You are on page 1of 700

P1: FXS/ABE P2: FXS

9780521740494c01.xml CUAU033-EVANS October 5, 2008 6:21


C H A P T E R
1
Matrices
Objectives
To be able to identify when two matrices are equal
To be able to add and subtract matrices of the same dimensions
To be able to perform multiplication of a matrix and a scalar
To be able to identify when the multiplication of two given matrices is possible
To be able to perform multiplication on two suitable matrices
To be able to find the inverse of a 2 2 matrix
To be able to find the determinant of a matrix
To be able to solve linear simultaneous equations in two unknowns using an
inverse matrix
1.1 Introduction to matrices
A matrix is a rectangular array of numbers. The numbers in the array are called the entries in
the matrix.
The following are examples of matrices:

1 2
3 4
5 6

[2 1 5 6]

2 3
0 0 1

2 0

[5]
Matrices vary in size. The size, or dimension, of the matrix is described by specifying the
number of rows (horizontal lines) and columns (vertical lines) that occur in the matrix.
The dimensions of the above matrices are, in order:
3 2, 1 4, 3 3, 1 1.
The rst number represents the number of rows and the second, the number of columns.
1
P1: FXS/ABE P2: FXS
9780521740494c01.xml CUAU033-EVANS October 5, 2008 6:21
2 Essential Advanced General Mathematics
Example 1
Write down the dimensions of the following matrices.
a

1 1 2
2 1 0

1
2
3
4

c

2 2 3

Solution
a 2 3 b 4 1 c 1 3
The use of matrices to store information is demonstrated by the following two examples.
Four exporters A, B, C and D sell televisions (t), CD players (c), refrigerators (r) and
washing machines (w). The sales in a particular month can be represented by a 4 4 array of
numbers. This array of numbers is called a matrix.
r c w t
A
B
C
D

120 95 370 250


430 380 1000 900
60 50 150 100
200 100 470 50

row 1
row 2
row 3
row 4
column 1 column 2 column 3 column 4
From the matrix it can be seen:
Exporter A sold 120 refrigerators, 95 CD players, 370 washing machines and 250 televisions.
Exporter B sold 430 refrigerators, 380 CD players, 1000 washing machines and 900 televisions.
The entries for the sales of refrigerators are made in column 1.
The entries for the sales of exporter A are made in row 1.
The diagram on the right represents a section of a road map.
The number of direct connecting roads between towns can be
represented in matrix form.
A B C D
A
B
C
D

0 2 1 1
2 0 1 0
1 1 0 0
1 0 0 0

B
A
C
D
If A is a matrix, a
ij
will be used to denote the entry that occurs in row i and column j of A.
Thus a 3 4 matrix may be written
A =

a
11
a
12
a
13
a
14
a
21
a
22
a
23
a
24
a
31
a
32
a
33
a
34

P1: FXS/ABE P2: FXS


9780521740494c01.xml CUAU033-EVANS October 5, 2008 6:21
Chapter 1 Matrices 3
For B, an m n matrix
B =

b
11
b
12
. . . . . b
1n
b
21
b
22
. . . . . b
2n
. .
. .
. .
. .
. .
b
m1
b
m2
. . . . . b
mn

Matrices provide a format for the storage of data. In this form the data is easily operated on.
Some graphics calculators have a built-in facility to operate on matrices and there are
computer packages which allow the manipulation of data in matrix form.
A car dealer sells three models of a certain make and his business operates through two
showrooms. Each month he summarises the number of each model sold by a sales
matrix S:
S =

s
11
s
12
s
13
s
21
s
22
s
23

, where s
i j
is the number of cars of model j sold by showroomi.
So, for example, s
12
is the number of sales made by showroom 1, of model 2.
If in January, showroom 1 sold three, six and two cars of models 1, 2 and 3 respectively, and
showroom 2 sold four, two and one car(s) of models 1, 2 and 3 (in that order), the sales matrix
for January would be:
S =

3 6 2
4 2 1

A matrix is, then, a way of recording a set of numbers, arranged in a particular way. As in
Cartesian coordinates, the order of the numbers is signicant, so that although the matrices

1 2
3 4

3 4
1 2

have the same numbers and the same number of elements, they are different matrices (just as
(2, 1), (1, 2) are coordinates of different points).
Two matrices A, B, are equal, and can be written as A = B when
each has the same number of rows and the same number of columns
they have the same number or element at corresponding positions.
e.g.

2 1 1
0 1 3

1 +1 1 1
1 1 1
6
2

P1: FXS/ABE P2: FXS


9780521740494c01.xml CUAU033-EVANS October 5, 2008 6:21
4 Essential Advanced General Mathematics
Example 2
If matrices A and B are equal, nd the values of x and y.
A =

2 1
x 4

B =

2 1
3 y

Solution
x = 3 and y = 4
Although a matrix is made from a set of numbers, it is important to think of a matrix as a
single entity, somewhat like a super number.
Example 3
There are four rows of seats of three seats each in a minibus. If 0 is used to indicate a seat is
vacant and 1 is used to indicate a seat is occupied, write down a matrix that represents
a the 1st and 3rd rows are occupied but the 2nd and 4th rows are vacant
b only the seat on the front left corner of the bus is occupied.
Solution
a

1 1 1
0 0 0
1 1 1
0 0 0

1 0 0
0 0 0
0 0 0
0 0 0

Example 4
There are four clubs in a local football league.
Team A has 2 senior teams and 3 junior teams
Team B has 2 senior teams and 4 junior teams
Team C has 1 senior team and 2 junior teams
Team D has 3 senior teams and 3 junior teams
Represent this information in a matrix.
Solution

2 3
2 4
1 2
3 3

Note: rows represent teams A, B, C, D and columns represent the number


of senior and junior teams respectively.
P1: FXS/ABE P2: FXS
9780521740494c01.xml CUAU033-EVANS October 5, 2008 6:21
Chapter 1 Matrices 5
Exercise 1A
1 Write down the dimensions of the following matrices.
Example 1
a

1 2
3 4

2 1 1
0 1 3

c
[a b c d ]
d

p
q
r
s

2 There are 25 seats arranged in ve rows and ve columns. If 0, 1 respectively are used to
Example 3
indicate whether a seat is vacant or occupied, write down a matrix which represents the
situation when
a only seats on the two diagonals are occupied
b all seats are occupied.
3 If seating arrangements (as in 2) are represented by matrices, consider the matrix in which
the i, j element is 1 if i = j , but 0 if i = j . What seating arrangement does this matrix
represent?
4 At a certain school there are 200 girls and 110 boys in Year 7, 180 girls and 117 boys in
Example 4
Year 8, 135 and 98 respectively in Year 9, 110 and 89 in Year 10, 56 and 53 in Year 11 and
28 and 33 in Year 12. Summarise this information in matrix form.
5 From the following, select those pairs of matrices which could be equal, and write down
Example 2
the values of x, y which would make them equal.
a

3
2

0
x

, [0 x ], [0 4 ]
b

4 7
1 2

1 2
4 x

x 7
1 2

, [4 x 1 2]
c

2 x 4
1 10 3

y 0 4
1 10 3

2 0 4
1 10 3

6 In each of the following nd the values of the pronumerals so that matrices A and B are
equal.
a A =

2 1 1
0 1 3

B =

x 1 1
0 1 y

b A =

x
2

B =

3
y

c A = [3 x] B = [y 4] d A =

1 y
4 3

B =

1 2
4 x

7 A section of a road map connecting towns A, B, C


and D is shown. Construct the 4 4 matrix which
shows the number of connecting roads between
each pair of towns.
B
D
A
C
P1: FXS/ABE P2: FXS
9780521740494c01.xml CUAU033-EVANS October 5, 2008 6:21
6 Essential Advanced General Mathematics
8 The statistics for the ve members of a basketball team are recorded as follows.
Player A: points 21, rebounds 5, assists 5
Player B: points 8, rebounds 2, assists 3
Player C: points 4, rebounds 1, assists 1
Player D: points 14, rebounds 8, assists 60
Player E: points 0, rebounds 1, assists 2
Express this data in a 5 3 matrix.
1.2 Addition, subtraction and multiplication
by a scalar
Addition will be dened for two matrices only when they have the same number of rows and
the same number of columns. In this case the sum of two matrices is found by adding
corresponding elements. For example,

1 0
0 2

0 3
4 1

1 3
4 3

and

a
11
a
12
a
21
a
22
a
31
a
32

b
11
b
12
b
21
b
22
b
31
b
32

a
11
+b
11
a
12
+b
12
a
21
+b
21
a
22
+b
22
a
31
+b
31
a
32
+b
32

Subtraction is dened in a similar way. When the two matrices have the same number of rows
and the same number of columns the difference is found by subtracting corresponding
elements.
Example 5
Find
a

1 0
2 0

2 1
4 1

2 3
1 4

2 3
1 4

Solution
a

1 0
2 0

2 1
4 1

1 1
6 1

2 3
1 4

2 3
1 4

0 0
0 0

It is useful to dene multiplication of a matrix by a real number. If A is an m n matrix,


and k is a real number, then kA is an m n matrix whose elements are k times the
corresponding elements of A. Thus
3

2 2
0 1

6 6
0 3

These denitions have the helpful consequence that if a matrix is added to itself, the result is
twice the matrix, i.e. A +A = 2A. Similarly the sum of n matrices each equal to A is n A
(where n is a natural number).
The m n matrix with all elements equal to zero is called the zero matrix.
P1: FXS/ABE P2: FXS
9780521740494c01.xml CUAU033-EVANS October 5, 2008 6:21
Chapter 1 Matrices 7
Example 6
Let X =

2
4

, Y =

3
6

, A =

2 0
1 2

, B =

5 0
2 4

Find X +Y, 2X, 4Y +X, X Y, 3A, 3A +B.


Solution
X +Y =

2
4

3
6

5
10

2X = 2

2
4

4
8

4Y +X = 4

3
6

2
4

12
24

2
4

14
28

X Y =

2
4

3
6

1
2

3A = 3

2 0
1 2

6 0
3 6

3A +B =

6 0
3 6

5 0
2 4

1 0
1 2

Example 7
If A =

3 2
1 1

and B =

0 4
2 8

, nd matrices Xsuch that 2A +X = B.


Solution
If 2A +X = B, then X = B 2A
X =

0 4
2 8

3 2
1 1

0 2 3 4 2 2
2 2 1 8 2 1

6 8
0 6

P1: FXS/ABE P2: FXS


9780521740494c01.xml CUAU033-EVANS October 5, 2008 6:21
8 Essential Advanced General Mathematics
Using the TI-Nspire
2-by-2 matrices are easiest entered using
the 2-by-2 matrix template ( 5) as
shown.
Notice that there is also a template for
entering m-by-n matrices.
To enter the matrix A=

3 6
6 7

, use the
NavPad to move between the entries of the
2-by-2 matrix template and store (/ )
the matrix as a.
Dene the matrix B=

3 6
5 6.5

in a
similar way.
Entering matrices directly
To enter matrix A without using the
template, enter the matrix row by row as
[[3,6][6,7]] and store (/ ) the matrix
as a.
Alternatively, enter the matrix by typing
[3,6;6,7]. Semicolon (;) can be obtained by
typing / .
P1: FXS/ABE P2: FXS
9780521740494c01.xml CUAU033-EVANS October 5, 2008 6:21
Chapter 1 Matrices 9
Addition, multiplication and multiplication by a scalar
Once A and B are dened as above, A + B,
AB and KA can easily be determined.
Using the Casio ClassPad
Matrices are entered using the
2D CALC menu on the
k. Tap , enter the
numbers required then store
this as a variable (using VAR).
Calculations can be
performed as shown in the
screen at the far right.
Exercise 1B
1 Let X =

1
2

, Y =

3
0

, A =

1 1
2 3

, B =

4 0
1 2

Find X +Y, 2X, 4Y +X, X Y, 3A and 3A +B.


Example 6
2 Each showroom of a car dealer sells exactly twice as many cars of each model in February
as in January. (See example in section 1.1.)
a Given that the sales matrix for January is

3 6 2
4 2 1

, write down the sales matrix for


February.
P1: FXS/ABE P2: FXS
9780521740494c01.xml CUAU033-EVANS October 5, 2008 6:21
10 Essential Advanced General Mathematics
b If the sales matrices for January and March (with twice as many cars of each model
sold in February as January) had been

1 0 0
4 2 3

and

2 1 0
6 1 4

respectively, nd the
sales matrix for the rst quarter of the year.
c Find a matrix to represent the average monthly sales for the rst three months.
3 Let A =

1 1
0 2

Find 2A, 3A and 6A.


4 A, B, C are m n matrices. Is it true that
a A +B = B +A b (A +B) +C = A +(B +C)?
5 A =

3 2
2 2

and B =

0 3
4 1

Calculate
a 2A b 3B c 2A + 3B d 3B 2A
6 P =

1 0
0 3

, Q =

1 1
2 0

, R =

0 4
1 1

Calculate
a P +Q b P +3Q c 2P Q+R
7 If A =

3 1
1 4

and B =

0 10
2 17

, nd matrices X and Y such that


Example 7
2A 3X = B and 3A +2Y = 2B.
8 Matrices X and Y show the production of four models a, b, c, d at two automobile factories
P, Q in successive weeks.
X =
P
Q
a b c d

150 90 100 50
100 0 75 0

Y =
P
Q
a b c d

160 90 120 40
100 0 50 0

week 1 week 2
Find X + Y and write what this sum represents.
P1: FXS/ABE P2: FXS
9780521740494c01.xml CUAU033-EVANS October 5, 2008 6:21
Chapter 1 Matrices 11
1.3 Multiplication of matrices
Multiplication of a matrix by a real number has been discussed in the previous section. The
denition for multiplication of matrices is less natural. The procedure for multiplying two
2 2 matrices is shown rst.
Let A =

1 3
4 2

and B =

5 1
6 3

Then AB =

1 3
4 2

5 1
6 3

1 5 +3 6 1 1 +3 3
4 5 +2 6 4 1 +2 3

23 10
32 10

and BA =

5 1
6 3

1 3
4 2

5 1 +1 4 5 3 +1 2
6 1 +3 4 6 3 +3 2

9 17
18 24

Note that AB = BA.


If A is an m n matrix and B is an n r matrix, then the product AB is the m r matrix
whose entries are determined as follows.
To nd the entry in row i and column j of AB single out row i in matrix A and column j in
matrix B. Multiply the corresponding entries from the row and column and then add up the
resulting products.
Note: The product AB is dened only if the number of columns of A is the same as the number
of rows of B.
Example 8
For A =

2 4
3 6

and B =

5
3

nd AB.
Solution
A is a 2 2 matrix and B is a 2 1 matrix. Therefore AB is dened.
The matrix AB is a 2 1 matrix.
AB =

2 4
3 6

5
3

2 5 +4 3
3 5 +6 3

22
33

P1: FXS/ABE P2: FXS


9780521740494c01.xml CUAU033-EVANS October 5, 2008 6:21
12 Essential Advanced General Mathematics
Example 9
Matrix X shows the number of cars of models a and b bought by four dealers, A, B, C and D.
Matrix Y shows the cost in dollars of model a and model b.
Find XY and explain what it represents.
a b
X =
A
B
C
D

3 1
2 2
1 4
1 1

Y =

26 000
32 000

a
b
Solution
a b
XY =
A
B
C
D

3 1
2 2
1 4
1 1

26 000
32 000

a
b
4 2 2 1
The matrix XY is a 4 1 matrix
XY =

3 26 000 +1 32 000
2 26 000 +2 32 000
1 26 000 +4 32 000
1 26 000 +1 32 000

110 000
116 000
154 000
58 000

The matrix XY shows dealer A spent $110 000, dealer B spent $116 000, dealer C
spent $154 000 and dealer D spent $58 000.
Example 10
For A =

2 3 4
5 6 7

and B =

4 0
1 2
0 3

nd AB.
Solution
A is a 2 3 matrix and B is a 3 2 matrix. Therefore AB is a 2 2 matrix.
AB =

2 3 4
5 6 7

4 0
1 2
0 3

2 4 +3 1 +4 0 2 0 +3 2 +4 3
5 4 +6 1 +7 0 5 0 +6 2 +7 3

11 18
26 33

P1: FXS/ABE P2: FXS


9780521740494c01.xml CUAU033-EVANS October 5, 2008 6:21
Chapter 1 Matrices 13
Exercise 1C
1 If X =

2
1

, Y =

1
3

, A =

1 2
1 3

, B =

3 2
1 1

, C =

2 1
1 1

, I =

1 0
0 1

,
Examples 8,10
nd the products AX, BX, AY, IX, AC, CA, (AC)X, C(BX), AI, IB, AB, BA,
A
2
, B
2
, A(CA) and A
2
C.
2 a Are the following products, of matrices given in 1, dened?
AY, YA, XY, X
2
, CI, XI
b If A =

2 0
0 0

and B =

0 0
3 2

, nd AB.
3 The matrices A and B are 2 2 matrices, and O is the zero 2 2 matrix. Is the following
argument correct?
If AB = O, and A = O, then B = O.
4 If L = [2 1], X =

2
3

, nd LX and XL.
5 A and B are both m n matrices. Are AB and BA dened and, if so, how many rows and
columns do they have?
6 Suppose

a b
c d

d b
c a

1 0
0 1

.
Show that ad bc = 1. What is the product matrix if the order of multiplication on the
left-hand side is reversed?
7 Using the result of 6, write down a pair of matrices A, B such that AB = BA = I where
I =

1 0
0 1

.
8 Select any three 2 2 matrices A, B and C.
Calculate A(B +C), AB +AC and (B +C) A.
9 It takes John ve minutes to drink a milk shake which costs $2.50, and twelve minutes to
Example 9
eat a banana split which costs $3.00.
Calculate the product

5 12
2.50 3.00

1
2

and interpret the result in milk bar economics.


Suppose two friends join John.
Calculate

5 12
2.50 3.00

1 2 0
2 1 1

and interpret the result.


P1: FXS/ABE P2: FXS
9780521740494c01.xml CUAU033-EVANS October 5, 2008 6:21
14 Essential Advanced General Mathematics
10 The reading habits of ve students A, B, C, D and E are shown in the rst matrix below
where the columns p, q, r, and s represent four weekly magazines. The second matrix
shows the cost in dollars of each magazine. Find the product of the two matrices and
interpret the result.
p q r s
A
B
C
D
E

0 0 1 1
1 0 1 1
1 0 0 0
1 1 1 1
0 1 0 1

p
q
r
s

2.00
3.00
2.50
3.50

11 Let S =

s
11
s
12
s
13
s
21
s
22
s
23

be the sales matrix for two showrooms selling three models of


cars. Here s
ij
is the number of cars of model j sold from showroom i. Let the prices of the
three models of cars be $c
1
, $c
2
, $c
3
.
Call the 3 1 matrix, C =

c
1
c
2
c
3

the price matrix.


a Find SC. b What is the practical meaning of SC?
c Suppose the car dealer sells both new and used cars and the price of two-year-old used
cars for the three models is $u
1
, $u
2
and $u
3
, respectively.
Form a new cost matrix
C =

c
1
u
1
c
2
u
2
c
3
u
3

Find SC and state its meaning.


d Suppose the car dealer makes 30% prot on his selling of new cars and 25% on used
cars.
If V =

0.3 0
0 0.25

, what is the meaning of CV?


1.4 Identities, inverses and determinants
for 2 2 matrices
Identities
A matrix with the same number of rows and columns is called a square matrix. For square
matrices of a given dimension, e.g. 2 2, a multiplicative identity I exists.
For example, for 2 2 matrices I =

1 0
0 1

and for 3 3 matrices I =

1 0 0
0 1 0
0 0 1

P1: FXS/ABE P2: FXS


9780521740494c01.xml CUAU033-EVANS October 5, 2008 6:21
Chapter 1 Matrices 15
If A =

2 3
1 4

, AI = IA = A, and this result holds for any square matrix multiplied by the
appropriate multiplicative identity.
Inverses
Given a 2 2 matrix A, is there a matrix B such that AB = BA = I?
Let B =

x y
u v

and A =

2 3
1 4

Then AB = I implies

2 3
1 4

x y
u v

1 0
0 1

i.e.

2x +3u 2y +3v
x +4u y +4v

1 0
0 1

2x +3u = 1 and 2y +3v = 0


x +4u = 0 y +4v = 1
These simultaneous equations can be solved to nd x, u, y, and v and hence B.
B =

0.8 0.6
0.2 0.4

B is said to be the inverse of A as AB = BA = I.


Let A be a 2 2 matrix with A =

a b
c d

and let B =

x y
u v

where B is the inverse of A.


Then AB = I. In full this is written

ax +bu ay +bv
cx +du cy +dv

1 0
0 1

Hence ax +bu = 1 ay +bv = 0


cx +du = 0 cy +dv = 1
which form two pairs of simultaneous equations, for x, u and y, v respectively.
Taking the x, u pair and eliminating u, (ad bc)x = d
Similarly, eliminating x, (bc ad)u = c
These two equations can be solved for x and u respectively provided ad bc = 0
i.e. x =
d
ad bc
and u =
c
cb ad
=
c
ad bc
In a similar way it can be found that
y =
b
ad bc
and v =
a
cb ad
=
a
ad bc
Therefore the inverse =

d
ad bc
b
ad bc
c
ad bc
a
ad bc

P1: FXS/ABE P2: FXS


9780521740494c01.xml CUAU033-EVANS October 5, 2008 6:21
16 Essential Advanced General Mathematics
The inverse of a square matrix A, is denoted by A
1
. The inverse is unique. ad bc has a
name, the determinant of A. This is denoted det(A).
i.e. for A =

a b
c d

, det(A) = ad bc
A 2 2 matrix has an inverse only if det(A) = 0
A square matrix is said to be regular if its inverse exists. Those square matrices which do
not have an inverse are called singular matrices; i.e. for a singular matrix det(A) = 0.
Using the TI-Nspire
The operation of matrix inverse is obtained
by raising the matrix to the power of 1.
The Determinant command is found in
the Matrix and Vector menu (b72)
and used as shown.
(a is the matrix A =

3 6
6 7

, dened
on page 8.)
Using the Casio ClassPad
The operation of matrix inverse is obtained
by entering A

1 in the entry line.


The determinant is obtained by entering
and highlighting A and tapping Interactive,
Matrix-Calc, det.
P1: FXS/ABE P2: FXS
9780521740494c01.xml CUAU033-EVANS October 5, 2008 6:21
Chapter 1 Matrices 17
Example 11
For the matrix A =

5 2
3 1

nd
a det(A) b A
1
Solution
a det(A) = 5 1 2 3 = 1 b A
1
=
1
1

1 2
3 5

1 2
3 5

Example 12
For the matrix A =

3 2
1 6

nd
a det(A) b A
1
c X if AX =

5 6
7 2

d Y if YA =

5 6
7 2

Solution
a det(A) = 3 6 2 = 16 b A
1
=
1
16

6 2
1 3

c AX =

5 6
7 2

Multiply both sides (from the left) by A


1
.
A
1
AX = A
1

5 6
7 2

IX = X =
1
16

6 2
1 3

5 6
7 2

=
1
16

16 30
16 0

1 2
1 0

d YA =

5 6
7 2

P1: FXS/ABE P2: FXS


9780521740494c01.xml CUAU033-EVANS October 5, 2008 6:21
18 Essential Advanced General Mathematics
Multiply both sides (from the right) by A
1
YAA
1
=
1
16

5 6
7 2

6 2
1 3

YI = Y =
1
16

24 8
40 8

Y =

3
2
1
2
5
2
1
2

Exercise 1D
1 For the matrices A =

2 1
3 2

and B =

2 2
3 2

nd
Example 11
a det(A) b A
1
c det(B) d B
1
2 Find the inverse of the following regular matrices ( is any real number, k is any non-zero
real number).
a

3 1
4 1

3 1
2 4

1 0
0 k

cos sin
sin cos

3 If A, B are the regular matrices A =

2 1
0 1

, B =

1 0
3 1

, nd A
1
, B
1
.
Also nd AB and hence nd, if possible, (AB)
1
.
Also nd from A
1
, B
1
, the products A
1
B
1
and B
1
A
1
. What do you notice?
4 For the matrix A =

4 3
2 1

Example 12
a nd A
1
b if AX =

3 4
1 6

, nd X c if YA =

3 4
1 6

, nd Y.
5 If A =

3 2
1 6

, B =

4 1
2 2

and C =

3 4
2 6

, nd
a X such that AX +B = C b Y such that YA +B = C
6 If A is a 2 2 matrix, a
12
= a
21
= 0, a
11
= 0, a
22
= 0, then show that A is regular and
nd A
1
.
7 Let A be a regular 2 2 matrix, B a 2 2 matrix and AB = 0. Show that B = 0.
8 Find all 2 2 matrices such that A
1
= A.
P1: FXS/ABE P2: FXS
9780521740494c01.xml CUAU033-EVANS October 5, 2008 6:21
Chapter 1 Matrices 19
1.5 Solution of simultaneous equations
using matrices
Inverse matrices can be used to solve certain sets of simultaneous linear equations. Consider
the equations
3x 2y = 5
5x 3y = 9
This can be written as

3 2
5 3

x
y

5
9

If A =

3 2
5 3

the determinant of A is 3(3) 5(2) = 1


which is not zero and so A
1
exists.
A
1
=

3 2
5 3

Multiplying the matrix equation

3 2
5 3

x
y

5
9

on the left hand side by A


1
and using
the fact that A
1
A = I yields the following:
A
1

x
y

= A
1

5
9

x
y

= A
1

5
9

x
y

3
2

since A
1

5
9

3
2

This is the solution to the simultaneous equations.


Check by substituting x = 3, y = 2 in the equations.
When dealing with simultaneous linear equations in two variables which represent parallel
straight lines, a singular matrix results.
For example the system
x +2y = 3
2x 4y = 6
has associated matrix equation

1 2
2 4

x
y

3
6

P1: FXS/ABE P2: FXS


9780521740494c01.xml CUAU033-EVANS October 5, 2008 6:21
20 Essential Advanced General Mathematics
Note that the determinant of

1 2
2 4

= 1 4 (2 2) = 0.
There is no unique solution to the system of equations.
Example 13
If A =

2 1
1 2

and K =

1
2

, solve the system AX = K where X =

x
y

.
Solution
If AX = K, then X = A
1
K
A
1
K =
1
5

2 1
1 2

1
2

0
1

X =

0
1

Example 14
Solve the following simultaneous equations.
3x 2y = 6
7x +4y = 7
Solution
The matrix equation is

3 2
7 4

x
y

6
7

Let A =

3 2
7 4

Then A
1
=
1
26

4 2
7 3

and

x
y

=
1
26

4 2
7 3

6
7

=
1
26

38
21

Since any linear system of n equations in n unknowns can be written as


AX = K, where A is an n n matrix, X =

x
1
x
2
.
.
x
n

and K =

k
1
k
2
.
.
k
n

,
P1: FXS/ABE P2: FXS
9780521740494c01.xml CUAU033-EVANS October 5, 2008 6:21
Chapter 1 Matrices 21
this method can be applied more generally when A is regular. In fact, as shown, an expression
for the solution can be written at once. Multiply AX, and K, on the left by A
1
, and
A
1
(AX) = A
1
K and A
1
(AX) = (A
1
A)X = IX = X.
Hence X = A
1
K, which is a formula for the solution of the system. Of course it depends
on the inverse A
1
existing, but once A
1
is found then equations of the form AX = K can be
solved for all possible n 1 matrices K.
Again this process can be completed using a calculator as long as matrices A and K have
been entered onto the calculator.
Example 15
Consider the system of ve equations in ve unknowns.
2a +3b c +d +2e = 9
a +b c = 4
a +2d 3e = 4
b +2c d +e = 6
a b +d 2e = 0
Use a graphics calculator to solve for a, b, c, d and e.
Solution
Enter 5 5 matrix A and 5 1 matrix B into the graphics calculator.
A =

2 3 1 1 2
1 1 1 0 0
1 0 0 2 3
0 1 2 1 1
1 1 0 1 2

B =

9
4
4
6
0

Then A
1
B =

4
9
23
9
1
7
9

2
3

a =
4
9
, b =
23
9
, c = 1, d =
7
9
and e =
2
3
It should be noted that just as for two equations in two unknowns, there is a geometric
interpretation for three equations in three unknowns. There is only a unique solution if the
equations represent three planes intersecting at a point.
P1: FXS/ABE P2: FXS
9780521740494c01.xml CUAU033-EVANS October 5, 2008 6:21
22 Essential Advanced General Mathematics
Exercise 1E
1 If A =

3 1
4 1

, solve the system AX = K where X =

x
y

, and
Example 13
a K =

1
2

b K =

2
3

2 If A =

3 1
2 4

, solve the system AX = K where


a K =

0
1

b K =

2
0

3 Use matrices to solve the following pairs of simultaneous equations.


Example 14
a 2x +4y = 6
3x + y = 1
b x +2y = 1
x +4y = 2
c 2x +5y = 10
y = x +4
d 1.3x +2.7y = 1.2
4.6y 3.5x = 11.4
4 Use matrices to nd the point of intersection of the lines given by the equations
2x 3y = 7 and 3x + y = 5.
5 Two children spend their pocket money buying some books and some CDs. One child
spends $120 and buys four books and four CDs. The other child buys three CDs and ve
books and spends $114. Set up a system of simultaneous equations and use matrices to nd
the cost of a single book and a single CD.
6 Consider the system 2x 3y = 3
4x 6y = 6
a Write this system in matrix form, as AX = K.
b Is A a regular matrix?
c Can any solutions be found for this system?
d How many pairs does the solution set contain?
7 Consider the system of four equations in four unknowns.
Example 15
p +q r s = 5
r +s = 1
2p q +2r = 2
p q +s = 0
Use a graphics calculator to solve for p, q, r and s.
P1: FXS/ABE P2: FXS
9780521740494c01.xml CUAU033-EVANS October 5, 2008 6:21
R
e
v
i
e
w
Chapter 1 Matrices 23
Chapter summary
A matrix is a rectangular array of numbers.
Two matrices A and B are equal when:
r
each has the same number of rows and the same number of columns, and
r
they have the same number or element at corresponding positions.
The size or dimension of a matrix is described by specifying the number of rows (m) and the
number of columns (n). The dimension is written m n.
Addition will be dened for two matrices only when they have the same dimension. The sum
is found by adding corresponding elements.

a b
c d

e f
g h

a +e b + f
c + g d +h

Subtraction is dened in a similar way.


If A is an m n matrix and k is a real number, kA is dened to be an m n matrix whose
elements are k times the corresponding element of A.
k

a b
c d

ka kb
kc kd

If A is an m n matrix and B is an n r matrix, then the product AB is the m r matrix


whose entries are determined as follows.
To nd the entry in row i and column j of AB, single out row i in matrix A and column j in
matrix B. Multiply the corresponding entries from the row and column and then add up the
resulting products.
The product AB is dened only if the number of columns of A is the same as the number
of rows of B.
If A and B are square matrices of the same dimension and AB = BA = I then A is said to
the inverse of B and B is said to be the inverse of A.
If A =

a b
c d

then A
1
=

d
ad bc
b
ad bc
c
ad bc
a
ad bc

det(A) = ad bc is the determinant of matrix A.


A square matrix is said to be regular if its inverse exists. Those square matrices which do
not have an inverse are called singular matrices.
Simultaneous equations can be solved using inverse matrices, for example
ax +by = c
dx +ey = f
can be written as

a b
d e

x
y

c
f

and

x
y

a b
d e

c
f

P1: FXS/ABE P2: FXS


9780521740494c01.xml CUAU033-EVANS October 5, 2008 6:21
R
e
v
i
e
w
24 Essential Advanced General Mathematics
Multiple-choice questions
1 The matrix A =

1 0
2 1
2 3
3 0

has dimension
A 8 B 4 2 C 2 4 D 1 4 E 3 4
2 If A =

2 0
1 3

and B =

1 3 4
1 3 1

then A +B =
A

3 3
2 0

3 4
2 2

1 2
2 3

2 1
1 3

E Cannot be determined
3 If C =

2 3 1
1 0 2

and D =

1 3 1
2 3 1

then D C =
A

1 0 0
1 3 1

2 6 4
2 0 4

1 0 0
1 3 1

1 6 0
1 3 1

E Cannot be determined
4 If M=

4 0
2 6

then M=
A

4 0
2 6

0 4
6 2

4 0
2 6

0 4
6 2

4 0
2 6

5 If M=

0 2
3 1

and N =

0 4
3 0

then 2M 2N =
A

0 0
9 2

0 2
6 1

0 4
12 2

0 4
12 2

0 2
6 1

6 If A and B are both m n matrices, where m = n, then A +B is an


A m n matrix B m m matrix C n n matrix
D 2m 2n matrix E Cannot be determined
7 If P is an m n matrix, and Q is a n p matrix, the dimension of matrix QP is
A n n B m p C n p D m n E Cannot be determined
8 The determinant of matrix A =

2 2
1 1

is
A 4 B 0 C 4 D 1 E 2
9 The inverse of matrix A =

1 1
1 2

is
A 1
B

2 1
1 1

1 1
1 2

1 1
1 2

2 1
1 1

P1: FXS/ABE P2: FXS


9780521740494c01.xml CUAU033-EVANS October 5, 2008 6:21
R
e
v
i
e
w
Chapter 1 Matrices 25
10 If M=

0 2
3 1

and N =

0 2
3 1

then NM=
A

0 4
9 1

4 2
2 8

0 4
9 1

6 2
3 5

6 2
3 5

Short-answer questions (technology-free)


1 If A =

1 0
2 3

and B =

1 0
0 1

, nd
a (A +B)(A B)
b A
2
B
2
2 Find all possible matrices A which satisfy the equation

3 4
6 8

A =

8
16

.
3 Let A =

1 2
3 1

, B = [3 1 2], C =

6
1

, D =

2 4

and E =

5
0
2

.
a State whether or not each of the following products exist: AB, AC, CD, BE
b Evaluate DA and A
1
.
4 If A =

1 2 1
5 1 2

, B =

1 4
1 6
3 8

and C =

1 2
3 4

, evaluate AB and C
1
.
5 Find the 2 2 matrix A such that A

1 2
3 4

5 6
12 14

6 If A =

2 0 0
0 0 2
0 2 0

, nd A
2
and hence A
1
.
7 If

1 2
4 x

is a singular matrix, nd the value of x.


8 a If M=

2 1
1 3

, nd the value of
i MM= M
2
ii MMM= M
3
iii M
1
b Find x and y given that M

x
y

3
5

P1: FXS/ABE P2: FXS


9780521740494c01.xml CUAU033-EVANS October 5, 2008 6:21
R
e
v
i
e
w
26 Essential Advanced General Mathematics
Extended-response questions
1 A =

3 1
1 4

, B =

2 1
5 2

a Find
i A +B ii A B iii 2A +3B iv C such that 3A +2C = B
b Find
i AB ii A
1
iii X such that AX = B iv Y such that YA = B
2 If A =

1 2 2
2 0 1
1 3 4

, B =

2 0 1
4 2 2
1 3 3

and C

2 0 2
3 0 1
1 3 1

, nd
a AB b AC c BC
d X such that AX = C e Y such that YA = B
f X such that AXC = CB g Y such that CYA = BA
3 a Consider the system of equations
2x 3y = 3
4x + y = 5
i Write this system in matrix form, as AX = K.
ii Find detA and A
1
.
iii Solve the system of equations.
iv Interpret your solution geometrically.
b Consider the system of equations
2x + y = 3
4x +2y = 8
i Write this system in matrix form, as AX = K.
ii Find detA and explain why A
1
does not exist.
c Interpret your ndings in part b geometrically.
4 The nal grades for Physics and Chemistry are made up of three components, tests,
practical work and exams. Marks out of 100 are awarded in each component each
semester.
Wendy scored the following marks in each of the three components for Physics.
Semester 1: tests 79, practical work 78, exam 80
Semester 2: tests 80, practical work 78, exam 82
a Represent this information in a 2 3 matrix.
To calculate the nal grade for each semester the three components are
weighted so that tests are worth 20%. Practical work is worth 30% and the exam is worth
50%.
(contd)
P1: FXS/ABE P2: FXS
9780521740494c01.xml CUAU033-EVANS October 5, 2008 6:21
R
e
v
i
e
w
Chapter 1 Matrices 27
b Represent this information in a 3 1 matrix.
c Calculate Wendys nal grade for Physics in each semester.
Wendy also scored the following marks in each of the three components for Chemistry.
Semester 1: tests 86, practical work 82, exam 84
Semester 2: tests 81, practical work 80, exam 70
d Calculate Wendys nal grade for Chemistry in each semester.
Students who gain an aggregate score for Physics and Chemistry of 320 or more over
the two semesters are awarded a Certicate of Merit in Science.
e Will Wendy be awarded a Certicate of Merit in Science?
She asks her teacher to remark her Semester 2 Chemistry Exam hoping that she will
gain the necessary marks to be awarded a Certicate of Merit.
f How many extra marks does she need?
5 A company runs Computing classes and employs full-time and part-time teaching staff as
well as technical support staff, cleaners and catering staff. The number of staff employed
depends on demand from term to term.
In one year they employed the following teaching staff:
Term 1: full-time 10, part-time 2
Term 2: full-time 8, part-time 4
Term 3: full-time 8, part-time 8
Term 4: full-time 6, part-time 10
a Represent this information in a 4 2 matrix.
Full-time teachers are paid $70 per hour and part-time teachers are paid $60 per hour.
b Represent this information in a 2 1 matrix.
c Calculate the cost per hour to the company for teaching staff for each term.
In the same year they also employed the following support staff
Term 1: technical staff 2, catering staff 2, cleaning staff 1.
Term 2: technical staff 2, catering staff 2, cleaning staff 1.
Term 3: technical staff 3, catering staff 4, cleaning staff 2.
Term 4: technical staff 3, catering staff 4, cleaning staff 2.
d Represent this information in a 4 3 matrix.
Technical staff are paid $60 per hour, catering staff $55 per hour and cleaners are paid
$40 per hour.
e Represent this information in a 3 1 matrix.
f Calculate the cost per hour to the company for support staff for each term.
g Calculate the total cost per hour to the company for teaching and support staff for each
term.
P1: FXS/ABE P2: FXS
9780521740494c02.xml CUAU033-EVANS August 21, 2009 15:24
C H A P T E R
2
Algebra I
Objectives
To express a number in standard form
To solve linear equations
To solve problems with linear equations and simultaneous linear equations
To use substitution and transposition with formulas
To add and multiply algebraic fractions
To solve literal equations
To solve simultaneous literal equations
2.1 Indices
In this section a review of indices is undertaken.
Review of index laws
a
m
a
n
= a
m+n
a
m
a
n
= a
mn
(a
m
)
n
= a
mn
a
n
=
1
a
n
n

a = a
1
n
(ab)
n
= a
n
b
n
a
0
= 1
Example 1
Simplify each of the following.
a x
2
x
3
b
x
4
x
2
c x
1
2
x
4
5
d (x
3
)
1
2
Solution
a x
2
x
3
= x
2+3
= x
5
b
x
4
x
2
= x
42
= x
2
c x
1
2
x
4
5
= x
1
2

4
5
= x

3
10
d (x
3
)
1
2
= x
3
2
28
P1: FXS/ABE P2: FXS
9780521740494c02.xml CUAU033-EVANS August 21, 2009 15:24
Chapter 2 Algebra I 29
Example 2
Evaluate
a 125
2
3
b

1000
27

2
3
Solution
a 125
2
3
=

(125)
1
3

2
= 5
2
= 25
b

1000
27

2
3
=

1000
27

1
3

2
=

10
3

2
=
100
9
Example 3
Simplify
4

x
2
y
3
x
1
2
y
2
3
.
Solution
4

x
2
y
3
x
1
2
y
2
3
=
(x
2
y
3
)
1
4
x
1
2
y
2
3
=
x
2
4
y
3
4
x
1
2
y
2
3
= x
2
4

1
2
y
3
4

2
3
= x
0
y
1
12
= y
1
12
Exercise 2A
1 Simplify each of the following using the appropriate index laws.
a x
3
x
4
Example 1a
b a
5
a
3
c x
2
x
1
x
2
d
y
3
y
7
Example 1b
e
x
8
x
4
f
p
5
p
2
g a
1
2
a
2
3
Example 1c
h (a
2
)
4
i (y
2
)
7
j (x
5
)
3
k (a
20
)
3
5
l

x

1
2

4
m (n
10
)
1
5
n 2x
1
2
4x
3
o (a
2
)
5
2
a
4
p
1
x
4
q

2n

2
5

5
(4
3
n
4
)
r x
3
2x
1
2
4x

3
2
s (ab
3
)
2
a
2
b
4

1
a
2
b
3
t (2
2
p
3
4
3
p
5
(6p
3
))
0
P1: FXS/ABE P2: FXS
9780521740494c02.xml CUAU033-EVANS August 21, 2009 15:24
30 Essential Advanced General Mathematics
2 Evaluate each of the following.
Example 2
a 25
1
2
b 64
1
3 c

16
9

1
2
d 16

1
2
e

49
36

1
2
f 27
1
3
g 144
1
2
h 64
2
3
i 9
3
2 j

81
16

1
4
k

23
5

0
l 128
3
7
3 Use your calculator to evaluate each of the following, correct to two decimal places.
a 4.35
2
b 2.4
5
c

34.6921 d (0.02)
3
e
3

0.729 f
4

2.3045 g (345.64)

1
3
h (4.568)
2
5
i
1
(0.064)

1
3
4 Simplify each of the following, giving your answer with positive index.
a
a
2
b
3
a
2
b
4
b
2a
2
(2b)
3
(2a)
2
b
4
c
a
2
b
3
a
2
b
4
d
a
2
b
3
a
2
b
4

ab
a
1
b
1
e
(2a)
2
8b
3
16a
2
b
4
f
2a
2
b
3
8a
2
b
4

16ab
(2a)
1
b
1
5 Write
2
n
8
n
2
2n
16
in the form 2
an+b
.
6 Write 2
x
3
x
6
2x
3
2x
2
2x
as a power of 6.
7 Simplify each of the following.
a 2
1
3
2
1
6
2

2
3
b a
1
4
a
2
5
a

1
10
c 2
2
3
2
5
6
2

2
3
d

2
1
3

2
1
2

5
e

2
1
3

2
2
1
3
2

2
5
8 Simplify each of the following.
Example 3
a
3

a
3
b
2

a
2
b
1
b

a
3
b
2

a
2
b
1
c
5

a
3
b
2

a
2
b
1
d

a
4
b
2

a
3
b
1
e

a
3
b
2
c
3

a
2
b
1
c
5
f
5

a
3
b
2

a
2
b
1
g

a
3
b
2
a
2
b
1
c
5

a
4
b
2
a
3
b
1

a
3
b
1
2.2 Standard form
Often when dealing with real world problems, the numbers involved may be very small or
very large. For example, the distance from the Earth to the Sun is approximately
150 000 000 kilometres, and the mass of an oxygen atom is approximately
0.000 000 000 000 000 000 000 026 grams. In order to deal with such numbers, a more
convenient way to express them can be used. This involves expressing the number as a product
of a number between 1 and 10 and a power of ten and is called standard form or scientic
notation.
P1: FXS/ABE P2: FXS
9780521740494c02.xml CUAU033-EVANS August 21, 2009 15:24
Chapter 2 Algebra I 31
These examples written in standard form would read
1.5 10
8
kilometres and 2.6 10
23
grams respectively.
Performing multiplication and division with very small or very large numbers can often be
simplied by rst converting the numbers into standard form. When simplifying algebraic
expressions or manipulating numbers in standard form, a sound knowledge of the index laws is
essential.
Example 4
Write each of the following in standard form.
a 3 453 000 b 0.00675
Solution
a 3 453 000 = 3.453 10
6
b 0.00675 = 6.75 10
3
Example 5
Find the value of
32 000 000 0.000 004
16 000
.
Solution
32 000 000 0.000 004
16 000
=
3.2 10
7
4 10
6
1.6 10
4
=
12.8 10
1
1.6 10
4
= 8 10
3
= 0.008
Example 6
Evaluate
5

a
b
2
if a = 1.34 10
10
and b = 2.7 10
8
.
Solution
5

a
b
2
=
5

1.34 10
10
(2.7 10
8
)
2
=
(1.34 10
10
)
1
5
2.7
2
(10
8
)
2
= 1.454 43 10
13
= 1.45 10
13
to three signicant gures.
Many calculators have scientic notation. The actual display will vary from calculator to
calculator.
For example, in standard form 3 245 000 = 3.245 10
6
may appear as 3.245E6 or 3.245
06
.
P1: FXS/ABE P2: FXS
9780521740494c02.xml CUAU033-EVANS August 21, 2009 15:24
32 Essential Advanced General Mathematics
Using the TI-Nspire
The TI-Nspire can be set to express answers in standard form by selecting Document
Settings or System Settings from 8: Systems Info. The number 3 245 000 will then
appear as 3.245E6.
The number of signicant gures can also be set through these menus. For example,
if two signicant places are selected (Float 2), 3 245 000 will appear as 3.2 E6.
Using the Casio ClassPad
The Classpad calculator can be set to express decimal answers in various forms. To
select a xed number of decimal places, including specifying scientic notation with
xed decimal accuracy, tap and in Basic format tap the arrow to select from the
various Number formats available.
Exercise 2B
1 Express each of the following numbers in standard form.
Example 4
a 47.8 b 6728 c 79.23 d 43 580
e 0.0023 f 0.000 000 56 g 12.000 34 h 50 million
i 23 000 000 000 j 0.000 000 0013 k 165 thousand l 0.000 014 567
2 Express each of the following in scientic notation.
a X-rays have a wavelength of 0.000 000 01 cm.
b The mass of a hydrogen atom is 0.000 000 000 000 000 000 000 0166 g.
c Visible light has wavelength 0.000 05 cm. d One nautical mile is 1853.18 m.
e A light year is 9 463 000 000 000 km.
f The speed of light is 29 980 000 000 cm/s.
3 Express each of the following as an ordinary number.
a The star Sirius is approximately 7.5684 10
13
km from Earth.
b A single blood cell contains 2.7 10
8
molecules of haemoglobin.
c The radius of an electron is 1.9 10
13
cm.
4 Find the value of
Example 5
a
324 000 0.000 0007
4000
b
5 240 000 0.8
42 000 000
5 Evaluate the following correct to three signicant gures.
Example 6
a
3

a
b
4
if a = 2 10
9
and b = 3.215 b
4

a
4b
4
if a = 2 10
12
and b = 0.05
P1: FXS/ABE P2: FXS
9780521740494c02.xml CUAU033-EVANS August 21, 2009 15:24
Chapter 2 Algebra I 33
2.3 Solving linear equations and linear
simultaneous equations
The solution to many problems may be found by translating them into a mathematical equation
which may then be solved using algebraic techniques. An equation is solved by nding the
value or values of the unknowns that would make the statement true.
Linear equations are simple equations that can be written in the form ax +b = 0. There are
a number of standard techniques that can be used for solving linear equations.
Example 7
Solve
x
5
2 =
x
3
Solution
x
5
2 =
x
3
x
5
15 2 15 =
x
3
15
Multiply both sides of the equation by the lowest common multiple of 3 and 5.
3x 30 = 5x
3x 5x = 30
2x = 30
x =
30
2
x = 15
Example 8
Solve
x 3
2

2x 4
3
= 5
Solution
x 3
2
6
2x 4
3
6 = 5 6
3(x 3) 2(2x 4) = 5 6
3x 9 4x +8 = 30
3x 4x = 30 +9 8
x = 31
x =
31
1
= 31
P1: FXS/ABE P2: FXS
9780521740494c02.xml CUAU033-EVANS August 21, 2009 15:24
34 Essential Advanced General Mathematics
Simultaneous linear equations
Finding the intersection of two straight lines can be
done graphically, however the accuracy of the solution
will depend on the accuracy of the graphs.
(1, 2)
x
+
2
y
=

3
x
2
x


y

=

4
0
1
1 2 3
1
1 2 3
2
3
4
2
3
4
y
Alternatively this point of intersection may be found
algebraically by solving the pair of simultaneous equations.
Three techniques for solving simultaneous equations will be
considered.
Example 9
Solve the equations 2x y = 4 and x +2y = 3.
Solution
1: By substitution
2x y = 4 (1)
x +2y = 3 (2)
First express one unknown from either equation in terms of the other unknown.
From equation (2) we get x = 3 2y.
Then substitute this expression into the other equation.
Equation (1) then becomes 2(3 2y) y = 4 reducing it to one equation
in one unknown.
Solving (1) 6 4y y = 4
5y = 10
y = 2
Substituting the value of y into (2) x +2(2) = 3
x = 1
Check in (1) LHS = 2(1) (2) = 4
RHS = 4
N.B. This means that the point (1, 2) is the point of intersection of the graphs of the
two linear relations.
P1: FXS/ABE P2: FXS
9780521740494c02.xml CUAU033-EVANS August 21, 2009 15:24
Chapter 2 Algebra I 35
2: By elimination
2x y = 4 (1)
x +2y = 3 (2)
If the coefcient of one of the unknowns is the same in both equations, we can
eliminate that unknown by subtracting one equation from the other. It may be
necessary to multiply one of the equations by a constant to make the coefcients of x
or y the same for the two equations.
To eliminate x multiply equation (2) by 2 and subtract the result from equation (1).
Equation (2) becomes 2x +4y = 6 (2

)
Then 2x y = 4 (1)
2x +4y = 6 (2

)
Subtracting (1) (2

) 5y = 10
y = 2
Now substitute for y in (1) to nd x, and check as in substitution method.
Using the TI-Nspire
The simultaneous equations can be solved in
a Calculator application. Solve( ) from the
Algebra menu (b31) can be used
with either the simultaneous equations
template (/ ) or with and as shown.
The and can either be typed or found in the
catalog ( 1 ).
The simultaneous equations can also be
solved graphically in a Graphs & Geometry
application. The equations are rearranged to
make y the subject. The equations in this
form are y = 2x 4 and y =
3 x
2
. Enter
these as shown.
P1: FXS/ABE P2: FXS
9780521740494c02.xml CUAU033-EVANS August 21, 2009 15:24
36 Essential Advanced General Mathematics
The intersection point is found by selecting
Intersection Point(s) from the Points and
Lines menu (b63). Use the NavPad
to move the hand to select each of the two
graphs as shown.
The coordinates of the intersection point
will appear on the screen.
Press to exit the Intersection Point(s)
menu.
Using the Casio ClassPad
The simultaneous equations can also be solved
graphically. First, the equations need to be
rearranged to make y the subject. In this form
the equations are y = 2x 4 and
y =
1
2
x
3
2
. Enter these in area as
shown. Select both equations by ticking the box
at the left then press to produce the graph.
To nd the solution, click into the graph
screen to select it and then click Analysis,
G-Solve, Intersect.
P1: FXS/ABE P2: FXS
9780521740494c02.xml CUAU033-EVANS August 21, 2009 15:24
Chapter 2 Algebra I 37
Exercise 2C
1 Solve the following linear equations.
a 3x +7 = 15 b 8
x
2
= 16 c 42 +3x = 22
d
2x
3
15 = 27 e 5(2x +4) = 13 f 3(4 5x) = 24
g 3x +5 = 8 7x h 2 +3(x 4) = 4(2x +5) i
2x
5

3
4
= 5x
Example 7
j 6x +4 =
x
3
3
2 Solve the following linear equations.
a
x
2
+
2x
5
= 16
Example 8
b
3x
4

x
3
= 8 c
3x 2
2
+
x
4
= 18
d
5x
4

4
3
=
2x
5
e
x 4
2
+
2x +5
4
= 6 f
3 3x
10

2(x +5)
6
=
1
20
g
3 x
4

2(x +1)
5
= 24 h
2(5 x)
8
+
6
7
=
4(x 2)
3
3 Solve each of the following pairs of simultaneous equations.
Example 9
a 3x +2y = 2
2x 3y = 6
b 5x +2y = 4
3x y = 6
c 2x y = 7
3x 2y = 2
d x +2y = 12
x 3y = 2
e 7x 3y = 6
x +5y = 10
f 15x +2y = 27
3x +7y = 45
2.4 Solving problems with linear equations
Many problems can be solved by translating them into mathematical language and using an
appropriate mathematical technique to nd the solution. By representing the unknown quantity
in a problem with a symbol (called a pronumeral) and constructing an equation from the
information, the value of the unknown can be found by solving the equation.
Before constructing the equation, state what the pronumeral is and what it stands for
(including the unit). It is essential to remember that all elements of the same type in the
equation must be in the same units.
Example 10
For each of the following, form the relevant linear equation and solve it for x.
a The length of the side of a square is (x 6) cm. Its perimeter is 52 cm.
b The perimeter of a square is (2x +8) cm. Its area is 100 cm
2
.
P1: FXS/ABE P2: FXS
9780521740494c02.xml CUAU033-EVANS August 21, 2009 15:24
38 Essential Advanced General Mathematics
Solution
a The perimeter = 4 length of a side
4(x 6) = 52
Therefore x 6 = 13
and x = 19
b The perimeter of the square is 2x +8
The length of one side =
2x +8
4
=
x +4
2
Therefore

x +4
2

2
= 100
In this case
x +4
2
= 10 as side length must be a positive number.
Therefore x = 16
Example 11
An athlete trains for an event by gradually increasing the distance she runs each week over a
ve-week period. If she runs an extra 5 km each successive week and over the ve weeks runs
a total of 175 km, how far did she run in the rst week?
Solution
Let the distance run in the rst week = x km.
Then the distance run in the second week = x +5 km.
The distance run in the third week = x +10 km.
So the total distance run = x + x +5 + x +10 + x +15 + x +20
5x +50 = 175
5x = 125
x = 25
The distance she ran in the rst week was 25 km.
Example 12
A man bought 14 CDs at a sale. Some cost him $15 each and the remainder cost $12.50 each.
In total he spent $190. How many $15 CDs and how many $12.50 CDs did he buy?
P1: FXS/ABE P2: FXS
9780521740494c02.xml CUAU033-EVANS August 21, 2009 15:24
Chapter 2 Algebra I 39
Solution
Let n equal the number of CDs costing $15.
Then 14 n = the number of CDs costing $12.50.
15n +12.5(14 n) = 190
15n +175 12.5n = 190
2.5n +175 = 190
2.5n = 15
n = 6
He bought 6 CDs costing $15 and 8 CDs costing $12.50.
Exercise 2D
1 For each of the cases below, write down a relevant equation involving the variables
dened and solve the equation for parts a, b and c.
Example 10
a The length of the side of a square is (x 2) cm. Its perimeter is 60 cm.
b The perimeter of a square is (2x +7) cm. Its area is 49 cm
2
.
c The length of a rectangle is (x 5) cm. Its width is (12 x) cm. The rectangle is
twice as long as it is wide.
d The length of a rectangle is (2x +1) cm. Its width is (x 3) cm. The perimeter of the
rectangle is y cm.
e n persons each has a meal costing $p. The total cost of the meal is $Q.
f S persons each has a meal costing $p. 10% service charge is added to the cost. The
total cost of the meal is $R.
g A machine working at a constant rate produces n bolts in 5 minutes. It produces 2400
bolts in 1 hour.
h The radius of a circle is (x +3) cm. A sector subtending an angle of 60

at the centre
is cut off. The arc length of the minor sector is a cm.
2 Bronwyn and Noel have a womens clothing shop in Summerland. Bronwyn manages the
shop and her sales are going up steadily over a particular period of time. They are going
up by $500 a week. If over a ve-week period her sales total $17 500, how much did she
earn in the rst week?
Example 11
3 Bronwyn and Noel have a womens clothing shop in Summerland and Bronwyn manages
the shop. Sally, Adam and baby Lana came into the shop and Sally bought dresses and
handbags. The dresses cost $65 each and the handbags cost $26 each. The total number of
items was 11 and in total she spent $598. How many dresses and how many handbags did
she buy?
Example 12
4 A rectangular courtyard is three times as long as it is wide. If the perimeter of the
courtyard is 67 m, nd the dimensions of the courtyard.
P1: FXS/ABE P2: FXS
9780521740494c02.xml CUAU033-EVANS August 21, 2009 15:24
40 Essential Advanced General Mathematics
5 A wine merchant buys 50 cases of wine. He pays full price for half of them but gets a
40% discount on the remainder. If he paid a total of $2260, how much was the full price
of a single case?
6 A real estate agent sells 22 houses in six months. He makes a commission of $11 500
per house on some and $13 000 per house on the remainder. If his total commission
over the six months was $272 500, on how many houses did he make a commission
of $11 500?
7 Three boys compare their marble collections. The rst boy has 14 less than the second
boy, who has twice as many as the third. If between them they have 71 marbles, how many
does each boy have?
8 Three girls are playing Scrabble. At the end of the game, the total of their scores adds up
to 504. Annie scored 10% more than Belinda, while Cassie scored 60% of the combined
scores of the other two. What did each player score?
9 A biathlon event involves running and cycling. Kim can cycle 30 km/h faster than she can
run. If Kim spends 48 minutes running and a third as much time again cycling in an event
that covers a total distance of 60 km, how fast can she run?
10 The mass of a molecule of a certain chemical compound is 2.45 10
22
g. If each
molecule is made up of two carbon atoms and six oxygen atoms and the mass of one
oxygen atom is
1
3
that of a carbon atom, nd the mass of an oxygen atom.
2.5 Solving problems using simultaneous
linear equations
When the relationships between two quantities is linear then the constants which determine the
linear relationship can be determined if two sets of information satisfying the relationship are
given. Simultaneous linear equations enable this to be done. Another situation in which
simultaneous linear equations may be used is where it is required to nd the point of the
cartesian plane which satises two linear relations.
Example 13
There are two possible methods for paying gas bills:
Method A: A xed charge of $25 per quarter + 50c per unit of gas used
Method B: A xed charge of $50 per quarter + 25c per unit of gas used.
Determine the number of units that must be used before method B becomes cheaper than
method A.
P1: FXS/ABE P2: FXS
9780521740494c02.xml CUAU033-EVANS August 21, 2009 15:24
Chapter 2 Algebra I 41
Solution
Let
C
1
= charge in $ using method A
C
2
= charge in $ using method B
x = number of units of gas used
Now C
1
= 25 +0.5x
C
2
= 50 +0.25x
It can be seen from the graph that if the number
of units exceeds 100 then method B is cheaper.
100
50
25
C
2
= 0.25x + 50
C
1
= 0.5x + 25
25 50 75 100125150
0
Units
D
o
l
l
a
r
s
C
x
The solution could also be obtained by solving simultaneous linear equations:
C
1
= C
2
25 +0.5x = 50 +0.25x
0.25x = 25
x = 100
Example 14
If 3 kg of jam and 2 kg of butter cost $29, and 6 kg of jam and 3 kg of butter cost $54, nd the
cost of 1 kg of jam and 1 kg of butter.
Solution
Let the cost of 1 kg of jam = x dollars and the cost of 1 kg of butter = y dollars.
Then
3x +2y = 29 1
and 6x +3y = 54 2
Multiply 1 by 2 :
6x +4y = 58 1

Subtract 1

from 2 :
y = 4
y = 4
Substituting in 2 gives:
6x +3(4) = 54
6x = 42
x = 7
The jam costs $7 per kilogram and the butter, $4 per kilogram.
Exercise 2E
1 A car hire rm offers the option of paying $108 per day with unlimited kilometres, or
$63 per day plus 32 cents per kilometre travelled. How many kilometres would you have to
travel in a given day to make the unlimited kilometre option more attractive?
Example 13
P1: FXS/ABE P2: FXS
9780521740494c02.xml CUAU033-EVANS August 21, 2009 15:24
42 Essential Advanced General Mathematics
2 Company A will cater for your party at a cost of $450 plus $40 per guest. Company B
offers the same service for $300 plus $43.00 per guest. How many guests are needed before
Company As charge is less than Company Bs?
3 A basketball nal is held in a stadium which can seat 15 000 people. All the tickets have
been sold, some to adults at $45 and the rest for children at $15. If the revenue from the
tickets was $525 000, nd the number of adults who bought tickets.
Example 14
4 A contractor employed eight men and three boys for one day and paid them a total of
$2240. Another day he employed six men and eighteen boys for $4200. What was the daily
rate he paid each man and each boy?
5 The sum of two numbers is 212 and their difference is 42. Find the two numbers.
6 A chemical manufacturer wishes to obtain 700 litres of a 24% acid solution by mixing a
40% solution with a 15% solution. How many litres of each solution should be used?
7 Two children had 220 marbles between them. After one child had lost half her marbles and
the other had lost 40 marbles they had an equal number of marbles. How many did each
child start with and how many did each child nish with?
8 An investor received $31 000 interest per annum from a sum of money, with part of it
invested at 10% and the remainder at 7% simple interest. She found that if she
interchanged the amounts she had invested she could increase her return by $1000 per
annum. Calculate the total amount she had invested.
9 Each adult paid $30 and each student paid $20 to attend a concert. A total of 1600 people
attended. The total paid was $37 000. How many adults and how many students attended
the concert?
2.6 Substitution and transposition of formulas
An equation that states a relationship between two or more quantities is called a formula,
e.g. the area of a circle A = r
2
. The value of A, the subject of the formula, may be found by
substituting a given value of r and the value of .
Example 15
Using the formula A = r
2
, nd the value of A correct to two decimal places, if r = 2.3,
= 3.142 (correct to two decimal places).
Solution
A = r
2
= 3.142(2.3)
2
= 16.621 18
A = 16.62, correct to two decimal places.
The formula can also be transposed to make r the subject. When transposing formulas
a similar procedure to solving linear equations is followed. Whatever has been done to
the pronumeral required is undone.
P1: FXS/ABE P2: FXS
9780521740494c02.xml CUAU033-EVANS August 21, 2009 15:24
Chapter 2 Algebra I 43
Example 16
Transpose the formula A = r
2
to make r the subject and nd the value of r, correct to two
decimal places, if A = 24.58, = 3.142 (correct to three decimal places).
Solution
A = r
2
A

= r
2

= r
r =

24.58
3.142
= 2.79697
r = 2.80, correct to two decimal places
Exercise 2F
1 Substitute the specied values to evaluate each of the following, giving the answers correct
to two decimal places.
a v if v = u +at and u = 15, a = 2, t = 5
b I if I =
PrT
100
and P = 600, r = 5.5, T = 10
c V if V = r
2
h and r = 4.25, h = 6
Example 15
d S if S = 2r(r +h) and r = 10.2, h = 15.6
e V if V =
4
3
r
2
h and r = 3.58, h = 11.4
f s if s = ut +
1
2
at
2
and u = 25.6, t = 3.3, a = 1.2
g T if T = 2

l
g
and l = 1.45, g = 9.8
h f if
1
f
=
1
v
+
1
u
and v = 3, u = 7
i c if c
2
= a
2
+b
2
and a = 8.8, b = 3.4
j v if v
2
= u
2
+2as and u = 4.8, a = 2.5, s = 13.6
2 Transpose each of the following to make the symbol in brackets the subject.
a v = u +at (a) b S =
n
2
(a +l) (l)
c A =
1
2
bh (b)
d P = I
2
R (I )
Example 16
e s = ut +
1
2
at
2
(a)
f E =
1
2
mv
2
(v)
g Q =

2gh (h) h xy z = xy + z (x)


i
ax +by
c
= x b (x) j
mx +b
x b
= c (x)
P1: FXS/ABE P2: FXS
9780521740494c02.xml CUAU033-EVANS August 21, 2009 15:24
44 Essential Advanced General Mathematics
3 The formula F =
9C
5
+32 is used to convert temperatures given in degrees Celsius
(C) to degrees Fahrenheit (F).
a Convert 28 degrees Celsius to degrees Fahrenheit.
b Transpose the formula to make C the subject and nd C if F = 135

.
4 The sum (S ) of the interior angles of a polygon with n sides is given by the formula
S = 180(n 2).
a Find the sum of the interior angles of an octagon.
b Transpose the formula to make n the subject and hence determine the number of sides
on a polygon whose interior angles add up to 1260

.
5 The volume (V) of a right cone is given by the formula V =
1
3
r
2
h where r is the radius
of the base and h is the height of the cone.
a Find the volume of a cone with radius 3.5 cm and height 9 cm.
b Transpose the formula to make h the subject and hence nd the height of a cone with
base radius 4 cm and volume 210 cm
3
.
c Transpose the formula to make r the subject and hence nd the radius of a cone with
height 10 cm and volume 262 cm
3
.
6 The sum (S) of a particular sequence of numbers is given by the formula S =
n
2
(a +l),
where n is the number of terms in the sequence, a is the rst term and l is the last term.
a Find the sum of the sequence of seven numbers whose rst term is 3 and whose last
term is 22.
b What is the rst term of a sequence containing thirteen terms, whose last term is 156
and whose sum is 1040?
c How many terms are there in the sequence 25 +22 +19 + +5 = 110?
2.7 Algebraic fractions
The principles involved in addition, subtraction, multiplication and division of algebraic
fractions are the same as for simple numerical fractions.
To add or subtract, all fractions must be written with a common denominator. When
multiplying, rst try to simplify the fractions by cancelling down. This process will involve
factorisation of either the numerators or denominators or both.
Addition and subtraction
Example 17
Simplify
a
x
3
+
x
4
b
2
x
+
3a
4
c
5
x +2

4
x 1
d
4
x +2

7
(x +2)
2
P1: FXS/ABE P2: FXS
9780521740494c02.xml CUAU033-EVANS August 21, 2009 15:24
Chapter 2 Algebra I 45
Solution
a
x
3
+
x
4
=
4x +3x
12
=
7x
12
b
2
x
+
3a
4
=
8 +3ax
4x
c
5
x +2

4
x 1
=
5(x 1) 4(x +2)
(x +2)(x 1)
=
5x 5 4x 8
(x +2)(x 1)
=
x 13
(x +2)(x 1)
d
4
x +2

7
(x +2)
2
=
4(x +2) 7
(x +2)
2
=
4x +1
(x +2)
2
Multiplication and division
Before multiplying and dividing algebraic fractions, it is best to factorise numerators and
denominators where possible so that common factors can be readily identied.
Example 18
Simplify
a
3x
2
10y
2

5y
12x
b
2x 4
x 1

x
2
1
x 2
c
x
2
1
2x 2

4x
x
2
+4x +3
d
x
2
+3x 10
x
2
x 2

x
2
+6x +5
3x +3
Solution
a
3x
2
10y
2

5y
12x
=
x
8y
b
2x 4
x 1

x
2
1
x 2
=
2(x 2)
x 1

(x 1)(x +1)
x 2
= 2(x +1)
c
x
2
1
2x 2

4x
x
2
+4x +3
=
(x 1)(x +1)
2(x 1)

4x
(x +1)(x +3)
=
2x
x +3
d
x
2
+3x 10
x
2
x 2

x
2
+6x +5
3x +3
=
(x +5)(x 2)
(x 2)(x +1)

3(x +1)
(x +1)(x +5)
=
3
x +1
Example 19
Express
3x
3

4 x
+3x
2

4 x as a single fraction.
P1: FXS/ABE P2: FXS
9780521740494c02.xml CUAU033-EVANS August 21, 2009 15:24
46 Essential Advanced General Mathematics
Solution
3x
3

4 x
+3x
2

4 x =
3x
3
+3x
2

4 x

4 x

4 x
=
3x
3
+3x
2
(4 x)

4 x
=
12x
2

4 x
Example 20
Express (x 4)
1
5
(x 4)

4
5
as a single fraction.
Solution
(x 4)
1
5
(x 4)

4
5
= (x 4)
1
5

1
(x 4)
4
5
=
(x 4)
1
5
(x 4)
4
5
1
(x 4)
4
5
=
x 5
(x 4)
4
5
Exercise 2G
1 Simplify each of the following.
Example 17
a
2x
3
+
3x
2
b
3a
2

a
4
c
3h
4
+
5h
8

3h
2
d
3x
4

y
6

x
3
e
3
x
+
2
y
f
5
x 1
+
2
x
g
3
x 2
+
2
x +1
h
2x
x +3

4x
x 3

3
2
i
4
x +1
+
3
(x +1)
2
j
a 2
a
+
a
4
+
3a
8
k 2x
6x
2
4
5x
l
2
x +4

3
x
2
+8x +16
m
3
(x 1)
+
2
(x 1)(x +4)
n
3
x 2

2
x +2
+
4
x
2
4
o
5
x 2

3
x
2
+5x +6
+
2
x +3
p x y
1
x y
q
3
x 1

4x
1 x
r
3
x 2
+
2x
2 x
P1: FXS/ABE P2: FXS
9780521740494c02.xml CUAU033-EVANS August 21, 2009 15:24
Chapter 2 Algebra I 47
2 Simplify each of the following.
Example 18
a
x
2
2y

4y
3
x
b
3x
2
4y

y
2
6x
c
4x
3
3

12
8x
4
d
x
2
2y

3xy
6
e
4 x
3a

a
2
4 x
f
2x +5
4x
2
+10x
g
(x 1)
2
x
2
+3x 4
h
x
2
x 6
x 3
i
x
2
5x +4
x
2
4x
j
5a
2
12b
2

10a
6b
k
x 2
x

x
2
4
2x
2
l
x +2
x(x 3)

4x +8
x
2
4x +3
m
2x
(x 1)

4x
2
x
2
1
n
x
2
9
x +2

3x +6
x 3

9
x
o
3x
9x 6

6x
2
x 2

2
x +5
3 Express each of the following as a single fraction.
a
1
x 3
+
2
x 3
b
2
x 4
+
2
x 3
c
3
x +4
+
2
x 3
d
2x
x 3
+
2
x +4
e
1
(x 5)
2
+
2
x 5
f
3x
(x 4)
2
+
2
x 4
g
1
x 3

2
x 3
h
2
x 3

5
x +4
i
2x
x 3
+
3x
x +3
j
1
(x 5)
2

2
x 5
k
2x
(x 6)
3

2
(x 6)
2
l
2x +3
x 4

2x 4
x 3
4 Express each of the following as a single fraction.
Example 19
a

1 x +
2

1 x
b
2

x 4
+
2
3
c
3

x +4
+
2

x +4
d
3

x +4
+

x +4
e
3x
3

x +4
3x
2

x +4
f
3x
3
2

x +3
+3x
2

x +3
5 Simplify each of the following.
Example 20
a (6x 3)
1
3
(6x 3)

2
3 b (2x +3)
1
3
2x(2x +3)

2
3
c (3 x)
1
3
2x (3 x)

2
3
2.8 Literal equations
A literal equation in x is an equation whose solution will be expressed in terms of pronumerals
rather than numbers. 2x +5 = 7 is an equation whose solution is the number 1. In the literal
equation ax +b = c, the solution is x =
c b
a
.
Literal equations are solved in the same way as solving numerical equations or transposing
formulas. Essentially, the literal equation is transposed to make x the subject.
P1: FXS/ABE P2: FXS
9780521740494c02.xml CUAU033-EVANS August 21, 2009 15:24
48 Essential Advanced General Mathematics
Example 21
Solve the following for x.
a px q = r b ax +b = cx +d c
a
x
=
b
2x
+c
Solution
a
px q = r
px = r +q
x =
r +q
p
b
ax +b = cx +d
ax cx = d b
x(a c) = d b
x =
d b
a c
c
a
x
=
b
2x
+c
Multiply both sides
by lowest common
denominator (2x)
2a = b +2xc
2a b = 2xc
2a b
2c
= x
Simultaneous literal equations
Simultaneous literal equations are solved by the same methods that are used for solving
simultaneous equations, i.e. substitution and elimination.
Example 22
Solve each of the following pairs of simultaneous equations for x and y.
a y = ax +c
y = bx +d
b ax y = c
x +by = d
Solution
a
ax +c = bx +d
ax bx = d c
x(a b) = d c
x =
d c
a b
and therefore
y = a

d c
a b

+c
=
ad bc
a b
b
ax y = c . . . 1
x +by = d . . . 2
Multiply 1 by b
abx by = cb . . . 1

Add 1

and 2
abx + x = cb +d
x(ab +1) = cb +d
x =
cb +d
ab +1
Substitute in 1
a

cb +d
ab +1

y = c
y = a

cb +d
ab +1

c
=
ad c
ab +1
P1: FXS/ABE P2: FXS
9780521740494c02.xml CUAU033-EVANS August 21, 2009 15:24
Chapter 2 Algebra I 49
Exercise 2H
1 Solve each of the following for x.
Example 21
a ax +n = m b ax +b = bx c
ax
b
+c = 0
d px = qx +5 e mx +n = nx m f
1
x +a
=
b
x
g
b
x a
=
2b
x +a
h
x
m
+n =
x
n
+m
i b(ax +b) = a(bx a)
j p
2
(1 x) 2pqx = q
2
(1 + x)
k
x
a
1 =
x
b
+2
l
x
a b
+
2x
a +b
=
1
a
2
b
2
m
p qx
t
+ p =
qx t
p
n
1
x +a
+
1
x +2a
=
2
x +3a
2 For the simultaneous equations ax + by = p and bx ay = q, show that x =
ap +bq
a
2
+b
2
and
y =
bp aq
a
2
+b
2
.
3 For the simultaneous equations
x
a
+
y
b
= 1 and
x
b
+
y
a
= 1, show that x = y =
ab
a +b
.
4 Solve each of the following pairs of simultaneous equations for x and y.
Example 22
a ax + y = c
x +by = d
b ax by = a
2
bx ay = b
2
c ax +by = t
ax by = s
d ax +by = a
2
+2ab b
2
bx +ay = a
2
+b
2
e (a +b)x +cy = bc
(b +c)y +ax = ab
f 3(x a) 2( y +a) = 5 4a
2(x +a) +3( y a) = 4a 1
5 Write s in terms of a only in the following pairs of equations.
a s = ah
h = 2a +1
b s = ah
h = a(2 +h)
c as = a +h
h +ah = 1
d as = s +h
ah = a +h
e s = h
2
+ah
h = 3a
2
f as = a +2h
h = a s
g s = 2 +ah +h
2
h = a
1
a
h 3s ah = a
2
as +2h = 3a
P1: FXS/ABE P2: FXS
9780521740494c02.xml CUAU033-EVANS August 21, 2009 15:24
50 Essential Advanced General Mathematics
Using the TI-Nspire with algebra
In this section a demonstration of the basic algebra properties of the TI-Nspire is
provided. To access these, open a Calculator application ( 1) and select the
Algebra menu (b3). The three main commands are solve, factor and expand.
Solve( )
This command is used to solve equations, simultaneous equations and some
inequalities. An approximate (decimal) answer can be obtained by pressing /
enter
or
by including a decimal number in the expression.
The following screens illustrate its use.
Factor( )
This command is used for factorisation.
Factorisation over the rational numbers is obtained by not specifying the variable,
whereas factorisation over the real numbers is obtained by specifying the variable.
The following screens illustrate its use.
P1: FXS/ABE P2: FXS
9780521740494c02.xml CUAU033-EVANS August 21, 2009 15:24
Chapter 2 Algebra I 51
Expand( )
This command is used for expansion of expressions.
By specifying the variable, the expanded expression will be ordered in decreasing
powers of that variable. Symbolic expressions can only be expanded for an appropriate
domain.
The following screens illustrate its use.
Using the Casio ClassPad with algebra
Using the Main menu
In this section the Main menu is explored.
Work through it to become acquainted with
the menu.
There are two ways to operate in this
menu, but the Interactive menu is the
simplest when used in conjunction with the
stylus. In this section we shall demonstrate
some examples of how this is used.
Solve
This is used to solve equation and
inequalities. The variables x, y and z are
found on the hard keyboard. Other variables
may be entered using the kand
selecting VAR. Variables are shown in bold
italics. The keyboard allows you to type
sentences, etc; however, the letters are not
always recognised as variables.
If you choose to use the keyboard, you must type a x, for example, because
ax will be treated as text.
P1: FXS/ABE P2: FXS
9780521740494c02.xml CUAU033-EVANS August 21, 2009 15:24
52 Essential Advanced General Mathematics
For example:
Enter ax +b = 0, highlight it with the stylus,
tap Interactive, Equation/inequality, solve and
ensure the variable selected is x. Solution
returned is x =
b
a
Enter x
2
+ x 1 = 0 and follow the same
instructions as above. The answer is as shown.
Note: x
2
+ x 1 = 0 is entered, but the
calculator converts this to standard algebraic
notation when EXE is pressed. Also note in
this example that = 0 has been omitted. If
the right-hand side of an equation is zero, it is
unnecessary to enter it.
Enter abt w +t = wt for w: follow the
instructions above and select w as the variable.
Solve x
3
x
2
x +1 = 0 for x.
Solve 2x +

2 < 3 for x. See screen at right


for solutions returned.
Note: is found in when kis
activated and is in . If the
answer is not in the form required, it is often
possible to cut and paste it in the next entry
line and use Interactive, Transformation,
Simplify as shown here.
Solve simultaneous equations uses a dedicated
entry style. Note carefully the screen at right.
To enter, tap in the window and
enter the equations and variables as shown.
For more than two equations, tap until the
number of equations required is given.
Factor
To factorise is to transform the expression to
a different form. This command is found in
Interactive, Transformation, factor.
Examples:
To factorise x
3
2x over the rational
numbers, use factor.
To factorise over the real numbers, select
rfactor.
P1: FXS/ABE P2: FXS
9780521740494c02.xml CUAU033-EVANS August 21, 2009 15:24
Chapter 2 Algebra I 53
Some further examples are provided here. See
the screens for the results.
Factor a
2
b
2
Factor a
3
b
3
Factor

2
x 1
+
1
(x 1)
2
+1

Factor (2x
4
x
2
) over the rationals
Factor (2x
4
x
2
) over the reals.
This command can also be used to give the
prime decomposition (factors) of integers.
Expand
To expand expressions, use Interactive,
Transformation, expand.
For example:
Expand (a +b)
3
Expand (a +b)
2
The expand command can also be used to
form partial fractions. In this case, enter and
highlight the expression, select Interactive,
transformation, expand, select the Partial
fraction option and set the variable as x.
For example:
Expand
1
x
2
1
Expand
x
3
+2x +1
x
2
1
Note: The top screen shows all the examples,
the bottom screen shows how to enter for
partial fractions.
P1: FXS/ABE P2: FXS
9780521740494c02.xml CUAU033-EVANS August 21, 2009 15:24
54 Essential Advanced General Mathematics
Zeros
To nd the zeros of an expression in the
menu, use Interactive, Equation/inequality,
Solve and ensure you set the variable. The
calculator assumes you are solving an
equation for which one side is zero.
For example:
Zeros of x
2
1 for x
Zeros of x
2
y
2
for y
Zeros of x
2
y
2
for x
Zeros of x
2
y for y
Zeros of x
2
4x +8 for x. No solutions.
Zeros of x
2
4x +1 for x. Two solutions.
Zeros of x
2
4x +4 for x. One solution.
Approximate
Switch mode in the status bar to Decimal. If
an answer is given in Standard (Exact) mode,
it can be converted by highlighting the
answer and tapping in the toolbar.
Combining fractions
This command gives an expression with a
common denominator and ratio form. The
denominator is returned in factored form.
For example:
Enter and highlight 1/(x 1) +1/(x +1)
then select Interactive, Transformation,
combine.
Enter and highlight y/(x y) + y/(x + y)
then select Interactive, Transformation,
combine.
P1: FXS/ABE P2: FXS
9780521740494c02.xml CUAU033-EVANS August 21, 2009 15:24
Chapter 2 Algebra I 55
Solve numerically
There are several ways to nd numerical
solutions to equations. In each of these ways,
only one solution is given. You can vary the
guess (Value) or the bounds of the search
(Lower and Upper) to nd particular
solutions. If an expression such as
x
2
x 2.1 is entered without an equals
sign, the calculator will assume the
expression is equal to zero and solve the
equation.
For example:
Enter and highlight x
2
x 2.1 = 0, select
Interactive, Equation/inequality, Solve and tap
the solve numerically option. Note that the
lower and upper bounds are set to and a
guess of 1 has been entered to return the rst
solution 1.03297.
For the second solution shown, the rst line
has been copied and pasted (or dragged) to the
next entry line and the guess x = 2 has been
entered to return the solution x = 2.03297.
Alternatively, use from the Main menu
and enter Lower and Upper bounds. A guess
may also be entered, but is not necessary.
(Note that the bounds selected arise from a
quick sketch which indicated that the
quadratic equation had a solution in the
negative and one in the positive domain.)
A third method involves using the Graph
program and nding the solution using the
G-Solve application.
P1: FXS/ABE P2: FXS
9780521740494c02.xml CUAU033-EVANS August 21, 2009 15:24
56 Essential Advanced General Mathematics
Exercise 2I
This exercise is to provide practice in some of the skills associated with a CAS calculator.
Other exercises in this chapter can also be attempted with CAS but it is recommended that you
also use this chapter to develop your by hand skills.
1 Solve each of the following equations for x.
a
a(a x)
b

b(b + x)
a
= x
b 2(x 3) +(x 2)(x 4) = x(x +1) 33
c
x +a
x +b
= 1
x
x b
d
x +a
x c
+
x +c
x a
= 2
2 Factorise each of the following.
a x
2
y
2
x
2
y
2
+1 b x
3
2 x +2x
2
c a
4
8a
2
b 48b
2
d a
2
+2bc (c
2
+2ab)
3 Solve each of the following pairs of simultaneous equations for x and y.
a axy +b = (a +c)y and bxy +a = (b +c)y
b x(b c) +by c = 0 and y(c a) ax +c = 0
P1: FXS/ABE P2: FXS
9780521740494c02.xml CUAU033-EVANS August 21, 2009 15:24
R
e
v
i
e
w
Chapter 2 Algebra I 57
Chapter summary
A number is expressed in standard form or scientic notation when written as a product
of a number between 1 and 10 and a power of ten; e.g. 1.5 10
8
Index laws
r
a
m
a
n
= a
m+n r
a
m
a
n
= a
mn r
(a
m
)
n
= a
mn
r
a
n
=
1
a
n
r n

a = a
1
n
r
(ab)
n
= a
n
b
n r
a

= 1
Linear equations
First identify the steps done to construct an equation; the equation is then solved by
undoing these steps.
This is achieved by doing the opposite in reverse order.
e.g. Solve for x, 3x +4 = 16
x has been multiplied by 3 and then 4 has been added
Subtract 4 from both sides 3x = 12
Divide both sides by 3 x = 4
A formula is an equation that states a relationship between two or more quantities, e.g. the
area of a circle A = r
2
. The value of A, the subject of the formula, may be found by
substituting a given value of r and the value of . A formula can be transposed to make a
different pronumeral the subject using a similar procedure to solving linear equations, i.e.
whatever has been done to the pronumeral required is undone.
Literal equations
Literal equations are solved using the same techniques as for solving numerical equations
or transposing formulas, i.e. by transposing the literal equation to make the required
pronumeral the subject.
Multiple-choice questions
1 For non-zero values of x and y, if 5x + 2y = 0 then the ratio
y
x
is equal to
A
5
2
B
2
5
C
2
5
D 1 E
5
4
2 The solution of the simultaneous equations 3x +2y = 36 and 3x y = 12 is
A x =
20
3
y = 8 B x = 2 y = 0 C x = 1 y = 3
D x =
20
3
y = 6 E x =
3
2
y =
3
2
3 The solution of the equation t 9 = 3t 17 is
A t = 4
B t =
11
2
C t = 4 D t = 2 E t = 2
4 For m =
n p
n + p
, p =
A
n(1 m)
1 +m
B
n(m 1)
1 +m
C
n(1 +m)
1 m
D
n(1 +m)
m 1
E
m(n 1)
m +1
P1: FXS/ABE P2: FXS
9780521740494c02.xml CUAU033-EVANS August 21, 2009 15:24
R
e
v
i
e
w
58 Essential Advanced General Mathematics
5
3
x 3

2
x +3
=
A 1 B
x +15
x
2
9
C
15
x 9
D
x +3
x
2
9
E
1
9
6 9x
2
y
3
(15(xy)
3
) is equal to
A
9x
15
B
18xy
5
C
3y
5x
D
3x
5
E
3
5x
7 For the formula V =
1
3
h(l +w)
A l =
hw
3V
B l =
3V
h
w C l =
3V 2w
h
D l =
3Vh
2
w E l =
1
3
h(V +w)
8
(3x
2
y
3
)
2
2x
2
y
=
A
9
2
x
2
y
7
B
9
2
x
2
y
5
C
9
2
x
6
y
7
D
9
2
x
6
y
6
E
9
2
x
6
y
7
9 If X is 50% greater than Y and Y is 20% less than Z, then
A X is 30% greater than Z B X is 20% greater than Z
C X is 20% less than Z D X is 10% less than Z E X is 10% greater than Z
10 The average of two numbers is 5x + 4. One of the numbers is x. The other number is
A 4x +4 B 9x +8 C 9x +4 D 10x +8 E 3x +1
Short-answer questions (technology-free)
1 Simplify the following.
a (x
3
)
4
b ( y
12
)
3
4
c 3x
3
2
5x
4
d (x
3
)
4
3
x
5
2 Express the product 32 10
11
12 10
5
in standard form.
3 Simplify the following.
a
3x
5
+
y
10

2x
5
b
4
x

7
y
c
5
x +2
+
2
x 1
d
3
x +2
+
4
x +4
e
5x
x +4
+
4x
x 2

5
2
f
3
x 2

6
(x 2)
2
4 Simplify the following.
a
x +5
2x 6

x
2
+5x
4x 12
b
3x
x +4

12x
2
x
2
16
c
x
2
4
x 3

3x 9
x +2

9
x +2
d
4x +20
9x 6

6x
2
x +5

2
3x 2
5 The human body can produce 2.5 million blood cells per second. If a person donates
500 mL of blood, how long will it take to replace the red blood cells if a litre of blood
contains 5 10
12
red blood cells?
P1: FXS/ABE P2: FXS
9780521740494c02.xml CUAU033-EVANS August 21, 2009 15:24
R
e
v
i
e
w
Chapter 2 Algebra I 59
6 The Sun is approximately 1.5 10
8
km from Earth and a comet is approximately
3 10
6
km from Earth. How many times further from Earth than the comet is the Sun?
7 Swifts Creek Soccer Team has played 54 matches over the past three seasons. They have
drawn one third of their games and won twice as many games as they have lost. How many
games have they lost?
8 A music store specialises in three types of CDs: classical, blues and heavy metal. In one
week they sold a total of 420 CDs. They sold 10% more classical than blues while sales of
heavy metal constituted 50% more than the combined sales of classical and blues CDs.
How many of each type of CD did they sell?
9 The volume (V) of a cylinder is given by the formula V = r
2
h, where r is the radius of the
base and h is the height of the cylinder.
a Find the volume of a cylinder with base radius 5 cm and height 12 cm.
b Transpose the formula to make h the subject and hence nd the height of a cylinder with
a base radius 5 cm and a volume of 585 cm
3
.
c Transpose the formula to make r the subject and hence nd the radius of a cylinder with
a height of 6 cm and a volume of 768 cm
3
.
10 Solve for x.
a xy +ax = b b
a
x
+
b
x
= c
c
x
a
=
x
b
+2
d
a dx
d
+b =
ax +d
b
11 Simplify
a
p
p +q
+
q
p q
b
1
x

2y
xy y
2
c
x
2
+ x 6
x +1

2x
2
+ x 1
x +3
d
2a
2a +b

2ab +b
2
ba
2
12 A is three times as old as B. In three years time, B will be three times as old as C. In fteen
years time, A will be three times as old as C. What are their present ages?
13 a Solve the following simultaneous equations for a and b.
a 5 =
1
7
(b +3) b 12 =
1
5
(4a 2)
b Solve the following simultaneous equations for x and y.
( p q)x +( p +q)y = ( p +q)
2
qx py = q
2
pq
14 A man has to travel 50 km in 4 hours. He does it by walking the rst 7 km at x km/h,
cycling the next 7 km at 4x km/h and motoring the remainder at (6x + 3) km/h. Find x.
15 Simplify each of the following.
a 2n
2
6nk
2
(3n)
b
8c
2
x
3
y
6a
2
b
3
c
3

1
2
xy
15abc
2
16 Solve the equation
x +5
15

x 5
10
= 1 +
2x
15
P1: FXS/ABE P2: FXS
9780521740494c02.xml CUAU033-EVANS August 21, 2009 15:24
R
e
v
i
e
w
60 Essential Advanced General Mathematics
Extended-response questions
1 Jack cycles home from work, a distance of 10x km. Benny leaves at the same time and
drives the 40x km to his home.
a Write an expression in terms of x for the time taken for Jack to reach home if he cycles
at 8 km/h on average.
b Write an expression in terms of x for the time taken for Benny to reach home if he
drives at an average speed of 70 km/h.
c In terms of x, nd the difference in times of the two journeys.
d If Jack and Benny arrive at their homes 30 minutes apart
i nd x, correct to three decimal places
ii nd the distance from work of each home, correct to the nearest kilometre.
2 Sams plastic dinghy has sprung a leak and water is pouring in the hole at a rate of
27 000 cm
3
per minute. He grabs a cup and frantically starts bailing the water out at a rate
of 9000 cm
3
per minute. The dinghy is shaped like a circular prism (cylinder) with a base
radius of 40 cm and a height of 30 cm.
a How fast is the dinghy lling with water?
b Write an equation showing the volume of water, V cm
3
, in the dinghy after t minutes.
c Find an expression for the depth of water, h cm, in the dinghy after t minutes.
d If Sam is rescued after nine minutes, is this before or after the dinghy has completely
lled with water?
3 Henry and Thomas Wong collect basketball cards. Henry has ve sixths the number of
cards that Thomas has. The Wright family also has a collection of cards. George Wright
has half as many cards again as Thomas, Sally Wright has 18 less than Thomas and Zeb
Wright has one third the number Thomas has.
a Write an expression for each childs number of cards in terms of the number Thomas
has.
b The Wright family owns six more cards than the Wong family. Write an equation
representing this information.
c Solve the above equation and use the result to nd the number of cards each child has
collected.
4 The gravitational force between two objects, F(N), is given by the formula
F =
6.67 10
11
m
1
m
2
r
2
where m
1
and m
2
are the mass (in kilograms) of each object and r (in metres) is the distance
between them.
a What would be the gravitational force between two objects each weighing 200 kg if
they are 12 m apart? Express the answer in standard form (to two signicant gures).
b Transpose the above formula to make m
1
the subject.
c The gravitational force between a planet and an object 6.4 10
6
m away from the
centre of the planet is found to be 2.4 10
4
N. If the object has a mass of 1500 kg,
P1: FXS/ABE P2: FXS
9780521740494c02.xml CUAU033-EVANS August 21, 2009 15:24
R
e
v
i
e
w
Chapter 2 Algebra I 61
calculate the approximate mass of the planet, giving the answer in standard form (two
signicant gures).
5 A water storage reservoir is 3 km wide, 6 km long and 30 m deep. (The water storage
reservoir is assumed to be a cuboid.)
a Write an equation to show the volume of water, V m
3
, in the reservoir when it is d
metres full.
b Calculate the volume of water in the reservoir when it is completely lled. The water
ows from the reservoir down a long pipe to a hydro electric power station in a valley
below. The amount of energy, E(J), that can be obtained from a full reservoir is given by
the formula
E = kVh
where k is a constant and h (m) is the length of the pipe.
c Find k given E = 1.06 10
15
when h = 200, expressing the answer in standard form
correct to three signicant gures.
d How much energy could be obtained from a full reservoir if the pipe was 250 m long?
e If the rate of water falling through the pipe is 5.2 m
3
/s, how many days without rain
could the station operate before emptying an initially full reservoir?
6 A new advertising symbol is to consist of three concentric
circles as shown, with the outer circle having a radius of 10 cm.
It is desired that the three coloured regions cover the same area.
Find the radius of the innermost circle in the gure shown. Yellow
Blue
Red
7 Temperatures of Fahrenheit (F) can be converted to
Celsius (C) by the formula
C =
5
9
(F 32)
Find the temperature which has the same numerical value in both scales.
8 A cyclist goes up a long slope at a constant speed of 15 km/h. He turns around and
comes down the slope at constant speed of 40 km/h. Find his average speed over a full
circuit.
9 A container has a cylindrical base and a hemispherical
top as shown in the gure at right. The height of the
container is 20 cm and its capacity is to be exactly
2 litres. Let r cm be the radius length of the base.
20 cm
r cm
a Express the height of the cylinder in terms of r.
b i Express the volume of the container in terms of r.
ii Find r and h if the volume is two litres.
P1: FXS/ABE P2: FXS
9780521740494c02.xml CUAU033-EVANS August 21, 2009 15:24
R
e
v
i
e
w
62 Essential Advanced General Mathematics
10 a Two bottles contain mixtures of wine and water. In bottle A there is two times as much
wine as water. In bottle B there is ve times as much water as wine. Bottle A and bottle
B are used to ll a third bottle which has a capacity of one litre. How much liquid must
be taken from each of bottle A and bottle B if the third bottle is to contain equal
amounts of wine and water?
b Repeat for the situation where the ratio of wine to water in bottle A is 1 : 2 and for bottle
B the ratio is to 3 : 1.
c Generalise the result for the ratio m : n for bottle A and p : q for bottle B.
11 A cylinder is placed so as to t into a cone as shown. The height of the cone is 20 cm and
the radius 10 cm.
The radius of the cylinder is r cm and the height h cm.
r cm
h cm
20 cm
a Use similar triangles to nd h in terms of r.
h cm
r cm
20 cm
10 cm
b The volume of the cylinder is given by the formula V = r
2
h. Find the volume of the
cylinder in terms of r.
c Use a graphics calculator to nd the values of r and h for which the volume of the
cylinder is 500 cm
3
.
P1: FXS/ABE P2: FXS
9780521740494c03.xml CUAU033-EVANS August 21, 2009 15:27
C H A P T E R
3
Number systems
and sets
Objectives
To understand and use the notation of sets including the symbols
, , , , ,
To be able to identify sets of numbers including natural numbers, integers, rational
numbers, irrational numbers, real numbers
To know and be able to apply the rules for:
r
simplification of surds
r
operations on surds
r
rationalisation of surds
To know the definition of factor, prime, highest common factor
To be able to solve problems with sets
Introduction
This chapter introduces set notation and discusses sets of numbers and their properties. Set
notation is used widely in mathematics and in this book it is employed where appropriate.
A set is a general name for any collection of things or numbers. There must be a way of
deciding whether any particular object is a member of the set or not. This may be done by
referring to a list of the members of the set or a statement describing them.
For example, A = {3, 3} = {x : x
2
= 9}
Note: {x: . . .} is read as the set of objects x such that . . ..
Number systems
Recall that the elements of {1, 2, 3, 4, . . .} are called natural numbers and the elements
{. . . , 2, 1, 0, 1, 2, . . .} are called integers.
The numbers of the form
p
q
with p and q integers, q = 0 , are called rational numbers.
63
P1: FXS/ABE P2: FXS
9780521740494c03.xml CUAU033-EVANS August 21, 2009 15:27
64 Essential Advanced General Mathematics
The real numbers which are not rational are called irrational, e.g. and

2.
The set of real numbers will be denoted by R.
The set of rational numbers will be denoted by Q.
The set of integers will be denoted by Z.
The set of natural numbers will be denoted by N.
R
2
= {(x, y) : x, y R}
These sets of numbers will be discussed further in Sections 3.2 and 3.3.
3.1 Set notation
The symbol means is a member of or is an element of. For example, 3 {prime
numbers} is read 3 is a member of the set of prime numbers.
The symbol / means is not a member of or is not an element of. For example,
4 / {prime numbers} is read 4 is not a member of the set of prime numbers.
Two sets are equal if they contain exactly the same elements, not necessarily in the same
order. For example, if A = {prime numbers less than 10} and B = {2, 3, 5, 7} then A = B.
Two sets A and B are equivalent if they have the same number of elements. For example,
{1, 2, 3} {a, b, c}.
A set which has no elements is called the empty or null set and is denoted by .
The universal set will be denoted by . The universal set is the set of all elements which are
being considered.
If all the elements of a set B are also members of a set A, then the set B is called a subset
of A. This is written B A. For example, {1, 2, 3} Z where Z is the set of integers, and
{3, 9, 27) {multiples of 3}. We note also A A and A.
Venn diagrams are used to illustrate sets. For example, let denote the set of all real
numbers less than 100, A denote the set of real numbers less than 50 and B the set of real
numbers between 90 and 100 (non-inclusive).
This may be illustrated by a Venn diagram.
A

B
A and B have no elements in common.
Two such sets are called disjoint sets.
The union of two sets
The set of elements that are in either set A or set B (or both) is the union of sets A and B. The
union of A and B is written A B.
P1: FXS/ABE P2: FXS
9780521740494c03.xml CUAU033-EVANS August 21, 2009 15:27
Chapter 3 Number systems and sets 65
Example 1
Let = {1, 2, 3, 4, 5, 6, 7, 8, 9, 10}, A = {1, 2, 3} and B = {1, 2, 9, 10}. Find A B and
illustrate on a Venn diagram.
4
3
9 1
2 10
5 6 7 8
B A

Solution
A B = {1, 2, 3, 9, 10}
The shaded area illustrates A B.
The intersection of two sets
The set of all the elements that are members both of set A and of set B is called the
intersection of A and B. The intersection of A and B is written A B.
Example 2
Let ={prime numbers less than 40}. If A = {3, 5, 7, 11} and B = {3, 7, 29, 37}, nd A B.
2 17 19 23 31 13
29
37 11
5
A B
3
7

Solution
A B = {3, 7}
Complement
If = {students at Highland Secondary College} and A = {students with blue eyes}, then the
complement of A is the set of all members of that are not members of A. In this case the
complement is the set of all students of Highland Secondary College that do not have blue
eyes. The complement of A is denoted by A

.
Similarly, if = {1, 2, 3, 4, 5, 6, 7, 8, 9, 10} and A = {1, 3, 5, 7, 9} then
A

= {2, 4, 6, 8, 10}.
Finite and infinite sets
When all the elements of a set may be counted the set is called a nite set, e.g. A = {months of
the year}. The number of elements of a set A will be denoted n(A). In this example n(A) = 12.
For the set C = {letters of the alphabet}, n(C) = 26.
Sets which are not nite are innite sets. For example, R, the set of real numbers, and Z, the
set of integers, are innite sets.
P1: FXS/ABE P2: FXS
9780521740494c03.xml CUAU033-EVANS August 21, 2009 15:27
66 Essential Advanced General Mathematics
Example 3
Given = {1, 2, 3, 4, 5, 6, 7, 8, 9, 10}
A = {odd numbers} = {1, 3, 5, 7, 9}
B = {multiples of 3} = {3, 6, 9}
show these sets on a Venn diagram.
Use the diagram to list the following sets.
a A

b B

c A B
d the complement of A B i.e. (A B)

e A

2 4
5
7
6 1
8 10
A B

3
9
Solution
(Each of the numbers in the given
sets is placed in the correct position
on this Venn diagram.)
From the diagram: a A

= {2, 4, 6, 8, 10}
b B

= {1, 2, 4, 5, 7, 8, 10} c A B = {1, 3, 5, 6, 7, 9}


d (A B)

= {2, 4, 8, 10} e A

= {2, 4, 8, 10}
Exercise 3A
1 = {1, 2, 3, 4, 5}, A = {1, 2, 3, 5}, B = {2, 4}.
Example 1
Show these sets on a Venn diagram and use the diagram to nd
a A

b B

c A B d (A B)

e A

2 = {natural numbers less than 17}, P = {multiples of 3}, Q = {even numbers} Show
Example 2
these sets on a Venn diagram and use it to nd
a P

b Q

c P Q d (P Q)

e P

3 = {1, 2, 3, 4, 5, 6, 7, 8, 9, 10, 11, 12}, A = {multiples of 4}, B = {even numbers} Show


Example 3
these sets on a Venn diagram and use this diagram to list the sets
a A

b B

c A B d (A B)

e A

4 = {natural numbers from 10 to 25}, P = {multiples of 4}, Q = {multiples of 5} Show


these sets on a Venn diagram and use this diagram to list the sets
a P

b Q

c P Q d (P Q)

e P

5 = {different letters in the word GENERAL},


A = {different letters in the word ANGEL},
B = {different letters in the word LEAN}
Show these sets on a Venn diagram and use this diagram to list the sets
a A

b B

c A B d A B e (A B)

f A

P1: FXS/ABE P2: FXS


9780521740494c03.xml CUAU033-EVANS August 21, 2009 15:27
Chapter 3 Number systems and sets 67
6 = { p, q, r, s, t, u, v, w}, X = {r, s, t, w}, Y = {q, s, t, u, v}
Show , X and Y on a Venn diagram, entering all members. Hence list the sets
a X

b Y

c X

d X

e X Y f (X Y)

Which two sets are equal?


7 = {1, 2, 3, 4, 5, 6, 7, 8, 9, 10, 11, 12}, X = {factors of 12},
Y = {even numbers}
Show , X and Y on a Venn diagram entering all members. Hence list the sets
a X

b Y

c X

d X

e X Y f (X Y)

Which two sets are equal?


8 Draw this diagram six times. Use
shading to illustrate each of the
following sets.
A B

a A

b B

c A

d A

e A B f (A B)

9 = {different letters in the word MATHEMATICS}


A = {different letters in the word ATTIC}
B = {different letters in the word TASTE}
Show , A and B on a Venn diagram entering all the elements. Hence list the sets
a A

b B

c A B d (A B)

e A

f A

3.2 Sets of numbers


The following notation was introduced earlier in this chapter.
R denotes the set of real numbers.
Q denotes the set of rational numbers.
Z denotes the set of integers.
N denotes the set of natural numbers.
A geometric construction of a line segment of
length
m
n
where m and n are non-zero integers is
shown in Chapter 9. Constructions of products
and quotients are also shown in that chapter.
It is clear that N Z Q R and this may
be represented by the diagram shown.
R
Q
Z
N
The set of all x such that (. . .) is denoted by {x : (. . .)}, where (. . .) stands for some condition.
Thus
{x : 0 < x < 1} is the set of all real numbers between 0 and 1.
{x : x > 0, x Q} is the set of all positive rational numbers.
{2n : n = 1, 2, 3, . . .} is the set of all even numbers.
P1: FXS/ABE P2: FXS
9780521740494c03.xml CUAU033-EVANS August 21, 2009 15:27
68 Essential Advanced General Mathematics
Examples of irrational numbers are

3,

2, , +2,

7 +

6. These numbers cannot be


written in the form
p
q
where p and q are integers. The decimal representation of these numbers
does not terminate or repeat.
Rational numbers
Every rational number can be expressed as a terminating or recurring decimal. For example
1
2
= 0.5,
1
5
= 0.2,
1
10
= 0.1,
1
3
= 0.

3,
1
7
= 0.

14285

7
Numbers of the form
m
n
, where m and n N have a terminating decimal representation if and
only if n = 2

, where and are members of the set N {0}.


In order to nd the decimal representation of a rational number
m
n
, the division m n is
undertaken. For example
a
1
20
0.05
20

1.00
Therefore
1
20
= 0.05
b
3
7
0.428571428 . . .
7

3.000000000 . . .
Therefore
3
7
= 0.

42857

1
The method to nd a rational number
m
n
from its decimal representation is demonstrated in the
following example.
Example 4
Write each of the following in the form
m
n
.
a 0.05 b 0.

42857

1
Solution
a 0.05 =
5
100
=
1
20
b 0.

42857

1 = 0.428571428571 . . . . . . 1
Multiply both sides by 10
6
0.

42857

1 10
6
= 428571.428571428571 . . . . . . 2
Subtract 1 from 2
0.

42857

1 (10
6
1) = 428571
0.

42857

1 =
428571
10
6
1
=
3
7
Irrational numbers
The set of irrational numbers has two important subsets, algebraic numbers and
transcendental numbers.
Algebraic numbers are those which are the solutions of an equation of the form
a
0
x
n
+a
1
x
n1
+ +a
n
= 0, where a
0
, a
1
, . . . . , a
n
are integers.
P1: FXS/ABE P2: FXS
9780521740494c03.xml CUAU033-EVANS August 21, 2009 15:27
Chapter 3 Number systems and sets 69
For example,

2 is an algebraic number, as it is a solution of the equation


x
2
2 = 0
is not an algebraic number, it is a transcendental number.
The proof that

2 is irrational is presented here. The proof is by contradiction.


Assume

2 =
a
b
, where a, b N
and
a
b
is a fraction in simplest form.
Then
a
2
b
2
= 2
a
2
=2b
2
which implies a
2
is even which implies a is even
a =2k for some k N
a
2
=4k
2
4k
2
=2b
2
b
2
=2k
2
which implies b
2
is even which implies b is even
But this contradicts the assumption that
a
b
is a fraction in simplest form, as a and b are both
divisible by 2.
Exercise 3B
1 Is the
a sum b product c quotient (if dened)
of two rational numbers rational?
2 Is the
a sum b product c quotient
of two irrational numbers irrational?
3 Write each of the following in the form
m
n
where m and n are integers.
Example 4
a 0.

7 b 0.12 c 0.

28571

4 d 0.

6 e 0.

2 f 0.45
4 Give the decimal representation of each of the following rational numbers.
a
2
7
b
5
11
c
7
20
d
4
13
e
1
17
5 Prove that

3 is not a rational number.


3.3 Surds
A number of the form

a where a is a rational number which is not a square of another


rational number is called a quadratic surd.
Note:

a is taken to mean the positive square root.


P1: FXS/ABE P2: FXS
9780521740494c03.xml CUAU033-EVANS August 21, 2009 15:27
70 Essential Advanced General Mathematics
If a is a rational which is not a perfect nth power,
n

a is called a surd of the nth order.

2,

7,

24,

9
7
,

1
2
are quadratic surds

9,

16,

9
4
are not surds
3

7,
3

15 are surds of the third order


4

100,
4

26 are surds of the fourth order


Quadratic surds hold a prominent position in school mathematics. For example, the solutions
of quadratic equations often involve surds, e.g.
x =
1
2
+
1
2

5 is a solution of the quadratic equation x


2
x 1 = 0
Values of trigonometric functions sometimes involve surds, e.g.
sin 60

3
2
, sin 15

2
4
In Mathematical Methods Units 3 and 4 and Specialist Mathematics Units 3 and 4, exact
solutions are often required.
Lengths such as

2,

3 or

6 can be constructed geometrically, using a straight edge and a


compass. For example, from the right-angled isosceles triangle ABC:
1
1
2
B
A
C

1
3
B
A
C

2
The length AB =

2 The length AB =

3
Any quadratic surd (of a natural number) may be constructed in this way. This makes it
possible to construct a line segment of the length determined by the solutions of many
quadratic equations.
For example, one solution of x
2
x 1 = 0 is x =
1
2
+
1
2

5.
The construction of a line segment of this length involves the right angled triangle XYZ.
5
2
1
X
Z
Y

1 Y X Z
5
The length is now achieved by bisecting the line segment.
P1: FXS/ABE P2: FXS
9780521740494c03.xml CUAU033-EVANS August 21, 2009 15:27
Chapter 3 Number systems and sets 71
Properties of square roots
The following properties of square roots are often used.

ab =

a

b, e.g.

50 =

25

2 = 5

a
b
=

b
, e.g.

7
9
=

9
=

7
3
Properties of surds
Simplest form
If possible, a factor is taken out of a square root. When the number under the square root has
no factors which are squares of a rational number, then the surd is said to be in simplest form.
Example 5
Write each of the following in simplest form.
a

72 b

28 c

700
117
d

99
64
Solution
a

72 =

36

2 = 6

2 b

28 =

4 7 = 2

7
c

700
117
=

700

117
=

7 100

9 13
=
10
3

7
13
d

99
64
=

99

64
=

9 11
8
=
3

11
8
Surds which have the same irrational factor are called like surds.
For example, 3

7, 2

7 and

7 are like surds.


The sum or difference of two like surds can be found.
i.e. m

p +n

p = (m +n)

p and m

p n

p = (m n)

p
Example 6
Express each of the following as a single surd in simplest form.
a

147 +

108

363 b

3 +

5 +

20 +

27

45

48
c

1
8

1
18
5

1
72
d

50 +

2 2

18 +

8
Solution
a

147 +

108

363
=

7
2
3 +

6
2
3

11
2
3
= 7

3 +6

3 11

3
= 2

3
P1: FXS/ABE P2: FXS
9780521740494c03.xml CUAU033-EVANS August 21, 2009 15:27
72 Essential Advanced General Mathematics
b

3 +

5 +

20 +

27

45

48
=

3 +

5 +2

5 +3

3 3

5 4

3
= 0

3 +0

5
= 0
c

1
8

1
18
5

1
72
=

1
4 2

1
9 2
5

1
36 2
=
1
2

1
2

1
3

1
2

5
6

1
2
=
3
6

1
2

2
6

1
2

5
6

1
2
=
4
6

1
2
=
2
3

1
2
d

50 +

2 2

18 +

8
= 5

2 +

2 2 3

2 +2

2
= 8

2 6

2
= 2

2
Rationalising the denominator
In the past, a labour saving procedure with surds was to rationalise any surds in the
denominator of an expression. It is still considered to be a neat way of expressing nal answers.
For

5 a rationalising factor is

5 as

5 = 5
For

2 +1 a rationalising factor is 1

2 as (1 +

2)(1

2) = 1 2 = 1
For

3 +

6 a rationalising factor is

6 as (

3 +

6) (

6) = 3 6 = 3
Example 7
Rationalise the denominator of each of the following.
a
1
2

7
b
1
2

3
c
1

6
d
3 +

8
3

8
Solution
a
1
2

7
=

7
14
b
1
2

3

2 +

3
2 +

3
=
2 +

3
4 3
= 2 +

3
c
1

3 +

3 +

6
=

3 +

6
3 6
=
1
3
(

3 +

6)
d
3 +

8
3

8
=
3 +2

2
3 2

2

3 +2

2
3 +2

2
=
9 +12

2 +8
9 8
= 17 +12

2
P1: FXS/ABE P2: FXS
9780521740494c03.xml CUAU033-EVANS August 21, 2009 15:27
Chapter 3 Number systems and sets 73
Example 8
Expand the brackets in each of the following and collect like terms, expressing surds in
simplest form.
a (3

2)
2
b (3

2)(1 +

2)
Solution
a (3

2)
2
= (3

2)(3

2)
= 3(3

2)

2(3

2)
= 9 3

2 3

2 +2
= 11 6

2
b (3

2)(1 +

2)
= 3(1 +

2)

2(1 +

2)
= 3 +3

2 2
= 1 +2

2
Using the TI-Nspire
A CAS calculator can be used to work with
irrational numbers.
Expressions can be reached and selected
using the up arrow ( ). This returns the
expression to the entry line and modications
can be made.
To illustrate this, evaluate
2
3
2
2
5
2
8
5
as
shown.
To nd the square root of this expression,
type / and move upwards by pressing
the up arrow ( ) so that the expression is
highlighted.
Press enter (
enter
) to paste this expression into
the square root sign and press enter once
more to evaluate the square root of this
expression.
P1: FXS/ABE P2: FXS
9780521740494c03.xml CUAU033-EVANS August 21, 2009 15:27
74 Essential Advanced General Mathematics
Exercise 3C
1 Express each of the following in terms of the simplest possible surds.
Example 5
a

8 b

12 c

27 d

50 e

45 f

1210
g

98 h

108 i

25 j

75 k

512
2 Simplify each of the following.
Example 6
a

8 +

18 2

2 b

75 +2

12

27
c

28 +

175

63 d

1000

40

90
e

512 +

128 +

32 f

24 3

216 +

294
3 Simplify each of the following.
a

75 +

108 +

14 b

847

567 +

63
c

720

245

125 d

338

288 +

363

300
e

12 +

8 +

18 +

27 +

300 f 2

18 +3

50 +

20

80
4 Express each of the following with rational denominators.
Example 7
a
1

5
b
1

7
c
1

2
d
2

3
e
3

6
f
1
2

2
g
1

2 +1
h
1
2

3
i
1
4

10
j
2

6 +2
k
1

3
l
3

5
m
1
3 2

2
5 Express each of the following in the form a +b

c.
Example 8
a
2
3 2

2
b (

5 +2)
2
c (1 +

2)(3 2

2) d (

3 1)
2
e

1
3

1

27
f

3 +2
2

3 1
g

5 +1

5 1
h

8 +3

18 +2
6 Expand and simplify each of the following.
a (2

a 1)
2
b (

x +1 +

x +2)
2
7 For real numbers a and b, a > b if and only if a b > 0. Use this to state the larger of
a 5 3

2 and 6

2 8 b 2

6 3 or 7 2

6
3.4 Natural numbers
Factors and composites
The factors of 8 are 1, 2, 4 and 8.
The factors of 24 are 1, 2, 3, 4, 6, 8, 12 and 24.
The factors of 5 are 1 and 5.
P1: FXS/ABE P2: FXS
9780521740494c03.xml CUAU033-EVANS August 21, 2009 15:27
Chapter 3 Number systems and sets 75
A natural number, a, is a factor of a natural number, b, if there exists a natural number, k,
such that b = ak.
If a number greater than 1 has only factors 1 and itself, it is said to be prime.
Among the rst 100 numbers, the following are prime:
2, 3, 5, 7, 11, 13, 17, 19, 23, 29, 31, 37, 41, 43, 47, 53, 59, 61, 67, 71, 73, 79, 83, 89, 97
A number, m, is called a composite if it can be written as a product, m = a b where a and
b are numbers greater than 1 and less than m.
Prime decomposition
3000 = 3 5
3
2
3
2294 = 2 31 37
This method of uniquely expressing a composite in terms of a product of powers of prime
numbers is called decomposition. It is useful for nding factors of numbers.
For example, the prime decomposition of 12 is given by 12 = 2
2
3
The factors of 12 are 1, 2, 2
2
, 3, 2 3, 2
2
3, i.e. 1, 2, 4, 3, 6, 12.
Example 9
Give the prime decomposition of 17 248 and hence list the factors of this number.
Solution
The prime decomposition can be determined by repeated division.
2 17248
2 8624
2 3112
2 2156
2 1078
7 539
7 77
11 11
1
The prime decomposition of 17248 is 17248 = 2
5
7
2
11
The factors can systematically be determined in the following way.
2
5
, 2
4
, 2
3
, 2
2
, 2, 1
2
5
7, 2
4
7, 2
3
7, 2
2
7, 2 7, 7
2
5
7
2
, 2
4
7
2
, 2
3
7
2
, 2
2
7
2
, 2 7
2
, 7
2
2
5
11, 2
4
11, 2
3
11, 2
2
11, 2 11, 11
2
5
7 11, 2
4
7 11, 2
3
7 11, 2
2
7 11, 2 7 11, 7 11
2
5
7
2
11, 2
4
7
2
11, 2
3
7
2
11, 2
2
7
2
11, 2 7
2
11, 7
2
11
P1: FXS/ABE P2: FXS
9780521740494c03.xml CUAU033-EVANS August 21, 2009 15:27
76 Essential Advanced General Mathematics
Highest common factor (greatest
common divisor)
The highest common factor of two natural numbers is the largest natural number which is a
factor of both the numbers. For example, the highest common factor of 15 and 24 is 3. The
prime decomposition can be used to nd the highest common factor of two numbers.
Consider the numbers 140 and 110. The prime factorisations of these numbers are
140 = 2
2
5 7 and 110 = 2 5 11.
The number which is a factor of 140 and 110 must have prime factors which occur in both
factorisations. The exponent (power) of each of these prime factors will be the smaller of
the two exponents occurring in the factorisation of 140 and 110. Thus the highest common
factor = 2 5 = 10.
Example 10
Find the highest common factor of 528 and 3168.
Solution
528 = 2
4
3 11 and 3168 = 2
5
3
2
11
highest common factor = 2
4
3 11 = 528
Example 11
Find the highest common factor of 3696 and 3744.
Solution
3696 = 2
4
3 7 11 and 3744 = 2
5
3
2
13
highest common factor = 2
4
3 = 48
Using the TI-Nspire
The calculator can be used to factor Natural numbers
by using factor( ) from the Algebra menu (b3
2) as shown.
The highest common factor (also called greatest
common divisor) of two numbers can be found using
the command gcd( ) from the Number menu (b2
4) as shown.
Note how nested gcd( ) may be used to nd the
greatest common divisor of several numbers.
P1: FXS/ABE P2: FXS
9780521740494c03.xml CUAU033-EVANS August 21, 2009 15:27
Chapter 3 Number systems and sets 77
Using the Casio ClassPad
To nd the highest common factor, tap
Interactive, Calculation, gcd and enter the
required numbers in the two lines provided.
Exercise 3D
1 Give the prime decomposition of each of the following numbers.
Example 9
a 68 640 b 96 096 c 32 032 d 544 544
2 Find the highest common factor of each of the following pairs of numbers.
Examples 10, 11
a 4361, 9281 b 999, 2160 c 5255, 716 845
d 1271, 3875 e 804, 2358
Note: Extended-answer questions 5, 6 and 7 are concerned with natural numbers.
3.5 Problems involving sets
Sets can be used to help sort information, as each of the following examples demonstrates.
Example 12
Two hundred and eighty students either travel by train or tram or both to get to school. One
hundred and fty travel by train and 60 travel by both train and tram.
a Show this information on a Venn diagram.
b Hence nd the number of students who travel by
i tram ii train but not tram iii just one of these modes of transport.
P1: FXS/ABE P2: FXS
9780521740494c03.xml CUAU033-EVANS August 21, 2009 15:27
78 Essential Advanced General Mathematics
Solution
a
b i n(TRAM) = 130 +60 = 190 ii n(TRAIN (TRAM)

) = 90
iii n(TRAIN (TRAM)

) +n((TRAIN)

TRAM) = 90 +130 = 220


Example 13
An athletics team consists of 18 members. Each member performs in at least one of three
events, sprints (S), jumps (J) and hurdles (H). Every hurdler either jumps or sprints. Also the
following information is available.
n(S) =11, n(J) =10, n(J H

) =5, n(J

S) =5 and n(J H

) =7
a Draw a Venn diagram.
b Find
i n(H) ii n(S H J) iii n(S J) iv n(S J H

)
Solution
a
The information given above can be summarised as equations in terms of p, q, r,
w, x, y, z.
x + y + z +w = 11 as n(S) = 11 . . . 1
p +q + z +w = 10 as n(J) = 10 . . . 2
x + y + z +w + p +q +r = 18 as all members compete . . . 3
p = 5 as n(J H

) = 5 . . . 4
s = 5 as n(J

S) = 5 . . . 5
r = 0 as every hurdler either jumps or sprints . . . 6
w + p = 7 as n(J H

) = 7 . . . 7
From 4 and 7 , w = 2
Equation 3 becomes
5 + y + z +2 +5 +q = 18
i.e. y + z +q = 6 . . . 8
P1: FXS/ABE P2: FXS
9780521740494c03.xml CUAU033-EVANS August 21, 2009 15:27
Chapter 3 Number systems and sets 79
Equation 1 becomes
y + z = 4
Therefore from 8, q = 2
Equation 2 becomes
5 +2 + z +2 = 10
z = 1
and y = 3
The Venn diagram can be completed.
5 2 5
3
1
2
0
H
J S

b i n(H) = 6 ii n(S H J) = 1
iii n(S J) = 18 iv n(S J H

) = 2
Exercise 3E
1 There are 28 students in a class all of whom take History or Economics or both. Fourteen
Example 12
take History, ve of whom also take Economics.
a Show this information on a Venn diagram.
b Hence nd the number of students who take
i Economics ii History but not Economics
iii just one of these subjects.
2 a Draw a Venn diagram to show three sets, A, B and C in a universal set . Enter
numbers in the correct parts of the diagram using the following information.
n(A B C) = 2, n(A B) = 7, n(B C) = 6,
n(A C) = 8, n(A) = 16, n(B) = 20, n(C) = 19, n() = 50
b Use the diagram to nd
i n(A

) ii n(A B

) iii n(A

B C

)
3 In a border town in the Balkans, 60% of people speak Bulgarian, 40% speak Greek and
20% speak neither. What percentage of the town speak both Bulgarian and Greek?
4 A survey of a class of 40 students showed that 16 own at least one dog and 25 at least one
cat. Six students had neither. How many students own both?
P1: FXS/ABE P2: FXS
9780521740494c03.xml CUAU033-EVANS August 21, 2009 15:27
80 Essential Advanced General Mathematics
5 At an international conference there were 105 delegates. Seventy spoke English, 50 spoke
French and 50 spoke Japanese. Twenty ve spoke English and French, 15 spoke French
and Japanese and 30 spoke Japanese and English.
a How many delegates spoke all three languages?
b How many spoke Japanese only?
6 A restaurant serves 350 people lunches. It offers three desserts, proteroles, gelati and
fruit. It is found that 40 people have all three desserts, 70 have gelati only, 50 have
proteroles only and 60 have fruit only. Forty ve people have fruit and gelati only, 30
people have gelati and proteroles only and 10 people have fruit and proteroles only.
How many people do not have a dessert?
7 Forty travellers were questioned about the various methods of transport they had used the
Example 13
previous day. Every traveller questioned travelled by at least one of the following: car (C),
bus (B), train (T).
Of those questioned, eight had used all three methods of transport.
Four had travelled by bus and car only.
Two had travelled by car and train only.
The number (x) who had travelled by train only was equal to the number who had
travelled by bus and train only.
If 20 travellers had used a train and 33 had used a bus, nd
a the value of x b the number who travelled by bus only
c the number who travelled by car only.
8 is the set of integers and
X = {x : 21 < x < 37}
Y = {3y : 0 < y 13}
Z = {z
2
: 0 < z < 8}
a Draw a Venn diagram representing the information.
b Find
i X Y Z ii n(X Y)
9 A number of students bought red, green and black pens. Three bought one of each colour.
Of the students who bought two colours, three did not buy red, ve not green and two not
black. The same number of students bought red only as bought red with other colours.
The same number bought black only as bought green only. More students bought red and
black but not green than bought black only. More bought only green than bought green
and black but not red. How many students were there and how many pens of each colour
were sold?
P1: FXS/ABE P2: FXS
9780521740494c03.xml CUAU033-EVANS August 21, 2009 15:27
Chapter 3 Number systems and sets 81
10 For three subsets, B, M and F of a universal set
n(B M) = 12, n(M F B) = n(F

), n(F B) > n(M F),


n(B F

) = 5, n(M B

) = 5, n(F M

) = 5,
n() = 28
Find n(M F).
11 A group of 80 students were interviewed about which sport they played. It was found that
23 did athletics, 22 swim and 18 play football. If 10 people do athletics and swim only
and 11 people do athletics and play football only, six people both swim and play football
only and 46 people do none of these activities on a regular basis, how many people do all
three?
12 At a certain secondary college students have to be procient in at least one of the
languages Italian, French or German. In a particular group of 33 students, two are
procient in all three languages, three in Italian and French only, four in French and
German only and ve in German and Italian only. The number procient in Italian only is
x, in French only is x and in German only is x +1. Find x and the total number procient
in Italian.
13 Two hundred and one students at a certain school studied one or more of Mathematics,
Physics and Chemistry. 35 took Chemistry only, 50% more students studied Mathematics
only than studied Physics only, four studied all three subjects, 25 studied both
Mathematics and Physics but not Chemistry, seven studied both Mathematics and
Chemistry but not Physics and 20 studied both Physics and Chemistry but not
Mathematics. Find the number of students studying Mathematics.
P1: FXS/ABE P2: FXS
9780521740494c03.xml CUAU033-EVANS August 21, 2009 15:27
R
e
v
i
e
w
82 Essential Advanced General Mathematics
Chapter summary
Set notation
is a member of
/ is not a member of
the empty set
the universal set
subset
The union of two sets
The set of elements that are in either set A or set B (or both) is the union of set A and B. The
union of A and B is written A B.
The intersection of two sets
The set of all the elements that are members both of set A and of set B is called the
intersection of A and B. The intersection of A and B is written A B.
The complement of A, written A

, is the set of all members of that are not members of A.


Sets of numbers
R denotes the set of real numbers
Q denotes the set of rational numbers
Z denotes the set of integers
N denotes the set of natural numbers
Note: N Z Q R
Numbers of the form
m
n
, where m, n N, have a terminating decimal representation if,
and only if, n = 2

where , are members of the set N {0}.


Algebraic numbers are those which are the solution(s) of an equation of the form
a
0
x
n
+a
1
x
n1
+ +a
n
= 0, where a
0
, a
1
, . . . , a
n
are integers.
A number of the form

a where a is a rational number which is not a square of another


rational number is called a quadratic surd.
If a is a rational which is not a perfect nth power
n

a is called a surd of the nth order.


When the number under the square root has no factors which are squares of a rational
number, then the surd is said to be in simplest form.
Surds which have the same irrational factor are called like surds.
The sum or difference of two like surds can be found
m

p +n

p = (m +n)

p and m

p n

p = (m n)

p
A natural number, a, is a factor of a natural number, b, if there exists a natural number, k,
such that b = ak.
If a natural number greater than 1 has only factors 1 and itself, it is said to be prime.
A natural number, m, is called a composite if it can be written as a product m = a b
where a and b are natural numbers greater than 1 and less than m.
The highest common factor of two natural numbers is the largest natural number which is
a factor of both numbers.
P1: FXS/ABE P2: FXS
9780521740494c03.xml CUAU033-EVANS August 21, 2009 15:27
R
e
v
i
e
w
Chapter 3 Number systems and sets 83
Multiple-choice questions
1
4
3 +2

2
expressed in the form a +b

2 is
A 12 8

2 B 3 +2

2 C
3
17

8
17

2 D
3
17
+
8
17

2 E 12 +8

2
2 The prime decomposition of 86400 is
A 2
5
3
2
5 B 2
6
3
3
5
2
C 2
7
3
3
5
D 2
7
3
3
5
2
E 2
6
3
3
5
3
3 (

6 +3)(

6 3) is equal to
A 3 12

6 B 3 6

6 C 3 +6

6 D 3 E 3
4 For the Venn diagram shown is the set of positive integers less than 20. A is the set of
positive integers less than 10 and B is the set of positive integers less than 20 divisible by 3.
The set B

A is
1
11 10 13 14 16 17 19
2
7
3
4 5 8
6
9
12
15 18
B
A
A {6, 3, 9}
B {12, 15, 18}
C {10,11,13,14,16,17,19}
D {1, 2, 4, 5, 7, 8}
E {1, 2, 3, 4, 5, 6, 7, 8, 9, 12, 15, 18}
5 (3,) (, 5] =
A (, 3) B (, 5] C (3, 5] D R E [3, 5]
6 A bell is rung every 6 minutes and a gong is sounded every 14 minutes. If these occur
together at a particular time then the smallest number of minutes until the bell and the gong
are again heard simultaneously is
A 10 B 20 C 72 D 42 E 84
7 If X is the set of multiples of 2, Y the set of multiples of 7 and Z the set of multiples of 5
then describe X Y Z =
A the set of multiples of 2 B the set of multiples of 70
C the set of multiples of 35 D the set of multiples of 14
E the set of multiples of 10
8 In a class of students, 50% play football, 40% play tennis and 30% play neither. The
percentage that plays both is
A 10 B 20 C 30 D 50 E 40
9

7 +

6
=
A 5 +2

7 B 13 +2

6 C 13 2

42 D 1 +2

42 E 13 2

13
10 There are 40 students in a class, all of whom take Literature or Economics or both. Twenty
take Literature and ve of these also take Economics. The number of students who only
take Economics is
A 20 B 5 C 10 D 15 E 25
P1: FXS/ABE P2: FXS
9780521740494c03.xml CUAU033-EVANS August 21, 2009 15:27
R
e
v
i
e
w
84 Essential Advanced General Mathematics
Short-answer questions (technology-free)
1 Express the following as fractions in their simplest form.
a 0.0

7 b 0.

5 c 0.005 d 0.405 e 0.2

6 f 0.1

71428

5
2 Express 504 as a product of powers of prime numbers.
3 Express each of the following with a rational denominator.
a
2

3 1

2
b

5 +2

5 2
c

3 +

2
4 Express
3 +2

75
3

12
in the form a +b

3 where a, b R\{0}.
5 Express each of the following with a rational denominator.
a
6

2
3

2 2

3
b

a +b

a b

a +b +

a b
6 In a class of 100 students, 55 are girls, 45 have blue eyes, 40 are blond, 25 are blond girls,
15 are blue-eyed blonds, 20 are blue-eyed girls, and 5 are blue-eyed blond girls. Find
a the number of blond boys
b the number of boys who are not blond or blue-eyed.
7 A group of 30 students received prizes in at least one of the subjects of English,
Mathematics, and French. Two students received prizes in all three subjects. Fourteen
received prizes in English and Mathematics but not French. Two received prizes in English
alone, two in French alone and ve in Mathematics alone. Four received prizes in English
and French but not Mathematics.
a How many received prizes in Mathematics and French but not English?
b How many received prizes in Mathematics?
c How many received prizes in English?
8 Fifty people are interviewed. Twenty-three say they like Brand X, 25 say they like Brand Y,
19 say they like Brand Z. Eleven say they like X and Z. Eight say they like Y and Z. Five say
they like X and Y. Two like all three. How many like none of them?
9 Three rectangles A, B and C overlap (intersect). Their areas are 20 cm
2
, 10 cm
2
and 16 cm
2
respectively. The area common to A and B is 3 cm
2
, that common to A and C is 6 cm
2
, that
common to B and C is 4 cm
2
. How much of the area is common to all three if the total area
covered is 35 cm
2
?
10 Express

112

63
224

28
in simplest form.
11 If

3
x
=
x

7 +

3
, nd the values of x.
12 Express
1 +

5 +

3
+
1

3
in the form a

5 +b

6.
13 Simplify

27

12 +2

75

48
25
.
P1: FXS/ABE P2: FXS
9780521740494c03.xml CUAU033-EVANS August 21, 2009 15:27
R
e
v
i
e
w
Chapter 3 Number systems and sets 85
14 A, B and C are three sets and = A B C. The number of elements in the regions of the
Venn diagram are as shown. Find
32
7 15 3
0
0
A B
C a the number of elements in A B
b the number of elements in C
c the number of elements in B

A.
15 Using the result that (

a +

b)
2
= a +b +2

ab, determine the square root of 17 +6

8.
Extended-response questions
1 a Show that (

x +

y)
2
= x + y +2

xy.
b Substitute x = 3 and y = 5 in the identity of a to show
(

3 +

5) =

8 +2

15
c Use this technique to nd the square root of
i 14 +2

33 (Hint: use x = 11 and y = 3)


ii 15 2

56 iii 51 36

2
2 In this question, {a +b

3 : a, b Q} is considered. Later in this book the set, C, of


complex numbers is introduced, where C = {a +b

1: a, b R}.
a If (2 +3

3) +(4 +2

3) = a +b

3, nd a and b.
b If (2 +3

3)(4 +2

3) = p +q

3, nd p and q.
c If
1
3 +2

3
= a +b

3, nd a and b.
d Solve each of the following equations for x.
i (2 +5

3)x = 2

3 ii (x 3)
2
3 = 0 iii (2x 1)
2
3 = 0
e Explain why every rational number is a member of {a +b

3 : a, b Q}.
3 a Show
1
2 +

3
= 2

3
b Use the substitution t =

2 +

x
and the result of a to show that the equation

2 +

x
+

x
= 4 can be written as t +
1
t
= 4.
c Show that the solutions of the equation are t = 2

3 and t = 2 +

3.
d Use this result to solve the equation

2 +

x
+

x
= 4
4 Use Venn diagrams to illustrate
a n(A B) = n(A) +n(B) n(A B)
b n(A B C) = n(A) +n(B) +n(C) n(A B) n(B C)
n(A C) +n(A B C)
5 The number 2

3 is a root of the quadratic equation with integer coefcients


x
2
+bx +c = 0.
P1: FXS/ABE P2: FXS
9780521740494c03.xml CUAU033-EVANS August 21, 2009 15:27
R
e
v
i
e
w
86 Essential Advanced General Mathematics
a If x = 2

3 is a solution to the equation, nd the values of b and c.


(Hint: Use the result that if m +n

3 = 0 then m = 0 and n = 0, m, n rational.)


b Find the other solution to the quadratic.
c If x
2
+bx +c = 0 and m n

q is a solution, show that


i b = 2m ii c = m
2
n
2
q
and hence that
iii x
2
+bx +c = (x (m n

q))(x (m +n

q))
6 A triple (x, y, z) is said to be Pythagorean if x
2
+ y
2
= z
2
, e.g. (3, 4, 5) is a Pythagorean
triple, (5, 12, 13) is a Pythagorean triple.
All Pythagorean triples may be generated by the following:
x = 2mn, y = m
2
n
2
, z = m
2
+n
2
where m, n N
e.g. if m = 2, n = 1 then x = 4, y = 3, z = 5
a Find the Pythagorean triple for m = 5, n = 2.
b Verify that for x = 2mn, y = m
2
n
2
, z = m
2
+n
2
where m, n N, x
2
+ y
2
= z
2
.
7 The factors of 12 are 1, 2, 3, 4, 6, 12.
a How many factors does each of the following have?
i 2
3
ii 3
7
b How many factors does 2
n
have?
c How many factors does each of the following have?
i 2
3
3
7
ii 2
n
3
m
d Every natural number may be expressed as a product of powers and primes. This is
called prime factorisation, e.g. 1080 = 2
3
3 5.
For x, a natural number, let p

1
1
p

2
2
p

3
3
. . . p

n
n
be the prime factorisation where
i
N
and each p
i
is a prime number.
How many factors does x have? (Answer to be given in terms of
i
.)
e Find the smallest number which has eight factors.
8 The least common multiple of natural numbers m and n is the smallest natural number
which is a multiple of both m and n, e.g. the least common multiple of 4 and 6 is 12.
a Give the prime decomposition of 1080 and 25 200.
b In order to nd the least common multiple of two numbers a and b, take the prime
decomposition of each of the numbers, i.e. a = p

1
1
p

2
2
p

3
3
. . . p

n
n
and
b = p

1
1
p

2
2
p

3
3
. . . p

m
m
and then the least common multiple of a and
b = p
max(
1
,
1
)
1
p
max(
2
,
2
)
2
. . . p
max(
n
,
n
)
n
. . . p
max(
m
,
m
)
m
, where all primes in the prime
decomposition of either a or b are included in this product, e.g. the least common
multiple of 24 = 2
3
3 and 18 = 3
2
2 is 2
3
3
2
= 72. Find the least common
multiple of 1080 and 25 200.
c Carefully explain why if m and n are integers mn = least common multiple of m and
n highest common factor of m and n.
(contd)
P1: FXS/ABE P2: FXS
9780521740494c03.xml CUAU033-EVANS August 21, 2009 15:27
R
e
v
i
e
w
Chapter 3 Number systems and sets 87
d i Find four consecutive even numbers such that the smallest is a multiple of 5, the
second a multiple of 7, the third a multiple of 9 and the largest a multiple of 11.
ii Find four consecutive natural numbers such that the smallest is a multiple of 5, the
second a multiple of 7, the third a multiple of 9 and the largest a multiple of 11.
9 a In the Venn diagram is the set of all students enrolled at Argos Secondary College. Set
R is the set of all students with red hair. Set B is the set of all students with blue eyes. Set
F is the set of all female students.
The numbers on the diagram indicate the eight different regions.
i Identify the region in the Venn diagram which represents male students who have
neither red hair nor blue eyes.
ii Describe the gender, hair colour and eye colour of students represented in
region 1 of the diagram.
iii Describe the gender, hair colour and eye colour of students represented in
region 2 of the diagram.
b It is known that at Argos Secondary College, 250 of the students study French (F),
Greek (G) or Japanese (J). Forty-one students do not study French. Twelve students
study French and Japanese, but not Greek. Thirteen students study Japanese and Greek,
but not French. Thirteen students study only Greek. Twice as many students study
French and Greek but not Japanese as study all three. The number studying only
Japanese is the same as the total of those studying both French and Greek.
i How many students study all three languages?
ii How many students study only French?
10 Consider the universal set as the set of all students enrolled at Sounion Secondary College.
Let B denote the set of students taller than 180 cm at Sounion Secondary College and A
denote the set of female students.
a Give a brief description of each of the following sets.
i B

ii A B iii A

b Use a Venn diagram to show (A B)

= (A

).
c Hence show that A B C = (A

),

where C is the set of students who play


sport.
P1: FXS/ABE P2: FXS
9780521740494c03.xml CUAU033-EVANS August 21, 2009 15:27
R
e
v
i
e
w
88 Essential Advanced General Mathematics
11 In a certain city three Sunday newspapers (A, B and C) are available. In a sample of
500 people from this city, it was found that
r
nobody regularly reads both A and C
r
a total of 100 people regularly read A
r
205 people regularly read only B
r
of those who regularly read C, exactly half of them also regularly read B
r
35 people regularly read A and B, but not C
r
35 people dont read any of the papers at all.
a Draw a set diagram showing the number of regular readers for each possible
combination of A, B and C.
b How many people in the sample were regular readers of C?
c How many people in the sample regularly read A only?
d How many people are regular readers of A, B and C?
P1: FXS/ABE P2: FXS
9780521740494c04.xml CUAU033-EVANS August 22, 2009 7:21
C H A P T E R
4
Variation
Objectives
To recognise relationships involving direct variation
To evaluate the constant of variation in cases involving direct variation
To solve problems involving direct variation
To recognise relationships involving inverse variation
To evaluate the constant of variation in cases involving inverse variation
To solve problems involving inverse variation
To establish the relationship that exists between variables from given data
To recognise relationships involving joint variation
To solve problems involving joint variation
To solve problems involving part variation
4.1 Direct variation
Emily sets out to drive from her home in Appleton to visit her friend Kim who lives 600 km
away in Brownsville. She drives at a constant speed and notes how far she has travelled every
hour. The distance and times are represented in the table below.
Time (t hours) 1 2 3 4 5 6
Distance (d km) 100 200 300 400 500 600
It can be seen that as t increases, d also increases. The rule relating time to distance is
d = 100t . This is an example of direct variation and 100 is the constant of variation. In this
case d varies directly as t or the distance travelled is proportional to the time spent travelling.
The graph of d against t is a straight line passing through the origin.
A metal ball is dropped from the top of a tall building and the distance it has fallen is
recorded each second.
Time (t s) 0 1 2 3 4 5
Distance (d km) 0 4.91 19.64 44.19 78.56 122.75
89
P1: FXS/ABE P2: FXS
9780521740494c04.xml CUAU033-EVANS August 22, 2009 7:21
90 Essential Advanced General Mathematics
It can be seen that as t increases, d also increases. This time the rule relating time and distance
is d = 4.91t
2
. This is another example of direct variation. In this case, d varies directly as
the square of t or the distance travelled is proportional to t
2
. The graph of d against t
2
is a
straight line passing through the origin.
The symbol used for varies as or is proportional to is . For example, d varies as t can be
written as d t , and d varies as t
2
can be written as d t
2
.
In the following, a proportional to a positive power of b is considered,
i.e. a varies directly as b
n
, n R
+
If a b
n
then a = kb
n
where k is a constant of variation.
For all examples of direct variation (where k is positive), as one variable increases the other
will also increase. The graph of a against b will show an upwards trend. It should be noted that
not all increasing trends will be examples of direct variation.
If a b
n
then the graph of a against b
n
is a straight line passing through the origin.
Example 1
Use the tables of values below to determine the constant of variation, k, in each case and hence
complete each of the tables.
a y x
2
x 2 4 6
y 12 108 192
b y

x

i.e. y x
1
2

x 2 4 6
y 1 1.225 1.414
Solution
a If y x
2
then y = kx
2
When x = 2, y = 12
12 = k(2
2
)
k = 3
Check:
When x = 6, y = 3(6
2
)
= 108
y = 3x
2
In order to complete the
table, consider the following.
When x = 4, y = 3(4
2
)
y = 48
When y = 192, 192 = 3x
2
64 = x
2
x = 8
b If y

x
then y = k

x
When x = 4, y = 1
1 = k(

4)
k = 0.5
Check:
When x = 6, y = 0.5(

6)
1.225
y = 0.5

x
In order to complete the table,
consider the following.
When x = 2, y = 0.5(

2)
y 0.7071
When y = 1.414, 1.414 0.5(

x)
2.828

x
x 8
P1: FXS/ABE P2: FXS
9780521740494c04.xml CUAU033-EVANS August 22, 2009 7:21
Chapter 4 Variation 91
x 2 4 6 8
y 12 48 108 192
x 2 4 6 8
y 0.707 1 1.225 1.414
Example 2
In an electrical wire, the resistance (R ohms) varies directly with the length (L m) of the wire.
a If a wire 6 m long has a resistance of 5 ohms, what would be the resistance in a wire of
length 4.5 m?
b How long is a wire for which the resistance is 3.8 ohms?
Solution
The constant of variation is determined rst.
R L
R = kL
When L = 6, R = 5
5 = k(6)
k =
5
6
i.e. the constant of variation is
5
6
Hence R =
5L
6
a When L = 4.5, R =
5 4.5
6
R = 3.75
The resistance of a wire of length
4.5 m is 3.75 ohms.
b When R = 3.8, 3.8 =
5L
6
L = 4.56
The length of a wire of resistance
3.8 ohms is 4.56 m.
Example 3
The volume of a sphere varies directly as the cube of its radius. By what percentage will the
volume increase if the radius is
a doubled b increased by 20%?
Solution
V r
3
i.e. V = kr
3
Initially set the radius equal to 1,
then V = k(1
3
) = k
a If r is doubled, then set r = 2
Then V = k(2
3
) = 8k
P1: FXS/ABE P2: FXS
9780521740494c04.xml CUAU033-EVANS August 22, 2009 7:21
92 Essential Advanced General Mathematics
the volume has increased from k to 8k, an increase of 7k
% increase of volume =
7k
k

100
1
= 700%
b If r is increased by 20%, then set r = 1.2.
Then V = k(1.2
3
) = 1.728k
% increase of volume = 72.8%
Exercise 4A
1 Determine the value of k, the constant of variation, in each of the following and hence
Example 1
complete the table of values.
a y x
2
x 2 4 6
y 8 32 128
b y x
x
1
2
1
3
2
y
1
6
1
2
2
3
c y

x
x 4 9 49
y 6 9 90
d y x
1
5
x
1
32
1 32
y
1
5
2
5
8
5
2 If V r
3
and V = 125 when r = 2.5, nd
a V when r = 3.2 b r when V = 200
3 If a b
2
3
and a =
2
3
when b = 1, nd
a a when b = 2 b b when a = 2
4 The area (A) of a triangle of xed base length varies directly as its perpendicular height
Example 2
(h). If the area of the triangle is 60 cm
2
when its height is 10 cm, nd
a the area when its height is 12 cm b the height when its area is 120 cm
2
.
5 The extension in a spring (E) varies directly with the weight (w) suspended from it. If a
weight of 452 g produces an extension of 3.2 cm, nd
a the extension produced by a weight of 810 g
b the weight that would produce an extension of 10 cm.
6 The weight (W) of a square sheet of lead varies directly with the square of its side length
(L). If a sheet of side length 20 cm weighs 18 kg, nd the weight of a sheet that has an
area of 225 cm
2
.
P1: FXS/ABE P2: FXS
9780521740494c04.xml CUAU033-EVANS August 22, 2009 7:21
Chapter 4 Variation 93
7 The volume (V) of a sphere varies directly with the cube of its radius (r). A sphere whose
radius is 10 cm has a volume of 4188.8 cm
3
. Find the radius of a sphere whose volume is
1 cubic metre.
8 The time taken for one complete oscillation of a pendulum is called its period. The period
Example 3
(T) of a pendulum varies directly with the square root of the length (L) of the pendulum. A
pendulum of length 60 cm has a period of 1.55 seconds. Find the period of a pendulum
that is one and a half times as long.
9 The distance (d) to the visible horizon varies directly with the square root of the height (h)
of the observer above sea level. An observer 1.8 m tall can see 4.8 km out to sea when
standing on the shoreline.
a How far could the person see if they climbed a 4 m tower?
b If the top of a 10 m mast on a yacht is just visible to the observer in the tower, how far
out to sea is the yacht?
10 In each of the following calculate the percentage change in y when x is
a doubled b halved c reduced by 20%
d increased by 40%
i y x
2
ii y

x iii y x
3
4.2 Inverse variation
A builder employs a number of bricklayers to build a brick wall. Three bricklayers will
complete the wall in eight hours but if he employs six bricklayers the wall will be complete in
half the time. The more bricklayers he employs, the shorter the time taken to complete the
wall. The time taken (t) decreases as the number of bricklayers (b) increases.
This is an example of inverse variation. The time taken to complete the wall varies
inversely as the number of bricklayers employed.
t varies inversely as b or t is inversely proportional to b
i.e. t
1
b
In general, inverse variation exists if a
1
b
n
where n is some positive number
i.e. a varies inversely as b
n
.
If a
1
b
n
then a =
k
b
n
where k is a positive constant called the constant of variation.
For all examples of inverse variation, as one variable increases the other will decrease and
vice versa. The graph of a against b will show a downward trend. It should be noted, however,
that any graph showing a decreasing trend will not necessarily be an example of inverse
variation.
If a
1
b
n
then the graph of a against
1
b
n
will be a straight line.
However, since if b = 0,
1
b
n
is undened, the line will not be dened at the origin.
P1: FXS/ABE P2: FXS
9780521740494c04.xml CUAU033-EVANS August 22, 2009 7:21
94 Essential Advanced General Mathematics
Example 4
Use the tables of values below to determine the value of the constant of variation, k, in each
case and hence complete each of the tables.
a y
1
x
2
x 2 5 10
y 0.1 0.016 0.001
b y
1

x
x 1 25 100
y 10 5 1
Solution
a y
1
x
2
y =
k
x
2
When x = 2, y = 0.1
0.1 =
k
2
2
k = 0.4
i.e. the constant of variation is 0.4
Check:
When x = 5, y =
0.4
5
2
= 0.16
y =
0.4
x
2
In order to complete the table,
consider the following.
When x = 10, y =
0.4
10
2
y = 0.004
When y = 0.001, 0.001 =
0.4
x
2
0.001x
2
= 0.4
x
2
=
0.4
0.001
x = 20
x 2 5 10 20
y 0.1 0.016 0.004 0.001
b y
1

x
y =
k

x
When x = 1, y = 10
10 =
k

1
k = 10
Check:
When x = 100, y =
10

100
= 1
y =
10

x
In order to complete the table,
consider the following.
When x = 4, y =
10

4
y = 5
When y = 2, 2 =
10

x
2

x = 10
x = 25
x 1 4 25 100
y 10 5 2 1
Example 5
For a cylinder of xed volume, the height (h cm) is inversely proportional to the square of the
radius (r cm).
P1: FXS/ABE P2: FXS
9780521740494c04.xml CUAU033-EVANS August 22, 2009 7:21
Chapter 4 Variation 95
a What percentage change in the height would result if its radius were reduced by 25%?
b If a cylinder 15 cm high has a base radius of 4.2 cm, how high would a cylinder of
equivalent volume be if its radius were 3.5 cm?
Solution
a h
1
r
2
i.e. h =
k
r
2
If r = 1, then h =
k
(1)
2
= k
If r is reduced by 25%, then
set r = 0.75
Then h =
k
(0.75)
2
=
k
0.5625
1.778k (correct to
three decimal places)
h is increased by 77.8%
b h =
k
r
2
When h = 15, r = 4.2
15 =
k
(4.2)
2
k = 15(4.2)
2
= 264.6
h =
264.6
r
2
Consider a cylinder of radius 3.5 cm.
If r = 3.5, then h =
264.6
(3.5)
2
h = 21.6
The height of the cylinder is 21.6 cm.
Exercise 4B
1 Determine the value of k, the constant of variation, in each of the following and hence
Example 4
complete the tables of values.
a y
1
x
x 2 4 6
y 1
1
2
1
16
b y
1

x
x
1
4
1 9
y 1
1
2
1
4
c y
1
x
2
x 1 2 3
y 3
3
4
1
12
d y
1
x
1
3
x
1
8
1 125
y
2
3
1
3
1
9
2 If a
1
b
3
and a = 4 when b =

2, nd
a a when b = 2

2 b b when a =
1
16
P1: FXS/ABE P2: FXS
9780521740494c04.xml CUAU033-EVANS August 22, 2009 7:21
96 Essential Advanced General Mathematics
3 If a
1
b
4
and a = 5 when b = 2, nd
a a when b = 4 b b when a = 20.
4 The gas in a cylindrical canister occupies a volume of 22.5 cm
3
and exerts a pressure
of 1.9 kg/cm
2
. If the volume (V) varies inversely as the pressure (P), nd the pressure if the
volume is reduced to 15 cm
3
.
5 The current (I amperes) that ows in an electrical appliance varies inversely as the
Example 5
resistance (R ohms) of the appliance. If the current is 3 amperes when the resistance is
80 ohms, nd
a the current when the resistance is 100 ohms
b the increase in resistance required to reduce the current to 80% of its original value.
6 The intensity of illumination (I) of a light varies inversely as the square of the distance (d)
from the light. At a distance of 20 m a light has an intensity of 100 candela. Find the
intensity of the light at a distance of 25 m.
7 The radius (r) of a cylinder of xed volume varies inversely as the square root of its height
(h). If the height is 10 cm when the radius is 5.64 cm, nd the radius if the height is 12 cm.
8 In each of the following, calculate the percentage change in y when x is
a doubled b halved c reduced by 20%
d increased by 40%.
i y
1
x
2
ii y
1

x
iii y
1
x
3
4.3 Fitting data
Sometimes the relationship that exists between two variables a and b is not known. By
inspection of a table of values, it is sometimes possible to ascertain whether the relationship
between the variables is direct or inverse proportion. Analysis is required to establish the rule
that best ts the given data. This may involve graphing the data.
Example 6
Establish the relationship between the two variables for each of the following tables of values.
a
b 0 2 4 6 8
a 0 12 48 108 192
b
x 1 3 6 12 15
y 30 10 5 2.5 2
Solution
a By inspection it can be conjectured that some type of direct variation exists. As b
increases, a also increases and when a = 0, b = 0.
P1: FXS/ABE P2: FXS
9780521740494c04.xml CUAU033-EVANS August 22, 2009 7:21
Chapter 4 Variation 97
Assume a b
n
for some positive number n
a = kb
n
i.e. k =
a
b
n
Select a value for n (it must be a positive number) and test each of the pairs of
values given in the table (do not use (0, 0)). If the value of k for each pair of values
is the same then the choice of n is correct.
Let n = 1 k =
a
b
Consider
a
b
for the values
given in the table.
Testing:
12
2
= 6
48
4
= 12
108
6
= 18
192
8
= 24
Since the quotients differ, n = 1.
Let n = 2 k =
a
b
2
Consider
a
b
2
for the values
given in the table.
Testing:
12
4
= 3
48
16
= 3
108
36
= 3
192
64
= 3
The quotients are all equal to 3.
k = 3 and n = 2
i.e. a = 3b
2
b By inspection it can be conjectured that some type of inverse variation exists.
As x increases, y decreases.
Assume y
1
x
n
for some positive number n
y =
k
x
n
i.e. k = yx
n
Let n = 1 k = yx
Consider the product yx for the values given in the table.
Testing: 30 1 = 30
10 3 = 30
5 6 = 30
2.5 12 = 30
2 15 = 30
k = 30 and n = 1
i.e. y =
30
x
The type of variation can also be investigated by graphical analysis. By plotting the graph of
a against b, an upward trend may indicate direct variation or a downward trend may indicate
inverse variation.
P1: FXS/ABE P2: FXS
9780521740494c04.xml CUAU033-EVANS August 22, 2009 7:21
98 Essential Advanced General Mathematics
To nd the specic type of variation that exists, the following can be used as a guide.
If direct variation exists (a b
n
), then the graph of a against b
n
will be a straight line
through the origin. The gradient of this line will be the constant of variation k.
If inverse variation exists

a
1
b
n

, then the graph of a against


1
b
n
will be a straight line.
This line will not be dened at the origin. The gradient of this line will be the constant of
variation k.
Example 7
For the table of values below, plot the graph of a against b
2
and hence establish the rule
relating a to b.
b 1 2 3 4 5
a 0.5 2 4.5 8 12.5
Solution
b
2
1 4 9 16 25
a 0.5 2 4.5 8 12.5
10
5
0
10 15 20 25
b
2
5
a
Since this is a straight line, it can be
conjectured that the relationship is
a = kb
2
where k corresponds to the
gradient of the graph.
From the graph it can be seen that a =
1
2
b
2
.
If it is known that the relationship between two variables x and y is of the form y = kx
n
where k R
+
and n Q\{0} then a CAS calculator can be used to nd n and k is
sufcient information is given.
Example 8
The following data was collected recording N, the number of calls to a company, D days
after the commencement of an advertising campaign.
Days (D) 5 10 15
Number of calls (N) 50 400 1350
Find a relationship between N and D using the graphics calculator.
P1: FXS/ABE P2: FXS
9780521740494c04.xml CUAU033-EVANS August 22, 2009 7:21
Chapter 4 Variation 99
Solution
Using the TI-Nspire
Store the x-values and y-values as
shown.
Select Power Regression from Stat
Calculations submenu of the Statistics
menu (b61 ) and complete
as shown. Press enter, and the result is
given as y = a x

b, a = 0.4, b = 3.
Hence y = 0.4x
3
, so the required
relationship is N = 0.4D
3
.
The graph and the data can be graphed
in a Graphs & Geometry application
( 2) as a Function (b31)
and a Scatter Plot (b34)
respectively.
Using the Casio ClassPad
In the program area, enter the data
into list 1 and list 2 then tap Calc, Power
Reg and ensure the settings are as shown.
Note that selecting y1 will copy the
formula to graph y1 in the program
area. Note the formula from the Stat
Calculation screen before tapping OK. The
required relationship is N = 0.4D
3
.
P1: FXS/ABE P2: FXS
9780521740494c04.xml CUAU033-EVANS August 22, 2009 7:21
100 Essential Advanced General Mathematics
Example 9
Establish a rule connecting y and x given the following data.
x 1 8 64
y 5 2.5 1.25
Solution
The solution is given in the screens below.
Using the TI-Nspire
Note thaty = 5x

1
3
=
5
x
1
3
Using the Casio ClassPad
The solution is given in the screens. Note that y = 5x

1
3
=
5
x
1
3
Exercise 4C
1 Each of the tables in parts a to e ts one of the following types of variation:
Example 6
direct y x inverse y
1
x
direct square y x
2
inverse square y
1
x
2
direct square root y

x
P1: FXS/ABE P2: FXS
9780521740494c04.xml CUAU033-EVANS August 22, 2009 7:21
Chapter 4 Variation 101
Establish the relationship between x and y in each case.
a
x 0 3 6 9 12
y 0 2 4 6 8
b
x 1 2 3 4 5
y 4 16 36 64 100
c
x 20 15 10 5 1
y
1
4
1
3
1
2
1 5
d
x 1 2 3 4 5
y 2 2.828 3.464 4 4.472
e
x 1 1.5 2 2.5 3
y 4 1.78 1 0.64 0.444
2 Which of the following graphs could represent examples of direct variation?
a
x
0
y
b
x
3
0
y c
x
0
y
d
x
0
y
e
0
y
x
1
f
x
0
y
3 Which of the following graphs could represent examples of inverse variation?
a
x
0
y
b
x
1
0
y c
x
0
y
d
x
0
y e
1
x
3
0
y
f
x
0
y
P1: FXS/ABE P2: FXS
9780521740494c04.xml CUAU033-EVANS August 22, 2009 7:21
102 Essential Advanced General Mathematics
4 Give the rule connecting y and x for each of the following.
a
(1, 3)
x
0
y
b
x
1
0
y
(2, 6)
c
x
2
0
y
(3, 10)
d
0
y
x
(1, 2)
e
(9, 3)
x
1
0
y
f
(1, 6)
x
3
0
y
5 Plot the graph of y against x
2
and hence establish the relationship between x and y.
Example 7
x 2 2.5 3 3.5 4
y 9.6 15 21.6 29.4 38.4
6 Plot the graph of y against

x and hence establish the relationship between x and y.


x 1 4 9 16 25
y 1.5 3 4.5 6 7.5
7 Plot the graph of y against
1
x
2
and hence establish the relationship between x and y.
x 0.2 0.3 0.4 0.5 1
y 50 22.2 12.5 8 2
8 Given that for each of the following y ax
b
use your graphics calculators PwrReg
Example 8
function to establish the values of a and b.
a x 4.00 8.00 12.00 16.00
y 0.50 0.71 0.87 1.00
b x 1 5 10 15
y 2.00 14.95 35.57 59.04
c x 1 10 100 1000
y 3.50 8.79 22.08 55.47
d x 10 20 30 40
y 46.42 73.68 96.55 116.96
e x 1 2 3 4
y 2.00 0.35 0.13 0.06
f x 1 3 5 7
y 3.20 2.06 1.68 1.47
P1: FXS/ABE P2: FXS
9780521740494c04.xml CUAU033-EVANS August 22, 2009 7:21
Chapter 4 Variation 103
9 The concentration of antibodies (C) in an animals bloodstream is directly proportional to
Example 9
time (t hours) after which the animal is injected with an antigen (i.e., C = at
b
). The
following data is collected.
t 1 2 3 4
C 100 114.87 124.57 131.95
a Find values for a and b. b Find the concentration after 10 hours.
10 The level of infestation (I) of a pest in a crop is proportional to the time (t days) after
which the crop is sprayed with an insecticide. The relationship can be modelled using the
rule I = at
b
, t 1.
The following data is collected.
t 1 2 3 4
I 1500 1061 866 750
a Find values for a and b. b Find the level of infestation after 10 days.
4.4 Joint variation
There are many situations where one variable depends on more than one other variable. The
variable is said to vary jointly as the other variables. For example, the volume of a cylinder
varies jointly as the square of the radius and the height.
i.e. V r
2
h
or V = kr
2
h (the value of k is known to be )
Example 10
Given that y
x
2
z
, use this table of values to determine
the value of the constant of variation k and hence
complete the table.
x 2 4 10
z 10 8 50
y 2 2.5 4
P1: FXS/ABE P2: FXS
9780521740494c04.xml CUAU033-EVANS August 22, 2009 7:21
104 Essential Advanced General Mathematics
Solution
y
x
2
z
y =
kx
2
z
When x = 2 and z = 10, y = 2
2 =
k(2
2
)
10
k = 5
i.e. y =
5x
2
z
When x = 4, z = 8
y =
5(4
2
)
8
y = 10
When z = 50, y = 2.5
2.5 =
5(x
2
)
50
25 = x
2
x = 5
When x = 10, y = 4
4 =
5(10
2
)
z
4z = 500
z = 125
x 2 4 5 10
z 10 8 50 125
y 2 10 2.5 4
Example 11
The speed (s) of a conveyor belt varies jointly as the diameter (d) of the cog around which it
passes and the number of revolutions per second (n) the cog makes. The speed of a belt that
passes round a cog of diameter 0.3 m, revolving 20 times per second, is 18.85 m/s. Find the
value of
a the constant of variation
b the speed of a belt passing around a cog half as big revolving 30 times per second.
P1: FXS/ABE P2: FXS
9780521740494c04.xml CUAU033-EVANS August 22, 2009 7:21
Chapter 4 Variation 105
Solution
a s dn
i.e. s = kdn
When n = 20 and d = 0.3, s = 18.85
18.85 = k(0.3)(20)
k = 3.142 (correct to three decimal places)
s = 3.142dn
b When d = 0.15 and n = 30
s = 3.142 (0.15) (30)
s = 14.14 m/s (correct to two decimal places)
Exercise 4D
1 Given that y
x
z
, use this table of values to
Example 10
determine the value of the constant of variation
k and hence complete the table.
x 2 4 10
z 10 2 60
y 1 10 0.5 4
2 Given that y xz, use this table of values to
determine the value of the constant of variation
k and hence complete the table.
x 2 4 10
z 10 8 50
y 10 25 15
3 Given that y
z
x
2
, use this table of values to
determine the value of the constant of variation
k and hence complete the table.
x 2 3 10
z 10 4 50
y
15
2
4
3
6 4
4 a varies directly as b
2
and inversely as c. If a = 0.54 when b = 1.2 and c = 2, nd a
when b = 2.6 and c = 3.5.
5 z varies as the square root of x and inversely as the cube of y. If z = 1.46 when x = 5 and
y = 1.5, nd z when x = 4.8 and y = 2.3.
6 The simple interest (I) earned on an investment varies jointly as the interest rate (r) and
Example 11
the time (t) for which it is invested. If a sum of money invested at 6.5% per annum for two
years earns $130, how much interest would the same amount of money earn if it were
invested at 5.8% for three years?
7 The kinetic energy (E) of an object varies directly as its mass (m) and the square of its
velocity (v). If the kinetic energy of an object with a mass of 2.5 kg moving at 15 m/s is
281.25 joules, nd the energy of an object with a mass of 1.8 kg moving at 20 m/s.
P1: FXS/ABE P2: FXS
9780521740494c04.xml CUAU033-EVANS August 22, 2009 7:21
106 Essential Advanced General Mathematics
8 The resistance (R) in an electrical wire varies directly as its length (l) and inversely as the
square of its diameter (d). Find the percentage change in R if
a l is increased by 50% and d is reduced by 50%
b l is decreased by 50% and d is increased by 50%.
9 The weight (W) that can be supported by a wooden beam varies directly as the square of
its diameter (d) and inversely as its length (l).
a What percentage increase in the diameter would be necessary for a beam twice as long
to support an equivalent weight?
b What percentage change in the weight would be capable of being supported by a beam
three times as long with twice the diameter?
10 If p varies as the square of q and inversely as the square root of r, what is the effect on p if
a both q and r are doubled b q is doubled and r is halved?
11 a The tension in a spring (T) varies directly with the extension (x) and inversely with the
natural length (l) of the spring. Compare the tension in a spring with a natural length
of 3 m that is extended by 1 m with the tension in a second spring with a natural length
of 2.7 m that is extended by 0.9 m.
b The work done (W) in stretching a spring varies directly with the square of the
extension (x) and inversely with the natural length of the spring (l). Compare the work
done on the two springs in part a.
4.5 Part variation
The total cost ($C) of printing cards is made up of a xed overhead charge ($b) plus an amount
that varies directly as the number printed (n).
i.e. C = b +kn
The total surface area (A) of a closed cylinder of xed height is made up of two parts. The
area of the curved surface (2rh), which varies as the radius, and the area of the two ends
(2r
2
), which varies as the square of the radius.
i.e. A = k
1
r +k
2
r
2
where k
1
= 2h and k
2
= 2 are the constants of variation. These are
examples of part variation.
Part variation exists when the value of one variable is the sum of two or more quantities
each of which varies independently in some way. In some cases, as in the rst example above,
one of those quantities may be constant.
Example 12
A monthly telephone account (A) is made up of a xed charge (c) for rental and servicing plus
an amount that is proportional to the number of calls made (n). In January, 220 calls were
made and the account was for $98.20. In February, 310 calls were made and the account was
for $120.70. Find the xed charge and the cost per call.
P1: FXS/ABE P2: FXS
9780521740494c04.xml CUAU033-EVANS August 22, 2009 7:21
Chapter 4 Variation 107
Solution
A = c +kn, where c equals the xed charge and k equals cost per call
98.20 = c +220k . . . 1
120.70 = c +310k . . . 2
Solving simultaneously, subtract 1 from 2
22.5 = 90k
k = 0.25
Substitute in 1
98.20 = c +220 (0.25)
= c +55
c = 43.2
The xed charge is $43.20 and the cost per call is $0.25, i.e. 25 cents.
Example 13
The stopping distance of a tram (d) (i.e. the distance travelled by the tram after its brakes are
applied) varies partly with the speed of the tram (s) and partly with the square of its speed. A
tram travelling at 15 km/h can stop in 57 m and at 20 km/h in 96 m. Find the formula that
relates s to d and hence the stopping distance of a tram travelling at 18 km/h.
Solution
d = k
1
s +k
2
s
2
57 = 15k
1
+225k
2
. . . 1
96 = 20k
1
+400k
2
. . . 2
Multiply 1 by 4 and 2 by 3
228 = 60k
1
+900k
2
. . . 3
288 = 60k
1
+1200k
2
. . . 4
Subtract 3 from 4
60 = 300k
2
k
2
=
1
5
Substitute in 1
57 = 15k
1
+225

1
5

57 = 15k
1
+45
k
1
=
12
15
P1: FXS/ABE P2: FXS
9780521740494c04.xml CUAU033-EVANS August 22, 2009 7:21
108 Essential Advanced General Mathematics
k
1
=
4
5
d =
4
5
s +
1
5
s
2
When s = 18
d =
4
5
(18) +
1
5
(18)
2
d = 79.2
The stopping distance of the tram will be 79.2 m.
Exercise 4E
1 The cost of a taxi ride (C) is partly constant (b) and partly varies with the distance travelled
Example 12
(d). A ride of 22 km costs $42.40 and a ride of 25 km costs $47.80. Find the cost of a
journey of 17 km.
2 The cost of a wedding reception at Hillview Reception Centre includes a xed overhead
charge and an amount per guest.
a If a reception for 50 people costs $2625 and a reception for 70 people costs $3575, nd
the xed overhead charge and the cost per guest.
b Hence nd the total cost of a reception for 100 guests.
3 p is the sum of two numbers, one of which varies as x and the other, as the square of y. If
Example 13
p = 14 when x = 3 and y = 4, and p = 14.5 when x = 5 and y = 3, nd p when x = 4
and y = 5.
4 The cost of running a ferris wheel in an amusement park varies partly as the number of
people who ride it and partly as the inverse of the number of people who ride it. If the
running cost is $32 when 200 people ride it and $61 if 400 people ride it, nd the running
cost on a day when 360 people ride it.
5 The distance travelled (s) by a particle varies partly with time and partly with the square of
time. If it travels 142.5 m in 3 s and 262.5 m in 5 s, nd
a how far it would travel in 6 s b how far it would travel in the sixth second.
6 The time taken (t) to load boxes onto a truck varies partly with the number of
boxes (b) and partly with the inverse of the number of men (m) loading the
boxes. If it takes one man 45 minutes to load ten boxes and two
men 30 minutes to load eight boxes, how long would it take four men to
load sixteen boxes?
P1: FXS/ABE P2: FXS
9780521740494c04.xml CUAU033-EVANS August 22, 2009 7:21
R
e
v
i
e
w
Chapter 4 Variation 109
Chapter summary
Direct variation
a b
n
, i.e. a varies directly as b
n
(n R
+
)
This implies a = kb
n
where k is the constant of variation (k R
+
).
As b increases, a will also increase.
If a b
n
, the graph of a against b
n
is a straight line through the origin.
Inverse variation
a
1
b
n
, i.e. a varies inversely as b
n
(n R
+
)
This implies a =
k
b
n
where k is the constant of variation (k R
+
).
As b increases, a will decrease.
If a
1
b
n
, the graph of a against
1
b
n
is a straight line but is undened at the origin.
Joint variation
One quantity varies with more than one other variable. This may be a combination of direct
and/or inverse variation.
e.g. V r
2
h implies V = kr
2
h
a
c

b
implies a =
kc

b
Part variation
The value of one variable is the sum of two or more quantities each of which is determined
by a variation. In some cases, one of those quantities may be constant.
e.g. A = k
1
r +k
2
r
2
where k
1
, k
2
are constants of variation.
Multiple-choice questions
1 For the values in the table shown, it is known that
y x
2
. The value of k, the constant of variation,
is equal to
x 2 3 6
y
4
3
3 12
A 3 B 9 C
1
3
D 2 E
4
3
2 For the values in the table shown, it is known that
y
1
x
. The value of k, the constant of variation,
is equal to
x 2 4 8
y
1
4
1
8
1
16
A
1
2
B 1 C 4 D 2 E
1
4
P1: FXS/ABE P2: FXS
9780521740494c04.xml CUAU033-EVANS August 22, 2009 7:21
R
e
v
i
e
w
110 Essential Advanced General Mathematics
3 a b
3
and a = 32 when b = 2. Find a when b = 4.
A 64 B 256 C 4 D 16 E 128
4 p
1
q
2
and p =
1
3
when q = 3. Find q when p = 1
A 3 B 3 C

3 D 1 E
1
3
5 The rule connecting y and x as shown in the graph is
A y = 3x B y = 3x
2
C y = 3

x
D y =
1
3
x
E y = x
2
+2
x
2
y
(2, 6)
6 The rule connecting y and x as shown in the graph is
A y =
1
4
x B y = 4x C y =

x
D y = 4

x E y = x
x
y
(1, 4)
7 For the values in the table shown, it is known that
y
x
z
2
. The value of k, the constant of variation,
is equal to
x 2 4 8
z 2 2 2
y
1
3
2
3
4
3
A 2 B
4
3
C
1
3
D 3 E
2
3
8 a varies directly as the square of p and inversely as q and a = 8 when p = 2 and q = 5. If
p = 3 and q = 6 then a =
A
1
2
B 12 C 120 D 15 E 5
9 If p q
2
and q is increased by 10%, p would be
A Increased by 10% B Increased by 20% C Increased by 100%
D Increased by 21% E Remain the same
10 If p
1
q
and q is decreased by 20%, p would be
A Decreased by 25% B Increased by 25% C Decreased by 20%
D Increased by 20% E Unchanged
P1: FXS/ABE P2: FXS
9780521740494c04.xml CUAU033-EVANS August 22, 2009 7:21
R
e
v
i
e
w
Chapter 4 Variation 111
Short-answer questions (technology-free)
1 a If a b
2
and a =
3
2
when b = 2, nd a when b = 4 and b when a = 8.
b If y x
1
3
and y = 10 when x = 2, nd y when x = 27 and x when y =
1
8
.
c If y
1
x
2
and y =
1
3
when x = 2, nd y when x =
1
2
and x when y =
4
27
.
d a varies directly as b and inversely as

c. If a =
1
4
when b = 1 and c = 4, nd a when
b =
4
9
and c =
16
9
.
2 The distance, d metres, which a body falls varies directly as the square of the time,
t seconds, for which it has been falling. If a body falls 78.56 m in 4 s, nd
a the formula connecting d and t b the distance fallen in 10 s
c the time taken to fall 19.64 m.
3 The velocity of a falling body (v metres per second) varies directly as the square root of the
distance (s metres) through which it has fallen. A body has a velocity of 7 metres per
second after falling 2.5 m.
a Find its velocity after falling 10 m.
b Find the distance through which it falls to attain a velocity of 28 metres per second.
c What variables would be plotted on the axis to obtain a straight line graph?
4 The time taken for a journey is inversely proportional to the average speed of travel. If it
takes 4 hours travelling at 30 km/h, how long will it take travelling at 50 km/h?
5 If y varies inversely as x, what is the effect on
a y if x is doubled b x if y is doubled c y if x is halved d x if y is halved?
6 The cost of running an electric appliance varies jointly as the time it is run, the electrical
resistance and the square of the current. It costs 9 cents to use an appliance of resistance
60 ohms, which draws 4 amps of current for 2.5 hours. How much will it cost to use an
appliance of resistance 80 ohms, which draws 3 amps of current for 1.5 hours?
7 The cost of printing is made up of two parts: a xed charge and a charge proportional to
the number of copies. If the cost of printing 100 copies is $20 and the cost of printing
500 copies is $30, what would be the cost of printing 700 copies?
8 For a constant resistance, the voltage (v volts) of an electric circuit varies directly as the
current (I amps). If the voltage is 24 volts when the current is 6 amps, nd the current when
the voltage is 72 volts.
9 The intensity of sound varies inversely as the square of the distance of the observer from the
source. If the observer moves to twice the distance from the source, compare the second
intensity I
2
with the rst intensity I
1
.
10 If y varies directly as x
2
and inversely as z, nd the percentage change in y when x is
increased by 10% and z is decreased by 10%.
P1: FXS/ABE P2: FXS
9780521740494c04.xml CUAU033-EVANS August 22, 2009 7:21
R
e
v
i
e
w
112 Essential Advanced General Mathematics
Extended-response questions
1 A certain type of hollow sphere is designed in such a way that the mass varies as the square
of the diameter. Three spheres of this type are made. One has mass 0.10 kg and diameter
9 cm, the second has diameter 14 cm and the third has mass 0.15 kg. Find
a the mass of the second sphere b the diameter of the third sphere.
2 The height (h m) to which a centrifugal pump raises water is proportional to the square of
its speed of rotation (n revs/min). If the pump raises water to a height of 13.5 m when it is
rotating at 200 revs/min, nd
a the formula connecting h and n
b the height that the water can be raised to when it is rotating at 225 revs/min
c the speed required to raise the water to a height of 16 m.
3 The maximum speed of yachts of normal dimensions varies as the square root of their
length. If a yacht 20 m long can maintain a maximum speed of 15 knots, nd the maximum
speed of a yacht 15 m long.
4 a The air in a tube occupies 43.5 cm
3
and the pressure is 2.8 kg/cm
2
. If the volume
(Vcm
3
) varies inversely as the pressure (P), nd the formula connecting V and P.
b Calculate the pressure when the volume is decreased to 12.7 cm
3
.
5 The weight (w kg) which a beam supported at each end will carry without breaking, varies
inversely as the distance (d m) between supports. A beam which measures 6 m between
supports will just carry a load of 500 kg.
a Find the formula connecting w and d.
b What weight would a similar beam carry if the distance between the supports
were 5 m?
c What weight would a similar beam carry if the distance between the supports
were 9 m?
6 The relationship between pressure and volume of a xed mass of gas when the temperature
is constant is shown by the following table.
Pressure ( p) 12 16 18
Volume (v) 12 9 8
a What is a possible equation relating p and v?
b Using this equation, nd
i the volume when the pressure is 72 units
ii the pressure when the volume is 3 units.
c Sketch the graph relating v and
1
p
.
7 The time taken to manufacture particular items of scientic equipment varies partly as
the diameter of the item and partly as the number of parts required in the item. If it takes
P1: FXS/ABE P2: FXS
9780521740494c04.xml CUAU033-EVANS August 22, 2009 7:21
R
e
v
i
e
w
Chapter 4 Variation 113
30 minutes to make a 3 cm diameter item with eight parts and 38 minutes to make a 5 cm
diameter item with ten parts, how long does it take to make a 4 cm diameter item with
twelve parts?
8 The cost of decorative wrought iron is the sum of two parts which vary as the length and
the square of the length respectively. When the length is 2 m, the cost is $18.40 and when
the length is 3 m, the cost is $33.60. Find the cost when the length is 5 m.
9 The sum of the rst n natural numbers is equal to the sum of two quantities, the rst of
which is proportional to n and the second to n
2
. Work out the sums of the rst three and
four natural numbers and hence nd the formula for the sum of the rst n natural numbers.
10 Data about the number of pies sold at football matches and the size of the crowds attending
has been recorded as follows
Attendance (N 1000) 20 30 60
Number of pies sold (P) 15 650 19 170 27 110
a Use a graphics calculator to nd an approximate relationship between N and P of the
form P = aN
b
.
b The crowd predicted for a forthcoming match is 55 000. Assuming the model found in
part a applies, how many pies would the caterers anticipate selling on that day?
c The caterers have only 25 000 pies available for sale. Again assuming the model found
in part a applies, what is the maximum crowd the caterers might be hoping for if they
are able to satisfy all customers wanting pies?
11 The effectiveness of an anaesthetic drug is being tested by varying the dose (d mL) given to
patients and recording both the time (t min) for the patient to lose consciousness and the
time (T min) for the patient to regain consciousness. The following data was recorded:
Dosage (d mL) 10 30 60
Time to lose consciousness (t min) 36 4 1
Time to regain consciousness (T min) 14 126 504
a Establish the relationship between d and t (assume t is proportional to a power of d).
b Establish the relationship between d and T (assume T is proportional to a power of d).
c If it is desirable to have a particular patient unconscious for no longer than 80 minutes,
what is the maximum dosage of the drug that should be given?
d How long would it take that patient to lose consciousness?
e Another patient is given a dose of 20 mL. How long will it take for the patient to lose
consciousness and how long will they remain unconscious?
12 The German astronomer Johannes Kepler collected data on the mean distance from the Sun
to the planets (R 10
6
km) and the period of the orbit (T years). He was able to establish a
relationship between R and T.
P1: FXS/ABE P2: FXS
9780521740494c04.xml CUAU033-EVANS August 22, 2009 7:21
R
e
v
i
e
w
114 Essential Advanced General Mathematics
a Using the data below (approximations only)
i establish the relationship between R and T (assume T is proportional to a power
of R)
ii complete the table of values showing the period of orbit of the remaining planets
Approximate radius of orbit Period of orbit
Planet (R 10
6
km) (T years)
Mercury 58 0.24
Venus 108 0.61
Earth 150 1
Mars 228
Jupiter 779
Saturn 1427
Uranus 2870
Neptune 4497
Pluto 5900
b A comet orbits the sun every 70 years. What is its radius of orbit?
13 To test the effectiveness of an advertising campaign for cheap ights to Hawaii, a travel
agent keeps a record of the number of enquiries she receives. It is estimated that the
number of enquiries, E, is proportional to the number of times, n, that the advertisement is
shown on television. The following data is collected.
Number of advertisements (n) 10 20 30
Number of enquiries (E) 30 40 47
a Assuming that the relationship between the number of enquiries and the number of
advertisements is modelled by the rule E = an
b
, use your graphics calculator to nd
values for a and b.
b Predict the number of enquiries received if the advertisement is shown 100 times.
After two weeks the advertisement has been shown 50 times and the advertising campaign
is stopped. The travel agent continues to get enquiries and continues to record them. It is
now estimated that the number of enquiries, E, is proportional to the number of days, d,
since the advertising campaign stopped. The following data is recorded.
Number of days (d) 3 5 7 10
Number of enquiries (E) 45 25 17 11
c Assuming that the relationship between the number of enquiries and the number of days
is modelled by the rule E = kd
p
, use your graphics calculator to nd values for k and p.
d Predict the number of enquiries received on the 14th day after the advertising campaign
has nished.
P1: FXS/ABE P2: FXS
9780521740494c05.xml CUAU033-EVANS August 24, 2009 9:30
C H A P T E R
5
Sequences and series
Objectives
To explore sequences of numbers and their difference equations
To use a CAS calculator to generate sequences and display graphs
To recognise arithmetic sequences
To find the terms, difference equation and number of terms for an arithmetic sequence
To calculate the sum of the terms of an arithmetic series
To recognise geometric sequences
To find the terms, difference equation and number of terms for a geometric sequence
To calculate the sum of the terms of a geometric series
To recognise and calculate the sum of the terms in an infinite geometric series
To use fixed point iteration to generate convergent sequences and hence solve
equations
To apply sequences and series to solving problems
5.1 Introduction to sequences
The following are examples of sequences of numbers:
A 1, 3, 5, 7, 9, . . . . . . B 0.1, 0.11, 0.111, 0.1111, . . . . . .
C
1
3
,
1
9
,
1
27
,
1
81
, . . . . . . D 10, 7, 4, 1, 2, . . . . . . E 0.6, 1.7, 2.8, 3.9, . . . . . .
Note each sequence is a set of numbers with order being important. For some sequences of
numbers a rule can be found for getting from one number to the next. For example:
for sequence A, a rule is: add 2
for sequence C, a rule is: multiply by
1
3
for sequence D, a rule is: subtract 3
for sequence E, a rule is: add 1.1
115
P1: FXS/ABE P2: FXS
9780521740494c05.xml CUAU033-EVANS August 24, 2009 9:30
116 Essential Advanced General Mathematics
The numbers of a sequence are called terms. The nth term of a sequence is denoted by the
symbol t
n
. So the rst term is t
1
, the 12th term is t
12
and so on.
A sequence may be dened by specifying a rule which enables each subsequent term to be
found using the previous term. In this case, the rule specied is called an iterative rule or a
difference equation. For example:
sequence A may be dened by t
1
= 1, t
n
= t
n1
+2
sequence C may be dened by t
1
=
1
3
, t
n
=
1
3
t
n1
Example 1
Use the difference equation to nd the rst four terms of the sequence
t
1
= 3, t
n
= t
n1
+5
Solution
t
1
= 3
t
2
= t
1
+5 = 8
t
3
= t
2
+5 = 13
t
4
= t
3
+5 = 18
The rst four terms are 3, 8, 13, 18.
Example 2
Find the rst four terms of the sequence dened by the rule t
n
= 2n +3.
Solution
t
1
= 2(1) +3 = 5
t
2
= 2(2) +3 = 7
t
3
= 2(3) +3 = 9
t
4
= 2(4) +3 = 11
The rst four terms are 5, 7, 9, 11.
Example 3
Find the difference equation for the following sequence.
9, 3, 1,
1
3
, . . .
Solution
3 =
1
3
9 i.e. t
2
=
1
3
t
1
1 =
1
3
3 i.e. t
3
=
1
3
t
2
t
n
=
1
3
t
n1
, t
1
= 9
P1: FXS/ABE P2: FXS
9780521740494c05.xml CUAU033-EVANS August 24, 2009 9:30
Chapter 5 Sequences and series 117
Alternatively a sequence may be dened by a rule that is stated in terms of n. For example:
t
n
= 2n denes the sequence t
1
= 2, t
2
= 4, t
3
= 6, t
4
= 8, . . .
t
n
= 2
n1
denes the sequence t
1
= 1, t
2
= 2, t
3
= 4, t
4
= 8, . . .
Example 4
Find the rule for the nth term for the sequence 1, 4, 9, 16 in terms of n.
Solution
t
1
= 1
t
2
= 4 = 2
2
t
3
= 9 = 3
2
t
4
= 16 = 4
2
t
n
= n
2
Example 5
At a particular school, the number of students studying General Mathematics increases each
year. If in 2006 there are 40 students studying General Mathematics
a set up the difference equation if the number is increasing by ve students each year
b write down an expression for t
n
in terms of n for the difference equation found in a
c nd the number of students expected to be doing General Mathematics at the school in
2011.
Solution
a t
n
= t
n1
+5
b t
1
= 40,
t
2
= t
1
+5 = 45 = 40 +1 5
t
3
= t
2
+5 = 50 = 40 +2 5
Therefore t
n
= 40 +(n 1) 5
t
n
= 35 +5n
c n = 6
t
6
= 40 +5 5 = 65
Sixty-ve students will study General Mathematics in 2011.
Example 6
The height of a sand dune is increasing by 10% each year. It is currently 4 m high.
a Set up the difference equation that describes the height of the sand dune.
b Write down an expression for t
n
in terms of n for the difference equation found in a.
c Find the height of the sand dune seven years from now.
P1: FXS/ABE P2: FXS
9780521740494c05.xml CUAU033-EVANS August 24, 2009 9:30
118 Essential Advanced General Mathematics
Solution
a t
n
= t
n1
1.1
b t
1
= 4
t
2
= 4 1.1 = 4.4
t
3
= 4 (1.1)
2
= 4.84
Therefore t
n
= 4 (1.1)
n1
c Seven years from now implies n = 8
t
8
= 4 (1.1)
7
7.795
The sand dune will be 7.795 m high in 7 years
Using the TI-Nspire
Sequences defined in terms of n
Add a Graphs & Geometry application,
(c2) and choose Graph Type and
Sequence, (b35). Enter in a known
initial value or leave clear as shown. We could
have entered 7 in this example for the initial
value.
Set appropriate Window and press and
/Tto display table of values
Iteratively defined sequences
This type of sequence is easiest handled in a Lists & Spreadsheet application (c
3).
P1: FXS/ABE P2: FXS
9780521740494c05.xml CUAU033-EVANS August 24, 2009 9:30
Chapter 5 Sequences and series 119
Example 7
Use a CAS calculator to generate the sequence dened by the difference equation
t
n
= t
n1
+3, t
1
= 1 and sketch the graph of the sequence against n.
Solution
Use the arrows ( ) to name the rst
two columns n and tn respectively.
Enter 1 in cell A1 and enter 1 in cell B1.
Enter = a1 +1 in cell A2 and = b1 +3 in
cell B2.
Highlight the cells A2 and B2 using g
and the NavPad, and use Fill Down (b
33) to generate the sequence of
numbers.
To graph the sequence, open a Graphs &
Geometry application (c2) and graph
the sequence as a Scatter Plot (b3
4), using an appropriate Window (b
4).
Note that it is possible to see the
coordinates of the points using Trace (b
51).
Another method is to use a Graphs & Geometry application and then b35and
enter the difference equation.
P1: FXS/ABE P2: FXS
9780521740494c05.xml CUAU033-EVANS August 24, 2009 9:30
120 Essential Advanced General Mathematics
Using the Casio ClassPad
The Casio Classpad spreadsheet is an efcient
way to produce and graph the sequence. It is
similar to operating in a computer spreadsheet
such as Microsoft Excel.
Enter the values for n in column A, then in B1,
enter the formula = 3 +4

A1.
Highlight cell B2 and the cells below it and tap
Edit, Fill Range to complete the sequence.
Click the arrow beside , select graph type
and click on this icon to produce the graph.
For an iteratively dened sequence, the procedure
is similar except the cells in column A are each
dened in terms of the cell(s) above.
The entry for the Fibonacci sequence is
A1 = 1, A2 = 1, A3 = A1 +A2. Highlight the
formula in A3, together with as many cells as
required below, and then tap Edit, Fill Range to
complete the operation.
Note: In the screen shown, the formula in cell A3 is displayed in the formula bar at the
bottom of the screen.
P1: FXS/ABE P2: FXS
9780521740494c05.xml CUAU033-EVANS August 24, 2009 9:30
Chapter 5 Sequences and series 121
Exercise 5A
1 In each of the following an iterative denition for a sequence is given. List the rst ve
terms.
Example 1
a t
1
= 3, t
n
= t
n1
+4 b t
1
= 5, t
n
= 3t
n1
+4 c t
1
= 1, t
n
= 5t
n1
d t
1
= 1, t
n
= t
n1
+2 e t
n+1
= 2t
n
+t
n1
, t
1
= 1, t
2
= 3
2 Each of the following is a rule for a sequence. In each case nd t
1
, t
2
, t
3
, t
4
.
Example 2
a t
n
=
1
n
b t
n
= n
2
+1 c t
n
= 2n d t
n
= 2
n
e t
n
= 3n +2 f t
n
= (1)
n
n
3
g t
n
= 2n +1 h t
n
= 2 3
n1
3 For each of the following sequences
Examples 3, 4
i nd a possible rule for t
n
in terms of n
ii nd the difference equation.
a 3, 6, 9, 12 b 1, 2, 4, 8
c 1,
1
4
,
1
9
,
1
16
d 3, 6, 12, 24 e 4, 7, 10, 13 f 4, 9, 14, 19
4 Consider a sequence for which t
n
= 3n +1. Find t
n+1
, t
2n
.
5 Hamish collects Pokemon cards. He currently has 15 and he adds three to his collection
every week.
Example 5
a Set up the difference equation that will generate the number of cards Hamish has in
any given week.
b Write down an expression for t
n
in terms of n for the difference equation found in a.
c Find the number of cards Hamish should have after another 12 weeks.
6 Isobel can swim 100 m in 94.3 s. She aims to reduce her time by 4% each week.
Example 6
a Set up the difference equation that generates Isobels time for the 100 m in any given
week.
b Write down an expression for t
n
in terms of n for the difference equation found in a.
c Find the time in which Isobel expects to be able to complete the 100 m after another
8 weeks.
7 Stephen is a sheep farmer with a ock of 100 sheep. He wishes to increase the size of his
ock by both breeding and buying new stock. He estimates that 80% of his sheep will
produce one lamb each year and he intends to buy 20 sheep to add to the ock each year.
Assuming no sheep die
a write the difference equation for the expected number of sheep at the end of each year
(let t
0
= 100)
b calculate the number of sheep at the end of each of the rst ve years.
P1: FXS/ABE P2: FXS
9780521740494c05.xml CUAU033-EVANS August 24, 2009 9:30
122 Essential Advanced General Mathematics
8 Alison invests $2000 at the beginning of the year. At the beginning of each of the
following years, she puts a further $400 into the account. Compound interest of 6% p.a. is
paid on the investment at the end of each year.
a Write down the amount of money in the account at the end of each of the rst three
years.
b Set up a difference equation to generate the sequence for the investment. (Let t
1
be the
amount in the investment at the end of the rst year.)
c With a calculator or spreadsheet, use the difference equation to nd the amount in the
account after ten years.
9 For each of the following difference equations, use a graphics calculator to nd the rst
six terms of the sequence dened and sketch the graph of these terms against n.
Example 7
a t
n
= t
n1
+3, t
1
= 1 b t
n
= t
n1
2, t
1
= 3
c t
n
= 2t
n1
, t
1
=
1
2
d t
n
=
1
2
t
n1
, t
1
= 32
e t
n
= (t
n1
)
2
, t
1
= 1.1 f t
n
=
2
3
t
n1
, t
1
= 27
g t
n
= 2t
n1
+5, t
1
= 1 h t
n
= 4 t
n1
, t
1
= 3
10 a For a sequence for which t
n
= 2
n1
, nd t
1
, t
2
, t
3
.
b For a sequence for which u
n
=
1
2
(n
2
n) +1, nd u
1
, u
2
, u
3
.
c What do you notice? d Find t
4
and u
4
.
11 If S
n
= an
2
+bn, a R, b R, nd S
1
, S
2
, S
3
and S
n+1
S
n
.
12 For the sequence dened by t
1
= 1, t
n+1
=
1
2

t
n
+
2
t
n

, nd t
2
, t
3
, t
4
. The terms of the
sequence are successive rational approximations of a real number. Can you recognise the
number?
13 The Fibonacci sequence is dened by t
1
= 1, t
2
= 1, t
n+2
= t
n+1
+t
n
(n N).
Use the rule to nd t
3
, t
4
, t
5
. Show that t
n+2
= 2t
n
+t
n1
(n N\{1}).
5.2 Arithmetic sequences
A sequence in which each successive term is found by adding a constant amount to the previous
term is called an arithmetic sequence. For example, 2, 5, 8, 11. . . . is an arithmetic sequence.
An arithmetic sequence can be dened by a difference equation of the form
t
n
= t
n1
+d, where d is a constant.
If the rst term of an arithmetic sequence t
1
= a then the nth term of the sequence can also
be described by the rule
t
n
= a +(n 1)d where a = t
1
and d = t
n
t
n1
where d is the common difference.
P1: FXS/ABE P2: FXS
9780521740494c05.xml CUAU033-EVANS August 24, 2009 9:30
Chapter 5 Sequences and series 123
Example 8
Find the 10th term of the arithmetic sequence 4, 1, 2, 5 . . .
Solution
a = 4, d = 3
t
n
= a +(n 1)d
t
10
= 4 +(10 1)3
t
10
= 23
Example 9
A national park has a series of huts along one of its mountain trails. The rst hut is 5 km from
the start of the trail, the second is 8 km from the start, the third 11 km and so on.
a How far from the start of the trail is the sixth hut?
b How far is it from the sixth hut to the twelfth hut?
Solution
a Distances of the huts from the start of the trail form an arithmetic sequence with
a = 5 and d = 3.
For the sixth hut t
6
= a +5d
t
6
= 5 +5 3 = 20
The sixth hut is 20 km from the start of the trail
b For the twelfth hut t
12
= a +11d
t
12
= 5 +11 3 = 38
distance from sixth to the twelfth hut = t
12
t
6
= 38 20 = 18 km
The twelfth hut is 18 km from the sixth hut.
Example 10
The 12th term of an arithmetic sequence is 9 and the 25th term is 100. Find a and d.
Solution
Since t
n
= a +(n 1)d
9 = a +11d . . . 1
100 = a +24d . . . 2
Subtract 1 from 2
91 = 13d
d = 7
P1: FXS/ABE P2: FXS
9780521740494c05.xml CUAU033-EVANS August 24, 2009 9:30
124 Essential Advanced General Mathematics
From 1 we have
9 = a +11(7)
a = 68
Arithmetic mean
The arithmetic mean of two numbers a and b is dened as
a +b
2
.
If the numbers a, c and b are consecutive terms of an arithmetic sequence, then
c a = b c
2c = a +b
c =
a +b
2
i.e. c is the arithmetic mean of a and b.
Exercise 5B
1 For the arithmetic sequence where t
n
= a +(n 1)d, nd the rst four terms given that
Example 8
a a = 0, d = 2 b a = 3, d = 5 c a = d =

5 d a = 11, d = 2
2 Find a and d and hence nd the rule of the arithmetic sequence whose rst few terms are
a 3, 7, 11 b 3, 1, 5 c
1
2
,
3
2
,
7
2
,
11
2
d 5

5, 5, 5 +

5
3 a An arithmetic sequence has a rst term of 5 and a common difference of 3. Find the
thirteenth term.
b An arithmetic sequence has a rst term of 12 and a common difference of 4. Find
the tenth term.
c In an arithmetic sequence a = 25 and d = 2.5. Find the ninth term.
d In an arithmetic sequence a = 2

3 and d =

3. Find the fth term.


4 David goes shing every day for 10 days. On the rst day he catches four sh and each
day after that he catches two more than the previous day.
Example 9
a How many sh did David catch on the sixth day?
b How many sh did he catch on the 10th day?
c On which day did he catch 10 sh?
5 An amphitheatre has 25 seats in row A, 28 seats on row B, 31 seats in row C and so on.
a How many seats in row P?
b How many seats are there in row X?
c Which row has 40 seats in it?
6 In each of the following, t
n
is the nth term of an arithmetic sequence.
a Find t
5
if t
1
= 6, t
2
= 10. b Find t
12
if t
1
= 5, t
2
= 2.
c Find n if t
1
= 16, t
2
= 13 and t
n
= 41.
d Find n if t
1
= 7, t
2
= 11 and t
n
= 227.
P1: FXS/ABE P2: FXS
9780521740494c05.xml CUAU033-EVANS August 24, 2009 9:30
Chapter 5 Sequences and series 125
7 For an arithmetic sequence the rst term is 7 and the thirtieth term is 108
1
2
. Find the
common difference.
Example 10
8 The number of people who go to see a movie over a period of a week follows an arithmetic
sequence. If on the rst day only three people go to the movie but on the sixth day 98 go,
nd the rule for the sequence and hence determine how many attend on the seventh day.
9 For an arithmetic sequence, t
3
= 18 and t
6
= 486, nd the rule for the sequence,
i.e. nd t
n
.
10 The number of laps a swimmer swims each week follows an arithmetic sequence. If in the
fth week she swims 24 laps and in the tenth week she swims 39 laps, how many laps did
she swim in the fteenth week?
11 In an arithmetic sequence, t
7
= 0.6 and t
12
= 0.4. Find t
20
.
12 An arithmetic sequence contains 10 terms. If the rst is 4 and the tenth is 30, what are the
other eight terms?
13 The number of goals kicked by a team in the rst six games of a season follows an
arithmetic sequence. If the team kicked 5 goals in the rst game and 15 in the sixth, how
many did they score in each of the other four games?
14 The rst term of an arithmetic sequence is a. The mth term is zero. Find the rule for t
n
for
the sequence.
15 For an arithmetic sequence, nd t
6
if t
15
= 3 +9

3 and t
20
= 38

3.
16 Find the arithmetic mean of
a 8 and 15
b
1
2

2 1
and
1
2

2 +1
17 Find x if 3x 2 is the arithmetic mean of 5x +1 and 11.
18 If a, 4a 4 and 8a 13 are successive terms of an arithmetic sequence, nd a.
19 If t
x
= y and t
y
= x, prove that t
x+y
, = 0 (t
x
and t
y
are the xth and yth terms of an
arithmetic sequence).
20 If a, 2a and a
2
are consecutive terms of an arithmetic sequence, nd a(a = 0).
5.3 Arithmetic series
The sum of the terms in a sequence is called a series. If the sequence in question is arithmetic,
the series is called an arithmetic series. The symbol S
n
is used to denote the sum of n terms of
a sequence.
i.e. S
n
= a +a +d +a +2d + +a +(n 1)d
P1: FXS/ABE P2: FXS
9780521740494c05.xml CUAU033-EVANS August 24, 2009 9:30
126 Essential Advanced General Mathematics
If this sum is written in reverse order, then
S
n
= a +(n l)d +a +(n 2)d + +a +d +a
Adding these two expression together gives
2S
n
= n[2a +(n 1)d]
S
n
=
n
2
[2a +(n 1)d]
and since the last term l = t
n
= a +(n 1)d
S
n
=
n
2
(a +l)
Example 11
A hardware store sells nails in a range of packets containing different numbers of nails. Packet
A contains 50 nails, packet B has 75 nails, packet C has 100 and so on.
a Find the number of nails in packet J.
b Lachlan buys one of each of packets A to J. How many nails in total does Lachlan have?
c Assuming he buys one of each packet starting at A, how many packets does he need to buy
to have a total of 1100 nails?
Solution
a a = 50, d = 25,
t
n
= a +(n 1)d
For packet J, t
10
= 50 +9 25
= 275
Packet J contains 275 nails
b a = 50, d = 25
S
n
=
n
2
(2a +(n 1)d)
S
10
=
10
2
(2(50) +11 25)
S
10
= 1625
Packets A to J contain a total of
1625 nails
c a = 50, d = 25, S
n
= 1100
S
n
=
n
2
(2a +(n 1)d)
S
n
=
n
2
(2(50) +(n 1)(25)) = 1100
n(100 +25n 25) = 2200
25n
2
+75n 2200 = 0
n
2
+3n 88 = 0
(n +11)(n 8) = 0
n = 11 or n = 8
since n > 0, n = 8
If Lachlan buys one of each of the rst eight packets (A to H) he will have
exactly 1100 nails.
P1: FXS/ABE P2: FXS
9780521740494c05.xml CUAU033-EVANS August 24, 2009 9:30
Chapter 5 Sequences and series 127
Example 12
For the arithmetic sequence 3, 6, 9, 12, . . . , calculate
a the sum of the rst 25 terms
b the number of terms in the series if S
n
= 1395.
Solution
a a = 3, d = 3, n = 25
S
n
=
n
2
[2a +(n 1)d]
=
25
2
[2(3) +(24)(3)]
= 975
b a = 3, d = 3, S
n
= 1395
S
n
=
n
2
[2a +(n 1)d]
1395 =
n
2
[2(3) +(n 1)(3)]
2790 = n[6 +3n 3]
2790 = 3n +3n
2
3n
2
+3n 2790 = 0
n
2
+n 930 = 0
(n 30)(n +31) = 0
n = 30 since n > 0
there are 30 terms in the series.
Example 13
For the arithmetic sequence 27, 23, 19, 15, . . . , 33, nd
a the number of terms b the sum of the terms.
Solution
a a = 27, d = 4, l = t
n
= 33
t
n
= a +(n 1)d
33 = 27 +(n 1)(4)
60 = (n 1)(4)
15 = n 1
n = 16
There are 16 terms in the series.
P1: FXS/ABE P2: FXS
9780521740494c05.xml CUAU033-EVANS August 24, 2009 9:30
128 Essential Advanced General Mathematics
b a = 27, l = t
n
= 33, n = 16
S
n
=
n
2
(a +l)
S
16
=
16
2
(27 33)
S
16
= 48
The sum of the terms is 48.
Example 14
The sum of the rst 10 terms of an arithmetic sequence is 48
3
4
. If the fourth term is 3
3
4
, nd
the rst term and the common difference.
Solution
t
4
= a +3d = 3
3
4
a +3d =
15
4
. . . 1
S
10
=
10
2
(2a +9d) = 48
3
4
10a +45d =
195
4
. . . 2
1 40 40a +120d = 150
2 4 40a +180d = 195
60d = 45
d =
3
4
Substitute in 1 a +3

3
4

=
15
4
a =
6
4
The rst term is 1
1
2
and the common difference is
3
4
.
Exercise 5C
1 For the arithmetic sequences
a 8, 13, 18, . . . , nd S
12
b 3.5, 1.5, 0.5, . . . , nd S
10
c
1

2
,

2,
3

2
, . . . , nd S
15
d 4, 1, 6, . . . , nd S
8
2 Greg goes shing every day for a week. On the rst day he catches seven sh and each
day he catches three more than the previous day. How many sh did he catch in total?
Example 11
3 There are 110 logs to be put in a pile, with 15 logs in the bottom layer, 14 in the next, 13
in the next and so on. How many layers will there be?
P1: FXS/ABE P2: FXS
9780521740494c05.xml CUAU033-EVANS August 24, 2009 9:30
Chapter 5 Sequences and series 129
4 Find the sum of the rst 16 multiples of 5.
5 Find the sum of all the even numbers between 1 and 99.
6 Doras walking club plans 15 walks for the summer. The rst walk is a distance of 6 km
and the last walk is distance of 27 km and the distances of each of the walks form an
arithmetic sequence,
a How far is the eighth walk?
b How far does the club plan to walk in the rst ve walks?
Dora goes away on holiday and misses the 9th, 10th and 11th walks but completes all
other walks.
c How far does Dora walk in total?
7 Liz has to proofread 500 pages of a new novel. She plans to do 30 pages on the rst day
and increase the number by ve each day.
a How many days will it take her to complete the proofreading?
She has only ve days to complete the task. She therefore decides to read 50 pages on
the rst day and increase the number she reads by a constant amount each day.
b By how many should she increase the number she reads each day if she is to meet her
deadline?
8 For the sequence 4, 8, 12, . . . , nd {n : S
n
= 180}.
9 The sum of m terms of an arithmetic sequence with rst term 5 and common difference
4 is 660. Find m.
Example 12
10 An assembly hall has 50 seats in row A, 54 seats in row B, 58 seats in row C, i.e. there are
four more seats in each row.
a How many seats in row J?
b How many seats are there altogether if the back row is row Z?
If on a particular day the front four rows are reserved for parents (and there is no other
seating for parents)
c how many parents can be seated
d how many students can be seated?
The hall is extended by adding more rows following the same pattern.
e If the nal capacity of the hall is 3410, how many rows were added?
11 A new golf club is formed with 40 members in its rst year. Each following year the
number of new members exceeds the number of retirements by 15. Each member pays
$120 p.a. in membership fees. Calculate the amount received from fees in the rst
12 years of the clubs existence.
12 In an arithmetic sequence, t
2
= 12 and S
12
= 18. Find a, d, t
6
and S
6
.
Example 14
13 The sum of the rst ten terms of an arithmetic sequence is 120 and the sum of the rst
twenty terms is 840. Find the sum of the rst thirty terms.
14 Evaluate 54 +48 +42 + +54.
Example 13
P1: FXS/ABE P2: FXS
9780521740494c05.xml CUAU033-EVANS August 24, 2009 9:30
130 Essential Advanced General Mathematics
15 If t
6
= 16 and t
12
= 28, nd S
14
.
16 For an arithmetic sequence, nd t
n
if:
a t
3
= 6.5, S
8
= 67
b t
4
=
6

5
, S
5
= 16

5
17 For the sequence with t
n
= bn (b R), nd
a t
n+1
t
n
b t
1
+t
2
+ +t
n
18 For a sequence where t
n
= 15 5n, nd t
5
and the sum of the rst 25 terms.
19 An arithmetic sequence has a common difference of d. If the sum of 20 terms is 25 times
the rst term, nd, in terms of d, the sum of 30 terms.
20 The sum of the rst n terms of a particular sequence is given by S
n
= 17n 3n
2
.
a Find an expression for the sum to (n 1) terms.
b Find an expression for the nth term of the sequence.
c Show that the corresponding sequence is arithmetic and nd a and d.
21 Three consecutive terms of an arithmetic sequence have a sum of 36 and a product of
1428. Find the three terms.
22 Show that the sum of the rst 2n terms of an arithmetic sequence is n times the sum of the
two middle terms.
5.4 Geometric sequences
A sequence in which each successive term is found by multiplying the previous term by a xed
amount is called a geometric sequence. For example, 2, 6, 18, 54, . . . is a geometric sequence.
A geometric sequence can be dened by an iterative equation of the form
t
n
= rt
n1
, where r is constant
If the rst term of a geometric sequence t
1
= a then the nth term of the sequence can also be
described by the rule
t
n
= ar
n1
, where r =
t
n
t
n1
and r is the common ratio.
Example 15
Calculate the tenth term of the sequence 2, 6, 18, . . .
Solution
a = 2, r = 3
t
n
= ar
n1
t
10
= 2 3
(101)
= 39 366
P1: FXS/ABE P2: FXS
9780521740494c05.xml CUAU033-EVANS August 24, 2009 9:30
Chapter 5 Sequences and series 131
Example 16
Georgina draws a pattern consisting of a number of similar equilateral triangles. The rst
triangle has sides of length 4 cm and the side length of each successive triangle is one and a
half times the side length of the previous one.
a How long is the side length of the fth triangle?
b Which triangle has a side length of 45
9
16
cm?
Solution
a a = 4, r =
3
2
t
n
= ar
n1
t
5
= ar
4
= 4

3
2

4
= 20
1
4
The fth triangle has a side length of 20
1
4
cm
b a = 4, r =
3
2
t
n
= 45
9
16
t
n
= ar
n1
= 45
9
16
which implies 4

3
2

n1
= 45
9
16
=
729
16
Hence

3
2

n1
=
729
64
Recognising that 729 = 3
6
and 64 = 2
6
yields

3
2

n1
=

3
2

6
Therefore n 1 = 6
and n = 7
The seventh triangle will have a side length of 45
9
16
cm
An application of geometric sequences is compound interest. Compound interest is interest
calculated at regular intervals on the total of the amount originally invested and the amount
accumulated in the previous years.
So $1000 invested at 10% per annum would grow to
1000 +10%(1000) = $1100 at the end of the rst year.
At the end of the second year, it will have grown to
(1000 +10%(1000)) +10%(1000 +10%(1000)) = $1210
P1: FXS/ABE P2: FXS
9780521740494c05.xml CUAU033-EVANS August 24, 2009 9:30
132 Essential Advanced General Mathematics
The value of the investment at the end of each year forms a geometric sequence. In the
above example
a = 1000, r = 1.1; i.e. r = 100%+10%
Example 17
Hamish invests $2500 at 7% p.a. compounded annually. Find
a the value of his investment after 5 years
b how long it takes until his investment is worth $10 000.
Solution
a = 2500, r = 1.07
a t
6
= ar
5
t
n
is the end of the (n 1)th year.
= 2500(1.07)
5
= 3506.38
The value of his investment after 5 years is $3506.38.
b t
n
= ar
n1
= 10 000
2500(1.07)
n1
= 10 000
(1.07)
n1
= 4
log
10
(1.07)
n1
= log
10
4
(n 1) log
10
(1.07) = log
10
4
n 1 =
log
10
4
log
10
(1.07)
n = 21.489
By the end of the 21st year, his investment will be worth in excess of $10 000.
Note: The number of years can also be found by trial and error or through using a
CAS calculator.
Example 18
The third term of a geometric sequence is 10 and the sixth term is 80. Find r and the rst term.
Solution
t
3
= ar
2
= 10 . . . 1
t
6
= ar
5
= 80 . . . 2
Divide 2 by 1
ar
5
ar
2
=
80
10
r
3
= 8
r = 2
P1: FXS/ABE P2: FXS
9780521740494c05.xml CUAU033-EVANS August 24, 2009 9:30
Chapter 5 Sequences and series 133
Substitute in 1 to nd a.
a 4 = 10
a =
5
2
The rst term is
5
2
Geometric mean
The geometric mean of two numbers a and b is

ab.
Note that if three numbers a, c, b are consecutive members of a geometric sequence
c
a
=
b
c
and c =

ab
Exercise 5D
1 For a geometric sequence t
n
= ar
n1
, nd the rst four terms given that
a a = 3, r = 2 b a = 3, r = 2 c a = 10 000, r = 0.1 d a = r = 3
2 Find the specied term in each of the following geometric sequences.
Example 15
a
15
7
,
5
7
,
5
21
, . . . nd t
6
b 1,
1
4
,
1
16
, . . . nd t
5
c

2, 2, 2

2, . . . nd t
10
d a
x
, a
x+1
, a
x+2
, . . . nd t
6
3 Find the rule for the geometric sequence whose rst few terms are
a 3, 2,
4
3
b 2, 4, 8, 16 c 2, 2

5, 10
4 For a geometric sequence the rst term is 25 and the fth term is
16
25
. Find the common
ratio.
5 A geometric sequence has rst term
1
4
and common ratio 2. Which term of the sequence
is 64?
6 If t
n
is the nth term of the following geometric sequences, nd n in each case.
a 2, 6, 18, . . . t
n
= 486 b 5, 10, 20, . . . t
n
= 1280
c 768, 384, 192, . . . t
n
= 3
d
8
9
,
4
3
, 2, . . . t
n
=
27
4
e
4
3
,
2
3
,
1
3
, . . . t
n
=
1
96
7 An art collector has a painting that is increasing in value by 8% each year. If the painting
is currently valued at $2500
Example 16
a how much will it be worth in 10 years
b how many years before its value exceeds $100 000?
P1: FXS/ABE P2: FXS
9780521740494c05.xml CUAU033-EVANS August 24, 2009 9:30
134 Essential Advanced General Mathematics
8 An algal bloom is growing in a lake. The area it covers triples each day.
a If it initially covers an area of 10 m
2
, how many square metres will it cover after
1 week?
b If the lake has a total area of 200 000 m
2
, how long before the entire lake is covered?
9 A ball is dropped from a height of 2 m and continues to bounce so that it rebounds to
3
4
of
the height from which it previously falls. Find the height it rises to on the fth bounce.
10 The Tour de Moravia is a cycling event which lasts for 15 days. On the rst day the
cyclists must ride 120 km and each successive day they ride 90% of the distance of the
previous day.
a How far do they ride on the eighth day? b On which day do they ride 30.5 km?
11 A child negotiates a new pocket money deal with her unsuspecting father in which she
receives 1 cent on the rst day of the month, 2 cents on the second, 4 cents on the third,
8 cents on the fourth and so on . . . until the end of the month. How much would the child
receive on the 30th day of the month? (Give your answer to the nearest thousand dollars.)
12 The number of sh in the breeding tanks of a sh farm follow a geometric sequence. The
third tank contains 96 sh and the sixth tank contains 768.
a How many sh are in the rst tank?
b How many sh are in the 10th tank?
13 The 12th term of a geometric sequence is 2 and the fteenth term is 54. Find the seventh
term.
Example 18
14 A geometric sequence has t
2
=
1
2

2
and t
4
=

2. Find t
8
.
15 The rst three terms of a geometric sequence are 4, 8, 16. Find the rst term which
exceeds 2000.
16 The rst three terms of a geometric sequence are 3, 9, 27. Find the rst term in the
sequence which exceeds 500.
17 $5000 is invested at 6% p.a. compounded annually.
Example 17
a Find the value of the investment after 6 years.
b Find how long it will take for the original investment to double in value.
18 How much would need to be invested at 8.5% p.a. compounded annually to yield a return
of $8000 after 12 years?
19 What annual compound interest rate would be required to triple the value of an investment
of $200 in 10 years?
20 The number of type A apple bugs present in an orchard is estimated to be 40 960 and is
reducing in number by 50% each week. At the same time it is estimated that there are
P1: FXS/ABE P2: FXS
9780521740494c05.xml CUAU033-EVANS August 24, 2009 9:30
Chapter 5 Sequences and series 135
40 type B apple bugs whose number is doubling each week. After how many weeks will
there be the same number of each type of bug?
21 Find the geometric means of
a 5 and 720 b 1 and 6.25 c
1

3
and

3 d x
2
y
3
and x
6
y
11
22 The fourth, seventh and sixteenth terms of an arithmetic sequence also form consecutive
terms of a geometric sequence. Find the common ratio of the geometric sequence.
5.5 Geometric series
The sum of the terms in a geometric sequence is called a geometric series. An expression for
S
n
, the sum of n terms of a geometric sequence, can be found using a similar method to that
used in the development of a formula for an arithmetic series.
Let S
n
= a +ar +ar
2
+ +ar
n1
. . . 1
Then r S
n
= ar +ar
2
+ar
3
+ +ar
n
. . . 2
Subtract 1 from 2
r S
n
, S
n
= ar
n
a
S
n
(r 1) = a(r
n
1)
and S
n
=
a(r
n
1)
r 1
For values of r such that 1 < r < 1, it is often more convenient to use the alternative formula
S
n
=
a(1 r
n
)
1 r
which is obtained by subtracting 2 from 1 above.
Example 19
Find the sum of the rst nine terms of the sequence
1
3
,
1
9
,
1
27
,
1
81
, . . .
Solution
a =
1
3
, r =
1
3
, n = 9
S
9
=
1
3

1
3

9
1

1
3
1
=
1
2

1
3

9
1

1
2
(0.999949)
0.499975
P1: FXS/ABE P2: FXS
9780521740494c05.xml CUAU033-EVANS August 24, 2009 9:30
136 Essential Advanced General Mathematics
Example 20
For the geometric sequence 1, 3, 9, . . . , nd how many terms must be added together to obtain
a sum of 1093.
Solution
a = 1, r = 3, S
n
= 1093
S
n
=
a(r
n
1)
r 1
S
n
=
1(3
n
1)
3 1
= 1093
3
n
1 = 1093 2
3
n
= 2187
Taking logarithms of both sides gives
log
10
3
n
= log
10
2187
n log
10
3 = log
10
2187
n =
log
10
2187
log
10
3
n = 7
Seven terms are required to give a sum of 1093.
A CAS calculator can also be used.
Example 21
In the 15-day Tour de Moravia the cyclists must ride 120 km and each successive day they ride
90% of the distance of the previous day.
a How far do they ride in total to the nearest km?
b After how many days will they have ridden half that distance?
Solution
a a = 120, r = 0.9
S
n
=
a(r
n
1)
r 1
S
15
=
120(1 (0.9)
15
)
1 0.9
= 952.93
953 km
b a = 120, r = 0.9, S
n
= 476.5 km
S
n
=
a(r
n
1)
r 1
S
n
=
120(1 (0.9)
n
)
1 0.9
= 476.5
P1: FXS/ABE P2: FXS
9780521740494c05.xml CUAU033-EVANS August 24, 2009 9:30
Chapter 5 Sequences and series 137
1 (0.9)
n
=
476.5 0.1
120
= 0.3971
(0.9)
n
= 1 0.3971
(0.9)
n
= 0.6029
Taking logarithms of both sides gives
log
10
(0.9)
n
= log
10
(0.6029)
n log
10
(0.9) = log
10
(0.6029)
n =
log
10
(0.6029)
log
10
(0.9)
n = 4.8023
on the fth day they pass the halfway mark.
Exercise 5E
1 Find the sum specied for each of the following geometric series.
Example 19
a 5 +10 +20 + , nd S
10
b 1 3 +9 , nd S
6
c
4
3
+
2
3

1
3
+ , nd S
9
2 Find
a 2 6 +18 +1458 b 4 +8 16 + 1024
c 6250 +1250 +250 + +2
3 Gerry owns a milking cow. On the rst day he milks the cow, it produces 600 mL of milk.
On each successive day, the amount of milk increases by 10%.
a How much milk does the cow produce on the seventh day?
b How much milk does it produce in the rst week?
4 An insurance salesman makes $15 000 commission on sales in his rst year. Each year, he
increases his sales by 5%.
Example 21
a How much commission would he make in his fth year?
b How much commission would he make in total over 5 years?
5 On Monday, William spends 20 minutes playing the piano. On Tuesday, he spends
25 minutes playing and on each successive day he increases the time he spends playing in
the same ratio.
a For how many hours does he play on Friday?
b How many hours in total does he play from Monday to Friday?
c On which day of the following week will his total time played pass 15 hours?
6 A ball dropped from a height of 15 m rebounds from the ground to a height of 10 m. With
each successive rebound, it rises two-thirds of the height of the previous rebound. What
total distance will it have travelled when it strikes the ground for the 10th time?
P1: FXS/ABE P2: FXS
9780521740494c05.xml CUAU033-EVANS August 24, 2009 9:30
138 Essential Advanced General Mathematics
7 Andrew invests $1000 at 20% simple interest for 10 years. Bianca invests her $1000 at
12.5% compound interest for 10 years. At the end of 10 years, whose investment is worth
more?
8 For the geometric sequence with nth term t
n
a t
3
= 20, t
6
= 160, nd S
5
b t
3
=

2, t
8
= 8, nd S
8
9 a How many terms of the geometric sequence where t
1
= 1, t
2
= 2, t
3
= 4, . . . must be
taken for S
n
= 255?
Example 20
b Let S
n
= 1 +2 +4 + +2
n1
. Find {n: S
n
> 1000 000}.
10 Find 1 x
2
+ x
4
x
6
+ + x
2m
(m is even).
5.6 Infinite geometric series
If the common ratio of a geometric sequence has a magnitude less than 1, i.e. 1 < r < 1,
then each successive term of the sequence is closer to zero.
e.g. 4, 2, 1,
1
2
,
1
4
, . . .
When the terms of the sequence are added, the corresponding series
a +ar +ar
2
+ +ar
n1
will approach a limiting value,
i.e. as n , S
n
a limiting value.
Such a series is called convergent.
In Example 19 from the previous section, it was found that for the sequence
1
3
,
1
9
,
1
27
,
1
81
, . . . , the sum of the rst nine terms, S
9
, was 0.499975
For the same sequence, S
20
= 0.4999999999 0.5
Given that S
n
=
a(1 r
n
)
1 r
S
n
=
a
1 r

ar
n
1 r
as n , r
n
0 and hence
ar
n
1 r
0
It follows then that the limit as n of S
n
is
a
1 r
So S

=
a
1 r
This is also referred to as the sum to innity of the series.
P1: FXS/ABE P2: FXS
9780521740494c05.xml CUAU033-EVANS August 24, 2009 9:30
Chapter 5 Sequences and series 139
Example 22
Find the sum to innity of the series 1 +
1
2
+
1
4
+
1
8
+
Solution
r =
1
2
, a = 1
S

=
1
1
1
2
= 2
Example 23
A square has a side length of 40 cm. A copy of the square is made so that the area of the copy
is 80% of the original. The process is repeated each time with the area of the new square being
80% of the previous one. If this process continues indenitely, nd the total area of all the
squares.
Solution
Area of rst square is 40
2
= 1600 cm
2
a = 1600, r = 0.8
S

=
a
1 r
S

=
1600
1 0.8
= 8000 cm
2
Example 24
Express the recurring decimal 0.

2 as a ratio of two integers.


Solution
0.

2 = 0.32 +0.0032 +0.000032 +


a = 0.32, r = 0.01
and S

=
0.32
0.99
=
32
99
i.e. 0.

2 =
32
99
Exercise 5F
1 Find
Example 22
a 1 +
1
5
+
1
25
+
1
125
+ b 1
2
3
+
4
9

8
27
+
P1: FXS/ABE P2: FXS
9780521740494c05.xml CUAU033-EVANS August 24, 2009 9:30
140 Essential Advanced General Mathematics
2 An equilateral triangle has perimeter p cm. The midpoints
Example 23
of the sides are joined to form another triangle, and this
process is repeated. Find the perimeter and area of the
nth triangle, and nd the limits as n of the
sums of perimeters and areas of the rst n triangles.
3 A rocket is launched into the air so that it reaches a height of 200 m in the rst second.
Each subsequent second it gains 6% less height. Find how high the rocket will climb.
4 A patient has an infection that, if it exceeds a certain level, will kill him. He is given a
drug that will inhibit the spread of the infection. The drug acts in such a way that the level
of infection only increases by 65% of the previous days level. On the rst day, the level of
infection is measured at 450.
The critical level of infection is 1280. Will the infection kill him?
5 A man can walk 3 km in the rst hour of a journey, but in each succeeding hour walks half
the distance covered in the preceding hour. Can he complete a journey of 6 km? Where
does this problem cease to be realistic?
6 A frog standing 10 m from the edge of a pond sets out to jump towards it. Its rst jump is
2 m, its second jump is 1
1
2
m, its third jump is 1
1
8
m and so on. Show that the frog will
never reach the edge of the pond.
7 A computer-generated virus acts in such a way that initially it blocks out a third of the
area of the screen of an infected computer. On each successive day, it blocks out a further
1
3
of the area it blocked the previous day. If the virus continues to act unchecked
indenitely, what percentage of the users screen will eventually be blocked out?
8 A stone is thrown so that it skips across the surface of a lake. If each skip is 30% less that
the previous skip, how long should the rst skip be if the total distance travelled by the
stone is 40 m?
9 A ball dropped from a height of 15 m rebounds from the ground to a height of 10 m. With
each successive rebound, it rises two-thirds of the height of the previous rebound. If it
continues to bounce indenitely, what is the total distance it will travel?
10 Express each of the following periodic decimals as the ratio of a pair of integers.
Example 24
a 0.

4 b 0.0

3 c 10.

3 d 0.0

5 e 0.

9 f 4.

1
11 The sum of the rst four terms of a geometric series is 30 and the sum to innity is 32.
Find the rst two terms.
12 Find the third term of a geometric sequence that has a common ratio of
1
4
and a sum to
innity of 8.
13 Find the common ratio of a geometric sequence that has a rst term of 5 and a sum to
innity of 15.
P1: FXS/ABE P2: FXS
9780521740494c05.xml CUAU033-EVANS August 24, 2009 9:30
Chapter 5 Sequences and series 141
5.7 Fixed point iteration
The solution(s) to equations may be found using a numerical method that involves generating a
sequence of numbers. This method is particularly useful when solving equations for which
using an analytic method may be problematic or impossible.
The solution to an equation of the form f (x) = 0 may be found by rst nding an
approximation to the solution and then using this rst approximation to generate a better
approximation, which is in turn used to produce an even better approximation and so on.
This process of using a previous value to generate the next value is called iteration. If the
sequence of numbers produced using the iterative process converges to a limit, this limit will
be the solution of the equation in question.
Consider the equation f (x) = 0. Begin by rewriting the equation in the form x = g(x).
If x
1
is the initial approximation for the solution to the equation, the second approximation is
found by evaluating
x
2
= g(x
1
).
This value is then used to generate a third approximation
x
3
= g(x
2
), and so on.
If the sequence is convergent, each successive term will be closer to the actual solution of
the original equation i.e., the sequence of numbers generated by the equation x
n
= g(x
n1
)
converges to a value which is the solution to the equation x = g(x) (which is of course the
solution to the equation f (x) = 0).
The iterative process is continued until the value of x
n
is equal to x
n1
to a pre-determined
level of accuracy such as ve decimal places. This type of iteration is called xed point
iteration.
Example 25
Write down the rst ve terms generated by the iterative equation x
n
=
x
n1
5
+1, x
1
= 2 and
hence state if the sequence produced appears to be convergent.
Solution
x
1
= 2
x
2
=
2
5
+1 = 1.4
x
3
=
1.4
5
+1 = 1.28
x
4
=
1.28
5
+1 = 1.256
x
5
=
1.256
5
+1 = 1.2512
x
6
=
1.2512
5
+1 = 1.25024
The sequence generated appears to be convergent.
P1: FXS/ABE P2: FXS
9780521740494c05.xml CUAU033-EVANS August 24, 2009 9:30
142 Essential Advanced General Mathematics
Example 26
Given that the solution to the equation f (x) = 0 where f (x) =

x x +4 is approximately
6.5, use xed point iteration to nd the solution to the equation correct to ve decimal places.
Solution
First rearrange the equation into the form x = g(x).
Note: There may be a number of ways to re-arrange the original equation producing
different functions g(x). The implications of these different forms will be discussed
later.

x x +4 = 0
Therefore x =

x +4 i.e., g(x) =

x +4
Using x
1
= 6.5
Therefore x
2
=

x
1
+4
=

6.5 +4
= 6.549509757
x
3
=

x
2
+4 x
4
=

x
3
+4
=

6.549509757 +4 =

6.559201 +4
= 6.559201 = 6.561093712
x
5
= 6.561463197
x
6
= 6.56153532 (= 6.56154 to ve decimal places)
x
7
= 6.561549398 (= 6.56155 to ve decimal places)
x
8
= 6.561552146 (= 6.56155 to ve decimal places)
Since the values of x
8
= x
7
to the required level of accuracy the iteration process is
terminated.
Hence the solution to the equation

x x +4 = 0 is 6.56155 correct to ve
decimal places.
Using the TI-Nspire
A solution using this technique can be
found efciently using the TI-Nspire
calculator.
Using the above example, start by
entering the initial approximation x
1
= 6.5
followed by enter (
enter
). Then enter

ans +4 and repeatedly press enter. The


successive terms of the sequence will be
generated until the required level of
accuracy is achieved.
P1: FXS/ABE P2: FXS
9780521740494c05.xml CUAU033-EVANS August 24, 2009 9:30
Chapter 5 Sequences and series 143
Using the Casio ClassPad
A solution using this technique can be found
efciently using the CAS calculator. Enter the
initial approximation 6.5 in the rst entry line.
Then, in the next line, enter 4 +

( as
shown. Repeatedly tapping EXE will produce
successive terms in the sequence until the
desired accuracy is achieved.
Note: It is useful to change the display mode
from Standard to Decimal to return answers in the
appropriate form.
In the above example a decision was made to rearrange the original equation

x x +4 = 0
into an equation of the form x =

x +4 so that g(x) =

x +4. An alternative
rearrangement could have been used.
Again consider the equation

x x +4 = 0
i.e.

x = x 4
x = (x 4)
2
i.e., in this case g(x) = (x 4)
2
Again using x
1
= 6.5 a solution can be sought using the iterative process
If x
1
= 6.5
Therefore x
2
= (x
1
4)
2
= (6.5 4)
2
= 6.25
x
3
= (x
2
4)
2
x
4
= (x
3
4)
2
= (6.25 4)
2
= (5.0625 4)
2
= 5.0625 = 1.12890625
x
5
= 8.243179321 x
6
= 18.00457075 x
7
= 196.128002 x
8
= 36913.16914
It is clear that the sequence of numbers generated by the iterative equation x
n
= (x
n1
4)
2
is not convergent and the solution to the equation

x x +4 = 0 cannot be found using the


iterative process with this particular rearrangement.
This method of nding solutions to equations is not universally applicable. It is, however,
possible to establish whether the sequence to be generated will converge, therefore producing a
solution, by considering the graph of y = g(x).
P1: FXS/ABE P2: FXS
9780521740494c05.xml CUAU033-EVANS August 24, 2009 9:30
144 Essential Advanced General Mathematics
In solving the equation

x x +4 = 0, two different rearrangements were used to


produce different functions.
First consider the graph of f (x) =

x x +4.
4
6
2
0
2
4
1
0
1 2 3 4 5 6 8 9 10
x
7
y
It appears that a solution to the equation
f (x) = 0 occurs between x = 6 and x = 7.
Now consider the graphs of y = x and y = g(x),
where g(x) =

x +4.
y = g (x)
y = x
4
6
8
2
0
0
1 2 3 4 5 6 8 9 10
x
7
y
Also consider the graphs of y = x and y = g(x),
where g(x) = (x 4)
2
.
y = g(x)
y = x
4
6
8
2
0
0 1 2 3 4 5 6 8 9 10
x
7
y
In both of these pairs of graphs it is clear that a
solution to the equation x = g(x) occurs between
x = 6 and x = 7. What is of interest in both
cases is the gradient of the graph of y = g(x) in the
vicinity of the actual solution.
In the rst of the two, the gradient of y = g(x) in
the vicinity of x = 6.5 is quite small (less than that
of the line y = x); however in the second the
gradient of y = g(x) in the vicinity of x = 6.5 is quite large (greater than that of the line
y = x).
It is the gradient of the function of y = g(x) in the vicinity of the solution which will
determine if the iterative process is to be successful.
(If students have studied differential calculus a more rigorous examination of the gradient of
y = g(x) may be done, however an informal recognition of the signicance of the gradient of
y = g(x) is sufcient for students to appreciate that the iterative process will not always
succeed.)
P1: FXS/ABE P2: FXS
9780521740494c05.xml CUAU033-EVANS August 24, 2009 9:30
Chapter 5 Sequences and series 145
Exercise 5G
1 a Write down the rst six terms generated by the following iterative equations
Example 25
i x
n
=
x
n1
4
+2, x
1
= 3
ii x
n
= x
2
n1
3, x
1
= 1
iii x
n
= 3x
2
n1
+1, x
1
= 2 iv x
n
=

x
n1
+2 +1, x
1
= 3
b Which of the sequences produced in a are convergent?
2 Use xed point iteration to nd a solution to the equations f (x) = 0. In each question, the
Example 26
initial approximation x
1
is given. Note: It may be necessary to try more than one
re-arrangement of f (x) = 0 before a solution can be found successfully.
a f (x) = x
3
+4x 3, x
1
= 1 b f (x) = x
3
+ x 1, x
1
= 1
c f (x) =
x
2
3
x 1, x
1
= 1
d f (x) = x
4
x 2, x
1
= 1
e f (x) = 2
x
4x, x
1
= 0.5 f f (x) = x +log
10
x +2, x
1
= 5
g f (x) = 4x 2
x
3, x
1
= 1 h f (x) = x
3
3x +1, x
1
= 0.5
P1: FXS/ABE P2: FXS
9780521740494c05.xml CUAU033-EVANS August 24, 2009 9:30
R
e
v
i
e
w
146 Essential Advanced General Mathematics
Chapter summary
Sequences
The nth term of a sequence is denoted using the symbol t
n
.
A difference equation enables each subsequent term to be found using the previous term.
A rule specied in this way is said to be dened iteratively.
e.g. t
1
= 1, t
n
= t
n1
+2
A sequence may be dened by a rule that is stated in terms of n.
e.g. t
n
= 2n
An arithmetic sequence is a sequence where
t
n
= a +(n 1)d with d = t
n
t
n1
where a is the rst term and d is called the common difference.
The arithmetic mean of two numbers a and b is dened as
a +b
2
The sum of the terms in an arithmetic sequence is called an arithmetic series.
The sum to n terms of an arithmetic sequence,
S
n
=
n
2
[2a +(n 1)d]
=
n
2
[a +l] where l = the last term (l = t
n
= a +(n 1)d)
A geometric sequence is a sequence where
t
n
= ar
n1
with r =
t
n
t
n1
a is the rst term and r is called the common ratio.
The sum of the terms in a geometric sequence is called a geometric series.
The sum of n terms of a geometric sequence is
S
n
=
a(r
n
1)
(r 1)
r = 1
or S
n
=
a(1 r
n
)
(1 r)
If 1 < r < 1, the sequence is convergent and S
n
approaches a limiting value.
The sum to an innite number of terms (sum to innity) is denoted by S

and S

=
a
1 r
.
Fixed point iteration can be used to nd the solution(s) to equations of the form f (x) = 0
by nding the sequence of numbers generated by the equation x
n
= g(x
n1
), as long as the
sequence is convergent. The equation x
n
= g(x
n1
) is found by an appropriate
rearrangement of the equation f (x) = 0.
P1: FXS/ABE P2: FXS
9780521740494c05.xml CUAU033-EVANS August 24, 2009 9:30
R
e
v
i
e
w
Chapter 5 Sequences and series 147
Multiple-choice questions
1 The rst three terms of the sequence dened by the rule t
n
= 3n +2 are
A 1, 2, 3 B 2, 4, 6 C 5, 7, 9 D 5, 8, 11 E 5, 8, 10
2 If t
1
= 3, t
n+1
= t
n
+3, then t
4
is
A 4 B 12 C 9 D 15 E 14
3 For the arithmetic sequence 10, 8, 6 . . . t
10
=
A 8 B 10 C 12 D 10 E 8
4 For the arithmetic sequence 10, 8, 6 . . . S
10
=
A 10 B 0 C 10 D 20 E 20
5 If 58 is the nth term of the arithmetic sequence 8, 13, 18 . . . then n =
A 12 B 11 C 10 D 5 E 3
6 The sixth term of the geometric sequence 12, 8,
16
3
, . . . . is . . .
A
16
3
B
128
27
C
64
81
D
128
81
E
256
81
7 For the sequence 8, 4, 2, . . . S
6
=
A
1
4
B 15
1
2
C 15
7
8
D 15 E 15
3
4
8 For the sequence 8, 4, 2, . . . S

=
A
1
2
B 0 C 16 D 4 E
9 $2000 is invested at 5.5% p.a. compounded annually. The value of the investment after
6 years is
A $13 766.10 B $11 162.18 C $2550 D $2613.92 E $2757.69
10 If S

= 37.5 and r =
1
3
, then a equals
A
2
3
B 12.5 C 16
2
3
D 25 E 56.25
Short-answer questions (technology-free)
1 Find the rst six terms of the following sequences
a t
1
= 3, t
n
= t
n1
4 b t
1
= 5, t
n
= 2t
n1
+2
2 Find the rst six terms of the following sequences
a t
n
= 2n b t
n
= 3n +2
3 Nick invests $5000 at 5% p.a. compound interest at the beginning of the year. At the
beginning of each of the following years he puts a further $500 into the account.
a Write down the amount of money in the account at the end of each of the rst two years.
b Set up a difference equation to generate the sequence for the investment.
4 The fourth term of an arithmetic sequence is 19 and the seventh term is 43. Find the 20th
term.
5 In an arithmetic sequence, t
5
= 0.35 and t
9
= 0.15. Find t
14
.
P1: FXS/ABE P2: FXS
9780521740494c05.xml CUAU033-EVANS August 24, 2009 9:30
R
e
v
i
e
w
148 Essential Advanced General Mathematics
6 An arithmetic sequence has t
6
= 24 and t
14
= 6. Find S
10
.
7 For the arithmetic sequence 5, 2, 9, . . . , nd {n: S
n
= 402}.
8 The sixth term of a geometric sequence is 9 and the tenth is 729. Find the fourth term.
9 One thousand dollars is invested at 3.5% p.a. compounded annually. Find the value of the
investment after n years.
10 The rst term of a geometric sequence is 9 and the third term is 4. Find the possible values
for the second and fourth terms.
11 The sum of three consecutive terms of a geometric sequence is 24 and the sum of the next
three terms is also 24. Find the sum of the rst 12 terms.
12 Find the sum of the rst eight terms of a geometric sequence with rst term 6 and common
ratio 3.
13 Find the sum to innity of 1
1
3
+
1
9

1
27
+
14 The numbers x, x +4, 2x +2 are three successive terms of a geometric sequence. Find the
value of x.
Extended-response questions
1 A rm offering a do-it-yourself picture frame kit makes the kit in various sizes. Size 1
contains 0.8 m of moulding, size 2 contains 1.5 m, size 3 contains 2.2 m, . . . and so on.
a Form the sequence of lengths of mouldings.
b Is the sequence of lengths of moulding an arithmetic sequence?
c Find the length of moulding contained in the largest kit, size 12.
2 A rm proposes to sell coated seeds in packs containing the following number of seeds:
50, 75, 100, 125,
a Is this an arithmetic sequence?
b Find a formula for the nth term.
c Find the number of seeds in the 25th size packet.
3 A number of telegraph poles are to be placed in a straight line between two towns, A and B,
which are 32 km apart. The rst is placed 5 km from town A, the last is placed 3 km from
town B. The poles are placed so that the intervals starting from town A and nishing at
town B are 5, 5 d, 5 2d, 5 3d, . . . , 5 6d, 3. There are seven poles. How far is the
fth pole from town A and how far is it from town B?
4 A new, electronic desk-top telephone exchange, for use in large organisations, is available
in various sizes.
Size 1 can handle 20 internal lines
Size 2 can handle 36 internal lines
Size 3 can handle 52 internal lines
Size 4 can handle 68 internal lines, and so on . . .
Size n can handle T
n
internal lines
(contd)
P1: FXS/ABE P2: FXS
9780521740494c05.xml CUAU033-EVANS August 24, 2009 9:30
R
e
v
i
e
w
Chapter 5 Sequences and series 149
a Continue the sequence up to T
8
.
b Write down a formula for T
n
in terms of n.
c A customer said he needed an exchange to handle 196 lines. Is there a version of the
desk-top exchange which will just do this? If so, which size is it? If not, which is the
next largest size?
5 A rm making nylon thread made it in the following deniers (thicknesses):
2, 9, 16, 23, 30, . . . etc.
a Find the denier number, D
n
, of the rms nth thread in order of increasing thickness.
A request came in for some very heavy 191 denier thread, but this turned out to be one
stage beyond the thickest thread made by the rm.
b How many different thicknesses did the rm make?
6 A new house appears to be slipping down a hillside. The rst year it slipped 4 mm, the
second year 16 mm, the third year 28 mm. If it goes on like this, how far will it slip during
the 40th year?
7 Anna sends 16 Christmas cards the rst year, 24 the second year, 32 the next year and so
on. How many Christmas cards will she have sent altogether after ten years if she keeps
increasing the number sent each year in the same way?
8 Each time Lee rinses her hair after washing it, the result is to remove a quantity of shampoo
from the hair. With each rinsing the quantity of shampoo removed is a tenth of the previous
rinse.
a If Lee rinses out 90 mg of shampoo with the rst rinse, how much will she have washed
out altogether after six rinses?
b How much shampoo do you think was present in her hair at the beginning?
9 A prisoner is trapped in an underground cell which is inundated with a sudden rush of
water which comes up to a depth of 1 m, a third of the height of the ceiling (3 m). After an
hour a second inundation occurs, but this time the water level rises by only
1
3
m. After a
second hour another inundation of water raises the level by
1
9
m. If this process continues
for 6 hours, write down
a the amount the water level will rise at the end of the sixth hour,
b the total height of the water level then.
If this process continues, do you think the prisoner, who cannot swim, will drown? Why?
10 After an undetected leak in a storage tank, the staff at an experimental station were
subjected to 500 curie hours of radiation the rst day, 400 curie hours the second day, 320
the third day and so on.
Find the number of curie hours they were subjected to
a on the 14th day b during the rst 5 days of the leak.
P1: FXS/ABE P2: FXS
9780521740494c05.xml CUAU033-EVANS August 24, 2009 9:30
R
e
v
i
e
w
150 Essential Advanced General Mathematics
11 A rubber ball is dropped from a height of 81 m. Each time it strikes the ground, it rebounds
two-thirds of the distance through which it has fallen.
a Find the height the ball reaches after the sixth bounce.
b Assuming the ball continues to bounce indenitely, nd the total distance travelled by
the ball.
12 In payment for loyal service to the king, a wise peasant asked to be given one grain of rice
for the rst square of a chessboard, two grains for the second square, four for the third
square and so on for all 64 squares of the board. The king thought this seemed fair and
readily agreed, but was horried when the court mathematician informed him of how many
grains of rice he would have to pay the peasant. How many grains of rice did the king have
to pay? (Leave your answer in index form.)
13 a In its rst month of operation a cement factory, A, produces 4000 tonnes of cement. In
each successive month, production rises by 250 tonnes per month. This growth in
production is illustrated for the rst ve months in the table shown.
Month number (n) 1 2 3 4 5
Amount of cement produced (tonnes) 4000 4250 4500 4750 5000
i Find an expression in terms of (n) for the amount of cement produced in the nth
month.
ii Find an expression in terms of n for the total amount of cement produced in the rst
n months.
iii In which month is the amount of cement produced 9250 tonnes?
iv In month m the amount of cement produced is T tonnes. Find m in terms of T
v The total amount of cement produced in the rst p months is 522 750. Find the
value of p.
b A second factory, B, commences production at exactly the same time as the rst. In its
rst month of production it produces 3000 tonnes of cement. In each successive month,
production increases by 8%.
i Find an expression for the total amount of cement produced by this factory after n
months.
ii Let Q
A
be the total amount of cement produced by factory A in the rst n months
and Q
B
be the total amount of cement produced by factory B in the rst n months.
Find an expression in terms of n for Q
B
Q
A
and nd the smallest value of n for
which Q
B
Q
A
0.
14 By using xed point iteration to solve the equation x
2
8 = 0, nd the value of

8 correct
to ve decimal places. Hint: Add x
2
to both sides of the equation and then re-arrange to
produce an iterative equation of the form x
n
= g(x
n1
).
P1: FXS/ABE P2: FXS
9780521740494c06.xml CUAU033-EVANS August 22, 2009 8:51
C H A P T E R
6
Algebra II
Objectives
To understand equality of polynomials
To use equating coefficients to solve problems
To solve quadratic equations by various methods
To use rates to solve problems
To resolve a rational algebraic expression into partial fractions
To find the coordinates of the points of intersection of linear graphs with
r
parabolas
r
rectangular hyperbolae
r
circles
6.1 Polynomial identities
Polynomials are introduced in Chapter 3 of Essential Mathematical Method 1 & 2 CAS. A
polynomial function has a rule of the type
y = a
n
x
n
+a
n1
x
n1
+. . . a
1
x +a
0
, n N
where a
0
, a
1
, . . . a
n
are numbers called coefcients.
The degree of a polynomial is given by the value of n, the highest power of x with non-zero
coefcient.
Two polynomials are equal if they give the same value for all x. If two polynomials are
equal then they are of the same degree, and corresponding coefcients are equal.
For example, if ax +b = cx
2
+dx +e, then c = 0, d = a and e = b
if x
2
x 12 = x
2
+(a +b)x +ab then ab = 12 and a +b = 1
This process is called equating coefcients.
Example 1
If the expressions (a +2b)x
2
(a b)x +8 and 3x
2
6x +8 are equal for all x, nd the
values of a and b.
151
P1: FXS/ABE P2: FXS
9780521740494c06.xml CUAU033-EVANS August 22, 2009 8:51
152 Essential Advanced General Mathematics
Solution
If (a +2b)x
2
(a b)x +8 = 3x
2
6x +8 for all x,
then a +2b = 3 and (a b) = 6
Solve as simultaneous equations
a +2b = 3 . . . 1
a +b = 6 . . . 2
Add 1 and 2
3b = 3
b = 1
Substitute into 2
a = 5
Example 2
Express x
2
in the form c(x 3)
2
+a(x 3) +d.
Solution
If x
2
= c(x 3)
2
+a(x 3) +d,
then x
2
= c(x
2
6x +9) +a(x 3) +d
= cx
2
+(a 6c)x +9c 3a +d
which implies c = 1 . . . 1
a 6c = 0 . . . 2
9c 3a +d = 0 . . . 3
From 2 a = 6
and from 3 9 18 +d = 0
i.e. d = 9
x
2
= (x 3)
2
+6(x 3) +9
Example 3
Find the values of a, b, c and d such that
x
3
= a(x +2)
3
+b(x +1)
2
+cx +d for all x.
Solution
Expand the right hand side.
a(x
3
+6x
2
+12x +8) +b(x
2
+2x +1) +cx +d
Collect like terms.
ax
3
+(6a +b)x
2
+(12a +2b +c)x +(8a +b +d)
If x
3
= ax
3
+(6a +b)x
2
+(12a +2b +c)x +(8a +b +d)
P1: FXS/ABE P2: FXS
9780521740494c06.xml CUAU033-EVANS August 22, 2009 8:51
Chapter 6 Algebra II 153
then a = 1 . . . 1
6a +b = 0 . . . 2
12a +2b +c = 0 . . . 3
8a +b +d = 0 . . . 4
Substituting a = 1 into 2 gives
b = 6a
= 6
Substituting a = 1 and b = 6 into 3 gives
12 12 +c = 0
c = 0
Substituting a = 1 and b = 6 into 4 gives
8 6 +d = 0
d = 2
x
3
= (x +2)
3
6(x +1)
2
2
Example 4
Show that 2x
3
5x
2
+4x +1 cannot be expressed in the form a(x +b)
3
+c.
Solution
Assume that 2x
3
5x
2
+4x +1 can be expressed in the form a(x +b)
3
+c
i.e., 2x
3
5x
2
+4x +1 = a(x +b)
3
+c
Expanding the right hand side
2x
3
5x
2
+4x +1 = a(x
3
+3bx
2
+3b
2
x +b
3
) +c
= ax
3
+3abx
2
+3ab
2
x +ab
3
+c
Equating coefcients a = 2 . . . 1
3ab = 5 . . . 2
3ab
2
= 4 . . . 3
and ab
3
+c = 1 . . . 4
From 2 b =
5
6
, but from 3 b =

2
3
=

6
3
We have a contradiction and therefore have shown that 2x
3
5x
2
+4x +1 cannot be
expressed in the form a(x +b)
3
+c.
Exercise 6A
1 If ax
2
+bx +c = 10x
2
7, nd the values of a, b and c.
Example 1
2 If (2a b)x
2
+(a +2b)x +8 = 4x
2
3x +8, nd the values of a and b.
P1: FXS/ABE P2: FXS
9780521740494c06.xml CUAU033-EVANS August 22, 2009 8:51
154 Essential Advanced General Mathematics
3 If (2a 3b)x
2
+(3a +b)x +c = 7x
2
+5x +7, nd the values of a, b and c.
4 If 2x
2
+4x +5 = a(x +b)
2
+c, nd the values of a, b and c.
5 Express x
2
in the form c(x +2)
2
+a(x +2) +d.
Example 2
6 Express x
3
in the form (x +1)
3
+a(x +1)
2
+b(x +1) +c.
7 Find the values of a, b and c such that x
2
= a(x +1)
2
+bx +c.
Example 3
8 a Show that 3x
3
9x
2
+8x +2 cannot be expressed in the form a(x +b)
3
+c.
Example 4
b If 3x
3
9x
2
+9x +2 can be expressed in the form a(x +b)
3
+c, then nd the
values of a, b and c.
9 Show that constants a, b, c and d can be found such that
n
3
= a(n +1)(n +2)(n +3) +b(n +1)(n +2) +c(n +1) +d
10 a Show that no constants can be found such that
n
2
= a(n +1)(n +2) +b(n +2)(n +3)
b Express n
2
in the form a(n +1)(n +2) +b(n +1) +c
11 a Express a(x +b)
2
+c in expanded form.
b Express ax
2
+bx +c in completed square form.
12 Prove that, if ax
3
+bx
2
+cx +d = (x 1)
2
( px +q), then
b = d 2a and c = a 2d
13 If 3x
2
+10x +3 = c(x a)(x b) for all values of x, nd the values of a, b and c.
14 If n is any number, show that n
2
can be expressed in the form
a(n 1)
2
+b(n 2)
2
+c(n 3)
2
, and nd the values of a, b and c.
15 If x
3
+3x
2
9x +c is of the form (x a)
2
(x b), show that c = 5 or c = 27 and
nd a and b for each of these cases.
6.2 Quadratics and rates
Quadratics
The general expression of a quadratic function is y = ax
2
+bx +c, x R, a = 0
The number of solutions to the quadratic equation ax
2
+bx +c = 0 can be determined by the
discriminant, , where = b
2
4ac
i If b
2
4ac > 0, the quadratic equation ax
2
+bx +c = 0 has two real solutions
ii If b
2
4ac = 0, the quadratic equation ax
2
+bx +c = 0 has one real solution
iii If b
2
4ac < 0, the quadratic equation ax
2
+bx +c = 0 has no real solutions
A quadratic equation may be solved by factorising, completing the square or using the general
quadratic formula x =
b

b
2
4ac
2a
. The following example demonstrates each method.
P1: FXS/ABE P2: FXS
9780521740494c06.xml CUAU033-EVANS August 22, 2009 8:51
Chapter 6 Algebra II 155
Example 5
Solve the following quadratic equations for x.
a x
2
+3x = 4 b 3x
2
+4x = 2 c 9x
2
+6x +1 = 0
Solution
a Rearranging the quadratic equation x
2
+3x 4 = 0
Factorising (x +4)(x 1) = 0
Applying the null factor law x +4 = 0 or x 1 = 1
Therefore x = 4 or 1
Note: = 3
2
4 1 (4) = 25, so there are two real solutions.
Using the TI-Nspire
Use Solve( ) from the Algebra menu (b
1) as shown.
Using the Casio ClassPad
Enter and highlight the equation
2x
2
+5x 12 = 0, tap Interactive,
Equation/inequality, solve and ensure
the variable is set to x.
P1: FXS/ABE P2: FXS
9780521740494c06.xml CUAU033-EVANS August 22, 2009 8:51
156 Essential Advanced General Mathematics
b Rearranging the quadratic equation 3x
2
+4x 2 = 0
3

x
2
+
4
3
x

2 = 0
Add and subtract

b
2

2
to complete the square
3

x
2
+
4
3
x +

2
3

2
3

2 = 0
3

x +
2
3

4
9

2 = 0
3

x +
2
3

4
3
2 = 0
3

x +
2
3

2
=
10
3

x +
2
3

2
=
10
9
x +
2
3
=

10
3
x =
2
3

10
3
Therefore x =
2 +

10
3
or
2

10
3
Note: = 4
2
4 3 (2) = 40, so there are two real solutions.
c
Consider 9x
2
+6x +1 = 0
Using the general quadratic formula x =
b

b
2
4ac
2a
x =
6

6
2
4 9 1
2 9
=
6

0
18
Therefore x =
1
3
More simply 9x
2
+6x +1 = (3x +1)
2
Note: = 6
2
4 9 1 = 0, so there is one real solution.
Example 6
Consider the quadratic equation x
2
4x = t . Make x the subject and give the values of t for
which real solution(s) to the equation can be found.
P1: FXS/ABE P2: FXS
9780521740494c06.xml CUAU033-EVANS August 22, 2009 8:51
Chapter 6 Algebra II 157
Solution
x
2
4x = t
Completing the square x
2
4x +4 = t +4
(x 2)
2
= t +4
x 2 =

t +4
x = 2

t +4
For real solutions to exist, t +4 0, i.e., t 4
Using the TI-Nspire
Use Solve( ) from the Algebra menu (b
1) as shown.
Using the Casio ClassPad
Enter and highlight the equation x
2
4x = t ,
tap Interactive, Equation/inequality, solve and
ensure the variable is set to x.
Note: Variable t is found in the menu in the
keyboard screen.
P1: FXS/ABE P2: FXS
9780521740494c06.xml CUAU033-EVANS August 22, 2009 8:51
158 Essential Advanced General Mathematics
Example 7
A rectangle has an area of 288 cm
2
. If the width is decreased by 1 cm, and the length increased
by 1 cm, the area would be decreased by 3 cm
2
. Find the original dimensions of the rectangle.
Solution
Let w and l be the width and length, in centimetres, of the original rectangle.
Therefore wl = 288 . . . 1
The dimensions of the new rectangle are w 1 and l +1, and the area is 285 cm
2
Therefore (w 1)(l +1) = 285 . . . 2
Rearranging 1 to make l the subject, and substituting in 2 gives
(w 1)

288
w
+1

= 285
288
288
w
+w 1 = 285
w
288
w
+2 = 0
w
2
+2w 288 = 0
Using the general quadratic formula x =
b

b
2
4ac
2a
w =
2

2
2
4 1 288
2 1
= 18 or 16
But w > 0, so w = 16. The original dimensions of the rectangle are 16 cm by 18 cm.
Rates
A rate describes how a certain quantity changes with respect to the change in another quantity
(often time). An example of a rate is speed. A speed of 60 km/h gives us a measure of how
fast an object is travelling. A further example is ow, where a rate of 20 L/min is going to ll
an empty swimming pool faster than, say, a rate of 6 L/min.
Many problems are solved using rates, which can be expressed as fractions. For example, a
speed of 60 km/h can be expressed in fraction form as
60(km)
1(h)
.
It is often rst necessary to add two or more fractions with different denominators, as shown
in the following examples.
P1: FXS/ABE P2: FXS
9780521740494c06.xml CUAU033-EVANS August 22, 2009 8:51
Chapter 6 Algebra II 159
Example 8
a Express
6
x
+
6
x +8
as a single fraction.
b Solve the equation
6
x
+
6
x +8
= 2 for x.
Solution
a
6
x
+
6
x +8
=
6(x +8)
x(x +8)
+
6x
x(x +8)
=
6x +48 +6x
x(x +8)
=
12(x +4)
x(x +8)
b Since
6
x
+
6
x +8
=
12(x +4)
x(x +8)
then
12(x +4)
x(x +8)
= 2
12(x +4) = 2x(x +8)
6(x +4) = x(x +8)
6x +24 = x
2
+8x
0 = x
2
+2x 24
0 = (x +6)(x 4)
x +6 = 0 or x 4 = 0
x = 6 or x = 4
Example 9
A tank is lled by two pipes. The smaller pipe alone will take 24 minutes longer than the larger
pipe alone, and 32 minutes longer than when both pipes are used. How long will each pipe take
to ll the tank alone? How long will it take for both pipes used together to ll the tank?
Solution
Let C cubic units be the capacity of the tank, and x minutes the time it takes for the
larger pipe alone to ll the tank. Therefore the average rate of ow for the larger pipe
is
C
x
cubic units per minute.
Since the smaller pipe alone takes (x +24) minutes to ll the tank, the average rate
of ow for the smaller pipe is
C
x +24
cubic units per minute.
P1: FXS/ABE P2: FXS
9780521740494c06.xml CUAU033-EVANS August 22, 2009 8:51
160 Essential Advanced General Mathematics
The average rate of ow when both pipes are used together is the sum of these two
rates,
C
x
+
C
x +24
cubic units per minute.
Expressed as a single fraction,
C
x
+
C
x +24
=
C(x +24) +Cx
x(x +24)
=
2C(x +12)
x(x +24)
The time taken to ll the tank using both pipes is
C
2C(x +12)
x(x +24)
= C
x(x +24)
2C(x +12)
=
x(x +24)
2(x +12)
The time taken for the smaller pipe to ll the tank can be also be expressed as
x(x +24)
2(x +12)
+32
i.e.
x(x +24)
2(x +12)
+32 = x +24
x(x +24)
2(x +12)
= x 8
x(x +24) = 2(x +12)(x 8)
x
2
+24x = 2x
2
+8x 192
0 = x
2
16x 192
0 = (x 24)(x +8)
x 24 = 0 or x +8 = 0
x = 24 or x = 8 (but x > 0)
It takes 24 minutes for the larger pipe alone to ll the tank, and 48 minutes for the
smaller pipe alone to ll the tank, and 16 minutes to ll the tank using both pipes.
Exercise 6B
1 Solve the following quadratic equations for x.
Example 5
a x
2
+2x = 1
b x
2
6x +9 = 0 c 5x
2
10x = 1
d 2x
2
+4x = 1 e 2x
2
+4x = 7 f 6x
2
+13x +1 = 0
2 Make x the subject in each of the following and give the values of t for which real
solution(s) to the equation can be found.
Example 6
a 2x
2
4t = x
b 4x
2
+4x 4 = t 2
c 5x
2
+4x +10 = t d t x
2
+4t x +10 = t
P1: FXS/ABE P2: FXS
9780521740494c06.xml CUAU033-EVANS August 22, 2009 8:51
Chapter 6 Algebra II 161
3 a Solve the quadratic equation x
2
+3x 9 = 0 for x, giving exact solutions.
b i Solve the quadratic equation x
2
+ px 16 = 0 in terms of p.
ii Find the value(s) of p, where 0 p 10 and p is a non-negative integer, for which
the quadratic equation in i has a non-negative integer solution.
x m
x m
8 m
6 m
10 m
4 A pole 10 m long leans against a wall. The bottom
of the pole is 6 m from the wall. If the bottom of the
pole is pulled away x m so that the top slides down
by the same amount, nd x.
5
Example 8
a Express
6
x

6
x +3
as a single fraction.
b Solve the equation
6
x

6
x +3
= 1 for x.
6 The sum of the reciprocals of two consecutive odd numbers is
36
323
. Form a quadratic
equation and hence determine the two numbers.
Example 9
7 A car travels from town A to town B, a distance of 600 km, in x hours. A plane, travelling
220 km/h faster than the car, takes ve and a half hours less to cover the same distance.
a Express, in terms of x, the average speed of the car and the average speed of the plane.
b Find the actual average speed of each of them.
8 A car covers a distance of 200 km at a speed of x km/h. A train covers the same distance at
a speed of (x +5) km/h. If the time taken by the car is 2 hours more than that taken by the
train, nd x.
9 A man travels 108 km, and nds that he could have made the journey in 4
1
2
hours less had
he travelled at an average speed 2 km/h faster. What was the mans average speed when he
made the trip?
10 A bus is due to reach its destination 75 km away at a certain time. The bus usually travels
with an average speed of x km/h. Its start is delayed by 18 minutes but, by increasing its
average speed by 12.5 km/h, the driver arrives on time.
a Find x. b How long did the journey actually take?
11 Ten minutes after the departure of an express train, a slow train starts, travelling at an
average speed of 20 km/h less. The slow train reaches a station 250 km away 3.5 hours
after the arrival of the express. Find the average speed of each of the trains.
12 When the average speed of a car is increased by 10 km/h the time taken for the car to
make a journey of 105 km is reduced by 15 minutes. Find the original average speed.
P1: FXS/ABE P2: FXS
9780521740494c06.xml CUAU033-EVANS August 22, 2009 8:51
162 Essential Advanced General Mathematics
13 A tank can be lled with water by two pipes running together in 11
1
9
minutes. If the larger
pipe alone takes 5 minutes less to ll the tank than the smaller pipe, nd the time that each
pipe will take to ll the tank.
14 At rst two different pipes running together will ll a tank in
20
3
minutes. The rate that
water runs through each of the pipes is then adjusted. If one pipe, running alone, takes
1 minute less to ll the tank at its new rate, and the other pipe, running alone, takes
2 minutes more to ll the tank at its new rate, then the two running together will ll the
tank in 7 minutes. Find in what time the tank will be lled by each pipe running alone at
the new rates.
15 The journey between two towns by one route consists of 233 km by rail followed by
126 km by sea. By a second route the journey consists of 405 km by rail followed by
39 km by sea. If the time taken for the rst route is 50 minutes longer than for the second
route, and travelling by rail is 25 km/h faster than travelling by sea, nd the average speed
by rail and the average speed by sea.
16 A freighter sailing due north at 12 km/h sights a cruiser straight ahead at an unknown
distance and speeding due east at unknown speed. After 15 minutes the vessels are 10 km
apart and then, 15 minutes later, they are 13 km apart. (Assume both travel at constant
speeds.) How far apart are the vessels when the cruiser is due east of the freighter?
17 A cask A, of capacity 20 litres, is lled with wine. A certain quantity of wine from A is
poured into a cask B which also has a capacity of 20 litres. Cask B is then lled with
water. After this cask A is lled with some of the mixture from cask B. A further
20
3
litres
of the mixture now in A is poured back into B, and the two casks now have the same
amount of wine. How much wine was rst taken out of cask A?
6.3 Partial fractions
A rational function is the quotient of two polynomials. If g(x) and h(x) are polynomials,
f (x) =
g(x)
h(x)
is a rational function,
e.g., f (x) =
x
2
+1
x
2
1
If the degree of g(x) < the degree of h(x), then
g(x)
h(x)
is a proper fraction.
If the degree of g(x) the degree of h(x), then
g(x)
h(x)
is an improper fraction.
It is convention to consider rational functions for their maximal domain. For example,
x
2
+1
x
2
1
is only considered for R\{1, 1}.
P1: FXS/ABE P2: FXS
9780521740494c06.xml CUAU033-EVANS August 22, 2009 8:51
Chapter 6 Algebra II 163
A rational algebraic function may be expressed as a sum of separate functions by resolving
it into what are called partial fractions. This can help in the sketching of graphs of these
functions or performing other mathematical procedures such as integration.
Proper fractions
For proper fractions, the technique used for obtaining partial fractions depends on the type of
factors in the denominator of the original algebraic fraction. Only examples where the
denominators have factors that are either 1st degree (linear) or 2nd degree (quadratic) will be
considered.
For every linear factor (ax +b) in the denominator, there will be a partial fraction of the
form
A
ax +b
For every repeated linear factor (cx +d)
2
in the denominator, there will be partial
fractions of the form
B
cx +d
and
C
(cx +d)
2
For every irreducible quadratic factor (ax
2
+bx +c) in the denominator, there will be a
partial fraction of the form
Dx + E
ax
2
+bx +c
To resolve an algebraic fraction into its partial fractions, rstly write a statement of identity
between the original fraction and a sum of the appropriate number of partial fractions. Then
express the sum of the partial fractions as a single fraction and note that the numerators of both
sides are equivalent. By choosing an appropriate value(s) for x and/or equating coefcients, the
values of the introduced constants A, B, C, etc. can be found.
Example 10
Resolve
3x +5
(x 1)(x +3)
into partial fractions.
Solution
Since the denominator has two linear factors, there will be two partial fractions of the
form
A
x 1
and
B
x +3
Let
3x +5
(x 1)(x +3)
=
A
x 1
+
B
x +3
(x R\{1, 3})
Express the right hand side as a single fraction.
3x +5
(x 1)(x +3)
=
A(x +3) + B(x 1)
(x 1)(x +3)

3x +5
(x 1)(x +3)
=
(A + B)x +3A B
(x 1)(x +3)
3x +5 = (A + B)x +3A B
P1: FXS/ABE P2: FXS
9780521740494c06.xml CUAU033-EVANS August 22, 2009 8:51
164 Essential Advanced General Mathematics
Equate the coefcients,
i.e., A + B = 3
3A B = 5
Solving these equations simultaneously,
4A = 8
i.e., A = 2
and B = 1
Therefore
3x +5
(x 1)(x +3)
=
2
x 1
+
1
x +3
Using the TI-Nspire
Use Expand( ) from the Algebra menu (b
) as shown.
Notice that you can access the fraction
template by typing / .
Using the Casio ClassPad
Tap Interactive, Transformation, expand and the Partial Fraction button. Enter the
expression and the variable.
P1: FXS/ABE P2: FXS
9780521740494c06.xml CUAU033-EVANS August 22, 2009 8:51
Chapter 6 Algebra II 165
Example 11
Resolve
2x +10
(x +1)(x 1)
2
into partial fractions.
Solution
Since there is a repeated linear factor and a single linear factor, there are three partial
fractions such that
2x +10
(x +1)(x 1)
2
=
A
x +1
+
B
x 1
+
C
(x 1)
2

2x +10
(x +1)(x 1)
2
=
A(x 1)
2
+ B(x +1)(x 1) +C(x +1)
(x +1)(x 1)
2
Therefore 2x +10 = A(x 1)
2
+ B(x +1)(x 1) +C(x +1)
To nd A, B and C, a combination of methods will be used.
First let x = 1
2(1) +10 = C(1 +1)
12 = 2C
C = 6
Let x = 1
2(1) +10 = A(1 1)
2
8 = 4A
A = 2
Substitute these values for A and C.
2x +10 = 2(x 1)
2
+ B(x +1)(x 1) +6(x +1) . . . 1
= 2(x
2
2x +1) + B(x
2
1) +6(x +1)
= (2 + B)x
2
+2x +8 B
Now by equating coefcients,
2 + B = 0
and 8 B = 10
B = 2
So
2x +10
(x +1)(x 1)
2
=
2
x +1

2
x 1
+
6
(x 1)
2
The value of B may also be found by substituting x = 0 into equation 1 .
In the exercises for this section, the following result is established: that it is impossible
to nd A and C such that
2x +10
(x +1)(x 1)
2
=
A
(x +1)
+
C
(x 1)
2
P1: FXS/ABE P2: FXS
9780521740494c06.xml CUAU033-EVANS August 22, 2009 8:51
166 Essential Advanced General Mathematics
Example 12
Resolve
x
2
+6x +5
(x 2)(x
2
+ x +1)
into partial fractions.
Solution
The denominator contains a quadratic factor, which cannot be reduced to linear
factors (an irreducible quadratic), as well as a single linear factor.

x
2
+6x +5
(x 2)(x
2
+ x +1)
=
A
x 2
+
Bx +C
x
2
+ x +1

x
2
+6x +5
(x 2)(x
2
+ x +1)
=
A(x
2
+ x +1) +(Bx +C)(x 2)
(x 2)(x
2
+ x +1)
x
2
+6x +5 = A(x
2
+ x +1) +(Bx +C)(x 2)
Let x = 2
2
2
+6(2) +5 = A(4 +2 +1)
21 = 7A
A = 3
Also x
2
+6x +5 = A(x
2
+ x +1) +(Bx +C)(x 2) . . . 1
= A(x
2
+ x +1) + Bx
2
2Bx +Cx 2C
= (A + B)x
2
+(A 2B +C)x + A 2C
Since A = 3, then
x
2
+6x +5 = (3 + B)x
2
+(3 2B +C)x +3 2C
Equating coefcients
3 + B = 1
B = 2
and 3 2C = 5
C = 1
[checking: 3 2B +C = 3 2(2) +(1) = 6]
Therefore
x
2
+6x +5
(x 2)(x
2
+ x +1)
=
3
x 2
+
2x 1
x
2
+ x +1
or
x
2
+6x +5
(x 2)(x
2
+ x +1)
=
3
x 2

2x +1
x
2
+ x +1
Note: The values of B and C can also be found by substituting x = 0 and x = 1
respectively in equation 1 .
Improper fractions
Improper algebraic fractions can be expressed as a sum of partial fractions by rst dividing the
denominator into the numerator to produce a quotient and a proper fraction. The resulting
proper fraction can then be resolved into its partial fractions using the techniques outlined
above.
P1: FXS/ABE P2: FXS
9780521740494c06.xml CUAU033-EVANS August 22, 2009 8:51
Chapter 6 Algebra II 167
Example 13
Express
x
5
+2
x
2
1
in partial fractions.
Solution
Dividing through
x
3
+ x
x
2
1

x
5
+2
x
5
x
3
x
3
+2
x
3
x
x +2

x
5
+2
x
2
1
= x
3
+ x +
x +2
x
2
1
Expressing
x +2
x
2
1
=
x +2
(x 1)(x +1)
as partial fractions,
x
5
+2
x
2
1
= x
3
+ x
1
2 (x +1)
+
3
2 (x 1)
Using the TI-Nspire
Use Expand( ) from the Algebra menu (b
) as shown.
Notice that you can access the fraction
template by typing / .
Using the Casio ClassPad
Tap Interactive, Transformation, expand and the
Partial Fraction button. Enter the expression and
the variable.
P1: FXS/ABE P2: FXS
9780521740494c06.xml CUAU033-EVANS August 22, 2009 8:51
168 Essential Advanced General Mathematics
Partial fractions are summarised through examples, as follows.
Linear factors
3x 4
(2x 3)(x +5)
=
A
2x 3
+
B
x +5
Repeated linear factors
3x 4
(2x 3)(x +5)
2
=
A
2x 3
+
B
x +5
+
C
(x +5)
2
Irreducible quadratic factors
3x 4
(2x 3)(x
2
+5)
=
A
2x 3
+
Bx +C
x
2
+5
If f (x) =
g(x)
h(x)
and the degree of g(x) is greater than or equal to the degree of h(x) then
division must be performed rst.
Exercise 6C
1 Resolve the following rational expressions into partial fractions.
Example 10
a
5x +1
(x 1)(x +2)
b
1
(x +1)(2x +1)
c
3x 2
x
2
4
d
4x +7
x
2
+ x 6
e
7 x
(x 4)(x +1)
2 Resolve the following rational expressions into partial fractions.
Example 11
a
2x +3
(x 3)
2
b
9
(1 +2x)(1 x)
2
c
2x 2
(x +1)(x 2)
2
3 Resolve the following rational expressions into partial fractions.
Example 12
a
3x +1
(x +1)(x
2
+ x +1)
b
3x
2
+2x +5
(x
2
+2)(x +1)
c
x
2
+2x 13
2x
3
+6x
2
+2x +6
4 Resolve
3x
2
4x 2
(x 1)(x 2)
into partial fractions.
Example 13
5 Find values of A and C such that
2x +10
(x +1)(x 1)
2
=
A
x +1
+
C
(x 1)
2
P1: FXS/ABE P2: FXS
9780521740494c06.xml CUAU033-EVANS August 22, 2009 8:51
Chapter 6 Algebra II 169
6 Express each of the following in partial fractions.
a
1
(x 1)(x +1)
b
x
(x 2)(x +3)
c
3x +1
(x 2)(x +5)
d
1
(2x 1)(x +2)
e
3x +5
(3x 2)(2x +1)
f
2
x
2
x
g
3x +1
x
3
+ x
h
3x
2
+8
x(x
2
+4)
i
1
x
2
4x
j
x +3
x
2
4x
k
x
3
x
2
1
x
2
x
l
x
3
x
2
6
2x x
2
m
x
2
x
(x +1)(x
2
+2)
n
x
2
+2
x
3
3x 2
o
2x
2
+ x +8
x(x
2
+4)
p
1 2x
2x
2
+7x +6
q
3x
2
6x +2
(x 1)
2
(x +2)
r
4
(x 1)
2
(2x +1)
s
x
3
2x
2
3x +9
x
2
4
t
x
3
+3
(x +1)(x 1)
u
2x 1
(x +1)(3x +2)
6.4 Simultaneous equations
In this section, methods for nding the coordinates of the points of intersection of a linear
graph with different non-linear graphs are discussed. The non-linear graphs are parabolas,
circles and rectangular hyperbolae. The associated relations have been discussed in Essential
Mathematical Methods 1 and 2 CAS.
Example 14
Find the coordinates of the points of intersection of the parabola with equation
y = x
2
2x 2 with the straight line with equation y = x +4.
Solution
Consider x +4 = x
2
2x 2
Then 0 = x
2
3x 6
x =
3

9 4 6 1
2
=
3

33
2
The points of intersection have
coordinates A

33
2
,
11

33
2

and
B

3 +

33
2
,
11 +

33
2

y = x + 4
y = x
2
2x 2
x
A
4
4
2
0
y
B
P1: FXS/ABE P2: FXS
9780521740494c06.xml CUAU033-EVANS August 22, 2009 8:51
170 Essential Advanced General Mathematics
Using the TI-Nspire
Use Solve( ) from the Algebra menu (b
1) as shown.
The and can either be typed or found in
the catalog ( 1 ).
Use the NavPad to move the cursor up to
the solution and see all the solutions.
The simultaneous equations template
can also be used.
Using the Casio ClassPad
The CAS calculator will give exact values for the
points of intersection.
Turn on the screen keyboard, tap 2D and select
the simultaneous equations symbol .
Enter the equations in the spaces provided and
the variables x, y as shown.
Example 15
Find the coordinates of the points of intersection of the circle with equation
(x 4)
2
+ y
2
= 16 and the line with equation x y = 0.
x
(4, 0)
(4, 4)
0
y
Solution
Rearrange x y = 0 to make y the subject.
Substitute y = x into the equation of the circle.
i.e., (x 4)
2
+ x
2
= 16
x
2
8x +16 + x
2
= 16
i.e., 2x
2
8x = 0
2x(x 4) = 0
x = 0 or x = 4
The points of intersection are (0, 0) and (4, 4)
Example 16
Find the point of contact of the line with equation
1
9
x + y =
2
3
and the curve with equation
xy = 1.
P1: FXS/ABE P2: FXS
9780521740494c06.xml CUAU033-EVANS August 22, 2009 8:51
Chapter 6 Algebra II 171
x
y =
x
+
9
1
y =
x
1
3
2
0
y
Solution
Rewrite the equations as y =
1
9
x +
2
3
and y =
1
x
Consider
1
9
x +
2
3
=
1
x
x +6 =
9
x
and x
2
+6x = 9
Therefore x
2
6x +9 = 0
and (x 3)
2
= 0, i.e. x = 3
The point of intersection is

3,
1
3

Using the TI-Nspire


Use Solve( ) from the Algebra menu (b
1) as shown.
The and can either be typed or found in
the catalog ( 1 ).
The multiplication sign between x and y
is required because the calculator will
consider xy a variable.
Using the Casio ClassPad
Turn on the screen keyboard, tap 2D and
select the simultaneous equations symbol
.
Enter the equations in the spaces
provided and the variables x, y as shown.
P1: FXS/ABE P2: FXS
9780521740494c06.xml CUAU033-EVANS August 22, 2009 8:51
172 Essential Advanced General Mathematics
Exercise 6D
1 Find the coordinates of the points of intersection for each of the following.
Example 14
a y = x
2
y = x
b y 2x
2
= 0
y x = 0
c y = x
2
x
y = 2x +1
2 Find the coordinates of the points of intersection for each of the following.
Example 15
a x
2
+ y
2
= 178
x + y = 16
b x
2
+ y
2
= 125
x + y = 15
c x
2
+ y
2
= 185
x y = 3
d x
2
+ y
2
= 97
x + y = 13
e x
2
+ y
2
= 106
x y = 4
3 Find the coordinates of the points of intersection for each of the following.
Example 16
a x + y = 28
xy = 187
b x + y = 51
xy = 518
c x y = 5
xy = 126
4 Find the coordinates of the points of intersection of the straight line with equation y = 2x
and the circle with equation (x 5)
2
+ y
2
= 25.
5 Find the coordinates of the points of intersection of the curves with equation
y =
1
x 2
+3 and y = x.
6 Find the coordinates of the points A and B for which the line with equation x 3y = 0
meets the circle with equation x
2
+ y
2
10x 5y +25 = 0.
7 Find the coordinates of the points of intersection of the line with equation
y
4

x
5
= 1 and
the circle with equation x
2
+4x + y
2
= 12.
8 Find the coordinates of the points of intersection of the curve with equation
y =
1
x +2
3 and the line with equation y = x.
9 Find the coordinates of the point where the line 4y = 9x +4 touches the parabola with
equation y
2
= 9x.
10 Find the coordinates of the point where the line with equation y = 2x +3

5 touches the
circle x
2
+ y
2
= 9.
11 Find the coordinates of the point where the straight line with equation y =
1
4
x +1
touches the curve with equation y =
1
x
.
12 Find the coordinates of the points of intersection of the curve with equation y =
2
x 2
and the line y = x 1.
P1: FXS/ABE P2: FXS
9780521740494c06.xml CUAU033-EVANS August 22, 2009 8:51
R
e
v
i
e
w
Chapter 6 Algebra II 173
Chapter summary
The general expression of a quadratic function is y = ax
2
+bx +c, x R
A quadratic equation may be solved by
r
Factorising
e.g., 2x
2
+5x 12 = 0
(2x 3)(x +4) = 0 x =
3
2
or 4
r
Completing the square
e.g., x
2
+2x 4 = 0
Add and subtract

b
2

2
to complete the square.
x
2
+2x +1 1 4 = 0
(x +1)
2
5 = 0
(x +1)
2
= 5
x +1 =

5 x = 1 +

5
r
Using the general quadratic formula x =
b

b
2
4ac
2a
e.g., 3x
2
12x 7 = 0
x =
(12)

(12)
2
4(3)(7)
2(3)
=
6

15
3
The number of solutions to the quadratic equation ax
2
+bx +c = 0 can be determined by
the discriminant, , where = b
2
4ac
r
If b
2
4ac > 0, the quadratic equation ax
2
+bx +c = 0 has two real solutions
r
If b
2
4ac = 0, the quadratic equation ax
2
+bx +c = 0 has one real solution
r
If b
2
4ac < 0, the quadratic equation ax
2
+bx +c = 0 has no real solutions
A function of the form f (x) =
g(x)
h(x)
, where g(x) and h(x) are polynomials in x, is called a
rational algebraic function, e.g. f (x) =
x +1
x
2
1
Some rational algebraic functions may be expressed as a sum of partial fractions. For every
linear factor (ax +b) in the denominator there will be a partial fraction of the form
A
ax +b
For every repeated linear factor (cx +d)
2
in the denominator there will be two partial
fractions of the form
B
(cx +d)
and
C
(cx +d)
2
P1: FXS/ABE P2: FXS
9780521740494c06.xml CUAU033-EVANS August 22, 2009 8:51
R
e
v
i
e
w
174 Essential Advanced General Mathematics
For every irreducible quadratic factor (ex
2
+ f x + g) in the denominator there will be a
partial fraction of the form
Dx + E
(ex
2
+ f x + g)
e.g.
2x +10
(x +1) (x 1)
2
may be expressed as partial fractions in the form
A
(x +1)
+
B
(x 1)
+
C
(x 1)
2
where A = 2, B = 2 and C = 6
Multiple-choice questions
1 If x
2
is written in the form (x +1)
2
+b(x +1) +c, then the values of b and c are
A b = 0, c = 0 B b = 2, c = 0 C b = 2, c = 1
D b = 1, c = 2 E b = 1, c = 2
2 If x
3
= a(x +2)
3
+b(x +2)
2
+c(x +2) +d, then the values of a, b, c and d are
A a = 0, b = 8, c = 10, d = 6 B a = 0, b = 6, c = 10, d = 8
C a = 1, b = 8, c = 10, d = 6 D a = 1, b = 6, c = 12, d = 8
E a = 1, b = 8, c = 12, d = 6
3 The quadratic equation 3x
2
6x +3 = 0 has
A two real solutions, x = 1 B one real solution, x = 1
C no real solutions D one real solution, x = 1
E two real solutions, x = 1 and x = 2
4 The quadratic equation whose solutions are 4 and 6 is
A (x +4)(x 6) = 0 B x
2
2x 24 = 0 C 2x
2
+4x = 48
D x
2
+2x 24 = 0 E x
2
+2x +24 = 0
5
3
x +4

5
x 2
is equal to
A
2
(x +4)(x 2)
B
2(x +1)
(x +4)(x 2)
C
2(x 7)
(x +4)(x 2)
D
2(4x +13)
(x +4)(x 2)
E
2(x +13)
(x +4)(x 2)
6
4
(x +3)
2
+
2x
x +1
is equal to
A
8x
(x +3)
2
(x +1)
B
2(3x
2
+ x +18)
(x +3)
2
(x +1)
C
3x
2
+13x +18
(x +3)
2
(x +1)
D
2(3x
2
+13x +18)
(x +3)
2
(x +1)
E
2(x
3
+6x
2
+11x +2)
(x +3)
2
(x +1)
7 If
7x
2
+13
(x 1)(x
2
+ x +2)
is expressed in the form
a
x 1
+
bx +c
x
2
+ x +2
, then the values of a,
b and c are
A a = 5, b = 0, c = 13 B a = 5, b = 0, c = 10 C a = 5, b = 2, c = 3
D a = 7, b = 2, c = 3 E a = 7, b = 3, c = 13
P1: FXS/ABE P2: FXS
9780521740494c06.xml CUAU033-EVANS August 22, 2009 8:51
R
e
v
i
e
w
Chapter 6 Algebra II 175
8
4x 3
(x 3)
2
is equal to
A
3
x 3
+
1
x 3
B
4x
x 3

3
x 3
C
9
x 3
+
4
(x 3)
2
D
4
x 3
+
9
(x 3)
2
E
4
x 3

15
(x 3)
2
9
8x +7
2x
2
+5x +2
is equal to
A
2
2x +1

3
x +2
B
2
2x +1
+
3
x +2
C
4
2x +2

1
x +1
D
4
2x +2
+
1
x +1
E
4
2x +2

1
x +1
10
3x
2
+2x 1
(x
2
+1)(x +1)
is equal to
A
2
x
2
+1
+
3
x +1
B
2
x
2
+1

3
x +1
C
5
x
2
+1
+
2
x +1
D
3
x
2
+1

2
x +1
E
3
x
2
+1
+
2
x +1
Short-answer questions (technology-free)
1 If (3a +b)x
2
+(a 2b)x +b +2c = 11x
2
x +4, nd the values of a, b and c.
2 Express x
3
in the form (x 1)
3
+a(x 1)
2
+b(x 1) +c.
3 Prove that, if ax
3
+bx
2
+cx +d = (x +1)
2
( px +q), then b = 2a +d and c = a +2d.
4 Prove that, if ax
3
+bx
2
+cx +d = (x 2)
2
( px +q), then b = 4a +
1
4
d and
c = 4a d.
5 Solve the following quadratic equations for x.
a x
2
+ x = 12 b x
2
2 = x c x
2
+3x +11 = 1
d 2x
2
4x +1 = 0 e 3x
2
2x +5 = t f t x
2
+4 = t x
6 Solve the equation
2
x 1

3
x +2
=
1
2
for x.
7 Express the following as partial fractions.
a
3x +4
(x 3)(x +2)
b
7x +2
x
2
4
c
7 x
x
2
+2x 15
d
3x 9
x
2
4x 5
e
3x 4
(x +3)(x +2)
2
f
6x
2
5x 16
(x 1)
2
(x +4)
g
x
2
6x 4
(x
2
+2)(x +1)
h
x +4
(x 1)(x
2
+ x +1)
i
4x +5
(x +4)(x 3)
j
2x +8
(x +4)(x 3)
8 Express each of the following in partial fractions.
a
14(x 2)
(x 3)(x
2
+ x +2)
b
1
(x +1)(x
2
x +2)
c
3x
3
x
2
5x +4
P1: FXS/ABE P2: FXS
9780521740494c06.xml CUAU033-EVANS August 22, 2009 8:51
R
e
v
i
e
w
176 Essential Advanced General Mathematics
9 Find the coordinates of the points of intersection for each of the following.
a y = x
2
y = x
b x
2
+ y
2
= 16
x + y = 4
c x + y = 5
xy = 4
10 Find the coordinates of the points of intersection of the line with equation 3y x = 1 and
the circle with equation x
2
+2x + y
2
= 9.
Extended-response questions
1 A train completes a journey of 240 km at a constant speed.
a If it had travelled 4 km/h slower, it would have taken two hours more for the journey.
Find the actual speed of the train.
b If it had travelled a km/h slower, and still taken two hours more for the journey of
240 km, what would have been the actual speed? (Answer in terms of a.) Discuss the
practical possible values of a and also the possible values for the speed of the train.
c If the train had travelled a km/h slower, and taken a hours more for the journey of
240 km, and if a is an integer and the speed is an integer, nd the possible values for a
and the speed of the train.
2 An upholsterer purchased some fabric for $a. If he had bought the fabric from another
supplier who charged $b per metre more he would have received b metres less for the same
amount of money.
a How many metres did he purchase, in terms of a and b?
b If a and b, and the number of metres purchased, are natural numbers, nd the possible
values of a given a < 100.
3 Two trains are travelling at uniform speeds. The slower train takes a hours longer to cover
b km. It travels 1 km less than the faster one in c hours.
a What is the speed of the faster train, in terms of a, b and c?
b If a, b and c, and the speeds of the trains, are rational numbers, nd ve sets of values for
a, b and c. Choose and discuss two sensible sets of values.
4 A tank can be lled using two pipes. The smaller pipe alone will take a minutes longer than
the larger pipe alone to ll the tank. Also the smaller pipe will take b minutes longer to ll
the tank than when both pipes are used.
a Find, in terms of a and b, how long it will take each of the pipes to ll the tank.
b If a = 24 and b = 32, nd how long it takes for each of the pipes to ll the tank.
c If a and b are consecutive positive integers, nd ve pairs of values of a and b such that
b
2
ab is a perfect square. Interpret these results in the context of this tank problem.
P1: FXS/ABE P2: FXS
9780521740494c07.xml CUAU033-EVANS October 16, 2008 21:15
C H A P T E R
Revision
7
Revision of chapters 16
7.1 Multiple-choice questions
1 If P
2
= 4I, then P
1
equals
A
1
4
P B
1
2
P C
1
2
I D 2P E 4P
2 If R = [5 3 1] and S =

0
1
2

, then RS is
A not dened B [1] C

0 0 0
5 3 1
10 6 2

D [0 3 2] E

0
3
2

3 If A =

9 8
11 5

, then det (A) equals


A 43 B
1
43
C
1
133
D 17 E 133
4 If A =

1
2
5

and B = [2 6 4], then BA has dimensions


A 1 1 B 3 1 C 1 3 D 3 3 E 3 2
5 Given that A =

5 2
2 1

, B =

2 1
6 7

and C =

5 4
8 9

, then if AX +B = C,
X equals
A
1
20

2 19
2 6

1 1
4 0

2 19
2 6

3 10
4 10

E
1
20

1 3
4 5

177
P1: FXS/ABE P2: FXS
9780521740494c07.xml CUAU033-EVANS October 16, 2008 21:15
R
e
v
i
s
i
o
n
178 Essential Advanced General Mathematics
6 Let P =

2 1
3 2

, Q =

4 2
6 5

, R =

2 1
3 2

and X = PQR. The number of zero


elements in X is
A 0 B 1 C 2 D 3 E 4
7 If X =

3 5
1 2

, then X
1
is
A

2 5
1 3

2 5
1 3

1
3
1
5
1
1
2

3 1
5 2

3 1
5 2

8 The determinant of the matrix

4 6
2 4

is
A 16 B 4 C 16
D
1
4
E 4
9 If S =

5 7
2 2

, then S
1
is
A

5 7
2 2

5 7
2 5

C
1
4

2 7
2 5

D
1
4

2 7
2 5

E
1
4

2 7
2 5

10 In algebraic form, ve is seven less than three times one more than x can be written as
A 5 = 7 3(x +1) B 3x +1 = 5 7 C (x +1) 7 = 5
D 5 = 7 3x +1 E 5 = 3x 4
11
3
x 3

2
x +3
is equal to
A 1 B
x +15
x
2
9
C
15
x 9
D
x 3
x
2
9
E
1
6
12 p varies directly as x and inversely as the square of y. If x is decreased by 30% and y is
decreased by 20%, the percentage change in p is best approximated by
A increase by 10% B decrease by 10% C increase by 9.4%
D decrease by 9.4% E no change
13 The sum of the odd numbers from 1 to n inclusive is 100. The value of n is
A 13 B 15 C 17 D 19 E 21
14 If the sum of the rst n terms of a geometric sequence is 2
n+1
2 , the nth term of the
geometric sequence is
A 2
n1
B 2
n
C 2
n
1 D 2
n1
+1 E 2
n
+1
P1: FXS/ABE P2: FXS
9780521740494c07.xml CUAU033-EVANS October 16, 2008 21:15
R
e
v
i
s
i
o
n
Chapter 7 Revision of chapters 16 179
15 If m n and m = 9 when n = 4, then k, the constant of variation, equals
A
9
4
B 13 C 36 D
4
9
E 5
16 If A = {1, 2, 3, 4}, B = {2, 3, 4, 5, 6} and C = {3, 4, 5, 6, 7} then A (B C) is equal to
A {1, 2, 3, 4, 5, 6, 7} B {1, 2, 3, 4, 5, 6} C {2, 3, 4}
D {3, 4} E {2, 3, 4, 5, 6, 7}
17 The price of painting the outside of a cylindrical tank (the bottom and top are not painted)
of radius r and height h varies directly as the total surface area. If r = 5 and h = 4, the
price is $60. The price when r = 4 and h = 6 is
A $45 B $57.60 C $53.50 D $62.80 E $72
18 If x y and x = 8 when y = 2, the value of x when y = 7 is
A 20 B 13 C 11 D 28 E 1.75
19 The recurring decimal 0.

2 is equal to
A
72
101
B
72
100
C
72
99
D
72
90
E
73
90
20 If x varies directly as y
2
and inversely as z, the percentage increase of x when y is
increased by 25% and z is decreased by 20% is best approximated by
A 5% B 50% C 85% D 95% E 100%
21
4
x 1

3
1 x
+
x
x 1
is equal to
A 1 B 1 C
7x
x 1
D
1
1 x
E none of these
22
x +2
3

5
6
is equal to
A
x 3
6
B
2x +4
6
C
2x 1
6
D
2x 5
6
E
x 3
3
23 If a = 1 +
1
1 +b
, then b equals
A 1
1
a 1
B 1 +
1
a 1
C
1
a 1
1 D
1
a +1
+1 E
1
a +1
1
24 When the repeating decimal 0.

6 is written in simplest fractional form, then the sum of


the numerator and denominator is
A 15 B 45 C 114 D 135 E 150
25 If
2x y
2x + y
=
3
4
, then
x
y
equals
A
2
7
B
7
2
C
3
4
D
4
3
E Not possible unless the values of x and y are known
26 The sum to innity of the series
1
2

1
4
+
1
8

1
16
+ is
A 2 B 1 C
1
2
D
1
3
E
2
3
P1: FXS/ABE P2: FXS
9780521740494c07.xml CUAU033-EVANS October 16, 2008 21:15
R
e
v
i
s
i
o
n
180 Essential Advanced General Mathematics
27 If x varies directly as y and inversely as the square of z and x = 10 when y = 4 and
z = 14, then when y = 16 and z = 7, x equals
A 180 B 160 C 154 D 140 E 120
28 If
3
3 + y
= 4, then y equals
A
1
4
B
9
4
C
9
4
D 0 E
4
9
29 The coordinates of the point where the lines with equations 3x + y = 7 and
2x +5y = 4 intersect are
A (3, 16) B (3, 2) C (3, 2) D (2, 3) E no solution
30 If
m +2
4

2 m
4
=
1
2
then m is equal to
A 1 B 1 C
1
2
D 0 E
1
2
31 46 200 can be written as
A 2 3 5 7 11 B 2
2
3
2
5
2
7 11
C 2 3
2
5 7
2
11 D 2
3
3 5
2
7 11
E 2
2
3 5
3
7 11
32 Three numbers, y, y 1 and 2y 1, are consecutive numbers of an arithmetic sequence.
y equals
A 1 B 1 C 0 D 2 E 2
33 If the integers n +1, n 1, n 6, n 5, n +4 are arranged in increasing order of
magnitude then the middle number is
A n +1 B n 1 C n 6 D n 5 E n +4
34 If x
1
y
, and y is multiplied by 5, then x will be
A decreased by 5 B increased by 5 C multiplied by 5
D divided by 5 E none of these
35 An arithmetic sequence has 3 as its rst term and 9 as its fourth term. The eleventh term is
A 23 B 11 C 63 D 21 E none of these
36 The expression
4
n +1
+
3
n 1
is equal to
A
7n 1
1 n
2
B
1 7n
1 n
2
C
7n 1
n
2
+1
D
7
n
2
1
E
7
n
37 If the second number is twice the rst number and a third number is half the rst number
and the three numbers sum to 28, then the numbers are
A (8, 16, 4) B (2, 3, 12) C (7, 9, 11) D (6, 8, 16) E (12, 14, 2)
38 (

7 +3)(

7 3) is equal to
A 2 B 10 C

14 19 D 2

7 9 E 45
P1: FXS/ABE P2: FXS
9780521740494c07.xml CUAU033-EVANS October 16, 2008 21:15
R
e
v
i
s
i
o
n
Chapter 7 Revision of chapters 16 181
39 If
13x 10
2x
2
9x +4
=
P
x 4
+
Q
2x 1
then the values of P and Q are
A P = 1 and Q = 1 B P = 1 and Q = 1 C P = 6 and Q = 1
D P = 6 and Q = 1 E P = 1 and Q = 6
40 The rst term of a geometric sequence is a and the innite sum of the geometric sequence
is 4a. The common ratio of the geometric sequence is
A 3 B 4 C
3
4
D
3
4
E
4
3
41 If
5x
(x +2)(x 3)
=
P
x +2
+
Q
x 3
, then
A P = 2, and Q = 3 B P = 2, and Q = 3 C P = 2, and Q = 3
D P = 2, and Q = 3 E P = 1, and Q = 1
42 If n is a perfect square then the next largest perfect square greater than n is
A n +1 B n
2
+1 C n
2
+2n +1 D n
2
+n E n +2

n +1
43 The area of triangle varies directly as the base length provided the altitude is constant. If
the area equals 14 when the base is 2.4, then the base length (correct to three decimal
places) when the area is 18 will equal
A 3.086 B 5.000 C 6.400 D 9.600 E 0.324
44 Which of the following is not a rational number?
A 0.4 B
3
8
C

5 D

16 E 4.125
45 If
1
x
=
a
b
and
1
y
= a b, then x + y equals
A
2
a
B
a
2
b
2
a
C
ba b
2
+a
a(a b)
D
2a
a
2
b
2
E
2b
a
2
b
2
46 9x
2
4mx +4 is a perfect square when m equals
A 5 B 12 C 2 D 1 E 3
47 If x = (n +1)(n +2)(n +3) where n is a positive integer, then x is not always divisible
by
A 1 B 2 C 3 D 5 E 6
48 The numbers 4, a, b, c, d, e, f, 10 are consecutive terms of an arithmetic sequence. The
sum a +b +c +d +e + f is equal to
A 6 B 10 C 18 D 24 E 48
49 If n and p are both odd numbers, which one of the following numbers must be an even
number?
A n + p B np C np +2 D n + p +1 E 2n + p
P1: FXS/ABE P2: FXS
9780521740494c07.xml CUAU033-EVANS October 16, 2008 21:15
R
e
v
i
s
i
o
n
182 Essential Advanced General Mathematics
7.2 Extended-response questions
1 The diagram represents a glass containing milk. When the height
of the milk in the glass is h cm, the diameter, d cm, of the surface
of the milk is given by the formula
d =
h
5
+6
a Find d when h = 10 b Find d when h = 8.5
c What is the diameter of the bottom of the glass?
d The diameter of the top of the glass is 9 cm. What is the
height of the glass?
d cm
h cm
2 The formula A = 180
360
n
gives the size of each interior angle, A

, of a regular polygon
with n sides.
a Find the value of A when n equals
i 180 ii 360 iii 720 iv 7200
b As n becomes very large
i what value does A approach? ii what shape does the polygon approach?
c Find the value of n when A = 162. d Make n the subject of the formula.
e Three regular polygons, two of which are octagons, meet at a point so that they t
together without any gaps. Describe the third polygon.
3 The gure shows a solid consisting of three parts, a cone, a cylinder
and a hemisphere, all of the same base radius.
a Find in terms of w, s, t and the volume of each part.
b i If the volume of each of the three parts is the same,
nd the ratio w : s : t .
ii If also w +s +t = 11, nd the total volume in terms of .
w
s
t
4 The cost, $C, of manufacturing each jacket of a particular type is
given by the formula
C = an +b for 0 < n 300
where a and b are constants and n is the size of the production run of this type of jacket.
For making 100 jackets, the cost is $108 each.
For making 120 jackets, the cost is $100 each.
a Find the values of a and b.
b Sketch the graph of C against n for 0 < n 300.
c Find the cost of manufacturing each jacket if 200 jackets are made.
d If the cost of manufacturing each jacket is $48.80, nd the size of the production run.
P1: FXS/ABE P2: FXS
9780521740494c07.xml CUAU033-EVANS October 16, 2008 21:15
R
e
v
i
s
i
o
n
Chapter 7 Revision of chapters 16 183
5 a In the diagram, OPQ is a sector of radius R.
A circle, centre C
1
and radius r
1
is inscribed
in this sector.
R
Q
O
P
60
r
3
r
2
r
1
r
1
r
2
r
3
C
3
C
2
C
1
i Express OC
1
in terms of R and r
1
.
ii Show that
r
1
OC
1
=
1
2
and hence express r
1
in terms of R.
b Another circle, centre C
2
, is inscribed in the sector as shown.
i Express OC
2
in terms of r
2
and R.
ii Express r
2
in terms of R.
c Circles C
3
, C
4
, . . . are constructed in a similar way. Their radii are r
3
, r
4
, . . .
respectively. It is known that r
1
, r
2
, r
3
, . . . is a geometric sequence.
i Find the common ratio. ii Find r
n
.
iii Find the sum to innity of the sequence, and interpret the result geometrically.
iv Find in terms of R and , the sum to innity of the areas of the circles with radii r
1
,
r
2
, r
3
, . . . .
6 At the beginning of 1997, Andrew and John bought a small catering business. The prot,
$P, in a particular year is given by
P = an +b
where n is the number of years of operation and a and b are constants.
a Given the table, nd the values of a and b.
Year 1997 2001
Number of years of operation (n) 1 5
Prot 9000 15 000
b Find the prot when n = 12. c In which year was the prot $45 000?
7 Two companies produce the same chemical. For Company A the number of tonnes
produced increases by 80 tonnes per month. For Company B production increases by 4%
per month. Each company produced 1000 tonnes in January 2003. (Let n be the number of
months of production. Use n = 1 for January 2003.)
a Find, to the nearest tonne where appropriate,
i the production of Company A in the nth month
ii the production of each company in December 2004 (i.e. for n = 24)
(contd)
P1: FXS/ABE P2: FXS
9780521740494c07.xml CUAU033-EVANS October 16, 2008 21:15
R
e
v
i
s
i
o
n
184 Essential Advanced General Mathematics
iii the total production of Company A over n months (starting with n = 1 for January
2003)
iv the total production of each company for the period January 2003 to December
2004 inclusive.
b Find in which month of which year the total production of Company A passed 100 000
tonnes.
8 The square shown has each side of length one.
a The perimeter of the square is denoted by P
1
.
What is the value of P
1
?
1
1
1
1
b A new gure is formed by joining two squares of
side length
1
2
to this square, as shown. The
perimeter is denoted by P
2
. What is the value of P
2
?
1
1
1
1
2
1
2
1
2
1
2
1
2
1
2
c What is the perimeter, P
3
, of this gure?
1
1
1
1
2
1
2
1
2
1
2
1
2
1
4
1
4
1
4 4
1
4
1
4
1
4
1
4
1
4
d It is known that P
1
, P
2
, P
3
, . . . are the terms of an arithmetic sequence with rst term
P
1
. What is the common difference ?
e i Find P
4
. ii Find P
n
in terms of P
n1
. iii Find P
n
in terms of n.
iv Draw the diagram of the gure corresponding to P
4
.
9 A piece of wire 28 cm long is cut into
two parts, one to make a rectangle three
times as long as it is wide and the other
to make a square.
3x cm
x cm
a What is the perimeter of the rectangle in terms of x?
b What is the perimeter of the square in terms of x?
c What is the length of each side of the square in terms of x?
Let A be the sum of the areas of the two gures.
d Show that A = 7(x
2
4x +7)
e Use a graphics calculator to help sketch the graph of
A = 7(x
2
4x +7) for 0 < x < 5
f Find the minimum value that A can take and the corresponding value of x.
P1: FXS/ABE P2: FXS
9780521740494c07.xml CUAU033-EVANS October 16, 2008 21:15
R
e
v
i
s
i
o
n
Chapter 7 Revision of chapters 16 185
10 A particular plastic plate manufactured at a factory sells at $1.50. The cost of production
consists of an initial cost of $3500 and then $0.50 per plate. Let x be the number of plates
produced.
a Let $C be the cost of production of x plates. Write an expression for C in terms of x.
b Let $I be the income from selling x plates. Write an expression for I in terms of x.
c On the one set of axes, sketch the graphs of I against x and C against x.
d How many plates must be sold for the income to equal the cost of production?
e How many plates must be sold for a prot of $2000 to be made?
f Let P = I C. Sketch the graph of P against x. What does P represent?
11 n is a natural number less than 50 such that n +25 is a perfect square.
a Show that there exists an integer a such that
n = a(a +10)
b Any natural number less than 100 can be written in the form 10p +q where p and q are
digits. For this representation of n show that q = p
2
.
c Give all possible values of n.
12 a i For the equation

7x 5

2x =

15 7x square both sides to show that this


equation implies
8x 10 =

14x
2
10x
ii Square both sides of this equation and simplify to form the equation
x
2
3x +2 = 0 1
iii The solutions to the equation 1 are x = 1 or x = 2.
Test these solutions for the equation

7x 5

2x =

15 7x
and hence show that x = 2 is the only solution for the original equation.
b Use the techniques of a to solve the equations
i

x +2 2

x =

x +1 ii 2

x +1 +

x 1 = 3

x
13 A geometric series is dened by
x +1
x
2

1
x
+
1
x +1

a Let r be the common ratio. Find r in terms of x.
b i Find the innite sum if x = 1.
ii Find the innite sum if x =
1
4
.
iii Find the innite sum if x = 2.
c Find the possible values of x for which the innite sum is dened.
P1: FXS/ABE P2: FXS
9780521740494c07.xml CUAU033-EVANS October 16, 2008 21:15
R
e
v
i
s
i
o
n
186 Essential Advanced General Mathematics
14 a The area, A, of the shaded region varies directly
as the cube of a.
i If A =
4
3
when a = 2, nd an expression
for A in terms of a.
ii Find A when a = 3.
iii If A = 4500, nd a.
0
x
y = x(x a)
y
b The area, A
1
, of the shaded region varies directly
as the cube of a
i If A
1
= 1152 when a = 24, nd an
expression for A
1
in terms of a.
ii Find A
1
when a = 18.
iii Find a when A
1
= 3888.
x
y
y = x(x a)
a
2
4
a
2
,
0
c The area, A
2
, of the shaded region varies partly as
the reciprocal of a and partly as the reciprocal of b.
x
y
y =
1
x
2
a b
i Find A
2
in terms of a and b if,
when a = 1 and b = 2, A
2
=
1
2
and
when a = 3 and b = 4, A
2
=
1
12
ii Find A
2
when a = 1 and b = 6.
iii Find A
2
when a =
1
4
and b = 3.
iv Find A
2
when a =
1
100
and b = 100.
v Find A
2
when a =
1
1000
and b = 1000.
15 In a vegetable garden carrots are planted in rows parallel to the fence.
F
e
n
c
e
r
o
w

1
r
o
w

2
r
o
w

3
r
o
w

4
rabbit
burrow
0.5 m 1.5 m 1.5 m 1.5 m
a Calculate the distance between the fence and the 10th row of carrots.
b If t
n
represents the distance between the fence and the nth row, nd a formula for t
n
in
terms of n.
P1: FXS/ABE P2: FXS
9780521740494c07.xml CUAU033-EVANS October 16, 2008 21:15
R
e
v
i
s
i
o
n
Chapter 7 Revision of chapters 16 187
c Given that the last row of carrots is less than 80 m from the fence, what is the largest
number of rows possible in this vegetable garden?
d A systematic rabbit has its burrow under
the fence as shown in the diagram. It runs
to the rst row, takes a carrot and returns
it to its burrow. It then runs to the
second row, collects a carrot and
returns it to its burrow.
It continues in this way until it has
15 carrots. Calculate the shortest distance the rabbit has to run to accomplish this.
F
e
n
c
e
rabbit
burrow
Trip 1
Trip 2
r
o
w

1
r
o
w

2
16 The potential energy, P joules, of a body varies jointly as the mass, m kg, of the body and
the height, h m, of the body above the ground.
a For a body of mass 5 kg
i nd P in terms of h if P = 980 when h = 20
ii sketch the graph of P against h iii nd P if h = 23.2.
b i Find P in terms of h and m if P = 980 when h = 20 and m = 5.
ii Find the percentage change in potential energy if the height (h m) is doubled and the
mass remains constant.
iii Find the percentage change in potential energy if a body has a quarter of the
original height (h m) and double the original mass (m kg).
c If a body is dropped from a height, h m, above ground level its speed, V m/s, when it
reaches the ground is given by V =

19.6h.
i Find V when h = 10. ii Find V when h = 90.
d In order to double the speed a given body has when it hits the ground, by what factor
must the height from which it is dropped be increased?
17 In its rst month of operation a soft drink manufacturer produces 50 000 litres of a type of
soft drink. In each successive month the production rises 5000 litres a month.
a i The quantity of soft drink, t
n
, produced in the nth month can be determined from a
rule of the form
t
n
= a +(n 1)d
Find the values of a and d.
ii In which month will the factory double its original production?
iii How many litres in total will be produced in the rst 36 months of operation?
b Another soft drink manufacturer sets up a factory at the same time as the rst. In the
rst month the production is 12 000 litres. The production of this factory increases by
10% every month.
i Find a rule for q
n
, the quantity of soft drink produced in the nth month.
ii Find the total amount of soft drink produced in the rst 12 months.
c If the two factories start production in the same month, in which month will the
production of the second factory exceed the production of the rst factory?
P1: FXS/ABE P2: FXS
9780521740494c07.xml CUAU033-EVANS October 16, 2008 21:15
R
e
v
i
s
i
o
n
188 Essential Advanced General Mathematics
18 In a certain country grain production and population statistics are produced.
In December 1986 the population of the country was 12.5 million.
In 1986 the grain production was 10 million tonnes.
It was found that since then the population has grown by 5% each year and grain
production has increased by 0.9 million tonnes each year.
Let P
1
denote the population in December 1986.
Let p
2
denote the population in December 1987.
P
n
denotes the population n 1 years after December 1986.
Let t
1
denote the grain production in 1986.
Let t
2
denote the grain production in 1987.
t
n
denotes the grain production in the (n 1)th year after 1986.
a Find, in millions of tonnes, the grain production in
i 1992 ii 1999
b Find an expression for t
n
.
c Find the total grain production for the 20 years starting 1986.
d How many years does it take for the grain production to double?
e Find an expression for P
n
.
f How many years does it take for the population to double?
19 The diagram shows a straight road OD where
OD = 6 km. A hiker is at A, 2 km from O.
2 km
x km O
A
X D
6 km
The hiker can walk at 3 km/h when off-road
but at 8 km/h along the road.
a Calculate the time taken, in hours and
minutes, correct to the nearest minute,
if he hikes directly to X then along the
road to D where OX = 3 km.
b Calculate OX, correct to one decimal place (in km) if the total time taken was
1
1
2
hours.
20 Seventy-six photographers submitted work for a photographic exhibition in which they
were permitted to enter not more than one photograph in each of the three classes, black
and white (B), colour prints (C), transparencies (T). Eighteen entrants had all their work
rejected while 30 B, 30 T and 20 C were accepted.
From the exhibitors, as many showed T only as showed T and C.
There were three times as many exhibitors showing B only as showing C only.
Four exhibitors showed B and T but not C.
a Write the last three sentences in symbolic form.
b Draw a Venn diagram representing the information.
c Find
i n(B C T) ii n(B C T

)
P1: FXS/ABE P2: FXS
9780521740494c07.xml CUAU033-EVANS October 16, 2008 21:15
R
e
v
i
s
i
o
n
Chapter 7 Revision of chapters 16 189
21 Let A =

a b
c d

with b = 0 and c = 0
a Find
i A
2
ii 3A
b If A
2
= 3A I, show that
i a +d = 3 ii det(A) = 1
c If A has the properties
r
a +d = 3
r
det(A) = 1
show that A
2
= 3A I.
22 The trace of square matrix A is dened to be the sum of the leading diagonal of A, and it is
denoted by Tr(A).
For example, if A =

6 3
2 2

, Tr(A) = 8
a Prove each of the following for any 2 2 matrices X and Y.
i Tr(X +Y) = Tr(X) +Tr(Y)
ii Tr(X) = Tr(X) iii Tr(XY) = Tr(YX)
b Use the results of a to show that there do not exist 2 2 matrices X and Y such that
XY YX = I.
P1: FXS/ABE P2: FXS
9780521740494c08.xml CUAU033-EVANS August 22, 2009 12:4
C H A P T E R
8
Transformations
Objectives
To define translations
To define reflections in the axes and in the line y = x
To define dilations from the x axis and the y axis
To apply these transformations to points and figures
To find algebraic rules for these transformations
To find the composition of two transformations and give the rule for this
transformation
To apply transformations to graphs
To determine the rule which transforms one graph to another (within a suitable
family of graphs)
To sketch the graph of the absolute value function, the integer part function and
transformations of these graphs
To describe transformations with function notation
Introduction
In this chapter three different types of transformations of the cartesian plane are discussed.
These are
Translations Reections Dilations.
These transformations are very useful in the graphing of functions.
A transformation is a rule which associates each point in the cartesian plane to another
point in the plane. These points uniquely dene each other through the rule.
For example, one transformation can be dened by:
Add 5 to each x coordinate.
This can be expressed algebraically
(x, y) (x +5, y)
e.g., (1, 6) (6, 6)
190
P1: FXS/ABE P2: FXS
9780521740494c08.xml CUAU033-EVANS August 22, 2009 12:4
Chapter 8 Transformations 191
This is read as:
The point with coordinates (1, 6) is mapped to the point with coordinates (6, 6).
A formal denition is the following:
A transformation T is a mapping from R
2
to R
2
such that if T(a, b) = T(c, d) then a = c and
b = d.
The transformation dened above can be written
T : R
2
R
2
, T(x, y) = (x +5, y)
This formal notation is avoided in this book and the transformation will be dened by the rule
given in the form
(x, y) (x +5, y)
8.1 Translations
The transformation dened above, i.e. (x, y) (x +5, y), is an example of a translation. A
translation is a transformation for which each point in the plane is moved the same distance in
the same direction.
In this section, a vector will mean a column of two numbers. The rst number indicates a
move in the positive or negative direction of the x axis and the second indicates a move in
the positive or negative direction of the y axis. A directed line segment is used to illustrate a
vector.
For example, the vector
_
2
3
_
is the vector 2 to the right and 3 up.
2
3
The image of the point (1, 2) would be (3, 5) under the translation
determined by the vector
_
2
3
_
.
The top number gives the displacement in the positive or negative direction of the x axis and
the lower number gives the displacement in the positive or negative direction of the y axis.
1
2
3
2
4
3
2
1
If the top number is negative, the displacement is to the left and if the lower number is
negative, the displacement is downwards.
P1: FXS/ABE P2: FXS
9780521740494c08.xml CUAU033-EVANS August 22, 2009 12:4
192 Essential Advanced General Mathematics
Vectors can be used to describe translations. They will be studied in a more general context
in Chapter 15.
Example 1
The point A has coordinates (1, 2). Find the image of A under the translation dened by the
vector
_
4
2
_
.
Solution
4 2 0 2
2
4
A'
A
x
y
The image of A is (3, 4).
Example 2
Find the vector which denes the translation which takes A(3, 4) to A

(2, 6).
Solution
Let (3, 4) (2, 6)
The vector
_
a
b
_
denes this translation if (3 +a, 4 +b) = (2, 6)
This implies
3 +a = 2 and 4 +b = 6
i.e., a = 1 and b = 2
The vector is
_
1
2
_
Example 3
A translation is dened by the rule (x, y) (x 3, y +2) and the point A with coordinates
(a, b) is mapped to A

(7, 1). Find the values of a and b.


Solution
(a, b) (a 3, b +2) = (7, 1)
a 3 = 7 and b +2 = 1
a = 10 and b = 3
P1: FXS/ABE P2: FXS
9780521740494c08.xml CUAU033-EVANS August 22, 2009 12:4
Chapter 8 Transformations 193
Exercise 8A
1 Find the images of the points in each of the following under the translations described by
the given vectors.
Example 1
a (3, 1),
_
4
2
_
b (4, 5),
_
2
4
_
c (2, 4),
_
4
3
_
d (3, 2),
_
2
3
_
e (4, 5),
_
3
2
_
2 For each of the following, nd the vectors describing the translations that map A to A

.
Example 2
a A(1, 2), A

(5, 3) b A(3, 8), A

(2, 9) c A(1, 2), A

(5, 4)
d A(3, 0), A

(4, 6) e A(4, 3), A

(0, 0)
3 In each of the following the given point A

is the image of an object point A under the


translation described by the given vector. Find the coordinates of A.
a A

(7, 9),
_
2
3
_
b A

(3, 6),
_
1
4
_
c A

(0, 6),
_
2
3
_
4 Give the vectors describing the translations
which map
a ABC to PQR
b ABC to LMN
c XYZ to ABC
d ABC to ABC
x
N
L M
A B
R
Q
Y
Z
X
P
C
4
2
0
4
4 2 2 6
y
5 Draw axes for x and y from 4 to 5. Draw
the following triangles.
ABC withA(2, 2), B(4, 2), C(2, 5)
PQR with P(1, 2), Q(3, 2), R(1, 1)
XYZ with X(3, 1), Y(1, 1), Z(3, 4).
Give the vectors describing the translations which map
a ABC to PQR b PQR to ABC
c PQR to XYZ d ABC to ABC
6 a Find the image of the point (2, 3) under the translation determined by the vector
_
3
4
_
followed by the translation determined by the vector
_
1
5
_
.
b Find the image of the point (5, 6) under the translation determined by the vector
_
3
4
_
followed by the translation determined by the vector
_
1
5
_
.
c Describe how one translation may be used to obtain the nal image in a and b.
P1: FXS/ABE P2: FXS
9780521740494c08.xml CUAU033-EVANS August 22, 2009 12:4
194 Essential Advanced General Mathematics
7 A translation has a rule (x, y) (x 5, y +3). Find
Example 3
a the image of the point (1, 3) under this translation
b a and b if (a, b) (6, 7) under this translation.
8 A translation has a rule (x, y) (x +1, y).
a The points (0, 0), (1, 1), (2, 4), (3, 9) all lie on
the graph of y = x
2
. Find the image of each of
these points under the translation.
b Sketch the graph of y = x
2
for x 0 as shown,
and complete.
c Describe the image of all the points on the
graph of y = x
2
under this transformation.
x
y
(3, 9) ( , )
( , )
(2, 4) (3, 4)
(1, 1)
(0, 0) (1, 0)
8.2 Reflections
P
1
is the image of P under a reection in the line m.
The line m is the perpendicular bisector of line PP
1
.
m
P
P
1
A
1
is the image of A under the transformation
reection in the y axis.
A
2
is the image of A under the transformation
reection in the x axis.
A
3
is the image of A under the transformation
reection in the line y = x.
x
y
3
2
1
2
2 1
1
0 1 2 3 4 3
A
2
(2, 1)
A(2, 1)
y = x
A
3
(1, 2)
A
1
(2, 1)
For reection in the x axis the rule is (x, y) (x, y)
For reection in the y axis the rule is (x, y) (x, y)
For reection in the line y = x the rule is (x, y) (y, x)
Example 4
The triangle ABC has coordinates A(1, 0), B(1, 6) and C(4, 6). Find the image of triangle ABC
under a reection in
a the x axis b the line y = x.
P1: FXS/ABE P2: FXS
9780521740494c08.xml CUAU033-EVANS August 22, 2009 12:4
Chapter 8 Transformations 195
Solution
a A(1, 0) A

(1, 0)
B(1, 6) B

(1, 6)
C(4, 6) C

(4, 6)
b A(1, 0) A

(0, 1)
B(1, 6) B

(6, 1)
C(4, 6) C

(6, 4)
x
y
6
5
4
3
2
1
1 2 3 4 5 6
1
2
3
4
5
6
A"(0, 1)
A(1, 0)
B"(6, 1)
B'(1, 6) C'(4, 6)
C"(6, 4)
C(4, 6) B(1, 6)
y = x
0
Exercise 8B
1 Draw axes for x from 5 to 5 and for y from 0 to 5. Draw triangle ABC by plotting
Example 4
A(1, 2), B(3, 2) and C(3, 5). Draw the image A

when ABC is reected in the y axis.


2 Draw axes for x from 0 to 5 and for y from 2 to 2. Draw triangle PQR where P is (1, 1),
Q is (5, 1) and R is (4, 0). Draw the image P

when PQR is reected in the x axis.


3 Draw axes for x and y from 5 to 0. Draw rectangle WXYZ where W is (3, 1), X is
(3, 2), Y is (5, 2) and Z is (5, 1). Draw the mirror line y = x. Draw the image
W

when WXYZ is reected in the mirror line.


4 Draw axes for x and y from 1 to 8. Plot the points A(2, 1), B(5, 1), C(7, 3) and D(4, 3).
Draw the parallelogram ABCD and its image by reection in the line y = x.
5 Draw axes for x and y from 6 to 7. Draw triangle ABC where A is (6, 2), B is
(3, 4) and C is (2, 1). Draw the following images of triangle ABC
a triangle A
1
B
1
C
1
by reection in the y axis
b triangle A
2
B
2
C
2
by reection in the line y = x (this is the straight line through the
points (2, 2), (4, 4))
c triangle A
3
B
3
C
3
by reection in the x axis.
6 Find the image of (6, 2) under each of the following
a reection in the line y = x b reection in the line x = 0
c reection in the line y = 0
P1: FXS/ABE P2: FXS
9780521740494c08.xml CUAU033-EVANS August 22, 2009 12:4
196 Essential Advanced General Mathematics
7 Find the image of (0, 1) under each of the following
a reection in the line y = x b reection in the line x = 0
c reection in the line y = 0 d reection in the line y = x
8.3 Dilations from the axes
For reections and translations, lengths and angles are preserved. In this section dilations from
the axes are introduced. These transformations do not preserve distances or angles.
The transformation dilation from the y axis of factor k, is dened by the rule
(x, y) (kx, y); k R
+
For example, for k = 2, the unit square
A(0, 0), B(0, 1), C(1, 1), D(1, 0)
is transformed to the rectangle
A(0, 0), B(0, 1), C

(2, 1), D

(2, 0)
x
y
B(0, 1)
A(0, 0) D(1, 0) D'(2, 0)
C(1, 1) C'(2, 1)
Example 5
Triangle ABC has vertices A(1, 2), B(3, 4), C(5, 1).
Find the image of the triangle under a dilation of factor 2 from the y axis.
Solution
(3, 4) (6, 4)
(1, 2) (2, 2)
(5, 1) (10, 1)
x
y
A(1, 2)
B(3, 4) B'(6, 4)
C'(10, 1)
C(5, 1)
0
1
1
2
3
4
5
2 3 4 5 6 7 8 9 10
A'(2, 2)
The transformation dilation from the x axis of factor k is dened by the rule:
(x, y) (x, ky); k R
+
For example, a triangle with vertices A(1, 2), B(3, 4), C(5, 1) is mapped to the triangle with
vertices A

(1, 4), B

(3, 8), C

(5, 2) under a dilation of factor 2 from the x axis.


P1: FXS/ABE P2: FXS
9780521740494c08.xml CUAU033-EVANS August 22, 2009 12:4
Chapter 8 Transformations 197
x
y
B'(0, 2) C'(1, 2)
C(1, 1)
D(1, 0) A(0, 0)
B(0, 1)
The unit square with vertices
A(0, 0), B(0, 1),
C(1, 1), D(1, 0)
is mapped to the rectangle with vertices
A(0, 0), B

(0, 2),
C

(1, 2), D

(1, 0)
Exercise 8C
1 Find the image of the point (1, 3) under each of the following
Example 5
a a dilation of factor 3 from the x axis b a dilation of factor 2 from the y axis
c a dilation of factor 4 from the y axis.
2 Give the rule for the dilation from the x axis which maps (1, 2) (1, 8).
3 Give the rule for the dilation from the y axis which maps (3, 2) (9, 2).
4 a Find the image of the unit square A(0, 0), B(0, 1), C(1, 1), D(1, 0) under a dilation
from the x axis of factor 3.
b Find the image of the unit square A(0, 0), B(0, 1), C(1, 1), D(1, 0) under a dilation
from the y axis of factor 3.
5 Triangle ABC has vertices A(0, 0), B(3, 0), C(3, 4).
a Find the image of triangle ABC under
i a dilation of factor
1
2
from the x axis
ii a dilation of factor
1
2
from the y axis.
b Illustrate triangle ABC and its images.
8.4 Rules for transformations
Transformations can be described through a rule given in terms of ordered pairs. For example,
it has been seen that (x, y) (x +2, y +3) describes the transformation of 2 units in the
positive direction of the x axis and 3 units in the positive direction of the y axis.
P1: FXS/ABE P2: FXS
9780521740494c08.xml CUAU033-EVANS August 22, 2009 12:4
198 Essential Advanced General Mathematics
Example 6
Find the rules for each of the following transformations.
a a translation determined by the vector
_
3
2
_
b a reection in the line y = x c a dilation of factor
1
4
from the x axis.
Solution
a (x, y) (x +3, y 2) b (x, y) (y, x) c (x, y)
_
x,
1
4
y
_
A point (a, b) is said to be invariant under a transformation if (a, b) (a, b) under that
transformation.
The transformation which maps (x, y) (x, y) for all (x, y) R
2
is called the identity
transformation.
Example 7
A transformation has rule (x, y) (2x +3, y +4).
a Find the image of (2, 3) under this transformation.
b Find the coordinates of the point which maps to (11, 12).
Solution
a If x = 2 and y = 3
(x, y) (2 2 +3, 3 +4) = (7, 7)
i.e., (2, 3) (7, 7)
b If (2x +3, y +4) = (11, 12)
Then 2x +3 = 11 and y +4 = 12
x = 4 and y = 8
Exercise 8D
1 For each of the following transformations nd
Example 6
i the rule ii the invariant points (if they exist).
a a translation determined by the vector
_
5
6
_
b a dilation from the x axis of factor 4 c a dilation from the y axis of factor
1
3
d a translation determined by the vector
_
2
3
_
e a reection in the line y = x f a reection in the y axis
2 A transformation has rule (x, y) (3 x, 2y +1)
Example 7
a Find the image of (2, 3) under this transformation.
P1: FXS/ABE P2: FXS
9780521740494c08.xml CUAU033-EVANS August 22, 2009 12:4
Chapter 8 Transformations 199
b Find the coordinates of the point which maps to (6, 12).
c Find the coordinates of the point which is invariant under this transformation.
3 A transformation has rule (x, y) (3 y, 4 x). Find the coordinates of the invariant
point.
4 A transformation has rule (x, y) (3 2x, 2y +1).
a Find the image of (4, 1) under this transformation.
b Find the coordinates of the point which maps to (7, 12).
c Find the coordinates of the point which is invariant under this transformation.
5 A transformation has rule (x, y) (x, 2y).
a Find the image of (1, 3) under this transformation.
b Find the coordinates of the point which maps to (0, 0).
c Find the coordinates of the point which is invariant under this transformation.
8.5 Composition of transformations
Consider a transformation determined by the vector
_
2
3
_
followed by a reection in the line
x = 0.
The rules for these two transformations are (x, y) (x +2, y +3) and (x, y) (x, y)
respectively.
The triangle A(1, 1), B(5, 1), C(5, 2) is mapped to A

(3, 4), B

(7, 4), C

(7, 5)
x
y
0
1 1
1
2
3
4
5
6
2 3 4 5 6 7 2 3 4 5 6 7
C"
B" A" A' B'
C'
A B
C
One rule can be used to describe the transformation of triangle ABC to triangle A

.
Consider
(x, y) (x +2, y +3) ((x +2), y +3)
translation reection
The rule is (x, y) ((x +2), y +3)
Note: (1, 1) (3, 4) (5, 1) (7, 4) (5, 2) (7, 5)
This new rule is called the composition of the two transformations.
P1: FXS/ABE P2: FXS
9780521740494c08.xml CUAU033-EVANS August 22, 2009 12:4
200 Essential Advanced General Mathematics
Example 8
Find the rule for the transformation dened by a dilation of factor 2 from the x axis followed
by a translation determined by the vector
_
1
3
_
.
Solution
The dilation is given by the rule (x, y) (x, 2y) and the translation by the rule
(x, y) (x 1, y +3).
The composition is determined by
(x, y) (x, 2y) (x 1, 2y +3)
dilation translation
i.e., the rule is (x, y) (x 1, 2y +3)
Exercise 8E
1 For each of the following nd the rule for
Example 8
i transformation of column A followed by transformation of column B
ii transformation of column B followed by transformation of column A.
Column A Column B
a a translation determined by the
vector
_
2
3
_
a translation determined by the
vector
_
3
2
_
b a dilation from the x axis of
factor 2
a dilation from the y axis of factor 2
c a translation determined by the
vector
_
4
5
_
a dilation from the x axis of factor 3
d a translation determined by the
vector
_
1
2
_
a reection in the y axis
e a reection in the line y = x a reection in the line y = x
f a reection in the line x = 2 a reection in the x axis
g a dilation from the y axis of
factor
1
2
a translation determined by the
vector
_
1
2
_
h a dilation from the y axis of
factor 2
a translation determined by the
vector
_
2
3
_
P1: FXS/ABE P2: FXS
9780521740494c08.xml CUAU033-EVANS August 22, 2009 12:4
Chapter 8 Transformations 201
8.6 Applying transformations to graphs
of functions
As mentioned in the introduction, transformations have an important role in graphing
functions. In this section techniques are developed for graphing functions using
transformations.
Consider {(x, y) : y = x
2
}. This is the set of points on the graph of y = x
2
. Transformations
can be applied to this set of points.
For example, consider the translation determined
by the vector
_
2
0
_
applied to this set of points.
It can be seen that all the images of the points of
y = x
2
lie on the graph of y = (x 2)
2
x
(1, 1)
(2, 4)
(0, 0) (2, 0)
(3, 1)
(4, 4)
(2, 4)
(1, 1)
(0, 4)
y
The rule for this transformation is (x, y) (x +2, y).
Each coordinate (x

, y

) of the image must be of this


form, i.e. x

= x +2, y

= y. The relationship between


x and y is known to be y = x
2
. Therefore if (x

, y

) is
the image of a point on y = x
2
, and x = x

2, then
y

= (x

2)
2
.
Example 9
Find the equation of the image of the line y = x +1 under the translation determined by the
vector
_
2
1
_
. Sketch the graph of y = x +1 and its image.
Solution
The rule for the translation is (x, y) (x +2, y +1).
Let (x

, y

) be the coordinates of the image of (x, y)


x

= x +2, y

= y +1 and y = x +1
Thus x = x

2 and y = y

1
and the relation y = x +1
becomes y

1 = x

2 +1
y

= x

The points (1, 0), (0, 1), (1, 2) and their


images are shown,
i.e., (1, 0) (1, 1)
(0, 1) (2, 2)
(1, 2) (3, 3)
x
y
0
(1, 1)
(0, 1)
(1, 2)
(1, 0)
(2, 2)
(3, 3)
y = x + 1 y = x
P1: FXS/ABE P2: FXS
9780521740494c08.xml CUAU033-EVANS August 22, 2009 12:4
202 Essential Advanced General Mathematics
Example 10
Find the equation of the image of the parabola y = x
2
under a dilation from the y axis of factor
1
2
. Sketch the graph of y = x
2
and its image.
Solution
The rule for the dilation is (x, y)
_
1
2
x, y
_
Therefore, if (x

, y

) is a point of the image of y = x


2
Then x

=
x
2
and y

= y
Also y = x
2
Therefore the relationship between x

and y

is
y

= (2x

)
2
= 4(x

)
2
x
y
(1, 1)
(1, 4)
(2, 4) (2, 4)
(1, 4)
y = 4x
2
y = x
2
0
, 1
1
2
Note:
(1, 1)
_
1
2
, 1
_
(2, 4) (1, 4)
(2, 4) (1, 4)
Example 11
For y = x
2
a nd the equation for the image of y = x
2
under a dilation of factor
1
2
from the x axis
b sketch the graph of y = x
2
and its image under this transformation.
c i Find the equation for the image of y = x
2
under the dilation of a followed by a
translation determined by the vector
_
1
2
_
.
ii Sketch the graph of this second image on the same set of axes as b.
Solution
a The rule for dilation is (x, y)
_
x,
1
2
y
_
If (x

, y

) is a point on the image


x

= x and y

=
1
2
y
i.e., x

= x and y = 2y

Also y = x
2
P1: FXS/ABE P2: FXS
9780521740494c08.xml CUAU033-EVANS August 22, 2009 12:4
Chapter 8 Transformations 203
The image will have equation 2y

= (x

)
2
i.e., y

=
1
2
(x

)
2
b
x
y
0
(1, 1)
(2, 4)
(2, 2)
y = x
2
1,
1
2
1
2
y = x
2
c i The rule becomes
(x, y)
_
x,
1
2
y
_

_
x +1,
1
2
y +2
_
dilation translation
i.e., let (x

, y

) be a point on the image.


x

= x +1, y

=
1
2
y +2
x = x

1 and y = 2(y

2)
y = x
2
is mapped to 2(y

2) = (x

1)
2
y

=
1
2
(x

1)
2
+2
ii
x
y
0
(1, 1)
(1, 2)
(2, 4)
(2, 2)
y = x
2
1
2
y = (x 1)
2
+ 2
1,
1
2
1
2
y = x
2
P1: FXS/ABE P2: FXS
9780521740494c08.xml CUAU033-EVANS August 22, 2009 12:4
204 Essential Advanced General Mathematics
In Exercise 8F, reference is made to the following basic graphs.
x
y
y = x
0
x
y
0
y = x
2
x
y
0
y =
1
x
x
y
0
x
2
+ y
2
= 1
x
y
0
y = 2
x
(0, 1)
Example 12
Find the image of each of the following curves under a dilation of factor 3 from the y axis
followed by a translation determined by the vector
_
1
2
_
.
a y = x
2
b y =
1
x
c x
2
+ y
2
= 1
Solution
The rule is given by (x, y) (3x, y) (3x 1, y +2)
dilation translation
x

= 3x 1 and y

= y +2
i.e., x =
x

+1
3
and y = y

2
a y = x
2
is mapped to y

2 =
_
x

+1
3
_
2
The image has equation y =
_
x +1
3
_
2
+2
b y =
1
x
is mapped to y

2 =
1
x

+1
3
y

=
3
x

+1
+2
The image has equation y =
3
x +1
+2
P1: FXS/ABE P2: FXS
9780521740494c08.xml CUAU033-EVANS August 22, 2009 12:4
Chapter 8 Transformations 205
c x
2
+ y
2
= 1 is mapped to
_
x

+1
3
_
2
+(y

2)
2
= 1
The image has equation
(x +1)
9
2
+(y 2)
2
= 1
The image is an ellipse.
Exercise 8F
1 Find the image of each of the following curves under the translation determined by the
vector
_
1
4
_
. State the equation of the image and sketch the graph of both the original
relation and its image on the one set of axes.
Example 9
a y = x b y = x
2
c y =
1
x
d y =
1
x
2
e x
2
+ y
2
= 1
2 Find the image of each of the following curves under the dilation from the x axis given by
Example 10
the rule (x, y) (x, 2y), i.e., a dilation of factor 2 from the x axis. State the equation of
the image and sketch the graph of both the original relation and its image on the one set of
axes.
a y = x b y = x
2
c y =
1
x
d y =
1
x
2
e x
2
+ y
2
= 1
3 For each of the curves considered in 1, nd the equation of its image under the
transformation dened by a reection in the line x = 0 followed by a translation
determined by the vector
_
3
2
_
.
Examples 11, 12
4 For each of the curves considered in 1, nd the equation of its image under the
transformation dened by a dilation of factor 2 from the y axis followed by a translation
determined by the vector
_
3
1
_
.
5 Find the image of the graph of y = 2x +3 under the transformation dened by rst
reecting in the line y = x and then translating as determined by the vector
_
4
6
_
.
6 Find the image of the line {(x, y) : y = x +2} under each of the following
transformations. Sketch a graph showing {(x, y) : y = x +2} and its image in each case.
a a translation determined by the vector
_
2
4
_
b a reection in the x axis
c a dilation of factor 4 from the x axis d a reection in the line y = x
e a reection in the y axis
P1: FXS/ABE P2: FXS
9780521740494c08.xml CUAU033-EVANS August 22, 2009 12:4
206 Essential Advanced General Mathematics
7 Repeat 1 for the circle {(x, y) : x
2
+ y
2
= 4}.
8 Consider {(x, y) : y = x
2
}. Find the image after
a reection in the x axis then a dilation of factor 2 from the x axis
b translation where (0, 0) (5, 2) then a dilation of factor
1
2
from the y axis
c dilation of factor
1
2
from the x axis followed by translation where (0, 0) (5, 2)
d translation where (0, 0) (2, 1) followed by a reection in the y axis
e reection in the line y = x followed by a translation where (0, 0) (0, 2).
9 Sketch the graph of the image of {(x, y) : y = x
2
} in 8 ae above.
10 Consider {(x, y) : y = 2
x
}. Find the image after
a translation where (0, 0) (1, 0) b translation where (0, 0) (0, 1)
c dilation of factor 2 from the y axis followed by dilation of factor
1
3
from the x axis
d dilation of factor
1
3
from the x axis followed by dilation of factor 2 from the y axis
e translation where (0, 0) (2, 4) followed by reection in the y axis
f translation where (0, 0) (0, 2) then dilation of factor
1
2
from the y axis then
reection in the x axis.
11 Sketch the graph of the image of {(x, y) : y = 2
x
} in 10 af above.
8.7 Determining transformations
In the previous section a method was presented for determining the equation of the image of a
graph under a given transformation. In this section a procedure for determining the
transformations which have produced a particular image is discussed.
Example 13
Find a sequence of transformations which maps y = x
2
to y = 2(x +3)
2
4
Solution
The composition of transformations maps (x, y) to (x

, y

)
y

= 2(x

+3)
2
4
Rearrange to make the transformation from y = x
2
more obvious
y

+4
2
= (x

+3)
2
P1: FXS/ABE P2: FXS
9780521740494c08.xml CUAU033-EVANS August 22, 2009 12:4
Chapter 8 Transformations 207
It can be seen that to obtain this, take
y =
y

+4
2
and x = (x

+3)
i.e., y

= 2y 4 and x

= x 3
A dilation of factor 2 from the x axis followed by a translation determined by the
vector
_
3
4
_
gives this image.
Example 14
Find a sequence of transformations which maps y = 2
x
to y = 3(2
x2
) 4.
Solution
Assume the composition of transformations maps (x, y) to (x

, y

)
Write y

= 3(2
x

2
) 4
Rearrange to
y

+4
3
= 2
x

2
It can be seen to obtain this, take
y =
y

+4
3
and x = x

2
y

= 3y 4 and x

= x +2
A dilation of factor 3 from the x axis followed by a translation determined by the
vector
_
2
4
_
gives this image.
Exercise 8G
1 Find the single transformation which maps
Example 13
a {(x, y) : y = x
2
} to {(x, y) : y = 2x
2
}
b {(x, y) : y = x
2
} to {(x, y) : y = (x +2)
2
}
c {(x, y) : y = x
2
} to {(x, y) : x = y
2
}
d {(x, y) : y = 2
x
} to {(x, y) : y = 2
2x
}
e {(x, y) : y = 2
x
} to {(x, y) : y = 3 2
x
}
f {(x, y) : y = 2
x
} to {(x, y) : y = 2
x3
}
g {(x, y) : y = 2
x
} to {(x, y) : y = 2
x
}
h {(x, y) : y = 2
x
} to {(x, y) : y = 2
x
}
2 Find a sequence of transformations which maps
Example 14
a {(x, y) : y = x
2
} to {(x, y) : y = 2(x 3)
2
}
b {(x, y) : y = x
2
} to {(x, y) : y = 2x
2
3}
c {(x, y) : y = x
2
} to {(x, y) : y = 2(x 3)
2
+1}
P1: FXS/ABE P2: FXS
9780521740494c08.xml CUAU033-EVANS August 22, 2009 12:4
208 Essential Advanced General Mathematics
d
_
(x, y) : y =
1
x
_
to
_
(x, y) : y =
2
x 3
_
e
_
(x, y) : y =
1
x
_
to
_
(x, y) : y =
2
x
3
_
f
_
(x, y) : y =
1
x
_
to
_
(x, y) : y =
1
3 x
_
g y = 2
x
to y = 2
x1
3
+4
h y =
1
x
to y =
3
2x 4
8.8 Absolute value function and integer
value function
Absolute value function
Let f : R R be dened as
f (x) =
_
x if x 0
x if x < 0
This function is written as f (x) = |x|
The graph of this function is as shown.
It is known as the modulus function or absolute value function.
x
y
0
The images of this graph under several transformations are considered.
Example 15
Find the image of {(x, y) : y = |x|} and sketch the graph of the image for each of the following
transformations.
a a translation dened by the vector
_
2
3
_
b a dilation of factor 4 from the y axis
c a reection in the line y = x d a reection in the x axis
Solution
a (x, y) (x +2, y +3)
(x, y) (x

, y

)
x

= x +2, y

= y +3
x = x

2 and y = y

3
{(x, y) : y = |x|} is mapped to
{(x

, y

) : y

3 = |x

2|}
x
y
0
(2, 3)
(0, 5)
y = |x 2| + 3
P1: FXS/ABE P2: FXS
9780521740494c08.xml CUAU033-EVANS August 22, 2009 12:4
Chapter 8 Transformations 209
b (x, y) (4x, y)
(x, y) (x

, y

)
x

= 4x and y

= y
{(x, y) : y = |x|} is mapped to
_
(x

, y

) : y

_ x
y
(4, 1) (4, 1)
y =
x
4
0
c (x, y) (y, x)
(x, y) (x

, y

)
x

= y and y

= x
{(x, y) : y = |x|} is mapped to
{(x

, y

) : x

= |y

|}
x
y
0
x = |y|
d (x, y) (x, y)
(x, y) (x

, y

)
x

= x, y

= y
{(x, y) : y = |x|} is mapped to
{(x

, y

) : y

= |x

|}
x
y
0
y = |x|
Integer value function
The integer value function I : R Z is dened by I (x) = [x] where [x] is the greatest integer
not exceeding x. For example,
[3.9] = 3, [5] = 5, [4.1] = 5, [] = 3
For all real numbers x, we have x 1 [x] x
x
y
1
1
1
1
2
3
2 3 4
y = I(x)
2
2
P1: FXS/ABE P2: FXS
9780521740494c08.xml CUAU033-EVANS August 22, 2009 12:4
210 Essential Advanced General Mathematics
Using the TI-Nspire
Absolute Value function
The abs( ) function can be found in the
catalog ( 1 ) or typed directly. It
works as shown.
The graph of the absolute value function
can be obtained in the usual way in a
Graphs & Geometry application (c2).
The graph of f 1(x) = abs(x) is shown
with medium line width. The graph of the
composite function f 2(x) = abs(x
2
4) is
also shown.
Integer Value function
The integer value function rounds down to
the nearest integer, so on the TI-Nspire it is
called the oor( ) function and can be
found in the Number Tools submenu of the
Number menu (b286). It works
as shown.
(Note the difference from the Integer Part
Function (b282) that returns
the integer part of a number.)
The graph of the integer value function can
be obtained in the usual way in a Graphs &
Geometry application (c2).
The graph of f 1(x) = oor(x) is shown
with medium line width. The graph of the
composite function f 2(x) = oor
_
x
2
_
is
also shown.
P1: FXS/ABE P2: FXS
9780521740494c08.xml CUAU033-EVANS August 22, 2009 12:4
Chapter 8 Transformations 211
Using the Casio ClassPad
To enter the absolute value function, switch on
the screen keyboard and in tap .
This is entered in the menu.
Tick the selection box and then tap $to
produce the graph.
Note: The Zoom is set to Quick Initialize for the
graph shown.
The absolute value function can be composed
with other functions. The small graph shows
y = |x
2
4|.
The CAS calculator has the absolute value and
integer part functions as built-in features.
(absolute value function)
Turn on the screen keyboard and tap
then enter the number.
(integer part function)
This function is found in the catalogue. Tap and use the alphabetic shortcut at the
bottom of the screen to nd a function beginning with i. The required function is intg(.
This function returns the next whole number which is less than the given number.
P1: FXS/ABE P2: FXS
9780521740494c08.xml CUAU033-EVANS August 22, 2009 12:4
212 Essential Advanced General Mathematics
Using the Casio ClassPad
The CAS calculator has the absolute value
and integer part functions as built-in
features.
(absolute value function)
Turn on the screen keyboard and tap
then enter the number.
(integer part function)
This function is found in the catalog. Tap
and use the alphabetic shortcut at the bottom of the
screen to nd a function beginning with i. The
required function is intg(. This function returns the
next whole number which is less than the given
number.
Exercise 8H
1 Find the image of {(x, y) : y = |x|} and sketch the graph of the image for the following.
Example 15
a a translation corresponding to the vector
_
1
3
_
b a reection in the line y = x
c a dilation from the y axis of factor 4
d a reection in the x axis
e a translation corresponding to the vector
_
1
3
_
followed by a reection in the line y = x
f a reection in the line y = x then a translation corresponding to the vector
_
1
3
_
g a dilation from the x axis of factor 2 followed by a reection in the x axis.
2 State the transformation(s) which map(s)
a {(x, y) : y = |x|} to {(x, y) : y = |x| +3}
b {(x, y) : y = |x|} to {(x, y) : y = |x 3| +3}
c {(x, y) : y = |x|} to {(x, y) : y = |2x|}
d {(x, y) : y = |x|} to {(x, y) : y = 2|x|}
3 Find the image of {(x, y) : y = [x]} and sketch the graph of the image of the following.
a a dilation from the y axis of factor 2
P1: FXS/ABE P2: FXS
9780521740494c08.xml CUAU033-EVANS August 22, 2009 12:4
Chapter 8 Transformations 213
b a translation corresponding to the vector
_
2
0
_
c a translation corresponding to the vector
_
0
2
_
d a reection in the line y = x
e a reection in the line y = x
f a translation corresponding to the vector
_
4
0
_
g a dilation from the y axis of factor 2 followed by a reection in the x axis
h a reection in the x axis followed by a translation corresponding to the vector
_
0
2
_
.
8.9 Function notation with transformations
In this section the image of the graph y = f (x) (where f is an arbitrary function) under a
transformation is considered. This is particularly useful when using a graphics or CAS
calculator.
Example 16
Let y = f (x) be the equation of a curve.
Find the image of y = f (x) under each of the following transformations.
a a translation determined by the vector
_
h
k
_
b a reection in the x axis
c a dilation of factor k from the y axis d a dilation of factor k from the x axis
Solution
a The rule is (x, y) (x +h, y +k)
Consider x

= x +h and y

= y +k
x = x

h and y = y

k
y = f (x) is mapped to y

k = f (x

h)
i.e., the image is y = f (x h) +k
b The rule is (x, y) (x, y)
Consider x

= x and y

= y
i.e., the image is y = f (x)
c The rule is (x, y) (kx, y)
Consider x

= kx and y

= y
x =
x

k
and y = y

y = f (x) is mapped to y

= f
_
x

k
_
i.e., the image of y = f (x) is y = f
_
x
k
_
P1: FXS/ABE P2: FXS
9780521740494c08.xml CUAU033-EVANS August 22, 2009 12:4
214 Essential Advanced General Mathematics
d The rule is (x, y) (x, ky)
Consider x

= x and y

= ky
x = x

and y =
y

k
y = f (x) is mapped to
y

k
= f (x

)
i.e., the image of y = f (x) is y = k f (x)
Example 17
For f (x) = x
2
nd 4 f (x +3) and state the transformations which take
y = f (x) to y = 4 f (x +3)
Solution
4 f (x +3) = 4(x +3)
2
y = x
2
is mapped to y = 4(x +3)
2
If (x, y) is mapped to (x

, y

)
Then
y

4
= (x

+3)
2
x = x

+3 and y =
y

4
i.e., x

= x 3 and y

= 4y
a dilation of factor 4 from the x axis followed by a translation
_
3
0
_
takes
y = f (x) to y = 4 f (x +3)
Using the TI-Nspire
The notation for transformations can be
used with a calculator.
In the screen shown, the transformation
which takes the curve of y = f (x) to the
curve of y = f (x 5) is applied to the
curve with equation f (x) = x
2
by dening
f 1(x) = x
2
and f 2(x) = f 1(x 5).
P1: FXS/ABE P2: FXS
9780521740494c08.xml CUAU033-EVANS August 22, 2009 12:4
Chapter 8 Transformations 215
Consider the following sequence of
transformations:
dilation of factor 2 from the x-axis
translation of 3 to the left
translation 4 down
These transformations will take the curve
of y = f (x) to the curve of
y = 2 f (x +3) 4. In the screen shown,
this is applied to the curve with equation
f (x) = x
2
.
Using the Casio ClassPad
The notation for transformations can be
used with the CAS calculator.
Consider the transformation which takes
the curve of y = f (x) to the curve of
y = f (x 5). The function for y1 can be
entered and then y2 can be dened as
shown.
Note: The y must be selected from the
tab on the screen keyboard, not the y on the
physical keyboard.
A dilation factor
1
2
from the y-axis is
applied to the curve with rule f (x) = x
2
.
The rule has been entered as y3 = y1 (2x).
Consider the following sequence of transformations:
dilation of factor 2 from the x-axis
translation of 3 to the left
translation 4 down
These transformations will take the curve
of y = f (x) to the curve of
y = 2 f (x +3) 4. In the screen shown,
this is applied to the curve with equation
y = x
2
.
P1: FXS/ABE P2: FXS
9780521740494c08.xml CUAU033-EVANS August 22, 2009 12:4
216 Essential Advanced General Mathematics
Exercise 8I
1 Let y = f (x) be the equation of a curve. Find the image of y = f (x) under each of the
following transformations.
Example 16
a a translation determined by the vector
_
2
3
_
b a reection in the line y = x
c a dilation of factor
1
2
from the y axis d a dilation of factor 2 from the x axis
2 For f (x) = 2
x
, nd
Example 17
a f (x +3) b f (2x) c f
_
x
2
_
d 4 f (x 3)
3 For f (x) = x
2
, nd 4 f
_
x
2
_
and state the transformations which take
y = f (x) to y = 4 f
_
x
2
_
4 For f (x) =
1
x
nd f (2x 3) +4 and state the transformations which take
y = f (x) to y = f (2x 3) +4
5 For f (x) = x
2
nd 3 f (2 x) and state the transformations which take
y = f (x) to y = 3 f (2 x)
8.10 Summary of transformations
A summary of some of the transformations and their rules is presented here. Suppose (x

, y

) is
the image of (x, y) under the mapping in the rst column of the table below.
Mapping Rule
Reection in the x axis x

= x = x +0y
y

= y = 0x +y
Dilation by factor k from the y axis x

= kx = kx +0y
y

= y = 0x + y
Rotation of

c
2
about O in an anticlockwise direction x

= y = 0x +y
y

= x = x +0y
Expansion of factor k from the origin x

= kx = kx +0y
y

= ky = 0x +ky
Reection in the line y = x x

= y = 0x + y
y

= x = x +0y
Translation dened by a vector
_
a
b
_
x

= x +a
y

= y +b
P1: FXS/ABE P2: FXS
9780521740494c08.xml CUAU033-EVANS August 22, 2009 12:4
Chapter 8 Transformations 217
The rst ve mappings are special cases of a general kind of mapping dened by
x

= ax +by
y

= cx +dy
where a, b, c, d are real numbers.
These equations can be rewritten as
x

= a
11
x +a
12
y
y

= a
21
x +a
22
y
which yields the equivalent matrix equation
_
x

_
=
_
a
11
a
21
a
12
a
22
__
x
y
_
A transformation of the form
(x, y) (a
11
x +a
12
y, a
21
x +a
22
y)
is called a linear transformation.
Example 18
Consider a linear transformation such that (1, 0) (3, 1) and (0, 1) (2, 4). Find the
image of (3, 5).
Solution
_
a
11
a
21
a
12
a
22
__
1
0
_
=
_
3
1
_
and
_
a
11
a
21
a
12
a
22
__
0
1
_
=
_
2
4
_
a
11
= 3, a
21
= 1 and a
12
= 2, a
22
= 4
i.e., the transformation can be dened by the 2 2 matrix
_
3
1
2
4
_
Let (3, 5) (x

, y

_
x

_
=
_
3 2
1 4
__
3
5
_
=
_
3 3 +2 5
1 3 4 5
_
=
_
1
17
_
(3, 5) (1, 17)
The image of (3, 5) is (1, 17)
Note: Non-linear mappings cannot be represented by a matrix in the way indicated above.
P1: FXS/ABE P2: FXS
9780521740494c08.xml CUAU033-EVANS August 22, 2009 12:4
218 Essential Advanced General Mathematics
Thus for the translation dened by (0, 0) (a, b)
x

= x +a
y

= y +b
While this cannot be represented by a square matrix, the dening equations
suggest
_
x

_
=
_
x
y
_
+
_
a
b
_
where the sum has denition:
for each x, y, a, b in R,
_
x
y
_
+
_
a
b
_
=
_
x +a
y +b
_
Composition of mappings
Consider a linear transformation dened by the matrix A =
_
a
11
a
21
a
12
a
22
_
composed with a
linear transformation dened by the matrix B =
_
b
11
b
21
b
12
b
22
_
The composition consists of the transformation of A being applied rst and then the
transformation of B.
The matrix of the resulting composition is the product
BA =
_
b
11
a
11
+b
12
a
21
b
21
a
11
+b
22
a
21
b
11
a
12
+b
12
a
22
b
21
a
12
+b
22
a
22
_
Exercise 8J
1 Find
a
_
2
5
3
4
__
1
2
_
b
_
3
1
1
3
__
a
b
_
2 If a linear transformation is dened by the matrix A =
_
2
4
1
3
_
, nd the image of
(1, 0), (0, 1) and (3, 2) under this transformation.
Example 18
3 Find the images of (1, 0) and (1, 2) under the linear transformation whose matrix is
a
_
2
1
1
1
_
b
_
2
0
0
1
_
c
_
2
3
3
1
_
P1: FXS/ABE P2: FXS
9780521740494c08.xml CUAU033-EVANS August 22, 2009 12:4
Chapter 8 Transformations 219
4 a Find the matrix of the linear transformation which maps (1, 2) (4, 5) and
(3, 4) (18, 5).
b The images of two points are given for a linear transformation. Investigate whether this
is sufcient information to determine the matrix of the transformation.
c Find the matrix of the linear transformation such that (1, 0) (1, 1) and
(0, 1) (2, 2). What is the range of this transformation?
5 By nding the images of (1, 0) and (0, 1), write down the matrix of each of the following
transformations.
a reection in the line x = 0
b reection in the line y = x
c reection in the line y = x
d dilation of factor 2 from the x axis
e expansion of factor 3 from the origin
f dilation of factor 3 from the y axis
P1: FXS/ABE P2: FXS
9780521740494c08.xml CUAU033-EVANS August 22, 2009 12:4
R
e
v
i
e
w
220 Essential Advanced General Mathematics
Chapter summary
A transformation T is a mapping from R
2
to R
2
such that if
T(a, b) = T(c, d) then a = c and b = d
A translation is a transformation for which each point in the plane is moved the same
distance in the same direction. A translation of 5 units in the positive direction of the x axis
can be represented by the rule (x, y) (x +5, y)
A vector will mean a column of two numbers. The rst number indicates a move in the
positive or negative direction of the x axis and the second indicates a move in the positive
or negative direction of the y axis. Directed line segments are used to illustrate vectors.
For example, the vector
_
2
3
_
is the vector 2 to the right and 3 up.
The rule for the translation described by this vector is (x, y) (x +2, y +3)
For reection in the x axis the rule is (x, y) (x, y)
For reection in the y axis the rule is (x, y) (x, y)
For reection in the line y = x the rule is (x, y) (y, x)
Lengths and angles are preserved by reections and translations.
The transformation dilation from the y axis of factor k is dened by the rule
(x, y) (kx, y); k R
+
The transformation dilation from the x axis of factor k is dened by the rule
(x, y) (x, ky); k R
+
Lengths and angles are not preserved by dilations.
A point (a, b) is said to be invariant under a transformation if (a, b) (a, b) under that
transformation.
The composition of two transformations is dened by applying one transformation, and
then the second, to the images of the rst.
e.g., (x, y) (x +2, y +3) ((x +2), y +3)
translation reection
Applying transformations to graphs of functions
e.g., let (x, y) (2x +1, y +3)
Let (x

, y

) be the image of (x, y)


Then x

= 2x +1 and y

= y +3
and x =
x

1
2
and y = y

3
y = f (x) is mapped to y

3 = f
_
x

1
2
_
If f (x) = x
2
, y = f (x) is mapped to y =
_
x 1
2
_
2
+3
P1: FXS/ABE P2: FXS
9780521740494c08.xml CUAU033-EVANS August 22, 2009 12:4
R
e
v
i
e
w
Chapter 8 Transformations 221
Determining transformations
For example, if y = f
_
x +3
2
_
4
then y +4 = f
_
x +3
2
_
Let (x, y) (x

, y

)
Then y

+4 = y and
x

+3
2
= x
Hence y

= y 4 and x

= 2x 3
The transformation is given by the rule (x, y) (2x 3, y 4)
|x| =
_
x if x 0
x if x < 0
x
y
0
y = | x|
This is called the absolute value function.
The integer value function is dened by
I (x) = [x]
where [x] is the greatest integer not
exceeding x.
x
y
0
1
1
2
2 3 1
1
2
2
Function notation
The image of the graph of y = f (x) under the translation
(x, y) (x +h, y +k) is the graph of y = f (x h) +k
The image of the graph of y = f (x) under the reection
(x, y) (x, y) is the graph of y = f (x)
The image of the graph of y = f (x) under the reection
(x, y) (x, y) is the graph of y = f (x)
The image of the graph of y = f (x) under the dilation
(x, y) (kx, y) is the graph of y = f
_
x
k
_
P1: FXS/ABE P2: FXS
9780521740494c08.xml CUAU033-EVANS August 22, 2009 12:4
R
e
v
i
e
w
222 Essential Advanced General Mathematics
Multiple-choice questions
1
x
y
y = f(x)
0
2 2
2
2
4
4
4 6 4 6
x
y
y = g(x)
0
2 2
2
2
4
4
4 6 4 6
The transformation which maps the graph of y = f (x) to the graph of y = g(x) is
A rotation of 180

about the origin B rotation of 270

about the origin


C reection in the y axis D reection in the x axis
E reection in the line y = x
2 If the graph of y = g(x) is obtained by reecting the graph of y = f (x) in the x axis, the
equation relating f (x) and g(x) is
A f (x) = g(x) B f (x) = g(x) C f (x) = g(x)
D f (x) =
1
g(x)
E f (x) = 2g(x)
3 The translation that maps the graph of y = x
2
to the graph of y = (x 5)
2
2 can be
described as
A 2 units in the negative direction of the x axis and 5 units in the positive direction of the
y axis
B 2 units in the positive direction of the x axis and 5 units in the positive direction of the
y axis
C 2 units in the negative direction of the x axis and 5 units in the negative direction of the
y axis
D 5 units in the positive direction of the x axis and 2 units in the negative direction of the
y axis
E 5 units in the negative direction of the x axis and 2 units in the negative direction of the
y axis
4 The translation that maps the graph of f (x) = (x +2)
2
+8 to the graph of g(x) = x
2
is
A 8 units in the negative direction of the x axis and 2 units in the positive direction of the
y axis
B 2 units in the positive direction of the x axis and 8 units in the positive direction of the
y axis
C 2 units in the negative direction of the x axis and 8 units in the positive direction of the
y axis
P1: FXS/ABE P2: FXS
9780521740494c08.xml CUAU033-EVANS August 22, 2009 12:4
R
e
v
i
e
w
Chapter 8 Transformations 223
D 2 units in the positive direction of the x axis and 8 units in the negative direction of the
y axis
E 2 units in the negative direction of the x axis and 8 units in the negative direction of the
y axis
5 If y = f (x) =
1
x
2
, the graph of y = 3 f (x) is as shown in
A
y = 3f(x) y = f(x)
0
x
y
B
y = 3f(x) y = f(x)
0
x
y
C
y = f(x)
y = 3f(x)
0
x
y
D
y = f(x) y = 3f(x)
0
x
y
E
y = f(x) y = 3f(x)
0
x
y
6 The equation of the image of the graph of y =

x after a reection in the y axis followed


by a translation of 2 units in the positive direction of the x axis and 3 units in the positive
direction of the y axis is
A y =

x 2 3 B y =

x 2 3 C y =

x +2 3
D y =

x 2 +3 E y =

x +2 +3
7 The equation of the image of the graph of y =
1
x
2
after a reection in the x axis followed
by a dilation of factor 2 from the x axis is
A y =
2
x
2
B y =
1
2x
2
C y =
2
x
2
D y =
1
2x
2
E y =
2
(x)
2
8 The equation of the image of the graph of y = |x| after a dilation of factor
1
3
from the
y axis followed by a translation of 5 units in the negative direction of the x axis and 2 units
in the positive direction of the y axis is
A y = 3|x 5| +2 B y = 3|x +5| +2 C y =

x 5
3

+2
D y =

x +5
3

+2
E y =

x +15
3

+2
9 The value of [4.6] +[7.2] +[8.7] is
A 9 B 10 C 8 D 13 E 11
10 The equation of the image of the graph of y = [x] after a reection in the x axis followed
by a translation of 5 units in the positive direction of the x axis and 4 units in the negative
direction of the y axis is
A y = [x] +1 B y = [x 5] 4 C y = [x +5] 4
D y = [x] +9 E y = [x +5] +4
P1: FXS/ABE P2: FXS
9780521740494c08.xml CUAU033-EVANS August 22, 2009 12:4
R
e
v
i
e
w
224 Essential Advanced General Mathematics
Short-answer questions (technology-free)
1 Find the image of the point (3, 1) under each of the following transformations.
a a dilation of factor 3 from the y axis b a dilation of factor 2 from the x axis
c a translation determined by the vector
_
3
2
_
d a reection in the line x = 0
e a reection in the line y = 0 f a reection in the line y = x
2 Find the equation of the image of the graph of y = x
2
under each of the following
transformations.
a a translation determined by the vector
_
2
3
_
b a reection in the line y = x
c a reection in the line y = 0
3 Find the rule for each of the following compositions of transformations.
a a reection in the line y = x followed by a translation determined by the vector
_
2
3
_
b a dilation of factor 5 from the x axis followed by a reection in the x axis
c a dilation of factor 4 from the y axis followed by a translation determined by the
vector
_
2
3
_
d a translation determined by the vector
_
2
3
_
followed by a dilation of factor 4 from the
x axis
4 Find the image of y = 2x 1 under each of the transformations dened in 1.
5 Find the image of y = 2 x
2
under each of the transformations dened in 3.
6 Sketch the graph of each of the following. Indicate the coordinates of the vertex and at least
one other point.
a y = |x| b y = |2x| +3 c y = 4 |2x|
7 Give the rule for the transformations which would map
a y =
1
x
to y =
2
x 3
+4 b y = x
2
to y = 2(x 4)
2
+3
c y = |x| to y = |3x| 4 d y =
1
x
to y =
x +1
x 1
e y = x
2
to y = (x 2)
2
+3
8 Sketch the graph of each of the following using transformations.
a x
2
= y 1 b (x 1)
2
= y +2 c (x 2)
2
= 3(y 2)
d y = [4x] e y = 3|2x 1| f y = 2 3|x 2|
Extended-response questions
1 a Find the image of the point with coordinates (2, 6) under reection in the line x = 3.
b Find the rule for the following sequence of transformations.
P1: FXS/ABE P2: FXS
9780521740494c08.xml CUAU033-EVANS August 22, 2009 12:4
R
e
v
i
e
w
Chapter 8 Transformations 225
r
a translation determined by the vector
_
3
0
_
r
a reection in the y axis
r
a translation determined by the vector
_
3
0
_
c Hence give the rule for the transformation, reection in the line x = 3.
d i Give a sequence of three transformations which would determine a reection in the
line x = m.
ii Give the rule for the transformation, reection in the line x = m.
e i Give a sequence of three transformations which would determine a reection in the
line y = n.
ii Give the rule for the transformation, reection in the line y = n.
f Find the image of each of the relations under the transformation, reection in the line
x = 3.
i y = x 3 ii y = x iii y = x
2
iv y = (x 3)
2
2 a Find the coordinates of the image of the point A(3, 1)
under a rotation of 90

in an anticlockwise
direction around the origin.
x
y
0
A'
A(3, 1)
b i State the gradient of line OA.
ii State the gradient of line OA

.
c A point A has coordinates (p, q).
i State the gradient of line OA.
ii The point is rotated about the origin by 90

in an anticlockwise direction. Find the


coordinates of A

, the image of A.
d Find the rule for the transformation rotation about the origin by 90

in an anticlockwise
direction.
e Find the equation of the image of each of the following curves under this
transformation.
i y = x ii y = x
2
iii x
2
+ y
2
= 1 iv y =
1
x
3 a Find the image of the point A(1, 3) under a rotation of 180

about the origin.


b Find the image of the point (a, b) under a rotation of 180

about the origin.


c If the curve with equation y = f (x) is rotated 180

about the origin, nd the rule for its


image in terms of f.
d i Find the rule for the following sequence of transformations.
r
a translation determined by the vector
_
3
0
_
r
a rotation of 180

about the origin


r
a translation determined by the vector
_
3
0
_
ii Find the image of the line with equation y = 3x +1 under the transformation with
rule determined in d i. (contd.)
P1: FXS/ABE P2: FXS
9780521740494c08.xml CUAU033-EVANS August 22, 2009 12:4
R
e
v
i
e
w
226 Essential Advanced General Mathematics
e Find the rule for rotation of 180

in an anticlockwise direction about the point (m, n).


f Find the rule for rotation of 90

in a clockwise direction about the point (m, n).


g Find the rule for rotation of 90

in an anticlockwise direction about the point (m, n).


h i Find the image of the curve with rule y = x
2
after a rotation of 90

in a clockwise
direction about the point (0, 1).
ii Sketch the graph of the curves on one set of axes.
4 a i Find the dilation from the x axis which takes y = x
2
to the parabola with vertex at
the origin and which passes through the point (25, 15).
ii State the rule which reects this dilated parabola in the x axis.
iii State the rule which takes the reected parabola of ii to a parabola with x intercepts
(0, 0) and (50, 0) and vertex (25, 15).
iv State the rule which takes the curve of y = x
2
to the parabola dened in iii.
b The plans for the entrance of a new building involve twin parabolic arches as shown in
the diagram.
x
y
50 m 50 m
15 m
Arch 1 Arch 2
0
i From the results of a give the equation for the curve of Arch 1.
ii Find the translation which maps the curve of Arch 1 to the curve of Arch 2.
iii Find the equation of the curve of Arch 2.
c The architect wishes to have exibility in his planning and so wants to develop an
algorithm for determining the equations of the curves given arch width m metres and
height n metres.
i Find the rule for the transformation which takes y = x
2
to Arch 1 with these
dimensions.
ii Find the equation for the curve of Arch 1.
iii Find the equation for the curve of Arch 2.
5 a Let f (x) = [x]
i Find the equation of the image of the graph of y = f (x) under the transformation
with rule (x, y) (2x 1, y +3)
ii Sketch the graph of the image of y = f (x) under this transformation.
iii Describe a sequence of transformations which takes the graph of
y = f (x) to y = f (x 3) 2
b Let g(x) = |x|
i Sketch the graph of y = g(x 3) +3 and the graph of y = g(2x 1)
ii Solve the equation g(x 3) +3 = g(2x 1)
P1: FXS/ABE P2: FXS
9780521740494c09.xml CUAU033-EVANS October 5, 2008 7:1
C H A P T E R
9
Ratios and similarity
Objectives
To divide a quantity in a given ratio
To determine the ratio in which a quantity has been divided
To apply the transformations which are expansions from the origin
To define similarity of two figures
To determine when two triangles are similar by using the conditions
r
equal angles (AAA)
r
equal ratios (PPP)
r
corresponding sides having the same ratio and the included angle equal (PAP)
To apply similarity to solve problems
To determine and apply expansion factors for areas and volumes
9.1 Ratios
This section is revision of work of previous years. Several examples are presented.
Example 1
Divide 300 in the ratio 3 : 2.
Solution
one part = 300 5 = 60
two parts = 60 2 = 120
three parts = 60 3 = 180
Example 2
Divide 3000 in the ratio 3 : 2 : 1.
227
P1: FXS/ABE P2: FXS
9780521740494c09.xml CUAU033-EVANS October 5, 2008 7:1
228 Essential Advanced General Mathematics
Solution
one part = 3000 6 = 500
two parts = 500 2 = 1000
three parts = 500 3 = 1500
Example 3
A day is divided into 10 new-hours, each new-hour is divided into 100 new-minutes and each
new-minute is divided into 100 new-seconds. What is the ratio of a new-second to an ordinary
second?
Solution
There are 10 10
2
10
2
new-seconds in a day
and 24 60 60 ordinary seconds in a day
the ratio of new-seconds : ordinary seconds
=
1
10
5
:
1
8.64 10
4
= 864 : 1000
= 108 : 125
Example 4
Two positive integers are in the ratio 2 : 5. If the product of the integers is 40 nd the larger
integer.
Solution
Let a and b denote the integers
a
b
=
2
5
. . . 1
and ab = 40 . . . 2
From 1 a =
2
5
b Substitute in 2

2
5
b
2
= 40
b
2
= 100
b = 10
and as b is a positive integer, b = 10 and a = 4
The larger integer is 10.
Exercise 9A
1 Divide 9000 in the ratio 2 : 7.
Example 1
2 Divide 15 000 in the ratio 2 : 2 : 1.
Example 2
P1: FXS/ABE P2: FXS
9780521740494c09.xml CUAU033-EVANS October 5, 2008 7:1
Chapter 9 Ratios and similarity 229
3 x : 6 = 9 : 15. Find x.
4 The ratio of the numbers of orange owers to pink owers in a garden is 6 : 11. There are
144 orange owers. How many pink owers are there?
5 15 : 2 = x : 3. Find x.
6 The angles of a triangle are in the ratio 6 : 5 : 7. Find the sizes of the three angles.
7 Three men X, Y and Z share an amount of money in the ratio 2 : 3 : 7. If Y receives $2
more than X, how much does Z obtain?
8 An alloy consists of copper, zinc and tin in the ratios 1 : 3 : 4 (by weight). If there is 10 g
of copper in the alloy, nd the weights of zinc and tin.
9 In a bag the ratio of red beads to white beads to green beads is 7 : 2 : 1. If there are 56 red
beads, how many white beads and how many green beads are there?
10 On a map the length of a road is represented by 45 mm. If the scale is 1 : 125 000, nd the
actual length of the road.
11 Five thousand two hundred dollars was divided between a mother and daughter in the
ratio 8 : 5. Find the difference between the sums they received.
12 Points A, B, C and D are placed in that order on a line so that AB = 2BC = CD. Express
BD as a fraction of AD.
13 If the radius of a circle is increased by two units, nd the ratio of the new circumference
to the new diameter.
14 In a class of 30 students the ratio of boys to girls is 2 : 3. If six boys join the class, nd the
new ratio of boys to girls in the class.
15 If a : b = 3 : 4 and a : (b +c) = 2 : 5, nd the ratio a : c.
16 The scale of a map reads 1 : 250 000. Find the distance, in kilometres, between two towns
which are 3.5 cm apart on the map.
9.2 An introduction to similarity
The two triangles ABC and A

are similar.
Note: OA

= 2OA, OB

= 2OB, OC

= 2OC.
2
0
2
4
6
8
10
12
A'(4, 12)
B'(10, 6)
C'(8, 2)
A(2, 6)
C(4, 1)
4 6 8 10 12
B(5, 3)
Triangle A

can be considered as the image of


triangle ABC under a mapping of the plane in which
the coordinates are multiplied by 2.
This mapping is called an expansion from the origin
of factor 2.
This can be written in transformation notation:
(x, y) (2x, 2y).
P1: FXS/ABE P2: FXS
9780521740494c09.xml CUAU033-EVANS October 5, 2008 7:1
230 Essential Advanced General Mathematics
There is also a mapping from A

to ABC which is an expansion from the origin of


factor
1
2
.
The rule for this is (x, y)

1
2
x,
1
2
y

.
Two gures are similar if one is congruent to an image of the other under an expansion
from the origin of factor k.
For example, the rectangle of side lengths 1 and 2 is similar
to the rectangle with side lengths 3 and 6.
Note here the expansion factor is 3 and the rule is
(x, y) (3x, 3y).
0
1
1
2
3
4
5
6
7
8
9
(3, 9) (6, 9)
(6, 3) (3, 3)
(2, 3) (1, 3)
(1, 1) (2, 1)
2 3 4 5 6
Note:
any two circles are similar
any two squares are similar
any two equilateral triangles are similar
For a triangle ABC with side lengths a, b, c and a similar
triangle A

with corresponding side lengths a

, b

, c

it can be seen that


a

a
=
b

b
=
c

c
= k
where k is the appropriate expansion factor.
Similar statements can be made about other pairs of similar polygons.
Note also that the measure of an angle does not change under an expansion: i.e., for two
similar gures, corresponding angles are equal.
Similar triangles
Two triangles are similar if one of the following conditions holds:
triangles have equal angles (AAA)
45
100
35
C
B
A
A'
B'
C'
35
100
45
corresponding sides are in the same ratio (PPP)
A

AB
=
B

BC
=
A

AC
= k, where k is the expansion (enlargement) factor
two pairs of corresponding sides have the same ratio and the included angles are equal,
(PAP)
C A
B
45
A'
B'
C'
45
A

AB
=
A

AC
P1: FXS/ABE P2: FXS
9780521740494c09.xml CUAU033-EVANS October 5, 2008 7:1
Chapter 9 Ratios and similarity 231
two pairs of corresponding sides have the same ratio and two corresponding non-included
angles are equal, provided these angles are right angles or obtuse.
A
B
C
120
B'
C'
120
A'
Triangle ABC is similar to triangle A

can be written symbolically as


ABC A

The triangles are named so that angles of equal magnitude hold the same position i.e., A
corresponds to A

, B corresponds to B

, C corresponds to C

.
i.e.
BC
B

=
AC
A

or
BC
B

=
BA
B

Example 5
a Give the reason for triangle ABC being
similar to triangle A

.
b Find the value of x.
C'
B'
6.25 cm
3.75 cm
3.013 cm
20
A'
20
3 cm
A
5 cm
B
C
x cm
Solution
a Triangle ABC is similar to triangle
A

as
5
6.25
=
3
3.75
= 0.8
and the magnitude of ABC = magnitude of A

= 20

PAP is the condition for similarity.


b
x
3.013
=
5
6.25
x =
5
6.25
3.013
= 2.4104
Example 6
a Give the reason for triangle ABC being similar to
triangle AXY.
b Find the value of x.
A
3 cm 2.5 cm
Y
X
6 cm
x cm
C
B
P1: FXS/ABE P2: FXS
9780521740494c09.xml CUAU033-EVANS October 5, 2008 7:1
232 Essential Advanced General Mathematics
Solution
a Corresponding angles are of equal magnitude (AAA).
b AB
AX
=
AC
AY
i.e.,
x
x +6
=
3
5.5
5.5x = 3(x +6)
2.5x = 18
x = 7.2
Exercise 9B
1 Give reasons why the following pairs of triangles are similar and nd the value of x in
Example 5
each case.
a
56
A
5 cm
C B
4 cm
82
9 cm
A'
B'
x cm
C'
b
A
12 cm
C
10 cm
D
E
13 cm
14 cm
B
x cm

c
2 cm
x cm
D 4 cm B 6 cm A
C
E

d C
B
10 cm
A
x cm

6 cm
Q
8 cm
P
R

P1: FXS/ABE P2: FXS


9780521740494c09.xml CUAU033-EVANS October 5, 2008 7:1
Chapter 9 Ratios and similarity 233
2 Give reasons why the following pairs of triangles are similar and nd the value of x in
Example 6
each case.
a
12 cm
x cm
16 cm
8 cm
A
Q
C
P
B
b
2 cm
3 cm 2 cm
x cm
A
E
C
D
B
c
2 cm x cm
6 cm 8 cm
A
Q
C
P
B
d
1.5 cm
x cm
2 cm 10 cm
C
B
E
D
A
3 Given that AD = 14, ED = 12, BC = 15 and
EB = 4, nd AC, AE and AB.
C
B
E
A
D
14
12
4
15
4 A tree casts a shadow of 33 m and at the same time a stick 30 cm long casts a shadow
224 cm long. How high is the tree?
tree
0.3 m
5 A 20 m high neon sign is supported by a 40 m
steel cable as shown. An ant crawls along the
cable starting at A. How high is the ant when
it is 15 m from A?
40 m
A
20 m
6 A hill has gradient of 1 in 20, i.e. for every 20 m horizontally there is a 1 m increase in
height. If you go 300 m horizontally, how high up will you be?
P1: FXS/ABE P2: FXS
9780521740494c09.xml CUAU033-EVANS October 5, 2008 7:1
234 Essential Advanced General Mathematics
7 A man stands at A and looks at point Y across the river. He gets a friend to place a stone at
X so that A, X and Y are collinear. He then measures AB, BX and XC to be 15 m, 30 m and
45 m respectively. Find CY, the distance across the river.
Y
C X B
A
15 m
45 m
30 m
8 Find the height, h m, of a tree that casts a shadow 32 m long at the same time that a
vertical straight stick 2 m long casts a shadow 6.2 m long.
9 A plank is placed straight up stairs that are 20 cm wide
and 12 cm deep. Find x, where x cm is the width of the
widest rectangular box of height 8 cm that can be placed
on a stair under the plank.
12 cm
20 cm
8 cm
x cm p
l
a
n
k
10 The sloping edge of a technical drawing table is 1 m
from front to back. Calculate the height above the
ground of a point A, which is 30 cm from the front
edge.
92 cm
80 cm
A
3
0
c
m
1
m
11 Two similar rods 1.3 m long have to be hinged together
to support a table 1.5 m wide. The rods have been xed
to the oor 0.8 m apart. Find the position of the hinge
by nding the value of x.
1.5 m
0.8 m
(1.3 x) m
x m
12 A man whose eye is 1.7 m from the ground, when standing 3.5 m in front of a wall 3 m
high, can just see the top of a tower that is 100 m away from the wall. Find the height of
the tower.
13 A man is 8 m up a 10 m ladder, the top of which leans against a vertical wall and touches
it at a height of 9 m above the ground. Find the height of the man above the ground.
14 A spotlight is at a height of 0.6 m above ground
level. A vertical post 1.1 m high stands 3 m away
and 5 m further away there is a vertical wall. How
high up the wall does the shadow reach?
spotlight
0.6 m
wall
vertical post
1.1 m
3 m 5 m
P1: FXS/ABE P2: FXS
9780521740494c09.xml CUAU033-EVANS October 5, 2008 7:1
Chapter 9 Ratios and similarity 235
15 Measurements in the diagram shown are in cm.
a Prove that ABC EDC.
b Find x.
c Use Pythagoras theorem to nd y and z.
d Verify y : z = ED : AB.
5
4
2
A
B
x
z
y
D
C
E
16 Find a.
10
7
12
B'
C'
C
a
AA'
B
17 A man who is 1.8 m tall casts a shadow of 0.76 m in length. If at the same time a
telephone pole casts a 3 m shadow, nd the height of the pole.
18 In the diagram shown, RT = 4 cm, ST = 10 cm.
Find the length NT.
S
N
T
R
19 ABC is a triangular frame with AB = 14 m, BC = 10 m, CA = 7 m. A point P on AB,
1.5 m from A, is linked by a rod to a point Q on AC, 3 m from A. Calculate the length
of PQ.
20 Using this diagram, nd a, x and y.
4
6
x
y
a
9.3 Areas, volumes and similarity
If two shapes are similar and the expansion (enlargement) factor is k, i.e., for any length AB of
one shape, the length of the corresponding length A

of the similar shape has length kAB,


then the
area of similar shape = k
2
area of the original shape
For two triangles ABC and A

which are similar, i.e., ABC A

with
A

= kAB,
area of triangle A

= k
2
area of triangle ABC
A C
D
B
c
b
h
a
A' C'
D'
B'
c'
b'
h'
a'
P1: FXS/ABE P2: FXS
9780521740494c09.xml CUAU033-EVANS October 5, 2008 7:1
236 Essential Advanced General Mathematics
This can be shown by observing that ABC A

and
area of triangle A

=
1
2
b

=
1
2
kb kh,
(where AC = b and A

= b

)
= k
2

1
2
bh

= k
2
area of triangle ABC
Some examples of similar shapes and the ratio of their areas are considered in the following.
3 cm
Area = .3
2
Similar circles
Scale factor =
4
3 4 cm
Area = .4
2
Ratio of areas =
.4
2
.3
2
=
4
2
3
2
=

4
3

2
3 cm
2 cm
Area = 6 cm
2
Similar rectangles
Scale factor = 2
6 cm
4 cm
Area = 24 cm
2
Ratio of areas =
24
6
= 4 = 2
2
4 cm
5 cm
3 cm
8 cm
6 cm
10 cm
Area = 6 cm
2
Area = 24 cm
2
Similar triangles
Scale factor = 2
Ratio of areas =
24
6
= 4 = 2
2
Example 7
The two rectangles shown below are similar. The area of rectangle ABCD is 20 cm
2
. Find the
area of rectangle A

.
P1: FXS/ABE P2: FXS
9780521740494c09.xml CUAU033-EVANS October 5, 2008 7:1
Chapter 9 Ratios and similarity 237
3 cm
B C
D A
5 cm
B' C'
D' A'
Solution
The ratio of the length of their bases =
A

AB
=
5
3
The ratio of their areas =
Area of A

Area of ABCD
=

5
3

2
=
25
9
Area of A

=
25
9
20 cm
2
= 55
5
9
cm
2
Two solids are considered to be similar if they have the same shape and the ratio of their
corresponding linear dimensions are equal.
E H
G
C
B
A D
F
3 cm
2 cm
1 cm
E'
H'
G'
C' B'
A'
D'
F'
7.5 cm
5 cm
2.5 cm
Scale factor = 2.5
The cuboids ABCDEFGH and A

are similar.
For similar solids, if the scale factor is k then the
volume of the similar solid = k
3
volume of the original solid
For example,
Volume of ABCDEFGH = (3 2 1) cm
3
= 6 cm
3
Volume of A

= (5 2.5 7.5) cm
3
= 93.75 cm
3
The ratio of volumes =
93.75
6
= 15.625 = 2.5
3
P1: FXS/ABE P2: FXS
9780521740494c09.xml CUAU033-EVANS October 5, 2008 7:1
238 Essential Advanced General Mathematics
Here is another example.
3 cm
3

c
m
3 cm
A C
V
B
5 cm
5

c
m
5 cm
A'
C'
V'
B'
Scale factor =
5
3
Ratio of volumes =
5
3
3
3
=

5
3

3
Example 8
The two square pyramids are similar. VO = 9 cm.
4 cm
9 cm
A
D
B
O
V
C
5 cm
D'
A'
B'
O'
V'
C'
a Find the ratio of the length of their bases, and hence the height, V

, of the pyramid
V

.
b The volume of VABCD is 48 cm
3
. Find the ratio of their volumes, and hence nd the
volume of V

.
Solution
a The ratio of the length of their bases =
C

CD
=
5
4
V

=
5
4
9
=
45
4
The length of V

is 11.25 cm.
b The volume of VABCD is 48 cm
3
The ratio of their volumes =
Volume of V

Volume of VABCD
=

5
4

3
=
125
64
Volume of V

=
125
64
48 cm
3
= 93.75 cm
3
P1: FXS/ABE P2: FXS
9780521740494c09.xml CUAU033-EVANS October 5, 2008 7:1
Chapter 9 Ratios and similarity 239
Exercise 9C
1 These four rectangles are similar.
a Write down the ratio of the lengths of their bases.
b By counting rectangles, write down the ratio of their areas.
c Is there a relationship between these two ratios?
2 These four parallelograms are similar.
a Write down the ratio of the lengths of their bases.
b By counting parallelograms, write down the ratio of their areas.
c Is there a relationship between these two ratios?
3 The two rectangles shown are similar. The area of rectangle ABCD is 7 cm
2
.
Example 7
B
A D
C
3 cm
B'
A'
C'
D'
5 cm
Find the area of rectangle A

.
4 Triangle ABC is similar to triangle XYZ.
XY
AB
=
YZ
BC
=
ZX
CA
= 2.1
The area of triangle XYZ is 20 cm
2
. Find the area of triangle ABC.
5 Triangles ABC and A

are equilateral
triangles.
2 cm
2 cm
2 cm
A C
F
B
2 cm
A' C'
F'
B'
a cm
a Find the length of BF. b Find a.
c Find the ratio
Area of triangle A

Area of triangle ABC


P1: FXS/ABE P2: FXS
9780521740494c09.xml CUAU033-EVANS October 5, 2008 7:1
240 Essential Advanced General Mathematics
6 The areas of two similar triangles are 16 and 25. What is the ratio of a pair of
corresponding sides?
7 The areas of two similar triangles are 144 and 81. If the base of the large triangle is 30,
what is the corresponding base of the smaller triangle?
8 These three solids are similar.
C B A
a Write down the ratio of the lengths of the
bases.
b Write down the ratio of the lengths of the
heights.
c By counting cuboids equal in shape and size
to the cuboid given in A, write down the ratio of the volumes.
d Is there a relationship between the answers to a, b and c?
9 These are two similar rectangular blocks.
8 cm
4 cm
3 cm
12 cm
6 cm
cm
4
1
2
a Write down the ratio of their
i longest edges ii depths iii heights.
b By counting cubes of side 1 cm, write down the ratio of their volumes.
c Is there any relationship between the ratios in a and b?
10 These three solids are spheres.
Example 8
5cm
2cm
3cm
a Write down the ratio of the radii of the three spheres.
b The volume of a sphere of radius r is given by the formula
V =
4
3
r
3
. Express the volume of each sphere as a multiple
of .
Hence write down the ratio of their volumes.
c Is there any relationship between the ratios found in a and b?
In 11 to 20, objects referred to in the same question are
mathematically similar.
11 The sides of two cubes are in the ratio 2 : 1. What is the ratio of their volumes?
12 The radii of two spheres are in the ratio 3 : 4. What is the ratio of their volumes?
13 Two regular tetrahedrons have volumes in the ratio 8 : 27. What is the ratio of their sides?
P1: FXS/ABE P2: FXS
9780521740494c09.xml CUAU033-EVANS October 5, 2008 7:1
Chapter 9 Ratios and similarity 241
14 Two right cones have volumes in the ratio 64 : 27. What is the ratio of
a their heights b their base radii?
15 Two similar bottles are such that one is twice as high as the other. What is the ratio of
a their surface areas b their capacities?
16 Each linear dimension of a model car is
1
10
of the corresponding car dimension.
Find the ratio of
a the areas of their windscreens b the capacities of their boots
c the widths of the cars d the number of wheels they have.
17 Three similar jugs have heights 8 cm, 12 cm and 16 cm. If the smallest jug holds
1
2
litre,
nd the capacities of the other two.
18 Three similar drinking glasses have heights 7.5 cm, 9 cm and 10.5 cm. If the tallest glass
holds 343 millilitres, nd the capacities of the other two.
19 A toy manufacturer produces model cars which are similar in every way to the actual cars.
If the ratio of the door area of the model to the door area of the car is 1 : 2500, nd
a the ratio of their lengths b the ratio of the capacities of their petrol tanks
c the width of the model, if the actual car is 150 cm wide
d the area of the rear window of the actual car if the area of the rear window of the
model is 3 cm
2
.
20 The ratio of the areas of two similar labels on two similar jars of coffee is 144 : 169. Find
the ratio of
a the heights of the two jars b their capacities.
21 a In the gure, if M is the midpoint of AF and K is the midpoint
of AB, the area of ABF is how many times as great as the
area of AKM?
b If the area of ABF is 15, nd the area of AKM.
F
M
A
K
B
22 In the diagram, ABC is equilateral. BDE = CAF
and D is the midpoint of AC. Find the ratio of
area of BDE : area ofACF.
B
E
C
D
A
F
P1: FXS/ABE P2: FXS
9780521740494c09.xml CUAU033-EVANS October 5, 2008 7:1
242 Essential Advanced General Mathematics
23 The areas of two similar triangles are 144 cm
2
and 81 cm
2
. If the length of one side of the
rst triangle is 6 cm, what is the length of the corresponding side of the second?
9.4 Geometric representation of
arithmetic operations
Simple arithmetic operations correspond to elementary geometrical constructions. In many
cases the validity of these constructions can be established through similar triangles. If two
segments are given with lengths a and b (as measured by a given unit segment) then
a +b, a b, ra (where r is any rational number), ab,
a
b
and

a can be constructed.
a
b
Construction of a + b
Draw a straight line and mark off with a compass, as
shown in the diagram, the distance OA and AB where
OA = a and AB = b. Then OB = a +b.
a + b
A B O
Construction of a -- b
Draw a straight line and mark off with a compass the
distance OA and AB where OA = a and AB = b, but
this time AB is constructed in the other direction. Then
OB = a b.
a b
b
B O A
a
Construction of ra
To construct 3a = a +a +a, three copies of the line segment of length a are constructed.
For na = a +a + +a, where n is a natural number, n copies of the line segment of length
a are constructed.
Construction of ab
Mark off line segments OA and OB of length a units and
b units respectively. Construct OC of length 1 unit. Join
points C and B and draw a line parallel to the line CB
through A. The line segment OD has length ab.
Note that triangle OAD is similar to triangle OCB and
OA = aOC. Therefore OD = aOB = ab.
C 1
a
A
O
b
B
ab
D
P1: FXS/ABE P2: FXS
9780521740494c09.xml CUAU033-EVANS October 5, 2008 7:1
Chapter 9 Ratios and similarity 243
Construction of
1
b
This will be done for b = 5.
Line segment AB is of unit length. Draw any line
AX. Choose a line segment AC and then replicate
this line segment four times to form line segments
CC

, C

, C

and C

D. Draw line segment


DB and then parallel line segments CY, C

, C

and C

to divide line segment AB into ve equal


segments. Each of these segments has length
1
5
of a
unit.
A
C
C
C
C
D
X
Y Y' Y" Y"'
B
'
''
'''
Note that triangle ACY is similar to triangle ADB. And 5AC = AD. Hence AB = 5AY
Construction of
a
b
One way of constructing
a
b
is to mark off line
segments OA and OB of length a units and b units
respectively. Construct OD of length 1 unit. Join
points A and B and draw a line parallel to the line
AB through D. The line segment OC has length
a
b
a
A
O
B
b
D
C
1
a
b
Note that triangle OAB is similar to triangle OCD and
OB = bOD. Therefore OA = bOC and this implies OC =
a
b
.
Construction of

a
Construct line segments of length a and 1, and a circle of diameter a +1. In the diagram
OA = a and AB = 1. Angle ODB is a right angle (right angle subtended at the circle by a
diameter), and OAD is a right angle by construction.
Therefore triangle ODB is similar to triangle OAD and to triangle DAB.
Hence
OA
AD
=
AD
AB
D
B
A
O
a
AD
=
AD
1
Therefore AD
2
= a
and hence AD =

a
Exercise 9D
1 Construct a line segment of length

3 units.
2 Construct a line segment of length

5 units.
P1: FXS/ABE P2: FXS
9780521740494c09.xml CUAU033-EVANS October 5, 2008 7:1
244 Essential Advanced General Mathematics
3 Draw a line segment of length 10 cm and use a construction described above to divide it
into three equal intervals.
4 Draw a line segment of length 20 cm and use a construction described above to divide it
into nine equal intervals.
5 Draw two line segments OA and OB of lengths 4 cm and 14 cm respectively. Use a
construction described above to construct a line segment of length
2
7
units.
6 Draw two line segments OA and OB of lengths 9 cm and 13 cm respectively. Use a
construction described above to construct a line segment of length
9
13
units.
7 Describe the method for constructing a line of length
10
3
units.
8 Illustrate the construction of a line segment of length 3 4 units, given line segments of
length 3 units, 4 units and 1 unit.
9.5 Golden ratio
If
a
b
=
c
a
then a is said to be the geometric mean of c and b (or sometimes the mean
proportional of b and c).
Let AB be a line segment length a units and C a point on AB such that
AB
AC
=
AC
CB
.
Let AC = x. Therefore CB = a x
A C B
a x x
The relation
a
x
=
x
a x
holds. AC is the geometric mean of AB and CB.
If
a
x
=
x
a x
then a(a x) = x
2
Which implies that x
2
+ax a
2
= 0
Therefore using the general quadratic formula x =
a

a
2
4 1 a
2
2 1
=
a

5a
2
2 1
x = a
1

5
2
Only one of these is possible as AC is a length. Thus AC = a
1 +

5
2
(which is positive)
Therefore
AB
AC
= a
(1 +

5)a
2
=
2
1 +

5
=
2
1 +

5

1 +

5
1 +

5
=
1 +

5
2
P1: FXS/ABE P2: FXS
9780521740494c09.xml CUAU033-EVANS October 5, 2008 7:1
Chapter 9 Ratios and similarity 245
Hence the ratio
AB
AC
is independent of the length of AB and is always the same number. This
number is known as the golden ratio or section and is denoted by , i.e., =
1 +

5
2
is the only number which when diminished by one becomes its own reciprocal,
i.e., 1 =
1

This is shown as 1 =
1 +

5
2
1
=
1 +

5
2
=
1 +

5
2

1 +

5
1 +

5
=
2
1 +

5
A construction of the golden ratio is as follows.
Let AB be a segment of unit length. Draw BD of
length
AB
2
, perpendicular to AB. Draw line segment AD.
With centre D draw an arc of radius DB cutting AD
at E. Draw an arc of radius AE with centre A cutting
AB at C.
AB
AC
=
1
x
=
A
x
x
E
D
C 1 x B
The golden rectangle
The rectangle ABHF shown is known as the golden rectangle.
A
E
B
F
G
H
L K
1
1

1
The ratio of the side lengths AB : AF = 1 + :
and
1 +

1 +
1 +

5
2

1 +

5
2

=
3 +

5
2

2
1 +

5
=
3 +

5 +1
=
2 +2

5
4
=
1 +

5
2
=
That is, the ratio of the side lengths is .
This rectangle has some very pleasant properties, as observed in the following explorations.
P1: FXS/ABE P2: FXS
9780521740494c09.xml CUAU033-EVANS October 5, 2008 7:1
246 Essential Advanced General Mathematics
Forming a sequence of similar golden rectangles
In the golden rectangle ABHF, construct square AEGF
with side length .
A
E
B
F
G
H
L K
1
1

1
The remaining rectangle EBHG has side lengths 1 and .
Construct the square LKHG with side length 1. The sides
of the remaining rectangle EBKL are 1 and 1.
It was established earlier in the section that 1 =
1

.
Thus the rectangles ABHF, EBHG, EBKL are all similar as
they all have sides in the ratio : 1. This pattern continues.
Consider the golden rectangle EBKL.
E
Y
B
L
X
K
Now rectangle YBKX has sides
1

and 1
1

.
The ratio
1

: 1
1

= 1 : 1
and as shown previously 1 =
1 +

5
2
1
=
1 +

5
2
=
1

Therefore 1 : 1 = : 1
It can be shown that all the rectangles formed in this way are similar to each other.
Forming a sequence of squares and rectangles, the areas of
which are in geometric sequence with common ratio
1

The ratio of the areas of the squares and rectangles is also worth considering. The areas in
sequence are
A
E Y
B
L
X
K
H
G
F
Area rectangle ABHF = (+1) =
3
Area of square AEGF =
2
Area of rectangle EBHG =
Area of square LKHG = 1
Area of rectangle EBKL =
1

Area of square EYXL =


1

2
Area of rectangle YBKX =
1

3
P1: FXS/ABE P2: FXS
9780521740494c09.xml CUAU033-EVANS October 5, 2008 7:1
Chapter 9 Ratios and similarity 247
Exercise 9E
1 For the golden ratio show that
a 1 =
1

b
3
= 2+1
c 2 = (1)
2
=
1

2
2 ABC is a right-angled triangle with the right angle at C.
CX is the altitude of the triangle from C.
A
C
X
B
a Prove that
AX
CX
=
CX
XB
; i.e., the length CX is the
geometric mean of lengths AX and XB.
b Find CX if
i AX = 2 and XB = 8 ii AX = 1 and XB = 10.
3 A square is inscribed in a semicircle as shown.
Prove that
AD
BD
=
BD
CD
= 1.
A D E C
B
4 A regular decagon is inscribed in a circle with unit radius as shown.
a Find the magnitude of angle
i AOB ii OAB
b The line AX bisects angle OAB. Prove that
i triangle AXB is isosceles
ii triangle AXO is isosceles
iii triangle AOB is similar to triangle BXA
A
B
X
O
c Find the length of AB, to two decimal places.
d Describe a construction for
i a regular decagon ii a regular pentagon.
5 Calculate
0
,
1
,
2
,
3
,
4
and
1
,
2
,
3
,
4
. Show that each power of is equal to
the sum of the two powers before it, i.e.,
n+1
=
n
+
n1
6 The Fibonacci sequence is dened by t
1
= t
2
= 1 and t
n+1
= t
n1
+t
n
. The sequence is
1, 1, 2, 3, 5, . . . Consider the sequence
t
2
t
1
,
t
3
t
2
,
t
4
t
3
,
t
5
t
4
. . . and show that as n gets very large
(n approaches innity),
t
n+1
t
n
approaches .
P1: FXS/ABE P2: FXS
9780521740494c09.xml CUAU033-EVANS October 5, 2008 7:1
R
e
v
i
e
w
248 Essential Advanced General Mathematics
Chapter summary
Two gures are similar to each other if one is congruent to the other under an expansion
from the origin of factor k. An expansion of factor k from the origin has rule
(x, y) (kx, ky)
Similar triangles
Two triangles are similar if one of the following conditions holds.
r
Triangles have equal angles (AAA)
r
Corresponding sides are in the same ratio (PPP)
r
Two pairs of corresponding sides have the same ratio and the included angles are equal
(PAP)
45
A
B
C
AB
A'B'
AC
A'C'
=
45
A'
B'
C'
If triangle ABC is similar to triangle XYZ, this can be written symbolically as
ABC XYZ. The triangles are named so that angles of equal magnitude hold the same
position, i.e., A corresponds to X, B corresponds to Y, C corresponds to Z.
If two shapes are similar and the scale factor is k, i.e. for any length AB of one shape, the
corresponding length A

of the similar shape has length kAB, then the area of the similar
shape = k
2
area of the original shape.
For similar solids, if the scale factor is k, then the volume of the similar solid is k
3
volume
of the original solid.
Multiple-choice questions
1 If 5 : 3 = 7 : x then x is equal to
A 12 B
35
3
C 5 D
21
5
E
5
21
2 Brass is composed of a mixture of copper and zinc. If the ratio copper : zinc is 85 : 15, then
the amount of copper in 400 kg of brass is
A 60 kg B 340 kg C 360 kg D 380 kg E 150 kg
3 If the total cost of P articles is Q dollars, then the cost of R articles of the same type is
A PQR B
P
QR
C
PQ
R
D
QR
P
E
R
PQ
4 A car is 3.2 m long. The length in cm of a model of the car if the scale is 1 : 100 is
A 0.032 B 0.32 C 3.2 D 320 E 32
5 An athlete runs 75 m in 9 seconds. If she were to maintain the same average speed for
100 m her time for 100 m in seconds would be
A 11.6 B 12.0 C 11.8 D 12.2 E 12.4
P1: FXS/ABE P2: FXS
9780521740494c09.xml CUAU033-EVANS October 5, 2008 7:1
R
e
v
i
e
w
Chapter 9 Ratios and similarity 249
6 If 50 is divided into three parts in the ratio 1 : 3 : 6 then the largest part is
A 5 B 15 C
50
3
D 30 E 3
7 Two similar cylinders are shown.
10 cm
15 cm
45 cm
30 cm
The ratio of the volume of the smaller cylinder to
the larger cylinder is
A 1 : 3 B 1 : 9 C 1 : 27
D 1 : 5 E 2 : 9
8 The radius of sphere A is
4
5
times the radius of
sphere B. Hence, the ratio of the volume of sphere A to the volume of sphere B is
A 16 : 25 B 4 : 5 C 5 : 4 D 25 : 16 E 64 : 125
9 Triangles ABC and XYZ are similar isosceles triangles.
10 cm 10 cm
3 cm
A B
C
12 cm 12 cm
Z
X Y
The length of XY is
A 4 cm B 5 cm C 4.2 cm
D 2.5 cm E 3.6 cm
10 YZ is parallel to Y

and Y

Y =
1
3
YX. The area of triangle
XYZ is 60 cm
2
. The area of triangle XY

is
X
Z'
Y'
Y
Z
A 20 cm
2
B 30 cm
2
C
20
9
cm
2
D
20
3
cm
2
E
80
3
cm
2
Short-answer questions (technology-free)
1 In triangle XYZ, P is a point on XY and Q is a point on XZ such that PQ is parallel to YZ.
a Show that the two triangles XYZ and XPQ are similar.
b If XY = 36 cm, XZ = 30 cm and XP = 24 cm, nd
i XQ ii QZ
c Write down the values of XP : PY and PQ : YZ.
2 Triangles ABC and DEF are similar. If the area of triangle ABC is 12.5 cm
2
, the area of
triangle DEF is 4.5 cm
2
and AB = 5 cm, nd
a the length of DE b the value of AC : DF c the value of EF : BC.
3 If a 1 m stake casts a shadow 2.3 m long, nd the height of a tree (in metres) which casts a
shadow 21 m long.
P1: FXS/ABE P2: FXS
9780521740494c09.xml CUAU033-EVANS October 5, 2008 7:1
R
e
v
i
e
w
250 Essential Advanced General Mathematics
4 ABC is a right-angled triangle with AB = 4 and AC = 3.
If the triangle is folded along the line XY, vertex C
coincides with vertex B.
Find the length of XY.
A
C B
Y
X
5 Points A, B and C lie on a straight line. The squares are
adjacent and have side lengths 4, 7 and x.
Find the value of x.
A
4 7
x
B
C
6 Find the value of y in the diagram on the right.
y
7.2
2.2
6.6
26.4
7 An alloy is produced by mixing metal X with metal Y in the ratio of 5 : 3 by volume. The
mass of 1 cm
3
of metal X is
8
5
g and of 1 cm
3
of metal Y is
4
3
g. Calculate
a the mass of a solid cube of alloy of edge 4 cm
b the ratio, in the form n : 1, by mass, of metal X to metal Y in the alloy
c the volume, to the nearest cm
3
, of a cubic block of alloy whose mass is 1.5 kg
d the length, in mm, of the edge of this cubic block.
8 ABCD is a rectangle in which AB = 40 cm and AD = 60 cm.
M is the midpoint of BC, and DP is perpendicular to AM.
B
P
A
C
M
D
40 cm
60 cm
a Prove that the triangles BMA and PAD are similar.
b Calculate the ratio of the areas of the triangles BMA
and PAD.
c Calculate the length of PD.
9 A sculptor is commissioned to create a bronze statue 2 m high. He begins by making a clay
model 30 cm high.
a Express, in simplest form, the ratio of the height of the completed bronze statue to the
height of the clay model.
b If the total surface area of the model is 360 cm
2
, nd the total surface area of the statue.
c If the total volume of the model is 1000 cm
3
, nd the volume of the statue.
10 The radius of a spherical soap bubble increases by 1%. Find, correct to the nearest whole
number, the percentage increase in
a its surface area b its volume.
P1: FXS/ABE P2: FXS
9780521740494c09.xml CUAU033-EVANS October 5, 2008 7:1
R
e
v
i
e
w
Chapter 9 Ratios and similarity 251
11 AC is the diagonal of a rhombus ABCD. The line XYZ
is parallel to AD, AX = 3 cm and AB = 9 cm.
B C
D A
X
Y
Z
Find
a
XY
BC
b
AY
AC
c
CY
AC
d
YZ
AD
e
area triangle AXY
area triangle ABC
f
area triangle CYZ
area triangle ACD
12 AB and DC are parallel sides of a trapezium and DC = 3AB. The diagonals AC and DB
intersect at O. Prove that AO =
1
4
AC.
13 Triangles ABC and PQR are similar. The medians AX and PY are drawn. (X is the midpoint
of BC and Y is the midpoint of QR.) Prove
a that triangles ABX and PQY are similar b
AX
PY
=
BC
QR
Extended-response questions
1 a In this diagram which other triangle is similar
to DAC?
D
p
A B C
F
E
x y
q
h
b Explain why
h
p
=
y
x + y
.
c Use another pair of similar triangles to write
down an expression for
h
q
in terms of x and y.
d Explain why h

1
p
+
1
q

= 1.
e Calculate h when p = 4 and q = 5.
2 ABCDE is a regular pentagon whose sides are each
1 unit long. Each diagonal is of length d units.
In a regular pentagon, each diagonal is parallel to
one of the sides of the pentagon.
C
D E
A
B
F
1 1
1 1
1
d
a What kind of shape is ABCF and what is the
length of CF?
b Explain why the length of EF is d 1.
c Which triangle is similar to EFD?
d Use the pair of similar triangles to write an equation
for d and show that the equation can be rewritten as d
2
d 1 = 0.
e Find d.
3 Place conditions upon x such that DE is parallel to AB
given that CD = x 3, DA = 3x 19, CE = 4 and
EB = x 4.
C
A B
E D
3x 19
x 3
x 4
4
P1: FXS/ABE P2: FXS
9780521740494c09.xml CUAU033-EVANS October 5, 2008 7:1
R
e
v
i
e
w
252 Essential Advanced General Mathematics
4 a If BR, CS and DT are perpendicular to BD, name
the pairs of similar triangles.
b Which is correct:
z
y
=
p
q
or
z
y
=
p
p +q
?
B D
T
R
C p q
S
z
x
y
c Which is correct:
z
x
=
q
p
or
z
x
=
q
p +q
?
d Show that
1
x
+
1
y
=
1
z
5 In the diagram, PQ is parallel to BC and PR is parallel to AC.
AQ = 2 cm, QC = 6 cm, AP = 3 cm and PQ = 4 cm.
A
P
3 cm
2 cm
6 cm
4 cm
B
R
C
Q
a Calculate
i PB ii BR
iii
area APQ
area ABC
iv
area BPR
area ABC
b If the area of triangle APQ is a cm
2
, express in terms of a:
i area ABC ii area CPQ
6 Construct a triangle ABC such that BC = 10 cm, AC = 9 cm and AB = 6 cm. Find a point
D on AB and a point E on AC, such that DE is parallel to BC and the area of ADE is
one-ninth the area of ABC.
7 A triangular lot has boundaries of lengths AB = 130 m,
BC = 40

10 m and CA = 150 m. The length of CD is 120 m.


A fence is to be erected which runs at right angles from AB.
If the lot is to be divided into two equal areas, nd x.
C
A
x m
B
D
fence
8 The Greek historian Herodotus wrote that the proportions
of the great pyramid at Giza in Egypt were chosen so that
the area of a square, for which the side lengths are equal
to the height of the great pyramid, is equal to the area of
one of the triangular faces.
C
D
V
A
b m
h m
k m
B
Let h m be the height of the pyramid, k m the altitude of
one of the face triangles, and b m be the length of a side
of the square base.
Show that Herodotus denition gives k :
b
2
= .
P1: FXS/ABE P2: FXS
9780521740494c10.xml CUAU033-EVANS August 22, 2009 9:35
C H A P T E R
10
Circular functions I
Objectives
To use radians and degrees for the measurement of angles
To convert radians to degrees and vice versa
To define the circular functions sine, cosine and tangent
To explore the symmetry properties of circular functions
To find standard exact values of circular functions
To understand and sketch the graphs of circular functions
10.1 Measuring angles in degrees and radians
The diagram shows a unit circle, i.e., a circle of radius one unit.
The circumference of the unit circle = 2 1
= 2 units
the distance in an anticlockwise direction around
the circle from
A to B =

2
units
A to C = units
A to D =
3
2
units
x
y
B
C
O
D
A
1
1 1
1
Definition of a radian
In moving around the circle a distance of one unit from A to P, the angle POA is dened. The
measure of this angle is one radian.
One radian (written 1
c
) is the angle subtended at the centre of the unit circle by an arc of
length one unit.
253
P1: FXS/ABE P2: FXS
9780521740494c10.xml CUAU033-EVANS August 22, 2009 9:35
254 Essential Advanced General Mathematics
Note: Angles formed by moving anticlockwise around
the circumference of the unit circle are dened as
positive. Those formed by moving in a clockwise
direction are said to be negative.
x
y
P
O A
1 unit
1
1
1
1
c
1
Degrees and radians
The angle, in radians, swept out in one revolution of a circle is 2
c
.
2
c
=360


c
=180

1
c
=
180

or 1

=

c
180
Example 1
Convert 30

to radians.
Solution
Since 1

=

c
180
30

=
30
180
=

c
6
Example 2
Convert

c
4
to degrees.
Solution
Since 1
c
=
180


c
4
=
180
4
= 45

Note: Often the symbol for radian,


c
, is omitted. For example, angle 45

is written as

4
rather
than

c
4
.
P1: FXS/ABE P2: FXS
9780521740494c10.xml CUAU033-EVANS August 22, 2009 9:35
Chapter 10 Circular functions I 255
Exercise 10A
1 Express the following angles in radian measure in terms of .
Example 1
a 60

b 144

c 240

d 330

e 420

f 480

2 Express in degrees the angles with the following radian measures.


Example 2
a
2
3
b
5
6
c
7
6
d 0.9 e
5
9
f
9
5
g
11
9
h 1.8
3 Use a calculator to convert the following angles from radians to degrees.
a 0.6 b 1.89 c 2.9 d 4.31 e 3.72 f 5.18 g 4.73 h 6.00
4 Use a calculator to express the following in radian measure.
a 38

b 73

c 107

d 161

e 84

10

f 228

g 136

40

h 329

5 Express in degrees the angles with the following radian measures.


a

3
b 4 c 3 d
e
5
3
f
11
6
g
23
6
h
23
6
6 Express each of the following in radian measure in terms of .
a 360

b 540

c 240

d 720

e 330

f 210

10.2 Defining circular functions: sine and cosine


Consider the unit circle.
The position of point P on the circle can be described by relating
the cartesian coordinates x and y and the angle . The point P
on the circumference corresponding to an angle is written P().
x
y
y
P()
O 1
1
1

c
1
1
x
Many different angles will give the same point, P, on
the circle, so the relation linking an angle to the
coordinates is a many-to-one function. There are, in
fact, two functions involved and they are called sine
and cosine and are dened as follows:
The x coordinate of P, x = cosine , R
The y coordinate of P, y = sine , R
Note: These functions are usually written in an
abbreviated form as follows:
x
y
P() = (cos, sin)
O
1
1
1

cos
sin
1
1
c
x = cos
y = sin
Note: cos (2 +) = cos and
sin (2 +) = sin , as adding 2 results in a return
to the same point on the unit circle.
P1: FXS/ABE P2: FXS
9780521740494c10.xml CUAU033-EVANS August 22, 2009 9:35
256 Essential Advanced General Mathematics
Example 3
Evaluate
a sin and cos b sin
_

3
2
_
and cos
_

2
_
c sin
5
2
and sin
7
2
d sin
9
2
and cos 27.
Solution
a In moving anticlockwise through an angle of , the position is P() which is
(1, 0)
cos =1
sin =0
b In moving clockwise through an angle of
3
2
, the position is P
_

3
2
_
which is
(0, 1)
sin
_

3
2
_
= 1
cos
_

2
_
= 0
c sin
_
5
2
_
= sin
_
2
1
2

_
= sin
_
2 +

2
_
= sin

2
= 1
sin
_
7
2
_
= sin
_
3
1
2

_
= sin
_
2 +
3
2
_
= sin
3
2
= 1
d sin
_
9
2
_
= sin
_
4 +

2
_
= sin

2
= 1
cos(27) = cos (26 +) = cos = 1
Exercise 10B
1 For each of the following angles, t, determine the values of sin t and cos t.
Example 3
a t = 0 b t =
3
2
c t =
3
2
d t =
5
2
e t = 3 f t =
9
2
g t =
7
2
h t = 4
2 Evaluate, using a calculator (with calculator in Radian mode).
a sin 1.9 b sin 2.3 c sin 4.1 d cos 0.3
e cos 2.1 f cos (1.6) g sin (2.1) h sin (3.8)
3 For each of the following angles, , determine the values of sin and cos .
a = 27 b =
5
2
c =
27
2
d =
9
2
e =
11
2
f = 57 g = 211 h = 53
P1: FXS/ABE P2: FXS
9780521740494c10.xml CUAU033-EVANS August 22, 2009 9:35
Chapter 10 Circular functions I 257
10.3 Another circular function: tangent
Again consider the unit circle.
If a tangent to the unit circle at A is drawn, then the
y coordinate of C, the point of intersection of the
extension of OP and the tangent is called tangent
(abbreviated to tan ). By considering the similar
triangles OPD and OCA:
tan
1
=
sin
cos
tan =
sin
cos
Now when cos = 0, tan is undened.
x
y
O D
A
1
1
1

cos
tan
sin
P()
1
C(1, y)
B
Hence tan is undened when =

2
,
3
2
,
5
2
,
Domain of tan = R\{ : cos = 0}.
Example 4
Use a calculator to evaluate, correct to two decimal places
a tan 1.3 b tan 1.9 c tan (2.8) d tan 59

e tan 138

Solution
a tan 1.3 = 3.60 (Dont forget calculator must be in Radian mode.)
b tan 1.9 = 2.93
c tan (2.8) = 0.36
d tan 59

= 1.66 (Put calculator in Degree mode.)


e tan 138

= 0.90
Exercise 10C
1 Use a calculator to nd, correct to two decimal places
Example 4
a tan 1.6 b tan (1.2) c tan 136

d tan (54

)
e tan 3.9 f tan (2.5) g tan 239

2 Evaluate
a tan b tan () c tan
7
2
d tan (2) e tan
5
2
f tan
_

2
_
3 For each of the following values of nd tan .
a = 180

b = 360

c = 0
d = 180

e = 540

f = 720

P1: FXS/ABE P2: FXS


9780521740494c10.xml CUAU033-EVANS August 22, 2009 9:35
258 Essential Advanced General Mathematics
10.4 Reviewing trigonometric ratios
For right-angled triangles
sin =
opp
hyp
cos =
adj
hyp
tan =
opp
adj
opposite side
hypotenuse
adjacent side

Applying these trigonometric ratios to the right-angled


triangle, OAB, in the unit circle
sin =
opp
hyp
=
y
1
= y
cos =
adj
hyp
=
x
1
= x
tan =
opp
adj
=
y
x
=
sin
cos
x
y
x
y
O 1
1
1

1
1
B
A
For 0 < <

2
, functions sin, cos and tan are dened by the trigonometric ratios and are the
same as the respective circular functions introduced earlier.
Exercise 10D
1 Find the value of the pronumeral for each of the following.
8
3

a b c
d e f
g h i
5
25
x
6
25
x
10
50
x

5
6
x
20
10
5
x
65
7
70
x
5
40
x
P1: FXS/ABE P2: FXS
9780521740494c10.xml CUAU033-EVANS August 22, 2009 9:35
Chapter 10 Circular functions I 259
2 a Use a calculator to nd a and b, correct to four
decimal places.
b Hence nd the values of c and d.
c i Use a calculator to nd cos 140

and
sin 140

.
ii Write cos 140

in terms of cos 40

.
x
y
(c, d)
(a, b)
1
0
40
140
10.5 Symmetry properties of circular functions
The coordinate axes divide the unit circle
into four quadrants. The quadrants can be
numbered anticlockwise from the positive
direction of the x axis, as shown.
x
y
0
Quadrant 2
Quadrant 3
Quadrant 1
Quadrant 4
Relationships, based on symmetry between circular functions, for angles in different
quadrants can be determined.
(0, b)
(0, b)
(a, 0) (a, 0)
P(2 )
P( )
P( + )
=
(a, b)
P()
=
(cos, sin)
Quadrant 1 Quadrant 2
By symmetry
sin( )
=
b
=
sin
cos( )
=
a
=
cos
tan( ) = = tan
0

a
Quadrant 3
sin( + )
=
b
=
sin
cos( + )
=
a
=
cos
tan( + ) = = tan
b
a
b
a
Quadrant 4
sin(2 )
=
b
=
sin
cos(2 )
=
a
=
cos
tan(2 ) = = tan
Note: These relationships are true for all values of .
P1: FXS/ABE P2: FXS
9780521740494c10.xml CUAU033-EVANS August 22, 2009 9:35
260 Essential Advanced General Mathematics
Signs of circular functions
These symmetry properties can be summarised for the
signs of sin, cos and tan for the four quadrants as follows:
1st quadrant All are positive (A)
2nd quadrant Sin is positive (S)
3rd quadrant Tan is positive (T)
4th quadrant Cos is positive (C)
x
y
S A
T C
Negative of angles
By symmetry
cos () = cos
sin () = sin
tan () =
sin
cos
= tan
x
y
P()
P()

0
1
1 1
1
Example 5
If sin x = 0.6, nd the value of
a sin( x) b sin ( + x) c sin (2 x) d sin (x)
Solution
a sin ( x)
= sin x
= 0.6
b sin ( + x)
= sin x
= 0.6
c sin (2 x)
= sin x
= 0.6
d sin (x)
= sin x
= 0.6
Example 6
If cos x

= 0.8, nd the value of


a cos (180 x)

b cos (180 + x)

c cos (360 x)

d cos (x)

Solution
a cos (180 x)

= cos x

= 0.8
b cos (180 + x)

= cos x

= 0.8
c cos (360 x)

= cos x

= 0.8
d cos (x)

= cos x

= 0.8
P1: FXS/ABE P2: FXS
9780521740494c10.xml CUAU033-EVANS August 22, 2009 9:35
Chapter 10 Circular functions I 261
Exercise 10E
1 If sin = 0.42, cos x = 0.7 and tan = 0.38, write down the values of
Example 5
a sin ( +) b cos ( x) c sin (2 ) d tan ( )
e sin ( ) f tan (2 ) g cos ( + x) h cos (2 x)
2 If sin x

= 0.7, cos

= 0.6 and tan

= 0.4, write down the values of


Example 6
a sin (180 + x)

b cos (180 +)

c tan (360 )

d cos (180 )

e sin (360 x)

f sin (x)

g tan (360 +)

h cos ()

3 Write down the values of


a a = cos ( )
b b = sin ( )
c c = cos ()
d d = sin ()
e tan ( )
f tan ()
x
y

0
1
2
3 1
2
,
(a, b)
(c, d)

4 Write down the values of
a d = sin ( +)
b c = cos ( +)
c tan ( +)
d sin (2 )
e cos (2 )
x
y

0
1
(c, d)
+
2
3 1
2
,
10.6 Exact values of circular functions
A calculator can be used to nd the values of the circular functions for different values of .
For many values of the calculator gives an approximation. Consider some values of such
that sin, cos and tan can be calculated exactly.
Exact values for 0 (0

) and

2
(90

)
From the unit circle
When = 0,
sin 0 = 0
cos 0 = 1
tan 0 = 0
x
y

0 1
1
1
1
cos
sin
When =

2
,
sin

2
= 1
cos

2
= 0
tan

2
is undened
P1: FXS/ABE P2: FXS
9780521740494c10.xml CUAU033-EVANS August 22, 2009 9:35
262 Essential Advanced General Mathematics
Exact values for

6
(30

) and

3
(60

)
Consider an equilateral triangle ABC of side length two units. In
ACD, by Pythagoras theorem DC =
_
AC
2
AD
2
=

3
sin 30

=
AD
AC
=
1
2
sin 60

=
CD
AC
=

3
2
cos 30

=
CD
AC
=

3
2
cos 60

=
AD
AC
=
1
2
tan 30

=
AD
CD
=
1

3
tan 60

=
CD
AD
=

3
1
=

3
C
A B
D
2
1 1
60 60
30 30
2
Exact values for

4
(45

)
AC =
_
1
2
+1
2
=

2
sin 45

=
BC
AC
=
1

2
cos 45

=
AB
AC
=
1

2
tan 45

=
BC
AB
= 1
45
1
1
A B
C
As an aid to memory, the exact values for circular functions can be tabulated.
Summary
(

) sin cos tan


0 0 1 0

6
(30

)
1
2

3
2
1

4
(45

)
1

2
1

2
1

3
(60

3
2
1
2

2
(90

) 1 0 undened
Example 7
Evaluate
a cos
5
4
b sin
11
6
P1: FXS/ABE P2: FXS
9780521740494c10.xml CUAU033-EVANS August 22, 2009 9:35
Chapter 10 Circular functions I 263
Solution
a cos
5
4
= cos

4
(by symmetry)
=
1

2
b sin
11
6
= sin

6
(by symmetry)
=
1
2
Exercise 10F
1 Write down the exact values of
Example 7
a sin
2
3
b cos
3
4
c tan
5
6
d sin
7
6
e cos
_
5
4
_
f tan
4
3
g sin
5
3
h cos
7
4
i tan
11
6
2 Without using a calculator, evaluate the sin, cos and tan of each of the following.
a 120

b 135

c 210

d 240

e 315

f 390

g 420

h 135

i 300

j 60

3 Write down the exact values of


a sin
_

2
3
_
b cos
_
11
4
_
c tan
_
13
6
_
d tan
_
15
6
_
e cos
_
14
4
_
f cos
_
3
4
_
g sin
_
11
4
_
h cos
_

21
3
_
10.7 Graphs of sine and cosine
Graphs of sine functions
The graph of f (x) = sin x is given below. It has been plotted for x 3.
x
y

2
3
4
5
4
3
2
7
4
9
4
5
2
11
4
2 3 3
4

4
0
2
1
2
1
1
1
1
1 2 3 4 5 6 7 8 9 2 3
f (x) = sinx
Observations from the graph
The graph repeats itself after an interval of 2 units, i.e. f (x +2k) = f (x) for all
x R, k Z. A function which repeats itself regularly is called a periodic function and
the interval between the repetitions is called the period of the function.
P1: FXS/ABE P2: FXS
9780521740494c10.xml CUAU033-EVANS August 22, 2009 9:35
264 Essential Advanced General Mathematics
The maximum and minimum values of sin x are 1 and 1 respectively. The distance
between the mean position and the maximum position is called the amplitude. The graph
of f (x) = sin x has an amplitude of 1.
Graphs of cosine functions
The graph of g(x) = cos x for x 3 is as shown.
x
y

2
3
4
5
4
3
2
7
4
9
4
5
2
11
4
2 3

3
4

4
0
1
1
1
1 2 3 4 5 6 7 8 9 2 3
g(x) = cosx
Dilations of sine and cosine functions
A dilation of factor 2 from the y axis has the rule (x, y) (2x, y).
Hence (0, 0) (0, 0),
_

2
, 1
_
(, 1) and (, 0) (2, 0).
When this transformation is applied to y = sin x, it will be stretched away from the y axis.
Let (x, y) (x

, y

) under this transformation.


Since (x, y) (2x, y), then x

= 2x and y

= y, and thus x =
x

2
and y = y

.
Hence y = sin x is mapped to y = sin
x
2
.
A dilation of factor
1
2
from the y axis will map y = sin x to y = sin 2x.
x
y
x
y
x
y
2 2 4
1 1 1
0
1 1 1
0 0
2
y = sin
x
2
y = sin x y = sin 2x
Period = 4 Period = 2 Period =
Range = [1, 1] Range = [1, 1] Range = [1, 1]
P1: FXS/ABE P2: FXS
9780521740494c10.xml CUAU033-EVANS August 22, 2009 9:35
Chapter 10 Circular functions I 265
In general:
f : R R, f (x) = sin (nx) f : R R, f (x) = cos (nx)
Period =
2
n
Period =
2
n
Amplitude = 1 Amplitude = 1
Range = [1, 1] Range = [1, 1]
A dilation of factor 3 from the x axis has the rule (x, y) (x, 3y).
Hence (0, 1) (0, 3),
_

2
, 0
_

2
, 0
_
and (, 1) (, 3).
When this transformation is applied to y = cos x, it will be stretched away from the x axis.
y = cos x is mapped to y = 3 cos x

2
2 2
2
3
2
3
2
3
1
1
0
2
3 3
1
0
1
2
3
2
x
y y
x
y = cos x y = 3 cos x
Period = 2 Period = 2
Range = [1, 1] Range = [3, 3]
In general:
f : R R, f (x) = a sin (nx), n, a > 0 f : R R, f (x) = a cos (nx), n, a > 0
Period =
2
n
Period =
2
n
Amplitude = a Amplitude = a
Range = [a, a] Range = [a, a]
Example 8
Sketch the graphs of
a y = 2 cos 2 b y =
1
2
sin
x
2
Show one complete cycle.
P1: FXS/ABE P2: FXS
9780521740494c10.xml CUAU033-EVANS August 22, 2009 9:35
266 Essential Advanced General Mathematics
Solution
a period =
2
2
=
amplitude = 2
b period = 2
1
2
= 4
amplitude =
1
2
x
y
2
0
2

2
3
x
y
2
0
3 4
1
2

1
2
Using the TI-Nspire
Check that the calculator is in Radian mode. Open a Graphs & Geometry application
( 2), enter the function and select appropriate Window Settings (b41) as
shown.
a f 1 (x) = 2 cos (2x)
xMin = 0
xMax =
xScale =

4
yMin = 2
yMax = 2
b f 1 (x) =
1
2
sin
_
x
2
_
xMin = 0
xMax = 4
xScale =
yMin = 0.5
yMax = 0.5
P1: FXS/ABE P2: FXS
9780521740494c10.xml CUAU033-EVANS August 22, 2009 9:35
Chapter 10 Circular functions I 267
Using the Casio ClassPad
Ensure that the calculator is set to Radian mode (bottom left of the screen; tap to change).
a Enter the function y = 2 cos (2x) in the
window. Tap $to produce the
graph.
To show one complete cycle as required,
tap 6and make the window settings
x min = 0, max = and scale =

4
.
The appropriate values for y should also
be set.
Do not be concerned with the setting
for dot, it will take care of itself.
b Enter the function y =
1
2
sin
_

2
_
in the
window then tap $to produce
the graph.
The 6settings for the graph are
shown below.
P1: FXS/ABE P2: FXS
9780521740494c10.xml CUAU033-EVANS August 22, 2009 9:35
268 Essential Advanced General Mathematics
Example 9
Sketch the graph of y = 5 sin 3 for
4
3
2.
Solution
The amplitude = 5, the period =
2
3
x
y

3
2
3
4
3
5
3
4
3
2
3
2
3
0
5
5
The x axis intercepts can also be found by observing that one half period is

3
.
Using the TI-Nspire
Check that the calculator is in Radian mode. Open a Graphs & Geometry application
( 2), enter the function and select appropriate Window Settings (b41) as
shown.
f 1(x) = 5 sin(3x)
xMin =
4
3
xMax = 2
xScale =

6
yMin = 5
yMax = 5
P1: FXS/ABE P2: FXS
9780521740494c10.xml CUAU033-EVANS August 22, 2009 9:35
Chapter 10 Circular functions I 269
Using the Casio ClassPad
Check that the calculator is in Radian
mode.
Enter the function as shown with the
window settings shown to produce the
graph.
Exercise 10G
1 For each of the following, write down
i the period and ii the amplitude.
a 2 sin b 3 sin 2 c
1
2
cos 3 d 3 sin
1
2

e 4 cos 3 f
1
2
sin 4 g 2 cos
1
2

2 Sketch the graph of each of the following, showing one complete cycle. State the amplitude
Example 8
and period.
a y = 3 sin 2x b y = 2 cos 3 c y = 4 sin

2
d y =
1
2
cos 3x e y = 4 sin 3x f y = 5 cos 2x
g y = 3 cos
_

2
_
h y = 2 cos (4) i y = 2 sin
_

3
_
3 Sketch the graph of
Example 9
a f (x) = sin 2x for x [2, 2] b f (x) = 2 sin
x
3
for x [6, 6]
c f (x) = 2 cos 3x for x [0, 2] d f (x) = 2 sin 3x for x [0, 2]
4 Sketch the graph of f : [0, 2] R, f (x) =
5
2
cos
_
2x
3
_
Hint: For endpoints nd f (0) and f (2).
5 a On the one set of axes, sketch the graphs of f : [0, 2] R, f (x) = sin x and
g: [0, 2] R, g(x) = cos x
b By inspection from the graph state the values of x for which sin x = cos x.
P1: FXS/ABE P2: FXS
9780521740494c10.xml CUAU033-EVANS August 22, 2009 9:35
270 Essential Advanced General Mathematics
10.8 Further transformations of sine and
cosine graphs
Reflection in the axes
The function with rule f (x) = sin x is described as an odd function, i.e., f (x) = f (x). A
reection in the y axis gives the same result as a reection in the x axis when applied to the
graph of y = sin x.
The function with rule f (x) = cos x is described as an even function, i.e., f (x) = f (x).
The graph of f (x) = cos x is mapped onto itself when reected in the y axis.
Example 10
Sketch the graphs of
a f () = 3 cos 2 for 0 2 b g() = 5 sin(3) for 0 2
Solution
a

y
3
3
0
2
f () = 3 cos 2
Period =
Amplitude = 3
b

y
5
5
0 2
g() = 5 sin(3)

3
2
3
4
3
5
3
Period =
2
3
Amplitude = 5
P1: FXS/ABE P2: FXS
9780521740494c10.xml CUAU033-EVANS August 22, 2009 9:35
Chapter 10 Circular functions I 271
Translations
Translations in the direction of the y axis
The graph of y = sin x +1 is obtained
from the graph of y = sin x by a
translation of one unit in the positive
direction of the y axis.
x
y
2
0
4 3 2
Period = 2
Range = [0, 2]
Amplitude = 1
The graph of y = cos 2x 2 is obtained from
the graph of y = cos 2x by a translation of two
units in the negative direction of the y axis.
x
y
2 3 4
0
1
2
3
Translations in the direction of the x axis
The graph of y = sin
_
x

3
_
is obtained from
the graph of y = sin x by a translation of

3
in the positive direction of the x axis.
x
y
0

6
5
6
4
3
7
3
11
6
17
6

3
The graph of y = cos 2
_
x +

3
_
is obtained
from the graph of y = cos 2x by a translation
of

3
in the negative direction of the x axis.
x
y
0

12

6
5
12
2
3
7
6
11
12
17
12
5
3
P1: FXS/ABE P2: FXS
9780521740494c10.xml CUAU033-EVANS August 22, 2009 9:35
272 Essential Advanced General Mathematics
Example 11
On separate axes sketch the graphs of
a y = 3 sin 2
_
t

4
_
for t 2 b y = 2 cos 3
_
t +

3
_
for t
Solution
a The transformations applied
to y = sin t are
r
a dilation of factor 3 from
the x axis
r
a dilation of factor
1
2
from
the y axis
r
a translation of

4
in the positive
direction of the x axis.
t
y
3
4

2
3
3

2
3
4
5
4
3
2
7
4
2
Period =
Amplitude = 3
Range = [3, 3]
0
Note: This is the graph of y = 3 cos 2t .
b Note: The transformations applied to y = cos t are
r
a dilation of factor 2 from the x axis
r
a dilation of factor
1
3
from the y axis
r
a translation of

3
in the negative direction of the x axis.
0
t
y
5
6
5
6
2
3

6
2
2

2
2
3

Exercise 10H
1 Sketch the graph of each of the following showing one complete cycle. State the period,
Example 10
amplitude and the greatest and least values.
a y = 4 cos (2x) b f () =

2 sin 2 c f (x) = 2 sin (3x)


2 Sketch the graph of each of the following showing one complete cycle. State the period,
Example 11
amplitude and the greatest and least values.
a y = 3 sin
_


2
_
b y = sin 2( +) c y = 2 sin 3
_
+

4
_
P1: FXS/ABE P2: FXS
9780521740494c10.xml CUAU033-EVANS August 22, 2009 9:35
Chapter 10 Circular functions I 273
d y =

3 sin 2
_


2
_
e y = 3 sin 2x f y = 2 cos 3
_
+

4
_
g y =

2 sin 2
_


3
_
h y = 3 sin 2x i y = 3 cos 2
_
+

2
_
3 For the function f : [0, 2] R, f (x) = cos
_
x

3
_
a nd f (0), f (2) b sketch the graph of f.
4 For the function f : [0, 2] R, f (x) = sin 2
_
x

3
_
a nd f (0), f (2) b sketch the graph of f.
5 For the function f : [, ] R, f (x) = sin 3
_
x +

4
_
a nd f (), f () b sketch the graph of f.
10.9 Solution of circular function equations
Example 12
Find all solutions to the equation sin =
1
2
for [0, 4].
Solution
It is clear from the graph that there are
four solutions in the interval [0, 4].
The solution for
_
0,

2
_
is =

6
.
This solution can be obtained from a
knowledge of exact values or using
sin
1
on a calculator.

y
y = sin

6
1
2
y =
3 4 2
1
1
0
The second solution is obtained by
symmetry. The function is positive in
the second quadrant and
sin ( ) = sin .
Therefore =
5
6
is the second
solution.
It can be seen that further solutions
can be achieved by adding 2, as
sin = sin ( +2).

y
1
1
y = sin
1
2
y =

6 6
5
6
13
6
17 0
Thus =
13
6
and
17
6
are also solutions.
P1: FXS/ABE P2: FXS
9780521740494c10.xml CUAU033-EVANS August 22, 2009 9:35
274 Essential Advanced General Mathematics
Using the TI-Nspire
Check that the calculator is in Radian
mode.
Use solve( ) from the Algebra menu
(b 1) as shown.
The symbol can be found in the
catalog ( 4), by typing , or by
typing / .
Using the Casio ClassPad
Set the calculator to Radian mode.
Enter the functions y = sin (x) and
y =
1
/
2
.
The window settings should be as
shown. Tap $to produce the graph.
Ensure the graph window is selected (bold
border) and tap Analysis, G-solve, intersect
to nd decimal approximations for the
solutions. The scroll key moves the cursor
between solutions.
To nd exact solutions, the window
is used.
Enter and highlight the equation
sin (x) =
1
2
. Tap Interactive,
Equation/inequality, solve and ensure the
variable is set to x.
The answer returned is
x = 2 constn (9) +

6
, x = 2 constn (10) +
5
6
.
This may be read as
x = 2m +

6
, 2n +
5
6
.
Note: The restricted domain can be entered. See the Appendix for directions.
P1: FXS/ABE P2: FXS
9780521740494c10.xml CUAU033-EVANS August 22, 2009 9:35
Chapter 10 Circular functions I 275
It should be clear that there are 4 solutions to the problem. Hence, the values for each
of m and n will be required which produce a solution in the domain. In this case the
values are m = 0, 1 and n = 0, 1. The solutions are x =

6
,
11
6
,
13
6
,
23
6
.
Example 13
For the following, nd two values of x in the range 0 x 360.
a sin x

= 0.3 b cos x

= 0.7
Solution
a First solve the equation sin x

= 0.3.
Use a calculator to nd the solution for x [0, 90]; x = 17.46
Now the value of sin is negative for P(x) in the 3rd and 4th quadrants.
From the symmetry relationships (or from the graph of y = sin x

)
3rd quadrant x = 180 +17.46
= 197.46
4th quadrant x = 360 17.46
= 342.54
if sin x

= 0.3, x = 197.46 or x = 342.54


b First solve the equation cos x

= 0.7.
Use a calculator to nd the solution for x [0, 90]; x = 45.57
Now the value of cos is negative for P(x) in the 2nd and 3rd quadrants
2nd quadrant x = 180 45.57
= 134.43
3rd quadrant x = 180 +45.57
= 225.57
if cos x

= 0.7, x = 134.43 or x = 225.57


Example 14
Solve the equation sin 2 =

3
2
for [, ].
Solution
It is clear that there are four solutions.
To solve the equation let x = 2.
Note: if [, ]
then 2 = x [2, 2]
Consider the equation sin x =

3
2
for x [2, 2]
y
1
1
y = sin2

2

3
2
y =

0

P1: FXS/ABE P2: FXS
9780521740494c10.xml CUAU033-EVANS August 22, 2009 9:35
276 Essential Advanced General Mathematics
The 1st quadrant solution of the
equation sin x =

3
2
is x =

3
Symmetry gives the solutions to
sin x =

3
2
for x [0, 2] as
x = +

3
and x = 2

3
i.e., x =
4
3
or x =
5
3
The other two solutions are obtained by subtracting 2,
i.e.,
4
3
2 and
5
3
2
the required solutions for x are
2
3
or

3
or
4
3
or
5
3
the required solutions for are

3
or

6
or
2
3
or
5
6
x
y
1
1
y = sinx
2 2
3
2
y =

0
Exercise 10I
1 Find, without using a calculator, all the values of between 0 and 2 for each of the
Example 12
following.
a

2 sin x +1 = 0 b

2 cos x 1 = 0
2 Find all values of x between and for which
a cos x =
1

2
b sin x =

3
2
c cos x =
1
2
3 Find, without using a calculator, all the values of between 0

and 360

for each of the


following.
a cos

3
2
b sin

=
1
2
c cos

=
1
2
d 2 cos

+1 = 0 e 2 sin

3 f

2 sin

1 = 0
4 Find all the values of x between 0 and 2 for which
Example 13
a sin x = 0.8 b cos x = 0.4 c sin x = 0.35
d sin x = 0.4 e cos x = 0.7 f cos x = 0.2
5 Find all the values of x between 0 and 4 for which
a sin x = 0.6 b sin x =
1

2
c sin x =

3
2
P1: FXS/ABE P2: FXS
9780521740494c10.xml CUAU033-EVANS August 22, 2009 9:35
Chapter 10 Circular functions I 277
6 Solve the following equations for [0, 2].
Example 14
a sin 2 =
1
2
b cos 2 =

3
2
c sin 2 =
1
2
d sin 3 =
1

2
e cos 2 =

3
2
f sin 2 =
1

2
7 Solve the following equations for [0, 2].
a sin 2 = 0.8 b sin 2 = 0.6 c cos 2 = 0.4 d cos 3 = 0.6
8 a Sketch the graph of f : [2, 2] R, f (x) = cos x.
b On the graph, mark the points which have y coordinate
1
2
and give the associated
x values.
c On the graph, mark the points which have y coordinate
1
2
and give the associated
x values.
10.10 Further sketch graphs
Sketch graphs of y = sin n(t + ) + b
and y = a cos n(t + ) + b
Consider the graph of each of the following functions.
a y = 3 sin 2
_
t

4
_
+2,

4
t
5
4
b y = 2 cos 3
_
t +

3
_
1,

3
t

3
a
t
y
0
1
1
2
3
4
5
y = 3sin 2 t + 2

2
3
4
5
4
b
t
y
1
1
3
y = 2cos 3 t + 1

3
0

3

6

2
Observations
1 The graph of y = 3 sin 2
_
t

4
_
+2 is the same shape as the graph of
y = 3 sin 2
_
t

4
_
but it is translated two units in the positive direction of the y axis.
2 Similarly, the graph of y = 2 cos 3
_
t +

3
_
1 is the same shape as the graph of
y = 2 cos 3
_
t +

3
_
but it is translated one unit in the negative direction of the y axis.
P1: FXS/ABE P2: FXS
9780521740494c10.xml CUAU033-EVANS August 22, 2009 9:35
278 Essential Advanced General Mathematics
In general, the effect of b is to translate the graph b units in the positive direction of the y axis
when b > 0, and in the negative direction of the y axis when b < 0.
Finding axis intercepts
Example 15
Sketch the graphs of each of the following for x [0, 2]. Clearly indicate axis intercepts.
a y =

2 sin x +1 b y = 2 cos 2x 1 c y = 2 sin 2


_
x

3
_

3
Solution
a To determine the axis intercepts, the equation

2 sin x +1 = 0 must be solved.

2 sin x +1 = 0
sin x =
1

2
x = +

4
, 2

4
x =
5
4
,
7
4
intercepts
_
5
4
, 0
_
,
_
7
4
, 0
_
y
2 + 1
x
1
0
2

4
5 2
2
3
4
7

y = 2sin x + 1
2 + 1

Using the TI-Nspire


Check that the calculator is in Radian mode.
Open a Graphs & Geometry application
( 2), enter f 1(x) =

2 sin(x) +1
and select appropriate Window Settings
(b41) as shown.
xMin = 0
xMax = 2
yMin = 1
_
(2)
yMax = 1 +
_
(2)
Open a Calculator application ( 1)
to nd the exact values of the x-intercepts
using solve( ) from the Algebra menu
(b 1) as shown.
The symbol can be found in the
catalog ( 4), by typing ,
or by typing / .
P1: FXS/ABE P2: FXS
9780521740494c10.xml CUAU033-EVANS August 22, 2009 9:35
Chapter 10 Circular functions I 279
Using the Casio ClassPad
Ensure that the calculator is in Radian mode.
Enter and highlight the equation

2 sin(x) +1 = 0.
Tap Interactive, Equation/inequality, solve.
The required solutions x =
7
4
or
5
4
are found by selecting the appropriate values for
the constant, constn(2) and constn(2).
b
2 cos 2x 1 = 0
cos 2x =
1
2
2x =

3
,
5
3
,
7
3
,
11
3
x =

6
,
5
6
,
7
6
,
11
6
intercepts
_

6
, 0
_
,
_
5
6
, 0
_
,
_
7
6
, 0
_
,
_
11
6
, 0
_
x
5
6

6
7
6
11
6
2
y = 2cos2x 1
1
0
1
2
3
y
c 2 sin 2
_
x

3
_

3 = 0
sin 2
_
x

3
_
=

3
2
2
_
x

3
_
=

3
,
2
3
,
7
3
,
8
3
x

3
=

6
,

3
,
7
6
,
4
3
x =

2
,
2
3
,
3
2
,
5
3
axis intercepts
_

2
, 0
_
,
_
2
3
, 0
_
,
_
3
2
, 0
_
,
_
5
3
, 0
_
x
y

y = 2

3
y =

2

3

3
(0, 2
3)
3
2
5
3
2
3
2
0
y = 2 sin 2

3
x 3
P1: FXS/ABE P2: FXS
9780521740494c10.xml CUAU033-EVANS August 22, 2009 9:35
280 Essential Advanced General Mathematics
Exercise 10J
1 Sketch the graphs of each of the following for x [0, 2]. List the x axis intercepts of
Example 15
each graph for this interval.
a y = 2 sin x +1 b y = 2 sin 2x

3 c y =

2 cos x +1
d y = 2 sin 2x 2 e y =

2 sin
_
x

4
_
+1
2 Sketch the graphs of each of the following for x [2, 2].
a y = 2 sin 3x 2 b y = 2 cos 3
_
x

4
_
c y = 2 sin 2x 3 d y = 2 cos 2x +1
e y = 2 cos 2
_
x

3
_
1 f y = 2 sin 2
_
x +

6
_
+1
3 Sketch the graphs of each of the following for x [, ].
a y = 2 sin 2
_
x +

3
_
+1 b y = 2 sin 2
_
x +

6
_
+1
c y = 2 cos 2
_
x +

4
_
+

3
10.11 Rotation of a point about the origin
Consider the points with coordinates (0, 1) and (1, 0) under a rotation of
c
in an anticlockwise
direction around the origin.
cos

2
+

2
+ , sin
(cos, sin)
(0, 1)
(1, 0) 0

c
x
y
From the diagram
(1, 0) (cos , sin )
(0, 1)
_
cos
_

2
+
_
, sin
_

2
+
__
and
_
cos
_

2
+
_
, sin
_

2
+
__
= (sin , cos )
The matrix dening the transformation =
_
cos
sin
sin
cos
_
P1: FXS/ABE P2: FXS
9780521740494c10.xml CUAU033-EVANS August 22, 2009 9:35
Chapter 10 Circular functions I 281
Hence if (x, y) (x

, y

)
_
x

_
=
_
cos
sin
sin
cos
__
x
y
_
=
_
x cos y sin
x sin + y cos
_
i.e., x

= x cos y sin
and y

= x sin + y cos
Example 16
a Find the matrix, A, for rotation of

3
about O in an anticlockwise direction, and hence nd
the coordinates of the image of the point (1, 0) under this rotation.
b Find the matrix, B, for rotation of

3
about O in a clockwise direction, and hence nd the
coordinates of the image of the point (1, 0) under this rotation.
Solution
a A =
_
_
_
cos

3
sin

3
sin

3
cos

3
_

_
=
_
_
_
_
1
2

3
2

3
2
1
2
_

_
_
_
_
x

_
=
_
_
_
_
1
2

3
2

3
2
1
2
_

_
_
_
_
1
0
_

_
=
_
_
_
_
1
2

3
2
_

_
b
B =
_
_
_
cos

3
sin

3
sin

3
cos

3
_

_
=
_
_
_
_
1
2

3
2

3
2
1
2
_

_
_
_
x

_
_
=
_
_
_
_
1
2

3
2

3
2
1
2
_

_
_
_
1
0
_
_
=
_
_
_
_
1
2

3
2
_

_
Example 17
Describe the rotation represented by the matrix
_
_
_
_

2
1

2
_

_
P1: FXS/ABE P2: FXS
9780521740494c10.xml CUAU033-EVANS August 22, 2009 9:35
282 Essential Advanced General Mathematics
Solution
The matrix dening a rotation of
c
in an anticlockwise direction around the origin is
given by
_
cos sin
sin cos
_
Let
_
cos
sin
sin
cos
_
=
_
_
_
_

2
1

2
_

_
Therefore cos =
1

2
and sin =
1

2
, and the smallest positive solution for is
3
4
The matrix represents a rotation of
3
4
radians about the origin in an anticlockwise
direction.
Note: This can also be described as a rotation of
5
4
radians about the origin in a
clockwise direction.
Exercise 10K
1 For each of the following, nd the matrix associated with the rotation and hence the image
Example 16
of the given point under that transformation.
a rotation of

6
about O in an anticlockwise direction, point (1, 0)
b rotation of

4
about O in a clockwise direction, point (0, 1)
c rotation of

2
about O in an anticlockwise direction, point (1, 0)
d rotation of
2
3
about O in a clockwise direction, point (0, 1)
e rotation of
5
4
about O in an anticlockwise direction, point
_
1

2
,
1

2
_
f rotation of
5
6
about O in a clockwise direction, point
_

3
2
,
1
2
_
2 Describe the rotation represented by each of the following matrices.
Example 17
a
_
_
_

3
2
1
2

1
2

3
2
_

_
b
_
_
_

1
2

3
2

3
2

1
2
_

_
c
_
_
_
4
5
3
5

3
5
4
5
_

_
3 a Find the matrix R
4
of rotation of

4
about O in an anticlockwise direction.
b Find the image of (2, 1) under this rotation.
P1: FXS/ABE P2: FXS
9780521740494c10.xml CUAU033-EVANS August 22, 2009 9:35
Chapter 10 Circular functions I 283
c Let R

denote the matrix dening a rotation of


c
in an anticlockwise direction about O.
Find:
i R
3
4
ii R
2
iii R
5
6
iv R
4
d If 0 < <

2
and cos =
4
5
, nd sin and write down the matrix R

.
10.12 Applications
Example 18
It is suggested that the height h(t) metres of the tide above mean sea level on 1 January at
Warnung is given approximately by the rule h(t ) = 4 sin
_

6
t
_
where t is the number of hours
after midnight.
a Draw the graph of y = h(t ) for 0 t 24. b When was high tide ?
c What was the height of the high tide ? d What was the height of the tide at 8 am ?
e A boat can only cross the harbour bar when the tide is at least 1 m above mean sea level.
When could the boat cross the harbour bar on 1 January?
Solution
a
period = 2

6
= 12
t
y
4
0 6 12 18 24
4
y = h(t)
b High tide occurs when h(t ) = 4
4 sin
_

6
t
_
= 4
implies sin
_

6
t
_
= 1


6
t =

2
,
5
2
t = 3, 15
i.e., high tide occurs at 3.00 (3 am) and 15.00 (3 pm)
c The high tide has height 4 m above the mean height.
d h(8) = 4 sin
_
8
6
_
= 4 sin
_
4
3
_
= 4

3
2
= 2

3
The water is 2

3 m below the mean height at 8 am.


P1: FXS/ABE P2: FXS
9780521740494c10.xml CUAU033-EVANS August 22, 2009 9:35
284 Essential Advanced General Mathematics
e First consider 4 sin
_

6
t
_
= 1
Thus sin
_

6
t
_
=
1
4


6
t = 0.2527, 2.889, 6.5359, 9.172
t = 0.48, 5.52, 12.48, 17.52
i.e., the water is at height 1 m at 00:29, 05:31, 12:29, 17:31.
Thus the boat can pass across the harbour bar between 00:29
and 05:31 and between 12:29 and 17:31.
Exercise 10L
1 The number of hours of daylight at a point on the Antarctic Circle is given approximately
Example 18
by d = 12 +12 cos
1
6

_
t +
1
3
_
where t is the number of months which have elapsed
since 1 January.
a Find d
i on 21 June (t 5.7) ii on 21 March (t 2.7).
b When will there be ve hours of daylight ?
2 The depth, D(t ) metres, of water at the entrance to a harbour at t hours after midnight on a
particular day is given by D(t ) = 10 +3 sin
_
t
6
_
, 0 t 24.
a Sketch the graph of D(t ) for 0 t 24.
b Find the values of t for which D(t ) 8.5.
c Boats which need a depth of w metres are permitted to enter the harbour only if the depth
of the water at the entrance is at least w metres for a continuous period of one hour.
Find, correct to one decimal place, the largest value of w which satises this condition.
3 The depth of water at the entrance to a harbour t hours after high tide is D metres, where
D = p +q cos (rt )

for suitable constants p, q, r. At high tide the depth is 7 m; at low tide,


six hours later, the depth is 3 m.
a Show that r = 30 and nd the values of p and q.
b Sketch the graph of D against t for 0 t 12.
c Find how soon after low tide a ship which requires a depth of at least 4 m of water will
be able to enter the harbour.
4 A particle moves on a straight line, Ox, and its distance x metres from O at time t (seconds)
is given by x = 3 +2 sin 3t.
a Find its greatest distance from O. b Find its least distance from O.
c Find the times at which it is 5 m from O for 0 t 5.
d Find the times at which it is 3 m from O for 0 t 3.
e Describe the motion of the particle.
P1: FXS/ABE P2: FXS
9780521740494c10.xml CUAU033-EVANS August 22, 2009 9:35
Chapter 10 Circular functions I 285
5 The temperature A

C inside a house at t hours after 4 am is given by


A = 21 3 cos
_
t
12
_
for 0 t 24, and the temperature B

C outside the house at the


same time is given by B = 22 5 cos
_
t
12
_
for 0 t 24.
a Find the temperature inside the house at 8 am.
b Write down an expression for D = A B, the difference between the inside and
outside temperatures.
c Sketch the graph of D versus t for 0 t 24.
d Determine when the inside temperature is less than the outside temperature.
6 The high-water mark on a beach wall is a sinusoidal function, i.e., it has a rule of the form
a sin (nt +) +b. In this case the function is d(t ) = 6 +4 cos
_

6
t

3
_
where t is the
number of hours after midnight and d is the depth of the water in metres.
a What is the earliest time of day at which the water is at its highest ?
b When is the water 2 m up the wall ?
7 The graph shows the distance d(t ) of the top of the hour hand of a large clock from the
ceiling at time t hours.
d (m)
t (h)
5
2
0
12 24
a d(t ) is the rule of a sinusoidal function. Find
i the amplitude ii the period
iii the rule for d(t ) iv the length of the hour hand.
b At what times is the distance less than 3.5 m from the ceiling ?
8 In a tidal river the time between high tide and low tide is 8 hours. The average depth of
water in a point on the river is 4 m; at high tide the depth is 5 m.
a Sketch a graph of the depth of the water at the point over time if the relationship
between time and depth is sinusoidal and there is a high tide at noon.
b If a boat requires a depth of 4 m of water in order to sail, how many hours before noon
can it enter the point and by what time must it leave to avoid being stranded?
c If a boat requires a depth of 3.5 m of water in order to sail, at what time before noon can
it enter the point and by what time must it leave to avoid being stranded ?
P1: FXS/ABE P2: FXS
9780521740494c10.xml CUAU033-EVANS August 22, 2009 9:35
286 Essential Advanced General Mathematics
9 The population, N, of a particular species of ant varies with time. The population at time t
weeks after 1 January 2006 is given by
N = 3000 sin
((t 10))
26
+4000
a For N(t ) = 3000 sin
((t 10))
26
+4000 state
i the period ii the amplitude iii the range.
b i State the values of N(0) and N(100).
ii Sketch the graph of y = N(t ) for t [0, 100].
c Find the values of t (t [0, 100]) for which the population is
i 7000 ii 1000
d Find {t : N(t ) > 5500} for t [0, 100], i.e., nd the intervals of time during the rst
hundred days for which the population of ants is greater than 5500.
e A second population M(t ) of ants also varies with time. The population has the
following properties.
r
minimum population is 10 000 at t = 20
r
no maximum or minimum value between t = 10 and t = 20
r
maximum population is 40 000 at t = 10
r
M(t ) = a sin
((t c))
b
+d where a, b, c and d are positive constants
Find a set of possible values of a, b, c and d.
P1: FXS/ABE P2: FXS
9780521740494c10.xml CUAU033-EVANS August 22, 2009 9:35
R
e
v
i
e
w
Chapter 10 Circular functions I 287
Chapter summary
Denition of a radian
One radian (written 1
c
) is the angle formed at the
centre of the unit circle by an arc of length 1 unit.
x
y
1
1 0
1
1 unit
1
c
1
1
1
c
=
180

=

c
180
Sine and cosine
x coordinate of P(Q) in unit circle,
x = cosine , R
y coordinate of P() in unit circle,
y = sine , R
Abbreviated to
x = cos , y = sin
x
y
x
y
1
1 0
1
1
1
P()

Tangent
If the tangent to the unit circle at A is drawn then the
y coordinate of B is called tangent (abbreviated to tan ).
x
y
1
A
B
0
1
cos
tan
1
1

sin
Also by similar triangles,
tan =
sin
cos
Circular functions and trigonometric ratios
x
y
x
y
0
1

opposite
side
adjacent
side
hypotenuse
sin =
opp
hyp
=
y
1
= y
cos =
adj
hyp
=
x
1
= x
tan =
opp
adj
=
y
x
=
sin
cos
P1: FXS/ABE P2: FXS
9780521740494c10.xml CUAU033-EVANS August 22, 2009 9:35
R
e
v
i
e
w
288 Essential Advanced General Mathematics
Symmetry properties of circular functions
x
1
1
1
1
b
b

y
Quadrant 2
(sin is positive)
sin( ) = b = sin
Quadrant 1
(all function are positive)
e.g. sin = b
sin(2 ) =

b =

sin
Quadrant 4 (cos is positive)
sin( + ) =

b =

sin
Quadrant 3 (tan is positive)
Further symmetry properties
Negative angles
cos () = cos
sin () = sin
tan () =
sin
cos
= tan
x
y
1
1 1
1

Exact values of circular functions


sin cos tan
0 0 1 0

6
1
2

3
2
1

4
1

2
1

2
1

3
2
1
2

2
1 0 undened
P1: FXS/ABE P2: FXS
9780521740494c10.xml CUAU033-EVANS August 22, 2009 9:35
R
e
v
i
e
w
Chapter 10 Circular functions I 289
Graphs of sine and cosine functions
y
2
1
y = sin
amplitude = 1
period = 2

y
1
1
y = cos
3
2

2
2

amplitude = 1
period = 2
Solutions of circular function equations of the type sin x

= a and cos x

= a
e.g., If cos x

= 0.7, nd the two values of x in the range 0 x 360.


If cos x

= 0.7, then x = 45.6


Since cos is negative in the 2nd and 3rd quadrants
x = 180 45.6 = 134.4
and x = 180 +45.6 = 225.6
Sketch graphs of circular functions of the type
y = a sin n(t +) +b and y = a cos n(t +) +b
e.g., y = 2 cos 3
_
t +

3
_
1
amplitude, a = 2
period =
2
n
=
2
3
1
2
3
t
y

6

3
0
1
The graph is the same shape as y = 2 cos 3t but is translated
i

3
units in the negative direction of the x axis and
ii 1 unit in the negative direction of the y axis.
Multiple-choice questions
1 In the triangle ABC, cos x is equal to
A
a

a
2
+b
2
B
b

a
2
+b
2
C
a
b
D
b
a
E

a
2
+b
2
a
A
x
b
B
C
a
P1: FXS/ABE P2: FXS
9780521740494c10.xml CUAU033-EVANS August 22, 2009 9:35
R
e
v
i
e
w
290 Essential Advanced General Mathematics
2 The period of the graph of y = 2 sin (3x ) +4 is
A
2
3
B 2 C 3 D E 2
3 The amplitude of the graph of y = 5 cos 5x +3 is
A 5 B 2 C 2 D 5 E 8
4 The number of solutions of 5 sin (2x ) +2 = 0 in the interval [0, 2] is
A 1 B 2 C 3 D 4 E 8
5 An angle of
3
11
radians expressed in degrees (correct to two decimal places) is
A 49 B 154.22 C 49.09 D 0.01 E 0.00
6 The solutions of 2 sin 3x +

2 = 0 in the interval
_
5
12
,
23
12
_
are
A
5
12
,
7
12
,
13
12
,
5
4
,
7
4
,
23
4
B 1.83, 3.40, 3.93, 5.50
C
4
9
,
5
9
,
10
9
,
11
9
,
16
9
,
17
9
D
7
12
,
13
12
,
5
4
,
7
4
E none of the above
7 cos
_
13
6
_
is equal to
A cos
_
13
6
_
B

3
2
C cos
_
7
6
_
D
1
2
E sin
_
2
3
_
8 tan (180 )

is equal to
A
sin (90 +)

cos (90 )

B
cos (180 )

sin (180 )

C
sin (90 )

cos (90 +)

D
cos (90 )

sin (90 +)

E
cos (90 +)

sin (90 )

.
9 The period of the graph of f (x) = 4 sin (3x) 3 cos (2x) is
A 1 B 2 C 3 D 4 E
2
3
10 The transformation given by the matrix
_
_
_
_
_
1
2

3
2

3
2
1
2
_

_
is
A a rotation of 30

about the origin in an anticlockwise direction


B a rotation of 60

about the origin in an anticlockwise direction


C a dilation by factor
1
2
from the x axis
D a reection in the line y = x,
E a rotation of 60

about the origin in a clockwise direction


P1: FXS/ABE P2: FXS
9780521740494c10.xml CUAU033-EVANS August 22, 2009 9:35
R
e
v
i
e
w
Chapter 10 Circular functions I 291
Short-answer questions (technology-free)
1 Change each of the following to radian measure in terms of .
a 330

b 810

c 1080

d 1035

e 135

f 405

g 390

h 420

i 80

2 Change each of the following to degree measure.


a
5
c
6
b
7
c
4
c
11
c
4
d
3
c
12
e
15
c
2
f
3
c
4
g

c
4
h
11
c
4
i
23
c
4
3 Give exact values of each of the following.
a sin
_
11
4
_
b cos
_
7
4
_
c sin
_
11
6
_
d cos
_
7
6
_
e cos
_
13
6
_
f sin
_
23
6
_
g cos
_
23
3

_
h sin
_
17
4

_
4 State the amplitude and period of each of the following.
a 2 sin

2
b 3 sin 4 c
1
2
sin 3
d 3 cos 2x e 4 sin
x
3
f
2
3
sin
2x
3
5 Sketch the graphs of each of the following (showing one cycle).
a y = 2 sin 2(2x) b y = 3 cos
_
x
3
_
c y = 2 sin 3x
d y = 2 sin
x
3
e y = sin
_
x

4
_
f y = sin
_
x +
2
3
_
g y = 2 cos
_
x
5
6
_
h y = 3 cos
_
x +

6
_
6 Solve each of the following equations for R.
a sin =

3
2
, [, ] b sin 2 =

3
2
, [, ]
c sin
_


3
_
=
1
2
, [0, 2] d sin
_
+

3
_
= 1, [0, 2]
e sin
_

3

_
=
1
2
, [0, 2]
7 Sketch the graphs of each of the following for x [, 2].
a f (x) = 2 sin 2x +1 b f (x) = 1 2 cos x c f (x) = 3 cos
_
x +

3
_
d f (x) = 2 cos
_
x +

3
_
e f (x) = 1 2 sin 3x
P1: FXS/ABE P2: FXS
9780521740494c10.xml CUAU033-EVANS August 22, 2009 9:35
R
e
v
i
e
w
292 Essential Advanced General Mathematics
Extended-response questions
1 The depth, D metres, of sea water in a bay, t hours after midnight on a particular day, may be
represented by the function with rule
D(t ) = a +b cos
_
2t
k
_
, where a, b and k are real numbers.
The water is at a maximum depth of 15.4 m at midnight and noon, and is at a minimum
depth of 11.4 m at 6.00 and 18.00 hours.
a Find the value of
i a ii b iii k
b Find the times when the depth of the water is 13.4 m.
c Find the values of t for which the depth of the bay is less than 14.4 m.
2 The temperature (

) in a small town in the mountains over a day is modelled by the function


with rule
T = 15 8 cos
_
t
12
+6
_
, where t is the time in hours after midnight, 0 t 24.
a What is the temperature at midnight, correct to two signicant gures?
b What are the maximum and minimum temperatures reached?
c At what times of the day, to the nearest minute, are temperatures warmer than 20

?
d Sketch the graph for the temperatures over a day.
3 A particle oscillates back and forth, in a straight line, between points A and B about a point
O. Its position, x(t) metres, relative to O at time t seconds is given by the rule
x(t ) = 3 sin (2t a). The position of the particle when t = 1 is x = 1.5.
A O B
a If a [0,

2
], nd the value of a.
b Sketch the graph of x(t ) against t for t [0, 2]. Label maximum and minimum points,
axes intercepts and endpoints with their coordinates.
c How far from O is point A?
d At what time does the particle rst pass through A?
e How long is it before the particle returns to A?
f How long does it take for the particle to go from A to O?
g How far does the particle travel in
i the rst 2 s of its motion ii the rst 2.5 s of its motion?
P1: FXS/ABE P2: FXS
9780521740494c10.xml CUAU033-EVANS August 22, 2009 9:35
R
e
v
i
e
w
Chapter 10 Circular functions I 293
4 The depth of water, h(t ) m, at a particular jetty in a harbour at time t hours after midnight is
given by the rule h(t ) = p +q sin
_
t
6
_
. The graph of h(t ) against t for t [0, 24] is as
shown.
h(t)
10.2
6
t
(24, 6)
1.8
O
The maximum depth is 10.2 m and the minimum depth is 1.8 m.
a Find the values of p and q.
b State the times at which the depth of water is a maximum for the interval of time [0, 24].
c What is the average depth of the water in the time interval [0, 24]?
d At what times in the time interval [0, 24] is the depth of the water 3.9 m?
e For how long in the 24-hour period from midnight is the water more than 8.1 m in depth?
5 For the function f : [0, 2] R with rule f (x) = 2 sin(3x) +1
a Find the values of k such that the equation f (x) = k has
i six solutions for x [0, 2] ii three solutions for x [0, 2]
iii no solutions for x [0, 2]
b Find a sequence of transformations which takes the graph of y = f (x) to the graph of
y = sin x.
c Find the values of h [0, 2] such that
i there is a maximum for the graph of y = f (x +h) at the point
_

3
, 3
_
ii there is a minimum for the graph of y = f (x +h) at the point
_

3
, 1
_
6 a Find a sequence of transformations which takes the graph of y = cos x to the graph of
y = sin x.
b Find a sequence of transformations which takes the graph of y = 2 cos x to the graph of
y =
1
2
sin 2x.
c i Find the rule for the image of the graph of f (x) = sin x under a dilation of factor
2

from the y axis, followed by reection in the line y = 2.


ii Find the range and period of the new function.
P1: FXS/ABE P2: FXS
9780521740494c10.xml CUAU033-EVANS August 22, 2009 9:35
R
e
v
i
e
w
294 Essential Advanced General Mathematics
7 Two pistons A and B move backwards and forwards in a cylinder as shown.
x cm
y cm
O
A B
The distance x centimetres of the right hand end of piston A from the point O at time t
seconds is modelled by the rule
x = 4 sin (3t ) +4
and the distance y centimetres of the left hand end of piston B from the point O at time t
seconds is modelled by the rule
y = 2 sin
_
2t

6
_
+10
The pistons are set in motion at time t = 0.
a State the value of x and the value of y when t = 0
b i State the amplitude of the motion of piston A.
ii State the amplitude of the motion of piston B.
c i State the maximum and minimum values of x.
ii State the maximum and minimum values of y.
d i State the period of the motion of piston A.
ii State the period of the motion of piston B.
e Find the time(s) in the rst cycle of A that its distance from O is a maximum.
f Find the next four values of t for which x takes its maximum value.
g Find the values of t, 0 t 4, for which y attains its minimum value.
h On the one set of axes draw the graphs of x = 4 sin (3t ) +4 and
y = 2 sin
_
2t

6
_
+10 over the interval [0, ].
i State the time when the pistons rst touch each other.
j How many seconds are there between the rst and second times the pistons touch?
8 The pistons A and B (from 7) are adjusted so that the distance x cm of the right hand end of
piston A from point O at time t seconds is modelled by the formula
x = a sin (nt ) +b
and the distance y centimetres of the left hand end of piston B from the point O at time t
seconds is modelled by the formula
y = c sin (mt ) +d
The pistons meet every second at a point 8 cm from O. The right hand end of piston A
cannot go to the left of the point O.
a Find one possible set of values of a, b, n and c, m, d and explain your solution.
b Using the set of values found in a, sketch the graphs of x against t and y against t on the
one set of axes.
P1: FXS/ABE P2: FXS
9780521740494c10.xml CUAU033-EVANS August 22, 2009 9:35
R
e
v
i
e
w
Chapter 10 Circular functions I 295
9 The population, N, of a particular species of ant varies with the seasons. The population is
modelled by the equation N = 3000 sin
_
(t 1)
6
_
+4000, where t is the number of
months after 1 January in a given year.
The population, M, of a second species of ant also varies with time. Its population is
modelled by the equation M = 3000 sin
_
(t 3.5)
5
_
+5500, where t is again the
number of months after 1 January in a given year. Use a graphics calculator to sketch the
graphs of both equations over a period of one year on the same axes and nd
a the maximum and minimum populations of both species and the months in which those
maxima and minima occur
b during which month of the year the populations of both species are equal and the
population of each species at that time
c by formulating a third equation, when the combined population of species N and M is at
a maximum and what that maximum is
d by formulating a fourth equation, when the difference between the two populations is a
maximum.
10 Passengers on a ferris wheel ride access their seats from a platform 5 m above the ground.
As each seat is lled the ferris wheel moves around so that the next seat can be lled. Once
all seats are lled the ride begins and lasts for 6 minutes. The height h m of Isobels seat
above the ground t seconds after the ride has begun is given by the equation
h = 15 sin (10t 45)

+16.5.
a Use a graphics calculator to sketch the graph of the equation for the rst 2 minutes of
the ride.
b How far above the ground is Isobels seat at the commencement of the ride?
c After how many seconds does Isobels seat pass the access platform?
d How many times will her seat pass the access platform in the rst 2 minutes?
e How many times will her seat pass the access platform during the entire ride?
Due to a malfunction the ride stops abruptly 1 minute and 40 seconds into the ride.
f How far above the ground is Isobel stranded?
g If Isobels brother Hamish had a seat 1.5 m above the ground at the commencement of
the ride, how far above the ground was Hamish stranded?
11 The transformation of rotation about the origin in an anticlockwise direction by

is
dened by the matrix
R

=
_
cos sin
sin cos
_
a Find the matrices
i R

and hence deduce identities for cos ( +) and sin ( +)


ii R

= R
2

and hence deduce identities for cos 2 and sin 2


iii R
2

= R
3

and hence deduce identities for cos 3 and sin 3


P1: FXS/ABE P2: FXS
9780521740494c10.xml CUAU033-EVANS August 22, 2009 9:35
R
e
v
i
e
w
296 Essential Advanced General Mathematics
b Find the matrix R
1

. What does this matrix represent?


c Rotation about another point with coordinates (a, b) can be dened by the following
sequence of transformations:
r
translation dened by the vector
_
a
b
_
r
rotation dened by the matrix R

r
translation dened by the vector
_
a
b
_
i Find the image of the point (x, y) following rotation about the point (a, b).
ii Find the image of the point (x, y) following rotation about the point (1, 1).
d i If R

_
x
y
_
=
_
x

_
, nd x

and y

in terms of x and y.
ii Find the image of the curve of y = x
2
under a rotation of 45

around the origin in an


anticlockwise direction.
iii Find the image of the line y = x under a rotation of

around the origin in an


anticlockwise direction.
e i Find the image of the points with coordinates
(1, 0) and (0, 1) after reection in the line
with equation y = (tan )x.
x
y
(0, 1)
(1, 0)
0

y = (tan)x
ii Given that this transformation
may be represented by a 2 2 matrix
_
a b
c d
_
, nd the values of a, b, c and d
in terms of .
iii Use a matrix technique to show that
a rotation of degrees about the origin
can be achieved through the composition
of two reections.
f Find the image of the curve with equation y = x
2
under a reection in the line y = x.
P1: FXS/ABE P2: FXS
9780521740494c11.xml CUAU033-EVANS August 24, 2009 8:32
C H A P T E R
11
Circular functions II
Objectives
To further explore the symmetry properties of circular functions
To further understand and sketch the graphs of circular functions
To solve circular function equations
To evaluate simple trigonometric expressions using trigonometric identities
To prove simple trigonometric identities
To apply addition theorems for circular functions
To apply double angle formulas for circular functions
To simplify expressions of the form acos x + bsinx
To sketch graphs of functions of the form f (x) = acos x + bsinx
To solve equations of the form acos x + bsinx = c
y
x

a
b
a
b
P()

2
P

2

y
x
b
b
a
a

2
+
P()
P

2
+

11.1 Further symmetry properties


Complementary relationships
sin

= a
and since a = cos
sin

= cos
Similarly
cos

= b
and since b = sin
cos

= sin
sin

2
+

= a = cos
cos

2
+

= b = sin
297
P1: FXS/ABE P2: FXS
9780521740494c11.xml CUAU033-EVANS August 24, 2009 8:32
298 Essential Advanced General Mathematics
Example 1
If sin = 0.3 and cos = 0.8, nd the values of
a sin

b cos

2
+

c sin()
Solution
a
sin

= cos
= 0.8
b
cos

2
+

= sin
= 0.3
c
sin () = sin
= 0.3
Exercise 11A
1 If sin x = 0.3, cos = 0.6 and tan = 0.7, nd the values of
Example 1
a cos() b sin

2
+

c tan() d cos

2
x

e sin(x) f tan

g cos

2
+ x

h sin

i sin

3
2
+

j cos

3
2
x

11.2 Addition of ordinates


Example 2
Using the same scale and axes, sketch the graphs of y
1
= 2 sin x and y
2
= 3 cos 2x for
0 x 2.
Use addition of ordinates to sketch the graph of y = 2 sin x +3 cos 2x.
x
y
3
2
1
0
1
2
3
5
4

2
2
y
1
= 2sinx
y
2
= 3cos2x
y = 2sinx + 3cos2x
3
2
Solution
The graphs of y
1
= 2 sin x and y
2
= 3 cos 2x are shown below.
To obtain points on the graph of y = 2 sin x +3 cos 2x the process of addition of
ordinates is used.
Let y = y
1
+ y
2
when y
1
= 2 sin x and y
2
= 3 cos 2x
e.g., at
x = 0, y = 0 +3 = 3
x =

4
, y =
2

2
+0 =
2

2
=

2
x =

2
, y = 2 3 = 1
x = , y = 0 +3 = 3
x =
3
2
,y = 2 3 = 5
and so on.
P1: FXS/ABE P2: FXS
9780521740494c11.xml CUAU033-EVANS August 24, 2009 8:32
Chapter 11 Circular functions II 299
Using the TI-Nspire
Check that the calculator is in Radian
mode. Open a Graphs & Geometry
application ( 2) and enter the
functions
f 1 (x) = 2 sin (x)
f 2 (x) = 3 cos (2x)
f 3 (x) = f 1 (x) + f 2 (x)
The graph of f 3 (x) is the heavier line.
Add Function Table (b2 ) and split
the screen as shown using the Tools menu
(/ 523) to see that the values
of f 1 (x) and f 2 (x) add to give f 3 (x).
Use the down arrow ( ) to see more x
values.
The table of values may also be obtained
by selecting a Lists & Spreadsheet
application and pressing /T.
Using the Casio ClassPad
Check that the calculator is Radian mode. Choose
Graphs & Tab. from the Menu.
Enter
y1 = 2 sin(x)
y2 = 3 cos(2x)
y3 = 2 sin(x) +3 cos(2x).
Tap the graph icon to obtain the graph.
Tap the table icon to obtain a table and then the
table input icon to obtain the desired table.
Graph icon Table icon Table input
P1: FXS/ABE P2: FXS
9780521740494c11.xml CUAU033-EVANS August 24, 2009 8:32
300 Essential Advanced General Mathematics
The table input window has been completed as
shown.
Another way to achieve this addition of functions is to Dene f (x) = 2 sin(x) and
g(x) = 3 cos(2x). Then enter the functions as shown.
Note also that the graph of ( f + g)(x) has been drawn with a thick line. The menu
shown to choose this as shown. It can be accessed by taping on the line segment to the
right of y3.
Exercise 11B
1 Use addition of ordinates to sketch the graphs of
Example 2
a y = 2 sin +cos b y = 3 cos 2 +2 sin 2
c y =
1
2
sin 2 cos d y = 3 sin +cos 2
e y = 4 sin 2 cos
11.3 Sketch graphs of the tangent function
A table of values for y = tan x is given below. Use a calculator to check these values.
x
3
4

4
0

4

2
3
4

5
4
3
2
7
4
2
9
4
5
2
11
4
3
y 0 1 undened 1 0 1 undened 1 0 1 undened 1 0 1 undened 1 0
P1: FXS/ABE P2: FXS
9780521740494c11.xml CUAU033-EVANS August 24, 2009 8:32
Chapter 11 Circular functions II 301
The graph of y = tan x is given below.
x
y

1
1
0
2
2 3
2 3
8 9 4 5 6 7 10

2
3
2

2
5
2
3 4 2 1
1
2

Note: x =

2
,

2
,
3
2
and
5
2
are asymptotes.
Observations from the graph
1 The graph repeats itself every units, i.e., the period of tan is .
2 The range of tan is R.
Exercise 11C
1 Sketch the graph of each of the following, showing one complete cycle.
a y = tan 2x b y = 2 tan 3x c y = 2 tan

x +

4

d y = 3 tan x +1 e y = 2 tan

x +

2

+1 f y = 3 tan 2

x

4

2
11.4 General solution of circular function equations
The solution of circular function equations has been discussed in Section 10.9 for functions
over a restricted domain. In this section, we consider the general solutions of such equations
over the maximal domain for each function.
If a circular function equation has one or more solutions in one cycle, then it will have
corresponding solutions in each cycle of its domain, i.e., there will be an innite number of
solutions.
For example, if cos x = a, then the solution in the interval [0, ] is given by:
x = cos
1
(a)
By the symmetry properties of the cosine function, other solutions are given by:
cos
1
(a), 2 +cos
1
(a), 2 cos
1
(a), 4 +cos
1
(a),
4 cos
1
(a), . . . and so on.
In general, if cos (x) = a,
x = 2n cos
1
(a), where n Z and a [1, 1]
P1: FXS/ABE P2: FXS
9780521740494c11.xml CUAU033-EVANS August 24, 2009 8:32
302 Essential Advanced General Mathematics
Similarly, if tan (x) = a,
x = n +tan
1
(a), where n Z and a R
If sin (x) = a,
x = 2n +sin
1
(a) or x = (2n +1) sin
1
(a), where n Z and a [1, 1]
Note: An alternative and more concise way to express the general solution of sin (x) = a is:
x = n +(1)
n
sin
1
(a), where n Z and a [1, 1]
Example 3
Find the general solution to each of the following equations.
a cos (x) = 0.5 b

3 tan (3x) = 1 c 2 sin (x) =

2
Solution
a x = 2n cos
1
(0.5)
= 2n

3
=
(6n 1)
3
, n Z
b tan (3x) =
1

3
3x = n +tan
1

= n +

6
=
(6n +1)
6
x =
(6n +1)
18
, n Z
c sin (x) =
1

2
x = 2n +sin
1

or x = (2n +1) sin


1

= 2n +

4
= (2n +1)

4
=
(8n +1)
4
, n Z =
(8n +3)
4
, n Z
Using the TI-Nspire
Check that the calculator is in Radian mode.
a Use Solve( ) from the Algebra menu
(b31) and complete as shown.
Note the use of
1
2
rather than 0.5 to
ensure that the answer is exact.
P1: FXS/ABE P2: FXS
9780521740494c11.xml CUAU033-EVANS August 24, 2009 8:32
Chapter 11 Circular functions II 303
b Complete as shown.
c Complete as shown.
Using the Casio ClassPad
a Enter and highlight the equation cos (x) = 0.5,
tap Interactive, Equation/inequation, solve
and ensure the variable is set to x.
b Enter and highlight the equation

3 tan (3x) = 1, tap Interactive,


Equation/inequation, solve and ensure the
variable is set to x.
c Enter and highlight the equation

2 sin (x) = 1,
tap Interactive, Equation/inequation, solve
and ensure the variable is set to x.
Example 4
Find the rst three positive solutions to each of the following equations.
a cos (x) = 0.5 b

3 tan (3x) = 1 c 2 sin (x) =

2
P1: FXS/ABE P2: FXS
9780521740494c11.xml CUAU033-EVANS August 24, 2009 8:32
304 Essential Advanced General Mathematics
Solution
a The general solution (from Example 3) is given by x =
(6n 1)
3
, n Z
When n = 0, x =

3
, and when n = 1, x =
5
3
or x =
7
3
The rst three positive solutions of cos (x) = 0.5 are x =

3
,
5
3
,
7
3
b The general solution (from Example 3) is given by x =
(6n +1)
18
, n Z
When n = 0, x =

18
, and when n = 1, x =
7
18
, and when n = 2, x =
13
18
The rst three positive solutions of

3 tan (3x) = 1 are x =



18
,
7
18
,
13
18
c The general solution (from Example 3) is given by x =
(8n +1)
4
or
x =
(8n +3)
4
, n Z
When n = 0, x =

4
or
3
4
, and when n = 1, x =
9
4
or x =
11
4
The rst three positive solutions of 2 sin (x) =

2 are x =

4
,
3
4
,
9
4
Exercise 11D
1 Find the general solution to each of the following equations.
Example 3
a sin (x) = 0.5 b 2 cos (3x) =

3 c

3 tan (x) = 3
2 Find the rst two positive solutions to each of the following equations.
Example 4
a sin (x) = 0.5 b 2 cos (3x) =

3 c

3 tan (x) = 3
3 Find the general solution to 2 cos

2x +

4

2, and hence nd all the solutions for x in


the interval (2, 2).
4 Find the general solution to

3 tan

6
3x

1 = 0, and hence nd all the solutions for


x in the interval [, 0].
5 Find the general solution to 2 sin (4x) +

3 = 0, and hence nd all the solutions for x in


the interval [1, 1].
11.5 Trigonometric identities
Reciprocal functions
The functions sin, cos, and tan can be used to form three other functions called the reciprocal
circular functions.
sec =
1
cos
(cos = 0) cosec =
1
sin
(sin = 0)
cot =
cos
sin
(sin = 0)
Note: For cos = 0 and sin = 0, cot =
1
tan
and tan =
1
cot
P1: FXS/ABE P2: FXS
9780521740494c11.xml CUAU033-EVANS August 24, 2009 8:32
Chapter 11 Circular functions II 305
Example 5
Find the exact value of each of the following.
a sec
2
3
b cot
5
4
c cosec
7
4
Solution
a sec
2
3
=
1
cos
2
3
=
1
cos

=
1
cos

3
=
1

1
2
= 2
b cot
5
4
=
cos
5
4
sin
5
4
=
cos

+

4

sin

+

4

=
1

2

1

2
= 1
c cosec
7
4
=
1
sin

2

4

=
1
sin

=
1

2
=

2
Example 6
Find the value(s) of x between 0 and 2 for which
a sec x = 2 b cot x = 1
Solution
a sec x = 2

1
cos x
= 2
cos x =
1
2
x =

3
or x = +

3
i.e., x =
2
3
or x =
4
3
b cot x = 1
implies
tan x = 1
x =

4
or x = 2

4
i.e. x =
3
4
or x =
7
4
Using the TI-Nspire
Check that the calculator is in Radian mode.
Use solve( ) from the Algebra menu (b31) as shown.
P1: FXS/ABE P2: FXS
9780521740494c11.xml CUAU033-EVANS August 24, 2009 8:32
306 Essential Advanced General Mathematics
Using the Casio ClassPad
a Enter and highlight the equation
1
cos(x)
= 2,
tap Interactive, Equation/inequation,
solve and ensure the variable is set to x.
b Enter and highlight the equation
1
tan(x)
= 1,
tap Interactive, Equation/inequation,
solve and ensure the variable is set to x.
Note: The restriction to the domain can be
used as shown in the Appendix.
The Pythagorean identity
Consider a point, P(), on the unit circle.
By Pythagoras theorem:
OP
2
= OM
2
+MP
2
1 = (cos )
2
+(sin )
2
Now (cos )
2
and (sin )
2
may be written as cos
2
and sin
2
.
x
y
1
1
1
1
sin
cos M O
P()
Since this is true for all values of it is called an identity.
In particular this is called the Pythagorean identity.
cos
2
+sin
2
= 1
Other forms of the identity can be derived.
Dividing both sides by cos
2
gives:
cos
2

cos
2

+
sin
2

cos
2

=
1
cos
2

1 +tan
2
= sec
2

Dividing both sides by sin


2
gives:
cos
2

sin
2

+
sin
2

sin
2

=
1
sin
2

cot
2
+1 = cosec
2

P1: FXS/ABE P2: FXS


9780521740494c11.xml CUAU033-EVANS August 24, 2009 8:32
Chapter 11 Circular functions II 307
Example 7
a If cosec x =
7
4
, nd cos x. b If sec x =
3
2
, nd sin x where

2
x .
Solution
a Since cosec x =
7
4
, sin x =
4
7
Now cos
2
x +sin
2
x = 1
so cos
2
x +
16
49
= 1
cos
2
x =
33
49
cos x =

33
7
b Since sec x =
3
2
, cos x =
2
3
cos
2
x +sin
2
x = 1

4
9
+sin
2
x = 1
sin x =

5
3
For P(x) in the 2nd quadrant, sin x is
positive
sin x =

5
3
Example 8
If sin =
3
5
and

2
< < , nd the value of cos and tan .
Solution
Since cos
2
+sin
2
= 1
then cos
2
+
3
2
5
2
= 1
cos
2
= 1
9
25
=
16
25
cos =
4
5
since

2
< <
tan =
3
4
as tan =
sin
cos
P1: FXS/ABE P2: FXS
9780521740494c11.xml CUAU033-EVANS August 24, 2009 8:32
308 Essential Advanced General Mathematics
Example 9
Prove the identity
1
1 cos
+
1
1 +cos
= 2 cosec
2

Solution
LHS =
1
1 cos
+
1
1 +cos
=
1 +cos +1 cos
1 cos
2

=
2
1 cos
2

=
2
sin
2

= 2 cosec
2

= RHS
Exercise 11E
1 Find the exact value of each of the following.
Example 5
a cot
3
4
b cot
5
4
c sec
5
6
d cosec

2
e sec
4
3
f cosec
13
6
g cot
7
3
h sec
5
3
2 Without using a calculator write down the exact value of each of the following.
a cot 135

b sec 150

c cosec 90

d cot 240

e cosec 225

f sec 330

g cot 315

h cosec 300

i cot 420

3 Find the values of x between 0 and 2 for which


Example 6
a cosec x = 2 b cot x =

3 c sec x +

2 = 0 d cosec x = sec x
4 If sec =
17
8
and

2
< < , nd
Example 7
Example 8
a cos b sin c tan
5 If tan =
7
24
and
3
2
< < 2, nd cos and sin .
6 Find the value of sec if tan = 0.4 and is not in the 1st quadrant.
7 If tan =
4
3
and < <
3
2
, evaluate
sin 2 cos
cot sin
.
8 If cos =
2
3
and is in the 4th quadrant, nd the simplest expression in surd form for
tan 3 sin
cos 2 cot
.
P1: FXS/ABE P2: FXS
9780521740494c11.xml CUAU033-EVANS August 24, 2009 8:32
Chapter 11 Circular functions II 309
9 Prove each of the following identities for suitable values of and .
Example 9 Example 9
a (1 cos
2
)(1 +cot
2
) = 1 b cos
2
tan
2
+sin
2
cot
2
= 1
c
tan
tan
=
tan +cot
cot +tan
d (sin +cos )
2
+(sin cos )
2
= 2
e
1 +cot
2

cot cosec
= sec f sec +tan =
cos
1 sin
11.6 Addition formulas and double angle formulas
Addition formulas
Consider a unit circle.
Let arc length AB = v units
arc length AC = u units
arc length CB = u v units
x
y
u v
C
A
B
(cos u, sin u)
(cos v, sin v)
1
O
1
1
Rotate OCB so that B is coincident with A.
The point P has coordinates
(cos (u v), sin (u v)).
Since the triangles CBO and PAO are congruent,
CB = PA
u v
P
A
O 1
x
1
(1, 0)
(cos(u v), sin(u v))
y
Applying the coordinate distance formula
CB
2
= (cos u cos v)
2
+(sin u sin v)
2
= 2 2(cos u cos v +sin u sin v)
PA
2
= (cos (u v) 1)
2
+(sin (u v) 0)
2
= 2 2(cos (u v))
Equating these
2 2(cos u cos v +sin u sin v) = 2 2(cos (u v))
cos (u v) = cos u cos v +sin u sin v
Using the TI-Nspire and the ClassPad
Use tExpand( )from the Trigonometry
submenu of the Algebra menu (b3
1) as shown.
ClassPad
The command and tExpand is found
through Interactive Transformation
tExpand.
P1: FXS/ABE P2: FXS
9780521740494c11.xml CUAU033-EVANS August 24, 2009 8:32
310 Essential Advanced General Mathematics
Replacing v with v
cos (u (v)) = cos u cos (v) +sin u sin (v)
From symmetry properties
cos () = cos
sin () = sin
cos (u +v) = cos u cos v sin u sin v
Example 10
Evaluate cos 75

.
Solution
cos 75

= cos (45

+30

)
= cos 45

cos 30

sin 45

sin 30

=
1

3
2

1

2

1
2
=

3 1
2

2
=

3 1
2

2
=

2
4
Replacing u with

2
u in cos (u v)
cos

2
u

= cos

2
u

cos v +sin

2
u

sin v
Applying symmetry properties
sin = cos

and cos = sin

cos

2
(u +v)

= sin u cos v +cos u sin v

sin (u +v) = sin u cos v +cos u sin v


Replacing v with v
sin (u v) = sin u cos (v) +cos u sin (v)
sin (u v) = sin u cos v cos u sin v
Example 11
Evaluate
a sin 75

b sin 15

.
P1: FXS/ABE P2: FXS
9780521740494c11.xml CUAU033-EVANS August 24, 2009 8:32
Chapter 11 Circular functions II 311
Solution
a sin 75

= sin (30

+45

)
= sin 30

cos 45

+cos 30

sin 45

=
1
2

1

2
+

3
2

1

2
=
1 +

3
2

2
=
1 +

3
2

2
=

2 +

6
4
b sin 15

= sin (45

30

)
= sin 45

cos 30

cos 45

sin 30

=
1

3
2

1

2

1
2
=

3 1
2

2
=

3 1
2

2
=

2
4
Addition formula for tangent
Also tan (u +v) =
sin (u +v)
cos (u +v)
=
sin u cos v +cos u sin v
cos u cos v sin u sin v
Divide the numerator and denominator by cos u cos v = 0
tan (u +v) =
tan u +tan v
1 tan u tan v
Similarly it can be shown that
tan (u v) =
tan u tan v
1 +tan u tan v
Example 12
If tan u = 4 and tan v =
3
5
and u and v are acute angles, show that u v =

4
.
P1: FXS/ABE P2: FXS
9780521740494c11.xml CUAU033-EVANS August 24, 2009 8:32
312 Essential Advanced General Mathematics
Solution
tan (u v) =
tan u tan v
1 +tan u tan v
=
4
3
5
1 +4
3
5
=
17
5
17
5
= 1
u v =

4
Note: tan is a one-to-one function for 0 < <

2
Derivation of the addition formulas using
rotation about the origin
The use of matrices to describe rotations about
the origin has been discussed in Section 10.11.
We can use matrices as an alternative method to
derive the addition formulas. Consider, for
example, the point with coordinates
(cos (u +v), sin (u +v)), which can be regarded
as the image of a point with coordinates (cos u, sin u)
under a rotation of v
c
in an anticlockwise
direction around the origin.
x
y
O
v
u
(cos(u + v), sin(u + v))
(cos u, sin u)
The matrix that denes a rotation of v radians anticlockwise about the origin is given by

cos v sin v
sin v cos v

i.e.,

cos v sin v
sin v cos v

x
y

becomes

cos (u +v)
sin (u +v)

cos v sin v
sin v cos v

cos u
sin u

i.e., cos (u +v) = cos v cos u sin v sin u or cos (u +v) = cos u cos v sin u sin v
and sin (u +v) = sin v cos u +cos v sin u or sin (u +v) = sin u cos v +cos u sin v
Similarly, consider a point
(cos (u v), sin (u v)), which can be
regarded as the image of a point with
coordinates (cos u, sin u) under a rotation
of v
c
in a clockwise direction around the
origin.
O
u
v
(cos(u v), sin(u v))
(cos u, sin u)
x
y
P1: FXS/ABE P2: FXS
9780521740494c11.xml CUAU033-EVANS August 24, 2009 8:32
Chapter 11 Circular functions II 313
The matrix that denes a rotation of v radians clockwise about the origin is given by

cos (v) sin (v)


sin (v) cos (v)

cos v sin v
sin v cos v

i.e.,

cos v sin v
sin v cos v

x
y

becomes

cos (u v)
sin (u v)

cos v sin v
sin v cos v

cos u
sin u

i.e., cos (u v) = cos v cos u +sin v sin u or cos (u v) = cos u cos v +sin u sin v
and sin (u v) = sin v cos u +cos v sin u or sin (u v) = sin u cos v cos u sin v
Double angle formulas
cos (u +v) = cos u cos v sin u sin v
Replacing v with u
cos (u +u) = cos u cos u sin u sin u
cos 2u = cos
2
u sin
2
u
= 2 cos
2
u 1
= 1 2 sin
2
u
since sin
2
u = 1 cos
2
u
since cos
2
u = 1 sin
2
u
Similarly, replacing v with u in sin (u +v) = sin u cos v +cos u sin v
sin 2u = sin u cos u +cos u sin u
sin 2u = 2 sin u cos u
Replacing v with u in tan (u +v) =
tan u +tan v
1 tan u tan v
tan (u +u) =
tan u +tan u
1 tan u tan u
tan 2u =
2 tan u
1 tan
2
u
Example 13
If tan =
4
3
and 0 < <

2
, evaluate
a sin 2 b tan 2
P1: FXS/ABE P2: FXS
9780521740494c11.xml CUAU033-EVANS August 24, 2009 8:32
314 Essential Advanced General Mathematics
5 4
3

Solution
a sin =
4
5
and cos =
3
5
sin 2 = 2 sin cos
= 2
4
5

3
5
=
24
25
b tan 2 =
2 tan
1 tan
2

=
2
4
3
1
16
9
=
8
3
7
9
=
24
7
Example 14
Prove each of the following identities.
a
2 sin cos
cos
2
sin
2

= tan 2 b
sin
sin
+
cos
cos
=
2 sin ( +)
sin 2
c
1
cos +sin
+
1
cos sin
= tan 2 cosec
Solution
a LHS =
2 sin cos
cos
2
sin
2

=
sin 2
cos 2
= tan 2
= RHS
Note: Identity holds when cos 2 = 0
b LHS =
sin
sin
+
cos
cos
=
sin cos +cos sin
sin cos
=
sin ( +)
1
2
sin 2
=
2 sin ( +)
sin 2
Note: Identity holds when sin 2 = 0
c LHS =
1
cos +sin
+
1
cos sin
=
cos sin +cos +sin
cos
2
sin
2

=
2 cos
cos 2
But 2 cos =
2 sin cos
sin
=
sin 2
sin
LHS =
sin 2
cos 2 sin
=
tan 2
sin
= tan 2 cosec
Note: Identity holds when cos 2 = 0
P1: FXS/ABE P2: FXS
9780521740494c11.xml CUAU033-EVANS August 24, 2009 8:32
Chapter 11 Circular functions II 315
Sometimes the easiest way to prove two expressions are equal is to simplify each of them. This
is demonstrated in the following example.
Example 15
Prove that (sec A cos A)(cosecA sin A) =
1
tan A +cot A
Solution
LHS = (sec A cos A)(cosecA sin A) RHS =
1
tan A +cot A
=

1
cos A
cos A

1
sin A
sin A

=
1
sin A
cos A
+
cos A
sin A
=
1 cos
2
A
cos A

1 sin
2
A
sin A
=
1
sin
2
A +cos
2
A
cos A sin A
=
sin
2
A cos
2
A
cos A sin A
=
cos A sin A
sin
2
A +cos
2
A
= cos A sin A = cos A sin A
LHS = RHS
Exercise 11F
1 By making use of the appropriate addition formula nd the exact values for each of the
Example 10
following.
a cos 15

b cos 105

2 By making use of the appropriate addition formula nd exact values for each of the
Example 11
following.
a sin 165

b tan 75

3 Find exact values of


a cos
5
12
b sin
11
12
c tan

12
4 If sin u =
12
13
and sin v =
3
5
, evaluate sin (u +v). (Note: There is more than one answer.)
Example 12
5 Simplify the following.
a sin

+

6

b cos

c tan

+

3

d sin

6 Simplify
a cos (u v) sin v +sin (u v) cos v b sin (u +v) sin v +cos (u +v) cos v
P1: FXS/ABE P2: FXS
9780521740494c11.xml CUAU033-EVANS August 24, 2009 8:32
316 Essential Advanced General Mathematics
7 If sin =
3
5
and is in the 3rd quadrant and cos =
5
13
and is in the 2nd quadrant, Example 13
evaluate each of the following without using a calculator.
a cos 2 b sin 2 c tan 2 d sec 2
e sin ( +) f cos ( ) g cosec ( +) h cot 2
8 If tan u =
4
3
and tan v =
5
12
and both u and v are acute angles evaluate:
a tan (u +v) b tan 2u c cos (u v) d sin 2u
9 If sin =
3
5
and sin =
24
25
and

2
< < < evaluate
a cos 2 b sin ( ) c tan ( +) d sin (2)
10 If sin =

3
2
and cos =
1
2
evaluate
a sin 2 b cos 2
11 Simplify each of the following expressions.
a (sin cos )
2
b cos
4
sin
4

12 Prove the following identities,


Examples 14, 15
a

2 sin

= sin cos b cos

+cos

+

3

= cos
c tan

+

4

tan

= 1 d cos

+

6

+sin

+

3

3 cos
e tan

+

4

=
1 +tan
1 tan
f
sin (u +v)
cos u cos v
= tan v +tan u
g
tan u +tan v
tan u tan v
=
sin (u +v)
sin (u v)
h cos 2 +2 sin
2
= 1
i sin 4 = 4 sin cos
3
4 cos sin
3
j
1 sin 2
sin cos
= sin cos
11.7 acos x + bsinx
In Section 11.2 the method of addition of ordinates was used in the plotting of the sums of
circular functions. In this section it will be shown how functions with rule of the form
f (x) = a cos x +b sin x may have the rule written in terms of a single circular function.
First write
a cos x +b sin x =

a
2
+b
2

a
2
+b
2
cos x +
b

a
2
+b
2
sin x

a
2
+b
2
(cos cos x +sin sin x)
where cos =
a

a
2
+b
2
and sin =
b

a
2
+b
2
Let r =

a
2
+b
2
and thus
a cos x +b sin x = r cos (x )
P1: FXS/ABE P2: FXS
9780521740494c11.xml CUAU033-EVANS August 24, 2009 8:32
Chapter 11 Circular functions II 317
Similarly it may be shown that
a cos x +b sin x = r sin (x +)
where r =

a
2
+b
2
, sin =
a

a
2
+b
2
and cos =
b

a
2
+b
2
Example 16
Express cos x

3 sin x in the form r cos (x ) and hence nd the range of the function
with rule f (x) = cos x

3 sin x, and the maximum and minimum values of the function.


Solution
a = 1, b =

3 r =

1 +3 = 2
also cos =
a
r
=
1
2
and sin =
b
r
=

3
2
=

3
cos x

3 sin x = 2 cos

x +

3

Range of f is [2, 2]
The maximum and minimum values of f are 2 and 2 respectively.
Using the TI-Nspire
Use tCollect( ) from the Trigonometry
submenu of the Algebra menu (b3
2) as shown.
Example 17
Solve cos x

3 sin x = 1 for x [0, 2].


Solution
FromExample 16, cos x

3 sin x = 2 cos

x +

3

2 cos

x +

3

= 1
cos

x +

3

=
1
2
x +

3
=

3
,
5
3
,
7
3
x = 0,
4
3
, 2
P1: FXS/ABE P2: FXS
9780521740494c11.xml CUAU033-EVANS August 24, 2009 8:32
318 Essential Advanced General Mathematics
Using the TI-Nspire
Use solve( ) from the Algebra menu (b
31) as shown.
The symbol can be found in the
catalog ( 4), by typing , or by
typing / .
Example 18
Express

3 sin 2x cos 2x in the form r sin (2x +).


Solution
A slightly different technique is used.
Let

3 sin 2x cos 2x = r sin (2x +)
Then

3 sin 2x cos 2x = r[sin 2x cos +cos 2x sin ]
This is to hold for all x.
If x =

4
,

3 = r cos ... 1
If x = 0, 1 = r sin ... 2
Squaring and adding 1 and 2 gives
r
2
cos
2
+r
2
sin
2
= 4
i.e., r
2
= 4
r = 2
The positive solution is taken. Substituting in 1 and 2 gives

3
2
= cos and
1
2
= sin
=


3 sin 2x cos 2x = 2 sin

2x

6

Expand the right hand side of the equation to verify.


P1: FXS/ABE P2: FXS
9780521740494c11.xml CUAU033-EVANS August 24, 2009 8:32
Chapter 11 Circular functions II 319
Exercise 11G
1 Find the maximum and minimum values of the following.
Example 16
a 4 cos x +3 sin x b

3 cos x +sin x c cos x sin x
d cos x +sin x e 3 cos x +

3 sin x f sin x

3 cos x
g cos x

3 sin x +2 h 5 +3 sin x 2 cos x


2 Solve each of the following for x [0, 2] or

[0, 360].
Example 17
a sin x cos x = 1 b

3 sin x +cos x = 1
c sin x

3 cos x = 1 d 3 cos x

3 sin x = 3
e 4 sin

+3 cos

= 5 f 2

2 sin

2 cos

= 3
3 Write

3 cos 2x sin 2x in the form r cos (2x +).


4 Write cos 3x sin 3x in the form r sin (3x ).
Example 18
5 Sketch the graphs of the following, showing one cycle.
a f (x) = sin x cos x b f (x) =

3 sin x +cos x
c f (x) = sin x +cos x d f (x) = sin x

3 cos x
P1: FXS/ABE P2: FXS
9780521740494c11.xml CUAU033-EVANS August 24, 2009 8:32
R
e
v
i
e
w
320 Essential Advanced General Mathematics
Chapter summary
Further symmetry properties: complementary angles
sin

= cos
sin

2
+

= cos
cos

= sin
cos

2
+

= sin
Addition of ordinates

y
3
2
1
0
y
1
= 2 sin x
y
2
= 3 cos 2x
y = 2 sin x + 3 cos 2x

2
2
3
2
1
4
5
2
3
Graph of tangent function
y

1
2
2

2
3
2 2
5
1


y = tan
period =
General solution of circular function equations
If cos (x) = a, x = 2n cos
1
(a), where n Z and a [1, 1]
If tan (x) = a, x = n +tan
1
(a), where n Z and a R
If sin (x) = a, x = 2n +sin
1
(a), or
x = (2n +1) sin
1
(a), where n Z and a [1, 1]
Reciprocal circular functions
secant = sec =
1
cos
cosecant = cosec =
1
sin
cotangent = cot =
1
tan
, sin = 0 and cos = 0
P1: FXS/ABE P2: FXS
9780521740494c11.xml CUAU033-EVANS August 24, 2009 8:32
R
e
v
i
e
w
Chapter 11 Circular functions II 321
Pythagorean identity
cos
2
+sin
2
= 1
1 +tan
2
= sec
2

cot
2
+1 = cosec
2

Addition formulas
cos (u v) = cos u cos v +sin u sin v
cos (u +v) = cos u cos v sin u sin v
sin (u +v) = sin u cos v +cos u sin v
sin (u v) = sin u cos v cos u sin v
tan (u +v) =
tan u +tan v
1 tan u tan v
tan (u v) =
tan u tan v
1 +tan u tan v
Double angle formulas
cos 2u = cos
2
u sin
2
u
= 2 cos
2
u 1
= 1 2 sin
2
u
sin 2u = 2 sin u cos u
tan 2u =
2 tan u
1 tan
2
u
a cos x +b sin x can be written as r cos (x )
where r =

a
2
+b
2
and cos =
a

a
2
+b
2
and sin =
b

a
2
+b
2
It can also be written as r sin (x +)
where r =

a
2
+b
2
and sin =
a

a
2
+b
2
and cos =
b

a
2
+b
2
Multiple-choice questions
1 cosec x sin x is equal to
A cos x cot x B cosec x tan x C 1 sin
2
x
D sin x cosec x E
1 sin x
sin x
2 If cos x =
1
3
, the possible values of sin x are
A
2

2
3
,
2

2
3
B
2
3
,
2
3
C
8
9
,
8
9
D

2
3
,

2
3
E
1
2
,
1
2
P1: FXS/ABE P2: FXS
9780521740494c11.xml CUAU033-EVANS August 24, 2009 8:32
R
e
v
i
e
w
322 Essential Advanced General Mathematics
3 If cos =
a
b
and 0 < <

2
, then tan in terms of a and b is
A

a
2
+b
2
b
B

b
2
a
2
a
C
a

b
2
a
2
D
a

b
2
+a
2
E
a
b

b
2
+a
2
4 The magnitude of ABX is , AX = 4 cm, XC = x cm
and BC = 2 cm. In terms of x, tan is equal to
A
8
(x +2)
2
B
4
x
C 8 x D 8 + x
E
8

x
2
+4
2 cm

4 cm
x cm
X
C
A
B
5 For

2
< A < and < B <
3
2
, with cos A = t and sin B = t , sin (B + A) is equal to
A 0 B 1 C 2t
2
1 D 1 2t
2
E 1
6
sin 2A
cos 2A 1
is equal to
A cot 2A 1 B sin 2A +sec 2A
C
sin A
cos A 1
D sin 2A tan 2A E cot A
7 sin

2
x

is not equal to
A cos (2 x) B sin

3
2
+ x

C sin x D cos (x) E sin

2
+ x

8 (1 +cot x)
2
+(1 cot x)
2
is equal to
A 2 +cot x +2 cot 2x B 2 C 4 cot x D 2 +cot 2x E 2cosec
2
x
9 If sin 2A = m and cos A = n, tan A in terms of m and n is equal to
A
m
2n
2
B
n
m
C
2n
m
2
D
2n
m
E
2n
2
m
10 cos x +sin x, in the form r sin (x +) where r > 0, is
A

2 sin

x +

4

B sin

x +

4

C

2 sin

x +
5
4

D

2 sin

x +
7
4

E

2 sin

x +
3
4

Short-answer questions (technology-free)


1 Prove each of the following identities.
a sec +cosec cot = sec cosec
2
b sec sin =
tan
2
+cos
2

sec +sin
2 Find the maximum and minimum values of each of the following.
a 3 +2 sin b 1 3 cos c 4 sin
3
2

d 2 sin
2
1
2
e
1
2 +cos
P1: FXS/ABE P2: FXS
9780521740494c11.xml CUAU033-EVANS August 24, 2009 8:32
R
e
v
i
e
w
Chapter 11 Circular functions II 323
3 Find the values of , [0, 2], for which
a sin
2
=
1
4
b sin 2 =
1
2
c cos 3 =

3
2
d sin
2
2 = 1 e tan
2
=
1
3
f tan 2 = 1
g sin 3 = 1 h sec 2 =

2
4 Solve the equation tan = 2 sin for values of from 0

to 360

.
5 If sin A =
5
13
, sin B =
8
17
where A and B are acute, nd
a cos (A + B) b sin (A B) c tan (A + B)
6 Find
a cos 80

cos 20

+sin 80

sin 20

b
tan 15

+tan 30

1 tan 15

tan 30

7 If A + B =

2
, nd the value of
a sin A cos B +cos A sin B b cos A cos B sin A sin B
8 Find the maximum and minimum values of the function with rule
a 3 +2 sin b 4 5 cos
9 Prove each of the following.
a sin
2
A cos
2
B cos
2
A sin
2
B = sin
2
A sin
2
B b
sin
1 +cos
+
1 +cos
sin
=
2
sin
c
sin 2 sin
3

2 cos
3
cos
= tan
10 Given that sin A =

5
3
and that A is obtuse, nd the value of each of the following:
a cos 2A b sin 2A c sin 4A
11 Prove
a
1 tan
2
A
1 +tan
2
A
= cos 2A b
sin A
1 +cos A
+
1 +cos A
sin A
=
2
sin A
12 a Find tan 15

in simplest surd form.


b Using the identities for sin (u v), express 2 sin x cos y as the sum of two sines.
13 Given f : [0, 2] R, f (x) = 2

3 cos x 2 sin x, nd the coordinates of


a the y intercept b the x intercepts
c the maximum point d the minimum point.
Hence sketch the graph of f (x) = 2

3 cos x 2 sin x
14 Solve for x, 0 x 2.
a sin x +cos x = 1 b sin
1
2
x cos
1
2
x =
1
4
c 3 tan 2x = 2 tan x d sin
2
x = cos
2
x +1
e sin 3x cos x cos 3x sin x =

3
2
f 2 cos

2x

3

3
P1: FXS/ABE P2: FXS
9780521740494c11.xml CUAU033-EVANS August 24, 2009 8:32
R
e
v
i
e
w
324 Essential Advanced General Mathematics
15 Sketch graphs of
a y = 2 cos
2
x b y = 1 2 sin

2

x
2

c f (x) = tan 2x
16 It is given that tan A = 2. Find the exact value of tan , given that tan ( + A) = 4.
17 a Express 2 cos +9 sin in the form r cos ( ), where r > 0 and 0 < <

2
b i Give the maximum value of 2 cos +9 sin
ii Give the cosine of for which this maximum occurs.
iii Find the smallest positive solution of the equation 2 cos +9 sin = 1
Extended-response questions
1 The diagram shows a rectangle ABCD inside a semicircle, centre O and radius 5 cm.
BOA = COD =

a Show that the perimeter, P cm, of the rectangle


is given by
P = 20 cos +10 sin
b Express P in the form r cos ( ) and hence
nd the value of for which P = 16.

A
B C
D O
5 cm 5 cm
c Find the value of k for which the area of the rectangle is k sin 2 cm
2
.
d Find the value of for which the area is a maximum.
2 The diagram shows a vertical section through a tent
in which AB = 1 m, BC = 2 m and
BAD = BCD = . CD is horizontal.
The diagram is symmetrical about the vertical AD.
a Obtain an expression for AD in terms of .
b Express AD in the form
r cos ( ), where r is positive.
c State the maximum length of AD
and the corresponding value of .
A
B
C D
1 m
2 m

d Given that AD = 2.15 m, nd the value of for which > .


3 a Prove the identity cos 2 =
1 tan
2

1 +tan
2

b
i Use the result of a to show 1 + x
2
=

2x
2

2 where x = tan

67
1
2

ii Hence nd the values of integers a and b such that tan

67
1
2

= a +b

2
c Find the value of tan

7
1
2

.
P1: FXS/ABE P2: FXS
9780521740494c11.xml CUAU033-EVANS August 24, 2009 8:32
R
e
v
i
e
w
Chapter 11 Circular functions II 325
4 In the diagram triangle ABC has a right angle at B.
Length of BC = 1 unit.
a Find in terms of
i h
1
ii h
2
iii h
3
iv h
n
A
h
1
h
2
h
3
B
C

b Show that the innite sum h


1
+h
2
+h
3
+. . . =
cos
1 sin
c If the innite sum =

2, nd .
5 ABCD is a regular pentagon with side length one unit.
The exterior angles of a regular pentagon each have
magnitude
2
5
.
a i Show that the magnitude of BCA is

5
ii Find the length of CA
b i Show the magnitude of DCP is
2
5
ii Use the fact that AC = 2CQ = 2CP +PR
to show that 2 cos

5
= 2 cos
2
5
+1
B
C
P Q R
A
D E
2
5
iii Use the identity cos 2 = 2 cos
2
1 to form a quadratic equation in terms of
cos

5
iv Find the exact value of cos

5
6 a Prove each of the identities
i cos =
1 tan
2

2
1 +tan
2

2
ii sin =
2 tan

2
1 +tan
2

2
b Use the result of a to nd the value of tan

2
, given 8 cos sin = 4
P1: FXS/ABE P2: FXS
9780521740494c12.xml CUAU033-EVANS October 22, 2008 22:46
C H A P T E R
12
Trigonometric ratios
and applications
Objectives
To solve practical problems using the trigonometric ratios
To use the sine rule and the cosine rule to solve problems
To find the area of a triangle given two sides and an included angle
To find the area of a sector and a segment of a circle
To find the length of an arc
To solve problems involving angles of depression and angles of elevation
To identify the line of greatest slope of a plane
To solve problems in three dimensions including determining the angle between
planes
12.1 Defining sine, cosine and tangent
The unit circle is a circle of radius 1 with centre at the origin.
x
y
(1, 0) (0, 0)
(0, 1)
(1, 1)
(0, 1)
Sine and cosine may be dened for any angle
through the unit circle.
P(cos(), sin())

x
y
(0, 0)
For the angle of

, a point P on the unit circle is


dened as illustrated here. The angle is measured
in an anticlockwise direction from the positive
direction of the x axis.
326
P1: FXS/ABE P2: FXS
9780521740494c12.xml CUAU033-EVANS October 22, 2008 22:46
Chapter 12 Trigonometric ratios and applications 327
Cos (

) is dened as the x coordinate of the point P and sin (

) is dened as the y coordinate


of P. A calculator gives approximate values for these coordinates.
30
(0.8660, 0.5)
x
y
135
(0.7071, 0.7071)
x
y
100
(0.1736, 0.9848)
x
y
sin 30

= 0.5 (exact value) sin 135

=
1

2
0.7071 cos 100

= 0.1736
cos 30

3
2
0.8660 cos 135

=
1

2
0.7071 sin 100

= 0.9848
In this chapter, angles greater than 180

or less than 0

will
not be considered.
For a right-angled triangle OBC, a similar triangle OB

can be constructed that lies in the unit circle. From the diagram,
OC

= cos (

) and C

= sin (

)
The scale factor is the length OB.
Hence BC = OB sin (

) and OC = OB cos (

)
This implies
BC
OB
= sin (

) and
OC
OB
= cos (

)
B'
O
C'
C
B
1

This gives the ratio denition of sine and cosine for a right-angled triangle. The naming of
sides with respect to an angle

is as shown.

B
O
C
hypotenuse
adjacent
opposite
sin

=
opp
hyp

opposite
hypotenuse

cos

=
adj
hyp

adjacent
hypotenuse

tan

=
opp
adj

opposite
adjacent

P1: FXS/ABE P2: FXS


9780521740494c12.xml CUAU033-EVANS October 22, 2008 22:46
328 Essential Advanced General Mathematics
From the unit circle, note that
sin (

) = sin (180 )

, e.g. sin 45

= sin 135

and cos (

) = cos (180 )

, e.g. cos (45

) = cos (135

)
(cos(), sin())

(cos(180 ), sin(180 ))
x
y
(180 )
0
This result will be used later in this chapter.
Example 1
Find the value of x correct to two decimal places.
B
x cm
C
80 cm
A
29.6
Solution
x
80
= sin 29.6

x = 80 sin 29.6

= 39.5153 . . .
x = 39.52 correct to two decimal places
Example 2
Find the length of the hypotenuse correct to two decimal places.
C
A
B
15
10 cm
Solution
10
AB
= cos 15

10 = ABcos 15

AB =
10
cos 15

= 10.3527 . . .
The length of the hypotenuse = 10.35 cm correct to two decimal places.
Example 3
Find the magnitude of ABC.
A
B C
3 cm
11 cm
x
Solution
tan x =
11
3
x = tan
1
11
3
x = (74.74 . . .)

x = 74

44

42

(to the nearest second).


Remember that this is read as 74 degrees, 44 minutes and 42 seconds.
P1: FXS/ABE P2: FXS
9780521740494c12.xml CUAU033-EVANS October 22, 2008 22:46
Chapter 12 Trigonometric ratios and applications 329
Exercise 12A
1 Find the value of x in each of the following.
Example 1
a
x cm
5 cm
35
b
10 cm
x cm
5
c
x cm
20.16
8 cm
d
x cm
7 cm
3015'
e
x
15 cm
10 cm
f
40
x cm
10 cm
2 An equilateral triangle has altitudes of length 20 cm. Find the length of one side.
Example 2
3 The base of an isosceles triangle is 12 cm long and the equal sides are 15 cm long. Find
Example 3
the magnitude of each of the three angles of the triangle.
4 A pole casts a shadow 20 m long when the altitude
of the sun is 49

. Calculate the height of the pole.


20 m
pole
49
5 This gure represents a ramp.
A
B
C
6 m
1 m
a Find the magnitude of angle ACB.
b Find the distance BC.
6 This gure shows a vertical mast PQ, which
stands on horizontal ground. A straight wire 20 m
long runs from P at the top of the mast to a point R
on the ground, which is 10 m from the foot of the mast.
a Calculate the angle of inclination,

, of the wire to the ground.


b Calculate the height of the mast.
20 m
10 m
R
P
Q

P1: FXS/ABE P2: FXS


9780521740494c12.xml CUAU033-EVANS October 22, 2008 22:46
330 Essential Advanced General Mathematics
7 A ladder leaning against a vertical wall makes an angle of 26

with the wall. If the foot of


the ladder is 3 m from the wall, calculate
a the length of the ladder b the height it reaches above the ground.
8 An engineer is designing a straight concrete entry ramp, 60 m long, for a car park 13 m
above street level. Calculate the angle of the ramp to the horizontal.
9 A vertical mast is secured from its top by straight cables 200 m long xed at the ground.
The cables make angles of 66

with the ground. What is the height of the mast?


10 A mountain railway rises 400 m at a uniform slope of 16

with the horizontal. What is the


distance travelled by a train for this rise?
11 The diagonals of a rhombus bisect each other at
right angles.
A D
B C
If BD = AC = 10 cm, nd
a the length of the sides of the rhombus
b the magnitude of angle ABC.
12 A pendulum swings from the vertical through an
angle of 15

on each side of the vertical. If the


pendulum is 90 cm long, what is the distance x cm
between its highest and lowest point?
90 cm 90 cm
x cm
13 A picture is hung symmetrically by means of
a string passing over a nail with its ends attached
to two rings on the upper edge of the picture.
The distance between the rings is 30 cm and the
angle between the two portions is 105

. Find the
length of the string.
105
30 cm
14 The distance AB = 50 m. If the line of sight of a
person standing at A to the tree makes an angle of 32

with the bank, how wide is the river?


50 m
B A
32
P1: FXS/ABE P2: FXS
9780521740494c12.xml CUAU033-EVANS October 22, 2008 22:46
Chapter 12 Trigonometric ratios and applications 331
15 A ladder 4.7 m long is placed against a wall. The foot of the ladder must not be placed in
a ower bed, which extends a distance of 1.7 m from the foot of the wall. How high up the
wall can the ladder reach?
16 A river is known to be 50 m wide. A swimmer sets
off from A to cross the river and the path of the
swimmer AB is as shown. How far does the person
swim?
50 m
B
A
60
12.2 The sine rule
In Section 12.1, methods for nding unknown lengths and angles for right-angled triangles
were discussed. In this section and the next, methods for nding unknown quantities in
non-right-angled triangles are discussed.
The sine rule is used to nd unknown quantities in a triangle when one of the following
situations arises:
one side and two angles are given
two sides and a non-included angle are given.
In the rst of the two cases, a unique triangle is dened, but for the second it is possible for two
triangles to exist.
Labelling convention
The following convention is followed in the remainder of this
module. Interior angles are denoted by upper case letters and
the length of the side opposite an angle is denoted by the
corresponding lower case letter.
B
A C
a c
b
For example, the magnitude of angle BAC is denoted by A,
and the length of side BC is denoted by a.
The sine rule states that for triangle ABC
B
A C
a c
b
a
sin A
=
b
sin B
=
c
sin C
A proof will only be given for the acute-angled triangle case. The proof for obtuse-angled
triangles is similar.
Proof
C
a
B
h
b
A
D
In triangle ACD, sin A =
h
b
h = b sin A
In triangle BCD, sin B =
h
a
h = a sin B
a sin B = b sin A
i.e.,
a
sin A
=
b
sin B
P1: FXS/ABE P2: FXS
9780521740494c12.xml CUAU033-EVANS October 22, 2008 22:46
332 Essential Advanced General Mathematics
Similarly, starting with a perpendicular from A to BC would give
b
sin B
=
c
sin C
Example 4
Use the sine rule to nd the length of AB.
Solution
B
70
c
A 10 cm
31
C
c
sin 31

=
10
sin 70

c =
10 sin 31

sin 70

c = 5.4809 . . .
The length of AB is 5.48 cm correct to two decimal places.
Example 5
Use the sine rule to nd the magnitude of angle
XZY in the triangle, given that Y = 25

, y = 5 cm,
and z = 6 cm.
Z
5 cm
6 cm
25
Y X
Solution
5
sin 25

=
6
sin Z

sin Z
6
=
sin 25

5
sin Z =
6 sin 25

5
= 0.5071 . . .
Z = sin
1
(0.5071 . . .)
Z = (30.4736 . . .)

or (180 30.4736 . . .)

Z = 30

28

25

or Z = 149

31

35

(to the nearest second)


Remember: sin (180 ) = sin
Z
1
Z
2
30 28' 25"
5 cm
X
5 cm
6 cm
25
Y
149 31' 35"
There are two solutions for the equation sin Z = 0.5071 . . .
Note: When using the sine rule in the situation where two sides and a non-included
angle are given, the possibility of two such triangles existing must be considered.
Existence can be checked through the sum of the given angle and the found angle not
exceeding 180

.
P1: FXS/ABE P2: FXS
9780521740494c12.xml CUAU033-EVANS October 22, 2008 22:46
Chapter 12 Trigonometric ratios and applications 333
Exercise 12B
1 Find the value of the pronumeral for each of the following triangles.
Example 4
a
Y
70
50
X
10 cm
Z
x cm
b
Z
65
y cm
X
6 cm
37
Y
c
Z
5.6 cm
100
x cm
Y
28
X
d
Y X
x cm
12 cm
Z
90
38
2 Find the value of for each of the following triangles.
Example 5
a
72
7 cm
A
8 cm

B
C
b
A

9.4 cm
C
8.3 cm
B
42
c
C
10 cm
108
8 cm
A

B
d
B
8 cm

9 cm
C
38
A
3 Solve the following triangles (i.e. nd all sides and angles).
a a = 12, B = 59

, C = 73

b A = 75.3

, b = 5.6, B = 48.25

c A = 123.2

, a = 11.5, C = 37

d A = 23

, a = 15, B = 40

e B = 140

, b = 20, A = 10

4 Solve the following triangles (i.e. nd all sides and angles).


a b = 17.6, C = 48.25

, c = 15.3
b B = 129

, b = 7.89, c = 4.56
c A = 28.35

, a = 8.5, b = 14.8
5 A landmark A is observed from two points B and C, which are 400 m apart. The magnitude
of angle ABC is found to be 68

and the magnitude of angle ACB is 70

. Find the distance


of A from C.
P1: FXS/ABE P2: FXS
9780521740494c12.xml CUAU033-EVANS October 22, 2008 22:46
334 Essential Advanced General Mathematics
6 P is a point at the top of a lighthouse. Measurements
of the length of AB and angles PBO and PAO are taken
and are as shown in the diagram. Find the height of the
lighthouse.
P
O
46.2
B
27.6
34 m A
7 A and B are two points on a coastline. They are 1070 m apart. C is a point at sea. The angles
CAB and CBA have magnitudes of 74

and 69

respectively. Find the distance of C from A.


8 Find
a AX b AY
X
A
88
32
50 m
20
89
B
Y
12.3 The cosine rule
The cosine rule is used to nd unknown quantities in a triangle when one of the following
situations arises:
two sides and an included angle are given
three sides are given.
The cosine rule states that for triangle ABC
B
a
b
c
A C
a
2
= b
2
+c
2
2bc cos A or equivalently
cos A =
b
2
+c
2
a
2
2bc
The symmetrical results also hold, i.e.
b
2
= a
2
+c
2
2ac cos B
c
2
= a
2
+b
2
2ab cos C
The result will be proved for an acute-angled triangle. The proof for obtuse-angled triangles is
similar.
Proof
In triangle ACD
C
b
h
a
B
c
A
D
x
b
2
= x
2
+h
2
(Pythagoras theorem)
cos A =
x
b
and therefore x = b cos A
P1: FXS/ABE P2: FXS
9780521740494c12.xml CUAU033-EVANS October 22, 2008 22:46
Chapter 12 Trigonometric ratios and applications 335
In triangle BCD
a
2
= (c x)
2
+h
2
(Pythagoras theorem)
Expanding gives
a
2
= c
2
2cx + x
2
+h
2
= c
2
2cx +b
2
(as x
2
+h
2
= b
2
)
a
2
= b
2
+c
2
2bc cos A (as x = b cos A)
Example 6
For triangle ABC, nd the length of AB in centimetres
correct to two decimal places.
c
A
B
5 cm
67
10 cm
C
Solution
c
2
= 5
2
+10
2
2 5 10 cos 67

= 85.9268 . . .
c 9.2697
The length of AB is 9.27 cm correct to two decimal places.
Example 7
Find the magnitude of angle ABC for triangle ABC.
Solution
cos B =
a
2
+c
2
b
2
2ac
=
12
2
+6
2
15
2
2 12 6
= 0.3125
B = (108.2099 . . .)

B 108

12

35.845

The magnitude of angle ABC is 108

12

36

(to the nearest second).


B
12 cm
15 cm
6 cm
A
C
Exercise 12C
1 Find the length of BC.
Example 6
10 cm
B
15
15 cm
C A
P1: FXS/ABE P2: FXS
9780521740494c12.xml CUAU033-EVANS October 22, 2008 22:46
336 Essential Advanced General Mathematics
2 Find the magnitude of angles ABC and ACB.
Example 7
B
8 cm
10 cm
A
5 cm
C
3 For triangle ABC with
a A = 60

b = 16 c = 30, nd a
b a = 14 B = 53

c = 12, nd b
c a = 27 b = 35 c = 46, nd the magnitude of angle ABC
d a = 17 B = 120

c = 63, nd b
e a = 31 b = 42 C = 140

, nd c
f a = 10 b = 12 c = 9, nd the magnitude of angle BCA
g a = 11 b = 9 C = 43.2

, nd c
h a = 8 b = 10 c = 15, nd the magnitude of angle CBA
4 A section of an orienteering course is as shown.
Find the length of leg AB.
B
A
6 km
4 km
20
C
5 Two ships sail from point O. At a particular time their
positions A and B are as shown. Find the distance between
the ships at this time.
N
B
4 km
O
30
6 km
A
6 ABCD is a parallelogram. Find the length of the diagonals:
a AC
b BD
5 cm B
4 cm
48
A
D
C
P1: FXS/ABE P2: FXS
9780521740494c12.xml CUAU033-EVANS October 22, 2008 22:46
Chapter 12 Trigonometric ratios and applications 337
7 A weight is hung from two hooks in a ceiling by strings
of length 54 cm and 42 cm, which are inclined at 70

to
each other. Find the distance between the hooks.
54 cm
42 cm
70
8 a Find the length of diagonal BD.
b Use the sine rule to nd the length of CD.
B
4 cm
5 cm
92
6 cm
88
D
C
A
9 Two circles of radius 7.5 cm and 6 cm have a
common chord of length 8 cm.
a Find the magnitude of angle AO

B.
b Find the magnitude of angle AOB.
A
7.5 cm
O
8 cm O'
6 cm
B
10 Two straight roads intersect at an angle of 65

. A point
A on one road is 90 m from the intersection and a point
B on the other road is 70 m from the intersection,
as shown on the diagram.
a Find the distance of A from B.
b C is the midpoint of AB. Find the distance
of C from the intersection.
A
C
B
65
90 m
O
70 m
12.4 Area of a triangle
It is known that the area of a triangle is given by the formula
Area =
1
2
bh
B
h
A
b
C
Area =
1
2
base length height
By observing that h = c sin A the following formula can be found.
P1: FXS/ABE P2: FXS
9780521740494c12.xml CUAU033-EVANS October 22, 2008 22:46
338 Essential Advanced General Mathematics
Area of triangle =
1
2
bc sin A
i.e., the area is given by half the product of the length of two sides and the sine of the
angle included between them.
Example 8
Find the area of triangle ABC shown in the
diagram.
B
140 6.5 cm
7.2 cm
A
C
Solution
Area =
1
2
7.2 6.5 sin 140

= 15.04 cm
2
The area of triangle ABC is 15.04 cm
2
correct to two decimal places.
Example 9
Find the area of each of the following triangles, correct to three decimal places.
a
A
10 cm
6.4 cm
C
8 cm
B
b
8.2 cm
70
85
F
D
E
c
G
7 cm
I
10 cm
H
12
Solution
a Using the cosine rule,
8
2
= 6.4
2
+10
2
2 6.4 10 cos C
64 = 140.96 128 cos C
cos C = 0.60125
C

= (126.95 . . .)

(the exact value can be stored on the


graphics calculator as C, say)
Area of triangle ABC =
1
2
6.4 10 sin C
= 25.570 cm
2
, correct to three decimal places.
P1: FXS/ABE P2: FXS
9780521740494c12.xml CUAU033-EVANS October 22, 2008 22:46
Chapter 12 Trigonometric ratios and applications 339
b E

= (180 (70 +85))

= 25

Using the sine rule,


DF = sin (25

)
8.2
sin (85

)
= 3.47 . . . (the exact value can be stored on
the graphics calculator as E, say)
Area of triangle DEF =
1
2
8.2 E sin (70

)
= 13.403 cm
2
, correct to three decimal places.
c Using the sine rule,
sin I = 10
sin (12

)
7
= 0.2970 . . .
I

= (180 17.27 . . .)

since I is an obtuse angle


= (162.72 . . .)

(the exact value can be stored on


the graphics calculator as I, say)
G

= (180 (12 + I ))

= (5.27 . . .)

(the exact value can be stored on


the graphics calulator as G, say)
Area of triangle GHI =
1
2
10 7 sin (G

)
= 3.220 cm
2
, correct to three decimal places.
Exercise 12D
1 Find the area of each of the following triangles.
Example 8
a C
70
6 cm
4 cm
B A
b X
5.1 cm
72.8 6.2 cm
Z
Y
c
N
3.5 cm
130
M
8.2 cm
L
d
A
C
B
5 cm
25
5 cm
P1: FXS/ABE P2: FXS
9780521740494c12.xml CUAU033-EVANS October 22, 2008 22:46
340 Essential Advanced General Mathematics
2 Find the area of each of the following triangles, correct to three decimal places.
Example 9
a A
B
C
5.9 cm
4.1 cm
3.2 cm
b A
B
C
9 cm
100
7 cm
c E
F
D
6.3 cm
55
65
d
E
F
D
5.7 cm
5.9 cm
5.1 cm
e
5 cm
12 cm
24
G
I
H
f
4 cm
19
10
G
I
H
12.5 Circle mensuration
Terminology
In this circle with centre O, the interval AB is called a chord of
the circle. A chord is an interval with endpoints on the circle.
If the centre of the circle is on the chord, the interval is called
a diameter, e.g. interval CD in the diagram. Any two points on
a circle divide the circle into arcs. The shorter arc is called the
minor arc, the longer is the major arc, e.g. arc ACB is a minor
arc and ADB is a major arc in this diagram. Note that arc DBC and arc DAC are semicircular
arcs in this diagram. Every chord divides the interior of a circle into two regions called
segments. The smaller is called the minor segment, the larger is the major segment. In the
above diagram the minor segment has been shaded.
A
C
B
O
D
Two radii and an arc dene a region called a sector. In this
diagram with circle centre O, the shaded region is a
minor sector and the unshaded region is a major sector.
Formulas to nd arc lengths, chord lengths and areas of
regions inside a circle will now be developed.
A
C
B
O
D
P1: FXS/ABE P2: FXS
9780521740494c12.xml CUAU033-EVANS October 22, 2008 22:46
Chapter 12 Trigonometric ratios and applications 341
Arc length
The arc ACB and the corresponding chord AB are said to subtend
the angle AOB at the centre of the circle. If the magnitude of
AOB =

and radius length is r units, then l units, the length


of arc ACB, will be a fraction of the circumference.
A
O
r
C
B
D
Since circumference = 2r
l =

360
2r
=
r
180
Now since

180
= where

=
c
l = r where
c
= mag AOB
Chord length
From the diagram, the cosine rule gives
A
O
r
r
B

AB
2
= r
2
+r
2
2r
2
cos
= 2r
2
(1 cos )
AB =

2r
2
(1 cos )
In triangle OAP,
B
A
r
O
P

2
AP = r sin

2
AB = 2r sin

2
Note: 1 cos = 1

1 2 sin
2

2

= 2 sin
2

2
Area of sector
If mag AOB =

the area of the sector is a fraction of the


area of the circle. Now area of circle is given by
r
A
B
O
area of circle = r
2
area of sector = fraction of r
2
=
r
2

360
Again using =

180
Area of sector =
1
2
r
2
where
c
= mag AOB
Example 10
In this circle, centre O, radius length 10 cm, the angle subtended
at O by arc ACB has magnitude 120

. Find
B
A
C
O
10 cm
10 cm
120
a the exact lengths of
i the chord AB ii the arc ACB
P1: FXS/ABE P2: FXS
9780521740494c12.xml CUAU033-EVANS October 22, 2008 22:46
342 Essential Advanced General Mathematics
b the exact area of the minor sector AOB
c the magnitude of angle AOC, in degrees and minutes, if the minor arc AC has length 4 cm.
Solution
a i Use chord length = 2r sin

2
where r = 10 and = 120

chord length = 20 sin 60

= 20

3
2
= 10

3
Length of chord is 10

3 cm.
ii Use l = r where r = 10, =
2
3
(note use of radians)
= 10
2
3
=
20
3
Length of arc is
20
3
cm.
(Verify that length of arc is greater than length of chord as a check.)
b Use area of sector =
1
2
r
2
where r = 10, =
2
3
(note use of radians)
=
1
2
10
2

2
3
=
100
3
So area of minor sector AOB =
100
3
cm
2
.
c Use arc length = r
A
C
O

4 cm
10 cm
4 = 10
=
4
10
angle AOC = 0.4
180

= (22.918 . . .)

= 22

55

(to the nearest minute)


Area of segment
Area of segment shaded = area of minor sector OAB area of AOB
A
B
r
O

So A =
r
2

360

1
2
r
2
sin
Where mag AOB =

but if mag AOB =


c
, =

180
A =
1
2
r
2

1
2
r
2
sin
=
1
2
r
2
(sin )
P1: FXS/ABE P2: FXS
9780521740494c12.xml CUAU033-EVANS October 22, 2008 22:46
Chapter 12 Trigonometric ratios and applications 343
Generally speaking the formulas are simpler if mag AOB is measured in radians. The
following formulas assume is in radians.
Arc length = r
Chord length = 2r sin

2
Area of sector =
1
2
r
2

Area of segment =
1
2
r
2
( sin )
Example 11
A circle, centre O, with radius length 20 cm has a chord AB that is 10 cm from the centre of the
circle. Calculate the area of the minor segment formed by this chord.
Solution
Now area of segment =
1
2
r
2
( sin )
20 cm
10 cm
A
O
B
C

r = 20 but needs to be calculated.


In OCB, cos

2
=
10
20


2
= 60
and = 120
Hence mag AOB =
2
c
3
Area of segment =
1
2
20
2

2
3
sin
2
3

cm
2
= 200

2
3

3
2

cm
2
= 200

4 3

3
6

cm
2
=
100

4 3

3
cm
2
Exercise 12E
1 Find the arc length which subtends an angle of magnitude 105

at the centre of a circle of


Example 10
radius length 25 cm.
2 Find the magnitude, in degrees and minutes, of the angle subtended at the centre of a
circle of radius length 30 cm, by
a an arc of length 50 cm
b a chord of length 50 cm.
3 A chord of length 6 cm is drawn in a circle of radius 7 cm. Find
Example 11
a the length of the minor arc cut off by the chord
b the area of the smaller region inside the circle cut off by the chord.
P1: FXS/ABE P2: FXS
9780521740494c12.xml CUAU033-EVANS October 22, 2008 22:46
344 Essential Advanced General Mathematics
4 Sketch, on the same set of axes, the graphs of
A = {(x, y) : x
2
+ y
2
16} and B = {(x, y) : y 2}
Find the area measure of the region A B.
5 Use results from Chapter 11 to show that

2r
2
(1 cos ) = 2r sin

2
6 Find the area of the region between an equilateral triangle of side length 10 cm and the
circumcircle of the triangle (the circle that passes through the three vertices of the
triangle).
7 A person stands on level ground 60 m from the nearest point of a cylindrical tank of
radius length 20 m. Calculate
a the circumference of the tank
b the percentage of the circumference that is visible to the person.
8 The minute hand of a large clock is 4 m long.
a How far does the tip of the minute hand move between 12.10 p.m. and 12.35 p.m?
b What is the area covered by the minute hand between 12.10 p.m. and 12.35 p.m?
9 Two circles of radii 3 cm and 4 cm have their centres 5 cm apart. Calculate the area of the
region common to both circles.
10 A sector of a circle has perimeter of 32 cm and an area of 63 cm
2
. Find the radius length
and the magnitude of the angle subtended at the centre of the two possible sectors.
11 Two wheels (pulleys) have radii of length 15 cm and 25 cm and have their centres 60 cm
apart. What is the length of the belt required to pass tightly around the pulleys without
crossing?
12 A frame in the shape of an equilateral triangle encloses three circular discs of radius
length 5 cm so that the discs touch each other. Find
a the perimeter of the smallest frame which can enclose the discs
b the area enclosed between the discs.
12.6 Angles of elevation and depression
and bearings
The angle of elevation is the angle between the horizontal
and a direction above the horizontal.
angle of elevation
eye level
lin
e
o
f s
ig
h
t
The angle of depression is the angle between the
horizontal and a direction below the horizontal.
angle of depression
lin
e
o
f
s
ig
h
t
eye level
cliff
P1: FXS/ABE P2: FXS
9780521740494c12.xml CUAU033-EVANS October 22, 2008 22:46
Chapter 12 Trigonometric ratios and applications 345
Example 12
The pilot of a helicopter ying at 400 m observes
a small boat at an angle of depression of 1.2

.
Calculate the horizontal distance of the boat to
the helicopter.
(diagram not to scale)
400 m
1.2 (angle of depression)
A
B
H
Solution
AH
AB
= tan 1.2

400
AB
= tan 1.2

AB =
400
tan 1.2

AB = 19 095.800 56 . . .
The horizontal distance is 19 100 m to the nearest 10 m.
Example 13
The light on a cliff-top lighthouse, known to be 75 m
above sea level, is observed from a boat at an angle
of elevation of 7.1

.
Calculate the distance of the boat from the lighthouse.
75 m
A
B
L
7.1
Solution
75
AB
= tan (7.1

)
AB =
75
tan (7.1

)
= 602.135 . . .
The distance of the boat from the lighthouse is 602 m to the nearest metre.
Example 14
From the point A, a man observes that the angle of elevation
of the summit of a hill is 10

. He then walks towards the hill


for 500 m along at ground. The summit of the hill is now
at an angle of elevation of 14

.
Find the height of the hill above the level of A.
166
4
14
500 m
10
A
B
C
H
Solution
The magnitude of angle HBA = (180 14)

= 166

The magnitude of angle AHB = [180 (166 +10)]

= 4

P1: FXS/ABE P2: FXS


9780521740494c12.xml CUAU033-EVANS October 22, 2008 22:46
346 Essential Advanced General Mathematics
Using the sine rule in triangle ABH:
500
sin 4

=
HB
sin 10

HB =
500 sin 10

sin 4

= 1244.67 . . .
In triangle BCH:
HC
HB
= sin 14

HC = HBsin 14

= 301.11 . . .
The height of the hill is 301 m to the nearest metre.
Bearings
The bearing (or compass bearing) is the direction
measured from north clockwise.
N
A
D
C
W E
S
B
30
210
120
330
O
The bearing of A from O is 030

The bearing of B from O is 120

The bearing of C from O is 210

The bearing of D from O is 330

Example 15
The road from town A runs due west for 14 km to town B.
A television mast is located due south of B at a distance
of 23 km. Calculate the distance and bearing of the
mast from the centre of town A.
14 km
23 km
B A
T
N

Solution
tan =
23
14
= 58.67

(to two decimal places)


bearing = 180

+(90 58.67)

= 211.33

By Pythagoras theorem
AT
2
= AB
2
+BT
2
= 14
2
+23
2
= 725
AT = 26.925 . . .
The mast is 27 km from the centre of town (to the nearest kilometre) and on a
bearing of 211.33

.
P1: FXS/ABE P2: FXS
9780521740494c12.xml CUAU033-EVANS October 22, 2008 22:46
Chapter 12 Trigonometric ratios and applications 347
Example 16
A yacht starts from a point A and sails on a bearing of 038

for
3000 m. It then alters its course to one in a direction with a
bearing of 318

and after sailing for 3300 m it reaches a point B. Find


a the distance AB
b the bearing of B from A.
3300 m
3000 m
A
B
C
38
42
318
N
N
Solution
a The magnitude of angle ACB needs to be determined
so that the cosine rule can be applied in triangle ABC.
A
B
C
38
38
42
N
N
The magnitude of angle ACB = (180 (38 +42))

= 100

In triangle ABC
AB
2
= 3000
2
+3300
2
2 3000 3300 cos (100

)
= 23 328 233.92
AB = 4829.931 04 . . .
The distance of B from A is 4830 m (to the nearest metre).
b To nd the bearing of B from A, the magnitude of angle BAC must rst be found.
The sine rule may be used.
3300
sin A
=
AB
sin 100

sin A =
3300 sin 100

AB
sin A = 0.672 8 . . .
A = (42.288 . . .)

The bearing of B from A


= 360

(42.29

38

)
= 355.71

.
The bearing of B from A is 356

to the nearest degree.


Exercise 12F
1 From the top of a vertical cliff 130 m high the angle of depression of a buoy at sea is 18

.
Example 12
What is the distance of the buoy from the foot of the cliff?
2 The angle of elevation of the top of an old chimney stack at a point 40 m from its base is
Example 13
41

. Find the height of the chimney.


P1: FXS/ABE P2: FXS
9780521740494c12.xml CUAU033-EVANS October 22, 2008 22:46
348 Essential Advanced General Mathematics
3 A man standing on top of a mountain observes that the angle of depression to the foot of a
building is 41

. If the height of the man above the foot of the building is 500 m, nd the
horizontal distance from the man to the building.
4 A man lying down on top of a cliff 40 m high observes the angle of depression to a buoy
in the sea below to be 20

. If he is in line with the buoy, calculate the distance between the


buoy and the foot of the cliff, which may be assumed to be vertical.
5 A man standing on top of a cliff 50 m high is in line with two buoys whose angles of
Example 14
depression are 18

and 20

. Calculate the distance between the buoys.


6 A ship sails 10 km north and then 15 km east. What is its bearing from the starting point?
Example 15
7 A ship leaves port A and steams 15 km due east. It then turns and goes 22 km due north.
a What is the bearing of the ship from A?
b What is the bearing of port A from the ship?
8 A yacht sails from point A on a bearing of 035

for 2000 m. It then alters course to a


Example 16
direction with bearing of 320

and after sailing for 2500 m it reaches point B.


a Find the distance AB. b Find the bearing of B from A.
9 The bearing of a point A from a point B is 207

. What is the bearing of B from A?


10 The bearing of a ship S from a lighthouse A is 055

. A second lighthouse B is due east of


A. The bearing of S from B is 302

. Find the magnitude of angle ASB.


11 A yacht starts from L and sails 12 km due east to M. It then sails 9 km on a bearing of
142

to K. Find the magnitude of angle MLK.


12 The bearing of C from A is 035

. The bearing of B from


A is 346

. The distance of C from A is 340 km. The


distance of B from A is 160 km.
a Find the magnitude of angle BAC.
b Use the cosine rule to nd the distance
from B to C.
N
340 km
160 km
346
35
B
A
C
13 From a ship S two other ships P and Q are on bearings 320

and 075

respectively. The
distance PS = 7.5 km and the distance QS = 5 km. Find the distance PQ.
12.7 Problems in three dimensions
Problems in three dimensions are solved by picking out triangles from a main gure and
nding lengths and angles through these triangles.
P1: FXS/ABE P2: FXS
9780521740494c12.xml CUAU033-EVANS October 22, 2008 22:46
Chapter 12 Trigonometric ratios and applications 349
Example 17
ABCDEFGH is a cuboid.
Find
a distance DB b distance HB
c the magnitude of angle HBD
d the magnitude of angle HBA.
H
G
F
C
D
E
A
B
7 cm
8 cm
10 cm
Solution
a
DB
2
= 8
2
+10
2
= 164
DB =

164
= 12.806 . . .
The length of DB is 12.81 cm correct to two decimal places.
8 cm
10 cm
D
A
B
b
HB
2
= HD
2
+DB
2
= 7
2
+164
= 49 +164
= 213
HB =

213
= 14.59 . . .
The length of HB is 14.59 cm correct to two decimal places.
H
D
B
7 cm

164 cm
c tan =
HD
BD
=
7

164
= 0.5466 . . .
= 28

40

to the nearest minute.


H
D
B
7 cm

164 cm
d From triangle HBA
cos B =
10

213
10

213
213
B = 46

45

to the nearest minute.


H
A
B
10 cm
Example 18
The gure shows a pyramid with a square base. The base has sides
6 cm long and the edges VA, VB, VC, VD are each 10 cm long.
a Find the length of DB.
b Find the length of BE.
c Find the length of VE.
d Find the magnitude of angle VBE.
V
B
C
D
A
E
6 cm
10 cm
P1: FXS/ABE P2: FXS
9780521740494c12.xml CUAU033-EVANS October 22, 2008 22:46
350 Essential Advanced General Mathematics
Solution
a
DB
2
= 6
2
+6
2
= 72
DB = 6

2
= 8.4852 . . .
The length of DB is 8.49 cm to two decimal places.
A B
C D
E
6 cm
6 cm
b BE =
1
2
DB
BE =
1
2

72
= 3

2
= 4.2426 . . .
The length of BE is 4.24 cm correct to two decimal places.
c
VE
2
= VB
2
EB
2
= 100
1
4
72
= 100 18
= 82
VE =

82
= 9.0553 . . .
The length of VE is 9.06 cm correct to two decimal places.
V
E
B
10 cm
d
sin =
VE
VB
=

82
10
= 0.9055 . . .
= 64

54

The magnitude of angle VBE is 64

54

to the nearest minute.


V
E B
10 cm

Example 19
A communications mast is erected at the corner, A, of a
rectangular courtyard ABCD whose sides measure 60 m
and 45 m. If the angle of elevation of the top of the mast
from C is 12

, nd
12
60 m
45 m
H
A
B
D
C
a the height of the mast
b the angle of elevation of the top of the mast from
B (where AB = 45 m).
P1: FXS/ABE P2: FXS
9780521740494c12.xml CUAU033-EVANS October 22, 2008 22:46
Chapter 12 Trigonometric ratios and applications 351
Solution
a
AC
2
= AB
2
+CB
2
= 45
2
+60
2
= 5625
AC = 75
HA
75
= tan 12

HA = 75 tan 12

= 15.9417
A
B C
60 m
45 m
The height of the mast is 15.94 m,
correct to two decimal places.
H
A
C
75 m
12
b tan =
HA
45
= 0.3542 . . .
19

30

The angle of elevation of the top of the mast, H, from B


is 19

30

to the nearest minute.

B A
H
45 m
Exercise 12G
1 ABCDEFGH is a cuboid with dimensions as shown. Find
Example 17
a the length of FH
b the length of BH
c the magnitude of angle BHF
d the magnitude of angle BHG.
A
B
C
D
E
H
G
F
12 cm
5 cm
8 cm
2 VABCD is a right pyramid with a square base. The sides of the base are 8 cm in length.
Example 18
The height, VF, of the pyramid is 12 cm. Find
a the length of EF
b the magnitude of angle VEF
c the length of VE
d the length of a sloping edge
e the magnitude of angle VAD
f the surface area of the pyramid.
V
C
A B
D
E
F
8 cm
3 A tree stands at the corner of a square playing eld. Each
Example 19
side of the square is 100 m long. At the centre of the eld
the tree subtends an angle of 20

. What angle does it


subtend at each of the other three corners of the eld?
100 m
20
100 m
A
B
C
T
P1: FXS/ABE P2: FXS
9780521740494c12.xml CUAU033-EVANS October 22, 2008 22:46
352 Essential Advanced General Mathematics
4 Suppose that A, C, and X are three points in a horizontal plane
and B is a point vertically above X. If the length of
AC = 85 m and the magnitudes of angles BAC, ACB and
BCX are 45

, 90

and 32

respectively, nd
a the distance CB
b the height XB.
A
B
X
C
85 m
45
32
5 Standing due south of a tower 50 m high, the angle of elevation of the top is 26

. What is
the angle of elevation after walking a distance 120 m due east?
6 From the top of a cliff 160 m high two buoys are observed. Their bearings are 337

and
308

. Their respective angles of depression are 3

and 5

. Calculate the distance between


the buoys.
7 Find the magnitude of each of the following angles
for the cuboid shown.
a ACE
b HDF
c ECH
H
E
A
D
F
C
B
G
12 cm
5 cm
6 cm
8 From a point A due north of a tower, the angle of elevation to the top of the tower is 45

.
From point B, 100 m on a bearing of 120

from A, the angle of elevation is 26

. Find the
height of the tower.
9 A and B are two positions on level ground. From an advertising balloon at a vertical
height of 750 m, A is observed in an easterly direction and B at a bearing of 160

. The
angles of depression of A and B as viewed from the balloon are 40

and 20

respectively.
Find the distance between A and B.
10 A right pyramid, height 6 cm, stands on a square base of side 5 cm. Find
a the length of a sloping edge
b the area of a triangular face.
11 A light aircraft ying at a height of 500 m above the ground
is sighted at a point A

due east of an observer stationed at


a point O on the ground, measured horizontally to be 1 km
from the plane. The aircraft is ying south west (along
A

) at 300 km/h.
A'
A
B
O
O'
B'
500 m
45
1000 m
a How far will it travel in one minute?
b Find its bearing from O (O

) at this time.
c What will be its angle of elevation from O at this time?
P1: FXS/ABE P2: FXS
9780521740494c12.xml CUAU033-EVANS October 22, 2008 22:46
Chapter 12 Trigonometric ratios and applications 353
12.8 Angles between planes and more
difficult 3-D problems
Angles between planes
Consider any point P on the common line of two planes

1
and
2
. If PA and PB are drawn at right angles to the
common line so that PA is in
1
and PB is in
2
, then
angle APB is the angle between
1
and
2
.
P

B
A

2
Note: If one of the planes,
2
say, is horizontal, then
PA is called a line of greatest slope in the plane
1
.
P
A
lines of
greatest slope
angle of
greatest slope

2
Example 20
Given the cuboid shown in the diagram, nd
a the angle between AC

and the plane ABB

b the angle between the planes ACD

and DCD

.
D'
D
A
A'
B'
C'
C
B
3a
3a
a
Solution
a To nd the angle between AC

and the plane ABB

,
we need the projection of AC

in the plane. So we
drop a perpendicular from C

to the plane, i.e. the


line C

, and join the foot of the perpendicular to


A, i.e. B

A. The required angle lies between C

A
and B

A.
D'
D
A
A'
C
C'
B'
B
a
3a
3a

Drawing separate diagrams showing the base and the section through A, C

and B

we have
A
B'
B
A'
3a
3a and

A
B'
C'
a
Thus AB

(3a)
2
+(3a)
2
= 3a

2
and tan =
a
3a

2
=
1
3

2
Hence the required angle, , is 13.3

.
P1: FXS/ABE P2: FXS
9780521740494c12.xml CUAU033-EVANS October 22, 2008 22:46
354 Essential Advanced General Mathematics
b The line common to the planes ACD

and DCD

is CD

. If M is the midpoint of this


line, then
MD is perpendicular to D

C in plane DCD

and
MA is perpendicular to D

C in plane D

CA.
Thus is the angle between the planes DCD

and D

CD.
D
D'
A
A'
M
C
C'
B'
a
B
3a
3a

D
A
M

But DM =
1
2
DC

=
1
2
(3a

2)
Hence tan = a

3a

2
2

2
3
i.e. the required angle is 25.2

Example 21
Three points A, B and C are on a horizontal line such that
AB = 70 m, and BC = 35 m. The angles of elevation of
the top of a tower are , and where tan =
1
13
,
tan =
1
15
and tan =
1
20
(as shown in the diagram).
The foot of the tower is at the same level as A, B and C.
Find the height of the tower.
P
A C
B
Q

70 m 35 m
tan =
1
13
tan =
1
15
tan =
1
20
Solution
Let the height of the tower, PQ, be h m.
Then h = QA tan = QB tan = QC tan
which implies QA = 13h, QB = 15h, QC = 20h
Now consider the base triangle ABCQ.
Q
A
B
C
15h
35 m 70 m
13h
20h

Using the cosine formula in AQB and CQB,


cos =
(70)
2
+(15h)
2
(13h)
2
2(70)(15h)
and cos (180 ) = cos =
(35)
2
+(15h)
2
(20h)
2
2(35)(15h)
P1: FXS/ABE P2: FXS
9780521740494c12.xml CUAU033-EVANS October 22, 2008 22:46
Chapter 12 Trigonometric ratios and applications 355
Hence
(70)
2
+(15h)
2
(13h)
2
2(70)(15h)
=
(20h)
2
(15h)
2
(35)
2
2(35)(15h)
4900 +56h
2
= 2(175h
2
1225)
7350 = 294h
2
Hence h = 5
The height of the tower is 5 m.
Example 22
A sphere rests on the top of a vertical cylinder which is open at the top. The inside diameter of
the cylinder is 8 cm. The sphere projects 8 cm above the top of the cylinder. Find the radius
length of the sphere.
Solution
This 3-D problem can be represented by a 2-D diagram without loss of information.
From the diagram, in OBC, if radius length of sphere is r cm,
OC = (8 r) cm, OB = r cm, BC = 4 cm
Using Pythagoras theorem
(8 r)
2
+4
2
= r
2
64 16r +r
2
+16 = r
2
16r +80 = 0
r = 5
So radius length of sphere is 5 cm.
A B
C
O
8 cm
8 cm
Example 23
A box contains two standard golf balls that t snugly inside. The box is 85 mm long. What
percentage of the space inside the box is air?
Solution
2-D diagrams may be used to represent the 3-D situation.
85 mm
side view end view
Use r mm = radius length of a ball
Now length of box = 85 mm = 4r mm
r =
85
4
i.e. r = 21.25
P1: FXS/ABE P2: FXS
9780521740494c12.xml CUAU033-EVANS October 22, 2008 22:46
356 Essential Advanced General Mathematics
So box has dimensions 85 mm by 42.5 mm by 42.5 mm
Now volume of box in mm
3
= 42.5
2
85 (using V = Ah)
volume of two golf balls = 2
4
3
21.25
3

using V =
4
3
r
3

=
8
3
21.25
3
So percentage air =
100

42.5
2
85
8
3
21.25
2

42.5
2
85
= 47.6% (to one decimal place)
Exercise 12H
1 The diagram shows a rectangular prism.
Example 20
AB = 4a units, BC = 3a units, GC = a units.
H
E
A
D
F
B
C
G
a Calculate the areas of the faces ABFE, BCGF, ABCD.
b Calculate the magnitude of the angle which
plane GFAD makes with the base.
c Calculate the magnitude of the angle which plane HGBA makes with the base.
d Calculate the magnitude of the angle which AG makes with the base.
2 VABCD is a right pyramid with square base ABCD.
AB = 2a and OV = a.
a Find the slope of edge VA, i.e., the magnitude of VAO.
b Find the slope of the face VBC.
D
O
B
C
V
A
3 A hill has gradient
5
12
. If BF makes an angle of 45

with
the line of greatest slope, nd
a the gradient of BF
b the magnitude of FBD.
F E
C
12
5
B
A
D
4 The cross-section of a right prism is an isosceles triangle ABC. AB = BC = 16 cm and
the magnitude of ABC = 58

. The equal edges AD, BE and CF are parallel and each of


length 12 cm. Calculate
a the length of AC b the length of AE
c the magnitude of the angle between AE and EC.
5 A vertical tower, AT, of height 50 m, stands at a point A on a horizontal plane. The points
Example 21
A, B, and C lie on the same horizontal plane, B is due west of A and C is due south of A.
The angles of elevation of the top, T, of the tower from B and C are 25

and 30

respectively.
a Calculate, giving answers to the nearest metre, the distances
i AB ii AC iii BC
b Calculate the angle of elevation of T from the midpoint, M, of AB.
P1: FXS/ABE P2: FXS
9780521740494c12.xml CUAU033-EVANS October 22, 2008 22:46
Chapter 12 Trigonometric ratios and applications 357
6 A right square pyramid, vertex O, stands on a square base ABCD. The height is 15 cm and
base side length is 10 cm. Find
a the length of the slant edge b the inclination of a slant edge to the base
c the inclination of a sloping face to the base
d the magnitude of the angle between two adjacent sloping faces.
7 A post stands at one corner of a rectangular courtyard. The elevations of the top of the
post from the nearest corners are 30

and 45

. Find the elevation from the diagonally


opposite corner.
8 VABC is a regular tetrahedron with base ABC. (All faces are equilateral triangles.) Find
the magnitude of the angle between
a a sloping edge and the base b adjacent sloping faces.
9 An observer at a point A at sea level notes an aircraft due east at an elevation of 35

. At
the same time an observer at B, 2 km due south of A, reports the aircraft on a bearing of
50

. Calculate the altitude of the aircraft.


10 Four congruent spheres, radius length 10 cm, are placed on a horizontal table so that each
Example 23
touches two others and their centres form a square. A fth congruent sphere rests on
them. Find the height of the top of this fth sphere above the table.
11 ABFE represents a section of a ski run which has a
uniform inclination of 30

to the horizontal.
AE = 100 m, AB = 100 m.
A skier traverses the slope from A to F. Calculate
A
D
E
F
C
B
a the distance that the skier has traversed
b the inclination of the skiers path to the horizontal.
12 A sphere of radius length 8 cm rests on the top of a hollow inverted cone of height 15 cm
Example 22
whose vertical angle is 60

. Find the height of the centre of the sphere above the vertex of
the cone.
13 A cube has edge length a cm. What is the radius length, in terms of a, of
a the sphere that just contains the cube b the sphere that just ts inside the cube?
14 In this diagram AB is vertical and BCD is horizontal. CBD is a right angle.
AB = 20 m, BD = 40 m, BC = 30 m. Calculate the inclination to the horizontal of
a AD
b AE where AE is the line of greatest slope
c AE where E is the midpoint of CD.
A
B
D
E
C
P1: FXS/ABE P2: FXS
9780521740494c12.xml CUAU033-EVANS October 22, 2008 22:46
R
e
v
i
e
w
358 Essential Advanced General Mathematics
Chapter summary
The sine rule is used to nd unknown quantities in a triangle when one of the following
situations arises:
r
one side and two angles are given
r
two sides and the non-included angle are given.
In the rst of the two cases a unique triangle is dened but for the second it is possible for
two triangles to exist.
Labelling convention
The following convention is followed. Interior angles are
denoted by upper case letters and the length of the side
opposite an angle is denoted by the corresponding
lower case letter. e.g.
A C
B
b
c
a
The magnitude of angle BAC is denoted by A.
The length of side BC is denoted by a.
The sine rule states that for a triangle ABC
A C
B
b
c
a
a
sin A
=
b
sin B
=
c
sin C
The cosine rule is used to nd unknown quantities in a triangle when one of the following
situations arises:
r
two sides and an included angle are given
r
three sides are given.
The cosine rule states that for a triangle ABC
A
C
B
b
c
a
a
2
= b
2
+c
2
2bc cos A or equivalently
cos A =
b
2
+c
2
a
2
2bc
The symmetrical results also hold, i.e.
b
2
= a
2
+c
2
2ac cos B
c
2
= a
2
+b
2
2ab cos C
It is known that the area of a triangle is given by the formula
Area =
1
2
bh
Area =
1
2
base length height
A C
B
b
c a
h
By observing that h = c sin A the following formula can be found:
Area of triangle =
1
2
bc sin A
i.e. the area is given by half the product of the length of two sides and the sine of the angle
included between them.
P1: FXS/ABE P2: FXS
9780521740494c12.xml CUAU033-EVANS October 22, 2008 22:46
R
e
v
i
e
w
Chapter 12 Trigonometric ratios and applications 359
The length of the minor arc AB (red line) is given by the formula
l = r
The area of sector AOB (shaded) is given by the formula
Area =
1
2
r
2

Chord length (red line) is given by


l = 2r sin

2
The area of a segment (shaded) is given by
Area =
1
2
r
2
( sin )
O
c
l
A
B
r
Angle between planes
Consider any point P on the common line of two
planes
1
and
2
. If PA and PB are drawn at right
angles to the common line so that PA is in
1
and
PB is in
2
then angle APB is the angle between

1
and
2
.
O
c
A
B
r
Note: If one of the planes,
2
say, is
horizontal, then PA is called a line of greatest slope
in the plane
1
.
P

B
A

2
P
A
lines of
greatest slope
angle of
greatest slope

2
Multiple-choice questions
1 In a triangle XYZ, x = 21 cm, y = 18 cm and YXZ = 62

. The magnitude of XYZ,


correct to one decimal place, is
A 0.4

B 0.8

C 1.0

D 49.2

E 53.1

2 In a triangle ABC, a = 30, b = 21 and cos C =


51
53
. The value of c, to the nearest whole
number, is
A 9 B 10 C 11 D 81 E 129
3 In a triangle ABC, a = 5.2 cm, b = 6.8 cm and c = 7.3 cm. The magnitude of ACB,
correct to the nearest degree, is
A 43

B 63

C 74

D 82

E 98

4 The area of the triangle ABC, where b = 5 cm, c = 3 cm, A = 30

and B = 70

, is
A 2.75 cm
2
B 3.75 cm
2
C 6.50 cm
2
D 7.50 cm
2
E 8 cm
2
P1: FXS/ABE P2: FXS
9780521740494c12.xml CUAU033-EVANS October 22, 2008 22:46
R
e
v
i
e
w
360 Essential Advanced General Mathematics
5 The length of the radius of the circle shown, correct to two decimal places, is
A 5.52 cm
B 8.36 cm
C 9.01 cm
D 12.18 cm
E 18.13 cm
10 cm
130
6 A chord of length 5 cm is drawn in a circle of radius 6 cm. The area of the smaller region
inside the circle cut off by the chord, correct to one decimal place, is
A 1.8 cm
2
B 2.3 cm
2
C 3.9 cm
2
D 13.6 cm
2
E 15.5 cm
2
7 From a point on a cliff 500 m above sea level, the angle of depression to a boat is 20

. The
distance from the foot of the cliff to the boat, to the nearest metre, is
A 182 m B 193 m C 210 m D 1374 m E 1834 m
8 A tower 80 m high is 1.3 km away from a point on the ground. The angle of elevation to the
top of the tower from this point, correct to the nearest degree, is
A 1

B 4

C 53

D 86

E 89

9 A man walks 5 km due east followed by 7 km due south. The bearing he must take to return
to the start is
A 036

B 306

C 324

D 332

E 348

10 A boat sails at a bearing of 215

from A to B. The bearing it must take from B to return


to A is
A 035

B 055

C 090

D 215

E 250

Short-answer questions (technology-free)


1 a Find x.
b Find y.
A C
B
10 cm
x cm
30
y
6 cm
2 Find
a AH, where AH is the altitude
b CM, where CM is the median.
B
A
C
40 cm
40 cm
30
3 From a port P, a ship Q is 20 km away on a bearing of 112

, and a ship R is 12 km away on


a bearing of 052

. Find the distance between the two ships.


4 In a quadrilateral ABCD, AB = 5 cm, BC = 5 cm, CD = 7 cm, B = 120

and C = 90

.
Find
a the length of the diagonal AC b the area of triangle ABC
c the area of triangle ADC d the area of the quadrilateral.
P1: FXS/ABE P2: FXS
9780521740494c12.xml CUAU033-EVANS October 22, 2008 22:46
R
e
v
i
e
w
Chapter 12 Trigonometric ratios and applications 361
5 If sin x = sin 37

and x is obtuse, nd x.
6 A point T is 10 km due north of a point S, and a point R, which is east of a straight line
joining T and S, is 8 km from T and 7 km from S. Calculate the cosine of the bearing of R
from S.
7 In ABC, AB = 5 cm, magnitude of BAC = 60

and AC = 6 cm. Calculate the sine of


ABC.
8 The area of a sector of a circle with radius 6 cm is 33 cm
2
. Calculate the angle of the sector.
N N
T
A B 300 m
60 45
9 The diagram shows two survey points, A and
B, which are on an eastwest line on level
ground. From point A, the bearing of the foot
of a tower is 060

, while from B the bearing


of the tower is 045

. Find
a i the magnitude of TAB
ii the magnitude of ATB
b Given that sin 15

2
4
, nd
AT and BT.
10 A boat sails 11 km from a harbour on a bearing of 220

. It then sails 15 km on a bearing of


340

. How far is the boat from the harbour?


11 A helicopter leaves a heliport A and ies 2.4 km on a bearing of 150

to a check point B. It
then ies due east to its base C.
a If the bearing of C from A is 120

, nd the distances AC and BC.


b The helicopter ies at a constant speed throughout and takes ve minutes to y from A
to C. Find its speed.
A
C
B
O
24
13
13
12 The diagram shows a circle of radius length 13 cm
and a chord AB of length 24 cm. Calculate
a the length of arc ACB
b the area of the shaded region.
13 A sector of a circle has an arc length of 30 cm. If the radius of the circle is 12 cm, nd the
area of the sector.
14 A chord PQ of a circle, radius 5 cm, subtends an angle of two radians at the centre of the
circle. Taking to be 3.14, calculate, correct to one decimal place, the length of the major
arc PQ.
15 From a cliff top 11 m above sea level, two boats are observed. One has an angle of
depression of 45

and is due east, the other an angle of depression of 30

on a bearing of
120

. Calculate the distance between the boats.


P1: FXS/ABE P2: FXS
9780521740494c12.xml CUAU033-EVANS October 22, 2008 22:46
R
e
v
i
e
w
362 Essential Advanced General Mathematics
Extended-response questions
1 AB is a tower 60 m high on top of a hill. The magnitude
of ACO is 49

and the magnitude of BCO is 37

.
a Find the magnitude of angles ACB, CBO and CBA.
b Find the length of BC.
c Find the height of the hill, i.e. the length of OB.
O
C
B
A
2 The angle of a sector of a circle, centre O and radius length
12 cm has magnitude 2.5 radians. The sector is folded
so that OA and OA

are joined to form a cone. Calculate


a the base radius length of the cone
b the curved surface area of the cone
c the shortest distance between two points diametrically
opposed on the edge of the base.
O
2.5
c
A' A
3 A tower 110 m high stands on the top of a hill. From a
point A at the foot of the hill the angle of elevation of the
bottom of the tower is 7

, and that of the top is 10

.
a Find the magnitude of angles TAB, ABT and ATB.
b Use the sine rule to nd the length of AB.
c Find CB, the height of the hill.
T
110 m
10
7
C A
B
A
O
S
B
120 m
59
57
4 Point S is a distance of 120 m from the base of a building.
On the building is an aerial, AB. The angle of elevation
from S to A is 57

. The angle of elevation from S to B is 59

. Find
a the distance OA
b the distance OB
c the distance AB.
5 From the top of a communications tower, the angles of
depression of two points A and B on a horizontal line through
the foot of the tower are 30

and 40

. The distance between


the points is 100 m. Find
a the distance AT
b the distance BT
c the height of the tower.
T
A
B
100 m
base of tower
top of tower
40
30
6 Angles VBC, VBA and ABC are right angles. Find
a the distance VA
b the distance VC
c the distance AC
d the magnitude of angle VCA.
8 cm
8 cm
6 cm
A
B
C
V
P1: FXS/ABE P2: FXS
9780521740494c13.xml CUAU033-EVANS October 16, 2008 19:24
C H A P T E R
Revision
13
Revision of chapters
812
13.1 Multiple-choice questions
1 A ladder 2.6 m long rests with one end on horizontal ground
while the other end rests against a vertical wall at a point which
is 2.1 m from the ground. The angle between the ladder and the
wall, to the nearest degree, is
A 36

B 39

C 51

D 54

E 63

2.6 m
2.1 m
2 The graph shown has amplitude
A 2 B 3 C 4
D 6 E 2
x
y
2
4
0
2
3 Which one of the following equations gives the
correct value for c?
A c =
58 cos 38

cos 130

B c =
58 sin 38

sin 130

C c = 58 sin 38

D c =
58 cos 130

cos 38

E c =
58 sin 130

sin 38

58 cm
12 38
130
B
A
C
4 A map is drawn so that a wall 17.1 m long is represented by a line 45 mm long. The scale
is
A 1 : 3.8 B 1 : 38 C 1 : 380 D 1 : 3800 E 1 : 38000
5 The point (5, 2) is reected in the line y = x. The coordinates of its image are
A (5, 2) B (5, 2) C (2, 5) D (2, 5) E (5, 2)
6 If sin A =
5
13
, sin B =
8
17
where A and B are acute, then sin (A B) is given by
A
140
221
B
21
221
C
34 209
23 560
D
107
140
E
107
140
363
P1: FXS/ABE P2: FXS
9780521740494c13.xml CUAU033-EVANS October 16, 2008 19:24
R
e
v
i
s
i
o
n
364 Essential Advanced General Mathematics
7 In triangle ABC, c = 5, b = 9 and A = 43

. Which of the following statements are


correct?
i With the information we can nd the area of triangle ABC.
ii With the information given we can nd angle B.
iii With the information given we can nd side a.
A i and ii only B i and iii only C ii and iii only
D i, ii and iii E none of these
8 In the gure, AB = 15, CD = 5, BF = 6, GD = 6,
EG = 9. x is equal to
A 3 B 4 C 4.5
D 4.75 E 5
15
6
6
9
D
A
B
C
E
F
G 5
x
9 The point (2, 6) is reected in the line y = x. The coordinates of its image are
A (2, 6) B (2, 6) C (6, 2) D (6, 2) E (2, 6)
10 The graph shown is best described by
A y = sin (a) B y = 2 cos (a)
C y = sin (a) +1 D y = cos (2a)
E y = cos (a) +1
y
2
1
0

2
2 3
2
a
11 If sin A =
5
13
, sin B =
8
17
where A and B are acute, then tan (A + B) is given by
A
140
221
B
21
221
C
34 209
23 560
D
171
140
E
171
140
12 P is the point (5, 4). After translation by

2
3

and reection in the line y = 1, the


coordinates of the image of P are
A (7, 7) B (7, 9) C (5, 7) D (7, 11) E (7, 10)
13 A model car is 8 cm long and the real car is 3.2 m long. The scale factor is
A 1 : 8 B 1 : 32 C 1 : 24 D 1 : 400 E 1 : 40
14 If 2 sin

x

6

3 and 0 x 2, then x is equal to


A

3
or
2
3
B

6
or
5
6
C

6
or

2
D

2
or
5
6
E

3
or
15 Which one of the following expressions will give the area of triangle ABC?
A
1
2
6 7 sin 48

B
1
2
6 7 cos 48

C
1
2
6 7 sin 52

D
1
2
6 7 cos 52

E
1
2
6 7 tan 48

A
B
C
6 cm
7 cm
48 52
P1: FXS/ABE P2: FXS
9780521740494c13.xml CUAU033-EVANS October 16, 2008 19:24
R
e
v
i
s
i
o
n
Chapter 13 Revision of chapters 812 365
16 Given that cos = c and that is acute, cot can be expressed in terms of c as
A c

1 c
2
B

1 c
2
C
1

1 c
2
D
c

1 c
2
E 2c

1 c
2
17 A trigonometric graph has the following characteristics:
r
period of 120

r
amplitude is 3
r
range is [4, 2]
r
y = 1 when a = 0
This graph would be described by the equation
A y = 3 sin (a)

+1 B y = 3 cos (3a)

1 C y = 3 sin (3a)

1
D y = 3 sin (3a)

+1 E y = cos (3a)

2
18 The point (a, b) is reected in the line with equation x = m. The image point has
coordinates
A (2m a, b) B (a, 2m b) C (a m, b)
D (a, b m) E (2m +a, b)
19 A child on a swing travels through an arc of length 3 m. If the ropes of the swing are 4 m
in length, the angle which the arc makes at the top of the swing (where the swing is
attached to the support) is best approximated by
A 135

B 75

C 12

D 75
c
E 42

58

20 Compared with the graph of y = sin , the graph of y = sin

1
2

has
A the same amplitude but double the period
B the same amplitude but half the period
C double the amplitude but the same period
D half the amplitude but the same period
E the same amplitude but shifted
1
2
a unit to the left.
21 The image of the line {(x, y) : x + y = 4} after a dilation of factor
1
2
from the y axis
followed by a reection in x = 4 is
A {(x, y) : y = 2x} B {(x, y) : y +2 = 0} C {(x, y) : y +2x 16 = 0}
D {(x, y) : x + y = 0} E {(x, y) : y = 2x 12}
22 If A + B =

2
, the value of cos A cos B sin A sin B is
A 2 B 1 C 1 D 0 E 2
23 Given that sin A =

5
3
and that A is obtuse, the value of sin 2A is
A
16

5
243
B
1
9
C
8

5
27
D
5
9
E
4

5
9
24 A ladder rests against a wall, touching the wall at a height of 5.6 m. The bottom of the
ladder is 2 m from the wall. The distance (to the nearest centimetre) that a person, of
height 1.6 m, must be from the wall to just t under the ladder is
A 1.43 m B 0.57 m C 1.75 m D 0.25 m E 1.2 m
P1: FXS/ABE P2: FXS
9780521740494c13.xml CUAU033-EVANS October 16, 2008 19:24
R
e
v
i
s
i
o
n
366 Essential Advanced General Mathematics
25 A possible equation of the graph shown is
A y = sin 2

x

12

B y = cos 2

x

12

C y = sin 2

x +

12

D y = cos 2

x +

12

E y = sin 2

x

12

x
y
0
1

12
3
4
26 Let ABC and DEF be similar triangles such that AB = 4 cm and DE = 10 cm. If the
area of ABC is 24 cm
2
, then the area of DEF, in cm
2
, is
A 60 B 240 C 150 D 96 E none of these
27 Which of the following statements is true for f (x) = 2 tan (3x)

?
i The period is 60. ii The amplitude is 2. iii The period is 120.
iv The graph is a reection of the graph of h(x) = 2 tan (3x)

in the x axis.
v The graph is a reection of g(x) = tan (x)

in the y axis.
A i and iv only B i, ii and iv only C i, iv and v only
D ii and iv only E iii and iv only
28 The image of {(x, y) : y = x
2
} under a translation determined by the vector

3
2

followed
by a reection in the x axis is
A {(x, y) : y = (x 3)
2
+2} B {(x, y) : (x 3)
2
= y +2}
C {(x, y) : y = (x +3)
2
+2} D {(x, y) : y +2 = (x 3)
2
} E none of these
29 The area, in cm
2
correct to two decimal places, of a sector with included angle of 60

in a
circle of diameter 10 cm is
A 104.72 cm
2
B 52.36 cm
2
C 13.09 cm
2
D 26.16 cm
2
E 750 cm
30 KLMN is a parallelogram and OQ is parallel to KL.
If O divides KN in the ratio of 1 : 2,
the ratio
areaKOP
areaKLMN
is equal to
A
1
4
B
1
9
C
1
12
D
1
18
E
1
20
K
N
L
M
P
O
Q
31 VABCD is a right, square pyramid with base length 80 mm and
perpendicular height 100 mm. The angle between a sloping
face and the base ABCD, to the nearest degree, is
A 22

B 29

C 51

D 61

E 68

A
B
C
D
V

E

O
32 Given that cos = c and that is acute, sin 2 can be expressed in terms of c as
A c

1 c
2
B

1 c
2
C
1

1 c
2
D
c

1 c
2
E 2c

1 c
2
P1: FXS/ABE P2: FXS
9780521740494c13.xml CUAU033-EVANS October 16, 2008 19:24
R
e
v
i
s
i
o
n
Chapter 13 Revision of chapters 812 367
33 The image of {(x, y) : y = 2
x
} after a dilation of factor 2 from the x axis followed by a
dilation of factor
1
3
from the y axis is
A y =
1
3
2
3x
B y = 3 2
x
2
C y = 2 2
3x
D y = 2 2
x
3
E none of these
34 The angles between 0

and 360

which satisfy the equation 4 cos x 3 sin x = 1, given


correct to two decimal places, are
A 53.13

and 126.87

B 48.41

and 205.33

C 41.59

and 244.67

D 131.59

and 334.67

E 154.67

and 311.59

35 In the gure, the volume of the shaded solid B is 49 cm


3
.
The volume of A is
A 19.5 cm
3
B 17.3 cm
3
C 13.5 cm
3
D 12.5 cm
3
E 10.5 cm
3
A
B
3 cm
2 cm
36 The area of the shaded region in the diagram, in cm
2
(to the nearest cm
2
), is
A 951 B 992 C 1944
D 2895 E 110 424
110 45 cm
37 The expression 8 sin cos
3
8 sin
3
cos is equal to
A 8 sin cos B sin 8 C 2 sin 4 D 4 cos 2 E 2 sin 2 cos 2
38 A possible equation for the graph shown is
A y = tan

1
2
x

4

+3
B y = tan

1
2
x +

4

3
C y = 3 tan

1
2
x

4

D y = 3 tan

1
2
x +

4

E y = tan 3x
x
y
3
3

2
2
3
2
39 If the ratio volume of the hemisphere : volume of the right circular cone equals 27 : 4
where r is the radius of the base of the cone and R is the radius of the hemisphere,
then R : r is equal to
R
r
r
A 1 : 2 B 2 : 3 C 3 :

2 D 27 : 8 E 3 : 2
P1: FXS/ABE P2: FXS
9780521740494c13.xml CUAU033-EVANS October 16, 2008 19:24
R
e
v
i
s
i
o
n
368 Essential Advanced General Mathematics
40 Let T be the translation determined by the vector

2
3

and S the transformation, reection


in the line with equation x = 2. The rule for the composition TS is given by
A TS(x, y) = (2 x, y +3) B TS(x, y) = (x, y +3)
C TS(x, y) = (x +2, y +3) D TS(x, y) = (6 x, y +3) E none of these
41 The square shown is subject to successive transformations.
The rst transformation has matrix

1 0
0 1

and the second


transformation has matrix

0 1
2 1

.
x
y
1
1
0
(1, 1)
Which one of the following shows the image of the square after these two
transformations?
A
x
y
1 3
1
2
0
B
1 1
3
2
1
2
0
x
y
C
2 1 1
2
0
1
3
x
y
D
x
y
1
1
2
1
1
0
E
x
y
2
1
3
1
1
0
13.2 Extended-response questions
1 a Find the rule of the transformation which
maps triangle ABC to triangle A

.
b On graph paper, draw triangle ABC and its
image under reection in the x axis. The
coordinates of A, B and C are (4, 1), (2, 1)
and (2, 5) respectively.
c On the same set of axes draw the image of
ABC under a dilation of factor 2 from
the y axis.
d Find the image of the parabola y = x
2
under a dilation of factor 2 from the x axis
followed by a translation dened by the vector

3
2

.
y
2 4
0
1
8
4
5
4 2
A B
C
C'
A' B'
x
(contd.)
P1: FXS/ABE P2: FXS
9780521740494c13.xml CUAU033-EVANS October 16, 2008 19:24
R
e
v
i
s
i
o
n
Chapter 13 Revision of chapters 812 369
e Find the rule for the transformation which maps the graph of
y = x
2
to y = 2(x 3)
2
+4
f If f (x) = x
3
2x use a graphics calculator to help sketch the graph of
y = 3 f (x 2) +4
2 In ABC, A = 30

, a = 60 (i.e., for the diagram


BC = BC

= 60) and c = 80.


a Find the magnitudes of angles
i ABC and BCA ii ABC

and BC

A
b Find the length of line segment
i AC ii AC

iii CC

A
C
C'
B
80
30
60 60
c i Show that the magnitude of CBC

is 96.38

(correct to two decimal places).


Using this value,
ii nd the area of triangle BCC

iii the area of the shaded sector


iv the area of the shaded segment.
3 a Find the image of the point (1, 1) under a dilation D, of factor 4 from the y axis.
b i Describe the image of the square with vertices A(0, 0), B(0, 1), C(1, 1), E(1, 0)
under the dilation D.
ii Find the area of the square ABCE.
iii Find the area of the region dened by the image of ABCE.
iv If the dilation had been of factor k, what would the area of this region be?
c State the rule for the dilation.
d i Find the equation of the image of the curve with equation y = x
2
under the
dilation D.
ii Find the equation of the image of the curve with equation y = x
2
under the
dilation D followed by the translation dened by the vector

2
1

.
iii Sketch the graph of y = x
2
and of its image dened in ii on the one set of axes.
State the coordinates of the vertex and of the axes intercepts.
e State the rule for the transformation which maps the curve with equation
y = 5(x +2)
2
3 to the curve with equation y = x
2
.
4 A transformation is represented by the matrix M=

3
5
4
5

4
5
3
5

a Describe the transformation.


b Let C be the circle which passes through the origin and which has as its centre the
point (0, 1).
i Find the equation of C.
ii Find the equation of C

, the image of C under the transformation determined


by M.
c Find the coordinates of the points of intersection of C and C

.
P1: FXS/ABE P2: FXS
9780521740494c13.xml CUAU033-EVANS October 16, 2008 19:24
R
e
v
i
s
i
o
n
370 Essential Advanced General Mathematics
5 A transformation is dened by the matrix M=

4 1
2 3

.
a Find the image of (2, 5) under this transformation. b Find the inverse of M.
c Given that the point (11, 13) is the image of the point (a, b), nd the values of
a and b.
d Find in terms of a, the image of the point (a, a).
e If M

a
b

a
b

, nd the possible values of and the relationship between


a and b in each of these cases.
6 Let R
4
be the matrix of the transformation, rotation of

4
in an anticlockwise direction.
a Give the 2 2 matrix associated with this transformation.
b Find the inverse of this matrix.
c If the image of (a, b) is (1, 1), nd the value of a and b.
d If the image of (c, d) is (1, 2), nd the value of c and d.
e i If (x, y) (x

, y

) under this transformation, use the result of b to nd x and y in


terms of x

and y

.
ii Find the image of y = x
2
under this transformation.
7 A particle oscillates along a straight line. Its displacement x (m), at time t (s), from a point
O is given by x = 5 +3 sin

6
t

.
a Find its displacement at time
i t = 0 ii t = 3
b Sketch the graph of x against t for t [0, 24], labelling clearly all turning points.
c State
i the maximum distance of the particle from O
ii the minimum distance of the particle from O.
d At what times (t [0, 24]) is the particle
i 5 m from O ii 6 m from O, correct to two decimal places?
8 A logo for a Victorian team is as shown here. O is the
centre of the circle and A, B and C are points on the
circle. OC = OA = OB = 10 cm.
a i Convert 30

to radians.
ii Find the length of the minor arc AB.
b The magnitude of AOC is 167

and the
magnitude of BOC is 163

.
Find the length of chord BC, correct to two
decimal places.
167 163
30
A
O
C
B
c Find, correct to two decimal places,
i the area of triangle BOC ii the area of triangle AOC
iii the shaded area of the logo.
P1: FXS/ABE P2: FXS
9780521740494c13.xml CUAU033-EVANS October 16, 2008 19:24
R
e
v
i
s
i
o
n
Chapter 13 Revision of chapters 812 371
9 Triangle LMN is an isosceles, right-angled triangle.
P, Q and R are midpoints of LM, MN and LN respectively.
a Prove PRQ LMN. b State the scale factor.
c Find the area of triangle PQR in terms of a.
X, Y and Z are the midpoints of PR, PQ and RQ
respectively.
XYZ is similar to LMN.
a units
a units
R
L
X
P
Y
Z
N
Q M
d State the scale factor.
e Find the area of triangle XYZ in terms of a.
f Let A
1
be the area of triangle LMN.
Let A
2
be the area of triangle PQR.
Let A
3
be the area of triangle XYZ.
The process of forming triangles by joining midpoints of the previous triangle is
continued to form a sequence of triangles,
1
,
2
,
3
, . . . ,
n
, . . . and associated
areas A
1
, A
2
, A
3
, . . . , A
n
, . . .
i Find A
n
in terms of a and n.
ii Find in terms of a, the sum to innity of the series A
1
+ A
2
+ + A
n
+
10 It is known that y varies partly as x and partly as
1
x
2
; i.e. there exist constants k
1
and k
2
such that y = k
1
x +
k
2
x
2
.
a When x = 1, y = 1 and when x = 1, y = 5.
Find the values of k
1
and k
2
.
b The graph of y against x is as shown.
i Sketch the graph of the image of
y = k
1
x +
k
2
x
2
under the transformation
determined by reection in the x axis
followed by a translation determined
by the vector

3
0

.
(The answers to parts ii, iii and iv
below may help you answer this.)
ii Find the value of c and hence the
x axis intercept of the image.
iii The image of the point with coordinates
(e, f ) is (e

, f

). Find e

and f

in terms
of e and f.
x
y
y = k
1
x +
x
2
k
2
y = k
1
x
(c, 0)
(e, f )
0
iv Find the equation of the image of the curve with equation y = k
1
x +
k
2
x
2
under
this transformation.
11 Let M be the transformation reection in the line y = x

.
a i Find the coordinates of the image of the point A (1, 3) under this transformation.
P1: FXS/ABE P2: FXS
9780521740494c13.xml CUAU033-EVANS October 16, 2008 19:24
R
e
v
i
s
i
o
n
372 Essential Advanced General Mathematics
ii Find the coordinates of the triangle which is the image of the triangle with vertices
A (1, 3), B (1, 5), C (3, 3).
iii Illustrate triangle ABC and its image on a set of axes scaled from 5 to 5 on both
axes.
b i Show that the equation of the image of the curve with equation y = x
2
2 under
the transformation M is x = y
2
2.
ii Find the coordinates of the points of intersection of the curve y = x
2
2 with the
line y = x.
iii Show that the x coordinates of the points of intersection of y = x
2
2 and its
image may be determined by the equation x
4
4x
2
x +2 = 0.
iv Two solutions of the equation x
4
4x
2
x +2 = 0 are x =
1
2
(1 +

5) and
x =
1
2
(1

5).
Use this result and the result of b ii to nd the coordinates of the points of
intersection of y = x
2
2 and its image under M.
12 In the gure, AE = BE = BD = 1 unit.
BCD is a right angle.
a Show that the magnitude of BDE is 2.
b Use the cosine rule in triangle BDE to show that
DE = 2 cos 2.
A
B
D
E
C
1 1
1
3
c Show that
i DC = sin 3
ii AD =
sin 3
sin
d Use the results of b and c to show sin 3 = 3 sin 4 sin
3

13 a Adam notices a distinctive tree while orienteering on a at horizontal plane. He


discovers that the tree is 200 m from where he is standing on a bearing of 050

. Two
other people, Brian and Colin, who are both standing due east of Adam, claim the tree
is 150 m away from them. Given that their claim is true and that Brian and Colin are
not standing in the same place, how far apart are they? Give your answer to the nearest
metre.
b From the top of a vertical tower of height 10 m,
standing in the corner of a rectangular courtyard,
the angles of depression to the nearest corners
(B and D) are 32

and 19

respectively.
Find
i AB, correct to two decimal places
T
A
B
C
D
ii AD, correct to two decimal places
iii the angle of depression of corner C diagonally opposite the tower from T, correct
to the nearest degree.
c Two circles, each of radius length 10 cm, have their centres 16 cm apart. Calculate the
area common to both circles, correct to one decimal place.
P1: FXS/ABE P2: FXS
9780521740494c13.xml CUAU033-EVANS October 16, 2008 19:24
R
e
v
i
s
i
o
n
Chapter 13 Revision of chapters 812 373
14 A satellite travelling in a circular orbit 1600 km
above the Earth is due to pass directly over a
tracking station at noon. Assume that the
satellite takes two hours to make an orbit and
the radius of the Earth is 6400 km.
a If the tracking station antenna is aimed
at 30

above the horizon, at what time will


the satellite pass through the beam of the antenna?
satellite
direction of travel
6400 km
8000 km
30
S
O
T
b Find the distance between the satellite and the tracking station at 12.06 p.m.
c At what angle above the horizon should the antenna be pointed so that its beam will
intercept the satellite at 12.06 p.m.?
15 An athlete in a gymnasium is training on an exercise bike.
At time t = 0, the position of the pedal is as shown.
The height of the pedal, h cm, from the oor at time
t seconds, is given by
h(t ) = a +b cos ((t +c))
where a and b are in centimetres.
a Find the values of a, b and c.
b i Find the times at which the height of the pedal
above the oor is 60 cm, 0 t 4.
direction of
movement
Pedal
Floor
35 cm
25 cm
60
ii Find the times at which the height of the pedal above the oor is 37.5 cm,
0 t 4.
c Sketch the graph of h against t for 0 t 4.
16 ABCD is a parallelogram whose diagonals
intersect at angle

at the point E.
Let AB = CD = x, AD = BC = y, BD = p,
AC = q.
a Apply the cosine rule to triangle DEC to
nd x in terms of p, q and .
A
B C
E
D

b Apply the cosine rule to triangle DEA to nd y in terms of p, q and .


c Use the results of a and b to show that 2(x
2
+ y
2
) = p
2
+q
2
d A parallelogram has sides 8 cm and 6 cm and one diagonal of 13 cm. Find the length
of the other diagonal.
17 The gure shows the circular cross section of a
uniform log of radius 40 cm oating in water. The
points A and B are on the surface of the water
and the highest point X is 8 cm above the surface.
a Show that the magnitude of AOB is
approximately 1.29 radians.
b Calculate
i the length of arc AXB
X
B A
O
40 cm
8 cm
P1: FXS/ABE P2: FXS
9780521740494c13.xml CUAU033-EVANS October 16, 2008 19:24
R
e
v
i
s
i
o
n
374 Essential Advanced General Mathematics
ii the area of the cross section below the surface
iii the percentage of the volume of the log below the surface.
18 In ABC, AB = 7 cm , BC = 9 cm and ABC = .
a Show that AC
2
= 130 126 cos .
D is the point on the opposite side of AC from B such that ABCD is a cyclic
quadrilateral in which CD = 6 cm and DA = 5 cm.
b Obtain another expression for AC
2
in terms of and prove that cos =
23
62
.
c Calculate
i the length of AC ii the area of quadrilateral ABCD.
P1: FXS/ABE P2: FXS
9780521740494c14.xml CUAU033-EVANS October 5, 2008 7:48
C H A P T E R
14
Circle theorems
Objectives
To establish the following results and use them to prove further properties and
solve problems:
r
The angle subtended at the circumference is half the angle at the centre
subtended by the same arc
r
Angles in the same segment of a circle are equal
r
A tangent to a circle is perpendicular to the radius drawn from the point of contact
r
The two tangents drawn from an external point to a circle are the same length
r
The angle between a tangent and a chord drawn from the point of contact is
equal to any angle in the alternate segment
r
A quadrilateral is cyclic (that is, the four vertices lie on a circle) if and only if the
sum of each pair of opposite angles is two right angles
r
If AB and CD are two chords of a circle which cut at a point P (which may be
inside or outside a circle) then PA PB = PC PD
r
If P is a point outside a circle and T, A, B are points on the circle such that PT is
a tangent and PAB is a secant then PT
2
= PA PB
These theorems and related results can be investigated through a geometry package such as
Cabri Geometry.
It is assumed in this chapter that the student is familiar with basic properties of parallel lines
and triangles.
14.1 Angle properties of the circle
Theorem 1
The angle at the centre of a circle is twice the angle at
the circumference subtended by the same arc.
x
A
O
B
P
2x
375
P1: FXS/ABE P2: FXS
9780521740494c14.xml CUAU033-EVANS October 5, 2008 7:48
376 Essential Advanced General Mathematics
Proof
Join points P and O and extend the line through O as shown
in the diagram.
Note that AO = BO = PO = r the radius of the circle. Therefore
triangles PAO and PBO are isosceles.
Let APO = PAO = a

and BPO = PBO = b

Then angle AOX is 2a

(exterior angle of a triangle) and angle


BOX is 2b

(exterior angle of a triangle)


P
A
r
r r
O
X
B
a
a
b
b
AOB = 2a

+2b

= 2(a +b)

= 2APB
Note: In the proof presented above, the centre and point P are considered to be on the same side
of chord AB.
The proof is not dependent on this and the result always holds.
The converse of this result also holds:
i.e., if A and B are points on a circle with centre O and angle APB is equal to half angle
AOB, then P lies on the circle.
A segment of a circle is the part of the plane bounded by
an arc and its chord.
Arc AEB and chord AB dene a major segment which
is shaded.
Arc AFB and chord AB dene a minor segment which is not
shaded.
E
A
F
B
AEB is said to be an angle in segment AEB.
A
E
B
O
Theorem 2
Angles in the same segment of a circle are equal.
Proof
Let AXB = x

and AYB = y

Then by Theorem 1 AOB = 2x

= 2y

Therefore x = y
A
X
x
y
Y
B
O
Theorem 3
The angle subtended by a diameter at the circumference is equal
to a right angle (90

).
Proof
The angle subtended at the centre is 180

.
Theorem 1 gives the result.
A
E
B
O
P1: FXS/ABE P2: FXS
9780521740494c14.xml CUAU033-EVANS October 5, 2008 7:48
Chapter 14 Circle theorems 377
A quadrilateral which can be inscribed in a circle is called a cyclic quadrilateral.
Theorem 4
The opposite angles of a quadrilateral inscribed in a circle sum to two right angles (180

). (The
opposite angles of a cyclic quadrilateral are supplementary). The converse of this result also
holds.
Proof
O is the centre of the circle
By Theorem 1 y = 2b and x = 2d
Also x + y = 360
Therefore 2b +2d = 360
i.e. b +d = 180
The converse states: if a quadrilateral has opposite angles supplementary then the quadrilateral
is inscribable in a circle.
A
b
x
y
d
B
C
D
O
Example 1
Find the value of each of the pronumerals in the diagram. O is the
centre of the circle and AOB = 100

.
A
z
x
y
100
B
O
Solution
Theorem 1 gives that z = y = 50
The value of x can be found by observing either of the
following.
Reex angle AOB is 260

. Therefore x = 130 (Theorem 1)


or y + x = 180 (Theorem 4)
Therefore x = 180 50 = 130
Example 2
A, B, C, D are points on a circle. The diagonals of quadrilateral ABCD meet at X. Prove that
triangles ADX and BCX are similar.
Solution
DAC and DBC are in the same segment.
Therefore m = n
BDA and BCA are in the same segment.
Therefore p = q
Also AXD = BXC (vertically opposite).
Therefore triangles ADX and BCX are equiangular
and thus similar.
A
X
D C
m
q
p
n
B
P1: FXS/ABE P2: FXS
9780521740494c14.xml CUAU033-EVANS October 5, 2008 7:48
378 Essential Advanced General Mathematics
Example 3
An isosceles triangle is inscribed in a circle. Find the angles in
the three minor segments of the circle cut off by the sides of
this triangle.
A
C B
O
32
74 74
Solution
First, to determine the magnitude of AXC cyclic
quadrilateral AXCB is formed. Thus AXC and
ABC are supplementary.
Therefore AXC = 106

. All angles in the minor


segment formed by AC will have this magnitude.
A
X
C B
O
74
In a similar fashion it can be shown that the angles in the minor segment formed by
AB all have magnitude 106

and for the minor segment formed by BC the angles all


have magnitude 148

.
Exercise 14A
1 Find the values of the pronumerals for each of the following, where O denotes the centre of
Example 1
the given circle.
a
50
x
O
y
b
108
x
O
y
z
c
35
O
y
z
d
O
y
x
e
O
y
3x
z
x
f
O
y x
25
125
2 Find the value of the pronumerals for each of the following.
a
x
y
59
112
b
x y
93
68
c
x
y
70
130
P1: FXS/ABE P2: FXS
9780521740494c14.xml CUAU033-EVANS October 5, 2008 7:48
Chapter 14 Circle theorems 379
3 An isosceles triangle ABC is inscribed in a circle. What are the
angles in the three minor segments cut off by the sides of the
triangle?
40
A
C B
4 ABCDE is a pentagon inscribed in a circle. If AE = DE and BDC = 20

,
CAD = 28

and ABD = 70

, nd all of the interior angles of the pentagon.


5 If two opposite sides of a cyclic quadrilateral are equal, prove that the other two sides are
Example 2
parallel.
6 ABCD is a parallelogram. The circle through A, B and C cuts CD (produced if necessary) at
E. Prove that AE = AD.
7 ABCD is a cyclic quadrilateral and O is the centre of the circle through A, B, C and D. If
Example 3
AOC = 120

, nd the magnitude of ADC.


8 Prove that if a parallelogram is inscribed in a circle it must be a rectangle.
9 Prove that the bisectors of the four interior angles of a quadrilateral form a cyclic
quadrilateral.
14.2 Tangents
Line PC is called a secant and line segment AB a chord.
P
A
B
C
If the secant is rotated with P as the pivot point a
sequence of pairs of points on the circle is dened. As
PQ moves towards the edge of the circle the points of the
pairs become closer until they eventually coincide.
When PQ is in this nal position (i.e., where
the intersection points A and B collide)
it is called a tangent to the circle. PQ
touches the circle. The point at which the tangent
touches the circle is called the point of
contact. The length of a tangent from a point
P outside the tangent is the distance between
P and the point of contact.
P
Q
Q
Q
Q
Q
A
1
A
2
A
3
A
4
A
5
B
5
B
4
B
3
B
2
B
1
Theorem 5
A tangent to a circle is perpendicular to the radius drawn to the point of contact.
P1: FXS/ABE P2: FXS
9780521740494c14.xml CUAU033-EVANS October 5, 2008 7:48
380 Essential Advanced General Mathematics
O
P
T
S
Q
Proof
Let T be the point of contact of tangent PQ.
Let S be the point on PQ, not T, such that OSP is a right angle.
Triangle OST has a right angle at S.
Therefore OT > OS as OT is the hypotenuse of triangle OTS.
S is inside the circle as OT is a radius.
The line through T and S must cut the circle again. But PQ is a tangent. A contradiction.
Therefore T = S and angle OTP is a right angle.
Theorem 6
The two tangents drawn from an external point to a circle are of the same length.
Proof
Triangle XPO is congruent to triangle XQO as XO is
a common side.
XPO = XQO = 90

OP = OQ (radii)
Therefore XP = XQ
X
P
O
Q
r
r
Alternate segment theorem
The shaded segment is called the alternate segment in
relation to STQ.
The unshaded segment is alternate to PTS
P
T
Q
S
Theorem 7
The angle between a tangent and a chord drawn from the
point of contact is equal to any angle in the alternate segment.
Proof
Let STQ = x

, RTS = y

and TRS = z

where RT is
a diameter.
Then RST = 90

(Theorem 3, angle subtended by a


diameter)
Also RTQ = 90

(Theorem 5, tangent is perpendicular to radius)


Hence x + y = 90 and y + z = 90
Therefore x = z
But TXS is in the same segment as TRS and therefore TXS = x

O
P
z
y
x
X
R
Q
S
T
Example 4
Find the magnitude of the angles x and y in
the diagram.
P
T
S
Q x
y
30
P1: FXS/ABE P2: FXS
9780521740494c14.xml CUAU033-EVANS October 5, 2008 7:48
Chapter 14 Circle theorems 381
Solution
Triangle PTS is isosceles (Theorem 6, two tangents from the same point) and
therefore PTS = PST
Hence y = 75. The alternate segment theorem gives that x = y = 75
Example 5
Find the values of x and y.
PT is tangent to the circle centre O
60
x
y
T
P
O Solution
x = 30 as the angle at the circumference is half
the angle subtended at the centre and y = 60 as
OTP is a right angle.
Example 6
The tangents to a circle at F and G meet at H. If a chord FK is
drawn parallel to HG, prove that triangle FGK is isosceles.
Solution
Let XGK = y

Then GFK = y

(alternate segment theorem)


and GKF = y

(alternate angles)
Therefore triangle FGK is isosceles with FG = KG
H
F
Y
G
K
X
y
Exercise 14B
1 Find the value of the pronumerals for each of the following. T is the point of contact of the
Example 4
tangent and O the centre of the circle.
a
81
x
y
73
T
b
33
x
q
T
O
c
BC = BT
74
z
y
x
T
C
B
d
80
40
w
y
x
z
e
w
Q
S
T
P
z
y
x
54
S and T are points of
contact of tangents
from P.
TP is parallel to QS
P1: FXS/ABE P2: FXS
9780521740494c14.xml CUAU033-EVANS October 5, 2008 7:48
382 Essential Advanced General Mathematics
2 A triangle ABC is inscribed in a circle, and the tangent at C
Example 5
to the circle is parallel to the bisector of angle ABC.
A
B
D
C
Y
X
40
a Find the magnitude of BCX.
b Find the magnitude of CBD, where D is the point
of intersection of the bisector of angle ABC with AC.
c Find the magnitude of ABC.
3 AT is a tangent at A and TBC is a secant to the circle. Given
CTA = 30

, CAT = 110

, nd the magnitude of angles


ACB, ABC and BAT.
C
A
B
T
4 If AB and AC are two tangents to a circle and BAC = 116

, nd the magnitudes of the


angles in the two segments into which BC divides the circle.
5 From a point A outside a circle, a secant ABC is drawn cutting the circle at B and C, and a
Example 6
tangent AD touching it at D. A chord DE is drawn equal in length to chord DB. Prove that
triangles ABD and CDE are similar.
6 AB is a chord of a circle and CT, the tangent at C, is parallel to AB. Prove that CA = CB.
7 Through a point T, a tangent TA and a secant TPQ are drawn to a circle AQP. If the chord
AB is drawn parallel to PQ, prove that the triangles PAT and BAQ are similar.
8 PQ is a diameter of a circle and AB is a perpendicular chord cutting it at N. Prove that PN is
equal in length to the perpendicular from P on to the tangent at A.
14.3 Chords in circles
Theorem 8
If AB and CD are two chords which cut at a point P (which may be inside or outside the circle)
then PA PB = PC PD.
A
C
D
P
B
Proof
CASE 1 (The intersection point is inside the circle.)
Consider triangles APC and BPD.
APC = BPD (vertically opposite)
CDB = CAB (angles in the same segment)
ACD = DBA (angles in the same segment)
Therefore triangle CAP is similar to triangle BDP.
Therefore
AP
PD
=
CP
PB
and AP PB = CP PD, which can be written PA PB = PC PD
P1: FXS/ABE P2: FXS
9780521740494c14.xml CUAU033-EVANS October 5, 2008 7:48
Chapter 14 Circle theorems 383
CASE 2 (The intersection point is outside the circle.)
Show triangle APD is similar to triangle CPB
Hence
AP
CP
=
PD
PB
i.e. AP PB = PD.CP
which can be written PA PB = PC PD
A
B
D
C
P
Theorem 9
If P is a point outside a circle and T, A, B are points on the circle such that PT is a tangent and
PAB is a secant then PT
2
= PA PB
Proof
PTA = TBA (alternate segment theorem)
PTB = TAP (angle sum of a triangle)
Therefore triangle PTB is similar to triangle PAT

PT
PA
=
PB
PT
which implies PT
2
= PA PB
T
P
A
B
Example 7
The arch of a bridge is to be in the form of an arc of a circle. The span of the bridge is to be
25 m and the height in the middle 2 m. Find the radius of the circle.
Solution
By Theorem 8
RP PQ = MP PN
Therefore
2PQ = 12.5
2
PQ =
12.5
2
2
Also PQ = 2r 2 where r is the radius of the circle.
Hence 2r 2 =
12.5
2
2
and r =
1
2

12.5
2
2
+2

=
641
16
m
R
N
Q
O
2 m
M
12.5 m
P
12.5 m
P1: FXS/ABE P2: FXS
9780521740494c14.xml CUAU033-EVANS October 5, 2008 7:48
384 Essential Advanced General Mathematics
Example 8
If r is the radius of a circle, with center O, and if A is any point inside the circle, show that the
product CA AD = r
2
OA
2
, where CD is a chord through A.
Solution
Let PQ be a diameter through A
Theorem 8 gives that
CA AD = QA AP
Also QA = r OA and PA = r +OA
CA AD = r
2
OA
2
Q
C
A
O
P
D
Exercise 14C
1 Two chords AB and CD intersect at a point P within a circle. Given that
Example 7
a AP = 5 cm, PB = 4 cm, CP = 2 cm, nd PD
b AP = 4 cm, CP = 3 cm, PD = 8 cm, nd PB.
2 If AB is a chord and P is a point on AB such that AP = 8 cm, PB = 5 cm and P is 3 cm
from the centre of the circle, nd the radius.
3 If AB is a chord of a circle with centre O and P is a point on AB such that BP = 4PA,
OP = 5 cm and the radius of the circle is 7 cm, nd AB.
4 Two circles intersect at A and B and, from any point P on AB produced tangents PQ and PR
Example 8
are drawn to the circles. Prove that PQ = PR.
5 PQ is a variable chord of the smaller of two xed concentric circles.
PQ produced meets the circumference of the larger circle at R. Prove that the product
RP.RQ is constant for all positions and lengths of PQ.
6 ABC is an isosceles triangle with AB = AC. A line through A meets BC at D and the
circumcircle of the triangle at E. Prove that AB
2
= AD AE.
P1: FXS/ABE P2: FXS
9780521740494c14.xml CUAU033-EVANS October 5, 2008 7:48
R
e
v
i
e
w
Chapter 14 Circle theorems 385
Chapter summary
The angle subtended at the circumference is half the angle at
the centre subtended by the same arc.

2
O
B A
Angles in the same segment of a circle are equal.

B
O
A
A tangent to a circle is perpendicular to the radius
drawn from the point of contact.
O
T
P
The two tangents drawn from an external point are
the same length i.e. PT = PT

.
T
P
T'
O
The angle between a tangent and a chord drawn from
the point of contact is equal to any angle in the alternate
segment.
A
B

A quadrilateral is cyclic if and only if the sum of each pair of opposite angles is two right
angles.
If AB and CD are two chords of a circle which cut at a point P then PA PB = PC PD.
C
B
D
P
A
A
B
C
D
P
P1: FXS/ABE P2: FXS
9780521740494c14.xml CUAU033-EVANS October 5, 2008 7:48
R
e
v
i
e
w
386 Essential Advanced General Mathematics
Multiple-choice questions
1 In the diagram A, B, C and D are points on the circumference
of a circle. ABC = 115

, BAD = 70

and AB = AD. The


magnitude of ACD is
A 45

B 55

C 40

D 70

E 50

B
C
70
115
D
A
2 In the diagram, PA and PB are tangents to the circle centre O. Given that Q is a point on the
minor arc AB and that AOB = 150

the magnitudes of APB and AQB are


A APB = 30

and AQB = 105

B APB = 40

and AQB = 110

C APB = 25

and AQB = 105

D APB = 30

and AQB = 110

E APB = 25

and AQB = 100

150
A
Q
P
B
O
3 A circle centre O, passes through A, B and C. AT is the
tangent to the circle at A. CBT is a straight line. Given
that ABO = 68

and OBC = 20

the magnitude of
ATB is
A 60

B 64

C 65

D 70

E 66

O
C
20
B
T
68
A
4 In the diagram the points A, B and C lie on a circle centre O.
BOC = 120

and ACO = 42

. The magnitude of ABO is


A 18

B 20

C 22

D 24

E 26

A
O
42
C
120
B
5 ABCD is a cyclic quadrilateral with AD parallel to BC.
DCB = 65

. The magnitude of CBE is


A 100

B 110

C 115

D 120

E 122

D A
B
E
C
6 A chord AB of a circle subtends an angle of 50

at a point on the circumference of the circle.


The acute angle between the tangents at A and B has magnitude
A 80

B 65

C 75

D 85

E 82

P1: FXS/ABE P2: FXS


9780521740494c14.xml CUAU033-EVANS October 5, 2008 7:48
R
e
v
i
e
w
Chapter 14 Circle theorems 387
7 Chords AB and CD of circle intersect at P. If AP = 12 cm, PB = 6 cm and CP = 2 cm,
the length of PD in centimetres is
A 12 B 24 C 36 D 48 E 56
8 In the diagram AB is the diameter of a circle with centre O. PQ
is a chord perpendicular to AB. N is the point of intersection of
AB and PQ and ON = 5 cm. If the radius of the circle is 13 cm
the length of chord PB is, in centimetres,
A 12 B 4

13 C 2

13 D 14 E 8
A
13 cm
O
5 cm
N
B
P
Q
9 In the diagram A, B, C and D are points on the circumference
of a circle. ABD = 40

and angle AXB = 105

.
The magnitude of XDC is
A 35

B 40

C 45

D 50

E 55

B
C
105
40
A
X
D
10 A, B, C and D are points on a circle, centre O such that AC is a
diameter of the circle. If BAD = 75

and ACD = 25

The magnitude of BDC is


A 10

B 15

C 20

D 25

E 30

B
C
D
25
O
75
A
Short-answer questions (technology-free)
1 Find the value of the pronumerals in each of the following.
a
y
O
140
x
b
x
50
75
y
O
c
x
53
z
y
47
d
x
z
30
70
y
2 O is the centre of a circle and OP is any radius. A chord BA is drawn parallel to OP. OA and
BP intersect at C. Prove that
a CAB = 2CBA b PCA = 3PBA
P1: FXS/ABE P2: FXS
9780521740494c14.xml CUAU033-EVANS October 5, 2008 7:48
R
e
v
i
e
w
388 Essential Advanced General Mathematics
3 A chord AB of a circle, centre O, is produced to C. The straight line bisecting OAB meets
the circle at E. Prove that EB bisects OBC
4 Two circles intersect at A and B. The tangent at B to one circle meets the second again at D,
and a straight line through A meets the rst circle at P and the second at Q. Prove that BP is
parallel to DQ.
5 Find the values of the pronumerals for each of the following:
a
x
57
b
64
x
c
y
x
48
6 Two circles intersect at M and N. The tangent to the rst at M meets the second circle at P,
while the tangent to the second at N meets the rst at Q. Prove that MN
2
= NP QM.
7 If AB = 10 cm, BE = 5 cm and CE = 25 cm, nd DE.
A
B
D
C
E
Extended-response questions
1 The diagonals PR and QS of a cyclic quadrilateral PQRS intersect
at X. The tangent at P is parallel to QS. Prove that
a PQ = PS
b PR bisects QRS.
R
Q
X
S P
2 Two circles intersect at A and B. The tangents at C and D
intersect at T on AB produced. If CBD is a straight line
prove that
a TCAD is a cyclic quadrilateral
b TAC = TAD
c TC = TD.
A
T
D
B
C
P1: FXS/ABE P2: FXS
9780521740494c14.xml CUAU033-EVANS October 5, 2008 7:48
R
e
v
i
e
w
Chapter 14 Circle theorems 389
3 ABCD is a trapezium in which AB is parallel to DC and the diagonals meet at P.
The circle through D, P and C touches AD, BC at D and C respectively.
Prove that
a BAC = ADB
b the circle through A, P and D touches BA at A
c ABCD is a cyclic quadrilateral.
A
B
C
P
D
4 PQRS is a square of side length 4 cm inscribed in a circle with
centre O. M is the midpoint of the side PS. QM is produced to
meet RS produced at X.
a Prove that:
i XPR is isosceles
ii PX is a tangent to the circle at P.
P
Q
R
O
M
4 cm S
X
b Calculate the area of trapezium PQRX.
5 a An isosceles triangle ABC is inscribed in a circle. AB = AC and chord AD intersects BC
at E. Prove that
AB
2
AE
2
= BE CE
b Diameter AB of circle with centre O is extended to C and from C a line is drawn tangent
to the circle at P. PT is drawn perpendicular to AB at T. Prove that
CA CB TA TB = CT
2
P1: FXS/ABE P2: FXS
9780521740494c15.xml CUAU033-EVANS October 5, 2008 7:52
C H A P T E R
15
Vectors
Objectives
To understand the concept of vector
To apply basic operations to vectors
To understand the zero vector
To use the unit vectors i and j to represent vectors in two dimensions
To use the fact that, if a and b are parallel, then a = kb for a real value k. The
converse of this also holds.
To use the unit vectors i, j and k to represent vectors in three dimensions
15.1 Introduction to vectors
For experiments in science or engineering some of the things which are measured are
completely determined by their magnitude. For example, mass, length or time are determined
by a number and an appropriate unit of measurement.
e.g., length: 30 cm is the length of the page of a particular book
time: 10 s is the time for one athlete to run 100 m
More is required to describe velocity, displacement or force. The direction must be recorded
as well as the magnitude.
e.g., displacement: 30 km in a direction north
velocity: 60 km/h in a direction south east
Quantities that have direction as well as magnitude are represented by arrows that point in
the direction of the action and whose lengths give the magnitude of the quantity in terms of a
suitably chosen unit.
Arrows with the same length and direction are regarded as equivalent. These arrows are
directed line segments and the sets of equivalent segments are called vectors.
390
P1: FXS/ABE P2: FXS
9780521740494c15.xml CUAU033-EVANS October 5, 2008 7:52
Chapter 15 Vectors 391
The ve directed line segments shown all have the same
magnitude and direction.
y
A
B
D
C
O
P
x
F
E
G
H
A directed line segment from a point A to a point B is
denoted by

AB.
For simplicity of language this is also called vector

AB,
i.e., the set of equivalent segments can be named through
one member of that set.
Note:

AB =

CD =

OP =

EF =

GH
In Chapter 8, vectors were introduced in the context of
translations (in two dimensions). A column of numbers
was introduced to represent the translation and it was
called a vector.
This is consistent with the approach here as the column
of numbers corresponds to a set of equivalent directed
line segments.
The column

3
2

corresponds to the directed line segment


which goes 3 across and 2 up.
x
y
B
2 units
3 units
A
0
This notation will be used to represent a directed line segment in the rst section of this
chapter. The notation is widely used to represent vectors but not to a large extent in Victorian
schools and so the notation, although useful, will be abandoned in the latter sections of the
chapter.
Vectors are often denoted by a single bold face roman letter. The vector from A to B can be
denoted by

AB or by a single v. That is, v =

AB.
When a vector is handwritten the notation is v

.
Addition of vectors
Two vectors u and v can be added geometrically by drawing
a line segment representing u from A to B and then a line
segment from B to C representing v.
The sum u +v is the vector from A to C.
That is, u +v =

AC.
C
v
B
u
A
u + v
C
v
B
u
u + v
u
D
v
A
The same result is achieved if the order is reversed. This is
represented in the following diagram
i.e. u +v =

AC
and u +v = v +u
P1: FXS/ABE P2: FXS
9780521740494c15.xml CUAU033-EVANS October 5, 2008 7:52
392 Essential Advanced General Mathematics
The addition can also be achieved with the column
vector notation.
e.g., if u =

4
1

and v =

1
3

then u +v =

4
1

1
3

3
4

3
v
u
1
4
4
1
3
u + v
Scalar multiplication
Multiplication by a real number (scalar) changes the length of
the vector.
u
2u
u
1
2
e.g., 2u = u +u and
1
2
u +
1
2
u = u
2u is twice the length of u and
1
2
u is half the length of u.
The vector ku, k R
+
, with u = 0, has the same direction as u, but its length is
multiplied by a factor of k.
When a vector is multiplied by 2 the vectors direction is reversed
and its length doubled.
u
2u
When a vector is multiplied by 1 the vectors direction is reversed
and the length remains the same.
If u =

3
2

, u =

3
2

, 2u =

6
4

and 2u =

6
4

If u =

AB then u =

AB =

BA.
The directed line segment

AB goes from B to A.
Zero vector
The zero vector is denoted by 0 and represents a line segment of zero length. The zero vector
has no direction.
Subtraction of vectors
In order to subtract v from u, add v to u.
For example
u
v
v
u
u v
v
P1: FXS/ABE P2: FXS
9780521740494c15.xml CUAU033-EVANS October 5, 2008 7:52
Chapter 15 Vectors 393
Example 1
Draw the directed line segment dened by

3
2

.
Solution

3
2

is the vector 3 across to the right and 2 down


Note: Here the vector starts at (1, 1) and goes to
(4, 1). It can start at any point.
x
y
A
B
4 3 2
0
1
1
1
Example 2
The vector u is dened by the directed line segment from (2, 6) to (3, 1).
If u =

a
b

, nd a and b.
Solution
The vector u =

3
1
2
6

1
5

Hence a = 1 and b = 5
x
y
A
0
B
(3, 1)
(2, 6)
Example 3
If the vector u =

3
1

and the vector v =

2
2

, nd 2u +3v.
Solution
2u +3v = 2

3
1

+3

2
2

6
2

6
6

0
4

Polygons of vectors
For two vectors

AB and

BC,

AB +

BC =

AC
B
A
C
P1: FXS/ABE P2: FXS
9780521740494c15.xml CUAU033-EVANS October 5, 2008 7:52
394 Essential Advanced General Mathematics
For a polygon ABCDEF,

AB +

BC +

CD +

DE +

EF +

FA = 0
B
C
F E
D
A
Example 4
Illustrate the vector sum

AB +

BC +

CD, where A, B, C and D are points in the plane.
Solution

AB +

BC +

CD =

AD
B
C
A
D
Parallel vectors
The non-zero vectors u and v are said to be parallel if there exists k R\{0} such that u = kv.
If u =

2
3

and v =

6
9

then vector u is parallel to v as v = 3u.


Position vectors
A point O, the origin, can be used as a starting point for a vector to indicate the position of a
point in space relative to that point. In this chapter, O is the origin for a cartesian plane (three
dimensional work is considered briey).
For a point A the position vector is

OA.
Linear combinations of non-parallel vectors
If a and b are non-zero, non-parallel vectors, then
ma +nb = pa +qb
implies m = p and n = q
An argument is as follows:
ma pa = qb nb
(m p)a = (q n)b
a =
q n
m p
b or b =
m p
q n
a
but a and b are not parallel and not zero
q = n and m = p
P1: FXS/ABE P2: FXS
9780521740494c15.xml CUAU033-EVANS October 5, 2008 7:52
Chapter 15 Vectors 395
Example 5
Let A, B and C be the vertices of a triangle.
Let D be the midpoint of BC.
Let

AB = a and

BC = b.
Find the following in terms of a and b.
a

BD b

DC c

AC d

AD e

CA
B
D
C
A
Solution
a

BD =
1
2

BC =
1
2
b (same direction and half the length)
b

DC =

BD =
1
2
b (equivalent vectors)
c

AC =

AB +

BC = a +b
d

AD =

AB +

BD = a +
1
2
b
e

CA =

AC = (a +b) (

CA +

AC = 0)
Example 6
In the gure,

DC = kp where k R\{0}.
a Express p in terms of k, q and r.
b Express

FE in terms of k and p to show FE is parallel to DC.
c If

FE = 4

AB, nd the value of k.
A
p
B
r
q
D C
r
F
q
E
Solution
a p =

AB =

AD +

DC +

CB
= q +kp r
(1 k)p = q r
and hence p =
1
1 k
(q r)
b

FE = 2q +p +2r
= 2(r q) +p
But r q = kp p = (k 1)p . . . from a


FE = 2kp 2p +p
= (2k 1)p
c If

FE = 4

AB
(2k 1)p = 4p
2k 1 = 4
k =
5
2
Exercise 15A
1 On the same graph, draw the arrows which represent the following vectors.
Example 1
a

1
5

0
2

1
2

4
3

P1: FXS/ABE P2: FXS


9780521740494c15.xml CUAU033-EVANS October 5, 2008 7:52
396 Essential Advanced General Mathematics
2 The vector u is dened by the directed line segment from (1, 5) to (6, 6). If u =

a
b

,
Example 2
nd a and b.
3 The vector v is dened by the directed line segment from (1, 5) to (2, 10). If
v =

a
b

, nd a and b.
4 A = (1, 2), B = (3, 0), C = (2, 3) and O is the origin. Express the following vectors
in the form

a
b

.
a

OA b

AB c

BC d

CO e

CB
5 A = (2, 3), B = (4, 0), C = (1, 4) and O is the origin. Sketch the following vectors.
Example 4
a

OA b

AB c

BC d

CO e

CB
6 On graph paper, sketch the vectors joining the following pairs of points in the direction
indicated.
a (0, 0) (2, 1) b (3, 4) (0, 0) c (1, 3) (3, 4)
d (2, 4) (4, 3) e (2, 2) (5, 1) f (1, 3) (3, 0)
7 Identify vectors from 6 which are parallel to each other.
8 a Plot the points A (1, 0) , B (1, 4) , C (4, 3) , D (2, 1) on a set of coordinate axes.
b Sketch the vectors

AB,

BC,

AD, and

DC.
c Show that
i

AB =

DC ii

BC =

AD
d Describe the shape of the quadrilateral ABCD.
9 Let a =

1
2

, b =

1
3

and c =

2
1

.
Example 3
a Find
i a +b ii 2c a iii a +b c
b Show that a +b is parallel to c.
10 Find the values of m and n such that m

3
3

+n

2
4

19
61

A
M
B
C
N
D
11 In the gure A, B, C, D are the vertices of a parallelogram.

AB = a and

AD = b
M, N are the midpoints of AB and DC respectively.
a Express the following in terms of a and b.
i

MD ii

MN
b Find the relationship between

MN and

AD.
P1: FXS/ABE P2: FXS
9780521740494c15.xml CUAU033-EVANS October 5, 2008 7:52
Chapter 15 Vectors 397
12 The gure represents the triangle ABC with

AB = a
and

AC = b.
M, N are midpoints of

AB and

AC respectively.
a Express

CB and

MN in terms of a and b.
b Hence describe the relation between the
two vectors (or directed line segments).
A
M
C
N
B
13 The gure represents the regular hexagon ABCDEF
Example 5
with vectors

AF = a and

AB = b.
Express the following vectors in terms of a and b.
a

CD b

ED c

BE d

FC
e

FA f

FB
g

FE
B
A F
E
D C
14 In parallelogram ABCD,

AB = a and

BC = b. Express each of the following vectors in
Example 6
terms of a and b.
a

DC b

DA c

AC d

CA e

BD
15 In triangle OAB,

OA = a and

OB = b. P is a point on AB such that

AP = 2

PB and Q is a
point such that

OP = 3

PQ. Express each of the following in terms of a and b.
a

BA b

PB c

OP d

PQ e

BQ
16 PQRS is a quadrilateral in which

PQ = u,

QR = v,

RS = w. Express the following


vectors in terms of u, v and w.
a

PR b

QS c

PS
17 OABC is a parallelogram.

OA = u,

OC = v. M is the midpoint of AB.


a Express

OB and

OM in terms of u and v.
b Express

CM in terms of u and v.
c If P is a point on

CM and

CP =
2
3

CM, express

CP in terms of u and v.
d Find

OP and hence show that P lies on

OB.
e Find the ratio OP : PB.
x
y
B(5, 7)
A(2, 3)
X
O
15.2 Components of vectors
The vector

AB illustrated opposite can be described
by the column vector

3
4

. From the diagram it is


possible to see that

AB can be expressed as the sum
of two vectors

AX and

XB,
i.e.,

AB =

AX +

XB.
P1: FXS/ABE P2: FXS
9780521740494c15.xml CUAU033-EVANS October 5, 2008 7:52
398 Essential Advanced General Mathematics
In column vector notation

3
4

3
0

0
4

This suggests the introduction of two important vectors.


Let i be the vector of unit length with direction
the positive direction of the x axis.
x
y
O
j
i
O
x
y
u
yj
xi
Let j be the vector of unit length with direction
the positive direction of the y axis.
Note that using the column notation, i =

1
0

and j =

0
1

.
For the example above,

AX = 3i and

XB = 4j
Therefore

AB = 3i +4j.
It is possible to describe any two-dimensional vectors in this way.
For a vector u =

x
y

, u = xi + yj. It is said that u is


the sum of two components xi and yj.
The magnitude of vector u = xi + yj is denoted
by |u| and |u| =

x
2
+ y
2
Operations with vectors now look more like basic algebra.
(xi + yj) +(mi +nj) = (x +m) i +(y +n) j
k (xi + yj) = kxi +kyj
For a = xi + yj and b = mi +nj,
a = b if x = m and y = n
Example 7
a Find

AB if

OA = 3i and

OB = 2i j
b Find |2i 3j|.
Solution
a

AB =

OA +

OB
= 3i +(2i j)
= i j
b |2i 3j| =

4 +9 =

13
Example 8
A, B are points on the cartesian plane such that

OA = 2i +j and

OB = i 3j. Find

AB and |

AB|.
P1: FXS/ABE P2: FXS
9780521740494c15.xml CUAU033-EVANS October 5, 2008 7:52
Chapter 15 Vectors 399
Solution

AB =

AO +

OB
=

OA +

OB


AB = (2i +j) +i 3j
= i 4j
|

AB| =

1 +16 =

17
Unit vectors
A unit vector is a vector of length one unit (i and j are unit vectors).
The unit vector in the direction of a is a (pronounced a hat).
| a| = 1 so |a| a = a
a =
a
|a|
or
1
|a|
a
Example 9
Let a = 3i +4j.
Find |a|, the magnitude of a, and hence the unit vector in the direction of a.
Solution
a = 3i +4j so |a| =

x
2
+ y
2
=

3
2
+4
2
|a| = 5
a =
1
|a|
a
So a =
1
5
(3i +4j)
Exercise 15B
1 A, B are points on the cartesian plane such that

OA = i +2j and

OB = 3i 5j. Find

AB.
Example 7
2 OAPB is a rectangle in which the vector

OA = 5i and the vector

OB = 6j.
Express the following vectors in terms of i and j.
a

OP b

AB c

BA
3 Determine the magnitude of the following vectors.
Example 8
a 5i b 2j c 3i +4j d 5i +12j
4 The vectors u and v are given by u = 7i +8j and v = 2i 4j.
a Find |u v|.
b Find constants x and y such that xu + yv = 44j.
P1: FXS/ABE P2: FXS
9780521740494c15.xml CUAU033-EVANS October 5, 2008 7:52
400 Essential Advanced General Mathematics
5 In the triangle OAB,

OA = 10i and

OB = 4i +5j. If M is the midpoint of AB, nd

OM in
terms of i and j.
6 In the rectangle OPAQ,

OP = 2i and

OQ = j. M is the point on OP such that OM =
1
5
OP.
N is the point on MQ such that MN =
1
6
MQ.
a Find the following vectors in terms of i and j.
i

OM ii

MQ iii

MN iv

ON v

OA
b i Hence show that N is on the diagonal OA.
ii State the ratio of the lengths ON : NA.
7 The position vectors of A and B are given by

OA =

1
3

and

OB =

5
1

.
Find the distance between A and B.
8 Find the pronumerals in the following equations.
a 2i +3j = 2 (li +kj) b (x 1) i + yj = 5i +(x 4) j
c (x + y) i +(x y) j = 6i d k (i +j) = 3i 2j +l (2i j)
9 Let A = (2, 3) and B = (5, 1). Find
a

AB b |

AB|
10 Let OA = 3i, OB = i +4j and OC = 3i +j. Find
a

AB b

AC c |

BC|
11 Let A = (5, 1) , B = (0, 4) and C = (1, 0). Find
a D such that

AB =

CD b F such that

AF =

BC c G such that

AB = 2

GC
12 Let a = i +4j and b = 2i +2j. A, B and C are points such that

AO = a,

OB = b,

BC = 2a and O is the origin. Find the coordinates of A, B and C.


13 A, B, C and D are the vertices of a parallelogram and O is the origin.
A = (2, 1), B = (5, 4) and C = (1, 7).
a Find
i

OA ii

OB iii

OC iv

BC v

AD
b Hence nd the coordinates of D.
x
y
O
R
Q
P
14 The diagram shows a parallelogram OPQR.
The points P and Q have coordinates (12, 5) and (18, 13)
respectively. Find
a

OP and

PQ b |

RQ| and |

OR|
15 A(1, 6), B(3, 1), C(13, 5) are the vertices of a triangle ABC
a Find
i |

AB| ii |

BC| iii |

CA|
b Hence show that ABC is a right-angled triangle.
P1: FXS/ABE P2: FXS
9780521740494c15.xml CUAU033-EVANS October 5, 2008 7:52
Chapter 15 Vectors 401
16 A(4, 4), B(3, 1) and C(7, 3) are the vertices of the triangle ABC.
a Find the vector
i

AB ii

BC iii

CA
b Find
i |

AB| ii |

BC| iii |

CA|
c Hence show that triangle ABC is an isosceles right-angled triangle.
17 A(3, 2), B(0, 7) are points on the cartesian plane. O is the origin. M is the
midpoint of AB.
a Find
i

OA ii

OB iii

BA iv

BM
b Hence nd the coordinates of M. (Hint:

OM =

OB +

BM)
18 Find the unit vector in the direction of each of the following vectors.
Example 9
a a = 3i +4j b b = 3i j c c = i +j
d d = i j
e e =
1
2
i +
1
3
j f f = 6i 4j
y
x
(x, y, z)
z
y
z
k
i
j
1
1
1
x
z
A
yj
zk
B x
xi
O
y
15.3 Vectors in three dimensions
Points in three dimensions (3-D) are represented in axes
as shown. Vectors in 3-D are of the form
a =

x
y
z

= xi + yj + zk where
i =

1
0
0

, j =

0
1
0

and k =

0
0
1

i, j and k are represented in the gure.

OA = xi + yj + zk
By Pythagoras theorem
OB
2
= x
2
+ y
2
and OA
2
= OB
2
+BA
2
= x
2
+ y
2
+ z
2
|

OA| =

x
2
+ y
2
+ z
2
Example 10
Let a = i +j k and b = i +7k. Find
a a +b b b 3a c |a|
P1: FXS/ABE P2: FXS
9780521740494c15.xml CUAU033-EVANS October 5, 2008 7:52
402 Essential Advanced General Mathematics
Solution
a a +b = i +j k +i +7k = 2i +j +6k
b b 3a = i +7k 3(i +j k) = 2i 3j +10k
c |a| =

3
Example 11
OABCDEFG is a cuboid.

OA = 3j,

OC = k,

OD = i.
a Express each of the following in terms of i, j, k.
i

OE ii

OF iii

GF iv

GB
b M, N are the midpoints of

OD and

GF respectively.
Find MN.
D
G
C
O
F
B
A
E
Solution
a i

OE =

OA +

AE = 3j +i (

AE =

OD)
ii

OF =

OE +

EF = 3j +k +i (

EF =

OC)
iii

GF =

OA = 3j
iv

GB =

DA =

DO +

OA = i +3j
b

MN =

MD +

DN =

MD +

DG +

GN
=
1
2
i +k +
3
2
j
|

MN| =

1
4
+1 +
9
4
=

14
2
Example 12
If a = 3i +2j +2k, nd a.
Solution
|a| =

9 +4 +4 =

17
a =
1

17
(3i +2j +2k)
=
3

17
i +
2

17
j +
2

17
k
Exercise 15C
1 Let a = i +j +2k, b = 2i j +3k, c = i +k. Find
Example 10
a a b b 3b 2a +c c |b| d |b +c| e 3(a b) +2c
2 If a = 3i +j k
Example 12
a nd
i a ii 2 a
b nd the vector b in the direction of a such that |b| = 5.
P1: FXS/ABE P2: FXS
9780521740494c15.xml CUAU033-EVANS October 5, 2008 7:52
Chapter 15 Vectors 403
3 If a = i j +5k and b = 2i j 3k nd the vector c in the direction of a such that
|c| = |b|.
4 P and Q are points dened by the position vectors i +2j k and 2i j k respectively.
M is the midpoint of

PQ. Find
a

PQ b |

PQ| c

OM
G
D
C
O
F
B
E
A
5 OABCDEFG is a cuboid.

OA = 2j,

OC = 2k,

OD = i
Express the following vectors in terms of i, j and k.
a

OB b

OE c

OG d

OF
e

ED f

EG g

CE h

BD
Example 11
G
D
C
O
F
B
E
A
6 OABCDEFG is a cuboid.

OA = 3j,

OC = 2k,

OD = i
M is such that

OM =
1
3

OE.
N is the midpoint of BF. Find
a

MN b |

MN|
15.4 Applications
Example 13
Three points P, Q, and R have position vectors p, q, and k(2p +q) respectively, relative to a
xed origin O. O, P and Q are not collinear. Find the value of k if
a

QR is parallel to p b

PR is parallel to q c P, Q and R are collinear.
Solution
a

QR =

QO +

OR
= q +k(2p +q)
= q +2kp +kq
If

QR is parallel to p, there exists R\{0} such that
(k 1)q +2kp = p
k 1 = 0 and 2k =
k = 1
b

PR =

PO +

OR
= p +k(2p +q)
= (2k 1)p +kq
P1: FXS/ABE P2: FXS
9780521740494c15.xml CUAU033-EVANS October 5, 2008 7:52
404 Essential Advanced General Mathematics
If

PR is parallel to q there exists m R\{0} such that
(2k 1)p +kq = mq
k =
1
2
c If P, Q and R are collinear there exists n R\{0} such that
n

PQ =

QR
n(p +q) = (k 1)q +2kp
k 1 = n and 2k = n
which implies 3k 1 = 0
i.e., k =
1
3
Exercise 15D
1 In the diagram OR =
4
5
OP,

OP = p,

OQ = q
and PS : SQ = 1 : 4.
a Express each of the following in terms of p and q.
i

OR ii

RP iii

PQ
iv

PS v

RS
b What can be said about line segments RS and OQ?
c What type of quadrilateral is ORSQ?
d The area of triangle PRS is 5 cm
2
. What is the area of ORSQ?
q
Q
O
p
P
S
R
2 The position vectors of three points A, B and C relative to an origin O are a, b and ka
respectively. The point P lies on AB and is such that AP = 2PB. The point Q lies on BC and
is such that CQ = 6QB.
a Find in terms of a and b
i the position vector of P
ii the position vector of Q
b Given that OPQ is a straight line, nd
i the value of k
ii the ratio
OP
PQ
c The position vector of a point R is
7
3
a. Show that PR is parallel to BC.
3 The position vectors of two points A and B relative to an origin O are 3i +3.5j and
Example 13
6i 1.5j respectively.
a i Given that

OD =
1
3

OB and

AE =
1
4

AB, write down the position vectors of


D and E.
ii Hence nd |

ED|.
P1: FXS/ABE P2: FXS
9780521740494c15.xml CUAU033-EVANS October 5, 2008 7:52
Chapter 15 Vectors 405
b Given that OE and AD intersect at X and that

OX = p

OE and

XD = q

AD, nd the
position vector of X in terms of
i p ii q
c Hence determine the values of p and q.
4 The position vectors of P, Q with reference to an origin O are p and q and M is the point on
PQ such that


PM =

MQ
a Prove that the position vector of M is m where
m =
p +q
+
b The vector p = ka and the vector q = lb where k and l are positive real numbers and a
and b are unit vectors.
i Prove that the position vector of any point on the internal bisector of POQ has the
form (a +b)
ii If M is the point where the internal bisector of POQ meets PQ, show that

=
k
l
5 ORST is a parallelogram. U is the midpoint of RS and V is the midpoint of ST. Relative to
the origin O, r, s, t, u and v are the position vectors of R, S, T, U and V respectively.
a Express s in terms of r and t.
b Express v in terms of s and t.
c Hence or otherwise show that 4 (u +v) = 3 (r +s +t)
P1: FXS/ABE P2: FXS
9780521740494c15.xml CUAU033-EVANS October 5, 2008 7:52
R
e
v
i
e
w
406 Essential Advanced General Mathematics
Chapter summary
A vector is a set of equivalent directed line segments.
A directed line segment from a point A to a point B is denoted by

AB.
In two dimensions, a vector can be represented by a column of numbers, e.g.

2
3

is the
vector 2 across and 3 up.
The sum u +v can be shown diagrammatically
A
B
v
u
u + v
C
u +v = v +u
If u =

a
b

and v =

c
d

then u +v =

a +c
b +d

The vector ku, k R


+
and u = 0, has the same direction as u but its length is multiplied by
a factor k.
The vector v is in the opposite direction to v but it has the same length.
u v = u +(v)
Two non-zero vectors u and v are said to be parallel if there exists k R\{0} such that
u = kv.
For a point A, the position vector of A is

OA where O is the origin.
Every vector u can be expressed as the sum of two vectors
xi and yj, where i is the unit vector in the positive direction
of the x axis and j is the unit vector in the positive direction
of the y axis.
x
O
xi
yj
u
y
The magnitude of vector u = xi + yj is denoted by |u| and |u| =

x
2
+ y
2
The unit vector in the direction of a is
a
|a|
. This vector is denoted by a.
In three dimensions a vector u can be written as u = xi + yj + zk, where i, j and k are unit
vectors as shown.
y
z
x
(x, y, z)
y
k
i
j
x
z
If u = xi + yj + zk, |u| =

x
2
+ y
2
+ z
2
P1: FXS/ABE P2: FXS
9780521740494c15.xml CUAU033-EVANS October 5, 2008 7:52
R
e
v
i
e
w
Chapter 15 Vectors 407
Multiple-choice questions
1 The vector v is dened by the directed line segment from (1, 1) to (3, 5). If v = ai +bj then
A a = 3 and b = 5 B a = 2 and b = 4 C a = 2 and b = 4
D a = 2 and b = 3 E a = 4 and b = 2
2 If vector

AB = u and vector

AC = v then vector

CB is equal to
A u +v B v u C u v D u v E v +u
3 If vector a =

1
2

and vector b =

2
3

then a +b =
A

1
5

1
5

1
1

1
1

3
1

4 If vector a =

3
2

and vector b =

1
3

then 2a 3b =
A

9
13

9
7

9
7

3
13

3
5

5 PQRS is a parallelogram. If

PQ = p and

QR = q, then in terms of p and q,

SQ equals
A p +q B p q C q p D 2q E 2p
6 |3i 5j| =
A 2 B

34 C 34 D 8 E 16
7 If

OA = 2i +3j and

OB = i 2j then

AB equals
A i 5j B i +5j C i j D i +j E i +j
8 If

OA = 2i +3j and

OB = i 2j then

AB

equals
A 6 B 26 C

26 D

24 E 36
9 If a = 2i +3j then the unit vector parallel to a is
A 2i +3j B
1
13
(2i +3j) C
1

5
(2i +3j)
D
1

13
(2i +3j) E

13(2i +3j)
10 If a = 3i +j +3k then a is
A
1
7
(3i +j +3k) B
1

7
(3i +j +3k) C
1

19
(3i +j +3k)
D
1
19
(3i +j +3k) E 3i +j +3k
Short-answer questions (technology-free)
1 Given that a = 7i +6j and b = 2i + xj, nd the values of x for which
a a is parallel to b b a and b have the same magnitude.
2 ABCD is a parallelogram where

OA = 2i j,

AB = 3i +4j and

AD = 2i +5j. Find the
coordinates of the four vertices of the parallelogram.
P1: FXS/ABE P2: FXS
9780521740494c15.xml CUAU033-EVANS October 5, 2008 7:52
R
e
v
i
e
w
408 Essential Advanced General Mathematics
3 If a = 2i 3j +k, b = 2i 4j +5k and c = i 4j +2k, nd the values of p and
q if a +pb +qc is parallel to the x axis.
4 The position vectors of P and Q are 2i 2j +4k and 3i 7j +12k respectively.
Find
a |

PQ| b a unit vector parallel to



PQ.
5 The position vectors of A, B and C are 2j +2k, 4i +10j +18k and xi +14j +26k
respectively. Find x if A, B and C are collinear.
6

OA = 4i +3j and C is a point on OA such that

OC

=
16
5
.
a Find the unit vector in the direction of

OA.
b Hence nd

OC.
7 In the diagram, ST = 2TQ,

PQ = a,

SR = 2a and

SP = b.
a Find each of the following in terms of a and b.
R
Q
a
P
b
S
T
2a
i

SQ ii

TQ iii

RQ
iv

PT v

TR
b Show that P, T and R are collinear.
8 If a = 5i sj +2k and b = t i +2j +uk are equal vectors, nd
a i s ii t iii u
b |a|
9 The vector p has magnitude 7 units and bearing 050

and the vector q has magnitude


12 units and bearing 170

. (These are compass bearings on the horizontal plane.) Draw a


diagram (not to scale) showing p, q and p +q. Calculate the magnitude of p +q.
10 If a = 5i +2j +k and b = 3i 2j +k, nd
a a +2b b |a| c a d a b
11 O, A and B are the points (0, 0), (3, 4) and (4, 6) respectively.
a C is the point such that

OA =

OC +

OB. Find the coordinates of C.
b D is the point (1, 24) and

OD = h

OA +k

OB. Find the values of h and k.
12 Given that p = 3i +7j and q = 2i 5j, nd the values of m and n such that
mp +nq = 8i +9j.
13 The points A, B and C have position vectors a, b and c relative to an origin O. Write down
an equation connecting a, b and c for each of the following cases.
a OABC is a parallelogram.
b B divides AC in the ratio 3 : 2, i.e., AB : BC = 3 : 2.
P1: FXS/ABE P2: FXS
9780521740494c15.xml CUAU033-EVANS October 5, 2008 7:52
R
e
v
i
e
w
Chapter 15 Vectors 409
Extended-response questions
1 Let

1
0

represent a displacement 1 km due east and

0
1

represent a displacement 1 km due north. The diagram shows


a circle of radius 25 km with a centre at O(0, 0). A lighthouse
entirely surrounded by sea is located at O.
x
O
P
y
20
10
30
0
10
10
0
10 20 30
20
20
30
30
The lighthouse is not visible from points outside the circle.
The ship is initially at P, 31 km west and 32 km south of the lighthouse.
a Write down the vector

OP.
The ship is travelling parallel to vector u =

4
3

with speed 20 km/h. An hour after


leaving P the ship is at R.
b Show that the vector

PR =

16
12

, and hence nd the vector



OR.
c Show that when the ship reaches R, the lighthouse rst becomes visible.
2 Given that p = 3i +j and q = 2i +4j nd
a | p q| b | p| |q| c r, such that p +2q +r = 0
3 Let a =

2
1
2

, b =

11
7
3

, c =

7
9
7

and d =

26
12
2

a Find the value of the scalar k such that a +2b c = kd


b Find the scalars x and y such that xa + yb = d
c Use your answers to a and b to nd scalars p, q and r (not all zero) such that
pa +qb rc = 0
4 The quadrilateral PQRS is a parallelogram. The point P has coordinates (5, 8), the point R
has coordinates (32, 17) and the vector

PQ is given by

PQ =

20
15

.
a Find the coordinates of Q, and write down the vector

QR.
b Write down the vector

RS, and show that the coordinates of S are (12, 32).
5 The diagram shows the path of a light beam from its source
at O in the direction of the vector r =

3
1

. At P the beam
is reected by an adjustable mirror and meets the x axis at
M. The position of M varies, depending on the adjustment
of the mirror at P.
O
P
M
r

a Given that

OP = 4r, nd the coordinates of P.
b The point M has coordinates (k, 0). Find in terms of k an expression for the vector

PM.
c Find the magnitudes of vectors

OP,

OM and

PM, and hence nd the value of k for
which is equal to 90

.
d Find the value for which M has coordinates (9, 0).
P1: FXS/ABE P2: FXS
9780521740494c16.xml CUAU033-EVANS August 22, 2009 11:52
C H A P T E R
16
Polar coordinates and
complex numbers
Objectives
To describe points on the plane using polar coordinates
To describe graphs with polar coordinates
To transform polar coordinates to cartesian coordinates
To transform cartesian coordinates to polar coordinates
To understand the imaginary number i
To understand the set of complex numbers C
To understand the real-valued functions of the complex numbers, Re(z) and Im(z)
To represent complex numbers graphically on an Argand diagram
To understand the rules which define equality, addition, subtraction and
multiplication of complex numbers
To understand the concept of the complex conjugate
To understand the operation of division by complex numbers
To understand the modulus-argument form of a complex number and the basic
operations on complex numbers in that form
To understand the geometrical significance of multiplication and division of
complex numbers in the modulus-argument form
To be able to factorise quadratic polynomials over C
To be able to solve quadratic polynomials over C
A new set of numbers called Complex numbers is introduced in this chapter. The need for this
new set of numbers can be equated to the need for a solution of the equation x
2
+1 = 0. A
geometric interpretation is also shown to be useful.
Complex numbers can be expressed in two ways, cartesian form and polar form. As a
preliminary to this, polar coordinates are introduced.
410
P1: FXS/ABE P2: FXS
9780521740494c16.xml CUAU033-EVANS August 22, 2009 11:52
Chapter 16 Polar coordinates and complex numbers 411
16.1 Polar coordinates
In previous work the cartesian coordinate system has been used to represent points in
two-dimensional space. The point (x, y) is described in terms of its horizontal displacement (x)
and its vertical displacement (y) from a xed point called the origin (O).
An alternative way of locating the point P is to describe it in terms of its polar coordinates
[r, ] where r species the distance from the origin or pole and species the angle of the line
OP relative to the line OZ which extends to the right from O and is called the polar axis.
Note: An angle in an anticlockwise direction from OZ is
considered to be positive.

O Z
r
P[r, ]
For example, the point P[4, 60

] is located a
distance of 4 units along a line forming an angle
of 60

with the polar axis.


60
O Z
4
P[4, 60]
Using this system it is clear that any point can be specied in a number of different ways.
For example, the point [4, 60

] may also be specied by [4, 120

].
The angle = 120

is measured in a clockwise direction from O.


The diagram below and to the left illustrates the point P

[4, 120

] and the diagram to the


right, [4, 120

].
P'
4
120
O
Z
60
4
120
O Z
P
P[4, 60

] may also be specied by P[4, 300

] or P[4, 240

].
Example 1
Plot the point P with coordinates [3, 30

].
Solution
3
30
O Z
P
P1: FXS/ABE P2: FXS
9780521740494c16.xml CUAU033-EVANS August 22, 2009 11:52
412 Essential Advanced General Mathematics
The relationship between cartesian and
polar coordinates
If a set of cartesian axes is superimposed
over a polar axis, the relationship between
cartesian and polar coordinates can be
established.
x(Z)
y
O

r
r cos()
r sin()
P(x, y)
From the diagram
x = r cos 1
and y = r sin 2
The angle can be found by nding a solution which satises both equations 1 and 2
Squaring both sides of equations 1 and 2 and adding yields
x
2
+ y
2
= r
2
cos
2
+r
2
sin
2

= r
2
(cos
2
+sin
2
)
i.e. x
2
+ y
2
= r
2
Using these relationships, coordinates can be converted from cartesian to polar and vice versa.
Example 2
a Express (

3, 1) in polar form. b Express [

2, 45

] in cartesian form.
Solution
a
r
2
= x
2
+ y
2
sin =
1
2
= (

3)
2
+(1)
2
= 4 and cos =

3
2
r = 2 = 30

3, 1) species the same point as [2, 30

]
b
r =

2 and = 45

x =

2 cos 45

2
1

2
= 1
y =

2 sin 45

2
1

2
= 1
[

2, 45

] species the same point as (1, 1)


P1: FXS/ABE P2: FXS
9780521740494c16.xml CUAU033-EVANS August 22, 2009 11:52
Chapter 16 Polar coordinates and complex numbers 413
Curve sketching using polar coordinates
In the same way that graphs of relationships in cartesian form can be sketched, relationships
expressed in polar form can also be sketched. Some very interesting curves result from simple
polar equations.
It is recommended that to sketch these graphs a graphics calculator or a computer graphing
package be used. Graphs can of course be plotted using a table of values. A sheet of polar
graph paper is also useful although a sheet of blank paper will sufce as long as a ruler and a
protractor are used.
Using the TI-Nspire
Example 3
Plot the graph of r = 3 (1 cos ).
Solution
Open a Graphs & Geometry application
(c2) and select Polar from the Graph
Type menu (b33).
Enter r1 () = 3 (1 cos ()) as shown
Note that the domain of, as well as the
step size, can be adjusted in this window.
The graph is shown using the Zoom,
Fit command from the Window menu
(b4 ).
Note that every point on the graph satises
r = 3 (1 cos ()) . For example, for
= 60

r = 3 (1 cos (60

)) = 3
_
1
1
2
_
=
3
2
For = 180

, r = 3 (1 cos (180

))
= 3 (1 (1)) = 6
For = 90

, r = 3 (1 cos (90)) = 3

P1: FXS/ABE P2: FXS


9780521740494c16.xml CUAU033-EVANS August 22, 2009 11:52
414 Essential Advanced General Mathematics
Note that Trace (b5) can be used to
show the coordinates of the points on the
graph in the form [r, ].
To go to the point where = , simply
type followed by enter. The cursor will
then move to the point [r, ] = [6, ] as
shown.
Example 4
Sketch the graph of r = .
Solution
Open a Graphs & Geometry application
(c2) and select Polar from the Graph
Type menu (b33).
Enter r1 () = .
The graph is shown.
If the domain of is extended, the graph
continues to spiral out. This can be
observed by extending the domain to
0 < < 6.
The resulting graph is shown using the
Zoom, Out command from the Window
menu (b44).
P1: FXS/ABE P2: FXS
9780521740494c16.xml CUAU033-EVANS August 22, 2009 11:52
Chapter 16 Polar coordinates and complex numbers 415
Using the Casio ClassPad
Solution for Example 3
Plot the graph of r = 3 (1 cos ).
Ensure that the mode is set to radians.
In tap and from the menu select
.
Enter the equation r1 = 3 (1 cos ()) and tap
$to produce the graph.
In the screen shown, the window was selected by tapping Zoom, quick initialize.
Example 5
Find the polar equation of the circle whose cartesian equation is
x
2
+ y
2
= 4x
Solution
Let x = r cos and y = r sin
Then r
2
cos
2
+r
2
sin
2
= 4r cos
r
2
(cos
2
+sin
2
) = 4r cos
r
2
4r cos = 0
r(r 4 cos ) = 0
r = 0 or r = 4 cos
r = 4 cos is the polar equation of the circle
Example 6
Find the cartesian equation corresponding to each of the following polar equations.
a r = 3 b r =
1
1 +sin
c r = 3(1 cos )
P1: FXS/ABE P2: FXS
9780521740494c16.xml CUAU033-EVANS August 22, 2009 11:52
416 Essential Advanced General Mathematics
Solution
a
r = 3
_
x
2
+ y
2
= 3
x
2
+ y
2
= 9
The circle with centre (0, 0)
and radius 3
b
r =
1
1 +sin
implies r(1 +sin ) = 1
i.e. r +r sin = 1

_
x
2
+ y
2
= 1 y
x
2
+ y
2
= 1 2y + y
2
x
2
= 2
_
y
1
2
_
y =
x
2
2
+
1
2
c
r = 3(1 cos )
r
2
= 3r 3r cos (Multiplying both sides of equation by r)
x
2
+ y
2
= 3
_
x
2
+ y
2
3x
x
2
+ y
2
+3x = 3
_
x
2
+ y
2
Exercise 16A
1 Plot each of the following points using a polar axis.
Example 1
a A[2, 30

] b B[3, 45

] c C[2, 60

] d D[4, 30

]
e E[5, 50

] f F[5, 50

] g G[5, 130

] h H[5, 130

]
2 Plot each of the following points using a polar axis.
a A[1, ] b B[1, ] c C
_
2,

2
_
d D
_
3,
3
4
_
3 Convert the following cartesian coordinates to polar coordinates. (Remember to note
which quadrant each point is in.)
Example 2a
a (4, 4) b (1,

3) c (2

3, 2) d (5, 12)
e (6, 5) f (

3, 1) g (5, 12) h (4, 3)


4 Convert the following polar coordinates to cartesian coordinates.
Example 2b
a [2, 30

] b
_
4,

2
_
c
_
1,
5
4
_
d [4, 2]
e
_
2,
7
6
_
f [5, 240

] g [2, 180

] h [1, 120

]
5 Plot each of the following polar graphs.
Examples 3, 4
a r =
3
cos
b r =
4
sin
c r = 2 cos
d r = 2, 0 6 e r =

6
4 f r = cos 2
g r = 5(1 +cos ) h r = 2(1 sin ) i r = 3 cos +2
j r =

cos 2 k r =
_

, 0 6 l r = 2 sin 2
P1: FXS/ABE P2: FXS
9780521740494c16.xml CUAU033-EVANS August 22, 2009 11:52
Chapter 16 Polar coordinates and complex numbers 417
6 Obtain the polar equations of each of the following.
Example 5
a x
2
+ y
2
= 16 b x + y = 6 c x
2
= y d
x
2
4
+ y
2
= 1
7 Obtain the cartesian equations of each of the following.
Example 6
a r = 2 b r = a(1 +cos ) c r = a cos
d r = 2a(1 +sin 2) e r =
a
1 +cos
f r =
a
1 +sin
16.2 The set of complex numbers
In earlier work in mathematics it was assumed that an equation of the form x
2
= 1 had no
solutions. Mathematicians of the eighteenth century introduced the imaginary number i with
the property i
2
= 1. i is dened as i =

1 and the equation x


2
= 1 has two solutions, i
and i . By considering i such that i
2
= 1 then the square roots of all negative numbers may
be found.
For example

4 =

4 1
=

1
= 2i
Imaginary numbers led to the introduction of complex numbers, which further broadened the
scope of mathematical thinking. Today complex numbers are widely used in engineering, the
study of aerodynamics and many other branches of physics.
Consider the equation x
2
+2x +3 = 0. Using the quadratic formula to solve yields:
x =
2

4 12
2
=
2

8
2
= 1

2
This equation has no real solutions since the discriminant = b
2
4ac is less than zero.
However, for complex numbers
x = 1

2i
A complex number is an expression of the form a +bi , where a and b are real numbers. C
is the set of complex numbers, i.e. C = {a +bi : a, b R}. The letter often used to denote a
complex number is z.
Therefore z C implies z = a +bi where a, b R
If a = 0, z is said to be imaginary.
If b = 0, z is real.
Real numbers and imaginary numbers are subsets of C.
P1: FXS/ABE P2: FXS
9780521740494c16.xml CUAU033-EVANS August 22, 2009 11:52
418 Essential Advanced General Mathematics
Functions of complex numbers
Let z = a +bi
Re (z) is a function which denes the real component of z. Im(z) is a function which denes
the value of the imaginary component of z.
Re (z) = a and Im(z) = b
Note: Re (z) and Im(z) are both real-valued functions of z, i.e. Re : C R and Im: C R.
So for the complex number z = 2 +5i, Re (z) = 2 and Im(z) = 5.
Equality of complex numbers
Two complex numbers are equal if and only if both their real and imaginary parts are equal.
i.e. x
1
+ y
1
i = x
2
+ y
2
i
if and only if x
1
= x
2
and y
1
= y
2
Example 7
If 4 3i = 2a +bi nd the values of a and b.
Solution
2a = 4 and b = 3
a = 2
Example 8
Find the values of a and b such that (2a +3b) +(a 2b)i = 1 +3i
Solution
2a +3b = 1 1
a 2b = 3 2
2 2 gives
2a 4b = 6 3
1 3 gives
7b = 7
b = 1 and a = 1
Operations with complex numbers
Addition and subtraction
If z
1
= a +bi and z
2
= c +di (a, b, c, d R)
Then z
1
z
2
= (a c) +i (b d)
i.e. Re (z
1
z
2
) = Re (z
1
) Re (z
2
) and Im(z
1
z
2
) = Im(z
1
) Im(z
2
)
P1: FXS/ABE P2: FXS
9780521740494c16.xml CUAU033-EVANS August 22, 2009 11:52
Chapter 16 Polar coordinates and complex numbers 419
Example 9
If z
1
= 2 3i and z
2
= 4 +5i nd
a z
1
+ z
2
b z
1
z
2
Solution
a z
1
+ z
2
= 2(2 +4) +(3 +5)i
= 2 +2i
b z
1
z
2
= (2 4) +(3 5)i
= 6 8i
Multiplication by a real constant
If z = a +bi and k R
then kz = k(a +bi )
= ka +kbi
For example, if z = 3 6i then 3z = 9 18i
Multiplication by powers of i
Successive multiplication by powers of i gives the following:
i
1
= i
i
2
= 1
i
3
= i
i
4
= (1)
2
= 1
i
5
= i
and so on
In general, for n = 0, 1, 2, 3, . . .
i
4n
= 1
i
4n+1
= i
i
4n+2
= 1
i
4n+3
= i
When multiplying by powers of i, the usual index laws apply.
Example 10
Simplify
a i
13
b 3i
4
(2i )
3
Solution
a i
13
= i
43+1
= i
b 3i
4
(2i )
3
= 3 (2)
3
i
4
i
3
= 24i
7
= 24i
P1: FXS/ABE P2: FXS
9780521740494c16.xml CUAU033-EVANS August 22, 2009 11:52
420 Essential Advanced General Mathematics
Exercise 16B
1 State the values of Re (z) and Im(z) for each of the following.
a 2 +3i b 4 +5i
c
1
2

3
2
i
d 4 e 3i f

2 2

2i
2 Find the values of a and b in each of the following if
Examples 7, 8
a 2a 3bi = 4 +6i b a +b 2abi = 5 12i
c 2a +bi = 10 d 3a +(a b)i = 2 +i
3 Simplify the following.
Example 9
a (2 3i ) +(4 5i ) b (4 +i ) +(2 2i )
c (3 i ) (3 +i ) d (2

2i ) +(5

8i )
e (1 i ) (2i +3) f (2 +i ) (2 i )
g 4(2 3i ) (2 8i ) h (5 4i ) +(1 +2i )
i 5(i +4) +3(2i 7)
j
1
2
(4 3i )
3
2
(2 i )
4 Simplify
Example 10
a

16 b 2

9 c

2 d i
3
e i
14
f i
20
g 2i i
3
h 4i
4
3i
2
i

8i
5

2
5 Simplify
a i (2 i ) b i
2
(3 4i ) c

2i (i
_
2) d

3(

3 +

2)
16.3 Multiplication and division of
complex numbers
Multiplication of complex numbers
If z
1
= a +bi and z
2
= c +di (a, b, c, d R)
Then z
1
z
2
= (a +bi ) (c +di )
= ac +bci +adi +dbi
2
= (ac bd) +(bc +ad)i (bdi
2
= bd)
Example 11
If z
1
= 3 2i and z
2
= 1 +i , nd z
1
z
2
.
Solution
z
1
z
2
= (3 2i )(1 +i )
= 3 2i +3i 2i
2
= 5 +i
P1: FXS/ABE P2: FXS
9780521740494c16.xml CUAU033-EVANS August 22, 2009 11:52
Chapter 16 Polar coordinates and complex numbers 421
Conjugate of a complex number
If z = a +bi then the conjugate of z denoted by the symbol z is
z = a bi
For example, the conjugate of 4 +3i is 4 3i and vice versa.
Note that zz = (a +bi ) (a bi )
= a
2
+abi abi b
2
i
2
= a
2
+b
2
which is a real number
Using this result, a
2
+b
2
can now be factorised over the set of complex numbers.
Example 12
If z
1
= 2 3i and z
2
= 1 +2i nd
a (z
1
+ z
2
) and z
1
+ z
2
b z
1
z
2
and z
1
z
2
Solution
z
1
= 2 +3i and z
2
= 1 2i
a
z
1
+ z
2
= (2 3i ) +(1 +2i )
= 1 i
(z
1
+ z
2
) = 1 +i
z
1
+ z
2
= 2 +3i +1 2i
= 1 +i
b
z
1
z
2
= (2 3i )(1 +2i ) = 4 +7i
z
1
z
2
= 4 7i
z
1
z
2
= (2 +3i )(1 2i ) = 4 7i
In general it can be stated that
the conjugate of the sum of two complex numbers is equal to the sum of the conjugates
the conjugate of the product of two complex numbers is equal to the product of the
conjugates.
i.e. (z
1
+ z
2
) = z
1
+ z
2
and (z
1
z
2
) = z
1
z
2
Division of complex numbers
Division of one complex number by another relies on the fact that the product of a complex
number and its conjugate is a real number.
If z
1
= a +bi and z
2
= c +di (a, b, c, d R)
Then
z
1
z
2
=
(a +bi )
(c +di )
If the numerator and denominator are multiplied by the conjugate of z
2
then
z
1
z
2
=
(a +bi )
(c +di )

(c di )
(c di )
=
ac +bci adi bdi
2
c
2
+d
2
=
(ac +bd)
c
2
+d
2
+
(bc ad)i
c
2
+d
2
P1: FXS/ABE P2: FXS
9780521740494c16.xml CUAU033-EVANS August 22, 2009 11:52
422 Essential Advanced General Mathematics
Example 13
If z
1
= 2 i and z
2
= 3 +2i , nd
z
1
z
2
.
Solution
z
1
z
2
=
2 i
3 +2i

3 2i
3 2i
=
6 3i 4i +2i
2
3
2
+2
2
=
4 7i
13
=
1
13
(4 7i )
Example 14
Solve for z the equation (2 +3i ) z = 1 2i
Solution
(2 +3i )z = 1 2i
z =
1 2i
2 +3i
=
1 2i
2 +3i

2 3i
2 3i
z =
8 i
13
There is an obvious similarity in the process of expressing a complex number with a real
denominator and the process of rationalising the denominator of a surd expression.
Example 15
If z = 2 5i nd z
1
and express with a real denominator.
Solution
z
1
=
1
z
=
1
2 5i
=
1
2 5i

2 +5i
2 +5i
=
2 +5i
29
=
1
29
(2 +5i )
P1: FXS/ABE P2: FXS
9780521740494c16.xml CUAU033-EVANS August 22, 2009 11:52
Chapter 16 Polar coordinates and complex numbers 423
Using the TI-Nspire
The TI-Nspire can be used to deal with complex numbers. Select Rectangular form in
the Document Settings (c81) to perform calculations on complex numbers in
the form a +bi .
The square root of a negative number can now
be performed as shown.
The results of the operations +, , and

,
are illustrated using the two complex numbers
2 +3i and 3 +4i .
It is possible to perform arithmetic
operations with complex numbers as shown.
The Real Part command from the Complex
Number Tools submenu of the Number menu
(b292) can be used as shown to nd
the real part of a complex number.
The Magnitude command from the
Complex Number Tools submenu of the
Number menu (b295) can be used as
shown to nd the modulus of a complex number.
The Complex Conjugate command from the
Complex Number Tools submenu of the Number
menu (b291) can be used as shown
to nd the complex conjugate of a complex
number.
P1: FXS/ABE P2: FXS
9780521740494c16.xml CUAU033-EVANS August 22, 2009 11:52
424 Essential Advanced General Mathematics
There are also commands for factorising
polynomials over the complex numbers and for
solving polynomial equations over the complex
numbers. These are available from the Complex
submenu of the Algebra menu (b3 ).
Using the Casio ClassPad
In tap Real in the status bar at the bottom of the screen to enter Cplx mode. In this
mode enter

1 and tap to obtain the answer i. Similarly,

16 will return the


answer 4i.
Operations
i is found in in the on-screen keyboard.
With the calculator set to Complex mode, a
number of arithmetic operations can be carried out,
as shown in the screen at right using options from
Interactive, Complex.
Polynomials can be factorised and solved over the
complex number eld using Interactive, transformation
and Equation/inequality, solve.
Exercise 16C
1 Expand and simplify
Example 11
a (4 +i )
2
b (2 2i )
2
c (3 +2i )(2 +4i )
d (1 i )
2
e (

3i )(

2 +

3i ) f (5 2i )(2 +3i )
P1: FXS/ABE P2: FXS
9780521740494c16.xml CUAU033-EVANS August 22, 2009 11:52
Chapter 16 Polar coordinates and complex numbers 425
2 Write down the conjugate of each of the following complex numbers.
a 2 5i b 1 +3i c

5 2i d 5i
3 If z
1
= 2 i and z
2
= 3 +2i nd
Example 12
a z
1
b z
2
c z
1
z
2
d z
1
z
2
e z
1
z
2
f z
1
+ z
2
g z
1
+ z
2
h z
1
+ z
2
4 If z = 2 4i express each of the following in the form x + yi .
Example 15
a z b zz c z + z d z(z + z)
e z z f i (z z) g z
1
h
z
i
5 Find the values of a and b such that (a +bi )(2 +5i ) = 3 i
6 Express in the form x + yi
Example 13
a
2 i
4 +i
b
3 +2i
2 3i
c
4 +3i
1 +i
d
2 2i
4i
e
1
2 3i
f
i
2 +6i
7 Find the values of a and b if (3 i )(a +bi ) = 6 7i
8 Solve each of the following for z.
Example 14
a (2 i )z = 4 +2i b (1 +3i )z = 2 i c (3i +5)z = 1 +i
d 2(4 7i )z = 5 +2i e z(1 +i ) = 4
16.4 Argand diagrams
An Argand diagram is a geometrical representation of the set of complex numbers. In a
vector sense, a complex number has two dimensions; the real part and the imaginary part.
Therefore a plane is required to represent C.
An Argand diagram is drawn with two
perpendicular axes. The horizontal axis
represents Re(z), z C, and the vertical
axis represents Im(z), z C.
1
1
2
2
3
3 3
3
2
2
1
1
0
(2 + i) (3 + i)
(2 3i)
Re(z)
Im(z)
Each point on an Argand diagram represents
a complex number. The complex number a +bi
is situated at the point (a, b) on the equivalent
cartesian axes as shown by the examples in this
gure. The number written as a +bi is
called the cartesian form of the complex number.
P1: FXS/ABE P2: FXS
9780521740494c16.xml CUAU033-EVANS August 22, 2009 11:52
426 Essential Advanced General Mathematics
Example 16
Write down the complex number represented
by each of the points A to F on this Argand diagram.
4
5
5
5
5
0
Re(z)
Im(z)
A
B
C
D
F
E
Solution
A: 2 +3i B: 4i
C: 5 D: 1 +i
E : 5 2i F : 1 3i
Geometrical representation of the basic
operations on complex numbers
The addition of two complex numbers is similar to a vector sum and follows the triangle of
vectors rule.
The multiplication by a scalar follows vector properties of parallel position vectors.
Re(z)
Im(z)
0
z
1
+ z
2
z
1
z
2
Re(z)
Im(z)
0
bz
cz
az
z
a > 1
0 < b < 1
c < 0
The subtraction z
1
z
2
is represented by the sum z
1
+(z
2
).
Example 17
a Represent the following points on an Argand diagram.
i 2 ii 3i iii 2 i iv (2 +3i ) v 1 +2i
b Let z
1
= 2 +i and z
2
= 1 +3i .
Represent z
1
, z
2
, z
1
+ z
2
and z
1
z
2
on an Argand diagram and verify that the complex
number sum and difference follow the vector triangle properties.
P1: FXS/ABE P2: FXS
9780521740494c16.xml CUAU033-EVANS August 22, 2009 11:52
Chapter 16 Polar coordinates and complex numbers 427
Solution
a
1
1
2
2
2
3
3
3
3
2
2
1
1
0
(2 + 3i)
1 + 2i
2 i
3i
Re(z)
Im(z)
b
1
1
2
2
3
4
4 3 3 4
4
3
2
2
1
1
Re(z)
Im(z)
z
1
+ z
2
z
1

z
1
z
2
z
2
z
2
0
z
1
+ z
2
= (2 +i ) +(1 +3i )
= 1 +4i
z
1
z
2
= (2 +i ) (1 +3i )
= 3 2i
Rotation about the origin
When the complex number 2 +3i is multiplied by 1 the result is 2 3i .
This can be considered to be achieved through a rotation of 180

about the origin. When the


complex number 2 +3i is multiplied by i,
i.e. i (2 +3i ) = 2i +3i
2
= 2i 3
= 3 +2i
the result can be seen to be achieved through
a rotation of 90

in an anticlockwise direction
about the origin. If 3 +2i is multiplied by
i the result is 2 3i . This is again achieved
through a rotation of 90

in an anticlockwise
direction about the origin.
Re(z)
Im(z)
0
3 + 2i
2 3i
2 + 3i
Example 18
If z
1
= 1 4i and z
2
= 2 +2i , nd z
1
+ z
2
algebraically and illustrate z
1
+ z
2
on an
Argand diagram.
Solution
Im(z)
Re(z)
z
1
1 2 3 4
1
2
3
4
1 2 3 4
0
1
2
3
z
1
+ z
2
z
2
z
1
+ z
2
= (1 4i ) +(2 +2i )
= 1 2i
P1: FXS/ABE P2: FXS
9780521740494c16.xml CUAU033-EVANS August 22, 2009 11:52
428 Essential Advanced General Mathematics
Exercise 16D
1 Write down the complex numbers
represented on the following Argand diagram.
Example 16
5
1
2
1 2
1
2
Im(z)
Re(z) 1 2 3 44
3
4
F
0
A
E
D
C
B
3 4 5
3
2 Represent each of the following complex numbers as points on an Argand plane.
Example 17
a 3 4i b 4 +i c 4 +i d 3 +0i e 0 2i f 5 2i
3 If z
1
= 6 5i and z
2
= 3 +4i , nd algebraically and represent on an Argand diagram.
Example 18
a z
1
+ z
2
b z
1
z
2
4 If z = 1 +3i , represent on an Argand diagram
a z b z c z
2
d z e
1
z
5 If z = 2 5i , represent on an Argand diagram
a z b zi c zi
2
d zi
3
e zi
4
16.5 Solving equations over the complex field
Quadratic equations for which the discriminant is less than zero have no solutions for the real
numbers. The introduction of the complex number enables such quadratic equations to be
solved. Further solutions to higher degree polynomials may also be found using complex
numbers. Solution of higher degree polynomials appears in the Specialist Mathematics course.
In this chapter only quadratics will be considered.
Sum of two squares
Earlier it was seen that the product of a complex number a +bi and its conjugate a bi
yielded the result
(a +bi )(a bi ) = a
2
+b
2
Hence sums of two squares can be factorised enabling equations of the form z
2
+a
2
= 0 to
be solved.
P1: FXS/ABE P2: FXS
9780521740494c16.xml CUAU033-EVANS August 22, 2009 11:52
Chapter 16 Polar coordinates and complex numbers 429
Example 19
Solve the equations
a z
2
+16 = 0 b 2z
2
+6 = 0
Solution
a z
2
+16 = 0
z
2
16i
2
= 0
(z +4i )(z 4i ) = 0
z = 4i
b 2z
2
+6 = 0
2(z
2
+3) = 0
2(z
2
3i
2
) = 0
2(z +

3i )(z

3i ) = 0
z =

3i
Solution of quadratic equations
To solve quadratic equations where the discriminant is less than zero, still use the quadratic
formula in the usual way.
Example 20
Solve the equation 3z
2
+5z +3 = 0
Solution
Using the quadratic formula
z =
5

25 36
6
=
5

11
6
=
1
6
(5

11i )
Using the TI-Nspire
Each of the expressions in the above examples can be factorised using cFactor from the
Complex submenu of Algebra, for example,
cfactor (z
2
+16, z).
Each of the equations in the above examples can be solved using cSolve from the
Complex submenu of Algebra, for example
cSolve (3z
2
+5z +3 = 0, z).
P1: FXS/ABE P2: FXS
9780521740494c16.xml CUAU033-EVANS August 22, 2009 11:52
430 Essential Advanced General Mathematics
Using the Casio ClassPad
To factorise in the above examples, ensure the mode is set to Cplx.
Enter and highlight the expression z
2
+16 then
tap Interactive, Transformation, rFactor.
To solve in the above examples, the usual method for solving equations is used. For
example, enter and highlight 3z
2
+5z +3 = 0 then tap Interactive,
Equation/inequality, solve and ensure that the variable selected is z.
Exercise 16E
1 Solve each of the following equations over C.
Examples 19, 20
a z
2
+4 = 0 b 2z
2
+18 = 0 c 3z
2
= 15
d (z 2)
2
+16 = 0 e (z +1)
2
= 49 f z
2
2z +3 = 0
g z
2
+3z +3 = 0 h 2z
2
+5z +4 = 0 i 3z
2
= z 2
j 2z = z
2
+5 k 2z
2
6z = 10 l z
2
6z = 14
16.6 Polar form of a complex number
Earlier in this chapter it was shown that points on a cartesian plane (x, y) may be represented in
terms of polar coordinates [r, ]. Similarly, complex numbers may be represented in polar
form.
Recalling that x = r cos and y = r sin where
r
2
= x
2
+ y
2
then the point P in the complex plane
corresponding to the complex number in cartesian form,
z = x + yi , may be represented as shown in the diagram.
z = r cos +r sin i
= r(cos +sin i )
Im(z)
Re(z)
P(x + iy)
y r
x

0
The polar form is abbreviated to z = r cis .
r =
_
x
2
+ y
2
is called the absolute value or modulus of z. It is denoted by mod z or |z|.
Remember that is measured in an anticlockwise direction from the horizontal axis.
Note: The same point may be represented a number of ways in polar form, since
cos = cos( 2n) and sin = sin ( 2n), where n Z, the polar form of a complex
number is not unique.
i.e. z = r cis = r cis ( +2n), n Z
P1: FXS/ABE P2: FXS
9780521740494c16.xml CUAU033-EVANS August 22, 2009 11:52
Chapter 16 Polar coordinates and complex numbers 431
Usually the interval < is used. The corresponding value of is called the principal
value of the argument of z and is denoted by Arg z.
i.e. < Arg z
Example 21
Express in polar form the following complex numbers
a z = 1 +

3i b z = 2 2i
Solution
a First note that z = 1 +

3 i is a point in the 1st quadrant.


0 < <

2
Now x = 1 and y =

3
Therefore r =

1 +3
= 2
also =

3
(since cos =
1
2
and sin =

3
2
)
z = 1 +

3i
= 2 cis

3
b z = 2 2i is a point in the 4th quadrant.


2
< < 0
Now x = 2 and y = 2
Therefore r =

4 +4
=

8
= 2

2
Also =

4
(since cos =
1

2
and sin =
1

2
)
z = 2 2i
= 2

2 cis
_

4
_
P1: FXS/ABE P2: FXS
9780521740494c16.xml CUAU033-EVANS August 22, 2009 11:52
432 Essential Advanced General Mathematics
Example 22
Express in cartesian form z = 2 cis
_
2
3
_
Solution
x = r cos = 2 cos
_
2
3
_
= 2
_

1
2
_
= 1
y = r sin = 2 sin
_
2
3
_
= 2
_

3
2
_
=

3
z = 2 cis
_
2
3
_
= 1

3i
Multiplication and division in polar form
If z
1
= r
1
cis and z
2
= r
2
cis
2
Then z
1
z
2
= r
1
r
2
cis (
1
+
2
)
and
z
1
z
2
=
r
1
r
2
cis (
1

2
)
These results may be proved using the addition formulas for sine and cosine established in
Chapter 11. This is left as an exercise for the reader.
Example 23
If z
1
= 2 cis 30

and z
2
= 4 cis 20

nd the product z
1
z
2
and represent it on an Argand
diagram.
Solution
Im(z)
Re(z)
30
50
z
2
z
1
z
2
20
0
z
1
z
1
z
2
= r
1
r
2
cis (
1
+
2
)
= 2 4 cis (20

+30

)
= 8 cis 50

P1: FXS/ABE P2: FXS


9780521740494c16.xml CUAU033-EVANS August 22, 2009 11:52
Chapter 16 Polar coordinates and complex numbers 433
Example 24
If z
1
= 3 cis

2
and z
2
= 2 cis
_
5
6
_
, nd the product z
1
z
2
.
Solution
z
1
z
2
= r
1
r
2
cis (
1
+
2
)
= 6 cis
_

2
+
5
6
_
= 6 cis
_
4
3
_
z
1
z
2
= 6 cis
_
2
3
_
since < Arg z
Example 25
If z
1
=

3 +i and z
2
= 2

3 +2i , nd the quotient


z
1
z
2
and express it in cartesian form.
Solution
First express z
1
and z
1
in polar form.
|z
1
| =

3 +1 Arg z
1
=
5
6
, since sin
1
=
1
2
and cos
1
=

3
2
= 2 where Arg z
1
=
1
|z
2
| =

12 +4 Arg z
2
=

6
, since sin
2
=
1
2
and cos
2
=

3
2
= 4 where Arg z
2
=
2
z
1
= 2 cis
_
5
6
_
and z
2
= 4 cis

6
z
1
z
2
=
r
1
r
2
cis (
1

2
)
=
2
4
cis
_
5
6


6
_
=
1
2
cis
_
2
3
_
In cartesian form
z
1
z
2
=
1
2
cos
_
2
3
_
+
1
2
sin
_
2
3
_
i
=
1
2
_
1
2
_
+
1
2
_

3
2
_
i
=
1
4
(1

3)i
P1: FXS/ABE P2: FXS
9780521740494c16.xml CUAU033-EVANS August 22, 2009 11:52
434 Essential Advanced General Mathematics
Exercise 16F
1 Express each of the following in the simplest polar form.
Example 21
a 1 +

3i b 1 i c 2

3 +2i
d 4 4i e 12 12

3i
f
1
2
+
1
2
i
2 Express each of the following in the form x + yi .
Example 22
a 3 cis

2
b

2 cis

3
c 2 cis

6
d 5 cis
3
4
e 12 cis
5
6
f 3

2 cis

4
g 5 cis
4
3
h 5 cis
2
3
3 Simplify the following and express the answers in cartesian form.
Examples 23, 24, 25
a
_
2 cis

6
_
.
_
3 cis

12
_
b
_
4 cis

12
_
.
_
3 cis

4
_
c
_
cis

4
_
.
_
5 cis
5
12
_
d
_
12 cis

3
_
.
_
3 cis
2
3
_
e
_
12 cis
5
6
_
.
_
3 cis

2
_
f (

2 cis ).
_

3 cis
3
4
_
g
_
10 cis

4
_
_
5 cis

12
_ h
_
12 cis

3
_
3 cis
2
3
i
_
12

8 cis
3
4
_
_
3

2 cis

12
_ j
_
20 cis

6
_
_
8 cis
5
6
_
P1: FXS/ABE P2: FXS
9780521740494c16.xml CUAU033-EVANS August 22, 2009 11:52
R
e
v
i
e
w
Chapter 16 Polar coordinates and complex numbers 435
Chapter summary
The polar coordinates [r, ] may be represented as follows.
P[r, ]

pole
polar axis
O
is measured in an anticlockwise direction
from the polar axis.
For conversion of coordinates from cartesian to polar and vice versa
x = r cos , y = r sin and hence x
2
+ y
2
= r
2
.
Therefore
_
2,

3
_
describes the same point as
_
2 cos

3
, 2 sin

3
_
= (1,

3)
For (1, 1), r =

2
1 =

2 cos and 1 =

2 sin
Therefore cos =
1

2
and sin =
1

2
and =

4
(1, 1) =
_

2,

4
_
For conversion of an equation from cartesian to polar use
x = r cos , y = r sin and x
2
+ y
2
= r
2
Therefore x + y = 1
becomes
r cos +r sin = 1
i.e. r(cos +sin ) = 1
Consider y = x
2
This becomes
r sin = r
2
cos
2

r sin r
2
cos
2
= 0
r(sin r cos
2
) = 0
r = 0 or sin = r cos
2

r = 0 is the pole. The second equation becomes r =


tan
cos
i is an imaginary number with the property i
2
= 1.
C, the set of complex numbers, is dened by C = {a +bi : a, b R}.
Real numbers and Imaginary numbers are subsets of C.
Re (z) is the real component of z.
Im(z) is the value of the imaginary component of z.
P1: FXS/ABE P2: FXS
9780521740494c16.xml CUAU033-EVANS August 22, 2009 11:52
R
e
v
i
e
w
436 Essential Advanced General Mathematics
z
1
= z
2
Re (z
1
) = Re (z
2
) and Im(z
1
) = Im(z
2
)
The Argand diagram is a geometrical representation of C.
Let z
1
= a +bi and z
2
= c +di, then z
1
z
2
= (ac bd) +(ad +bc)i
The modulus of z, |z|, is the distance from the origin of the point represented by z.
The argument of z, arg z, is an angle measured anticlockwise about the origin from the
positive direction of the x axis to the line joining the origin to z.
The Argument of z, Arg z, is arg z expressed as an angle in the interval (, ].
The modulus-argument form of the complex number z is given as:
z = r(cos +i sin ) where r = |z|
cos =
Re (z)
|z|
and sin =
Im(z)
|z|
r(cos +i sin ) is usually written as r cis
The complex conjugate of z is denoted by z, where z = Re (z) Im(z)i ; zz, (z + z) R
The division of complex numbers:
z
1
z
2
=
z
1
z
2
|z
2
|
2
Multiplication and division of the modulus-argument form
Let z
1
= r
1
cis
1
, z
2
= r
2
cis
2
Then z
1
z
2
= r
1
r
2
cis (
1
+
2
),
z
1
z
2
=
r
1
r
2
cis (
1

2
)
Multiple-choice questions
1 The polar coordinates [3, 30

] dene a point that can also be described by


A [3, 30

] B [3, 30

] C [3, 150

] D [3, 150

] E [3, 150

]
2 The polar coordinates of point A are
A [2, 40

] B [2, 40

] C [2, 140

]
D [4, 40

] E [2, 140

]
A
40
2
Z O
3 The polar coordinates of the point with cartesian coordinates (1,

3) are
A
_
2,
4
3
_
B
_
2,

3
_
C
_
2,

3
_
D
_
2,
2
3
_
E
_
2,

3
_
4 The cartesian coordinates of the point with polar coordinates
_
3,

6
_
are
A (3,

3) B
_
3,
1

3
_
C
_
3
2
,
3

3
2
_
D
_
3

3
2
,
3
2
_
E
_
3
2
,

3
2
_
5 The polar equation of the circle with centre
_
3,

2
_
and radius 3 is
A r = 3 sin B r =

3 C r = 3 D r = 3 cos E r = 6 sin
P1: FXS/ABE P2: FXS
9780521740494c16.xml CUAU033-EVANS August 22, 2009 11:52
R
e
v
i
e
w
Chapter 16 Polar coordinates and complex numbers 437
6 The graph of r cos = 2 is
A
O
B
O
C
O
D
O
E
O
7 The polar equation of the circle with cartesian equation x
2
+ y
2
= 16 is
A r = 16 B r = 4 sin C r
2
+cos
2
= 4 D r = 4 cos E r = 4
8 If u = 1 +i , then
1
2 u
=
A
1
2

1
2
i B
1
5
+
2
5
i C
1
2
+
1
2
i D
1
2
+
1
5
i E 1 +5i
9 The point C on the Argand diagram represents the complex number z. Which point
represents the complex number i z?
A A B B C C D D E E
Re(z)
B
D E
A C
Im(z)
10 If |z| = 5 then

1
z

=
A
1

5
B
1

5
C
1
5
D
1
5
E

5
Short-answer questions (technology-free)
1 Graph each of the following.
a [3, ] b
_
2,

3
_
c [2, 210

] d
_
3,
11
6
_
2 Find the cartesian coordinates of the points in 1.
P1: FXS/ABE P2: FXS
9780521740494c16.xml CUAU033-EVANS August 22, 2009 11:52
R
e
v
i
e
w
438 Essential Advanced General Mathematics
3 Graph each of the following.
a {[r, ] : r = 3} b
_
[r, ] : =

3
_
c {[r, ] : r = 4} d
_
[r, ] : =
5
4
_
4 Express each of the following in polar form.
a (3, 3) b
_

3
2
,
1
2
_
c
_
5
2
,
5

3
2
_
d (4

2, 4

2)
5 Transform each of the following equations from cartesian to polar form.
a x
2
+ y
2
= 16 b x
2
+ y
2
= 9 c y
2
= 8x
d x
2
= 4y e x
2
+4y
2
= 64 f 2x y +2 = 0
6 Transform each of the following equations from polar to cartesian form.
a r = 5 b r = 3 sin c r
2
cos 2 = 9
d r(1 2 cos ) = 8 e r(2 cos ) = 7 f r(1 sin ) = 1
7 For z
1
= m +i n and z
2
= p +i q, express each of the following in the form a +i b.
a 2z
1
+3z
2
b z
2
c z
1
z
2
d
z
1
z
2
e z
1
+ z
1
f (z
1
+ z
2
)(z
1
z
2
)
g
1
z
1
h
z
2
z
1
i
3z
1
z
2
8 In the following, z = 1

3i . Express each in the form a +i b and mark each of the


following on an Argand diagram.
a z b z
2
c z
3
d
1
z
e z f
1
z
9 Write each of the following in polar form.
a 1 +i b 1

3i c 2

3 +i
d 3

2 +3

2i e 3

2 3

2i f

3 i
10 Write each of the following in cartesian form.
a 2 cis

3
b 3 cis

4
c 3 cis
3
4
d 3 cis
_
3
4
_
e 3 cis
_
5
6
_
f

2 cis
_

4
_
Extended-response questions
1 a Plot the graphs of r = 2 sin and r = 2 cos .
b Write the corresponding cartesian equation for each of these relations.
c Describe the curves you obtain from the polar equations r = 2a sin or r = 2a cos
where a is a non-zero constant.
P1: FXS/ABE P2: FXS
9780521740494c16.xml CUAU033-EVANS August 22, 2009 11:52
R
e
v
i
e
w
Chapter 16 Polar coordinates and complex numbers 439
2 Investigate each of the families of graphs dened by:
a r = a +b sin or r = a +b cos where a and b are non-zero constants
b r
2
= a
2
sin 2, r
2
= a
2
sin 2, r
2
= a
2
cos 2, r
2
= a
2
cos 2 where a is a positive
constant
c r = a
d r = a sin n and r = a cos n where a is a non-zero constant
3 a Find the exact solutions in C for the equation z
2
2

3z +4 = 0.
b i Plot the two solutions from a on an Argand diagram.
ii Find the equation of the circle, with centre the origin, which passes through these
two points.
iii Find the value of a Z such that the circle passes through (0, a)
4 Let z be a complex number with |z| = 6. Let A be the point representing z. Let B be the
point representing (1 +i )z.
Find
a |(1 +i )z| b |(1 +i )z z|
c Prove that OAB is an isosceles right-angled triangle.
5 Let z =
1

2
+
1

2
i
a On an Argand diagram O, A, Z, P, Q represent the complex numbers 0, 1, z, 1 + z and
1 z respectively. Show these points on a diagram.
b Prove that the magnitude of POQ =

2
. Find the ratio
|OP|
|OQ|
.
P1: FXS/ABE P2: FXS
9780521740494c17.xml CUAU033-EVANS October 5, 2008 8:1
C H A P T E R
17
Loci
Objectives
To find the cartesian equation of a locus, where each point P of the locus satisfies
the following properties:
r
that P is equidistant from two given points A and B, i.e., PA = PB
r
that P is a fixed distance from a given point A, i.e., there is a positive number k
such that PA = k
r
that PA = kPB, i.e., the distance from a fixed point A is k times its distance from
a fixed point B
r
that the sum of the distances from two points A and B is always a constant,
i.e. PA +PB = k
r
that the difference of the distances from two points A and B is always a
constant, i.e. PA PB = k
To sketch the graphs of circles, ellipses and hyperbolas
To consider the asymptotes of hyperbolas
17.1 Introduction and parabolas
Introduction
In order to nd the equation of a curve, some condition must be given which establishes which
points are on a curve. Up to now in this book, curves have been described through a
relationship between the x and y coordinates (and in Chapter 16, between the polar
coordinates). For example, y = 2x is the straight line through the origin with gradient 2 (and
for polar coordinates, r = 5 is the circle with center the origin and radius 5).
Many curves can also be described through a geometric description. For example, the set of
points equidistant from the points A(4, 0) and B(2, 0) lie on the line with equation x = 3.
A locus is a set of points which satisfy a condition. For the above example, the locus of the
points which are equidistant from A and B is the straight line with equation x = 3. Note that
every point which lies on the line x = 3 satises this condition. This is an important
observation which should be thought about with every locus problem.
440
P1: FXS/ABE P2: FXS
9780521740494c17.xml CUAU033-EVANS October 5, 2008 8:1
Chapter 17 Loci 441
Example 1
Find the equation of the locus of points P satisfying PA = PB, where A is the point with
coordinates (3, 0) and B is the point with coordinates (6, 4).
Solution
Let (x, y) be the coordinates of point P.
If PA = PB
Then

(x 3)
2
+(y 0)
2
=

(x 6)
2
+(y 4)
2
Squaring both sides and expanding
x
2
6x +9 + y
2
= x
2
12x +36 + y
2
8y +16
6x +9 = 12x 8y +52
8y +6x = 43
The locus is a straight line as shown.
Every point P on the line also satises the property that PA = PB.
x
y
43
8
43
(6, 4)
(3, 0)
6
O
Example 2
Find the equation of the locus of points P satisfying PA = 3, where A is the point with
coordinates (2, 1).
Solution
Let (x, y) be the coordinates of point P.
If PA = 3
Then

(x 2)
2
+(y 1)
2
= 3
Squaring both sides
(x 2)
2
+(y 1)
2
= 9
The locus is a circle with centre (2, 1) and radius 3.
Every point P on the circle satises the property that PA = 3.
Example 3
Find the equation of the locus of points P satisfying PO = 2PA, where A is the point with
coordinates (4, 0) and O is the origin.
Solution
Let (x, y) be the coordinates of point P.
If PO = 2PA
Then

x
2
+ y
2
= 2

(x 4)
2
+ y
2
P1: FXS/ABE P2: FXS
9780521740494c17.xml CUAU033-EVANS October 5, 2008 8:1
442 Essential Advanced General Mathematics
Squaring both sides
x
2
+ y
2
= 4[(x 4)
2
+ y
2
]
Expanding
x
2
+ y
2
= 4[x
2
8x +16 + y
2
]
x
2
+ y
2
= 4x
2
32x +64 +4y
2
0 = 3x
2
32x +64 +3y
2
0 = 3

x
2

32
3
x +
64
3

+3y
2
Completing the square
0 = 3

x
2

32
3
x +
256
9
+
64
3

256
9

+3y
2
0 = 3

x
16
3

2
+3y
2

64
3
64
9
=

x
16
3

2
+ y
2
The locus is a circle with centre

16
3
, 0

and radius
8
3
.
Every point P on this circle satises the property that PO = 2 PA.
Parabolas
Example 4
Find the equation of the locus of points P satisfying PM = PF, where F is the point with
coordinates (3, 0) and PM is the perpendicular distance from P to the line with equation
x = 3.
Solution
Let (x, y) be the coordinates of point P.
If PF = PM
Then

(x 3)
2
+ y
2
=

(x +3)
2
Squaring both sides
(x 3)
2
+ y
2
= (x +3)
2
Hence
x
2
6x +9 + y
2
= x
2
+6x +9
Therefore
y
2
= 12x
This is a parabola.
x
y
x = 3
O
M(3, y) P(x, y)
F(3, 0)
x = 3
x
y
P(x, y)
F(3, 0) O
M(3, y)
P1: FXS/ABE P2: FXS
9780521740494c17.xml CUAU033-EVANS October 5, 2008 8:1
Chapter 17 Loci 443
Exercise 17A
1 Sketch the locus of points P(x, y) for each of the following and hence write down its
cartesian equation.
a P is equidistant from the points A(3, 0) and B(6, 0)
Example 1
b P is equidistant from the points A(0, 8) and B(0, 12)
c P is always three units from the origin
Example 2
d A triangle OAP has vertices O(0, 0), A(4, 0) and P(x, y). The triangle has area
12 square units. Find the locus of P as it moves.
2 Find the locus of a point P(x, y) which moves so that it is equidistant from the origin and
the point (2, 5).
3 Find the locus of a point P(x, y) which moves so that it is equidistant from the points
(0, 6) and (2, 4).
4 Find the locus of a point P(x, y) which moves so that the sum of the squares of its
distances from the points (2, 0) and (2, 0) is 26 units.
5 A point P(x, y) moves so that its distance from the point K(2, 5) is twice its distance
from the line x = 1. Find its locus.
6 A point P moves so that its distance from the point (0, 20) is twice its distance from
B(4, 5). What is the locus of P?
Example 3
7 Find the locus of a point P(x, y) which moves so that it is equidistant from the points
(1, 2) and (2, 1).
8 A point P(x, y) moves so that its distance from the point K(4, 2) is twice its distance
from the origin. Find its locus.
9 Determine the locus of a point P which moves so that the difference of the squares of its
distances from the points A(4, 0) and B(4, 0) is 16.
10 Determine the locus of a point P which moves so that the square of its distance from the
origin is equal to the sum of its coordinates.
11 A(0, 0) and B(4, 0) are two of the vertices of a triangle ABP. The third vertex P is such
that PA : PB = 2. Find the locus of P.
12 Find the locus of the point P which moves so that it is always equidistant from two xed
points A(1, 2) and B(1, 0).
13 Given two xed points A(0, 1) and B(2, 5) nd the locus of P if the slope of AB equals
that of BP.
14 P moves so that its distance from the line y = 3 is always 2 units. Find the locus of P.
15 Find the equation of the locus of points P(x, y) which satisfy the property that the
Example 4
distance of P to the point F(2, 0) is equal to the distance PM, the perpendicular distance
to the line with equation x = 4. That is, PF = PM.
P1: FXS/ABE P2: FXS
9780521740494c17.xml CUAU033-EVANS October 5, 2008 8:1
444 Essential Advanced General Mathematics
16 Find the equation of the locus of points P(x, y) which satisfy the property that the
distance from P to the point F(0, 4) is equal to the distance PM, the perpendicular
distance to the line with equation y = 2. That is, PF = PM.
17 Describe the locus, in terms of equal distance from a line and a point, of a parabola with
equation y
2
= 3x.
17.2 Ellipses
The equation for an ellipse can be found in a similar way to those loci considered in
Section 17.1.
Example 5
Find the equation of the locus of points P satisfying PA +PB = 8, where A is the point with
coordinates (2, 0) and B is the point with coordinates (2, 0).
Solution
Let (x, y) be the coordinates of point P.
If PA +PB = 8
Then

(x +2)
2
+ y
2
+

(x 2)
2
+ y
2
= 8
Then

(x +2)
2
+ y
2
= 8

(x 2)
2
+ y
2
Squaring both sides
(x +2)
2
+ y
2
= 64 16

(x 2)
2
+ y
2
+(x 2)
2
+ y
2
Expanding and simplifying
x
2
+4x +4 + y
2
= 64 16

(x 2)
2
+ y
2
+ x
2
4x +4 + y
2
and x 8 = 2

(x 2)
2
+ y
2
Squaring both sides and expanding
x
2
16x +64 = 4(x
2
4x +4 + y
2
)
Simplifying yields
48 = 3x
2
+4y
2
or
x
2
16
+
y
2
12
= 1
This is an ellipse with centre the origin,
x axis intercepts 4 and 4 and y axis
intercepts 2

3 and 2

3.
Every point on the ellipse satises the property
that PA + PB = 8.
x
23

23

4 4 O
y
P1: FXS/ABE P2: FXS
9780521740494c17.xml CUAU033-EVANS October 5, 2008 8:1
Chapter 17 Loci 445
In general, an ellipse can be dened as the
locus of the point P so that, as it moves,
PA +PB = k for some k greater than the
distance between A and B. This is shown in
the diagram.
x
y
O
P
1
F
2
F
1
P
2
P
3
P
1
F
1
+ P
1
F
2
= P
2
F
1
+ P
2
F
2
= P
3
F
1
+ P
3
F
2
This can be pictured as a string of length P
1
F
1
+ P
1
F
2
being attached by nails to a board at
F
1
and F
2
and, considering the path mapped out by a pencil, extending the string so that it is
taut, and moving around the two points.
Example 6
Find the image of the circle x
2
+ y
2
= 1 under each of the following transformations.
a a dilation of factor 4 from the x axis followed by a dilation of factor 5 from the y axis
b a dilation of factor 4 from the x axis followed by a dilation of factor 5 from the y axis and
then a translation of 4 units in the positive direction of the x axis and 3 units in the negative
direction of the y axis
Solution
a The transformation is dened by the rule (x, y) (5x, 4y)
Therefore let x

= 5x and y

= 4y where (x

, y

) is the
image of (x, y) under the transformation.
Hence x =
x

5
and x =
y

4
. The image is
(x

)
2
25
+
(y

)
2
16
= 1. This is an ellipse with centre the
origin, x axis intercepts 5 and 5 and y axis intercepts 4 and 4.
x
y
4
5
4
5 O
b The transformation is dened by the rule
(x, y) (5x +4, 4y 3)
Therefore let x

= 5x +4 and y

= 4y 3 where
(x

, y

) is the image of (x, y) under the transformation.


Hence x =
x

4
5
and y =
y

+3
4
. The image is
(x

4)
2
25
+
(y

+3)
2
16
= 1. This an ellipse with
centre (4, 3).
x
y
4
4
(4, 3)
O
Example 7
Find the equation of the locus of points P(x, y) which satisfy the property that the distance of
P to the point F(1, 0) is half the distance PM, the perpendicular distance to the line with
equation x = 2. That is, PF =
1
2
PM.
P1: FXS/ABE P2: FXS
9780521740494c17.xml CUAU033-EVANS October 5, 2008 8:1
446 Essential Advanced General Mathematics
Solution
Let (x, y) be the coordinates of point P.
If PF =
1
2
PM

(x 1)
2
+ y
2
=
1
2

(x +2)
2
Squaring both sides
(x 1)
2
+ y
2
=
1
4
(x +2)
2
4(x
2
2x +1) +4y
2
= x
2
+4x +4
4x
2
8x +4 +4y
2
= x
2
+4x +4
3x
2
12x +4y
2
= 0
Completing the square
3[x
2
4x +4] +4y
2
= 12
3(x 2)
2
+4y
2
= 12 or equivalently
(x 2)
2
4
+
y
2
3
= 1
This is an ellipse with centre (2, 0).
x
y
x = 2
M(2, y)
P(x, y)
O F(1, 0)
It can be shown that the locus of points P(x, y) satisfying PF = ePM, where 0 < e < 1, F is a
xed point and PM is the perpendicular distance from P to a xed line l, is an ellipse. From the
symmetry of the ellipse it is clear that there is a second point F

and a second line l

such that
PF

= ePM

, where PM

is the perpendicular distance from P to l

, that denes the same


locus.
l
M
P
F F'
M'
l'
Exercise 17B
1 Sketch the graph of each of the following ellipses. Label axes intercepts.
a
x
2
9
+
y
2
64
= 1 b
x
2
25
+
y
2
100
= 1
c
y
2
9
+
x
2
64
= 1 d 25x
2
+9y
2
= 225
P1: FXS/ABE P2: FXS
9780521740494c17.xml CUAU033-EVANS October 5, 2008 8:1
Chapter 17 Loci 447
2 Sketch the graph of each of the following ellipses. State the centre and label the axes
intercepts.
a
(x 3)
2
9
+
(y 4)
2
64
= 1 b
(x +3)
2
9
+
(y +4)
2
25
= 1
c
(y 3)
2
16
+
(x 2)
2
4
= 1 d 25(x 5)
2
+9y
2
= 225
3 Sketch the graph of the image of the circle with equation x
2
+ y
2
= 1 transformed by a
dilation of factor 3 from the x axis and a dilation of factor 5 from the y axis. Give the
equation of this image.
4 Find the locus of the point P as it moves such that the sum of its distances from two xed
Example 5
points F(4, 0) and F

(4, 0) is 10 units.
5 Sketch the graph of the image of the circle with equation x
2
+ y
2
= 1 under the
transformation, dilation of factor 4 from the x axis and a dilation of factor 8 from the
y axis. Give the equation of this image.
6 Find the equation of the locus of points P(x, y) which satisfy the property that the distance
Example 7
of P to the point F(2, 0) is half the distance PM, the perpendicular distance to the line with
equation x = 4. That is, PF =
1
2
PM.
7 Find the equation of the locus of points P(x, y) which satisfy the property that the distance
of P to the point F(0, 8) is half the distance PM, the perpendicular distance to the line with
equation y = 4. That is, PF =
1
2
PM.
17.3 Hyperbolas
The curve with equation
x
2
a
2

y
2
b
2
= 1 is a hyperbola with centre at the origin. The axis
intercepts are (a, 0) and (a, 0). The hyperbola has asymptotes y =
b
a
x and y =
b
a
x. An
argument for this is as follows.
The equation
x
2
a
2

y
2
b
2
= 1
may be rearranged
x
2
a
2

y
2
b
2
= 1
y
2
b
2
=
x
2
a
2
1
y
2
=
b
2
x
2
a
2

1
a
2
x
2

But as x ,
a
2
x
2
0
x
y
(a, 0) (a, 0)
0
y =
a
b
x y =
a
b
x
y
2

b
2
x
2
a
2
i.e. y
bx
a
P1: FXS/ABE P2: FXS
9780521740494c17.xml CUAU033-EVANS October 5, 2008 8:1
448 Essential Advanced General Mathematics
The general equation for a hyperbola is formed by suitable translations.
The curve with equation
(x h)
2
a
2

(y k)
2
b
2
= 1
is a hyperbola with centre (h, k). The asymptotes are
y k =
b
a
(x h)
This hyperbola is obtained from the hyperbola with equation
x
2
a
2

y
2
b
2
= 1 by the
translation dened by (x, y) (x +h, y +k).
Example 8
For each of the following equations, sketch the graph of the corresponding hyperbola, give the
coordinates of the centre, the axes intercepts and the equations of the asymptotes.
a
x
2
9

y
2
4
= 1 b
y
2
9

x
2
4
= 1
c (x 1)
2
(y +2)
2
= 1 d
(y 1)
2
4

(x +2)
2
9
= 1
Solution
a
x
2
9

y
2
4
= 1
y
2
=
4x
2
9

1
9
x
2

Equations of asymptotes
y =
2
3
x
When y = 0, x
2
= 9 and therefore x = 3
x
y
y =
3
2
x
3
2
x
y =

(3, 0) (3, 0) 0
Axes intercepts (3, 0) and (3, 0), centre (0, 0)
b
y
2
9

x
2
4
= 1 is the reection of
x
2
9

y
2
4
= 1
in the line y = x
asymptotes are
x =
2
3
y
i.e. y =
3
2
x
The y axis intercepts are (0, 3) and (0, 3)
x
y
(0, 3)
0
(0, 3)
3
2
y = x
3
2
y = x
P1: FXS/ABE P2: FXS
9780521740494c17.xml CUAU033-EVANS October 5, 2008 8:1
Chapter 17 Loci 449
c (x 1)
2
(y +2)
2
= 1. The graph of
x
2
y
2
= 1 is sketched rst. The asymptotes
are y = x and y = x.
This hyperbola is called a rectangular
hyperbola as its asymptotes are perpendicular.
The centre is (0, 0) and the axes intercepts
are at (1, 0) and (1, 0).
x
y
(1, 0) (1, 0) 0
y =

x y = x
A translation of (x, y) (x +1, y 2) is
applied. The new centre is (1, 2) and the
asymptotes have equations y +2 = (x 1),
i.e., y = x 3 and y = x 1.
When x = 0, y = 2 and when y = 0,
(x 1)
2
= 5
x = 1

5
x
y
y
=
x 1
(0, 2) (2, 2)
3
1
1 3 0
y
=
x 3
(1, 2)
(1 5, 0) (1 + 5, 0)
d
(y 1)
2
4

(x +2)
2
9
= 1
This is obtained by translating the hyperbola
y
2
4

x
2
9
= 1 through the
translation dened by (x, y) (x 2, y +1)
x
y
2
3
y = x
(0, 2)
(0, 2)
0
2
3
y = x
x
2
9
= 1

y
2
4
(2, 1)
(2, 1)
(2, 3)
x
y
0
(x + 2)
2
9
= 1
( y 1)
2
4
y = x
3
2
3
1
y = x+
3
2
3
7
52
3
+ 1
52
3
1
Note: the asymptotes for
y
2
4

x
2
9
= 1 are the same as for those of the hyperbola
x
2
9

y
2
4
= 1. The two hyperbolae are called conjugate hyperbolae.
Example 9
Find the equation of the locus of points P(x, y) which satisfy the property that the distance of
P to the point F(1, 0) is twice the distance PM, the perpendicular distance to the line with
equation x = 2. That is, PF = 2PM.
P1: FXS/ABE P2: FXS
9780521740494c17.xml CUAU033-EVANS October 5, 2008 8:1
450 Essential Advanced General Mathematics
Solution
Let (x, y) be the coordinates of point P.
If PF = 2PM

(x 1)
2
+ y
2
= 2

(x +2)
2
Squaring both sides
(x 1)
2
+ y
2
= 4(x +2)
2
x
2
2x +1 + y
2
= 4(x
2
+4x +4)
x
2
2x +1 + y
2
= 4x
2
+16x +16
0 = 3x
2
+18x y
2
+15
Completing the square
x =

2
M(

2, y)
F(1, 0)
P(x, y)
O
x
y
0 = 3[x
2
+6x +9] y
2
+15 27
3(x +3)
2
y
2
= 12 or equivalently
(x +3)
2
4

y
2
12
= 1
This is a hyperbola with centre (3, 0)
x
y
y = 3(x + 3)
y = 3(x + 3)
5 3 1
O
It can be shown that the locus of points P(x, y) satisfying PF = ePM, where e > 1, F is a
xed point and PM is the perpendicular distance from P to a xed line l, is a hyperbola. From
the symmetry of the hyperbola it is clear that there is a second point F

and a second line l

such that PF

= ePM

, where PM

is the perpendicular distance from P to l

, that denes the


same locus.
F
P
M
l
M'
F'
l'
Hyperbolas may be dened in a manner similar to the methods discussed earlier in this
section for circles and ellipses.
Consider the set of all points, P, such that PF
1
PF
2
= k where k is a suitable constant and
F
1
and F
2
are points with coordinates (m, 0) and (m, 0) respectively. Then the equation of
the curve dened in this way is
x
2
a
2

y
2
m
2
a
2
= 1, k = 2a
P1: FXS/ABE P2: FXS
9780521740494c17.xml CUAU033-EVANS October 5, 2008 8:1
Chapter 17 Loci 451
Example 10
Find the equation of the locus of points P satisfying PA PB = 3 where A is the point with
coordinates (2, 0) and B is the point with coordinates (2, 0).
Solution
Let (x, y) be the coordinates of point P.
If PA PB = 3
Then

(x +2)
2
+ y
2

(x 2)
2
+ y
2
= 3
Then

(x +2)
2
+ y
2
= 3 +

(x 2)
2
+ y
2
Squaring both sides
(x +2)
2
+ y
2
= 9 +6

(x 2)
2
+ y
2
+(x 2)
2
+ y
2
Expanding and simplifying
x
2
+4x +4 + y
2
= 9 +6

(x 2)
2
+ y
2
+ x
2
4x +4 + y
2
and 8x 9 = 6

(x 2)
2
+ y
2
. Note that this only holds if x >
9
8
Squaring both sides
64x
2
144x +81 = 36[x
2
4x +4 + y
2
]
Simplifying yields
28x
2
36y
2
= 63
4x
2
9

4y
2
7
= 1 x
3
2
This is the right branch of a hyperbola with centre the origin, x axis intercept
3
2
.
The equations of the asymptotes are y =

7x
3
.
Exercise 17C
1 Sketch the graph of each of the following hyperbolas. Label axes intercepts and give the
Example 8
equation of the asymptotes.
a
x
2
9

y
2
64
= 1 b
x
2
25

y
2
100
= 1
c
y
2
9

x
2
64
= 1 d 25x
2
9y
2
= 225
P1: FXS/ABE P2: FXS
9780521740494c17.xml CUAU033-EVANS October 5, 2008 8:1
452 Essential Advanced General Mathematics
2 Sketch the graph of each of the following hyperbolas. State the centre and label axes
intercepts and asymptotes.
a
(x 3)
2
9

(y 4)
2
64
= 1 b
(x +3)
2
9

(y +4)
2
25
= 1
c
(y 3)
2
16

(x 2)
2
4
= 1 d 25(x 5)
2
9y
2
= 225
e x
2
y
2
= 4 f 2x
2
y
2
= 4
g x
2
4y
2
4x 8y 16 = 0 h 9x
2
25y
2
90x +150y = 225
3 Find the locus of the point P as it moves such that the difference of its distances from two
Example 10
xed points F(4, 0) and F

(4, 0) is 6 units.
4 Find the equation of the locus of points P(x, y) which satisfy the property that the distance
Example 9
of P to the point F(2, 0) is twice the distance PM, the perpendicular distance to the line
with equation x = 4. That is, PF = 2PM.
5 Find the equation of the locus of points P(x, y) which satisfy the property that the distance
of P to the point F(0, 8) is four times the distance PM, the perpendicular distance to the
line with equation y = 4. That is, PF = 4PM.
6 Find the equation of the locus of points P(x, y) satisfying PA PB = 4 where A is the
point with coordinates (3, 0) and B is the point with coordinates (3, 0).
P1: FXS/ABE P2: FXS
9780521740494c17.xml CUAU033-EVANS October 5, 2008 8:1
R
e
v
i
e
w
Chapter 17 Loci 453
Chapter summary
Lines
The general equation of a straight line may be written as ax +by = c.
For xed points A and B, the locus of P(x, y), as P moves such that PA = PB, is a straight
line.
Circles
The circle with centre the origin and radius a is the graph of the equation x
2
+ y
2
= a
2
.
The circle with centre (h, k) and radius a is the graph of the equation
(x h)
2
+(y k)
2
= a
2
. For a xed point A, the locus of P(x, y) as P moves such that
PA = k, where k > 0, is a circle.
Ellipses
The curve with equation
x
2
a
2
+
y
2
b
2
= 1 is an ellipse with centre the origin, x axis intercepts
(a, 0) and (a, 0), and y axis intercepts (0, b) and (0, b). For a > b, the ellipse will
appear as shown in the diagram to the left. If b > a, the ellipse is as shown in the diagram
to the right.
x
B'
B
O
b
A
a a
b
A'
y
x
B'
B
O
b
A
a a
b
A'
y
The curve with equation
(x h)
2
a
2
+
(y k)
2
b
2
= 1 is an ellipse with centre (h, k).
For xed points A and B, the locus of P(x, y) as P moves such that PA +PB = k, where k
is greater than the distance between A and B, is an ellipse.
Hyperbolas
The curve with equation
x
2
a
2

y
2
b
2
= 1 is a hyperbola
with centre the origin. The axis intercepts are (a, 0)
and (a, 0). The hyperbola has asymptotes
y =
b
a
x and y =
b
a
x.
The curve with equation
(x h)
2
a
2

(y k)
2
b
2
= 1
is a hyperbola with centre (h, k). The hyperbola
has asymptotes y k =
b
a
(x h) and y k =
b
a
(x h).
For xed points A and B, the locus of P(x, y) as P
moves such that |PA PB| = k, where k is a suitable constant, is a hyperbola.
x
y
0
(a, 0) (a, 0)
y = x
b
a y = x
b
a
P1: FXS/ABE P2: FXS
9780521740494c17.xml CUAU033-EVANS October 5, 2008 8:1
R
e
v
i
e
w
454 Essential Advanced General Mathematics
Multiple-choice questions
1 The equation of the ellipse shown is
A 5x
2
+ y
2
= 5
B 5x
2
+ y
2
= 25
C x
2
+5y
2
= 25
D x
2
+5y
2
= 5
E

x
5

2
+ y
2
= 1
x
y
0
1 1
1
1
2
3
2
3
4
2 3 4 5 6 2 3 4 5 6
2 The coordinates of the x axis intercepts of the graph of the ellipse with equation
x
2
25
+
y
2
9
= 1 are
A (5, 0) and (3, 0) B (3, 0) and (3, 0) C (0, 5) and (0, 5)
D (5, 0) and (5, 0) E (5, 0) and (3, 0)
3 The graph of the ellipse with equation

x
9

2
+

y
4

2
= 1 is
A
x
y
9
6
3
0
1 2 3 4 5 6 7 7 5 3 2 1 4 6
3
9
6
B
x
y
5
4
3
2
1
0
1 2 3 4 5 6 7 8 9 7 8 9 5
5
3
3
2
2
1
1
4
4
6
C
x
y
2 3
1
1 4
0
1
1 3 2
2
2
D
x
y
3 1
3
2
1
0
1
2
3
4
1 3 2 2
E
x
y
3 4
2
2
1
1
0
1
3 2 1
2
4
4
3
3
4 The coordinates of the y axis intercepts of the graph of the ellipse with equation

x
2
9

(y +2)
2
4

= 1 are
A (2, 0) and (2, 0) B (4, 0) and (4, 0) C (0, 4) and (0, 4)
D (0, 0) and (0, 4) E (3, 0) and (0, 2)
P1: FXS/ABE P2: FXS
9780521740494c17.xml CUAU033-EVANS October 5, 2008 8:1
R
e
v
i
e
w
Chapter 17 Loci 455
5 The graph of the equation ax
2
+by
2
= 8 has y axis intercept 2 and passes through the
point with coordinates

1,
1
2

10

. Then
A a = 2 and b = 3 B a = 4 and b = 3 C a =

3 and b = 2
D a = 3 and b = 2 E a = 2 and b = 2
6 The circle with equation (x a)
2
+(y b)
2
= 16 has its centre on the y axis and passes
through the point with coordinates (4, 4). Then
A a = 0 and b = 4 B a = 0 and b = 0 C a = 2 and b = 0
D a = 4 and b = 0 E a = 4 and b = 0
7 The circle with equation x
2
+ y
2
= 1 is transformed to an ellipse through the following
sequence of transformations:
r
dilation of factor 4 from the x axis
r
dilation of factor 3 from the y axis
r
translation of 4 units in the positive direction of the x axis
r
translation of 3 units in the positive direction of the y axis
The equation of the resulting ellipse is
A
(x 4)
2
16
+
(y 3)
2
9
= 1 B
(x 4)
2
9
+
(y 3)
2
16
= 1
C
(x +4)
2
3
+
(y +3)
2
4
= 1 D
(x 4)
2
36
+
(y 3)
2
16
= 1
E
(x +4)
2
20
+
(y +3)
2
48
= 1
8 The equation of the graph shown is
A
(x +2)
2
27

y
2
108
= 1
B
(x 2)
2
9

y
2
34
= 1
C
(x +2)
2
81

y
2
324
= 1
D
(x 2)
2
81

y
2
324
= 1
E
(x +2)
2
9

y
2
36
= 1
x
O
y
4
2
11
4
7
9 The locus of points P(x, y) which satisfy the property that PA = PB where A is the point
with coordinates (2, 5) and B is the point with coordinates (4, 1) is described by the
equation
A y = x 1 B y = x 6 C y = x 3
D y = x +1 E y = 3 x
10 The locus of points P(x, y) which satisfy the property that PA = 2PB where A is the point
with coordinates (2, 5) and B is the point with coordinates (4, 1) is
A a straight line B an ellipse C a circle
D a parabola E a hyperbola
P1: FXS/ABE P2: FXS
9780521740494c17.xml CUAU033-EVANS October 5, 2008 8:1
R
e
v
i
e
w
456 Essential Advanced General Mathematics
Short-answer questions (technology-free)
1 A circle has equation x
2
+4x + y
2
+8y = 0. Find the coordinates of the centre and radius
of the circle.
2 An ellipse has equation x
2
+4x +2y
2
= 0. Find the coordinates of the centre and the axes
intercepts of the ellipse.
3 Find the locus of the point P(x, y) such that PA = PB, where A is the point with coordinates
(0, 2) and B is the point with coordinates (6, 0).
4 Find the locus of the point P(x, y) such that PA = 6, where A is the point with coordinates
(3, 2).
5 State the equations of the asymptotes of the hyperbola with equation
x
2
9

y
2
4
= 1.
6 Find the locus of the point P(x, y) such that PA = 2PB, where A is the point with
coordinates (0, 2) and B is the point with coordinates (6, 0).
7 Sketch the graph of the ellipse with equation
(x 2)
2
9
+
y
2
4
= 1 and state its centre.
8 Determine the locus of a point P which moves so that the difference of the squares of its
distances from two xed points P
1
(4, 0) and P
2
(4, 0) is constant.
Extended-response questions
1 Let A, B and C be points with coordinates (6, 0), (6, 0) and (0, 6) respectively. Find the
locus of the points P which satisfy each of the following.
a PA = PC b PA = 6 c PA = 2PC
d PA = 2PB e PA =
1
2
PB f PA +PB = 20
g PA +PB = 12 h PA PB = 5 i PB PA = 5
2 Find the equation of the locus of points P(x, y) which satisfy the property that the distance
of P to the point F(0, 4) is
a equal to PM, the perpendicular distance to the line with equation y = 2
b half the distance PM, the perpendicular distance to the line with equation y = 2
c twice the distance PM, the perpendicular distance to the line with equation y = 2.
3 a The base of a triangle is xed and the distance from one end of the base to the midpoint
of the opposite side is a constant. Find the locus of the vertex joining the other two sides.
b The base of a triangle is xed and the ratio of the lengths of the other two sides is a
constant. Find the locus of the vertex joining the other two sides.
c Three vertices of a convex quadrilateral are xed. Find the locus of the fourth vertex if
the area of the quadrilateral is a constant.
P1: FXS/ABE P2: FXS
9780521740494c18.xml CUAU033-EVANS August 22, 2009 10:16
C H A P T E R
Revision
18
Revision of chapters
1417
18.1 Multiple-choice questions
1 ST is a tangent at T to the circle with center O.
If angle QOT = 150

then the magnitude of QTS is


A 70

B 75

C 95

D 105

E 150

Q
O
T
S
150
2 ML and MN are tangents to the circle at L and N.
The magnitude of angle LMN is
A 80

B 90

C 100

D 110

E 140

P
O
N
M
L
40
3 TS is a tangent at X and ZX bisects angle TXY.
Given these facts it can be proved that
A YZ = XT B YZ = XZ
C YZX = ZXT D SXY = ZXY
E TX
2
= XY.YZ
Z
Y
X
T
S
4 POQ is a diameter of the circle centre O. The size of angle QRS is
A 90

B 100

C 110

D 125

E 160

Q
R
O
P
S
70
457
P1: FXS/ABE P2: FXS
9780521740494c18.xml CUAU033-EVANS August 22, 2009 10:16
R
e
v
i
s
i
o
n
458 Essential Advanced General Mathematics
5 In the gure O is the centre of the circle and D is the midpoint of AB.
If AB = 8 cm, and CD = 2 cm, the radius of the circle is
A 3 cm B 4 cm C 5 cm
D 6 cm E 7 cm
A
D
O
B
C
6 In the gure, TA and TB are tangents to the circle.
If TA is perpendicular to TB and TA is perpendicular
to AC then the magnitude of BCA is
A 30

B 40

C 45

D 55

E 65

C
T
B
A
7 R, S and T are three points on the circumference of a circle,
with RST equal to 30

. The tangent to the circle at T meets


the line segment SR produced at P and RPT is equal to 40

.
RTS is equal to
A 70

B 80

C 90

D 100

E 110

40
30
P
R
S
T
8 If AB = AC, ADB = 60

and CAD = 50

then ABD is equal to


A 80

B 90

C 100

D 110

E 120

50
60
A
D
C
B
9 The unit vector in the direction of vector a = 3i 4 j is
A i j B
1
5
(3i 4j ) C i + j D
1
25
(3i 4 j ) E
3
5
i +
4
5
j
10 If

OA = 2i 4 j +k and

OB = 3i +4 j +k , then

AB equals
A 5i +2k B i 8 j C i +8 j +2k D i +8 j E i
11 If a = 2i +4 j and b = 3i 2 j , then a b equals
A 5i 6 j B i +6 j C 5i 2 j D 5i +2 j E i 6 j
12 The magnitude of vector a = 2i j +4k is
A

21 B 21 C 19 D

19 E 7
13 AB is parallel to OC, DC is parallel to OB,

OB = b,

OC = c, and AB = OB = OC = DC.

AD is equal to
A b +c B 2(c b) C 2(b c)
D 2b +2c E |b +c|
B C
A
O
D
b c
P1: FXS/ABE P2: FXS
9780521740494c18.xml CUAU033-EVANS August 22, 2009 10:16
R
e
v
i
s
i
o
n
Chapter 18 Revision of chapters 1417 459
14 If r = 2i j +k and s = i + j +3k, then 2r s equals
A 3i j +5k B 3i 3 j k C 5i j +5k
D 5i 3 j k E 6i 4 j 4k
15 PQR is a straight line and PQ = 2QR.

OQ = 2i 3 j and

OR = i +2 j .

OP is equal to
A 4i 13 j B 3i j C 2i 10 j
D 3i + j E i 5 j
R
Q
P
16 Let u = i +aj 5k and v = bi 3j +6k. Vectors u and v are parallel when
A a = 3 and b = 1 B a =
5
2
and b =
6
5
C a = 3 and b = 1
D a =
5
6
and b =
6
5
E a =
2
5
and b =
5
6
17 Given that a = 3i +4j , b = 2i j , x = i +5j and x = sa +tb, then the scalars s and t
are given by
A s = 1 and t = 1 B s = 1 and t = 1 C s = 1 and t = 1
D s = 1 and t = 1 E s = 5 and t = 5
18 In this diagram, OABC is a trapezium.

OA = a,

OC = c and

OA = 3

CB.

AB equals
A 3c B c
2
3
a C 3c 2a
D
2
3
a c E
4
3
a +c
C B
O
a
c
A
19 In this diagram,

OC = c,

OA = a,

OB = b and
AC : AB = 2 : 1. c is equal to
A a +2b B 3a 2b C 2a +b
D 2a b E 3a +b
C
c
O
b
a
A
B
20 If v, w and z are complex numbers such that v = 4 cis (0.3), w = 5 cis (0.6) and
z = vw, then Arg z is equal to
A 0.9 B 0.9 C 0.3 D 0.3 E 1.8
21 In cartesian form, 2 cis

2
3

is equal to
A

3 i B

3 +i C 1

3i D 1 +

3i E
1
3

3
2
i
22 If z =

3
2

1
2
i then Arg z is equal to
A
4
3
B
7
6
C

6
D
2
3
E
5
6
23 The imaginary part of the complex number 2 3i is
A 3 B 3i C 3 D 2 E 3i
24 If u = 3 cis

2
and v = 5 cis
2
3
then uv is equal to
A 15 cis

3
B 15 cis

2
3
C 15 cis

5
6

D 8 cis

2
3
E 8 cis
7
6
P1: FXS/ABE P2: FXS
9780521740494c18.xml CUAU033-EVANS August 22, 2009 10:16
R
e
v
i
s
i
o
n
460 Essential Advanced General Mathematics
25 The modulus of 12 5i is
A 169 B 7 C 13 D

119 E

7
26 Let z = x +i y, where x and y are real numbers which are not both zero. Which one of the
following expressions does not necessarily represent a real number?
A z
2
B zz C z
1
z D Im(z) E z + z
27 If z = 14 7i then the complex conjugate of z is equal to
A 7 14i B 14 +7i C 14 +7i D 14 7i E 7 +14i
28 The expression 3z
2
+9 is factorised over C. Which one of the following is a factor?
A 3z B z +3 C z +3i D z 3i E z +

3i
29 (1 +2i )
2
is equal to
A 3 B 3 +2i C 3 +4i D 1 +4i E 5 +4i
30 A point has polar coordinates

2,
4
3

. What are its cartesian coordinates, referring to the


same origin and with the polar axis in the direction of the positive x axis?
A (1,

3) B (1,

3) C (

3, 1) D (1,

3) E (

3, 1)
31 The point with cartesian coordinates (4

2, 4

2) has polar coordinates


A

2,

8,
3
4

2,
5
4

8,
7
4

8,

32 The polar equation of the circle with centre given by polar coordinates

2,

and radius
2 units is
A r = 2 sin B r = 2 cos C r = 4 sin D r = 4 cos E r = 2
33 Which one of the following graphs is the graph of r cos = 4?
A
O [4, 0] Z
B
O
[4, ]
Z
C
O [4, 0] Z
D
Z O
4,

2
[ ]
E
Z
O
[4, 0]
34 The circle with cartesian equation x
2
+ y
2
= 9 has polar equation
A r cos +r sin = 3 B r = 3 C r = 9 D r = 3 cos E r = 3 sin
35 The curve with polar equation r
2
= 1 r
2
sin
2
has cartesian equation
A x
2
+ y
2
+2y = 1 B x +2y = 1 C 2x
2
+ y
2
= 1
D x
2
+2y
2
= 1 E x +2y
2
= 1
P1: FXS/ABE P2: FXS
9780521740494c18.xml CUAU033-EVANS August 22, 2009 10:16
R
e
v
i
s
i
o
n
Chapter 18 Revision of chapters 1417 461
36 The graph of the relation r = 1 +cos is the cardioid shown.
Z O [2, 0]
Which one of the following graphs best represents the relation r =
1
1 +cos
?
A
Z O [2, ]
B
Z O
1
2
, 0
[ ]
C
Z O
1
2
, 0
[ ]
D
Z O
1
2
, 0
[ ]
E
Z O
1
2
, 0
[ ]
18.2 Extended-response questions
1 D is the midpoint of AC and E is a point on BC
such that BE : EC = 1 : t , where t > 0.
DE is produced to a point F such that
DE : EF = 1 : 7.
Let

AD = a and

AB = b.
a Express

AE in terms of t, a and b.
B
A
E
D
C
b Express

AE in terms of a and

AF. c Show that

AF =
9 7t
1 +t
a +
8t
1 +t
b
d If A, B and F are collinear, nd the value of t.
2 ABC is a triangle whose vertices have position
vectors a, b and c respectively, relative to an
origin in the plane ABC.
B
A C
a Show that an arbitrary point P on the segment
AB has position vector ma +nb where
m 0, n 0 and m +n = 1.
(Hint: Assume P divides AB in the ratio x : y.)
b Find

PC in terms of a, b and c.
c Let Q be an arbitrary point on line segment PC. Show that Q has position vector
a +b +c, where 0, 0, 0 and + + = 1.
3

OA = a,

OB = b,

OP =
4
5

OA and Q is the midpoint of AB.


a Express

AB and

PQ in terms of a and b.
b PQ is produced to meet OB produced at R so that

QR = n

PQ and

BR = kb. Express

QR in terms of
i n, a and b ii k, a and b
c Find the value of n and k.
P1: FXS/ABE P2: FXS
9780521740494c18.xml CUAU033-EVANS August 22, 2009 10:16
R
e
v
i
s
i
o
n
462 Essential Advanced General Mathematics
4 In the gure, O is the centre of a circle. TD and AC
are parallel. TA and TB are tangents to the circle.
Let BPT = x

.
a Prove that TBOA is a cyclic quadrilateral.
b Find BCA, BOA, TAB and TBA in terms of x.
B
T
A
O
C
D
P
x
5 a A man walks north at a rate of 4 km/h and notices that the wind appears to blow from
the west. He doubles his speed and now the wind appears to blow from the north-west.
What is the velocity of the wind? (Note: Direction and magnitude must be given.)
b A river 400 m wide ows from east to west at a steady speed of 1 km/h. A swimmer,
whose speed in still water is 2 km/h, starts from the south bank and heads north across
the river. Find the swimmers speed over the river bed and how far downstream he is
when he reaches the north bank.
c To a motorcyclist travelling due north at 50 km/h, the wind appears to come from the
north-west at 60 km/h. What is the true velocity of the wind?
d A dinghy in distress is 6 km on a bearing of 230

from a lifeboat and drifting in a


direction of 150

at 5 km/h. In what direction should the lifeboat travel to reach the


dinghy as quickly as possible if the maximum speed of the lifeboat is 35 km/h?
6 a Given that O, A, B and C are coplanar,

OA = a,

OB = b,

OC = c and A, B and C are


collinear with
c = a +b where , R,
show that + = 1.
b In the gure, G is the centroid of a triangle (i.e. the point where the lines joining each
vertex to the midpoint of the opposite side meet). A line passing through G meets ZX
and ZY at points H and K respectively, such that ZH = hZX and ZK = kZY.
i Prove that

ZG =
2
3

ZM
ii Express

ZG in terms of h, k,

ZHand

ZK
iii Find the value of
1
h
+
1
k
(use the result from a).
iv If h = k, nd the value of h and describe
geometrically what this implies.
v If the area of triangle XYZ is 1 cm
2
, what is
the area of triangle HZK when h = k?
H
X
M
G
K
Y
Z
vi If k = 2h, nd the value of h and describe geometrically what this implies.
vii Describe the restrictions on h and k and sketch the graph of h against k for suitable
values of k.
viii Investigate the area, A cm
2
, of triangle ZHK as a ratio with respect to the area of
triangle XYZ, as k varies. Sketch the graph of A against k. Be careful of the domain.
P1: FXS/ABE P2: FXS
9780521740494c19.xml CUAU033-EVANS August 22, 2009 10:52
C H A P T E R
19
Kinematics
Objectives
To model motion in a straight line and to use calculus to solve problems involving
motion in a line with constant and variable acceleration
To use graphical methods to solve problems involving motion in a straight line
Introduction
Kinematics is the study of motion without reference to the cause of the motion. In this chapter
we will consider the motion of a particle in a straight line only. Such motion is called
rectilinear motion. When referring to the motion of a particle we may in fact be referring to a
body of any size. However for the purposes of studying its motion we can consider that all
forces that act upon the body, causing it to move, act through a single point. Hence we can
consider the motion of a car or train in the same way as we would consider the motion of a
dimensionless particle.
It is important to make a distinction between vector and scalar quantities when studying
motion. Quantities such as displacement, velocity and acceleration must be specied by both
magnitude and direction. They are vector quantities. Distance, speed and time on the other
hand are specied by their magnitude only. They are scalar quantities.
Since we are considering movement in a straight line only, the direction of all vector
quantities is simply specied by the sign of the numerical value.
19.1 Position, velocity and acceleration
Position
The position coordinate of a particle moving in a straight line is determined by its distance
from a xed point O on the line, called the origin, and whether it is to the right or left of O.
Conventionally the direction to the right of the origin is considered to be positive.
O P
x
X
463
P1: FXS/ABE P2: FXS
9780521740494c19.xml CUAU033-EVANS August 22, 2009 10:52
464 Essential Advanced General Mathematics
Consider a particle which starts at O and begins to move. The position of a particle is
determined by a number, x, called the position coordinate. If the units are metres and if
x = 3, the position is 3 m to the left of O, while if x = 3, the displacement is 3 m to the
right of O.
The displacement is dened as the change in position of the particle relative to O.
Sometimes there is a rule which enables the position coordinate, at any instant, to be
calculated. In this case x is redened as a function of t. Hence x(t ) is the displacement at
time t. Specication of a displacement function together with the physical idealisation of
a real situation constitute a mathematical model of the situation.
An example of a mathematical model is the following.
A stone is dropped from the top of a vertical cliff 45 m high. Assume that the stone is a
particle travelling in a straight line. Let x(t ) be the downwards position of the particle from O,
the top of the cliff, t seconds after the particle is dropped. If air resistance is neglected, an
approximate model for the displacement is
x(t ) = 5t
2
for 0 t 3
It is important to distinguish between the scalar quantity distance and the vector quantity
displacement.
Consider a particle that starts at O and moves rstly ve units to the right to point P, and
then seven units to the left to point Q.
1 2 3 4 5 6 0 1 2 3 4
Q O P
Its nal position is x = 2. However the distance the particle has moved is 12 units.
Example 1
A particle moves in a straight line so that its position x cm relative to O at time t seconds is
given by x = t
2
7t +6, t 0. Find
a its initial position b its position at t = 4.
Solution
a At t = 0, x = +6 i.e. the particle is 6 cm to the right of O.
b At t = 4, x = (4)
2
7(4) +6 = 6 i.e. the particle is 6 cm to the left of O.
Velocity
You should already be familiar with the concept of a rate of change through your studies in
Mathematical Methods.
The velocity of a particle is dened as the rate of change of its position with respect to time.
We can consider the average rate of change, the change in position over a period of time, or
P1: FXS/ABE P2: FXS
9780521740494c19.xml CUAU033-EVANS August 22, 2009 10:52
Chapter 19 Kinematics 465
we can consider the instantaneous rate of change, which species the rate of change at a
given instant in time.
If a particle moves from x
1
at time t
1
to x
2
at time t
2
, then its
average velocity =
x
2
x
1
t
2
t
1
Velocity may be positive, negative or zero. If the velocity is positive the particle is moving to
the right and if it is negative the direction of motion is to the left. A velocity of zero means the
particle is instantaneously at rest.
The instantaneous rate of change of position with respect to time is the instantaneous
velocity. If the position, x, of the particle at time t is given as a function of t, then the velocity of
the particle at time t is determined by differentiating the rule for position with respect to time.
Common units of velocity (and speed) are:
1 metre per second = 1 m/s
1 centimetre per second = 1 cm/s
1 kilometre per hour = 1 km/h
The rst and third are connected in the following way:
1 km/h = 1000 m/h
=
1000
60 60
m/s
=
5
18
m/s
1 m/s =
18
5
km/h
Note the distinction between velocity and speed.
Speed is the magnitude of the velocity.
Average speed for a time interval [t
1
, t
2
] is equal to
distances travelled
t
2
t
1
Instantaneous velocity v =
dx
dt
where x is a function of time.
Example 2
A particle moves in a straight line so that its position x cm relative to O at time t seconds is
given by x = t
2
7t +6, t 0. Find
a its initial velocity b when and where its velocity equals zero
c its average velocity for the rst 4 s d its average speed for the rst 4 s.
Solution
a
x = t
2
7t +6
v =
dx
dt
= 2t 7
at t = 0, v = 7 i.e. the particle is moving to the left at 7 cm/s.
P1: FXS/ABE P2: FXS
9780521740494c19.xml CUAU033-EVANS August 22, 2009 10:52
466 Essential Advanced General Mathematics
b
2t 7 = 0
implies t = 3.5
When t = 3.5, x = (3.5)
2
7(3.5) +6
= 6.25
The particle is 6.25 cm to the left of O.
c average velocity =
change in position
change in time
at t = 4, x = 6
average velocity =
6 +6
4
= 3 cm/s
d average speed =
distance travelled
change in time
1 2 3 4 5 6 0 1 2 3 4 5 6
O
t = 4
t = 0
t = 3.5
1
4
6
Since the particle has stopped at t = 3.5 and begun to move in the opposite direction,
we must consider the distance travelled in the rst 3.5 s (from x = 6 to x = 6.25)
and then the distance travelled in the nal 0.5 s (from x = 6.25 to x = 6).
total distance travelled = 12.25 +0.25 = 12.5
average speed =
12.5
4
= 3.125 cm/s
Acceleration
The acceleration of a particle is dened as the rate of change of its velocity with respect to time.
Average acceleration for the time interval [t
1
, t
2
] is dened by
v
2
v
1
t
2
t
1
where v
2
is the
velocity at time t
2
and v
1
is the velocity at time t
1
.
Instantaneous acceleration a =
dv
dt
=
d
dt

dx
dt

=
d
2
x
dt
2
For kinematics, the second derivative
d
2
x
dt
2
is denoted by x

(t ) or x (t).
Acceleration may be positive, negative or zero. Zero acceleration means the particle is
moving at a constant velocity. Note that the direction of motion and the acceleration need not
coincide. For example, a particle may have a positive velocity indicating it is moving to the
right, but a negative acceleration indicating it is slowing down. Also, although a particle may
be instantaneously at rest its acceleration at that instant need not be zero. If acceleration has
the same sign as velocity then the particle is speeding up. If the sign is opposite the particle is
slowing down.
The most commonly used units for acceleration include cm/s
2
and m/s
2
.
P1: FXS/ABE P2: FXS
9780521740494c19.xml CUAU033-EVANS August 22, 2009 10:52
Chapter 19 Kinematics 467
Example 3
A particle moves in a straight line so that its position x cm relative to O at time t seconds is
given by x = t
3
6t
2
+5, t 0. Find
a its initial position, velocity and acceleration and hence describe its motion
b the times when it is instantaneously at rest and its position and acceleration at those times.
Solution
a
for x = t
3
6t
2
+5, v = 3t
2
12t and a = 6t 12
t = 0
x = 5, v = 0 and a = 12
Particle is instantaneously at rest 5 cm to right of O with an acceleration of
12 cm/s
2
.
b v = 3t
2
12t = 0
3t (t 4) = 0
t = 0 or t = 4
Particle is initially at rest and stops again after 4 s.
At t = 0, x = 5 and a = 12
At t = 4, x = (4)
3
6(4)
2
+5 = 27 and a = 6(4) 12 = 12
After 4 s the position of the particle is 27 cm to the left of O and its acceleration is
12 cm/s.
Exercise 19A
1 A particle moves in a straight line so that its position x cm relative to O at time t seconds
Examples 1, 2
(t 0) is given by x = t
2
7t +12. Find
a its initial position b its position at t = 5
c its initial velocity d when and where its velocity equals zero
e its average velocity in the rst 5 s f its average speed in the rst 5 s.
2 The position x metres at time t seconds (t 0) of a particle moving in a straight line is
Example 3
given by x = t
2
7t +10. Find
a when its velocity equals zero b its acceleration at this time
c the distance travelled in the rst 5 s d when and where its velocity is 2 m/s.
3 A particle moving in a straight line is x cm from the point O at time t seconds (t 0) where
x = t
3
11t
2
+24t 3. Find
a its initial position and velocity b its velocity at any time
c at what times the particle is stationary d where the particle is stationary
e for how long the particles velocity is negative f its acceleration at any time
g when the particles acceleration is zero and its velocity and position at that time.
P1: FXS/ABE P2: FXS
9780521740494c19.xml CUAU033-EVANS August 22, 2009 10:52
468 Essential Advanced General Mathematics
4 A particle moves in a straight line so that its position x cm relative to O at time t seconds
(t 0) is given by x = 2t
3
5t
2
+4t 5. Find
a when its velocity is zero and its acceleration at that time
b when its acceleration is zero and its velocity at that time.
5 A particle moving in a straight line is x cm from the point O at time t seconds (t 0) where
x = t
3
13t
2
+46t 48.
Find when it passes through O and its velocity and acceleration at those times.
6 Two particles are moving along a straight path so that their displacements, x cm from a
xed point P at any time, are given by x = t +2 and x = t
2
2t 2. Find
a the time when the particles are at the same position
b the time when they are moving with the same velocity.
19.2 Using antiderivatives for kinematics problems
So far we have considered examples where the equation of motion has dened the position of
the particle in terms of time and from it we have derived equations for the velocity and the
acceleration by differentiation.
We may be given a rule for acceleration at time t, and by the use of antidifferentiation with
respect to t and some additional information we can deduce rules for both velocity and
position.
Example 4
A body starts from O and moves in a straight line. After t seconds (t 0) its velocity (v cm/s)
is given by v = 2t 4. Find
a its position x in terms of t b its position after 3 s
c its average velocity in the rst 3 s d the distance travelled in the rst 3 s
e its average speed in the rst 3 s.
Solution
a Antidifferentiate with respect to t to nd the expression for position x m at time
t seconds
x = t
2
4t +c
When t = 0, x = 0 and therefore c = 0
x = t
2
4t
b When t = 3, x = 3. The body is 3 units to the left of O
c Average velocity =
3 0
3
= 1 m/s
P1: FXS/ABE P2: FXS
9780521740494c19.xml CUAU033-EVANS August 22, 2009 10:52
Chapter 19 Kinematics 469
d v = 0 when 2t 4 = 0, i.e., when t = 2
When t = 2, x = 4
Therefore the body goes from x = 0 to x = 4 in the rst 2 s, then back
to 3 in the next second. It has travelled 5 m in the rst 3 s.
e Its average speed is
5
3
m/s.
Example 5
A particle starts from rest 3 m from a xed point and moves in a straight line with an
acceleration of a = 6t +8. Find its position and velocity at any time t seconds.
Solution
a =
dv
dt
= 6t +8
by antidifferentiating v = 3t
2
+8t +c
at t = 0, v = 0 and so c = 0
v = 3t
2
+8t
by antidifferentiating again x = t
3
+4t
2
+d
at t = 0, x = 3 and so d = 3
x = t
3
+4t
2
+3
Example 6
A stone is projected vertically upward from the top of a building 20 m high with an initial
velocity of 15 m/s.
Find
a the time taken for the stone to reach its maximum height
b the maximum height reached by the stone
c the time taken for the stone to reach the ground
d the velocity of the stone as it hits the ground.
In this case we only consider the stones motion in a vertical direction so we can consider it as
rectilinear motion. Also we will assume that the acceleration due to gravity is approximately
10 m/s
2
(note that downward is considered the negative direction).
Solution
Given that a = 10
v = 10t +c
at t = 0, v = 15
v = 10t +15
x = 5t
2
+15t +d
at t = 0, x = 20
x = 5t
2
+15t +20
P1: FXS/ABE P2: FXS
9780521740494c19.xml CUAU033-EVANS August 22, 2009 10:52
470 Essential Advanced General Mathematics
a The stone will reach its maximum height when v = 0
10t +15 = 0
which implies t = 1.5
b
At t = 1.5, x = 5(1.5)
2
+15(1.5) +20
= 31.25
The maximum height reached by the stone is 31.25 m.
c The stone reaches the ground when x = 0
5t
2
+15t +20 = 0
5(t
2
3t 4) = 0
5(t 4)(t +1) = 0
t = 4 (solution of t = 1 is rejected since t 0)
i.e. the stone takes 4 s to reach the ground.
d At t = 4, v = 10(4) +15
= 25
i.e. velocity on impact is 25 m/s.
Exercise 19B
1 A body starts from O and moves in a straight line. After t seconds (t 0) its velocity
Example 4
(v cm/s) is given by v = 4t 6. Find
a its position x in terms of t b its position after 3 s
c the distance travelled in the rst 3 s d its average velocity in the rst 3 s
e its average speed in the rst 3 s.
2 The velocity (v m/s) at time t seconds (t 0) of a particle is given by v = 3t
2
8t +5. It
is initially 4 m to the right of a point O. Find
a its displacement and acceleration at any time
b its displacement when the velocity is zero
c its acceleration when the velocity is zero.
3 A body moves in a straight line with an acceleration of 10 m/s
2
. If after 2 s it passes
Example 6
through O and after 3 s it is 25 m from O, nd its initial displacement relative to O.
4 A body moves in a straight line so that its acceleration

a m/s
2

after time t seconds (t 0)


is given by a = 2t 3. If the initial position of the body is 2 m to the right of a point O and
its velocity is 3 m/s, nd the particles position and velocity after 10 s.
5 A body is projected vertically upwards with a velocity of 25 m/s. (Its acceleration due to
Example 5
gravity is 10 m/s
2
.) Find
a the particles velocity at any time
b its height above the point of projection at any time
P1: FXS/ABE P2: FXS
9780521740494c19.xml CUAU033-EVANS August 22, 2009 10:52
Chapter 19 Kinematics 471
c the time it takes to reach its maximum height
d the maximum height reached
e the time taken to return to the point of projection.
6 In a tall building the lift passes the 50th oor with a velocity of 8 m/s and an acceleration
of
1
9
(t 5) m/s
2
. If each oor spans a distance of 6 m, nd at which oor the lift
will stop.
19.3 Constant acceleration
When considering motion of a particle due to a constant force, e.g. gravity, the acceleration is
constant. There are a number of rules that we may establish by considering the case where
acceleration remains constant or uniform.
Given that
dv
dt
= a
by antidifferentiating we have
v = at +c where c is the initial velocity.
Using the symbol u for initial velocity we have
v = u +at 1
Now given that
dx
dt
= v
by antidifferentiating a second time we have
x = ut +
1
2
at
2
+d, where d is the initial position.
If we consider s = x d as the change in position of the particle from its starting point,
i.e. the particles displacement from its initial position, we have
v = ut +
1
2
at
2
2
If we transform the formula v = u +at so that t is the subject we have
t =
v u
a
By substitution in s = ut +
1
2
at
2
s =
u(v u)
a
+
a(v u)
2
2a
2
2as = 2u(v u) +(v u)
2
= 2uv 2u
2
+v
2
2uv +u
2
= v
2
u
2
i.e. v
2
= u
2
+2as 3
P1: FXS/ABE P2: FXS
9780521740494c19.xml CUAU033-EVANS August 22, 2009 10:52
472 Essential Advanced General Mathematics
Also we know that distance travelled = average velocity time.
i.e. s =
1
2
(u +v)t 4
These four formulas are very useful but it must be remembered that they only apply when
dealing with constant acceleration.
When approaching problems involving constant acceleration it is a good idea to list the
quantities you are given, establish which quantity or quantities you require and then use the
appropriate formula. Ensure that all quantities are converted to compatible units.
Constant acceleration summary
If acceleration is constant, the following formulas may be applied, where u is the initial
velocity, v is the nal velocity, a is the acceleration, t is the time and s is the displacement.
v = u +at s = ut +
1
2
at
2
v
2
= u
2
+2as s =
1
2
(u +v)t
Example 7
A body is moving in a straight line with uniform acceleration at an initial velocity of 12 m/s.
After 5 s its velocity is 20 m/s. Find
a the acceleration b the distance travelled in this time
c the time taken to travel a distance of 200 m.
Solution
Given u = 12
v = 20
t = 5
a
Find a using v = u +at
20 = 12 +5a
a = 1.6
The acceleration is 1.6 m/s
2
.
b Find s using s = ut +
1
2
at
2
= 12(5) +
1
2
(1.6)5
2
= 80
The distance travelled is 80 m.
c Using the formula s = ut +
1
2
at
2
gives
200 = 12t +
1
2
(1.6) t
2
200 = 12t +
4
5
t
2
1000 = 60t +4t
2
250 = 15t +t
2
i.e. t
2
+15t 250 = 0
(t 10)(t +25) = 0
t = 10 or t = 25
As t 0, t = 10 is the acceptable solution.
The body takes 10 s to travel a distance of 200 m.
P1: FXS/ABE P2: FXS
9780521740494c19.xml CUAU033-EVANS August 22, 2009 10:52
Chapter 19 Kinematics 473
Exercise 19C
1 How long does it take for a body at rest to travel a distance of 30 m if it is accelerated at
1.5 m/s
2
?
2 A car is travelling at 25 m/s when the brakes are applied. It is brought to rest with uniform
deceleration in 3 s. How far would it travel after the brakes were applied?
3 A motor cycle accelerates uniformly from 3 m/s to 30 m/s in 9 s. Find
Example 7
a the acceleration
b the time it will take to increase in speed from 30 m/s to 50 m/s
c the distance travelled in the rst 15 s (assuming it starts from rest)
d the time taken to reach a speed of 200 km/h (assuming it starts from rest).
4 A car accelerating uniformly from rest reaches a speed of 45 km/h in 5 s. Find
a its acceleration
b the distance travelled in the 5 s.
5 A train starts from rest at a station and accelerates uniformly at 0.5 m/s
2
until it reaches a
speed of 90 km/h.
a How long does the train take to reach this speed?
b How far does the train travel in reaching this speed?
6 A train travelling at 54 km/h begins to climb an incline of constant gradient that produces
a deceleration of 0.25 m/s
2
.
a How long will the train take to travel a distance of 250 m?
b What will the trains speed be then?
For 7 to 11 assume that the acceleration due to gravity is 9.8 m/s
2
and ignore air
resistance. Upward motion is considered to be in the positive direction.
7 A stone is projected vertically upwards from O with a speed of 20 m/s. Find
a the velocity of the stone after 4 s
b the distance of the stone from O after 4 s.
8 Repeat 7 for the stone being projected downwards from O with the same speed.
9 A body is projected vertically upwards with a velocity of 49 m/s.
a After what time will the body return to the point of projection?
b When will the body be at a height of 102.9 m above the point of projection?
10 A man dives from a springboard where his centre of gravity is initially 3 m above the
water and his initial velocity is 4.9 m/s upwards. Regarding the diver as a particle at his
centre of gravity, and assuming that the divers motion is vertical, nd
a the divers velocity after t seconds
P1: FXS/ABE P2: FXS
9780521740494c19.xml CUAU033-EVANS August 22, 2009 10:52
474 Essential Advanced General Mathematics
b the divers height above the water after t seconds
c the maximum height of the diver above the water
d the time taken for the diver to reach the water.
11 A stone is thrown vertically upwards from the top edge of a cliff 24.5 m high with a speed
of 19.6 m/s. Find
a the time taken for the stone to reach its maximum height
b the maximum height reached from the foot of the cliff
c the time taken for the stone to return to the point of projection
d the time taken for the stone to reach the foot of the cliff.
12 A body is travelling at 20 m/s when it passes point P and 40 m/s when it passes point Q.
Find its speed when it is halfway from P to Q, assuming uniform acceleration.
19.4 Velocitytime graphs
Many kinematics problems can be solved using velocitytime graphs. These are particularly
useful if acceleration is constant but with a broader knowledge of integral calculus they can
also be used when acceleration is variable.
First, we understand that if the acceleration is constant then v = u +at .
This constitutes a linear relationship between v and t where a is the gradient of the
corresponding velocitytime graph.
Since v =
dx
dt
it follows that

t
2
t
1
v(t )dt = x
2
x
1
where x
1
is the position at time t
1
and x
2
is the position at time t
2
.
Then the total area of the region(s) between
the velocitytime graph and the t axis
corresponds to the distance travelled by
the particle between times t
1
and t
2
.
area =
displacement
v
t
t
2
t
1
Consideration of the velocitytime graph
is particularly useful in situations where
there are several stages to the particles
motion.
Example 8
A car starts from rest and accelerates uniformly for 25 s until it is travelling at 25 m/s. It
then maintains this velocity for 3 minutes before decelerating uniformly until it stops in
another 15 s.
Construct a velocitytime graph and use it to determine the total distance travelled in
kilometres.
P1: FXS/ABE P2: FXS
9780521740494c19.xml CUAU033-EVANS August 22, 2009 10:52
Chapter 19 Kinematics 475
Solution
From the graph we can calculate
the area of the trapezium.
Area =
(a +b) h
2
=
1
2
(220 +180) 25
= 5000 m
= 5 km
The total distance travelled is 5 km.
25
25 205 220
O
t (s)
v (m/s)
Example 9
A motorist is travelling at a constant speed of 120 km/h when he passes a stationary police car.
He continues at that speed for another 15 s before uniformly decelerating to 100 km/h in 5 s.
The police car takes off after the motorist the instant it passes. It accelerates uniformly for 25 s
by which time it has reached 130 km/h. It continues at that speed until it catches up to the
motorist. After how long does the police car catch up to the motorist and how far has he
travelled in that time?
Solution
We start by representing the information
on a velocitytime graph.
130
100
20 15 25
120
t (s)
v (km/h)
police car
motorist
T O
The distance travelled by the motorist
and the police car will be the same so
the areas under each of the velocitytime
graphs will be equal. This fact can be
used to nd T, the time taken for the
police car to catch up to the motorist.
For the motorist, the distance travelled
after T seconds
=

120 15 +
1
2
(120 +100) 5 +100(T 20)

5
18
= (1800 +550 +100T 2000)
5
18
= (100T +350)
5
18
Note: The factor
5
18
changes velocities from km/h to m/s.
P1: FXS/ABE P2: FXS
9780521740494c19.xml CUAU033-EVANS August 22, 2009 10:52
476 Essential Advanced General Mathematics
Police car:

1
2
25 130 +130(T 25)

5
18
= (1625 +130T 3250)
5
18
= (130T 1625)
5
18
When the police car catches the motorist
100T +350 = 130T 1625
30T = 1975
T =
395
6
The police car catches the motorist after 65.83 s.
distance = (100T +350)
5
18
where T =
395
6
=
52 000
27
m
distance = 1.926 km
The police car has travelled 1.926 km when it catches the motorist.
Exercise 19D
It is suggested that you draw a velocitytime graph for each of these questions.
1 A particle starts from rest and accelerates uniformly for 5 s until it reaches a speed of
10 m/s. It immediately decelerates uniformly until it comes to rest after a further 8 s. How
far did it travel?
Example 8
2 A car accelerates uniformly from rest for 10 s to a speed of 15 m/s. It maintains this speed
for a further 25 s before decelerating uniformly to rest after a further 15 s. Find
a the total distance travelled by the car
b the distance it had travelled when it started to decelerate
c the time taken for it to reach the halfway point of its journey.
3 A particle starts from rest and travels 1 km before coming to rest again. For the rst 5 s it
accelerates uniformly. It next maintains a constant speed for 500 m, and then decelerates
uniformly for the last 10 s. Find the maximum speed of the particle.
4 A car passes point P with a speed of 36 km/h and continues at this speed for 12 s before
accelerating to a speed of 72 km/h in 6 s. How far from P is the car when it reaches a speed
of 72 km/h?
P1: FXS/ABE P2: FXS
9780521740494c19.xml CUAU033-EVANS August 22, 2009 10:52
Chapter 19 Kinematics 477
5 A tram decelerates uniformly from a speed of 60 km/h to rest in 60 s. Find
a the distance travelled by the tram
b how far it had travelled by the time it had reduced its speed by half
c the time taken for it to travel half the total distance.
6 A car passes a point A with a speed of 15 m/s and continues travelling at that speed. A
Example 9
second car starts from rest and accelerates uniformly until it reaches a speed of 25 m/s in
10 s. Both cars continue with a constant speed on to point B which they reach at the same
time.
a How long does it take for both cars to reach point B?
b How far is it from A to B?
7 Two stations A and B are 14 km apart. A train passes through station A, heading towards B,
maintaining a constant speed of 60 km/h. At the instant it passes through A, a second train
on the same track leaves station B, heading towards A, and accelerates uniformly. After 5
minutes the alarm is raised at both stations simultaneously that a collision is imminent.
Both trains are radioed and told to brake. The rst train decelerates uniformly so that it will
stop in 2.5 minutes. The second train, which has reached a speed of 80 km/h, will take 4
minutes to stop. Will they collide?
8 Two tram stops are 800 m apart. A tram starts at rest from the rst stop and accelerates at a
constant acceleration of a m/s
2
for a certain time and then decelerates at a constant rate of
2a m/s
2
, before coming to rest at the second stop. The time taken to travel between the
stops is 1 min 40 seconds. Find
a the maximum speed reached by the tram in km/h
b the time at which the brakes are applied
c the value of a.
P1: FXS/ABE P2: FXS
9780521740494c19.xml CUAU033-EVANS August 22, 2009 10:52
R
e
v
i
e
w
478 Essential Advanced General Mathematics
Chapter summary
The position coordinate of a particle moving in a straight line is determined by its distance
from a xed point O, called the origin, and whether the particle is to the right or left of O.
Conventionally, the direction to the right of the origin is considered to be positive.
r
displacement (x) is the position of the particle relative to O
r
velocity (v) is the rate of change of its position with respect to time, i.e. v =
dx
dt
r
speed is a scalar quantity and refers to the distance travelled per unit time
r
average velocity =
change in position
change in time
r
average speed =
distance travelled
change in time
r
acceleration (a) is the rate of change of its velocity with respect to time, i.e.
a =
dv
dt
=
d
2
x
dt
2
Constant acceleration
If acceleration is constant, the following formulae may be applied where u is the initial
velocity, v is the nal velocity, a is the acceleration, t is the time and s is the displacement
r
v = u +at
r
s = ut +
1
2
at
2
r
v
2
= u
2
+2as
r
s =
1
2
(u +v) t
Velocitytime graphs
The area of the region(s) between the velocitytime (v against t) graph and the t axis
between t = t
1
and t = t
2
corresponds to the distance travelled by the particle between
times t
1
and t
2
.
Multiple-choice questions
1 A particle moves in a straight line so that its position x cm from a xed point O at time t
seconds (t 0) is given by x = t
3
+7t
2
12t . The initial position of the particle
relative to O is
A 0 cm B 6 cm C 12 cm D 20 cm E 5 cm
2 A particle moves in a straight line so that its position x cm from a xed point O at time t
seconds (t 0) is given by x = t
3
+7t
2
12t . The average velocity of the particle in
the rst 2 s correct to two decimal places is
A 4 cm/s B 4 cm/s C 2 cm/s D 4.06 cm/s E 2 cm/s
3 A particle moves in a straight line with acceleration of 4 6t m/s
2
at time t seconds. The
particle has an initial velocity of 1 m/s and an initial position of 4 m from a xed point O.
The velocity of the particle when t = 1 is
A 1 m/s B 6 m/s C 0 m/s D 4 m/s E 2 m/s
P1: FXS/ABE P2: FXS
9780521740494c19.xml CUAU033-EVANS August 22, 2009 10:52
R
e
v
i
e
w
Chapter 19 Kinematics 479
4 A body starts from rest with a uniform acceleration of 1.8 m/s
2
. The time it will take for
the body to travel 90 m is
A 5 s B

10 s C 10 s D

10 E 10

2 s
5 A car accelerating uniformly from rest reaches a speed of 60 km/h in 4 s. The cars
acceleration is
A 15 km/h
2
B 15 m/s
2
C 54 m/s
2
D
25
6
km/h
2
E
25
6
m/s
2
6 A car accelerating uniformly from rest reaches a speed of 60 km/h in 4 s. The distance
travelled by the car in the 4 s is
A 200 m B 100 km C
100
3
m D 100 m E 360 m
7 A cars motion is represented by the
velocitytime graph shown.
10
25
20
15
15
10
t (s)
v (m/s)
5 6
0
5
4
The total distance travelled by the car
over the 15 s is
A 75 m B 315 m C 182.5 m
D 167.5 m E 375 m
8 A rock falls from the top of a cliff 40 m high. The rocks speed just before it hits the ground
in m/s

g = 9.8 m/s
2

is
A 20 B 22 C 24 D 26 E 28
9 A body initially travelling at 20 m/s is subject to a constant deceleration of 4 m/s
2
. The
time it takes to come to rest (t seconds) and the distance travelled before it comes to rest
(s metres) is given by
A t = 5, s = 50 B t = 5, s = 45 C t = 4, s = 20
D t = 5, s = 40 E t = 4, s = 35
10 A particle moves in a straight line with acceleration of 12t 5 m/s
2
at time t seconds. The
particle has an initial velocity of 1 m/s and an initial position of 0 m from a xed point O.
Find the velocity of the particle at t = 1.
A 1 m/s B 5 m/s C 7 m/s D 2 m/s E 3 m/s
Short-answer questions (technology-free)
1 A particle moves in a straight line so that its position x cm relative to O at time t seconds
(t 0) is given by x = t
2
4t 5. Find
a its initial position b its position at t = 3
c its initial velocity d when and where its velocity equals zero
e its average velocity in the rst 3 s f its average speed in the rst 3 s.
2 A particle moves in a straight line so that its position x cm relative to O at time t seconds
(t 0) is given by x = t
3
2t
2
+8. Find
P1: FXS/ABE P2: FXS
9780521740494c19.xml CUAU033-EVANS August 22, 2009 10:52
R
e
v
i
e
w
480 Essential Advanced General Mathematics
a its initial position, velocity and acceleration and hence describe its motion
b the times when it is stationary and its positions and acceleration at those times.
3 A particle moving in a straight line is x cm from the point O at time t seconds (t 0),
where x = 2t
3
+3t
2
+12t +7. Find
a when the particle passes through O and its velocity and its acceleration at those times
b when the particle is at rest
c the distance travelled in the rst 3 s.
4 Two particles A and B are moving in a straight line such that their displacements x cm from
the point O at time t seconds are given by x
1
and x
2
respectively, where
x
1
(t ) =t
3
t
2
t 0
x
2
(t ) =t
2
t 0
a Find
i the displacement of A after
1
2
s ii the acceleration of A after
1
2
s
iii the velocity of B after
1
2
s.
b Find
i the times when A and B collide (i.e., have the same displacement)
ii the maximum distance between A and B during the rst 2 s of motion.
5 A particle moving in a straight line has acceleration of 6t m/s
2
at time t seconds (t 0). If
the particle starts from rest at the origin O, nd
a the velocity after 2 s b the displacement at any time t.
6 A particle moving in a straight line has acceleration of (3 2t ) m/s
2
at time t seconds
(t 0). If the particle starts at the origin O with a velocity of 4 m/s, nd
a the time when the particle comes to rest
b the position of the particle at the instant it comes to rest
c the acceleration at this instant
d the time when the acceleration is zero
e the velocity at this time.
7 A particle moves in a straight line and, t seconds after it starts from O, its velocity is

2t
2
3t
3

m/s. Find
a the displacement after 1 s
b the velocity after 1 s
c the acceleration after 1 s.
8 For a particle moving in a straight line, the velocity function is v : R
+
R, where
v (t ) =
1
2t
2
and t is the time in seconds. Find
a the acceleration after t seconds
b the displacement at time t seconds, given that the particle is at O when t = 1.
9 The velocity, v m/s, of a body t seconds after it starts moving from O along a straight line is
given by v = t
3
11t
2
+24t, t 0.
P1: FXS/ABE P2: FXS
9780521740494c19.xml CUAU033-EVANS August 22, 2009 10:52
R
e
v
i
e
w
Chapter 19 Kinematics 481
a Find the acceleration at time t.
b Find the acceleration at the instant when the body rst changes direction.
c Find the displacement of the body from O after 5 s, and the total distance travelled in
the rst 5 s.
10 A car is travelling at 20 m/s when the brakes are applied. It is brought to rest with uniform
deceleration in 4 s. How far did it travel after the brakes were applied?
11 A car accelerates uniformly from 0 to 30 m/s in 12 s. Find
a the acceleration
b the time it will take to increase in speed from 30 m/s to 50 m/s
c the distance travelled in the rst 20 s
d the time taken to reach a speed of 100 km/h.
12 A train starts from rest at a station and accelerates uniformly at 0.4 m/s
2
until it reaches a
speed of 60 km/h.
a How long does the train take to reach this speed?
b How far does the train travel in reaching this speed?
For questions 13 and 14 assume that the acceleration due to gravity is 9.8 m/s
2
and ignore air
resistance. Upward motion is considered to be in the positive direction.
13 A body is projected vertically upward with a velocity of 35 m/s.
a After what time will the body return to the point of projection?
b When will the body be at a height of 60 m above the point of projection?
14 A stone is projected vertically upward from the top of a cliff 20 m high with a speed of
19.6 m/s. Find
a the time taken for the stone to reach its maximum height
b the maximum height reached, with respect to ground level
c the time taken for the stone to return to the point of projection
d the time taken for the stone to reach the foot of the cliff.
It is suggested that you draw a velocitytime graph for each of the questions 15 to 18.
15 A particle starts from rest and accelerates uniformly for 15 s until it reaches a speed of
25 m/s. It immediately decelerates uniformly until it comes to rest after a further 20 s. How
far did it travel?
16 A car accelerates uniformly from rest for 8 s to a speed of 12 m/s. It maintains this speed
for a further 15 s before decelerating uniformly to rest after a further 10 s. Find
a the total distance travelled by the car
b the time taken for it to reach the halfway point of its journey.
17 A vehicle starts from rest and travels 1 km before coming to rest again. For the rst 15 s it
accelerates uniformly, before maintaining a constant speed for 800 m then nally
decelerating uniformly to rest in 10 s. Find the maximum speed of the vehicle.
18 A car travels at a constant speed of 12 m/s along a straight road. It passes a second
stationary car which sets off in pursuit 3 s later. Find the constant acceleration required for
the second car so that it catches the rst car after a further 27 s has passed.
P1: FXS/ABE P2: FXS
9780521740494c19.xml CUAU033-EVANS August 22, 2009 10:52
R
e
v
i
e
w
482 Essential Advanced General Mathematics
19 A particle moves in a straight line so that t seconds after passing a xed point O in the line,
its velocity, v metres per second, is given by v =
t
2
4
3t +5.
Calculate:
a the velocity after 10 s b the acceleration when t = 0 c the minimum velocity
d the distance travelled in the rst 2 s e the distance travelled in the 3rd second.
20 A spot of light moves along a straight line so that its acceleration t seconds after passing a
xed point O on the line is (2 2t ) cm/s
2
. Three seconds after passing O the spot has a
velocity of 5 cm/s. Find, in terms of t, an expression for
a the velocity of the spot of light after t seconds
b the distance of the spot from O after t seconds.
21 A point P is moving along a straight line. It passes through a point O with a velocity 6 m/s
and, t seconds after passing through O, its acceleration is (4 4t ) m/s
2
.
a Show that, t seconds after passing through O, the velocity of P is

6 +4t 2t
2

m/s.
b Calculate
i the maximum velocity of P
ii the value of t when the velocity of P is again 6 m/s
iii the distance OP when the velocity of P is zero.
22 A particle travelling in a straight line passes a xed point O with a velocity 5 m/s. Its
acceleration, a m/s
2
, is given by a = 27 4t
2
, where t seconds is the time after passing O.
Calculate
a the acceleration of the particle as it passes O
b its velocity when t = 3
c the value of t when its velocity is again 5 m/s.
23 A particle passes a xed point O with a velocity of 2 m/s and moves in a straight line with
acceleration of 3 (1 t ) m/s
2
, where t is the time in seconds after passing O. Calculate
a the velocity when t = 4 b the position of the particle at this instant.
24 A particle P travels in a straight line from a xed point O so that its velocity, v m/s, is given
by v = t
2
10t +24, where t is the time in seconds after leaving O. Find
a the values of t for which P is instantaneously at rest
b the distance OP when t = 3
c the range of values of t for which the acceleration is negative.
Extended-response questions
1 A particle moves in a straight line so that its displacement x cm relative to O at time
t seconds is given by x =
1
3
t
3
2t
2
+4t 2
1
3
. Find
a its initial displacement b its initial velocity
c its acceleration after 3 s d the time when its velocity equals zero
P1: FXS/ABE P2: FXS
9780521740494c19.xml CUAU033-EVANS August 22, 2009 10:52
R
e
v
i
e
w
Chapter 19 Kinematics 483
e the displacement when its velocity equals zero
f the time when its displacement is zero.
2 A particle moves in a straight line so that its displacement, x cm, relative to O at time
t seconds (t 0) is given by x = t
4
+2t
2
8t . Show that
a the particle moves rst to the left
b the greatest distance of the particle to the left of O occurs after 1 s
c after this time, the particle always moves to the right.
3 A defective rocket rises vertically upwards into the air and then crashes back to the ground.
The rockets height above the ground, at time t seconds after take-off, is h metres where
h = 6t
2
t
3
(an approximate model).
a When does the rocket crash and what is its velocity at this time?
b At what time is the speed of the rocket zero, and what is its maximum height?
c When does the acceleration of the rocket become negative?
4 A body is projected vertically upwards at 20 m/s from the top of a tower 10 m high on the
edge of a vertical cliff. The upward displacement, x(t) metres, of the body from ground
level O at time t seconds after projection (t 0) is given by x (t ) = 4.9 t
2
+20t +10.
Use a graphics calculator or otherwise to evaluate the values
x (1) x (0) , x (2) x (1) , x (3) x (2) , . . . , x (10) x (9) .
Analyse your results and draw some inference about the motion of the body.
5 A particle is projected vertically upwards with a speed of u m/s. Prove that
a the time taken by the particle to reach its highest point is
u
g
seconds
b the total time taken for the particle to return to the point of projection is
2u
g
seconds
c the speed of returning to the point of projection is u m/s.
6 A stone is projected vertically upwards with a speed of 14 m/s, from a point O at the top of
a mine shaft. Five seconds earlier, a lift began to descend the mine shaft from O with a
constant speed of 3.5 m/s. Find the depth of the lift (to the nearest metre) at the instant
when the stone falls on it. (Neglect air resistance and take the acceleration due to gravity as
9.8 m/s
2
.)
7 A car is travelling along a straight road at 90 km/h when the brakes are applied. The car
comes to rest in 5 s and during this time the velocity decreases linearly with time. Find:
a the rule for the velocitytime function after the brakes are applied
b the distance travelled in the ve seconds.
8 A particle moves in a straight line so that its displacement x cm relative to O at time
t seconds (t 0) is given by x = 3t
4
4t
3
+24t
2
48t . Show that the particle moves at
rst to the left, comes to rest at a point A and then moves always to the right. Find the
position of A.
P1: FXS/ABE P2: FXS
9780521740494c19.xml CUAU033-EVANS August 22, 2009 10:52
R
e
v
i
e
w
484 Essential Advanced General Mathematics
9 A particle is projected vertically upwards with a velocity of u m/s from a point O on the
ground. T seconds later a second particle is projected vertically upwards from O with the
same velocity.
Prove that the time taken for the particles to collide is

u
g
+
T
2

seconds after the rst


particle was launched, and that the height of the particles at the instant they collide is
4u
2
g
2
T
2
8g
metres above O.
Interpret the case where T =
2u
g
. What happens if T >
2u
g
?
P1: FXS/ABE P2: FXS
9780521740494c20.xml CUAU033-EVANS October 5, 2008 8:10
C H A P T E R
20
Statics of a particle
Objectives
To be able to use a triangle of forces to solve problems
To be able to resolve forces, acting in a plane, in two directions which are at
right angles
Students should have completed some of the chapter on vectors, Chapter 15, before attempting
this chapter.
Introduction
A force is a measure of the strength of a push or pull. When considering the forces that act
upon a body, it is convenient to consider the forces acting on a single particle. The single
particle may be considered as a point at which the entire mass of the body is concentrated.
The unit of force used in this chapter is the kilogram weight (kg wt). If a body has a mass
of one kilogram then the force due to gravity acting on the body is one kilogram weight.
When a number of forces act simultaneously on a body, their combined effect is called the
resultant force. If the resultant force acting on a body is zero, the body will remain at rest or
continue moving with constant velocity. The body is said to be in equilibrium.
Every object near the surface of the Earth is subject to the force of gravity. We refer to this
force as the weight of the body. Weight is a force that acts vertically downwards on a body
(actually towards the centre of the Earth).
When an object such as a book is placed on a table it remains at rest. The book is obviously
being subjected to a force due to gravity but the fact that it does not fall to the ground indicates
that there must be a second force, equal in magnitude and opposite in direction, being applied
to the book. The table is exerting a force equal in magnitude to gravity on the book but in the
opposite direction. Hence it remains at rest. The book is said to be in equilibrium.
Any mass placed on a surface, either horizontal or inclined, experiences a force
perpendicular to the surface. This force is referred to as a normal force.
485
P1: FXS/ABE P2: FXS
9780521740494c20.xml CUAU033-EVANS October 5, 2008 8:10
486 Essential Advanced General Mathematics
N
W kg wt
N
W kg wt
Since a force is a quantity that is dened by both magnitude and direction, it may be
represented by a vector. Hence, if a particle under the action of a number of forces is in
equilibrium, it is clear that the vector sum of all forces acting must be zero.
20.1 Triangle of forces
If three forces are acting on a point in equilibrium, they can be represented by three vectors
forming a triangle.
If three forces F
1
, F
2
and F
3
act on a particle in equilibrium
as shown in the gure they can be rearranged into a triangle.
F
1
F
2
F
3
Since the particle is in equilibrium F
1
+ F
2
+ F
3
= 0
F
1
F
2
F
3
The magnitudes of the forces acting and the angles between the forces can be found using
trigonometric ratios (if the triangle contains a right angle) or using the sine or cosine rule.
In the following examples and exercises, strings and ropes are considered to have negligible
mass. A smooth light pulley is considered to have negligible mass and the friction between a
rope and pulley is considered to be negligible.
Example 1
A particle of mass 8 kg is suspended by two strings
attached to two points in the same horizontal plane.
If the two strings make angles of 30

and 40

to the
horizontal, nd the tension in each string.
8 kg wt
T
1
kg wt
T
2
kg wt
30 40
P1: FXS/ABE P2: FXS
9780521740494c20.xml CUAU033-EVANS October 5, 2008 8:10
Chapter 20 Statics of a particle 487
Solution
Representing the forces in a triangle and using the sine rule gives
T
1
sin 50

=
T
2
sin 60

=
8
sin 70

T
1
sin 50

=
8
sin 70

T
1
=
8
sin 70

sin 50

6.52 kg wt
T
2
=
8
sin 70

sin 60

7.37 kg wt
8 kg wt
T
1
kg wt
T
2
kg wt
70
60
50
Example 2
A particle of mass 15 kg is suspended vertically from a point
P by a string. The particle is pulled horizontally by a force of
F kg wt so that the string makes an angle of 30

with the
vertical. Find the value of F and the tension in the string.
T kg wt
15 kg wt
F kg wt
P
30
Solution
Representing the forces in a triangle gives
F
15
= tan 30

F = 15 tan 30

= 5

3
Also
15
T
= cos 30

T =
15
cos 30

= 10

3
The tension in the string is 10

3 kg wt.
T kg wt
15 kg wt
F kg wt
30
Example 3
A body weighing 20 kg is placed on a smooth inclined plane
which is inclined at 30

to the horizontal. A string is attached


to a point further up the plane which prevents the body from
moving. Find the tension in the string and the magnitude of
the force exerted on the body by the plane.
20 kg wt
F
30
N
P1: FXS/ABE P2: FXS
9780521740494c20.xml CUAU033-EVANS October 5, 2008 8:10
488 Essential Advanced General Mathematics
Solution
The triangle of forces appears as shown
150
90
120
20 kg wt
N kg wt
F kg wt
Rearrange into a triangle of forces as the body is in equilibrium
60
90
30
20 kg wt
N kg wt
F kg wt
Therefore F = 20 sin 30

= 10 kg wt and
N = 20 cos 30

= 10

3 kg wt
Exercise 20A
1 A particle of mass 5 kg is suspended by two strings
Example 1
attached to two points in the same horizontal plane.
If the two strings make angles of 45

with the horizontal,


nd the tension in each string.
5 kg wt
T
1
kg wt
T
2
kg wt
45 45
2 Using strings and pulleys, three weights of mass 6 kg,
8 kg and 10 kg are suspended in equilibrium as shown
in the diagram. Calculate the magnitude of the angle
ACB.
A B
C
6 kg
8 kg
10 kg
3 A weight of 20 kg is suspended from two strings of
length 10 cm and 12 cm, the ends of the strings being
attached to two points in a horizontal line, 15 cm apart.
Find the tension in each string.
20 kg wt
10 cm
T
1
T
2
15 cm
12 cm
4 If a boat is being pulled by two forces, of 40 kg wt towards the east and 30 kg wt to the
Example 2
north-west, what third force must be acting on the boat if it remains stationary? Give the
magnitude and direction.
P1: FXS/ABE P2: FXS
9780521740494c20.xml CUAU033-EVANS October 5, 2008 8:10
Chapter 20 Statics of a particle 489
5 A body weighing 104 kg is placed on a smooth,
Example 3
inclined plane which rises 5 cm vertically for every
12 cm horizontally. A string is attached to a point
further up the plane which prevents the body from
moving. Find the tension in the string and the
magnitude of the force exerted on the body by
the plane.
5
104 kg wt
12
N
T
6 A body weighing 12 kg is kept at rest on a smooth,
inclined plane of 30

by a force acting at an angle


of 20

to the plane. Find the magnitude of the force.


30
2
0

12 kg wt
N
F
7 In each of the following cases, determine whether the particle is in equilibrium.
a
6
10
8
160
80
b
120
150
4
2
23
8 Three forces of magnitude 4 kg wt, 7 kg wt and 10 kg wt are in equilibrium. Determine
the magnitude of the angles between the forces.
9 A mass of 15 kg is maintained at rest on a smooth, inclined plane by a string that is
parallel to the plane. Determine the tension in the string if:
a the plane is at 30

to the horizontal
b the plane is at 40

to the horizontal
c the plane is at 30

to the horizontal, but the string is held at an angle of 10

to the plane.
10 A string is connected to two points A and D in a horizontal line and weights of 12 kg and
W kg are attached at points B and C. If AB, BC and CD make angles of 40

, 20

and 50

respectively with the horizontal, calculate the tensions in the string and the weight W.
20.2 Resolution of forces
Obviously there are many situations where more than three forces (or in fact only two forces)
will be acting on a body. An alternative method is required to solve such problems.
P1: FXS/ABE P2: FXS
9780521740494c20.xml CUAU033-EVANS October 5, 2008 8:10
490 Essential Advanced General Mathematics
If all forces under consideration are acting in the same plane, then each of the forces and the
resultant force can be expressed as a sum of its i and j components. If a force F acts at an angle
of to the x axis, then F can be written as the sum of two components, one vertical and the
other horizontal.
So F = |F| cos i +|F| sin j
The force F is resolved into two components,
one parallel to the x axis (i component) and the
other parallel to the y axis (j component).
y
x

F
|F| cosi
|F| sinj
j
i
For a particle to be in equilibrium, the sum of the i components and the sum of the j
components are each zero.
Example 4
A particle of mass 8 kg is suspended by two
strings attached to two points in the same
horizontal plane. If the two strings make
angles of 30

and 60

to the horizontal, nd
the tension in each string.
8 kg wt
T
1
kg wt
T
2
kg wt
30 60
Solution
8
T
1
kg wt
T
2
kg wt
30
30
60
j
i
Resolution in the j direction
T
1
sin 30

+ T
2
sin 60

8 = 0
T
1

1
2

+ T
2

3
2

8 = 0 1
Resolution in the i direction
T
1
cos 30

+ T
2
cos 60

= 0
T
1

3
2

+ T
2

1
2

= 0 2
From 2

3 T
1
= T
2
P1: FXS/ABE P2: FXS
9780521740494c20.xml CUAU033-EVANS October 5, 2008 8:10
Chapter 20 Statics of a particle 491
Substituting in 1 gives
T
1

1
2

3 T
1

3
2

8 = 0
16 = 4T
1
T
1
= 4
T
2
= 4

3
The tensions in the strings are 4 kg wt and 4

3 kg wt.
Example 5
A body of mass 10 kg is held at rest on a smooth,
inclined plane by a string with tension 5 kg wt as
shown here. Find the angle between the string
and the inclined plane.
5 kg wt
10 kg wt

N
20
j
i
Solution
By resolving the forces parallel and perpendicular to the plane it can be seen that N
has no parallel component, since N is perpendicular to the plane.
Resolving in the i direction
5 cos

10 sin 20

= 0
cos

=
10 sin 20

5
= cos
1
(0.684)
= 46.84

Exercise 20B
For the following questions give answers correct to two decimal places.
1 A force F kg wt makes an angle of 40

with the
Example 3
horizontal. If its horizontal component is a force
of 10 kg wt, nd the magnitude of F.
40
F
2 Find the magnitude of a force, acting on a smooth,
Example 4
inclined plane of angle 35

, required to support a
mass of 10 kg resting on the plane.
10 kg wt
N F
35
P1: FXS/ABE P2: FXS
9780521740494c20.xml CUAU033-EVANS October 5, 2008 8:10
492 Essential Advanced General Mathematics
3 A body weighing 8 kg rests on a smooth, inclined
plane of angle 25

under the action of a horizontal


force. Find the magnitude of the force and the
reaction of the plane on the body.
8 kg wt
N
F
25
4 A body weighing 10 kg rests on a smooth, inclined
plane of angle 20

. Find the force that will keep it


in equilibrium when it acts at an angle of 54

with
the horizontal.
10 kg wt
20 54
N
F
5 If a body suspended by a string weighs 12 kg, nd the horizontal force required to hold it at
an angle of 30

from the vertical.


6 A force of 20 kg wt acting directly up a smooth plane inclined at an angle of 40

maintains
a body in equilibrium on the plane. Calculate the weight of the body and the pressure it
exerts on the plane.
7 Two men are supporting a block by ropes. One exerts a force of 20 kg wt, his rope making
an angle of 35

with the vertical, and the other exerts a force of 30 kg wt. Determine the
weight of the block and the angle of direction of the second rope.
8 A body A of weight 10 kg is supported
against a smooth plane of angle 50

.
Find the pressure of the body on the
plane and the tension in the string
which is parallel to the slope. A second
body B on a second plane of angle 40

is connected to A by a string passing


over a smooth pulley on the ridge. If
the system is in equilibrium, what is the weight of B?
N
40
10 kg wt
50
B
A
9 A sphere of radius 9 cm is attached to a point A on a
vertical wall by a string of length 15 cm. If the weight
of the sphere is 3 kg, nd the tension in the string.
A
15 cm
T
9 cm
3 kg
P1: FXS/ABE P2: FXS
9780521740494c20.xml CUAU033-EVANS October 5, 2008 8:10
R
e
v
i
e
w
Chapter 20 Statics of a particle 493
Chapter summary
When a number of forces act simultaneously on a body, their combined effect is called the
resultant force. If the resultant force acting on a body is zero, the body will remain at rest
or continue moving with constant velocity.
If three forces act upon a particle and the resultant force is zero, then vectors representing
the forces may be arranged to form a triangle. The magnitudes of the forces acting and the
angles between the forces can often be found using trigonometric ratios (if the triangle
contains a right angle) or using the sine or cosine rule.
Any mass placed on a surface, either horizontal or inclined, experiences a force
perpendicular to the surface called a normal force.
Resolution of forces
If all forces under consideration are
acting in two dimensions, it is possible
to express the forces in terms of their
components in the i direction and j
direction.
y
F
|F| cosi
|F| sinj
x
j

i
So F = |F| cos i +|F| sin j
For a particle to be in equilibrium, the sum of all i components must be zero and the sum of
all j components must be zero.
Multiple-choice questions
1 The magnitude of the component of force F in the i direction is
A 300 N B 50 N C 40 N D 20 N E 25 N
50 kg wt
60
i
Questions 2 and 3 refer to the following information.
A 20 kg weight is resting on a smooth plane inclined
at 30

to the horizontal and is prevented from slipping


down the plane by a string as shown in the diagram.
N
F
20 kg wt
30
2 The magnitude of N is
A 10 kg wt B 20 kg wt C

3
2
kg wt D 60 kg wt E 10

3 kg wt
3 The magnitude of the tension in the string is approximately
A 10 kg wt B 20 kg wt C

3
2
kg wt D 60 kg wt E 10 kg wt
P1: FXS/ABE P2: FXS
9780521740494c20.xml CUAU033-EVANS October 5, 2008 8:10
R
e
v
i
e
w
494 Essential Advanced General Mathematics
4 Two perpendicular forces have magnitudes 5 kg wt and 4 kg wt. The magnitude of the
resultant force is
A 3 kg wt B

11 kg wt C

41 kg wt D 1 kg wt E 9 kg wt
5 The diagram represents a particle in equilibrium acted on
by three forces in a plane of magnitudes A, B and C. Which one
of the following statements is not true?
A A = B cos 60

B A =
C cos 60

cos 30

C B = A cos 60

D B = A cos 60

+C cos 30

E C = B cos 30

A
C
150
90
B
6 A particle is acted on by a force of magnitude 7 kg wt acting on a bearing of 45

, and
another force of magnitude a kg wt acting on a bearing of 135

. If the magnitude of the


resultant force is 9 kg wt, the value of a must be
A 2 B 4

2 C

130 D 16 E 32
7 Two forces of magnitude 20 kg wt act on a particle at O as
shown. The magnitude of the resultant force in kg wt is
A 40 B 20

3 C 0
D 20 E 10
60
20N
20N
O
8 The resultant force when two forces of magnitude 300 kg wt and 200 kg wt act at an angle
of 60

to each other is
A 100

19 kg wt B 436 kg wt C 100 kg wt
D 350 kg wt E 500 kg wt
9 Two perpendicular forces have magnitudes 16 kg wt and 30 kg wt. The magnitude of the
resultant force is
A 50 kg wt B 10 kg wt C 34 kg wt
D 6

35 kg wt E 2 kg wt
10 A particle is acted on by a force of magnitude 8 kg wt acting on a bearing of 30

, and
another force of magnitude a kg wt acting on a bearing of 120

. If the magnitude of the


resultant force is 12 kg wt, the value of a must be
A 2 B 4

5 C

130 D 20 E 4

13
Short-answer questions (technology-free)
1 A mass of 15 kg is suspended from two strings of
length 6 cm and 8 cm, the ends of the strings being
attached to two points in a horizontal line, 10 cm
apart. Find the tension in each string.
10 cm
6 cm
8 cm
15 kg wt
P1: FXS/ABE P2: FXS
9780521740494c20.xml CUAU033-EVANS October 5, 2008 8:10
R
e
v
i
e
w
Chapter 20 Statics of a particle 495
2 An object is being pulled by two forces of 10 kg wt as
shown in the diagram. What is the magnitude and
direction of the third force acting on the object if
it remains stationary?
F
10 kg wt
60
10 kg wt
3 A body of mass 70 kg is placed on a smooth, inclined
plane which rises 6 cm vertically for every 12 cm
horizontally. A string is attached to a point further
up the plane which prevents the body from moving.
Find the tension in the string and the magnitude of
the force exerted on the body by the plane.
12 cm
70 kg wt
6 cm
4 A body of mass 15 kg is kept at rest on a smooth,
inclined plane of 30

by a force acting at an angle


of 30

to the plane. Find the magnitude of the force.


30
30
15 kg wt
F
5 Three forces of magnitude 5 kg wt, 8 kg wt and
12 kg wt are in equilibrium. Determine the
cosine of the angle between the 5 kg wt and 12 kg wt
forces.
5 kg wt 12 kg wt
8 kg wt
6 A force of F kg wt makes an angle of 30

with the vertical.


If its vertical component is a force of 20 kg wt, nd the
magnitude of F.
20 kg wt
30
F
7 Find the magnitude of the force, acting up a smooth,
inclined plane of angle 45

, required to support a
mass of 15 kg resting on the plane.
45
15 kg wt
F
8 A force of 14 kg wt acting directly up a smooth plane inclined at an angle of 30

maintains
a body in equilibrium on the plane. Calculate the weight of the body and the force it exerts
on the plane.
9 A body of mass 12 kg is kept at rest on a smooth incline of 30

by a horizontal force. Find


the magnitude of the force.
P1: FXS/ABE P2: FXS
9780521740494c21.xml CUAU033-EVANS October 5, 2008 8:12
C H A P T E R
Revision
21
Revision of chapters
19 and 20
21.1 Multiple-choice questions
Questions 1 to 9 refer to a particle that moves in a straight line so that its position x cm
relative to O at time t seconds (t 0) is given by x = 2t
2
5t 12.
1 The initial displacement of the particle is
A 12 cm B 0 cm C 15 cm D 12 cm E 3 cm
2 The initial velocity of the particle is
A 0 cm/s B 1 cm/s C 1 cm/s D 3 cm/s E 5 cm/s
3 The initial acceleration of the particle is
A 0 cm/s
2
B 4 cm/s
2
C 4 cm/s
2
D 3 cm/s
2
E 1 cm/s
2
4 The particle is stationary
A after 1 s B after 1.25 s C after 5 s D after 0.8 s E never
5 The particle passes through O at time t equals
A 1 s B 1 s C 4 s D 4 s E never
6 The particles displacement after 3 s is
A 9 cm B 0 cm C 9 cm D 13 cm E 21 cm
7 The average velocity of the particle over the rst 3 s is
A 1 cm/s B 1 cm/s C 7 cm/s D 7 cm/s E 0 cm/s
8 The distance travelled by the particle in the rst 3 s is
A 3 cm B 3 cm C 9 cm D 9.25 cm E 9 cm
496
P1: FXS/ABE P2: FXS
9780521740494c21.xml CUAU033-EVANS October 5, 2008 8:12
R
e
v
i
s
i
o
n
Chapter 21 Revision of chapters 19 and 20 497
9 The average speed of the particle over the rst 3 s is
A 1 cm/s B 1 cm/s C 3
1
12
cm/s D 3 cm/s E 3 cm/s
Questions 10 to 13 refer to a body projected upward from the ground with a velocity of 15 m/s.
Its acceleration due to gravity is 10 m/s
2
.
10 The bodys velocity at t = 3 s is
A 15 cm/s B 15 cm/s C 0 cm/s D 30 cm/s E 30 cm/s
11 The bodys velocity is 0 m/s at time t equals
A 1 s B 2 s C 0 s D 1.5 s E never
12 The maximum height reached by the body is
A 11 m B 15 m C 10 m D 11.25 m E 20 m
13 The body returns to the ground after
A 2 s B 4 s C 3 s D 1.5 s E never
Questions 14 and 15 refer to the following velocitytime
graph for a moving vehicle.
20
5 11
0
14
t (s)
v (m/s)
14 The distance travelled by the vehicle over the 14 s is
A 100 m B 150 m C 160 m D 180 m E 200 m
15 The acceleration of the vehicle over the rst 5 s is
A 20 m/s
2
B 10 m/s
2
C 2.5 m/s
2
D 4 m/s
2
E 4 m/s
2
Questions 16 and 17 refer to this system of
forces which is in equilibrium. F
1
F
2
8 kg wt
100
120
16 The magnitude of force F
1
is approximately
A 10.78 kg wt B 5.94 kg wt C 9.10 kg wt D 12.26 kg wt E 7.04 kg wt
17 The magnitude of force F
2
is approximately
A 10.78 kg wt B 5.94 kg wt C 9.10 kg wt D 12.26 kg wt E 7.04 kg wt
P1: FXS/ABE P2: FXS
9780521740494c21.xml CUAU033-EVANS October 5, 2008 8:12
R
e
v
i
s
i
o
n
498 Essential Advanced General Mathematics
Questions 18 and 19 refer to the following information. A 10 kg
weight is resting on a smooth plane inclined at 25

to the
horizontal and is prevented from slipping down the plane
by a string, as shown in the diagram.
N
F
25
10 kg wt
18 The magnitude of N is approximately
A 4.23 kg wt B 9.06 kg wt C 8.19 kg wt D 2.59 kg wt E 10 kg wt
19 The magnitude of the tension in the string is approximately
A 4.23 kg wt B 9.06 kg wt C 8.19 kg wt D 2.59 kg wt E 10 kg wt
20 If this system of forces is in equilibrium, is
approximately
A 120

B 136

C 102

D 110

E 100

14 kg wt
10 kg wt
12 kg wt

21 If this system of forces is in equilibrium, is


approximately
A 138

B 130

C 123

D 100

E 90

12 kg wt
15 kg wt
138
17.5 kg wt

22 The component of force F in the i direction is


A 34.64 kg wt B 30 kg wt C 40 kg wt
D 20 kg wt E 51.96 kg wt
|F| = 60 kg wt
30
F
i
j
23 The component of force F in the j direction is
A 34.64 kg wt B 30 kg wt C 40 kg wt
D 20 kg wt E 51.96 kg wt
|F| = 60 kg wt
30
F
i
j
21.2 Extended-response questions
1 The velocity, v m/s, of a particle moving in a straight line is given by
v = 6 + pt +qt
3
where t is the time in seconds after the particle passes through a xed point O.
Given that when t = 2 s the distance of the particle from O is 16 m and its acceleration is
32 m/s
2
, calculate
P1: FXS/ABE P2: FXS
9780521740494c21.xml CUAU033-EVANS October 5, 2008 8:12
R
e
v
i
s
i
o
n
Chapter 21 Revision of chapters 19 and 20 499
a the value of p and of q
b the velocity of the particle at the instant when the acceleration is zero.
2 A stone is projected vertically upwards from the top of a cliff 20 m high. After a time of 3 s
it passes the edge of the cliff on its way down. Calculate
a the speed of projection
b the speed when it hits the ground
c the times when it is 10 m above the top of the cliff
d the time when it is 5 m above the ground.
3 Trials are being undertaken on a horizontal road to test the performance of an electrically
powered car. The car has a top speed V. In a test run, the car moves from rest with uniform
acceleration a and is brought to rest with uniform retardation r.
a If the car is to achieve top speed during a test run, show that the length of the test run
must be at least
V
2
(a +r)
2ar
b Find the least time taken for a test run of length
i
2V
2
(a +r)
9ar
ii
2V
2
(a +r)
3ar
c Find, in terms of V, the average speed of the car for the test run described in b ii.
4 A particle X is projected vertically upwards from the ground with a velocity of 80 m/s.
Calculate the maximum height reached by X.
A particle Y is held at a height of 300 m above the ground. At the moment when X has
dropped 80 m from its maximum height, Y is projected downwards with a velocity of v m/s.
The particles reach the ground at the same time. Calculate the value of v.
5 a A particle X moves along a horizontal straight line so that its displacement, s m, from a
xed point O, t seconds after the motion has begun, is given by
s = 28 +4t 5t
2
t
3
.
Find expressions in terms of t for
i velocity ii acceleration.
b State
i the initial velocity of X
ii the initial acceleration of X.
c A second particle Y moves along the same horizontal straight line as X and starts from O
at the same instant that X begins to move. The initial velocity of Y is 2 m/s, and its
acceleration a m/s
2
, t seconds after motion has begun is given by a = 2 6t . Find the
value of t at the instant when X and Y collide.
d Find the velocity of X and the velocity of Y at this instant and comment on the direction
and motion of each of the particles.
P1: FXS/ABE P2: FXS
9780521740494c22.xml CUAU033-EVANS October 15, 2008 13:22
C H A P T E R
22
Describing the
distribution of a
single variable
Objectives
To introduce the two main types of datacategorical and numerical
To use bar charts to display frequency distributions of categorical data
To use histograms and frequency polygons to display frequency distributions of
numerical data
To use cumulative frequency polygons and cumulative relative frequency
polygons to display cumulative frequency distributions
To use the stem-and-leaf plot to display numerical data
To use the histogram to display numerical data
To use these plots to describe the distribution of a numerical variable in terms of
symmetry, centre, spread and outliers
To define and calculate the summary statistics mean, median, range, interquartile
range, variance and standard deviation
To understand the properties of these summary statistics and when each is
appropriate
To construct and interpret boxplots, and use them to compare data sets
22.1 Types of variables
A characteristic about which information is recorded is called a variable, because its value is
not always the same. Several types of variable can be identied. Consider the following
situations.
500
P1: FXS/ABE P2: FXS
9780521740494c22.xml CUAU033-EVANS October 15, 2008 13:22
Chapter 22 Describing the distribution of a single variable 501
Students answer a question by selecting yes, no or dont know.
Students say how they feel about a particular statement by ticking one of strongly agree,
agree, no opinion, disagree or strongly disagree.
Students write down the size shoe that they take.
Students write down their height.
These situations give rise to two different types of data. The data arising from the rst two
situations are called categorical data, because the data can only be classied by the name of
the category from which they come; there is no quantity associated with each category. The
data arising from the third and fourth examples is called numerical data. These examples
differ slightly from each other in the type of numerical data they each generate. Shoe sizes are
of the form . . . , 6, 6.5, 7, 7.5, . . . . These are called discrete data, because the data can only
take particular values. Discrete data often arise in situations where counting is involved. The
other type of numerical data is continuous data where the variable may take any value
(sometimes within a specied interval). Such data arise when students measure height. In fact,
continuous data often arise when measuring is involved.
Exercise 22A
1 Classify the data which arise from the following situations into categorical, or numerical.
a Kindergarten pupils bring along their favourite toy, and they are grouped together under
the headings: dolls, soft toys, games, cars, and other.
b The number of students on each of twenty school buses are counted.
c A group of people each write down their favourite colour.
d Each student in a class is weighed in kilograms.
e Each student in a class is weighed and then classied as light, average or heavy.
f People rate their enthusiasm for a certain rock group as low, medium, or high.
2 Classify the data which arise from the following situations as categorical or numerical.
a The intelligence quotient (IQ) of a group of students is measured using a test.
b A group of people are asked to indicate their attitude to capital punishment by selecting
a number from 1 to 5 where 1 = strongly disagree, 2 = disagree, 3 = undecided,
4 = agree, and 5 = strongly agree.
3 Classify the following numerical data as either discrete or continuous.
a The number of pages in a book.
b The price paid to ll the tank of a car with petrol.
c The volume of petrol used to ll the tank of a car.
d The time between the arrival of successive customers at an autobank teller.
e The number of tosses of a die required before a six is thrown.
P1: FXS/ABE P2: FXS
9780521740494c22.xml CUAU033-EVANS October 15, 2008 13:22
502 Essential Advanced General Mathematics
22.2 Displaying categorical datathe bar chart
Suppose a group of 130 students were asked to nominate their favourite kind of music under
the categories hard rock, oldies, classical, rap, country or other. The table shows the
data for the rst few students.
Students name Favourite music
Daniel hard rock
Karina classical
John country
Jodie hard rock
The table gives data for individual students. To consider the group as a whole the data
should be collected into a table called a frequency distribution by counting how many of each
of the different values of the variable have been observed.
Counting the number of students who responded to the question on favourite kinds of music
gave the following results in each category.
Hard rock Other Oldies Classical Rap Country
62 27 20 15 3 3
While a clear indication of the groups preferences can be seen from the table, a visual
display may be constructed to illustrate this. When the data are categorical, the appropriate
display is a bar chart. The categories are indicated on the horizontal axis and the
corresponding numbers in each category shown on the vertical axis.
20
30
60
70
50
40
10
0
N
u
m
b
e
r

o
f


s
t
u
d
e
n
t
s
Hard rock Other Oldies Classical Rap Country
Type of music
The order in which the categories are listed on the horizontal axis is not important, as no
order is inherent in the category labels. In this particular bar chart, the categories are listed in
decreasing order by number.
From the bar chart the music preferences for the group of students may be easily compared.
The value which occurs most frequently is called the mode of the variable. Here it can be seen
that the mode is hard rock.
P1: FXS/ABE P2: FXS
9780521740494c22.xml CUAU033-EVANS October 15, 2008 13:22
Chapter 22 Describing the distribution of a single variable 503
Exercise 22B
1 A group of students were asked to select their favourite type of fast food, with the
following results.
a Draw a bar chart for these data.
b Which is the most popular food type?
Food type Number of students
hamburgers 23
chicken 7
sh and chips 6
Chinese 7
pizza 18
other 8
2 The following responses were received to a
question regarding the return of capital punishment.
strongly agree 21
agree 11
dont know 42
disagree 53
strongly disagree 129
a Draw a bar chart for these data.
b How many respondents either agree or strongly
agree?
3 A video shop proprietor took note of the
type of lms borrowed during a particular day
with the following results.
a Construct a bar chart to illustrate these data.
b Which is the least popular lm type?
comedy 53
drama 89
horror 42
music 15
other 33
4 A survey of secondary school students preferred
ways of spending their leisure time at home gave the
following results.
a Construct a bar chart to illustrate these data.
b What is the most common leisure activity?
watch TV 42%
read 13%
listen to music 23%
watch a video 12%
phone friends 4%
other 6%
22.3 Displaying numerical datathe histogram
In previous studies you have been introduced to various ways of summarising and displaying
numerical data, including dotplots, stem-and-leaf plots, histograms and boxplots. Constructing
a histogram for discrete numerical data is demonstrated in Example 1.
P1: FXS/ABE P2: FXS
9780521740494c22.xml CUAU033-EVANS October 15, 2008 13:22
504 Essential Advanced General Mathematics
Example 1
The numbers of siblings reported by each student in Year 11 at a local school is as follows:
2 3 4 0 3 2 3 0 4 1 0 0 1 2 3
0 2 1 1 4 5 3 2 5 6 1 1 1 0 2
2 3 4 1 1 0 9 0 1 1 1 1 1 0 1
Construct a frequency distribution of the number of siblings.
Solution
To construct the frequency distribution count the numbers of students corresponding
to each of the numbers of siblings, as shown.
Number 0 1 2 3 4 5 6 7 8 9
Frequency 9 15 7 6 4 2 1 0 0 1
A histogram looks similar to a bar chart, but because the data are numeric there is a
natural order to the plot which may not occur with a bar chart. Usually for discrete data
the actual data values are located at the middle of the appropriate column, as shown.
0
0
5
15
10
1 2 3 4 5 6 7 8 9
F
r
e
q
u
e
n
c
y
Number of siblings
An alternative display for a frequency distribution is a frequency polygon. It is formed by
plotting the values in the frequency histogram with points, which are then joined by straight
lines. A frequency polygon for the data in Example 1 is shown by the red line in this diagram.
0
0
5
15
10
1 2 3 4 5 6 7 8 9
F
r
e
q
u
e
n
c
y
Number of siblings
When the range of responses is large it is usual to gather the data together into sub-groups
or class intervals. The number of data values corresponding to each class interval is called the
class frequency.
P1: FXS/ABE P2: FXS
9780521740494c22.xml CUAU033-EVANS October 15, 2008 13:22
Chapter 22 Describing the distribution of a single variable 505
Class intervals should be chosen according to the following principles:
Every data value should be in an interval
The intervals should not overlap
There should be no gaps between the intervals.
The choice of intervals can vary, but generally a division which results in about 5 to
15 groups is preferred. It is also usual to choose an interval width which is easy for the reader
to interpret, such as 10 units, 100 units, 1000 units etc (depending on the data). By convention,
the beginning of the interval is given the appropriate exact value, rather than the end. For
example, intervals of 049, 5099, 100149 would be preferred over the intervals 150,
51100, 101150 etc.
Example 2
A researcher asked a group of people to record how many cups of coffee they drank in a
particular week. Here are her results.
0 0 9 10 23 25 0 0 34 32 0 0 30 0 4
5 0 17 14 3 6 0 33 23 0 32 13 21 22 6
8 19 25 25 0 0 0 2 28 25 14 20 12 17 16
Construct a frequency distribution and hence a histogram of these data.
Solution
Because there are so many different results and they are spread over a wide range, the
data are summarised into class intervals.
As the minimum value is 0 and the
maximum is 34, intervals of width 5
would be appropriate, giving the
frequency distribution shown in the table.
Number of Frequency
cups of coffee
04 16
59 5
1014 5
1519 4
2024 5
2529 5
3034 5
The corresponding histogram
may then be drawn.
F
r
e
q
u
e
n
c
y
20
15
10
5
5
0
10 25 20 15 30 35
Number of cups of coffee
Example 2 was concerned with a discrete numerical variable. When constructing a frequency
distribution of continuous data, the data are again grouped, as shown in Example 3.
P1: FXS/ABE P2: FXS
9780521740494c22.xml CUAU033-EVANS October 15, 2008 13:22
506 Essential Advanced General Mathematics
Example 3
The following are the heights of the players in a basketball club, measured to the nearest
millimetre.
178.1 185.6 173.3 193.4 183.1 193.0 188.3 189.5 184.6 202.4 170.9
183.3 180.3 182.0 183.6 184.5 185.8 189.1 178.6 194.7 185.3 188.7
192.4 203.7 191.1 189.7 191.1 180.4 180.0 180.1 170.5 179.3 193.8
196.3 189.6 183.9 177.7 184.1 183.8 174.7 178.9
Construct a frequency distribution and hence a histogram of these data.
Solution
From the data it seems that intervals of width
5 will be suitable. All values of the variable
which are 170 or more, but less than 175,
have been included in the rst interval.
The second interval includes values from
175 to less than 180, and so on for the rest
of the table.
Player heights Frequency
170 4
175 5
180 13
185 9
190 7
195 1
200 2
The histogram of these
data is shown here.
F
r
e
q
u
e
n
c
y
Player heights
15
5
170 175 180 185 190 195 200 205
0
10
The interval in a frequency distribution which has the highest class frequency is
called the modal class. Here the modal class is 180.0184.9.
Using the TI-Nspire
The calculator can be used to construct a histogram for numerical data. This will be
illustrated using the basketball player height data from Example 3.
P1: FXS/ABE P2: FXS
9780521740494c22.xml CUAU033-EVANS October 15, 2008 13:22
Chapter 22 Describing the distribution of a single variable 507
The data is easiest entered in a Lists &
Spreadsheet application ( 3).
Firstly, use the up/down arrows ( ) to
name the rst column height.
Then enter each of the 41 numbers as
shown.
Open a Data & Statistics application (
5) to graph the data. At rst the data
displays as shown.
Specify the x variable by selecting Add X
Variable from the Plot Properties (b2
4) and selecting height. The data now
displays as shown.
(Note: It is also possible to use the NavPad
to move down below the x-axis and click to
add the x variable.)
Select Histogram from the Plot Type menu
(b13). The data now displays as
shown.
Select Bin Settings from the Histogram
Properties submenu of Plot Properties
menu (b222).
Let width = 5 and Alignment = 170.
Finally, select Zoom, Data from the
Window/Zoom menu (b52) to
display the data as shown.
P1: FXS/ABE P2: FXS
9780521740494c22.xml CUAU033-EVANS October 15, 2008 13:22
508 Essential Advanced General Mathematics
Using the Casio ClassPad
The calculator can be used to construct a histogram for numerical data. This will be
illustrated using the basketball player height data from Example 3.
In enter the data into list1, tapping EXE to enter and move down the column.
Tap SetGraph, Setting . . . and the tab for Graph 1, enter the settings shown and tap
SET.
Tap SetGraph, StatGraph1 and then tap the box
to tick and select the graph.
Tap to produce the graph selecting HStart
= 4 (the left bound of the histogram) and HStep =
4 (the desired interval width) when prompted. The
histogram is produced as shown.
With the graph window selected (bold border)
tap 6to adjust the viewing window for the
graph.
Tap Analysis, Trace and use the navigator key to
move from column to column and display the
count for that column.
P1: FXS/ABE P2: FXS
9780521740494c22.xml CUAU033-EVANS October 15, 2008 13:22
Chapter 22 Describing the distribution of a single variable 509
Relative and percentage frequencies
When frequencies are expressed as a proportion of the total number they are called relative
frequencies. By expressing the frequencies as relative frequencies more information is
obtained about the data set. Multiplying the relative frequencies by 100 readily converts them
to percentage frequencies, which are easier to interpret.
An example of the calculation of relative and percentage frequencies is shown in
Example 4.
Example 4
Construct a relative frequency distribution and a percentage frequency distribution for the
player height data.
Solution
Player Relative Percentage
heights (cm) Frequency frequency frequency
170 4
4
41
= 0.10 10%
175 5
5
41
= 0.12 12%
180 13
13
41
= 0.32 32%
185 9
9
41
= 0.22 22%
190 7
7
41
= 0.17 17%
195 1
1
41
= 0.02 2%
200 2
2
41
= 0.05 5%
From this table it can be
seen, for example, that nine
out of forty-one, or 22% of
players, have heights from
185 cm to less than 190 cm.
Both the relative frequency histogram and the percentage frequency histogram are identical to
the frequency histogramonly the vertical scale is changed. To construct either of these
histograms from a list of data use a graphics calculator to construct the frequency histogram,
and then convert the individual frequencies to either relative frequencies or percentage
frequencies one by one as required.
Cumulative frequency distribution
To answer questions concerning the number or proportion of the data values which are less
than a given value a cumulative frequency distribution, or a cumulative relative frequency
distribution can be constructed. In both a cumulative frequency distribution and a cumulative
relative frequency distribution, the number of observations in each class are accumulated from
low to high values of the variable.
P1: FXS/ABE P2: FXS
9780521740494c22.xml CUAU033-EVANS October 15, 2008 13:22
510 Essential Advanced General Mathematics
Example 5
Construct a cumulative frequency distribution and a cumulative relative frequency distribution
for the data in Example 4.
Solution
Player heights Cumulative Cumulative relative
(cm) Frequency frequency frequency
<170 0 0 0
<175 4 4 0.10
<180 5 9 0.22
<185 13 22 0.54
<190 9 31 0.76
<195 7 38 0.93
<200 1 39 0.95
<205 2 41 1.00
Each cumulative frequency was obtained by adding preceding values of the frequency.
In the same way the cumulative relative frequencies were obtained by adding
preceding relative frequencies. Thus it can be said that a proportion of 0.54, or 54%,
of players are less than 185 cm tall.
A graphical representation of a cumulative frequency
distribution is called a cumulative frequency
polygon and has a distinctive appearance, as it
always starts at zero and is non-decreasing.
C
u
m
u
l
a
t
i
v
e

f
r
e
q
u
e
n
c
y
Player heights
20
30
40
170 175 180 185 190 195 200 205
0
10
This graph shows, on the vertical axis, the
number of players shorter than any height
given on the horizontal axis. The cumulative
relative frequency distribution could also be
plotted as a cumulative relative frequency
polygon, which would differ from the cumulative
frequency polygon only in the scale on the vertical axis, which would run from 0 to 1.
Exercise 22C
1 The number of pets reported by each student in a class is given in the following table:
Example 1
2 3 4 0 3 2 3 0 4 1 0
0 2 1 1 4 5 3 2 5 6 1
Construct a frequency distribution of the numbers of pets reported by each student.
P1: FXS/ABE P2: FXS
9780521740494c22.xml CUAU033-EVANS October 15, 2008 13:22
Chapter 22 Describing the distribution of a single variable 511
2 The number of children in the family for each student in a class is shown in this histogram.
10
0
2 1 3 4 5 6 7 8 9 10
5
Size of family
N
u
m
b
e
r

o
f

s
t
u
d
e
n
t
s
a How many students are the only child in a family?
b What is the most common number of children in the family?
c How many students come from families with six or more children?
d How many students are there in the class?
3 The following histogram gives the scores on a general knowledge quiz for a class of Year
11 students.
10
0
20 30 40 50 60 70 80 90 10 100
5
Marks
N
u
m
b
e
r

o
f

s
t
u
d
e
n
t
s

a How many students scored from 1019 marks?
b How many students attempted the quiz? c What is the modal class?
d If a mark of 50 or more is designated as a pass, how many students passed the quiz?
4 The maximum temperatures for several capital cities around the world on a particular day,
in degrees Celsius, were:
17 26 36 32 17 12 32 2
16 15 18 25 30 23 33 33
17 23 28 36 45 17 19 37
31 19 25 22 24 29 32 38
a Use a class interval of 5 to construct a frequency distribution for these data.
Example 2
b Construct the corresponding relative frequency distribution.
Example 4
c Draw a histogram from the frequency distribution.
d What percentage of cities had a maximum temperature of less than 25

C?
P1: FXS/ABE P2: FXS
9780521740494c22.xml CUAU033-EVANS October 15, 2008 13:22
512 Essential Advanced General Mathematics
5 A student purchases 21 new text books from a school book supplier with the following
prices (in dollars).
21.65 14.95 12.80 7.95 32.50 23.99 23.99
7.80 3.50 7.99 42.98 18.50 19.95 3.20
8.90 17.15 4.55 21.95 7.60 5.99 14.50
a Draw a histogram of these data using appropriate class intervals.
Example 3
b What is the modal class?
c Construct a cumulative frequency distribution for these data and draw the cumulative
Example 5
frequency polygon.
6 A group of students were asked to draw a line which they estimated to be the same length
as a 30 cm ruler. The lines were then measured (in cm) with the following results.
30.3 30.9 31.2 32.3 31.3 30.7 32.8 31.0 33.3 30.7
32.2 30.1 31.6 32.1 31.4 31.8 32.9 31.9 29.4 31.6
32.1 31.2 30.7 32.1 30.8 29.7 30.1 28.9
a Construct a histogram of the frequency distribution.
b Construct a cumulative frequency distribution for these data and draw the cumulative
frequency polygon.
c Write a sentence to describe the students performance on this task.
7 The following are the marks obtained by a group of Year 11 Chemistry students on the end
of year exam.
21 49 58 68 72 31 49 59 68 72
33 52 59 68 82 47 52 59 70 91
47 52 63 71 92 48 53 65 71 99
a Using a graphics calculator, or otherwise, construct a histogram of the frequency
distribution.
b Construct a cumulative frequency distribution for these data and draw the cumulative
frequency polygon.
c Write a sentence to describe the students performance on this exam.
8 The following 50 values are the lengths (in metres) of some par 4 golf holes from
Melbourne golf courses.
302 272 311 351 338 325 314 307 336 310
371 334 369 334 320 374 364 353 366 260
376 332 338 320 321 364 317 362 310 280
366 361 299 321 361 312 305 408 245 279
398 407 337 371 266 354 331 409 385 260
a Construct a histogram of the frequency distribution.
b Construct a cumulative frequency distribution for these data and draw the cumulative
frequency polygon.
P1: FXS/ABE P2: FXS
9780521740494c22.xml CUAU033-EVANS October 15, 2008 13:22
Chapter 22 Describing the distribution of a single variable 513
c Use the cumulative frequency polygon to estimate:
i the proportion of par 4 holes below 300 m in length
ii the proportion of par 4 holes 360 m or more in length
iii the length which is exceeded by 90% of the par 4 holes.
22.4 Characteristics of distributions
of numerical variables
Distributions of numerical variables are characterised by their shapes and special features such
as centre and spread.
Two distributions are said to differ in centre if the values of the variable in one distribution
are generally larger than the values of the variable in the other distribution. Consider, for
example, the following histograms shown on the same scale.
a
0
5 10 15
b
0
5 10 15
It can be seen that plot b is identical to plot a but moved horizontally several units to the
right, indicating that these distributions differ in the location of their centres.
The next pair of histograms also differ, but not in the same way. While both histograms are
centred at about the same place, histogram d is more spread out. Two distributions are said to
differ in spread if the values of the variable in one distribution tend to be more spread out than
the values of the variable in the other distribution.
c
0
5 10 15
d
0
5 10 15
A distribution is said to be symmetric if it forms a mirror image of itself when folded in the
middle along a vertical axis; otherwise it is said to be skewed. Histogram e is perfectly
symmetrical, while f shows a distribution which is approximately symmetric.
e
0
5 10 15
f
0
5 10 15
P1: FXS/ABE P2: FXS
9780521740494c22.xml CUAU033-EVANS October 15, 2008 13:22
514 Essential Advanced General Mathematics
If a histogram has a short tail to the left and a long tail pointing to the right it is said to be
positively skewed (because of the many values towards the positive end of the distribution) as
shown in the histogram g.
If a histogram has a short tail to the right and a long tail pointing to the left it is said to be
negatively skewed (because of the many values towards the negative end of the distribution),
as shown in histogram h.
g
0
5 10 15
positively skewed
h
0
5 10 15
negatively skewed
Knowing whether a distribution is skewed or symmetric is important as this gives
considerable information concerning the choice of appropriate summary statistics, as will be
seen in the next section.
Exercise 22D
1 Do the following pairs of distributions differ in centre, spread, both or neither?
a
b
0 0
c
0 0
P1: FXS/ABE P2: FXS
9780521740494c22.xml CUAU033-EVANS October 15, 2008 13:22
Chapter 22 Describing the distribution of a single variable 515
2 Describe the shape of each of the following histograms.
a
0
b
0
c
0
3 What is the shape of the histogram drawn in 6, Exercise 22C?
4 What is the shape of the histogram drawn in 7, Exercise 22C?
5 What is the shape of the histogram drawn in 8, Exercise 22C?
22.5 Stem-and-leaf plots
An informative data display for a small (less than 50 values) numerical data set is the
stem-and-leaf plot. The construction of the stem-and-leaf plot is illustrated in Example 6.
Example 6
By the end of 2004 the number of test matches played, as captain, by each of the Australian
cricket captains was:
3 16 2 1 8 3 6 4 8 21 2 15 10 6
10 11 2 5 25 5 24 1 24 2 17 1 5 28
1 39 2 25 1 30 48 7 28 93 50 57 9 6
Construct a stem-and-leaf plot of these data.
P1: FXS/ABE P2: FXS
9780521740494c22.xml CUAU033-EVANS October 15, 2008 13:22
516 Essential Advanced General Mathematics
Solution
To make a stem-and-leaf plot nd the smallest and
the largest data values. From the table above, the
smallest value is 1, which is given a 0 in the tens
column, and the largest is 93, which has a 9 in the
tens column. This means that the stems are chosen
to be from 09. These are written in a column with
a vertical line to their right, as shown.
0
1
2
3
4
5
6
7
8
9
The units for each data point are then entered to the right of the dividing line. They are
entered initially in the order in which they appear in the data. When all data points are
entered in the table, the stem-and-leaf plot looks like this.
0 3 2 1 8 3 6 4 8 2 6 2 5 5 1 2 1 5 1 2 1 7 9 6
1 6 5 0 0 1 7
2 1 5 4 4 8 5 8
3 9 0
4 8
5 0 7
6
7
8
9 3
To complete the plot the leaves are ordered, and a key added to specify the place
value of the stem and the leaves.
0 1 1 1 1 1 2 2 2 2 2 3 3 4 5 5 5 6 6 6 7 8 8 9
1 0 0 1 5 6 7
2 1 4 4 5 5 8 8
3 0 9
4 8
5 0 7
6 3 | 9 indicates 39 matches
7
8
9 3
It can be seen from this plot that one captain has led Australia in many more test matches than
any other (Allan Border, who captained Australia in 93 test matches). When a value sits away
from the main body of the data it is called an outlier.
P1: FXS/ABE P2: FXS
9780521740494c22.xml CUAU033-EVANS October 15, 2008 13:22
Chapter 22 Describing the distribution of a single variable 517
Stem-and-leaf plots have the advantage of retaining all the information in the data set while
achieving a display not unlike that of a histogram (turned on its side). In addition, a
stem-and-leaf plot clearly shows:
the range of values
where the values are concentrated
the shape of the data set
whether there are any gaps in which no values are observed
any unusual values (outliers).
Grouping the leaves in tens is simplestother convenient groupings are in ves or twos, as
shown in Example 7.
Example 7
The birth weights, in kilograms, of the rst 30 babies born at a hospital in a selected month are
as follows.
2.9 2.7 3.5 3.6 2.8 3.6 3.7 3.6 3.6 2.9
3.7 3.6 3.2 2.9 3.2 2.5 2.6 3.8 3.0 4.2
2.8 3.5 3.3 3.1 3.0 4.2 3.2 2.4 4.3 3.2
Construct a stem-and-leaf plot of these data.
Solution
A stem-and-leaf plot of the birth weights, with the stem representing units and the
leaves representing one-tenth of a unit, may be constructed.
2 4 5 6 7 8 8 9 9 9
3 0 0 1 2 2 2 2 3 5 5 6 6 6 6 6 7 7 8
4 2 2 3 3 | 0 indicates 3.0 kilograms
The plot, which allows one row for each different stem, appears to be too compact.
These data may be better displayed by constructing a stem-and-leaf plot with two rows
for each stem. These rows correspond to the digits {0, 1, 2, 3, 4} in the rst row and
{5, 6, 7, 8, 9} in the second row.
2 4
2 5 6 7 8 8 9 9 9
3 0 0 1 2 2 2 2 3
3 5 5 6 6 6 6 6 7 7 8
4 2 2 3 3 | 0 indicates 3.0 kilograms
The only other possibility for a stem-and-leaf plot is one which has ve rows per
stem. These rows correspond to the digits {0, 1}, {2, 3}, {4, 5}, {6, 7} and {8, 9}.
P1: FXS/ABE P2: FXS
9780521740494c22.xml CUAU033-EVANS October 15, 2008 13:22
518 Essential Advanced General Mathematics
2 4 5
2 6 7
2 8 8 9 9 9
3 0 0 1
3 2 2 2 2 3
3 5 5
3 6 6 6 6 6 7 7
3 8
4 3 | 0 indicates 3.0 kilograms
4 2 2 3
None of the stem-and-leaf displays shown are correct or incorrect. A stem-and-leaf plot is
used to explore data and more than one may need to be constructed before the most
informative one is obtained. Again, from 5 to 15 rows is generally the most helpful, but this
may vary in individual cases.
When the data have too many digits for a convenient stem-and-leaf plot they should be
rounded or truncated. Truncating a number means simply dropping off the unwanted digits.
So, for example, a value of 149.99 would become 149 if truncated to three digits, but 150 if
rounded to three digits. Since the object of a stem-and-leaf display is to give a feeling for the
shape and patterns in the data set, the decision on whether to round or truncate is not very
important; however, generally when constructing a stem-and-leaf display the data is truncated,
as this is what commonly used data analysis computer packages will do.
Some of the most interesting investigations in statistics involve comparing two or more data
sets. Stem-and-leaf plots are useful displays for the comparison of two data sets, as shown in
the following example.
Example 8
The following table gives the number disposals by members of the Port Adelaide and Brisbane
football teams, in the 2004 AFL Grand Final.
Port Adelaide
25 20 19 18 18 17 16 15 14 13 12
12 11 11 11 11 10 10 9 9 7 7
Brisbane
25 19 19 18 17 16 15 15 13 13 13
10 10 9 9 8 8 7 6 5 4 0
Construct back to back stem-and-leaf plots of these data.
P1: FXS/ABE P2: FXS
9780521740494c22.xml CUAU033-EVANS October 15, 2008 13:22
Chapter 22 Describing the distribution of a single variable 519
Solution
To compare the two groups, the stem-and-leaf plots are drawn back to back, using two
rows per stem.
Port Adelaide Brisbane
0 0 4
9 9 7 7 0 5 6 7 8 8 9 9
4 3 2 2 1 1 1 1 0 0 1 0 0 3 3 3
9 8 8 7 6 5 1 5 5 6 7 8 9 9
0 2
5 2 5
0 | 2 represents 20 disposals 2 | 0 represents 20 disposals
The leaves on the left of the stem are centred slightly higher than the leaves on the
right, which suggests that, overall, Port Adelaide recorded more disposals. The spread
of disposals for Port Adelaide appears narrower than that of the Brisbane players.
Exercise 22E
1 The monthly rainfall for Melbourne, in a particular year, is given in the following table
Example 6
(in millimetres).
Month J F M A M J J A S O N D
Rainfall (mm) 48 57 52 57 58 49 49 50 59 67 60 59
a Construct a stem-and-leaf plot of the rainfall, using the following stems.
4
5
6
b In how many months is the rainfall 60 mm or more?
2 An investigator recorded the amount of time 24 similar batteries lasted in a toy. Her results
Example 7
in hours were:
25.5 39.7 29.9 23.6 26.9 31.3 21.4 27.4 19.5 29.8 33.4 21.8
4.2 25.6 16.9 18.9 46.0 33.8 36.8 27.5 25.1 31.3 41.2 32.9
a Make a stem-and-leaf plot of these times with two rows per stem.
b How many of the batteries lasted for more than 30 hours?
3 The amount of time (in minutes) that a class of students spent on homework on one
particular night was:
10 27 46 63 20 33 15 21 16 14 15
39 70 19 37 67 20 28 23 0 29 10
P1: FXS/ABE P2: FXS
9780521740494c22.xml CUAU033-EVANS October 15, 2008 13:22
520 Essential Advanced General Mathematics
a Make a stem-and-leaf plot of these times.
b How many students spent more than 60 minutes on homework?
c What is the shape of the distribution?
4 The cost of various brands of track shoes at a retail outlet are as follows.
$49.99 $75.49 $68.99 $164.99 $75.99 $39.99 $35.99 52.99
$210.00 $84.99 $36.98 $95.49 $28.99 $25.49 $78.99 $45.99
$46.99 $76.99 $82.99 $79.99 $149.99
a Construct a stem-and-leaf plot of these data.
b What is the shape of the distribution?
5 The students in a class were asked to write down the ages of their mothers and fathers.
Example 8
Mothers age
49 50 43 50 47 50 40 46 49 49 42 44 38
43 44 40 39 40 41 43 45 48 38 43 37 43
Fathers age
50 51 41 55 51 48 47 47 52 54 41 44 40
43 46 44 44 48 43 48 43 46 48 49 45 46
a Construct a back to back stem-and-leaf plot of these data sets.
b How do the ages of the students mothers and fathers compare in terms of shape, centre
and spread?
6 The results of a mathematics test for two different classes of students are given in the table.
Class A
22 19 48 39 68 47 58 77 76 89 85 82
85 79 45 82 81 80 91 99 55 65 79 71
Class B
12 13 80 81 83 98 70 70 71 72 72 73
74 76 80 81 82 84 84 88 69 73 88 91
a Construct a back to back stem-and-leaf plot to compare the data sets.
b How many students in each class scored less than 50%?
c Which class do you think performed better overall on the test? Give reasons for your
answer.
22.6 Summarising data
A statistic is a number that can be computed from data. Certain special statistics are called
summary statistics, because they numerically summarise special features of the data set under
consideration. Of course, whenever any set of numbers is summarised into just one or two
gures much information is lost, but if the summary statistics are well chosen they will also
help to reveal the message which may be hidden in the data set.
Summary statistics are generally either measures of centre or measures of spread. There
are many different examples for each of these measures and there are situations when one of
the measures is more appropriate than another.
P1: FXS/ABE P2: FXS
9780521740494c22.xml CUAU033-EVANS October 15, 2008 13:22
Chapter 22 Describing the distribution of a single variable 521
Measures of centre
Mean
The most commonly used measure of centre of a distribution of a numerical variable is the
mean. This is calculated by summing all the data values and dividing by the number of values
in the data set.
Example 9
The following data set shows the number of premierships won by each of the current AFL
teams, up until the end of 2004. Find the mean of the number of premiership wins.
Team Premierships
Carlton 16
Essendon 16
Collingwood 14
Melbourne 12
Fitzroy/Lions 11
Richmond 10
Hawthorn 9
Geelong 6
Kangaroos 4
Sydney 3
West Coast 2
Adelaide 2
Port Adelaide 1
W Bulldogs 1
St Kilda 1
Fremantle 0
Solution
mean =
16 +16 +14 +12 +11 +10 +9 +6 +4 +3 +2 +2 +1 +1 +1 +0
16
= 6.8
The mean of a sample is always denoted by the symbol x, which is called x bar.
In general, if n observations are denoted by x
1
, x
2
, . . . ., x
n
the mean is
x =
x
1
+ x
2
+ + x
n
n
or, in a more compact version
x =
1
n
n

i =1
x
i
where the symbol

is the upper case Greek sigma, which in mathematics means the sum
of the terms.
P1: FXS/ABE P2: FXS
9780521740494c22.xml CUAU033-EVANS October 15, 2008 13:22
522 Essential Advanced General Mathematics
Note: The subscripts on the xs are used to identify all of the n different values of x. They do not
mean that the xs have to be written in any special order. The values of x in the example are in
order only because they were listed in that way in the table.
Median
Another useful measure of the centre of a distribution of a numerical variable is the middle
value, or median. To nd the value of the median, all the observations are listed in order and
the middle one is the median.
The median of
median
2 3 4 5 5 6 7 7 8 8 11
is 6, as there are ve observations on either side of this value when the data are listed in order.
Example 10
Find the median number of premierships in the AFL ladder using the data in Example 9.
Solution
As the data are already given in order, it only remains to decide which is the middle
observation.
0 1 1 1 2 2 3 4 6 9 10 11 12 14 16 16
Since there are 16 entries in the table there is no actual middle observation, so the
median is chosen as the value half way between the two middle observations, in this
case the eighth and ninth (6 and 4). Thus the median is equal to
1
2
(6 + 4) = 5. The
interpretation here is that of the teams currently playing in the AFL, half (or 50%)
have won the premiership 5 or more times and half (or 50%) have have won the
premiership 5 or less times.
In general, to compute the median of a distribution:
Arrange all the observations in ascending order according to size.
If n, the number of observations, is odd, then the median is the
_
n + 1
2
_
th
observation from the end of the list.
If n, the number of observations, is even, then the median is found by averaging the
two middle observations in the list. That is, to nd the median the
nth
2
and the
_
n
2
+ 1
_
th
observations are added together, and divided by 2.
The median value is easily determined from a stem-and-leaf plot by counting to the required
observation or observations from either end.
P1: FXS/ABE P2: FXS
9780521740494c22.xml CUAU033-EVANS October 15, 2008 13:22
Chapter 22 Describing the distribution of a single variable 523
From Examples 10 and 11, the mean number of times premierships won (6.8) and the
median number of premierships won (5) have already been determined. These values are
different and the interesting question is: why are they different, and which is the better measure
of centre for this example? To help answer this question consider a stem-and-leaf plot of these
data.
0 0 1 1 1 2 2 3 4
0 6 9
1 0 1 2 4
1 6 6
From the stem-and-leaf plot it can be seen that the distribution is positively skewed. This
example illustrates a property of the mean. When the distribution is skewed or if there are one
or two very extreme values, then the value of the mean may be quite signicantly affected. The
median is not so affected by unusual observations, however, and is thus often a preferable
measure of centre. When this is the case, the median is generally preferred as a measure of
centre as it will give a better typical value of the variable under consideration.
Mode
The mode is the observation which occurs most often. It is a useful summary statistic,
particularly for categorical data which do not lend themselves to some of the other numerical
summary methods. Many texts state that the mode is a third option for a measure of centre but
this is generally not true. Sometimes data sets do not have a mode, or they have several modes,
or they have a mode which is at one or other end of the range of values.
Measures of spread
Range
A measure of spread is calculated in order to judge the variability of a data set. That is, are
most of the values clustered together, or are they rather spread out? The simplest measure of
spread can be determined by considering the difference between the smallest and the largest
observations. This is called the range.
Example 11
Consider the marks, for two different tasks, awarded to a group of students.
Task A
2 6 9 10 11 12 13 22 23 24 26 26 27 33 34
35 38 38 39 42 46 47 47 52 52 56 56 59 91 94
Task B
11 16 19 21 23 28 31 31 33 38 41 49 52 53 54
56 59 63 65 68 71 72 73 75 78 78 78 86 88 91
Find the range of each of these data sets.
P1: FXS/ABE P2: FXS
9780521740494c22.xml CUAU033-EVANS October 15, 2008 13:22
524 Essential Advanced General Mathematics
Solution
For Task A, the minimum mark is 2 and the maximum mark is 94.
Range for Task A = 94 2 = 92
For Task B, the minimum mark is 11 and the maximum mark is 91.
Range for Task B = 91 11 = 80
The range for Task A is greater than the range for Task B. Is the range a useful summary
statistic for comparing the spread of the two distributions? To help make this decision,
consider the stem-and-leaf plots of the data sets:
Task A Task B
9 6 2 0
3 2 1 0 1 1 6 9
7 6 6 4 3 2 2 1 3 8
9 8 8 5 4 3 3 1 1 3 8
7 7 6 2 4 1 9
9 6 6 2 2 5 2 3 4 6 9
6 3 5 8
7 1 2 3 5 8 8 8
8 6 8
4 1 9 1
From the stem-and-leaf plots of the data it appears that the spread of marks for the two tasks is
not well described by the range. The marks for Task A are more concentrated than the marks for
Task B, except for the two unusual values for Task A. Another measure of spread is needed, one
which is not so inuenced by these extreme values. For this the interquartile range is used.
Interquartile range
To nd the interquartile range of a distribution:
Arrange all observations in order according to size.
Divide the observations into two equal-sized groups. If n, the number of
observations, is odd, then the median is omitted from both groups.
Locate Q
1
, the rst quartile, which is the median of the lower half of the
observations, and Q
3
, the third quartile, which is the median of the upper half
of the observations.
The interquartile range IQR is dened as the difference between the quartiles.
That is
IQR = Q
3
Q
1
P1: FXS/ABE P2: FXS
9780521740494c22.xml CUAU033-EVANS October 15, 2008 13:22
Chapter 22 Describing the distribution of a single variable 525
Denitions of the quartiles of a distribution sometimes differ slightly from the one given here.
Using different denitions may result in slight differences in the values obtained, but these will
be minimal and should not be considered a difculty.
Example 12
Find the interquartile ranges for Task A and Task B data given in Example 11.
Solution
For Task A the marks listed in order are:
2 6 9 10 11 12 13 22 23 24 26 26 27 33 34
35 38 38 39 42 46 47 47 52 52 56 56 59 91 94
Since there is an even number of observations, then the lower half is:
2 6 9 10 11 12 13 22 23 24 26 26 27 33 34
The median of this lower group is the eighth observation, 22, so Q
1
= 22.
The upper half is:
35 38 38 39 42 46 47 47 52 52 56 56 59 91 94
The median of this upper group is 47, so Q
3
= 47
Thus, the interquartile range, IQR = 47 22
= 25
Similarly, for Task B data,
the lower quartile = 31 and
the upper quartile = 73,
giving an interquartile range for this data set of 42.
Comparing the two values of interquartile range shows the spread of Task A marks to
be much smaller than the spread of Task B marks, which seems consistent with the
display.
The interquartile range is a measure of spread of a distribution which describes the range of
the middle 50% of the observations. Since the upper 25% and the lower 25% of the
observations are discarded, the interquartile range is generally not affected by the presence of
outliers in the data set, which makes it a reliable measure of spread.
The median and quartiles of a distribution may also be determined from a cumulative
relative frequency polygon. Since the median is the observation which divides the data set in
half, this is the data value which corresponds to a cumulative relative frequency of 0.5 or 50%.
Similarly, the rst quartile corresponds to a cumulative relative frequency of 0.25 or 25%, and
the third quartile corresponds to a cumulative relative frequency of 0.75 or 75%.
P1: FXS/ABE P2: FXS
9780521740494c22.xml CUAU033-EVANS October 15, 2008 13:22
526 Essential Advanced General Mathematics
Example 13
Use the cumulative relative frequency polygon to nd the median and the interquartile range
for the data set shown in the graph.
0
25
50
75
% 100
2 4 6 8 10 12 14 16 18
Solution
From the plot of the data it can be seen that the median is 10, the rst quartile is 8, the
third quartile is 12 and hence the interquartile range is 12 8 = 4.
Standard deviation
Another extremely useful measure of spread is the standard deviation. It is derived by
considering the distance of each observation from the sample mean. If the average of these
distances is used as a measure of spread it will be found that, as some of these distances are
positive and some are negative, adding them together results in a total of zero. A more useful
measure will result if the distances are squared (which makes them all positive) and are then
added together. The variance is dened as a kind of average of these squared distances. When
the variance is calculated from a sample, rather than the whole population, the average is
calculated by dividing by n 1, rather than n. For the remainder of this discussion it will be
assumed that the data under consideration are from a sample.
Since the variance has been calculated by squaring the data values it is sensible to nd the
square root of the variance, so that the measure reverts to a scale comparable to the original
data. This results in measure of spread which is called the standard deviation. Standard
deviation calculated from a sample is denoted s.
Formally the standard deviation may be dened as follows.
If a data set consists of n observations denoted x
1
, x
2
, . . . , x
n
, the standard deviation is
s =
_
1
n 1
_
(x
1
x)
2
+ (x
2
x)
2
+ + (x
n
x)
2
_
or, in more compact notation,
s =

_
1
n 1
n

i =1
(x
i
x)
2
P1: FXS/ABE P2: FXS
9780521740494c22.xml CUAU033-EVANS October 15, 2008 13:22
Chapter 22 Describing the distribution of a single variable 527
Example 14
Calculate the standard deviation of the following data set.
13 12 14 6 15 12 7 6 7 8
Solution
Construct a table as shown.
x
i
x
i
x (x
i
x)
2
13 3 9
12 2 4
14 4 16
6 4 16
15 5 25
12 2 4
7 3 9
6 4 16
7 3 9
8 2 4
x
i
=

100 (x
i
x)
2
=

112
From the table, the standard deviation s is: s =
_
112
9
=

12.44 = 3.53
Interpreting the standard deviation
The standard deviation can be made more meaningful by interpreting it in relation to the data
set. The interquartile range gives the spread of the middle 50% of the data. Can similar
statements be made about the standard deviation? It can be shown that, for most data sets,
about 95% of the observations lie within two standard deviations of the mean.
Example 15
The cost of a lettuce at a number of different shops on a particular day is given in the table:
$3.85 $2.65 $1.90 $2.95 $2.40 $2.42 $2.63 $3.20 $4.20 $2.33 $0.85
$3.81 $1.69 $3.66 $2.60 $2.70 $3.10 $2.80 $1.80 $2.88 $1.40
Calculate the mean cost, the standard deviation and the interval equivalent to two standard
deviations above and below the mean.
P1: FXS/ABE P2: FXS
9780521740494c22.xml CUAU033-EVANS October 15, 2008 13:22
528 Essential Advanced General Mathematics
Solution
The mean cost is $2.66 and the standard deviation is $0.84.
The interval equivalent to two standard deviations above and below the mean is:
[2.66 2 0.84, 2.66 + 2 0.84] = [0.98, 4.34].
In this case, 20 of the 21 observations, or 95% of observations, have values within the
interval calculated.
Example 16
The prices of forty secondhand motorbikes listed in a newspaper are as follows:
$5442 $5439 $2523 $2358 $2363 $2244 $1963 $2142
$2220 $1356 $738 $656 $715 $1000 $1214 $1788
$3457 $4689 $8218 $11 091 $11 778 $11 637 $8770 $8450
$6469 $7148 $10 884 $14 450 $15 731 $13 153 $10 067 $9878
$5294 $3847 $4219 $4786 $2280 $3019 $7645 $8079
Determine the interval equivalent to two standard deviations above and below the mean.
Solution
The mean price is $5729 and the standard deviation is $4233 (to the nearest whole
dollar).
The interval equivalent to two standard deviations above and below the mean is:
[5729 2 4233, 5729 + 2 4233] = [2737, 14 195].
The negative value does not give a sensible solution and should be replaced by 0.
38 of the 40 observations, or 95% of observations, have values within the interval.
The exact percentage of observations which lie within two standard deviations of the mean
varies from data set to data set, but in general it will be around 95%, particularly for symmetric
data sets.
It was noted earlier that even a single outlier can have a very marked effect on the value of
the mean of a data set, while leaving the median unchanged. The same is true when the effect
of an outlier on the standard deviation is considered, in comparison to the interquartile range.
The median and interquartile range are called resistant measures, while the mean and standard
deviation are not resistant measures. When considering a data set it is necessary to do more
than just compute the mean and standard variation. First it is necessary to examine the data,
using a histogram or stem-and-leaf plot to determine which set of summary statistics is more
suitable.
P1: FXS/ABE P2: FXS
9780521740494c22.xml CUAU033-EVANS October 15, 2008 13:22
Chapter 22 Describing the distribution of a single variable 529
Using the TI-Nspire
The calculator can be used to calculate the values of all of the summary statistics in this
section. Consider the data from Example 16.
The data is easiest entered in a Lists &
Spreadsheet application ( 3).
Firstly, use the up/down arrows ( ) to
name the rst column bike.
Then enter each of the 40 numbers as
shown.
Open a Calculator application ( 1) to
calculate the summary statistics.
Select the One-Variable Statistics
command from the Stat Calculations
submenu of the Statistics menu (b6
11), specify in the dialog box that
there is only one list, and then complete the
nal dialog box as shown.
Press enter to calculate the values of the
summary statistics.
Use the up arrow ( ) to view the rest of the
summary statistics.
P1: FXS/ABE P2: FXS
9780521740494c22.xml CUAU033-EVANS October 15, 2008 13:22
530 Essential Advanced General Mathematics
The calculator can also be used to determine the summary statistics when the data is
given in a frequency table such as:
x 1 2 3 4
Frequency 5 8 7 2
The data is easiest entered in a Lists &
Spreadsheet application ( 3).
Firstly, use the up/down arrows ( ) to
name the rst column x and the second
column freq.
Then enter the data as shown.
Open a Calculator application ( 1) to
calculate the summary statistics.
Select the One-Variable Statistics
command from the Stat Calculations
submenu of the Statistics menu (b6
11), specify in the dialog box that
there is only one list, and then complete the
nal dialog box as shown. Press enter to
calculate the values of the summary
statistics.
Using the Casio ClassPad
Consider the following heights in cm of a group of eight women.
176, 160, 163, 157, 168, 172, 173, 169
Enter the data into list1 in the module. Tap Calc, One-Variable and when prompted
ensure that the XList is set to list1 and the Freq = 1 (since each score is entered
individually).
The calculator returns the results as shown and all univariate statistics can be viewed
by using the scroll bar. Note that the standard deviation is given by x
n1
.
Where data is grouped, the scores are entered in list1 and the frequencies in list2. In
this case, in Set Calculation use the drop-down arrow to select list2 as the location for
the frequencies.
P1: FXS/ABE P2: FXS
9780521740494c22.xml CUAU033-EVANS October 15, 2008 13:22
Chapter 22 Describing the distribution of a single variable 531
Exercise 22F
1 Find the mean and the median of the following data sets.
Examples 9, 10
a 29 14 11 24 14 14 28 14 18 22 14
b 5 9 11 3 12 13 12 6 13 7 3 15 12 15 5 6
c 8.3 5.6 8.2 6.5 8.2 7.0 7.9 7.1 7.8 7.5
d 1.5 0.2 0.7 0.7 0.2 0.2 0.1 1.7 0.5 1.2 2.0 1.7
1.0 3.4 1.3 0.9 1.1 5.8 2.7 3.2 0.6 4.6 0.5 3.1
2 Find the mean and the median of the following data sets.
a
x 1 2 3 4 5
Frequency 6 3 10 7 8
b
x 2 1 0 1 2
Frequency 5 8 11 3 2
3 The price, in dollars, of houses sold in a particular suburb during a one-week period are
given in the following list.
$187 500 $129 500 $93 400 $400 000 $118 000 $168 000 $550 000
$133 500 $135 500 $140 000 $186 000 $140 000 $204 000 $122 000
Find the mean and the median of the prices. Which do you think is a better measure of
centre of the data set? Explain your answer.
4 Concerned with the level of absence from his classes a teacher decided to investigate the
number of days each student had been absent from the classes for the year to date. These
are his results.
No. of days missed 0 1 2 3 4 5 6 9 21
No. of students 4 2 14 10 16 18 10 2 1
Find the mean and the median number of days each student had been absent so far that
year. Which is the better measure of centre in this case?
5 Find the range and the interquartile range for each of the following data sets.
Examples 11, 12
a
718 630 1002 560 715 1085 750 510 1112 1093
b
0.7 1.6 0.2 1.2 1.0 3.4 3.7 0.8
c
8.56 8.51 8.96 8.39 8.62 8.51 8.58 8.82 8.54
d
20 19 18 16 16 18 21 20 17 15 22 19
P1: FXS/ABE P2: FXS
9780521740494c22.xml CUAU033-EVANS October 15, 2008 13:22
532 Essential Advanced General Mathematics
6 The serum cholesterol levels for a sample of twenty people are:
231 159 203 304 248 238 209 193 225 244
190 192 209 161 206 224 276 196 189 199
a Find the range of the serum cholesterol levels.
b Find the interquartile range of the serum cholesterol levels.
7 Twenty babies were born at a local hospital on one weekend. Their birth weights, in kg,
are given in the stem-and-leaf plot below.
2 1
2 5 7 9 9
3 1 3 3 4 4
3 5 6 7 7 9
4 1 2 2 3
4 5 3|6 represent 3.6 kg
a Find the range of the birth weights.
b Find the interquartile range of the birth weights.
8 Find the standard deviation for the following data sets.
Example 14
a
30 16 22 23 18 18 14 56 13 26 9 31
b
$2.52 $4.38 $3.60 $2.30 $3.45 $5.40 $4.43 $2.27 $4.50
$4.32 $5.65 $6.89 $1.98 $4.60 $5.12 $3.79 $4.99 $3.02
c
200 300 950 200 200 300 840 350 200 200
d 86 74 75 77 79 82 81 75 78 79 80 75 78 78 81 80 76 77 82
9 For each of the following data sets
a calculate the mean and the standard deviation
b determine the percentage of observations falling within two standard deviations of the
Example 15
mean.
i
41 16 6 21 1 21 5 31 20 27 17 10 3 32 2 48 8 12
21 44 1 56 5 12 3 1 13 11 15 14 10 12 18 64 3 10
ii
141 260 164 235 167 266 150 255 168 245 258 239
152 141 239 145 134 150 237 254 150 265 140 132
10 A group of university students was asked to write down their ages with the following
Example 13
results.
17 17 17 17 17 17 17 18 18 18 18 18 18 18 18 18 18 18
18 18 18 18 18 19 19 19 20 20 20 21 24 25 31 41 44 45
a Construct a cumulative relative frequency polygon and use it to nd the median and
the interquartile range of this data set.
b Find the mean and standard deviation of the ages.
c Find the percentage of students whose ages fall within two standard deviations of the
mean.
P1: FXS/ABE P2: FXS
9780521740494c22.xml CUAU033-EVANS October 15, 2008 13:22
Chapter 22 Describing the distribution of a single variable 533
11 The results of a students chemistry experiment are as follows.
7.3 8.3 5.9 7.4 6.2 7.4 5.8 6.0
a
i Find the mean and the median of the results.
ii Find the interquartile range and the standard deviation of the results.
b Unfortunately when the student was transcribing his results into his chemistry book he
made a small error, and wrote:
7.3 8.3 5.9 7.4 6.2 7.4 5.8 60
i Find the mean and the median of these results.
ii Find the interquartile range and the standard deviation of these results.
c Describe the effect the error had on the summary statistics calculated in parts a
and b.
12 A selection of shares traded on the stock exchange had a mean price of $50 with a
Example 17
standard deviation of $3. Determine an interval which would include approximately 95%
of the share prices.
13 A store manager determined the stores mean daily receipts as $550, with a standard
deviation of $200. On what proportion of days were the daily receipts between $150 and
$950?
22.7 The boxplot
Knowing the median and quartiles of a distribution means that quite a lot is known about the
central region of the data set. If something is known about the tails of the distribution then a
good picture of the whole data set can be obtained. This can be achieved by knowing the
maximum and minimum values of the data. These ve important statistics can be derived from
a data set: the median, the two quartiles and the two extremes.
These values are called the ve-gure summary and can be used to provide a succinct
pictorial representation of a data set called the box and whisker plot, or boxplot.
For this visual display, a box is drawn with the ends at the rst and third quartiles. Lines are
drawn which join the ends of the box to the minimum and maximum observations. The median
is indicated by a vertical line in the box.
Example 17
Draw a boxplot to show the number of hours spent on a project by individual students in a
particular school.
24 4 166 147 97 90 36 92 226 37 111
59 102 13 108 2 71 102 147 56 181 35
9 3 48 27 264 86 9 40 146 19 76
P1: FXS/ABE P2: FXS
9780521740494c22.xml CUAU033-EVANS October 15, 2008 13:22
534 Essential Advanced General Mathematics
Solution
First arrange the data in order.
2 3 4 9 9 13 19 24 27 35 36
37 40 48 56 59 71 76 86 90 92 97
102 102 108 111 146 147 147 166 181 226 264
From this ordered list prepare the ve-gure summary.
median, m = 71
rst quartile, Q
1
=
24 + 27
2
= 25.5
third quartile, Q
3
=
108 + 111
2
= 109.5
minimum = 2
maximum = 264
The boxplot can then be drawn.
300 200 100 0
min = 2
max = 264 Q
1
= 25.5 Q
3
= 109.5
m = 71
In general, to draw a boxplot:
Arrange all the observations in order, according to size.
Determine the minimum value, the rst quartile, the median, the third quartile, and
the maximum value for the data set.
Draw a horizontal box with the ends at the rst and third quartiles. The height of the
box is not important.
Join the minimum value to the lower end of the box with a horizontal line.
Join the maximum value to the upper end of the box with a horizontal line.
Indicate the location of the median with a vertical line.
P1: FXS/ABE P2: FXS
9780521740494c22.xml CUAU033-EVANS October 15, 2008 13:22
Chapter 22 Describing the distribution of a single variable 535
The symmetry of a data set can be determined from a boxplot. If a data set is symmetric, then
the median will be located approximately in the centre of the box, and the tails will be of
similar length. This is illustrated in the following diagram, which shows the same data set
displayed as a histogram and a boxplot.
A median placed towards the left of the box, and/or a long tail to the right indicates a
positively skewed distribution, as shown in this plot.
A median placed towards the right of the box, and/or a long tail to the left indicates a
negatively skewed distribution, as illustrated here.
P1: FXS/ABE P2: FXS
9780521740494c22.xml CUAU033-EVANS October 15, 2008 13:22
536 Essential Advanced General Mathematics
A more sophisticated version of a boxplot can be drawn with the outliers in the data set
identied. This is very informative, as one cannot tell from the previous boxplot if an
extremely long tail is caused by many observations in that region or just one.
Before drawing this boxplot the outliers in the data set must be identied. The term outlier
is used to indicate an observation which is rather different from other observations. Sometimes
it is difcult to decide whether or not an observation should be designated as an outlier. The
interquartile range can be used to give a very useful denition of an outlier.
An outlier is any number which is more than 1.5 interquartile ranges above the upper
quartile, or more than 1.5 interquartile ranges below the lower quartile.
When drawing a boxplot, any observation identied as an outlier is indicated by an asterisk,
and the whiskers are joined to the smallest and largest values which are not outliers.
Example 18
Use the data from Example 17 to draw a boxplot with outliers.
Solution
median = 71
interquartile range = Q
3
Q
1
= 109.5 25.5
= 84
An outlier will be any observation which is less than 25.5 1.5 84 = 100.5,
which is impossible, or greater than 109.5 + 1.5 84 = 235.5. From the data it can
be seen that there is only one observation greater than this, 264, which would be
denoted with an asterisk.
The upper whisker is now drawn from the edge of the box to the largest observation
less than 235.5, which is 226.
300 200 100 0
*
P1: FXS/ABE P2: FXS
9780521740494c22.xml CUAU033-EVANS October 15, 2008 13:22
Chapter 22 Describing the distribution of a single variable 537
Using the TI-Nspire
The calculator can be used to construct a boxplot. Consider the data from Example 17.
The data is easiest entered in a Lists &
Spreadsheet application ( 3).
Firstly, use the up/down arrows ( ) to
name the rst column hours.
Then enter each of the 33 numbers as
shown.
Open a Data & Statistics application (
5) to graph the data. At rst the data
displays as shown.
Specify the x variable by selecting Add X
Variable from the Plot Properties (b2
4) and selecting hours. The data now
displays as shown.
(Note: It is also possible to use the NavPad
to move down below the x-axis and click to
add the x variable.)
Select Box Plot from the Plot Type menu
(b12). The data now displays as
shown.
Notice how the calculator, by default,
shows any outlier(s).
P1: FXS/ABE P2: FXS
9780521740494c22.xml CUAU033-EVANS October 15, 2008 13:22
538 Essential Advanced General Mathematics
To not show the outlier(s), select Extend
Box Plot Whiskers from the Plot Properties
menu (b23). The data now
displays as shown.
Note: It is possible to show the values of the
ve-point summary by moving the cursor
over the boxplot.
Using the Casio ClassPad
In the following consider the set of marks:
28 21 21 3 22 31 35 26 27 33 36 35 23 24 43 31 30 34 48
In enter the data into list1. Tap SetGraph, Setting . . . and the tab for Graph 2, enter
the settings shown including the tick box and tap SET. (Note
that on the Classpad you can store settings for a number of different graphs and return
to them quickly.)
Tap SetGraph, StatGraph2 and tap the box to
tick and select the graph (de-select any other
graphs). Tap to produce the graph. The
boxplot is produced as shown.
With the graph window selected (bold border),
tap 6to adjust the viewing window for the
graph.
Tap Analysis, Trace and use the navigator key to
move between the outlier(s), Minimum, Q1,
Median, Q3 and Maximum scores.
Starting from the left of the plot, we see that the:
Minimum value is 3: min X = 3. It is also an
outlier
Lower adjacent value is 21: X = 21
First quartile is 23: Q1 = 23
Median is 30: Med = 30
Second quartile is 35: Q3 = 35
Maximum value is 48: max X = 48.
P1: FXS/ABE P2: FXS
9780521740494c22.xml CUAU033-EVANS October 15, 2008 13:22
Chapter 22 Describing the distribution of a single variable 539
Exercise 22G
1 The heights (in centimetres) of a class of girls are
Example 17
160 165 123 143 154 180 133 123 157 157 135 140 140 150
154 159 149 167 176 163 154 167 168 132 145 143 157 156
a Determine the ve-gure summary for this data set.
b Draw a boxplot of the data.
c Describe the pattern of heights in the class in terms of shape, centre and spread.
2 A researcher is interested in the number of books people borrow from a library. She
Example 18
decided to select a sample of 38 cards and record the number of books each person has
borrowed in the previous year. Here are her results.
7 28 0 2 38 18 0 0 4 0 0 2 13
1 1 14 1 8 27 0 52 4 0 12 28 15
10 1 0 2 0 1 11 5 11 0 13 0
a Determine the ve-gure summary for this data set.
b Determine if there are any outliers.
c Draw a boxplot of the data, showing any outliers.
d Describe the number of books borrowed in terms of shape, centre and spread.
3 The winnings of the top 25 male tennis players in 2004 are given in the following table.
Player Winnings
Roger Federer 6 357 547
Lleyton Hewitt 2 766 051
Andy Roddick 2 604 590
Marat San 2 273 283
Guillermo Coria 1 697 155
Gaston Gaudio 1 639 171
Tim Henman 1 508 177
Carlos Moya 1 448 209
Andre Agassi 1 177 254
David Nalbandian 1 045 985
Jonas Bjorkman 927 344
Tommy Robredo 861 357
Nicolas Massu 854 533
Player Winnings
Joachim Johansson 828 744
Jiri Novak 813 792
Dominik Hrbaty 808 944
Guillermo Canas 780 701
Fernando Gonzalez 766 416
Sebastian Grosjean 755 795
Feliciano Lopez 748 662
Max Mirnyi 742 196
Juan Ignacio Chela 727 736
Mikhail Youzhny 725 948
Radek Stepanek 706 387
Vincent Spadea 704 105
a Draw a boxplot of the data, indicating any outliers.
b Describe the data in terms of shape, centre, spread and outliers.
4 The hourly rate of pay for a group of students engaged in part-time work was found to be:
$4.75 $8.50 $17.23 $9.00 $12.00 $11.69 $6.25
$7.50 $8.89 $6.75 $7.90 $12.46 $10.80 $8.40
$12.34 $10.90 $11.65 $10.00 $10.00 $13.00
P1: FXS/ABE P2: FXS
9780521740494c22.xml CUAU033-EVANS October 15, 2008 13:22
540 Essential Advanced General Mathematics
a Draw a boxplot of the data, indicating any outliers.
b Describe the hourly pay rate for the students in terms of shape, centre, spread and
outliers.
5 The daily circulation of several newspapers in Australia is:
570 000 327 654 299 797 273 248 258 700 230 487
217 284 214 000 212 770 171 568 170 000 125 778
98 158 77 500 56 000 43 330 17 398
a Draw a boxplot of the data, indicating any outliers.
b Describe the daily newspaper circulation in terms of shape, centre, spread and outliers.
22.8 Using boxplots to compare distributions
Boxplots are extremely useful for comparing two or more sets of data collected on the same
variable, such as marks on the same assignment for two different groups of students. By
drawing boxplots on the same axis, both the centre and spread for the distributions are readily
identied and can be compared visually.
Example 19
The number of hours spent by individual students on the project referred to in Example 17 at
another school were:
53 152 82 30 16 136 21 11 1 55 128
57 106 14 18 173 102 86 227 48 12 45
136 226 17 9 156 19 107 24 42 21 176
24 80 54 16 106 6 38 3
Use boxplots to compare the time spent on the project by students at this school with those in
Example 17.
Solution
The ve-gure summary for this data set is:
median, m = 48; rst quartile, Q
1
= 17.5; third quartile, Q
3
= 106.5; minimum = 1;
maximum = 227
In order to compare the time spent on the project by the students at each school,
boxplots for both data sets are drawn on the same axis.
300 200 100 0
*
School 1
School 2
P1: FXS/ABE P2: FXS
9780521740494c22.xml CUAU033-EVANS October 15, 2008 13:22
Chapter 22 Describing the distribution of a single variable 541
From the boxplots the distributions of time for the two schools can be compared in
terms of shape; centre, spread and outliers. Clearly the two distributions for both
schools are positively skewed, indicating a larger range of values in the upper half of
the distributions. The centre for School 1 is higher than the centre for School 2
(71 hours compared to 48 hours). As can be seen by comparing the box widths, which
indicate the IQR, the spread of the data is comparable for both distributions. There is
one outlier, a student who attended School 1 and spent 264 hours on the project.
The boxplot is useful for summarising large data sets and for comparing several sets of data. It
focuses attention on important features of the data and gives a picture of the data which is easy
to interpret. When a single data set is being investigated a stem-and-leaf plot is sometimes
better, as a boxplot may hide the local detail of the data set.
Exercise 22H
1 To test the effect of a physical tness course the number of sit-ups that a person could do
in 1 minute, both before and after the course, were recorded. Twenty randomly selected
participants scored as follows.
Example 19
Before 29 22 25 29 26 24 31 46 34 28
23 22 26 26 30 12 17 21 20 30
After 28 26 25 35 33 36 32 54 50 43
25 24 30 34 30 15 29 21 19 34
a Construct boxplots of these two sets of data on the same axis.
b Describe the effect of the physical tness course on the number of sit-ups achieved in
terms of shape, centre, spread and outliers.
2 The number of hours spent on homework per week by a group of students in Year 8 and a
group of students in Year 12 are shown in the tables.
Year 8 1 2 4 2 4 4 5 3 7 7 2 4 3 3
1 3 4 3 3 1 7 2 1 3 1 4 1 0
Year 12 1 2 3 5 6 7 7 6 7 8 7 5 4 1
2 3 1 1 4 7 8 9 6 7 8 7 2 3
Draw boxplots of these two sets of data on the same axis and use them to answer the
following questions.
a Which group does the most homework?
b Which group varies more in the number of hours homework they do?
P1: FXS/ABE P2: FXS
9780521740494c22.xml CUAU033-EVANS October 15, 2008 13:22
542 Essential Advanced General Mathematics
3 The ages of mothers at the birth of their rst child were noted, for the rst forty such
births, at a particular hospital in 1970 and again in 1990.
1970 21 29 25 32 37 30 24 36 23 19
37 22 26 31 26 27 19 21 33 17
24 21 22 36 22 25 31 20 18 20
16 21 25 26 34 27 18 39 24 21
1990 24 22 35 32 17 28 38 20 30 39
19 33 44 24 18 27 24 33 29 23
26 18 28 32 43 28 26 28 41 28
25 35 31 23 19 46 29 23 34 29
a Construct boxplots of these two sets of data on the same axis.
b Compare the ages of the mothers in 1970 and 1990 in terms of shape, centre, spread
and outliers.
P1: FXS/ABE P2: FXS
9780521740494c22-1.xml CUAU033-EVANS October 5, 2008 8:27
R
e
v
i
e
w
Chapter 22 Describing the distribution of a single variable 543
Chapter summary
Variables may be classied as categorical or numerical. Numerical data may be discrete
or continuous.
Examination of a data set should always begin with a visual display.
A bar chart is the appropriate visual display for categorical data.
When a data set is small, a stem-and-leaf plot is the most appropriate visual display for
numerical data.
When a data set is larger, a histogram, frequency polygon or boxplot is a more
appropriate visual display for numerical data.
Cumulative frequency distributions and cumulative relative frequency distributions are
useful for answering questions about the number or proportion of data values greater than
or less than a particular value. These are graphically represented in cumulative frequency
polygons or cumulative relative frequency polygons.
From a stem-and-leaf plot, histogram or boxplot, insight can be gained into the shape,
centre and spread of the distribution, and whether or not there are any outliers.
An outlier is a value which sits away from the main body of the data in a plot. It is formally
dened as a value more than 1.5IQR below Q
1
, or more than 1.5IQR above Q
3
.
For numerical data it is also very useful to calculate some summary statistics.
The mean is dened as x =
1
n
n

i =1
x
i
.
If n, the number of observations, is odd, then the median is the

n +1
2

th
observation
from the end of the ordered list. If n is even, then the median is found by averaging the two
middle observations in the list, i.e., the

n
2

th
and the

n
2
+1

th
observations are added
together and divided by 2.
The mode is the most common observation in a group of data.
The most useful measures of centre are the median and the mean.
To nd the interquartile range of a distribution:
r
Arrange all observations in order according to size.
r
Divide the observations into two equal sized groups. If n, the number of
observations, is odd, then the median is omitted from both groups.
r
Locate Q
1
, the rst quartile, which is the median of the lower half of the
observations, and Q
3
, the third quartile, which is the median of the upper half of the
observations.
r
The interquartile range IQR is dened as the difference between the quartiles. That is
IQR = Q
3
Q
1
The standard deviation is dened as s =

1
n 1
n

i =1
(x
i
x)
2
.
The most useful measures of spread are the interquartile range and the standard
deviation.
P1: FXS/ABE P2: FXS
9780521740494c22-1.xml CUAU033-EVANS October 5, 2008 8:27
R
e
v
i
e
w
544 Essential Advanced General Mathematics
The ve-gure summary of a set of data consists of the minimum, Q
1
, median, Q
3
, and
the maximum. A boxplot is a diagrammatic representation of this, e.g.
min
max
Q
1
Q
3
median
When the data set is symmetric any of the summary statistics are appropriate.
When the data set is not symmetric or when there are outliers the median and the
interquartile range are the preferred summary statistics.
In general, 95% of the values of the data set will fall within two standard deviations of the
mean.
When comparing the distribution of two or more data sets the comparison should be made
in terms of the shape, centre, spread and outliers for each distribution.
Multiple-choice questions
1 In a survey a number of subjects were asked to indicate how much they exercise by
selecting one of the following options.
1 Never 2 Seldom 3 Occasionally 4 Regularly
The resulting variable was named Level of Exercise, and the level of measurement of this
variable is
A variable B numerical C constant D categorical E metric
Questions 2, 3 and 4 relate to the following information.
The numbers of hours worked per week by employees in a large company are shown in this
percentage frequency histogram.
Hours worked weekly
P
e
r
c
e
n
t
a
g
e

F
r
e
q
u
e
n
c
y
0
10
20
30
40
20 40 60 80
P1: FXS/ABE P2: FXS
9780521740494c22-1.xml CUAU033-EVANS October 5, 2008 8:27
R
e
v
i
e
w
Chapter 22 Describing the distribution of a single variable 545
2 The percentage of employees who work from 20 to less than 30 hours per week is closest to
A 1% B 2% C 6% D 10% E 33%
3 The median number of hours worked is in the interval
A 10 to less than 20 B 20 to less than 30
C 30 to less than 40 D 40 to less than 50
E 50 to less than 60
Questions 4 and 5 relate to the following information.
A group of 19 employees of a company was asked to record the number of meetings that they
attended in the last month. Their responses are summarised in the following stem-and-leaf plot.
0 1 1 2 3 4 5 5 6 6 7 9
1 0 2 4 4 6
2 2 3
3
4 4
4 The median number of meetings is
A 6 B 6.5 C 7 D 7.5 E 9
5 The interquartile range (IQR) of number of meetings is
A 0 B 4 C 9.5 D 10 E 14
6 The cumulative frequency polygon shown gives the
examination scores in Mathematics for a group of
200 students.
0
40 50 60 70 80 90
100
200
Exam Score
N
u
m
b
e
r

o
f

s
t
u
d
e
n
t
s
The number of students who scored less than 70 on the
examination is closest to
A 30 B 100 C 150 D 175 E 200
Questions 7 and 8 relate to the following information.
The number of years that a sample of people has lived that their current address is summarised
in this boxplot.
Years lived
this address
0 10 20 30 40 50
7 The shape of the distribution of years lived at this address is:
A positively skewed B negatively skewed C bimodal
D symmetric E symmetric with outliers
8 The interquartile range years lived at this address is approximately equal to:
A 5 B 8 C 17 D 12 E 50
P1: FXS/ABE P2: FXS
9780521740494c22-1.xml CUAU033-EVANS October 5, 2008 8:27
R
e
v
i
e
w
546 Essential Advanced General Mathematics
Questions 9 and 10 relate to the following data.
The amount paid per week to the employees of each of ve large companies are shown in the
boxplots:
0
Company 1
Company 2
Company 3
20000 60000 80000 120000 100000 40000
Yearly income
9 The company with the lowest typical wage is
A Company 1 B Company 2
C Company 3 D Company 1 and Company 2
E Company 2 and Company 3
10 The company with the largest variation in wage is
A Company 1 B Company 2
C Company 3 D Company 1 and Company 2
E Company 2 and Company 3
Short-answer questions (technology-free)
1 Classify the data which arise from the following situations as categorical or numerical.
a The number of phones calls a hotel receptionist receives each day.
b Interest in politics on a scale from 1 to 5 where 1 = very interested, 2 = quite interested,
3 = somewhat interested, 4 = not very interested, and 5 = uninterested.
2 This bar chart shows the
percentage of people working who are
employed in private companies, work
for the Government or are self-employed
in a certain town.
Private Government Self-employed
Type of company worked for
P
e
r
c
e
n
t
0
10
30
40
50
20
a What kind of measurement is the
Type of company worked for?
b Approximately what percentage of
the people are self-employed?
P1: FXS/ABE P2: FXS
9780521740494c22-1.xml CUAU033-EVANS October 5, 2008 8:27
R
e
v
i
e
w
Chapter 22 Describing the distribution of a single variable 547
3 A researcher asked a group of people to record how many
cigarettes they had smoked on a particular day. Here are her results:
0 0 9 10 23 25 0 0 34 32 0 0 30 0 4
5 0 17 14 3 6 0 33 23 0 32 13 21 22 6
Using an appropriate class interval, construct a histogram of these data.
4 A teacher recorded the time taken (in minutes) by each of a class of students to complete a
test.
56 57 47 68 52 51 43 22 59 51 39
54 52 69 72 65 45 44 55 56 49 50
a Make a stem-and-leaf plot of these times, using one row per stem.
b Use this stem-and-leaf-plot to nd the median and quartiles for the time taken.
5 The weekly rentals, in dollars, for apartments in a particular suburb are given in the
following table.
285 185 210 215 320 680 280
265 300 210 270 190 245 315
Find the mean and the median of the weekly rental.
6 Geoff decided to record the time it takes him to complete his mail delivery round each
working day for four weeks. His data are recorded in the following table.
170 189 201 183 168 182 161 166 167 173
164 176 161 187 180 201 147 188 186 176
182 167 188 211 174 193 185 183
The mean of the time taken, x, is 179 and the standard deviation, s, is 14.
a Determine the percentage of observations falling within two standard deviations of the
mean.
b Is this what you would expect to nd?
7 A group of students were asked to record the number of SMS messages that they sent in
one 24-hour period, and the following ve-gure summary was obtained from the data set.
Use it to construct a simple boxplot of these data.
Min = 0, Q
1
= 3, Median = 5, Q
3
= 12, Max = 24
8 The following table gives the number of students absent each day from a large secondary
college on each of 36 randomly chosen school days.
7 22 12 15 21 16 23 23 17 23 8 16
7 3 21 30 13 2 7 12 18 14 14 0
15 16 13 21 10 16 11 4 3 0 31 44
Construct a boxplot of these data, with outliers.
P1: FXS/ABE P2: FXS
9780521740494c22-1.xml CUAU033-EVANS October 5, 2008 8:27
R
e
v
i
e
w
548 Essential Advanced General Mathematics
Extended-response questions
1 The divorce rates (in percentages) of 19 countries are
27 18 14 25 28 6 32 44 53 0
26 8 14 5 15 32 6 19 9
a What is the level of measurement of the variable, divorce rate?
b Construct an ordered stem-and-leaf plot of divorce rates, with one row per stem.
c What shape is the divorce rates?
d What percentage of countries have divorce rates greater than 30?
e Calculate the mean and median of the divorce rates for the 19 countries.
f Construct a histogram of the data with class intervals of width 10.
i What is the shape of the histogram?
ii How many countries had divorce rates from 10% to less than 20%?
g Construct a cumulative percentage frequency polygon of divorce rates.
i What percentage of countries has divorce rates less than 20%?
ii Use the cumulative frequency distribution to estimate the median percentage
divorce rate.
2 Hillside Trains have decided to improve their service on the Lilydale line. Trains were timed
on the run from Lilydale to Flinders Street, and their times recorded over a period of six
weeks at the same time each day. The time taken for each journey is shown below.
60 61 70 72 68 80 76 65 69 79 82
90 59 86 70 77 64 57 65 60 68 60
63 67 74 78 65 68 82 89 75 62 64
58 64 69 59 62 63 89 74 60
a Construct a histogram of the times taken for the journey from Lilydale to Flinders
Street, using class intervals 5559, 6064, 6569 etc.
i On how many days did the trip take from 6569 minutes?
ii What shape is the histogram?
iii What percentage of trains took less than 65 minutes to reach Flinders Street?
b Calculate the following summary statistics for the time taken (correct to two decimal
places).
x s Min Q
1
M Q
3
Max
c Use the summary statistics to complete the following report.
i The mean time taken from Lilydale to Flinders Street (in minutes) was . . .
ii 50% of the trains took more than . . . minutes to travel from Lilydale to Flinders
Street.
iii The range of travelling times was . . . minutes while the interquartile range was . . ...
minutes.
(contd)
P1: FXS/ABE P2: FXS
9780521740494c22-1.xml CUAU033-EVANS October 5, 2008 8:27
R
e
v
i
e
w
Chapter 22 Describing the distribution of a single variable 549
iv 25% of trains took more than . . . minutes to travel to Flinders Street.
v The standard deviation of travelling times was . . .
vi Approximately 95% of trains took between . . . and . . . minutes to travel to Flinders
St.
d Summary statistics for the year before Hillside Trains took over the Lilydale line from
the Met are indicated below:
Min = 55 Q
1
= 65 Median = 70 Q
3
= 89 Max = 99
Draw simple boxplots for the last year the Met ran the line and the data from Hillside
trains on the same axis.
e Use the information from the boxplots to compare travelling times for the two transport
corporations in terms of shape, centre and spread.
3 In a small company, upper management wants to know if there is a difference in the three
methods used to train its machine operators. One method uses a hands-on approach. A
second method uses a combination of classroom instruction and on-the-job training. The
third method is based completely on classroom training. Fifteen trainees are assigned
to each training technique. The following data are the results of a test undertaken by the
machine operators after completion of one of the different training methods.
Method 1 Method 2 Method 3
98 79 70
100 62 74
89 61 60
90 89 72
81 69 65
85 99 49
97 87 71
95 62 75
87 65 55
70 88 65
69 98 70
75 79 59
91 73 77
92 96 67
93 83 80
a Draw boxplots of the data sets, on the same axis.
b Write a paragraph comparing the three training methods in terms of shape, centre,
spread and outliers.
c Which training method would you recommend?
P1: FXS/ABE P2: FXS
9780521740494c22-1.xml CUAU033-EVANS October 5, 2008 8:27
R
e
v
i
e
w
550 Essential Advanced General Mathematics
4 It has been argued that there is a relationship between a childs level of independence and
the order in which they were born in the family. Suppose that the children in thirteen
three-children families are rated on a 50-point scale of independence. This is done when all
children are adults, thus eliminating age effects. The results are as follows.
Family 1 2 3 4 5 6 7 8 9 10 11 12 13
First-born 38 45 30 29 34 19 35 40 25 50 44 36 26
Second-born 9 40 24 16 16 21 34 29 22 29 20 19 18
Third-born 12 12 12 25 9 11 20 12 10 20 16 13 10
a Draw boxplots of the data sets on the same axis.
b Write a paragraph comparing the independence scores of rst-, second-
and third-born children.
P1: FXS/ABE P2: FXS
9780521740494c23.xml CUAU033-EVANS October 5, 2008 8:31
C H A P T E R
23
Investigating the
relationship between
two numerical variables
Objectives
To use scatterplots to display bivariate (numerical) data
To identify patterns and features of sets of data from scatterplots
To identify positive, negative or no association between variables from a scatterplot
To introduce the q-correlation coefficient to measure the strength of the
relationship between two variables
To introduce Pearsons product-moment correlation coefficient r to measure the
strength of the linear relationship between two variables
To fit a straight line to data by eye, and using the method of least squares
To interpret the slope of a regression line and its intercept, if appropriate
To predict the value of the dependent (response) variable from an independent
(explanatory) variable, using a linear equation
In Chapter 22 statistics of one variable were discussed. Sometimes values of a variable for
more than one group have been examined, such as age of mothers and age of fathers, but only
one variable was considered for each individual at a time.
When two variables are observed for each subject, bivariate data are obtained. For example,
it might be interesting to record the number of hours spent studying for an exam by each
student in a class and the mark they achieved in the exam. If each of these variables were
considered separately the methods discussed earlier would be used. It may be of more interest
to examine the relationship between the two variables, in which case new bivariate techniques
are required. When exploring bivariate data, questions arise such as, Is there a relationship
between two variables? or Does knowing the value of one of the variables tell us anything
about the value of the other variable?
551
P1: FXS/ABE P2: FXS
9780521740494c23.xml CUAU033-EVANS October 5, 2008 8:31
552 Essential Advanced General Mathematics
Consider the relationship between the number of cigarettes smoked per day and blood
pressure. Since one opinion might be that varying the number of cigarettes smoked may affect
blood pressure, it is necessary to distinguish between blood pressure, which is called the
dependent or response variable, and the number of cigarettes, which is called the
independent or explanatory variable. In this chapter some techniques are introduced which
enable questions concerning the nature of the relationship between such variables to be
answered.
23.1 Displaying bivariate data
As with data concerning one variable, the most important rst step in analysing bivariate data
is the construction of a visual display. When both of the variables of interest are numerical then
a scatterplot (or bivariate plot) may be constructed. This is the single most important tool in
the analysis of such bivariate data, and should always be examined before further analysis is
undertaken. The pairs of data points are plotted on the cartesian plane, with each pair
contributing one point to the plot. Using the normal convention, the variable plotted
horizontally is denoted as x, and the variable plotted vertically as y. The following example
examines the features of the scatterplot in more detail.
Example 1
The number of hours spent studying for an examination by each member of a class, and the
marks they were awarded, are given in the table.
Student 1 2 3 4 5 6 7 8 9 10
Hours 4 36 23 28 25 11 18 13 4 8
Mark 27 87 67 84 66 52 61 43 38 52
Student 11 12 13 14 15 16 17 18 19 20
Hours 4 19 6 19 1 29 33 36 28 15
Mark 41 54 57 62 23 65 75 83 65 55
Construct a scatterplot of these data.
Solution
The rst decision to be made when preparing this scatterplot is whether to show Mark
or Hours on the horizontal (x) axis. Since a students mark is likely to depend on the
hours that they spend studying, in this case Hours is the independent variable and
Mark is the dependent variable. By convention, the independent variable is plotted on
the horizontal (x) axis, and the dependent variable on the vertical (y) axis, giving the
scatterplot shown.
P1: FXS/ABE P2: FXS
9780521740494c23.xml CUAU033-EVANS October 5, 2008 8:31
Chapter 23 Investigating the relationship between two numerical variables 553
Hours x
0
20
20
40
40
60
80
10 30
Mark y
From this scatterplot, a general trend can be seen of increasing marks with increasing hours of
study. There is said to be a positive association between the variables.
Two variables are positively associated when larger values of y are associated with larger
values of x, as shown in the previous scatterplot.
Examples of variables which exhibit positive association are height and weight, foot size and
hand size, and number of people in the family and household expenditure on food.
Example 2
The age, in years, of several cars and their advertised price in a newspaper are given in the
following table.
Age (years) 4 6 5 7 4 2 3 3
Price ($) 13 000 9 800 11 000 8 300 10 500 15 800 14 300 13 800
Age (years) 7 6 4 6 4 8 6
Price ($) 9 700 9 500 13 200 10 000 11 800 8 000 12 200
Construct a scatterplot to display these data.
Solution
In this case the independent variable is the age of the car, which is plotted on the
horizontal axis. The dependent variable, price, is plotted on the vertical axis.
2 4 6 8 1 3 5 7
Age (years) x
8000
10000
12000
14000
16000
Price ($) y
P1: FXS/ABE P2: FXS
9780521740494c23.xml CUAU033-EVANS October 5, 2008 8:31
554 Essential Advanced General Mathematics
From the scatterplot a general trend of decreasing price with increasing age of car can be seen.
There is said to be a negative association between the variables.
Two variables are negatively associated when larger values of y are associated with smaller
values of x, as shown in the scatterplot above.
Examples of other variables which exhibit negative association are weight and number of
weeks spent on a diet program, hearing ability and age, and number of cold rainy days per
week and sales of ice creams.
The third alternative is that a scatterplot shows no particular pattern, indicating no
association between the variables.
2
2
4
4
6
6
8
8
1 3 5 7 x
y
0
There is no association between two variables when the values of y are not related to the
values of x, as shown in the preceding scatterplot.
Examples of variables which show no association are height and IQ for adults, price of cars
and fuel consumption, and size of family and number of pets.
When one point, or a few points, do not seem to t with the rest of the data they are called
outliers. Sometimes a point is an outlier, not because its x value or its y value is in itself
unusual, but rather because this particular combination of values is atypical. Consequently such
an outlier cannot always be detected from single variable displays, such as stem-and-leaf plots.
For example, consider this scatterplot. While the
variable plotted on the horizontal axis takes values
from 1 to 8 and the variable plotted on the vertical
axis takes values from 2 to 8, the combination (2, 8)
is clearly an outlier.
2
2
4
4
6
6
8
8
1 3 5 7 x
y
0
P1: FXS/ABE P2: FXS
9780521740494c23.xml CUAU033-EVANS October 5, 2008 8:31
Chapter 23 Investigating the relationship between two numerical variables 555
Using the TI-Nspire
The calculator can be used to construct a scatterplot of statistical data. The procedure is
illustrated using the age and price of car data from Example 2.
The data is easiest entered in a Lists &
Spreadsheet application ( 3).
Firstly, use the up/down arrows ( ) to
name the rst column age and the second
column price.
Then enter the data as shown.
Open a Data & Statistics application (
5) to graph the data. At rst the data
displays as shown.
Specify the x variable by selecting Add X
Variable from the Plot Properties (b2
4) and selecting age.
Specify the y variable by selecting Add Y
Variable from the Plot Properties (b2
6) and selecting price.
The data now displays as shown.
P1: FXS/ABE P2: FXS
9780521740494c23.xml CUAU033-EVANS October 5, 2008 8:31
556 Essential Advanced General Mathematics
Using the Casio ClassPad
The table represents the results of 12 students in two tests.
Test 1 score 10 18 13 6 8 5 12 15 15
Test 2 score 12 20 11 9 6 6 12 13 17
Enter the data into list1 (x) and list2 (y) in the module. Tap SetGraph,
Setting . . . and select the tab for Graph3.
(Note: Following on from the types of graphs in
univariate statistics, this allows the Scatterplot
settings to be remembered and called upon when
required.)
Ensure that all other graphs are de-selected and
tap to produce the graph shown in the full
screen.
Select the graph window (bold border) and tap Analysis, Trace to scroll from point to
point and display the coordinates at the bottom of the graph.
Exercise 23A
Note: Save your data for 14 in named lists as they will be needed for later exercises.
1 The amount of a particular pain relief drug given to each patient and the time taken for the
patient to experience pain relief are shown.
Patient 1 2 3 4 5 6 7 8 9 10
Drug dose (mg) 0.5 1.2 4.0 5.3 2.6 3.7 5.1 1.7 0.3 4.0
Response time (min) 65 35 15 10 22 16 10 18 70 20
a Plot the response time against drug dose.
b From the scatterplot, describe any association between the two variables.
c Identify outliers, if any, and interpret.
2 The proprietor of a hairdressing salon recorded the amount spent advertising in the local
paper and the business income for each month of a year, with the following results.
P1: FXS/ABE P2: FXS
9780521740494c23.xml CUAU033-EVANS October 5, 2008 8:31
Chapter 23 Investigating the relationship between two numerical variables 557
Month Advertising ($) Business ($)
1 350 9 450
2 450 10 070
3 400 9 380
4 500 9 110
5 250 5 220
6 150 3 100
Month Advertising ($) Business ($)
7 350 8 060
8 300 7 030
9 550 11 500
10 600 12 870
11 550 10 560
12 450 9 850
a Plot the business income against the advertising expenditure.
b From the scatterplot, describe any association between the two variables.
c Identify outliers, if any, and interpret.
3 The number of passenger seats on the most commonly used commercial aircraft, and the
airspeeds of these aircraft, in km/h, are shown in the following table.
Number of seats 405 296 288 258 240 230 193 188
Airspeed (km/h) 830 797 774 736 757 765 760 718
Number of seats 148 142 131 122 115 112 103 102
Airspeed (km/h) 683 666 661 378 605 620 576 603
a Plot the airspeed against the number of seats.
b From the scatterplot, describe any association between the two variables.
c Identify outliers, if any, and interpret.
4 The price and age of several secondhand caravans are listed in the table.
Age (years) Price ($)
7 4 800
7 3 900
8 4 275
9 3 900
4 6 900
8 6 500
1 11 400
Age (years) Price ($)
10 8 700
9 1 950
9 3 300
11 1 650
3 9 600
4 8 400
7 6 600
a Plot the price of the caravans against their age.
b From the scatterplot, describe any association between the two variables.
c Identify outliers, if any, and interpret.
23.2 The q-correlation coefficient
If the plot of a bivariate data set shows a basic trend, apart from some randomness, then it is
useful to provide a numerical measure of the strength of the relationship between the two
variables. Correlation is a measure of strength of a relationship which applies only to
P1: FXS/ABE P2: FXS
9780521740494c23.xml CUAU033-EVANS October 5, 2008 8:31
558 Essential Advanced General Mathematics
numerical variables. Thus it is sensible, for example, to calculate the correlation between the
heights and weights for a group of students, but not between height and gender, as gender is
not a numerical variable. There are many different numerical measures of correlation, and each
has different properties. In this section the q-correlation coefcient will be introduced.
Consider the scatterplot of the number of hours spent by each member of a class when
studying for an examination, and the mark they were awarded, from Example 1. This shows a
positive association. To calculate the q-correlation coefcient, rst nd the median value for
each of the variables separately. This can be done from the data, but it is usually simpler to
calculate directly from the plot. There are 20 data points, and the median values are halfway
between the 10th and 11th points, both vertically and horizontally. A vertical line is then drawn
through the median x value, and a horizontal line through the median y value. The effect of this
is to divide the plot into four regions, as shown.
Hours x
0
20
20
40
40
60
80
10 30
Marks y
Each of the four regions which have been created in this way is called a quadrant, and it can
be noticed immediately that most of the points in this plot are in the upper right and lower left
quadrants. In fact, wherever there is a positive association between variables this will be the
case.
Consider the scatterplot of the age of cars and the advertised price from Example 2, which
shows negative association. Again the median value for each of the variables is found
separately. There are 15 data points, giving the median values at the 8th points, both vertically
and horizontally. A vertical line is then drawn through the median x value, and a horizontal line
through the median y value. In this particular case they are coordinates of the same point, but
this need not be so.
4 6 8 1 2 3 5 7
Age (years) x
8000
10000
12000
14000
16000
Price ($) y
P1: FXS/ABE P2: FXS
9780521740494c23.xml CUAU033-EVANS October 5, 2008 8:31
Chapter 23 Investigating the relationship between two numerical variables 559
It can be seen in this example that most points are in the upper left and the lower right
quadrants, and this is true whenever there is a negative association between variables.
These observations lead to a denition of the q-correlation coefcient.
The q-correlation coefcient can be determined from the scatterplot as follows.
x
y
0
A
B
C D
Find the median of all the
x values in the data set, and
draw a vertical line through
this value.
Find the median of all the
y values in the data set, and
draw a horizontal line through
this value.
The plane is now divided into four quadrants. Label the quadrants A, B, C and D as shown
in the diagram.
Count the number of points in each of the quadrants A, B, C and D. Any points which lie
on the median lines are omitted.
Let a, b, c, d represent the number of points in each of the quadrants A, B, C and D
respectively. Then the q-correlation coefcient is given by
q =
(a +c) (b +d)
a +b +c +d
Example 3
Use the scatterplot from Example 1 to determine the q-correlation coefcient for the number
of hours each member of a class spent studying for an examination and the mark they were
awarded.
Solution
There are nine points in quadrant A, one in quadrant B, nine in quadrant C and one in
quadrant D.
Thus q =
(a +c) (b +d)
a +b +c +d
=
(9 +9) (1 +1)
9 +1 +9 +1
=
18 2
20
=
16
20
= 0.8
P1: FXS/ABE P2: FXS
9780521740494c23.xml CUAU033-EVANS October 5, 2008 8:31
560 Essential Advanced General Mathematics
Example 4
Use the scatterplot from Example 2 to determine the q-correlation coefcient for the age of
cars and their advertised price.
Solution
There is one point in quadrant A, six in quadrant B, one in quadrant C and six in
quadrant D.
q =
(a +c) (b +d)
a +b +c +d
=
(1 +1) (6 +6)
1 +6 +1 +6
=
2 12
14
=
10
14
= 0.71
From Examples 3 and 4 it can be seen that q-correlation coefcients may take both positive
and negative values. Consider the situation when all the points are in the quadrants A and C.
q =
(a +c) (b +d)
a +b +c +d
=
a +c
a +c
(since b and d are both equal to zero)
= 1
Thus the maximum value the q-correlation coefcient may take is 1, and this indicates a
measure of strong positive association.
Suppose all the points are in the quadrants B and D.
q =
(a +c) (b +d)
a +b +c +d
=
(b +d)
b +d
(since a and c are both equal to zero)
= 1
Thus the minimum value the q-correlation coefcient may take is 1, and this indicates a
measure of strong negative association.
P1: FXS/ABE P2: FXS
9780521740494c23.xml CUAU033-EVANS October 5, 2008 8:31
Chapter 23 Investigating the relationship between two numerical variables 561
When the same number of points are in each of the quadrants A, B, C and D then:
q =
(a +c) (b +d)
a +b +c +d
=
0
a +b +c +d
(since a = b = c = d)
= 0
This value of the q-correlation coefcient clearly indicates that no association exists.
q-correlation coefcients can be classied as follows:
1 q 0.75 strong negative relationship
0.75 < q 0.50 moderate negative relationship
0.50 < q 0.25 weak negative relationship
0.25 < q < 0.25 no relationship
0.25 q < 0.50 weak positive relationship
0.50 q < 0.75 moderate positive relationship
0.75 q 1 strong positive relationship
Exercise 23B
1 Use the table of q-correlation coefcients to classify the following.
a q = 0.20 b q = 0.30 c q = 0.85 d q = 0.33
e q = 0.95 f q = 0.75 g q = 0.75 h q = 0.24
i q = 1 j q = 0.25 k q = 1 l q = 0.50
2 Calculate the q-correlation coefcient for each pair of variables shown in the following
scatterplots.
a y
x
36
24
12
15.0 20.0 25.0 30.0 35.0
P1: FXS/ABE P2: FXS
9780521740494c23.xml CUAU033-EVANS October 5, 2008 8:31
562 Essential Advanced General Mathematics
b
x
y
140
120 160 200 240 280
210
280
c
2
4
6
8
x 1 2
y
3 4 5 6 7 8
0
d y
x
0
20
20
40
40
60
80
10 30
3 The amount of a particular pain relief drug given to each patient and the time taken for the
Example 4
patient to experience pain relief are shown.
Patient 1 2 3 4 5 6 7 8 9 10
Drug dose (mg) 0.5 1.2 4.0 5.3 2.6 3.7 5.1 1.7 0.3 4.0
Response time (min) 65 35 15 10 22 16 10 18 70 20
a Use your scatterplot from 1, Exercise 23A to nd the q-correlation coefcient for
response time and drug dosage.
P1: FXS/ABE P2: FXS
9780521740494c23.xml CUAU033-EVANS October 5, 2008 8:31
Chapter 23 Investigating the relationship between two numerical variables 563
b Classify the strength and direction of the relationship between response time and drug
dosage according to the table given.
4 The proprietor of a hairdressing salon recorded the amount spent advertising in the local
Example 3
paper and the business income for each month of a year, with the following results.
Month Advertising ($) Business ($)
1 350 9 450
2 450 10 070
3 400 9 380
4 500 9 110
5 250 5 220
6 150 3 100
Month Advertising ($) Business ($)
7 350 8 060
8 300 7 030
9 550 11 500
10 600 12 870
11 550 10 560
12 450 9 850
a Use your scatterplot from 2, Exercise 23A to nd the q-correlation coefcient for
advertising expenditure and total business conducted.
b Classify the strength and direction of the relationship between advertising expenditure
and business income according to the table given.
5 The number of passenger seats on the most commonly used commercial aircraft, and the
airspeeds of these aircraft, in km/h, are shown in the following table.
Number of seats 405 296 288 258 240 230 193 188
Airspeed (km/h) 830 797 774 736 757 765 760 718
Number of seats 148 142 131 122 115 112 103 102
Airspeed (km/h) 683 666 661 378 605 620 576 603
a Use your scatterplot from 3, Exercise 23A to nd the q-correlation coefcient for the
number of seats on an airline and the air speed.
b Classify the strength and direction of the relationship between the number of seats on
an airline and the air speed according to the table given.
6 The price and age of several secondhand caravans are listed in the table.
Age (years) Price ($)
7 4 800
7 3 900
8 4 275
9 3 900
4 6 900
8 6 500
1 11 400
Age (years) Price ($)
10 8 700
9 1 950
9 3 300
11 1 650
3 9 600
4 8 400
7 6 600
P1: FXS/ABE P2: FXS
9780521740494c23.xml CUAU033-EVANS October 5, 2008 8:31
564 Essential Advanced General Mathematics
a Use your scatterplot from 4, Exercise 23A to nd the q-correlation coefcient for price
and age of secondhand caravans.
b Classify the strength and direction of the relationship between price and age of
secondhand caravans according to the table given.
23.3 The correlation coefficient
When a relationship is linear the most commonly used measure of strength of the relationship
is Pearsons product-moment correlation coefcient, r. It gives a numerical measure of the
degree to which the points in the scatterplot tend to cluster around a straight line.
Pearsons product-moment correlation is dened to be
r =
degree which the variables vary together
degree which the two variables vary separately
Formally, if we call the two variables x and y and we have n observations then Pearsons
product-moment correlation for this set of observations is
r =
1
n 1
n

i =1

x
i
x
s
x

y
i
y
s
y

where x and s
x
are the mean and standard deviation of the x scores and y and s
y
are the mean
and standard deviation of the y scores.
There are two key assumptions made in calculating Pearsons correlation coefcient r. They
are
the data is numerical
the relationship being described is linear.
Pearsons correlation coefcient r has the following properties:
If there is no linear relationship, r = 0.
x
y
0
r = 0
For a perfect positive linear relationship,
r = +1.
x
y
0 r = +1
For a perfect negative linear relationship,
r = 1.
x
y
0
r = 1
P1: FXS/ABE P2: FXS
9780521740494c23.xml CUAU033-EVANS October 5, 2008 8:31
Chapter 23 Investigating the relationship between two numerical variables 565
Otherwise, 1 r +1
Pearsons correlation coefcient r can be classied as follows:
1 r 0.75 strong negative linear relationship
0.75 r 0.50 moderate negative linear relationship
0.50 r 0.25 weak negative linear relationship
0.25 < r < 0.25 no linear relationship
0.25 r < 0.50 weak positive linear relationship
0.50 r < 0.75 moderate positive linear relationship
0.75 r 1 strong positive linear relationship
The following scatterplots show linear relationships of various strengths together with the
corresponding value of Pearsons product-moment correlation coefcient.
100 150 200 250 300 350 400
10
12
14
Traffic volume
C
O

l
e
v
e
l
600 800 1000 1200 1400
20
25
15
30
Testosterone level
A
g
e
Carbon monoxide level in the atmosphere
and trafc volume: r = 0.985
Age rst convicted and testosterone (a male
hormone) level of a group of prisoners:
r = 0.814
80 100 120
60
80
100
120
140
160
Smoking ratio
M
o
r
t
a
l
i
t
y
90
100
110
8 10 12 14 16 18 20
Age 1st word
S
c
o
r
e
Mortality rate due to lung cancer and
smoking ratio (100 average): r = 0.716
Score on aptitude test (taken later in life) and
age (in months) when rst word spoken:
r = 0.445
700
600
600 800 700
500
500
400
V
e
r
b
a
l
Mathematics
20
15
10
5
30 40 50
Age
C
a
l
f
Scores on standardised tests of verbal
and mathematical ability: r = 0.275
Calf measurement and age of adult males:
r = 0.005
P1: FXS/ABE P2: FXS
9780521740494c23.xml CUAU033-EVANS October 5, 2008 8:31
566 Essential Advanced General Mathematics
Using the TI-Nspire
The value Pearsons product-moment correlation coefcient, r, can be calculated using
the calculator. This will be illustrated using the age and price of car data from
Example 2.
With the data entered as the two lists age and
price respectively, we now open a Calculator
application ( 1) to calculate Pearsons
product-moment correlation coefcient.
Use Linear Regression (mx +b) from the
Stat Calculations submenu of the Statistics
menu (b613) and complete the
dialog box as shown.
Press enter to obtain the regression
information including the value for r as
shown.
Using the Casio ClassPad
Consider the following set of data
x 1 3 5 4 7
y 2 5 7 2 9
Enter the data into list1 (x) and list2 (y).
Tap Calc, Linear Reg and select the settings shown. Tap
OK to produce the results shown in the second screen.
The value of r is shown in the answer box.
Tap OK to produce a scatterplot.
P1: FXS/ABE P2: FXS
9780521740494c23.xml CUAU033-EVANS October 5, 2008 8:31
Chapter 23 Investigating the relationship between two numerical variables 567
After the mean and standard deviation, Pearsons product-moment correlation is one of the
most frequently computed descriptive statistics. It is a powerful tool but it is also easily
misused. The presence of a linear relationship should always be conrmed with a scatterplot
before Pearsons product-moment correlation is calculated. And, like the mean and the
standard deviation, Pearsons correlation coefcient r is very sensitive to the presence of
outliers in the sample.
Exercise 23C
1 Use the table of Pearsons correlation coefcients r to classify the following.
a r = 0.20 b r = 0.30 c r = 0.85 d r = 0.33
e r = 0.95 f r = 0.75 g r = 0.75 h r = 0.24
i r = 0.50 j r = 0.25 k r = 1 l r = 1
2 By comparing the plots given to those on page 538 estimate the value of Pearsons
correlation coefcient r.
a
y
x 15.0 20.0 25.0 30.0 35.0
24
12
36
b
y
x 160 200
280
210
140
280 120 240
c
x
y
1
0
2
4
6
8
2 3 7 8 6 4 5
d
x
y
80
60
40
20
10 20 30 40
0
e
x
y
1 3 5 4 6 7 2 8
8000
10000
12000
14000
f
x
y
6
4
2
1 2 3 4 5 6 7 8
8
0
P1: FXS/ABE P2: FXS
9780521740494c23.xml CUAU033-EVANS October 5, 2008 8:31
568 Essential Advanced General Mathematics
3 The amount of a particular pain relief drug given to each patient and the time taken for the
patient to experience relief are shown.
Patient 1 2 3 4 5 6 7 8 9 10
Drug dose (mg) 0.5 1.2 4.0 5.3 2.6 3.7 5.1 1.7 0.3 4.0
Response time (min) 65 35 15 10 22 16 10 18 70 20
a Determine the value of Pearsons correlation coefcient.
b Classify the relationship between drug dose and response time according to the table
given.
4 The proprietor of a hairdressing salon recorded the amount spent on advertising in the
local paper and the business income for each month for a year, with the following results.
Month Advertising ($) Business ($)
1 350 9 450
2 450 10 070
3 400 9 380
4 500 9 110
5 250 5 220
6 150 3 100
Month Advertising ($) Business ($)
7 350 8 060
8 300 7 030
9 550 11 500
10 600 12 870
11 550 10 560
12 450 9 850
a Determine the value of Pearsons correlation coefcient.
b Classify the relationship between the amount spent on advertising and business income
according to the table given.
5 The number of passenger seats on the most commonly used commercial aircraft, and the
airspeeds of these aircraft, in km/h, are shown in the following table.
Number of seats 405 296 288 258 240 230 193 188
Airspeed (km/h) 830 797 774 736 757 765 760 718
Number of seats 148 142 131 122 115 112 103 102
Airspeed (km/h) 683 666 661 378 605 620 576 603
a Determine the value of Pearsons correlation coefcient.
b Classify the relationship between the number of passenger seats and airspeed
according to the table given.
P1: FXS/ABE P2: FXS
9780521740494c23.xml CUAU033-EVANS October 5, 2008 8:31
Chapter 23 Investigating the relationship between two numerical variables 569
6 The price and age of several secondhand caravans are listed in the table.
Age (years) Price ($)
7 4 800
7 3 900
8 4 275
9 3 900
4 6 900
8 6 500
1 11 400
Age (years) Price ($)
10 8 700
9 1 950
9 3 300
11 1 650
3 9 600
4 8 400
7 6 600
a Determine the value of Pearsons correlation coefcient.
b Classify the relationship between price and age according to the table given.
7 The following are the scores for a group of ten students who each had two attempts at a
test (out of 70).
Attempt 1 53 56 57 49 44 69 66 40 53 43 68 64
Attempt 2 63 66 67 58 54 70 70 55 63 53 70 70
a Construct a scatterplot of these data, and describe the relationship between scores on
attempt 1 and attempt 2.
b Is it appropriate to calculate the value of Pearsons correlation coefcient for these
data? Give reasons for your answer.
8 This table represents the results of two
different tests for a group of students.
Student Test 1 Test 2
1 214 216
2 281 270
3 212 281
4 324 326
5 240 243
6 208 213
7 303 311
8 278 290
9 311 320
a Construct a scatterplot of these data, and
describe the relationship between scores on
Test 1 and Test 2.
b Is it appropriate to calculate the value of
Pearsons correlation coefcient for these
data? Give reasons for your answer.
c Determine the values of the q-correlation
coefcient and Pearsons correlation
coefcient r.
d Classify the relationship between Test 1 and Test 2 using both the q-correlation
coefcient and Pearsons correlation coefcient r, and compare.
e It turns out that when the data was entered into the student records, the result for
Test 2, Student 9 was entered as 32 instead of 320.
i Recalculate the values of the q-correlation coefcient and Pearsons correlation
coefcient r with this new data value.
ii Compare these values with the ones calculated in c.
P1: FXS/ABE P2: FXS
9780521740494c23.xml CUAU033-EVANS October 5, 2008 8:31
570 Essential Advanced General Mathematics
23.4 Lines on scatterplots
If a linear relationship exists between two variables it is possible to predict the value of the
dependent variable from the value of the independent variable. The stronger the relationship
between the two variables the better the prediction that is made. To make the prediction it is
necessary to determine an equation which relates the variables and this is achieved by tting a
line to the data. Fitting a line to data is often referred to as regression, which comes from a
Latin word regressum which means moved back.
The simplest equation relating two variables x and y is a linear equation of the form
y = a +bx
where a and b are constants. This is similar to the general equation of a straight line, where a
represents the coordinate of the point where the line crosses the y axis (the y axis intercept),
and b represents the slope of the line. In order to summarise any particular (x, y) data set,
numerical values for a and b are needed that will make the line pass close to the data. There are
several ways in which the values of a and b can be found, of which the simplest is to nd the
straight line by placing a ruler on the scatter diagram, and drawing a line by eye, which
appears to follow the general trend of the data.
Example 6
The following table gives the gold medal winning distance, in metres, for the mens long jump
for the Olympic games for the years 1896 to 1996. (Some years were missing owing to the two
world wars.)
Find a straight line which ts the general trend of the data, and use it to predict the winning
distance in the year 2008.
Year 1896 1900 1904 1908 1912 1920 1924 1928 1932 1936 1948 1952 1956
Distance (m) 6.35 7.19 7.34 7.49 7.59 7.16 7.44 7.75 7.65 8.05 7.82 7.57 7.82
Year 1960 1964 1968 1972 1976 1980 1984 1988 1992 1996 2000 2004
Distance (m) 8.13 8.08 8.92 8.26 8.36 8.53 8.53 8.72 8.67 8.50 8.55 8.59
Solution
D
i
s
t
a
n
c
e
Year
1900 1920 1940 1960 1980 2000
9.0
8.5
8.0
7.5
7.0
6.5
6.0
P1: FXS/ABE P2: FXS
9780521740494c23.xml CUAU033-EVANS October 5, 2008 8:31
Chapter 23 Investigating the relationship between two numerical variables 571
Note that this scatterplot does not start at the origin. Since the values of the
coordinates that are of interest on both axes are a long way from zero, it is sensible to
plot the graph for that range of values only. In fact, any values less than 1896 on the
horizontal axis are meaningless in this context.
The line shown on the scatterplot is only one of many which could be drawn. To
enable the line to be used for prediction it is necessary to nd its equation. To do this,
rst determine the coordinates of any two points through which it passes on the
scatterplot. Appropriate points are (1932, 7.65) and (1976, 8.36). The equation of the
straight line is then found by substituting in the formula which gives the equation for a
straight line between two points.
y y
1
x x
1
=
y
2
y
1
x
2
x
1
y 7.65
x 1932
=
8.36 7.65
1976 1932
=
0.71
44
= 0.016
y 7.65 = 0.016(x 1932)
y = 0.016x 23.26
or distance = 23.26 +0.016 year
The intercept for this equation is 23.26 m. In theory, this is the winning distance
for the year 0! In practice, there is no meaningful interpretation for the y axis intercept
in this situation. But the same cannot be said about the slope. A slope of 0.016 means
that on average the gold medal winning distance increases by about 1.6 centimetres at
each successive games.
Using this equation the gold medal winning distance for the long jump in 2008
would be predicted as
y = 23.26 +0.016 2008 = 8.87 m
Obviously, attempting to project too far into the future may give us answers which are not
sensible. When using an equation for prediction, derived from data, it is sensible to use values
of the explanatory variable which are within a reasonable range of the data. The relationship
between the variables may not be linear if we move too far from the known values.
P1: FXS/ABE P2: FXS
9780521740494c23.xml CUAU033-EVANS October 5, 2008 8:31
572 Essential Advanced General Mathematics
Example 7
The following table gives the alcohol consumption per head (in litres) and the hospital
admission rate to each of the regions of Victoria in 199495.
Per capita Hospital
consumption admissions per
Region (litres of alcohol) 1000 residents
LoddonMallee 9.0 42.0
Grampians 8.4 44.7
Barwon 8.7 38.6
Gippsland 9.1 44.7
Hume 10.0 41.0
Western Metropolitan 9.0 40.4
Northern Metropolitan 6.7 36.2
Eastern Metropolitan 6.2 32.3
Southern Metropolitan 8.1 43.0
Find a straight line which ts the general trend of the data, and interpret the intercept and slope.
Solution
Alcohol consumption
A
d
m
i
s
s
i
o
n
s
7.0 6.0 8.0 9.0
30.0
0
35.0
40.0
45.0
10.0
One possible line passes through the points (7, 36) and (9, 42).
Thus
y y
1
x x
1
=
y
2
y
1
x
2
x
1
y 36
x 7
=
42 36
9 7
=
6
2
= 3
y 36 = 3(x 7)
y = 3x +15
or admission rate = 15 +3 alcohol consumption
The intercept for this equation is 15, implying that we predict a hospital admission
rate would be 15 per 1000 residents for a region with 0 alcohol consumption. While
this is interpretable, it would be a brave prediction as it is well out of the range of the
P1: FXS/ABE P2: FXS
9780521740494c23.xml CUAU033-EVANS October 5, 2008 8:31
Chapter 23 Investigating the relationship between two numerical variables 573
data. A slope of 3 means that on average the admission rate rises by 3 per 1000
residents for each additional litre of alcohol consumed per capita.
Exercise 23D
1 Plot the following set of data points on graph paper.
Example 6
x 0 1 2 3 4 5 6 7 8
y 1 3 6 7 7 11 13 18 17
Draw a straight line which ts the data by eye, and nd an equation for this line.
2 Plot the following set of points on graph paper.
x 3 2 1 0 1 2 3 4
y 5 2 0 6 7 11 13 20
Draw a straight line which ts the data by eye, and nd an equation for this line.
3 The numbers of burglaries during two successive years for various districts in one state are
Example 7
given in the following table.
District Year 1 (x) Year 2 (y)
A 3233 2709
B 2363 2208
C 4591 3685
D 4317 4038
E 2474 2792
F 3679 3292
G 5016 4402
H 6234 5147
I 6350 5555
J 4072 4004
K 2137 1980
a Make a scatterplot of the data.
b Find the equation of a straight line which
relates the two variables.
c Describe the trend in burglaries in this state.
4 The following data give a girls height (in cm) between the ages of 36 months and
60 months.
Age (x) 36 40 44 52 56 60
Height (y) 84 87 90 92 94 96
a Make a scatterplot of the data.
b Find the equation of a straight line which relates the two variables.
c Interpret the intercept and slope, if appropriate.
d Use your equation to estimate the girls height at age
i 42 months ii 18 years
e How reliable are your answers to d?
P1: FXS/ABE P2: FXS
9780521740494c23.xml CUAU033-EVANS October 5, 2008 8:31
574 Essential Advanced General Mathematics
5 The following table gives the adult heights (in cm) of ten pairs of mothers and daughters.
Mother (x) 170 163 157 165 175 160 164 168 152 173
Daughter (y) 178 175 165 173 168 152 163 168 160 178
a Make a scatterplot of the data.
b Find the equation of a straight line which relates the two variables.
c Estimate the adult height of a girl whose mother is 170 cm tall.
6 The manager of a company which manufactures MP3 players keeps a weekly record of the
cost of running the business and the number of units produced. The gures for a period of
eight weeks are shown in the table.
Number of MP3 players produced (x) 100 160 80 100 220 150 170 200
Cost in 000s $ (y) 2.5 3.3 2.4 2.6 4.1 3.1 3.5 3.8
a Make a scatterplot of the data.
b Find the equation of a straight line which relates the two variables.
c What is the manufacturers xed cost for operating the business each week?
d What is the cost of production of each unit, over and above this xed operating cost?
7 The amount of a particular pain relief drug given to each patient and the time taken for the
patient to experience pain relief are shown.
Patient 1 2 3 4 5 6 7 8 9 10
Drug dose (mg) 0.5 1.2 4.0 5.3 2.6 3.7 5.1 1.7 0.3 4.0
Response time (min) 65 35 15 10 22 16 10 18 70 20
a Find the equation of a straight line which relates the two variables.
b Interpret the intercept and slope if appropriate.
c Use your equation to predict the time taken for the patient to experience pain relief if 6
mg of the drug is given. Is this answer realistic?
8 The proprietor of a hairdressing salon recorded
the amount spent on advertising in the local
paper and the business income for each
month for a year, with the results shown.
Month Advertising ($) Business ($)
1 350 9 450
2 450 10 070
3 400 9 380
4 500 9 110
5 250 5 220
6 150 3 100
7 350 8 060
8 300 7 030
9 550 11 500
10 600 12 870
11 550 10 560
12 450 9 850
a Find the equation of a straight line
which relates the two variables.
b Interpret the intercept and slope if
appropriate.
c Use your equation to predict the business
income which would be attracted if the
proprietor of the salon spent the following
amounts on advertising:
i $1000 ii $0
P1: FXS/ABE P2: FXS
9780521740494c23.xml CUAU033-EVANS October 5, 2008 8:31
Chapter 23 Investigating the relationship between two numerical variables 575
23.5 The least squares regression line
Fitting a line to a scatterplot by eye is not generally the best way of modelling a relationship.
What is required is a method which uses a more objective criterion. A simple method is two
divide the data set into two halves on the basis of the median x value, and to t a line which
passes through the mean x and y values of the lower half, and the mean x and y values of the
upper half. This is called the two-mean line, and while easy to determine, it is not very widely
used. The most common procedure is the method of least squares. The least squares
regression line is the line for which the sum of squares of the vertical deviations from the data
to the line (as indicated in the diagram) is a minimum. These deviations are called the
residuals.
y
x 10 5 7 9 6 8 4 3 2 1
35
30
20
25
15
5
10
0
(x
i
, y
i
)
y = a + bx
Consider the line y = a +bx
We would like to nd a and b such that the sum of the residuals is zero. That is,
n

i =1
(y
i
a bx
i
) = 0 1
and the sum of residuals square is as small as possible. That is,
n

i =1
(y
i
a bx
i
)
2
is a minimum 2
We will use the symbol S to denote
n

i =1
(y
i
a bx
i
)
2
From 1 ,
n

i =1
(y
i
a bx
i
) = 0

i =1
y
i
na b
n

i =1
x
i
= 0
y a b x = 0
a = y b x 3
P1: FXS/ABE P2: FXS
9780521740494c23.xml CUAU033-EVANS October 5, 2008 8:31
576 Essential Advanced General Mathematics
Substituting this relationship in 2
S =
n

i =1
[y
i
( y b x) bx
i
]
2
=
n

i =1
[(y
i
y) b(x
i
x)]
2
=
n

i =1
[(y
i
y)
2
2b(x
i
x)(y
i
y) +b
2
(x
i
x)
2
]
This can be thought of as a quadratic expression in b.
In order to nd the value of b which minimises S, we will differentiate with respect to b and
set the derivative equal to zero.
dS
db
= 2
n

i =1
(x
i
x)(y
i
y) +2b
n

i =1
(x
i
x)
2
= 0
Simplifying gives b =
n

i =1
(x
i
x)(y
i
y)
n

i =1
(x
i
x)
2
4
Equations 3 and 4 can then be used to calculate the least squares estimates of the y axis
intercept and the slope.
Using the TI-Nspire
The calculator can be used to construct the least squares regression line. The procedure
is illustrated using the age and price of car data from Example 2.
It has previously been illustrated how to
enter the data as two lists, age and price
respectively, and from these construct the
scatterplot in a Data & Statistics
application resulting in the data displayed
as shown.
P1: FXS/ABE P2: FXS
9780521740494c23.xml CUAU033-EVANS October 5, 2008 8:31
Chapter 23 Investigating the relationship between two numerical variables 577
Now use Show Linear (mx +b) from the
Regression submenu of the Analyze menu
(b461) to place the regression
line on the scatterplot as shown.
Using the Casio ClassPad
The following data gives the heights (in cm) and weights (in kg) of 11 people.
Height (x) 177 182 167 178 173 184 162 169 164 170 180
Weight (y) 74 75 62 63 64 74 57 55 56 68 72
Enter the data into list1 (x) and list2 (y).
Tap Calc, Linear Reg and select the settings
shown. Tap OK to produce the results
shown in the second screen. The format of
the formula, y = ax +b is shown at the
top and the values of a, b are shown in the
answer box.
Tap OK to produce a scatterplot
showing the regression line.
Note: The formula can be automatically copied into a selected entry line if
desired by selecting a graph number, e.g. y1 in the Copy Formula line.
After the equation of the least squares line has been determined, we can interpret the
intercept and slope in terms of the problem at hand, and use the equation to make predictions.
The method of least squares is also sensitive to any outliers in the data.
P1: FXS/ABE P2: FXS
9780521740494c23.xml CUAU033-EVANS October 5, 2008 8:31
578 Essential Advanced General Mathematics
Example 8
Consider again the gold medal winning distance, in metres, for the mens long jump for
the Olympic games for the years 1896 to 2004.
Find the equation of the least squares regression line for these data, and use it to predict
the winning distance for the year 2008.
Year 1896 1900 1904 1908 1912 1920 1924 1928 1932 1936 1948 1952 1956
Distance (m) 6.35 7.19 7.34 7.49 7.59 7.16 7.44 7.75 7.65 8.05 7.82 7.57 7.82
Year 1960 1964 1968 1972 1976 1980 1984 1988 1992 1996 2000 2004
Distance (m) 8.13 8.08 8.92 8.26 8.36 8.53 8.53 8.72 8.67 8.50 8.55 8.59
Solution
Using a calculator or computer the equation is found to be
distance = 23.87 +0.0163 year
which is quite similar to the equation to the line tted by eye.
The predicted distance for the year 2008 is
distance = 23.87 +0.0163 2008 = 8.86 m
Example 9
Consider again the data from Example 7 which related alcohol consumption per head (in litres)
and the hospital admission rate to each of the regions of Victoria in 199495.
Per capita consumption Hospital admissions
Region (litres of alcohol) per 1000 residents
LoddonMallee 9.0 42.0
Grampians 8.4 44.7
Barwon 8.7 38.6
Gippsland 9.1 44.7
Hume 10.0 41.0
Western Metropolitan 9.0 40.4
Northern Metropolitan 6.7 36.2
Eastern Metropolitan 6.2 32.3
Southern Metropolitan 8.1 43.0
Find the equation of the least squares regression line which ts these data.
Solution
Using a calculator or computer the equation is found to be
admissions = 19.9 +2.45 alcohol
which is slightly different from the line tted by eye.
P1: FXS/ABE P2: FXS
9780521740494c23.xml CUAU033-EVANS October 5, 2008 8:31
Chapter 23 Investigating the relationship between two numerical variables 579
Correlation and causation
The existence of even a strong linear relationship between two variables is not, in itself,
sufcient to imply that altering one variable causes a change in the other. It only implies that
this might be the explanation. It may be that both the measured variables are affected by a third
and different variable. For example, if data about crime rates and unemployment in a range of
cities were gathered, a high correlation would be found. But could it be inferred that high
unemployment causes a high crime rate? The explanation could be that both of these variables
are dependent on other factors, such as home circumstances, peer group pressure, level of
education or economic conditions, all of which may be related to both unemployment and
crime rates. These two variables may vary together, without one being the direct cause of the
other.
Exercise 23E
1 The following data give a girls height (in cm) between the ages of 36 months and
Example 8
60 months.
Age (x) 36 40 44 52 56 60
Height (y) 84 87 90 92 94 96
a Using the method of least squares nd the equation of a straight line which relates the
two variables.
b Interpret the intercept and slope, if appropriate.
c Use your equation to estimate the girls height at age
i 42 months ii 18 years
d How reliable are your answers to part c?
2 The number of burglaries during two
successive years for various districts in
one state are given in the following table.
District Year 1 (x) Year 2 (y)
A 3 233 2 709
B 2 363 2 208
C 4 591 3 685
D 4 317 4 038
E 2 474 2 792
F 3 679 3 292
G 5 016 4 402
H 6 234 5 147
I 6 350 5 555
J 4 072 4 004
K 2 137 1 980
Using the method of least squares nd
the equation of a straight line which
relates the two variables.
P1: FXS/ABE P2: FXS
9780521740494c23.xml CUAU033-EVANS October 5, 2008 8:31
580 Essential Advanced General Mathematics
3 The following table gives the adult heights (in cm) of ten pairs of mothers and daughters.
Example 9
Mother (x) 170 163 157 165 175 160 164 168 152 173
Daughter (y) 178 175 165 173 168 152 163 168 160 178
a Using the method of least squares nd the equation of a straight line which relates the
two variables.
b Interpret the slope in this context.
c Estimate the adult height of a girl whose mother is 170 cm tall.
4 The manager of a company which manufactures MP3 players keeps a weekly record of the
cost of running the business and the number of units produced. The gures for a period of
eight weeks are:
Number of MP3
players produced (x) 100 160 80 100 220 150 170 200
Cost in 000s $ (y) 2.5 3.3 2.4 2.6 4.1 3.1 3.5 3.8
a Using the method of least squares nd the equation of a straight line which relates the
two variables.
b What is the manufactures xed cost for operating the business each week?
c What is the cost of production of each unit, over and above this xed operating cost?
5 The amount of a particular pain relief drug given to each patient and the time taken for the
patient to experience pain relief are shown.
Patient 1 2 3 4 5 6 7 8 9 10
Drug dose (mg) 0.5 1.2 4.0 5.3 2.6 3.7 5.1 1.7 0.3 4.0
Response time (min) 65 35 15 10 22 16 10 18 70 20
a Using the method of least squares nd the equation of a straight line which relates the
two variables.
b Interpret the intercept and slope if appropriate.
c Use your equation to predict the time taken for the patient to experience pain relief
if 6 mg of the drug is given. Is this answer realistic?
P1: FXS/ABE P2: FXS
9780521740494c23.xml CUAU033-EVANS October 5, 2008 8:31
Chapter 23 Investigating the relationship between two numerical variables 581
6 The proprietor of a hairdressing salon
recorded the amount spent on advertising
in the local paper and the business income
for each month for a year, with the
results shown.
Month Advertising ($) Business ($)
1 350 9 450
2 450 10 070
3 400 9 380
4 500 9 110
5 250 5 220
6 150 3 100
7 350 8 060
8 300 7 030
9 550 11 500
10 600 12 870
11 550 10 560
12 450 9 850
a Using the method of least squares nd
the equation of a straight line which
relates the two variables.
b Interpret the intercept and slope if
appropriate.
c Use your equation to predict the volume which would be attracted if the proprietor of
the salon spent the following amounts on advertising.
i $1000 ii $0
P1: FXS/ABE P2: FXS
9780521740494c23.xml CUAU033-EVANS October 5, 2008 8:31
R
e
v
i
e
w
582 Essential Advanced General Mathematics
Chapter summary
Bivariate data arises when measurements on two variables are collected for each subject.
A scatterplot is an appropriate visual display of bivariate data if both of the variables are
numerical.
A scatterplot of the data should always be constructed to assist in the identication of
outliers and illustrate the association (positive, negative or none).
Two variables are positively associated when larger values of y are associated with larger
values of x. Two variables are negatively associated when larger values of y are associated
with smaller values of x. There is no association between two variables when the values of
y are not related to the values of x.
When constructing the scatterplot, the independent or explanatory variable is plotted on the
horizontal (x) axis, and the dependent or response variable is plotted on the vertical (y) axis.
If a linear relationship is indicated by the scatterplot a measure of its strength can be found
by calculating the q-correlation coefcient, or Pearsons product-moment correlation
coefcient, r.
If the values on a scatterplot are divided by lines representing the median of x and the
median of y into four quadrants A, B, C and D, with a, b, c, d representing the number of
points in each quadrant respectively, then the q-correlation coefcient is given by
q =
(a +c) (b +d)
a +b +c +d
Pearsons product-moment correlation, r, is a measure of strength of linear relationship
between two variables, x and y. If we have n observations then for this set of observations
r =
1
n 1
n

i =1

x
i
x
s
x

y
i
y
s
y

where x and s
x
are the mean and standard deviation of the x scores and y and s
y
are the
mean and standard deviation of the y scores.
For these correlation coefcients
1 q 1
1 r 1
with values close to 1 indicating strong correlation, and those close to 0 indicating little
correlation.
If a linear relationship is indicated from the scatterplot a straight line may be tted to the
data, either by eye or using the least squares regression method.
The least squares regression line is the line for which the sum of squares of the vertical
deviations from the data to the line is a minimum.
The value of the slope (b) gives the extent of the change in the dependent variable
associated with a unit change in the independent variable.
Once found, the equation to the straight line may be used to predict values of the response
variable (y) from the explanatory variable (x). The accuracy of the prediction depends on
how closely the straight line ts the data, and an indication of this can be obtained from the
correlation coefcient.
P1: FXS/ABE P2: FXS
9780521740494c23.xml CUAU033-EVANS October 5, 2008 8:31
R
e
v
i
e
w
Chapter 23 Investigating the relationship between two numerical variables 583
Multiple-choice questions
1 For which one of the following pairs of variables would it be appropriate to construct a
scatterplot?
A eye colour (blue, green, brown, other) and hair colour (black, brown, blonde, red, other)
B score out of 100 on a test for a group of Year 9 students and a group of Year 11 students
C political party preference (Labor, Liberal, Other) and age in years
D age in years and blood pressure in mm Hg
E height in cm and gender (male, female)
2 For which one of the following plots would it be appropriate to calculate the value of the
q-correlation coefcient?
A B C
D E
3 A q-correlation coefcient of 0.32 would describe a relationship classied as
A weak positive B moderate positive C strong positive
D close to zero E moderately strong
4 The scatterplot shows the relationship between age and the number of alcoholic drinks
consumed on the weekend by a group of people.
The value of the q-correlation coefcient is closest to
A 1 B
7
9
C
5
6
D
7
9
E 1
0
5
10
15
20
10 20 30 40 50 60 70
Age
N
o
.

o
f

d
r
i
n
k
s
P1: FXS/ABE P2: FXS
9780521740494c23.xml CUAU033-EVANS October 5, 2008 8:31
R
e
v
i
e
w
584 Essential Advanced General Mathematics
5 The following scatterplot shows the relationship between height and weight for a group of
people.
The value of the Pearsons product-moment correlation coefcient r is closest to
A 1 B 0.8 C 0.5 D 0.3 E 0
180
160
120
180
160
140
100
140 190
220
80
200
200 150 170
W
e
i
g
h
t
Height (cm)
Questions 6 and 7 relate to the following information.
The weekly income and weekly expenditure on food for a group of 10 university students is
given in the following table.
Weekly
income ($) 150 250 300 600 300 380 950 450 850 1000
Weekly food
expenditure ($) 40 60 70 120 130 150 200 260 460 600
6 The value of the Pearson product-moment correlation coefcient r for these data is closest
to
A 0.2 B 0.4 C 0.6 D 0.7 E 0.8
7 The least squares regression line which would enable expenditure on food to be predicted
from weekly income is closest to
A 0.482 +42.864 weekly income B 0.482 42.864 weekly income
C 42.864 +0.482 weekly income D 239.868 +1.355 weekly income
E 1.355 +239.868 weekly income
P1: FXS/ABE P2: FXS
9780521740494c23.xml CUAU033-EVANS October 5, 2008 8:31
R
e
v
i
e
w
Chapter 23 Investigating the relationship between two numerical variables 585
Questions 8 and 9 relate to the following information.
Suppose that the least squares regression line which would enable expenditure on
entertainment (in dollars) to be predicted from weekly income is given by
Weekly expenditure on entertainment = 40 +0.10 Weekly income
8 Using this rule the expenditure on entertainment by an individual with an income of $600
per week is predicted to be
A $40 B $24 060 C $100 D $46 E $240
9 From this rule which of the following statements is correct?
A On average for each extra dollar of income an extra 10 cents is spent on entertainment
B On average for each extra 10 cents in income an extra $1 is spent on entertainment
C On average for each extra dollar of income an extra 40 cents is spent on entertainment
D On average people spend $40 per week on entertainment
E On average people spend $50 per week on entertainment
10 For the scatterplot shown the line of best t would
have a slope closest to:
16 10 12 14
150
18
50
22
100
200
250
20
A 0.1 B 0.1 C 10
D 10 E 200
Short-answer questions
Technology is required to answer some of the following questions.
1 The following table gives the number of times
the ball was inside the 50 metre line in an AFL
football game, and the teams score in that game
Inside 50 Score (points)
64 90
57 134
34 76
61 92
51 93
52 45
53 120
51 66
64 105
55 108
58 88
71 133
a Plot the score against the number of Inside 50s.
b From the scatterplot, describe any association
between the two variables.
2 Use the scatterplot constructed in 1 to determine
q-correlation between the score and the number of
Inside 50s.
P1: FXS/ABE P2: FXS
9780521740494c23.xml CUAU033-EVANS October 5, 2008 8:31
R
e
v
i
e
w
586 Essential Advanced General Mathematics
3 The distance traveled to work and the time taken for a group of company employees are
given in the following table. Determine the value of the Pearson product-moment
correlation r for these data.
Distance (kms) 12 50 40 25 45 20 10 3 10 30
Time (mins) 15 75 50 50 80 50 10 5 10 35
4 The following scatterplot shows the relationship between height and weight for a group of
people. Draw a straight line which ts the data by eye, and nd an equation for this line.
160
180
200
220
170 160 180 190 200 150
140
140
120
80
100
height (cm)
w
e
i
g
h
t
5 The time taken to complete a task, and the number of
errors on the task, were recorded for a sample of 10
primary school children. Determine the equation of
the least squares regression line which ts these data.
Time (seconds) Errors
22.6 2
21.7 3
21.7 3
21.3 4
19.3 5
17.6 5
17.0 7
14.6 7
14.0 9
8.8 9
6 For the data in 5:
a Interpret the intercept and slope of the least
squares regression line.
b Use the least squares regression line to predict
the number of errors which would be observed
for a child who took 10 seconds to complete the
task.
P1: FXS/ABE P2: FXS
9780521740494c23.xml CUAU033-EVANS October 5, 2008 8:31
R
e
v
i
e
w
Chapter 23 Investigating the relationship between two numerical variables 587
Extended-response questions
1 A marketing company wishes to predict the likely number of new clients each of its
graduates will attract to the business in their rst year of employment, by using their scores
on a marketing exam in the nal year of their course.
Number of new
Exam score clients
65 7
72 9
68 8
85 10
74 10
61 8
60 6
78 10
70 5
82 11
a Which is the independent variable
and which is the dependent variable?
b Construct a scatterplot of these data.
c Describe the association between the
Number of new clients and Exam
score.
d Determine the value of the q-correlation
coefcient for these data, and classify
the strength of the relationship.
e Determine the value of the Pearson
product-moment correlation coefcient
for these data and classify the strength
of the relationship.
f Determine the equation for the least squares regression line and write it down in terms of
the variables Number of new clients and Exam score.
g Interpret the intercept and slope of the least squares regression line in terms of the
variables in the study.
h Use your regression equation to predict to the nearest whole number the Number of new
clients for a person who scored 100 on the exam.
i How reliable is the prediction made in h?
2 To investigate the relationship between marks on an assignment and the nal examination
mark a sample of 10 students was taken. The table indicates the marks for the assignment
and the nal exam mark for each individual student.
Assignment mark Final exam mark
(max = 80) (max = 90)
80 83
77 83
71 79
78 75
65 68
80 84
68 71
64 69
50 66
66 58
a Which is the independent variable
and which is the dependent variable?
b Construct a scatterplot of these data.
c Describe the association between the
assignment mark and exam mark.
d Determine the value of the q-correlation
coefcient for these data, and classify the
strength of the relationship.
e Determine the value of the Pearson
product-moment correlation coefcient
for these data and classify the strength of
the relationship.
P1: FXS/ABE P2: FXS
9780521740494c23.xml CUAU033-EVANS October 5, 2008 8:31
R
e
v
i
e
w
588 Essential Advanced General Mathematics
f Use your answer to d to comment on the statement: Good nal exam marks are the
result of good assignment marks.
g Determine the equation for the least squares regression line and write it down in terms
of the variables Final exam mark and Assignment mark.
h Interpret the intercept and slope of the least squares regression line in terms of the
variables in the study.
i Use your regression equation to predict the Final exam mark for a student who scored 50
on the assignment.
j How reliable is the prediction made in i?
3 A marketing rm wanted to investigate the relationship between airplay and CD sales (in the
following week) of newly released songs. Data was collected on a randomsample of 10 songs.
No. of times the Weekly sales
song was played of the CD
47 3950
34 2500
40 3700
34 2800
33 2900
50 3750
28 2300
53 4400
25 2200
46 3400
a Which is the independent variable and which
is the dependent variable?
b Construct a scatterplot of these data.
c Describe the association between the number
of times the song was played and weekly sales.
d Determine the value of the q-correlation
coefcient for these data, and classify the strength
of the relationship.
e Determine the value of the Pearson
product-moment correlation coefcient for these
data and classify the strength of the relationship.
f Determine the equation for the least squares
regression line and write it down in terms of the
variables Number of times the song was played and Weekly sales.
g Interpret the intercept and slope of the least squares regression line in terms of the
variables in the study.
h Use your regression equation to predict the weekly sales for a song which was played 60
times.
i How reliable is the prediction made in h?
P1: FXS/ABE P2: FXS
9780521740494c24.xml CUAU033-EVANS October 5, 2008 8:35
C H A P T E R
Revision
24
Revision of chapters
22 and 23
24.1 Multiple-choice questions
1 For which of the following variables is a bar chart an appropriate display?
A weight (kg) B maximum daily temperature (C)
C exam results (A, B, C, D, E) D distance travelled to school each day (km)
E time taken to travel to school (mins)
2 For which of the following variables is a histogram an appropriate display?
A weight (kg) B religious afliation C exam results (A, B, C, D, E)
D weight (light, medium, heavy) E gender (male, female)
3 The shape of the distribution shown in this
histogram is best described as
A negatively skewed B symmetric
C positively skewed
D negatively skewed with an outlier
E positively skewed with an outlier
0
1
2
3
4
5
6
7
8
9
10
1 2 3 4 5 6 7 8 9 10 11 12 13 14 15
4 The following histogram shows the distance
travelled to school by a group of students
(in kilometres).
From the histogram we can say:
A Most students travel from 10 to less than
15 kilometres to school.
B Most students travel from 15 to less than
20 kilometres to school.
5
0
1
2
3
4
5
6
7
8
9
10
10 15 20 25 30 35 40 45 50
Distance (km)
Count
C Most students travel less than 15 kilometres to school.
D Most students travel less than 20 kilometres to school.
E Most students travel more than 20 kilometres to school.
589
P1: FXS/ABE P2: FXS
9780521740494c24.xml CUAU033-EVANS October 5, 2008 8:35
R
e
v
i
s
i
o
n
590 Essential Advanced General Mathematics
Questions 58 refer to the following information.
A class of students scored marks on a mathematics test as shown in the stem-and-leaf plot.
2 9
3 3 8
4 0 1 4 9
5 0 3 5 9
6 0 3 3 5 7 7
7 2 3 4 4 4 7 9
8 3 4 5 6
9 0 3
5 The number of students in the class is
A 8 B 26 C 30 D 38 E 40
6 If the pass mark for the test is a score of 50, the number of students who failed is
A 6 B 7 C 8 D 10 E 11
7 The teacher decides to award the top 10% of students on this test an A. The lowest mark
to score an A is thus
A 74 B 77 C 85 D 86 E 90
8 From the stem-and-leaf plot we can say that the shape of the distribution of test scores is
A positively skewed B symmetric C negatively skewed
D symmetrically skewed E unable to determine from the information available.
9 The following data are the driving-test scores for a group of people before and after they
completed an advanced driving course.
before after
7 2 1 5 9
9 9 8 6 6 6 5 4 3 2 2 1 0 2 1 4 5 5 6 8 9
4 1 0 0 3 0 0 2 3 4 4 5 6
6 4 3
5 0 4
From the stem-and-leaf plots we could say:
A The course has no effect on driving scores.
B Driving scores have in general increased after the course.
C Driving scores have in general decreased after the course.
D The course has increased some peoples score but not others.
E We are unable to determine any effect of the course from the information available.
P1: FXS/ABE P2: FXS
9780521740494c24.xml CUAU033-EVANS October 5, 2008 8:35
R
e
v
i
s
i
o
n
Chapter 24 Revision of chapters 22 and 23 591
10 The following stem-and-leaf plot shows the resting pulse rates for two groups of people:
Group 1, who exercise occasionally, and Group 2, who exercise regularly.
Group 1 Group 2
8 4
4 4 5 4
8 8 7 5 5 8 8
4 4 4 2 2 1 0 0 0 0 6 0 0 0 0 1 2 2 2 2 2 2 4 4 4 4
8 8 8 8 8 8 8 8 6 6 6 6 5 6 6 6 6 6 6 8 8 8 8 8 8 8 8 8 8 8
4 4 3 3 3 3 2 2 2 2 2 0 0 7 0 0 0 0 0 0 2 2 2 2 4 4 4 4 4
9 9 9 8 8 8 7 7 6 7 6 6 6 6 6 8 8 8 8 8
4 4 4 3 3 2 2 1 8 0 0 0 2 4 4
8 7 7 7 6 6 8 6 8
4 4 2 2 9 0 2 4
6 6 9
1 10
From the stem-and-leaf plots we could say:
A There does not seem to be any relationship between resting pulse rate and the amount
of exercise undertaken.
B The pulse rates of those who exercise regularly are in general lower than those who
exercise only occasionally.
C The pulse rates of those who exercise regularly are in general higher than those who
exercise only occasionally.
D The pulse rates of those who exercise regularly are in general less variable than those
who exercise only occasionally.
E The pulse rates of those who exercise regularly are in general more variable than
those who exercise only occasionally.
Questions 1114 relate to the following information.
An absent-minded professor calculated the following statistics for an examination.
mean = 50 range = 50 number of cases = 99
minium = 20 maximum = 70 median = 50
standard deviation = 12.0
He then found an additional exam with a score of 50, and recalculated the statistics.
11 The new value of the median is
A 21 B 50 C 51 D 71 E 100
12 The new value of the range is
A 21 B 50 C 51 D 71 E 100
13 The new value of the mean is
A 20 B 50 C 51 D 70 E 100
P1: FXS/ABE P2: FXS
9780521740494c24.xml CUAU033-EVANS October 5, 2008 8:35
R
e
v
i
s
i
o
n
592 Essential Advanced General Mathematics
14 The new value of the standard deviation is
A 11.0 B 11.9 C 12.0 D 13.1 E 13.0
15 The boxplots shown summarise the test scores for two classes of students, Class X and
Class Y.
25 50 75 100
Class X*
Class Y
*
From the boxplots we can say that, in general,
A Class X scored better than Class Y. B Class Y scored better than Class X.
C Class X scores are more variable than Class Y scores.
D Class Y scores are more variable than Class X scores.
E The scores for the two classes are about the same.
16 The median age of a group of subjects is 33 and the mean is 36. From this we can say that
the shape of the age distribution
A may be symmetric B may be positively skewed C may be negatively skewed
D has no outliers E may be symmetric with a very low outlier.
17 From the boxplot shown we can conclude:
A The distribution is symmetric.
B The distribution is positively skewed.
C The distribution is negatively skewed.
D Nothing can be determined about the skewness of a distribution from the boxplot.
E The distribution is symmetric with an outlier.
18 For the distribution shown in the following histogram we could say:
A The mode is larger than the mean.
B The mode is larger than the median.
C The mean and the median are about the same.
D The mean is smaller than the median.
E The mean is larger than the median.
P1: FXS/ABE P2: FXS
9780521740494c24.xml CUAU033-EVANS October 5, 2008 8:35
R
e
v
i
s
i
o
n
Chapter 24 Revision of chapters 22 and 23 593
19 Select which of the histograms A to E best corresponds to the boxplot given below.
A
B C
D E
20 Suppose a woman is on the 3rd quartile for height. This means that
A about 75% of women are taller than her
B about 75% of women are shorter than her
C about 50% of women are taller than her
D about 50% of women are shorter than her
E about 25% of women are shorter than her.
21 For which one of the following pairs of variables would it be appropriate to construct a
scatterplot?
A gender (male or female) and height (in cm)
B height (in cm) and weight (in kg) for a class of students
C attitude to abortion (for, against, no opinion) and age (in years)
D test scores for a group of male students and a group of female students
E religious afliation and age (in years).
22 For which one of the following scatterplots would it not be informative to calculate the
q-correlation coefcient?
A
10 5
0
5 10
200
150
100
50
B
100
80
60
50 60 70 80 90 100
C
80 100 120 140
20
25
30
35
D
0
0.2 0.4 0.6 0.8
0.2
0.4
0.6
0.8
P1: FXS/ABE P2: FXS
9780521740494c24.xml CUAU033-EVANS October 5, 2008 8:35
R
e
v
i
s
i
o
n
594 Essential Advanced General Mathematics
23 The q-correlation for the scatterplot shown
here is closest to
A 1 B 0.8 C 0
D 0.8 E 1
x
y
0
2 4 6 8 10
10
20
24 For the scatterplot shown,
the line of best t would
have equation closest to
A y = x
B y = 2x
C y = 2x +10
D y = x +10
E y = x +8
4
8
12
16
20
24
28
y
1 2 3 4 5 6 7 8 9 10
x
25 The Pearson correlation coefcient r relating the number of cigarettes smoked per capita
and the rate of heart disease in the population for several countries is 0.7. We may
interpret this as meaning:
A 70% of the people who smoke will get heart disease.
B Smoking causes heart disease.
C Countries with high smoking rates also tend to have high rates of heart disease.
D Countries with high smoking rates also tend to have low rates of heart disease.
E The probability of a smoker getting lung cancer is 70% more than that of a
non-smoker.
26 Jane calculates the Pearson correlation coefcient r
for the data shown in the scatter diagram as 0.35.
We may best interpret this by saying
A There is only a weak relationship between
x and y.
B There is obviously a relationship between
x and y, so Janes calculation must be wrong.
C A relationship exists between x and y but it is not linear.
D Knowing x does not tell us much about the corresponding value of y.
E There is a positive association between x and y.
x
y
4 8 12 16 20
50
100
27 If the correlation between x and y is negative then which of the following statements must
be true?
A y = x
B The values of y are in general more than the corresponding values of x.
C As the values of x decrease the values of y tend to increase.
D As the values of x increase the values of y tend to increase.
E The values of y are in general less than the corresponding values of x.
P1: FXS/ABE P2: FXS
9780521740494c24.xml CUAU033-EVANS October 5, 2008 8:35
R
e
v
i
s
i
o
n
Chapter 24 Revision of chapters 22 and 23 595
28 For the scatterplot shown,
the line of best t would
have equation closest to
A y = x
B y = x
C y = x +70
D y = x 70
E y = x +70
98
94
90
86
82
78
74
70
70 74 78 82 86 90 94 98
x
y
29 The production costs ($C) of a company which manufactures a particular computer game
is related to the number of games produced (N) as follows.
C = 12 000 +35 N
From this equation we can say, on average,
A cost of production increases at a rate of $12 000 per unit
B cost of production increases at a rate of $N per unit
C cost of production increases at a rate of $35 per unit
D cost of production is xed at $12 000
E cost of production decreases as the number of units produced increases
30 In calculating the least squares regression line, we choose the line which minimises
A the sum of the shortest distances from each point on the scatterplot to the line
B the sum of the horizontal distances from each point on the scatterplot to the line
C the sum of the squared horizontal distances from each point on the scatterplot to
the line
D the sum of the squared vertical distances from each point on the scatterplot to the line
E the sum of the squared perpendicular distances from each point on the scatterplot to
the line.
24.2 Extended-response questions
1 A researcher believes that the manner in which
an individual is approached by another (i.e. in
an aggressive or passive attitude) inuences
the emotional reaction of the person approached.
A researcher approached each of 15 subjects
on two separate occasions; once in an aggressive
manner and once in a passive manner. He
observed the distance he was permitted to
approach the subject before he or she stepped
back. This distance of approach was recorded in
centimetres for the 15 subjects.
Manner of approach
Subject Aggressive Passive
1 33 13
2 43 30
3 25 25
4 39 15
5 55 30
6 30 27
7 45 18
8 37 33
9 30 25
10 43 15
11 58 18
12 45 22
13 45 22
14 43 17
15 60 13
a Make ordered back to back stem-and-leaf
plots for the each of the approaches.
b Give the ve-gure summaries for
each of the approaches.
P1: FXS/ABE P2: FXS
9780521740494c24.xml CUAU033-EVANS October 5, 2008 8:35
R
e
v
i
s
i
o
n
596 Essential Advanced General Mathematics
c Draw boxplots for each of the approaches on the same scale.
d Calculate the mean and standard deviation for each of the groups.
e Compare the distributions of aggressive and passive approach distances.
2 To test the effect of alcohol on coordination twenty randomly selected participants were
timed to complete a task with both 0% blood alcohol and 0.05% blood alcohol. The times
taken are shown in the accompanying table.
0%blood alcohol
38 36 35 35 43 46 42 64 40 48
35 34 40 44 30 25 39 31 29 44
0.05%blood alcohol
39 32 35 39 36 34 41 56 44 38
43 42 46 46 50 32 32 41 40 50
a Make ordered back to back stem-and-leaf plots for the each of the sets of scores.
b Give the ve-gure summaries for each of the sets of scores.
c Draw boxplots for each of the sets of scores on the same scale.
d Calculate the mean and standard deviation for each of the sets of scores.
e Compare the distributions of the before and after scores.
3 The prices of several cars when new and when two years old are shown in the following
tables.
New price
10 817 16 664 12 477 15 450 13 239 11 585 16 391
17 200 11 490 15 070 16 660 12 049 15 424 13 410
18 050 17 999 10 700 14 955 11 595
Secondhand price
9 950 15 850 11 900 14 600 12 550 10 650 15 750
16 600 11 550 13 650 15 700 10 900 14 950 11 750
17 250 14 650 10 450 15 900 11 850
a Make ordered back to back stem-and-leaf for the each of the sets of car prices.
b Give the ve-gure summaries for each of the sets of car prices.
c Draw boxplots for each of the sets of car prices on the same scale.
d Calculate the mean and standard deviation of each of the sets of car prices.
e Compare the distributions of new prices and two year old prices.
4 A study was conducted to determine the effect
of choice on performance on student assignments.
A random sample of students were allowed to
choose their assignment topics from a long list
of possibilities. Another random sample of
students were given the same assignments but
without any choice of topic.
The marks, out of 50, obtained by the students
are given in the table.
(contd)
No choice Choice
44 36 36 36
38 35 26 30
26 42 40 35
21 17 35 25
44 24 48 36
49 48 50 25
28 42 44 30
35 50
P1: FXS/ABE P2: FXS
9780521740494c24.xml CUAU033-EVANS October 5, 2008 8:35
R
e
v
i
s
i
o
n
Chapter 24 Revision of chapters 22 and 23 597
a Make ordered back to back stem-and-leaf for the each of the sets of marks.
b Give the ve-gure summaries for each of the sets of marks.
c Draw boxplots for each of the sets of marks on the same scale.
d Calculate the mean and standard deviation for each of the sets of marks.
e Discuss the effect of students choice on the distribution of marks.
5 Consider the scores on a manual dexterity test for two groups of students. One group was
randomly selected from the population of normal children. The other group was randomly
selected from the population of children with a particular learning disability. The results
are shown in the following table.
Normal 32 26 28 28 28 31 24 24
children (x
1
) 30 28 29 27 28 30 26
Learning disabled 23 21 11 21 29 29 30
children (x
2
) 25 19 20 30 27 26
a Draw boxplots of the scores on the same scale.
b Compare the distributions for normal children and learning disabled children.
6 A researcher noted that loss of sleep affected the number of dreams experienced by an
individual. He also noted that as soon as people started to dream they exhibited rapid eye
movement (REM). To examine this apparent relationship be kept a group of volunteers
awake for various lengths of time by reading them spicy chapters from a statistics book.
After they fell asleep he recorded the number of times REM occurred. The following data
was obtained.
Subject 1 2 3 4 5 6 7 8
Hours of sleep deprivation 1.0 1.5 2.0 2.5 3.0 3.5 4.0 4.5
Number of times REM occurred 10 20 15 30 20 20 25 35
a Construct a scatterplot of these data, and use it to describe the relationship between
hours of sleep deprivation and number of times REM occurred.
b Determine the value of the q-correlation coefcient.
c Calculate the value of Pearsons r for these data.
d Determine the equation for the least squares regression line which relate hours of sleep
deprivation and number of times REM occurred.
e Interpret the slope, b, of this equation.
f Use the equation to predict the number of times REM occurred if the subject
experiences ve hours of sleep deprivation.
7 To test the effect of driving instruction on driving skill, ten randomly selected learner
drivers were given a driving skills test. The number of hours instruction for each learner
was also recorded. The results are displayed in the table shown.
P1: FXS/ABE P2: FXS
9780521740494c24.xml CUAU033-EVANS October 5, 2008 8:35
R
e
v
i
s
i
o
n
598 Essential Advanced General Mathematics
Learner A B C D E F G H I J
Hours 19 2 5 9 16 4 19 26 14 8
Test score 32 12 17 19 23 16 28 36 30 23
a Construct a scatterplot of these data, and use it to describe the relationship between the
number of hours instruction and the score.
b Determine the value of the q-correlation coefcient.
c Calculate the value of Pearsons r for these data.
d Determine the equation for the least squares regression line which relates number of
hours instruction and score.
e Interpret the slope, b, of this equation.
f Use the equation to predict the score after 10 hours of instruction.
8 Consider the data in the table, which shows the number of government schools in Victoria
over the period 19811992.
Year Number of schools
1981 2149
1982 2140
1983 2124
1984 2118
1985 2118
1986 2114
1987 2091
1988 2064
1989 2059
1990 2038
1991 2029
1992 2013
a Construct a scatterplot of these data, and use it to
describe the relationship between the number of
government schools and year.
b Determine the value of the q-correlation coefcient.
c Calculate the value of Pearsons r for these data.
d Determine the equation for the least squares
regression line which relates number of government
schools and year.
e Interpret the slope, b, of this equation.
f Use the equation to predict the number of
government schools in the year 2000.
g Comment on the reliability of the prediction in f.
9 The table shows the weights (kg) and blood glucose levels (mg/100 mL) of 16 apparently
healthy adult males.
Weight 64.0 75.2 73.1 82.1 76.3 95.7 59.4 93.4
Glucose 108 109 104 102 105 120 78 110
Weight 82.1 78.9 76.6 82.1 83.9 73.2 64.4 77.5
Glucose 101 85 98 100 108 104 102 89
a Construct a scatterplot of these data, and use in a describe the relationship between
weight and blood glucose level.
b Calculate the value of Pearsons r for these data.
c Determine the equation for the least squares regression line which relates weight and
blood glucose level.
d Interpret the slope, b, of this equation.
e Use the equation to predict the blood glucose levels for a male who weighs 70 kg.
P1: FXS/ABE P2: FXS
9780521740494c24.xml CUAU033-EVANS October 5, 2008 8:35
R
e
v
i
s
i
o
n
Chapter 24 Revision of chapters 22 and 23 599
10 The data shown in this table was collected in an
investigation to determine if the amount spent by
a school district on public education is dependent
on the per capita income of the school district.
Per capita Expenditure per
income ($) student ($)
14 198 1871
14 008 1850
16 197 2210
15 343 2188
15 928 2547
13 764 1834
13 244 1623
14 612 2052
14 918 2256
12 194 1476
a Construct a scatterplot of these data, and use it
to describe the relationship between expenditure
and per capita income.
b Determine the value of the q-correlation
coefcient.
c Calculate the value of Pearsons r for these data.
d Determine the equation for the least squares
regression line which relates expenditure and
per capita income.
e Interpret the slope, b, of this equation.
f Use the equation to predict the expenditure if the per capita income is $16 000.
g Comment on the reliability of the prediction made if the per capita income is
$30 000.
11 The owner of a fruit store records the number of boxes of mangoes she sells each month,
together with the average price of the mangoes in that month, giving the following data.
Month Sold Price ($) Month Sold Price ($)
Jan 25 0.95 July 6 2.50
Feb 30 0.90 Aug 5 3.00
Mar 20 1.20 Sep 5 3.20
April 18 1.40 Oct 7 2.40
May 11 2.00 Nov 15 1.90
June 8 2.40 Dec 15 1.75
a Construct a scatterplot of these data, and use it to describe the relationship between the
price of mangoes and the number of boxes sold.
b Determine the value of the q-correlation coefcient.
c Calculate the value of Pearsons r for these data.
d Determine the equation for the least squares regression line which relates number of
boxes sold and price.
e Interpret the slope, b, of this equation.
f Use the equation to predict the number of boxes of mangoes sold if the price of
mangoes is $ 1.40 each.
g Comment on the reliability of the predicted number of boxes of mangoes sold if their
price is $0.05 (5 cents) each.
P1: FXS/ABE P2: FXS
9780521740494c24.xml CUAU033-EVANS October 5, 2008 8:35
R
e
v
i
s
i
o
n
600 Essential Advanced General Mathematics
12 To study the effect of water hardness on taste, the data in this table were obtained from
specimens of drinking water from eight country towns (a higher taste rating means better
tasting water).
Amount of
magnesium (mg) Taste rating
8.7 25
9.0 25
11.0 26
8.5 48
9.2 65
12.0 87
12.0 90
18.0 100
a Construct a scatterplot of these data, and use
it to describe the relationship between taste
rating and the amount of magnesium in the
water.
b Determine the value of the q-correlation
coefcient.
c Calculate the value of the correlation coefcient
r for these data.
d Determine the equation for the least squares
regression line which relates taste rating and the
amount of magnesium.
e Interpret the slope, b, of this equation.
f Use the equation to predict the taste rating if the amount of magnesium is 80 mg.
13 The following table gives details of Australian Test Cricket Captains, based on all tests up
to and including the Third Test, January 1998.
Captain Tenure Tests W L D T
D W Gregory 187679 3 2 1 0 0
W L Murdoch 188090 16 5 7 4 0
T P Horan 188485 2 0 2 0 0
H H Massie 188485 1 1 0 0 0
J M Blackham 188495 8 3 3 2 0
H J H Scott 1886 3 0 3 0 0
P S McDonnell 188688 6 1 5 0 0
G Griffen 189495 4 2 2 0 0
G H S Trott 189698 8 5 3 0 0
J Darling 189905 21 7 4 10 0
H Trumble 190102 2 2 0 0 0
M A Noble 190309 15 8 5 2 0
C Hill 191012 10 5 5 0 0
S E Gregory 1912 6 2 1 3 0
W W Armstrong 192021 10 8 0 2 0
H L Collins 192126 11 5 2 4 0
W Bardsley 1926 2 0 0 2 0
(contd)
P1: FXS/ABE P2: FXS
9780521740494c24.xml CUAU033-EVANS October 5, 2008 8:35
R
e
v
i
s
i
o
n
Chapter 24 Revision of chapters 22 and 23 601
Captain Tenure Tests W L D T
J Ryder 192829 5 1 4 0 0
W M Woodfull 193034 25 14 7 4 0
V Y Richardson 193536 5 4 0 1 0
D G Bradman 193648 24 15 3 6 0
W A Brown 194546 1 1 0 0 0
A L Hassett 194953 24 14 4 6 0
A R Morris 195155 2 0 2 0 0
I W Johnson 195457 17 7 5 5 0
R R Lindwall 195657 1 0 0 1 0
I D Craig 195758 5 3 0 2 0
R Benaud 195864 28 12 4 11 1
R N Harvey 1961 1 1 0 0 0
R B Simpson 196378 39 12 12 15 0
B C Booth 196566 2 0 1 1 0
W M Lawry 196771 25 9 8 8 0
B N Jarman 1968 1 0 0 1 0
I M Chappell 197075 30 15 5 10 0
G S Chappell 197583 48 21 13 14 0
G N Yallop 197879 7 1 6 0 0
K J Hughes 197885 28 4 13 11 0
A R Border 198494 93 32 22 38 1
M A Taylor 19941999 50 26 13 11 0
S R Waugh 19992004 57 41 9 7 0
R Ponting 2004 8 6 1 1 0
a i Draw a boxplot of the number of tests captained by each captain.
ii Use the boxplot to describe the distribution of the tests captained.
iii Who do you think is Australias most successful captain from this plot?
b i Draw a boxplot of the percentage of matches won by each captain.
ii Use the boxplot to describe the distribution of percentage of matches won.
iii Who do you think is Australias most successful captain from this plot?
c i Plot the number of matches won against the number of matches played for each
captain.
ii Describe the relationship between number of matches played and number of
matches won.
iii Calculate the correlation coefcient r for the two variables.
iv Determine the equation of the least squares regression line.
v Write a sentence interpreting the slope of the regression line.
vi Add the least squares regression line to the plot, and consider the residuals. Who
do you think has been Australias most successful cricket captain from this plot?
P1: FXS/ABE P2: FXS
9780521740494c25.xml CUAU033-EVANS October 5, 2008 8:38
C H A P T E R
25
Proof and number
Objectives
To understand implication
To understand converse and equivalence
To understand the purpose of counter examples
To construct proofs
To understand the principle of mathematical induction
To solve linear Diophantine equations
To apply the Euclidean algorithm to find the highest common factor of two
numbers
To apply the Euclidean algorithm in the solution of linear Diophantine equations
25.1 An introduction to proof
Implication, converse and equivalence
For two statements p and q, p q is read p implies q.
For example,
x 3 = 4 x = 7
x is divisible by 3 2x is divisible by 6
x = 3 x
2
= 9.
In the rst two examples the converse statements also hold.
q p is the converse of p q
The converse of the two examples are
x = 7 x 3 = 4
2x is divisible by 6 x is divisible by 3.
Combining p q and q p we can write p qwhich may be read p is equivalent to q
Thus we can write
x 3 = 4 x = 7
x is divisible by 3 2x is divisible by 6.
602
P1: FXS/ABE P2: FXS
9780521740494c25.xml CUAU033-EVANS October 5, 2008 8:38
Chapter 25 Proof and number 603
An equivalent symbol for is iff which is read if and only if.
However, x
2
= 9 does not imply that x = 3, i.e., the converse does not hold.
In fact, x
2
= 9 x = 3 or x = 3
There are many other examples where the implication only holds one way.
For example,
x > 3 x
2
> 9, but x
2
> 9 does not imply x > 3
x rational x
2
rational, but x
2
rational does not imply x rational
x and y even x + y even, but x + y even does not imply x and y even
a = b a
2
= b
2
, but a
2
= b
2
does not imply a = b
Quadrilateral ABCD is a square the sides of quadrilateral ABCD are of equal length,
but the sides of the quadrilateral ABCD are of equal length does not imply quadrilateral
ABCD is a square.
Counter example
Consider numbers of the form n
2
+n +11 where n is a natural number.
n 1 2 3 4 5 6 7 8
n
2
+n +11 13 17 23 31 41 53 67 83
This table of values might lead to the conjecture:
Numbers of the form n
2
+n +11 are prime numbers.
However, if n = 11 it is evident that n
2
+n +11 is divisible by 11 and hence n
2
+n +11 is
not prime. The value of n
2
+n +11 when n = 11 provides a counter example to our
conjecture. The conjecture has been shown to be false.
Consider the conjecture: (a +b)
2
= a
2
+b
2
for all a and b R.
The values 2 and 3 for a and b respectively provide a counter
example.
Consider the conjecture: x
2
> x for all x R.
The value
1
3
for x provides a counter example.
Consider the conjecture: The cube of a natural number is greater than the natural number.
The natural number 1 provides a counter example.
This nal conjecture can be xed up by making the conjecture the cube of a natural number
greater than 1 is greater than the natural number.
Fix up the rst two conjectures.
Proof
In Year 10 you may have come across proofs in geometry or used them in your problem
solving. Proofs are a very important part of Mathematics. If we have a conjecture which we
suspect to be true and for which a counter example cannot be found, then we try to construct a
chain of reasoning which will enable us to deduce the result from assumptions which are as
simple as possible.
P1: FXS/ABE P2: FXS
9780521740494c25.xml CUAU033-EVANS October 5, 2008 8:38
604 Essential Advanced General Mathematics
Example 1
Consider this pattern.
4
2
3
2
+2
2
1
2
= 10
5
2
4
2
+3
2
2
2
= 14
6
2
5
2
+4
2
3
2
= 18
From this it could be conjectured that
If a, b, c, d are consecutive natural numbers with a < b < c < d then
d
2
c
2
+b
2
a
2
= d +c +b +a.
Prove this conjecture is true.
Solution
The result is not immediately obvious.
Let the numbers a, b, c, d be n 1, n, n +1, n +2 respectively.
Then d +c +b +a = n +2 +n +1 +n +n 1
= 4n +2
and d
2
c
2
+b
2
a
2
= (n +2)
2
(n +1)
2
+n
2
(n 1)
2
= n
2
+4n +4 n
2
2n 1 +n
2
n
2
+2n 1
= 4n +2
The result has been proved. In fact, it has been proved for any four consecutive
integers.
It can be seen from the proof that a more general result is true.
If a, b, c, d are consecutive integers then
d
2
c
2
+b
2
a
2
= a +b +c +d
The proof has actually added to the understanding of the problem by leading to a
generalisation of the original conjecture.
Example 2
Consider the following.
321 941 987 980
123 149 789 089
198 792 198 891
+ 891 + 297 + 891 + 198
1089 1089 1089 1089
Conjecture
Take any three digit number whose digits decrease as you read them from left to right. Make
another number by reversing the order of the digits and subtract the smaller from the larger.
Reverse the order of the digits of the difference and add the number so formed to the
difference. The result will always be 1089.
P1: FXS/ABE P2: FXS
9780521740494c25.xml CUAU033-EVANS October 5, 2008 8:38
Chapter 25 Proof and number 605
Solution
Let the number be a 10
2
+b 10 +c.
Reverse the digits to get the number c 10
2
+b 10 +a.
It will be assumed a > c without loss of generality.
a 10
2
+b 10 +c c 10
2
b 10 a = (a c) 10
2
+(c a)
Now c a is negative, but (10 +c a) is positive.
Write (a c) 10
2
+(c a) = (a c 1) 10
2
+90 +(10 +c a)
The digits are now correctly displayed.
Reverse the order to obtain (10 +c a) 10
2
+90 +(a c 1) and add.
(10 +c a +a c 1) 10
2
+(90 +90) +(10 +c a +a c 1)
= 9 10
2
+180 +9
= 900 +180 +9
= 1089
We can attempt to generalise in this case. The question arises What happens when a
four digit number is considered? The proof also indicates that the base chosen is
important. What is the result for different bases?
Exercise 25A
1 Insert or to make the following into true statements about integers.
a p is even . . . pq is even b p +q is odd . . . pq is even
c x = 0 . . . xy = 0 d ab = ac . . . b = c
2 State with reasons whether the following statements are true or false.
a n (A) = 5 and n (B) = 3 n (A B) = 8 b A B A B = A
c A B = A = or B = d A

= A =
3 Write down the converses of the statements given in 2 and state whether each new
statement is true or false.
4 State the converse of each of the following statements and also state whether the converse
is true or false.
a If n is odd then n
2
is odd. b N is divisible by 3 N
2
is divisible by 9
c x
2
> 4 x < 2
5 The sum of two consecutive odd numbers is divisible by 4. Can you make similar
statements about:
a the sum of three consecutive odd numbers
b the sum of four consecutive odd numbers?
Prove your assertions.
P1: FXS/ABE P2: FXS
9780521740494c25.xml CUAU033-EVANS October 5, 2008 8:38
606 Essential Advanced General Mathematics
6 Prove that the sum of the squares of ve consecutive integers is divisible by 5.
(Take n 2, n 1, n, n +1, n +2 as the integers.)
7 For each of the following statements give a counter example which proves that the given
statement is false.
a The sum of the squares of two numbers is equal to the square of the sum of these two
numbers.
b If a number is even then it is not divisible by 7.
c

a +b =

a +

b for all a, b R
d
1
2

a +
b +c
2

=
1
2

a +b
2
+c

for all a, b, c R
e The sum of two prime numbers is a prime number.
f
1
s +t
=
1
s
+
1
t
for all s, t R\ {0}
8 Write the converse of each of the following and state whether it is true or false.
a If a b is positive then a > b b If x = 0 and y = 0 then x = y
c If x + y = 0 then x = y d If x is even and y is odd then xy is even.
e The square of an even number is even.
9 For each of the following conjectures for the set of natural numbers either prove or
provide a counter example.
a A number N has an odd number of divisors if and only if it is a perfect square.
b For any odd number, N, there is a number with exactly N divisors.
c For any number, N, there is a number with exactly N divisors.
d There are innitely many numbers with exactly N divisors (N = 1).
10 a Show that a
2
+b
2
2ab for all real a and b.
b Hence, or otherwise, prove that if u 0, v 0, then
u +v
2

uv
11 Prove that for all positive integers a and b, (a +b)

1
a
+
1
b

4
12 Prove each of the following for a, b Z.
a If a and b are even, a +b is even. b If a and b are odd, a +b is even.
c If a is even and b is odd, ab is even. d If a and b are odd, ab is odd.
e a +b is even if and only if a b is even.
f a +b even and a b even implies ab is a difference of perfect squares.
13 The oor of a rectangular room is covered with square tiles. The room is m tiles wide and
n tiles long with m n.If exactly half of the tiles are on the perimeter, nd all possible
values of m and n.
14 Prove that no positive integer (except 1) all of whose digits are 1s is a perfect square.
P1: FXS/ABE P2: FXS
9780521740494c25.xml CUAU033-EVANS October 5, 2008 8:38
Chapter 25 Proof and number 607
15 Find all right-angled triangles which have integer length sides and for which the area has
the same numerical value as the perimeter.
16 If a, b and c are positive integers such that no integer greater than 1 divides them all and
1
a
+
1
b
=
1
c
, prove that a +b is a perfect square.
Note: This is a difcult question.
25.2 The principle of mathematical induction
Suppose that P(k) is a statement for each positive integer k.
For example, the statement could be k
2
2k +1 0 for each positive integer k or k
2
k
is an even integer for each positive integer k.
The method of mathematical induction is used to prove P(k) for all k as follows.
i Show P(1) is true.
ii Show P(k) P(k +1) for every positive integer k.
From this it can be seen that if both of the statements hold, then
P(1) P(2), P(2) P(3), P(3) P(4)
and continuing in this fashion it can be seen that P(k) is true for every positive integer k.
The method of proof by induction can be illustrated by a line of dominoes (starting with
domino 1) stretching away without an end. To be sure that all dominoes will be knocked over it
is enough to know that
i the rst domino is knocked over and
ii if one domino falls it will certainly knock over the next.
Another integer, besides 1, may be chosen for the starting value.
Example 3
Prove that the sum of the rst n integers is
n (n +1)
2
.
Solution
Let P(n) be the statement that the sum of the rst n integers is
n (n +1)
2
P(1) is certainly true as 1 =
1 2
2
Assume 1 +2 + +k =
k (k +1)
2
i.e. that P(k) is true
then 1 +2 + +k +k +1 =
k (k +1)
2
+(k +1)
=
(k +1)(k +2)
2
P(k +1) is true
and the principle of induction gives that P(n) is true for all n.
P1: FXS/ABE P2: FXS
9780521740494c25.xml CUAU033-EVANS October 5, 2008 8:38
608 Essential Advanced General Mathematics
It is sometimes convenient to begin an induction at a point other than k = 1. In the following
example, the starting point is at k = 5.
Example 4
Prove that 2
n
> 1 +n
2
for all n > 4, n N.
Solution
Let P(n) be the statement that 2
n
> 1 +n
2
.
If n = 5, 2
5
= 32 and 1 +5
2
= 26
As 2
5
> 26, P(5) is true.
Assume true for k.
i.e. 2
k
> 1 +k
2
For 2
k+1
= 2.2
k
> 2(1 +k
2
) = 2 +k
2
+k
2
1
But k
2
> 2k for k > 2
Therefore 2 +k
2
+k
2
> 2 +2k +k
2
= 1 +1 +2k k
2
= 1 +(1 +k)
2
2
From inequalities 1 and 2
2
k+1
> 1 +(1 +k)
2
P(k +1) is true
and the principle of induction gives that P(n) is true for all n 5.
Example 5
Prove that 3
2n
1 is divisible by 8 for all n N.
Solution
Let P(n) be the statement that 3
2n
1 is divisible by 8.
If n = 1, 3
2
1 = 8 is divisible by 8
i.e. P(l) is true.
Assume true for k.
i.e. P(k) is true and 3
2k
1 is divisible by 8
Consider 3
2(k+1)
1 = 3
2k+2
1 = 3
2
3
2k
3
2
+3
2
1
= 3
2
(3
2k
1) +8
Since 3
2k
1 is divisible by 8 and 8 is divisible by 8,
3
2(k+1)
1 is divisible by 8.
P(k +1) is true, P(n) is true for all n N.
P1: FXS/ABE P2: FXS
9780521740494c25.xml CUAU033-EVANS October 5, 2008 8:38
Chapter 25 Proof and number 609
Example 6
Prove that 2
n
> n
3
for n 10.
Solution
P(10) is true as 2
10
> 10
3
(1024 > 1000)
Assume P(k) is true i.e. 2
k
> k
3
for k > 9
then 2
k+1
= 2 2
k
> 2 k
3
2
k+1
> (k +1)
3
+k
3
3k
2
3k 1
2
k+1
> (k +1)
3
+k
3
3k
2
3k 459
2
k+1
> (k +1)
3
+(k
2
+6k +51)(k 9)
2
k+1
> (k +1)
3
for k > 9 (Note: k
2
+6k +51 > 0 for all k)
P(k +1) is true and P(n) is true for all n 10
Exercise 25B
Sums
1 Prove each of the following by induction, for all natural numbers n.
a 1 +2 + +n =
1
2
n (n +1)
b 1
2
+2
2
+ +n
2
=
1
6
n (n +1) (2n +1)
c
1
1 3
+
1
3 5
+ +
1
(2n 1) (2n +1)
=
n
2n +1
d 1 2 +2 3 +3 4 + +n(n +1) =
1
3
n(n +1)(n +2)
e 1 4 +2 7 +3 10 + +n(3n +1) = n(n +1)
2
f 1 +2 2
1
+3 2
2
+ +n 2
n1
= 1 +(n 1) 2
n
g
1
3 5
+
1
5 7
+
1
7 9
+
1
(2n +1) (2n +3)
=
1
2

1
3

1
2n +3

h 1
3
+3
3
+5
3
+ +(2n 1)
3
= n
2
(2n
2
1)
i 2 1 +3 2 + +n(n 1) =
1
3
n(n
2
1)
Divisibility
2 Prove by induction that, for all natural numbers,
a n(n +1)(n +2) is divisible by 3 b 4n
3
4n is divisible by 3
c 8
n
+2 7
n
1 is divisible by 7 d n(n
2
+2) is divisible by 3
e 23
n
1 is divisible by 22 f 8
n
5
n
is divisible by 3
g 5n
3
3n
2
2n is divisible by 6
P1: FXS/ABE P2: FXS
9780521740494c25.xml CUAU033-EVANS October 5, 2008 8:38
610 Essential Advanced General Mathematics
3 a Prove that 4
n
+5
n
is divisible by 9 for all positive odd integers, n.
b Prove that 3
n
1 is divisible by 8 for all positive even integers, n.
c Prove that 6
n+1
5 (n +1) 1 is divisible by 25 for all positive integers, n.
Inequalities
4 a Prove that 3
n
> n
3
for all positive integers greater than 3.
b Prove that 1
3
+2
3
+ +n
3
>
n
4
4
for all positive odd integers.
c Prove that 2
n
> n
2
for all positive integers greater than 4.
d Prove that 2
n
> 3n for all positive integers greater than 3.
e Prove that 2
n
n! for all n 4.
5 Prove that
a
1
1 2
+
1
2 3
+
1
3 4
+ +
1
n (n +1)
=
n
n +1
, n N
b 1
3
+2
3
+ +n
3
=
1
4
n
2
(n +1)
2
, n N
c 3
3n+1
+9 2
n+3
is divisible by 25, n N
6 Prove by induction that if S is a set of n elements, then S has 2
n
subsets.
25.3 Linear Diophantine equations
Consider the equation 3x +4y = 1. This equation denes a straight line. If the values of x and
y are integers, a family of solutions may be described. This equation is called a linear
Diophantine equation.
On the graph, a family of solutions is illustrated.
x
10
11
9
8
7
6
5
4
3
2
1
0
1
2
3
4
1 2 3 4 5 6 7 8 9 10 11 12 13 14 15 1 2 3 4 5
(5, 4)
(1, 1)
(3, 2)
(7, 5)
(11, 8)
(15, 11)
y
Notice that as the integer solutions for x increase by 4, the y integer solutions decrease by 3.
P1: FXS/ABE P2: FXS
9780521740494c25.xml CUAU033-EVANS October 5, 2008 8:38
Chapter 25 Proof and number 611
The solutions may be built up in the following way using (1, 1) as the starting point.
x y
1 +4 1 3
1 +2 4 1 2 3
1 +3 4 1 3 3
i.e. x y
3 2
7 5
11 8
The family of solutions may be described as
x = 1 +4t, y = 1 3t where t Z
Using set notation the solution is
{(x, y) : x = 1 +4t, y = 1 3t, t Z}
If a linear Diophantine equation has one solution then it has innitely many.
If ax +by = c is a linear Diophantine equation in two unknowns and (x
0
, y
0
) is found to be
one solution, the general solution is given by
x = x
0
+
b
d
t
y = y
0

a
d
t, where t Z
and d is the highest common factor of a and b.
Proof
Suppose x
1
, y
1
are solutions to the equation.
Then ax
1
+by
1
= c 1
and ax
0
+by
0
= c 2
Subtracting 2 from 1
a(x
1
x
0
) = b(y
0
y
1
)
Divide both sides by d
a
d
(x
1
x
0
) =
b
d
(y
0
y
1
)
a
d
and
b
d
have no common factors.
Hence x
1
x
0
must be divisible by
b
d
and x
1
= x
0
+
b
d
t
Similarly y
1
= y
0

a
d
t
It can be proved by substitution that
x = x
0
+
b
d
t, y = y
0

a
d
t, is a solution of the equation for any t Z.
P1: FXS/ABE P2: FXS
9780521740494c25.xml CUAU033-EVANS October 5, 2008 8:38
612 Essential Advanced General Mathematics
Example 7
A man has $200 in his wallet. This is made up of $50 and $20 notes. What are the possible
numbers of each of these types of notes?
Solution
Let x, y be the number of $50 and $20 notes respectively.
The linear Diophantine equation is
50x +20y = 200
5x +2y = 20
By inspection a solution is
x = 4, y = 0
The general solution will be
x = 4 +2t, y = 0 5t, t Z
We are only interested in the case where x, y 0
Thus 4 +2t 0 and 0 5t 0
Hence 2 t 0
For t = 2, x = 0, y = 10
t = 1, x = 2, y = 5
t = 0, x = 4, y = 0
Hence the man can have ten $20 notes
or two $50 notes and ve $20 notes
or four $50 notes.
Exercise 25C
1 Find all solutions of the following Diophantine equations
a 11x +3y = 1 b 2x +7y = 2 c 24x +63y = 99
d 22x +6y = 2 e 2x +7y = 22 f 10x +35y = 110
2 For a, c, e in 1 nd the solutions for which x, y are both positive.
3 Prove that if ax +by = c and the highest common factor of a and b does not divide c,
then there is no solution to the Diophantine equation.
4 A student puts a number of spiders (with eight legs) and a number of beetles (with six
legs) in a box. She counted 54 legs in all.
a Form a Diophantine equation.
b Find the number of spiders and the number of beetles in the box.
P1: FXS/ABE P2: FXS
9780521740494c25.xml CUAU033-EVANS October 5, 2008 8:38
Chapter 25 Proof and number 613
5 Helena has a number of coins in her wallet. They are all either 20c or 50c coins. The total
value of the coins is $5.00. What are the possible numbers of each type of coin?
6 One of the solutions of the equation 19x +83y = 1983 in positive integers x and y is
obviously x = 100, y = 1. Show that there is only one other pair of positive integers
which satisfy this equation and nd it. Consider the equation 19x +98y = 1998.
7 A man has $500 in his wallet made up of $50 and $10 notes. Find the possible
combinations of notes that he could have.
8 There are seven coconuts and 63 heaps of pineapples. Each heap has exactly the same
number of pineapples. The fruit is to be divided equally between 23 people.
Let x be the number of pineapples in each heap and y the number of pieces of fruit that
each person receives.
Form a Diophantine equation and nd the possible values for x and y.
9 A dealer spent $10 000 buying cattle, some at $410 each and the rest at $530 each. How
many of each sort did she buy?
10 Find the smallest positive number which, when divided by 7, leaves a remainder of 6, and
when divided by 11 leaves a remainder of 9. Also nd the general form of such numbers.
11 Given a 3 litre jug and a 5 litre jug can I measure exactly 7 litres of water? If it is possible,
explain how this may be done as efciently as possible.
12 The Guadeloupe Post Ofce has only 3c and 5c stamps. What amounts of postage can the
post ofce sell?
13 A man spent $29.60 buying party hats. There were two types of party hat. Type A cost
$1.70 while type B cost $1.00. How many of each type did he buy?
25.4 The Euclidean algorithm
The Euclidean algorithm provides a method for nding the highest common factor of two
numbers and also a method for solving linear Diophantine equations.
Theorem 1
If a, b are integers with a > 0, then there are unique integers q, r such that b = aq +r with
0 r < a.
Proof
Suppose there exists another pair of integers, q
1
and r
1
with
b = aq
1
+r
1
and 0 r
1
< a.
Suppose that r > r
1
. Then by subtraction
0 = aq
1
+r
1
(aq +r)
0 = a(q
1
q) +(r
1
r)
r r
1
= a(q
1
q)
P1: FXS/ABE P2: FXS
9780521740494c25.xml CUAU033-EVANS October 5, 2008 8:38
614 Essential Advanced General Mathematics
Now since the right hand side is an integer and is a multiple of a, then a divides the left hand
side. But the left hand side is an integer which is greater than zero and less than a.
Therefore the assumption, that r > r
1
, must be false.
But if it is assumed r < r
1
, then consider
0 = (aq +r) (aq
1
+r
1
)
and a similar contradiction will arise
Thus r = r
1
. In that case
r r
1
= 0 = a(q
1
q)
so q
1
= q and r
1
= r
and the uniqueness of integers q, r has been proved.
Example 8
Express 45 in the form 6q +r where 0 r < 6.
Solution
Here 45 = 6(8) +3
Note that 45 = 6(7) 3 is not a correct answer since the remainder 3 is less
than zero.
In the following, (a, b) denotes the highest common factor of the integers a and b.
Theorem 2
If a and b are two integers, a = 0 and b = aq +r where q, r are integers, then (a, b) = (a, r).
(This theorem may be used to determine the highest common factor of any two given integers.)
Proof
If d is a common divisor of a and r, then d divides the right-hand side of equation b = aq +r
and so d divides b.
This proves that all common divisors of a and r will be common divisors of a and b. But
(a, r) is a common divisor of a and r, and so (a, r) must divide a and b. It follows that (a, r)
must divide (a, b).
That is (a, b) = (a, r)x where x is an integer 1
Now rewrite equation b = aq +r as r = b aq
All common divisors of a and b divide the right hand side of this relation and so divide r.
Thus (a, b) must divide r as well as a. Hence (a, b) must divide (a, r).
It has now been proved (a, r) = (a, b)y, where y is an integer 2
P1: FXS/ABE P2: FXS
9780521740494c25.xml CUAU033-EVANS October 5, 2008 8:38
Chapter 25 Proof and number 615
From equations 1 and 2 obtain
(a, r) = [(a, r)x]y
Hence 1 = xy
This equation in integers x, y is possible only if both x, y are +1 or 1.
Hence (a, b) = +(a, r), since 1 is inappropriate to this problem.
Since the integer r is less than the integer b, the calculation of (a, r) is easier than the
calculation of (a, b).
Example 9
Find (1271, 3875).
Solution
3875 = 1271 3 +62
and (1271, 3875) = (1271, 62) by theorem 2.
Now 1271 = 62 20 +31
(62, 1271) = (62, 31) again by theorem 2.
As 62 = 31 2 +0
we have (1271, 3875) = 31 by using the theorem a nal time.
This procedure (algorithm) is called the Euclidean algorithm.
Method for finding a solution of a linear
Diophantine equation
The method presented here uses the Euclidean algorithm.
Example 10
Find a, b Z such that 22a +6b = 2
Solution
Apply the division algorithm to 22 and 6
22 = 3 6 +4 1
6 = 1 4 +2 2
4 = 2 2 3
i.e. (22, 6) = 2
Using these results
2 = 6 1 4 from 2
2 = 6 1(22 3 6) from 1
2 = 6 22 +3 6
2 = 4 6 1 22
or 1 22 +4 6 = 2
A solution is (1, 4)
P1: FXS/ABE P2: FXS
9780521740494c25.xml CUAU033-EVANS October 5, 2008 8:38
616 Essential Advanced General Mathematics
Example 11
Find a, b Z such that 125a +90b = 5
Solution
Divide by 5.
25a +18b = 1
Apply the division algorithm.
25 = 1 18 +7 1
18 = 2 7 +4 2
7 = 1 4 +3 3
4 = 1 3 +1 4
3 = 3 1 5
i.e., (25, 18) = 1
1 = 4 1 3 from 4
1 = 4 1(7 1 4) from 3
1 = 2 4 1 7
1 = 2 (18 2 7) 1 7 from 2
1 = 2 18 5 7
1 = 2 18 5(25 18) from 1
1 = 7 18 5 25
a = 5 and b = 7 is one solution.
Solution is given by a = 5 +18t and b = 7 25t ; t Z.
Exercise 25D
1 For the following, express b in the form b = aq +r with 0 r < a and show in each case
(a, b) = (a, r).
a a = 5, b = 43 b a = 13, b = 39 c a = 17, b = 37 d a = 16, b = 128
2 If d is a common factor of a and b, prove that d is a common divisor of a +b and a b.
3 Use the Euclidean algorithm to nd
a (4361, 9284) b (999, 2160) c (372, 762) d (5255, 716 485)
4 Solve in the integers the equations
a 804x +2358y = 6 b 18x +24y = 6
c 3x +4y = 478 d 3x 5y = 38
e 804x +2688y = 12 f 1816x +2688y = 8
P1: FXS/ABE P2: FXS
9780521740494c25.xml CUAU033-EVANS October 5, 2008 8:38
R
e
v
i
e
w
Chapter 25 Proof and number 617
Chapter summary
For two statements p and q, p q is read p implies q.
q p is the converse of p q
Combining p q and q p we can write p q which may be read p is equivalent to q
An equivalent symbol for is iff which is read if and only if .
A counter example is an example which proves a conjecture to be false.
The method of mathematical induction is used to prove P(k) for all k as follows
i Show P(1) is true.
ii Show P(k) P(k +1) for every positive integer k.
Consider the equation 3x +4y = 1. This equation denes a straight line. The coefcients
of the left hand side of the equation and the right hand side are integers. If the values of x
and y are integers, a family of solutions may be described. This equation is called a linear
Diophantine equation.
If a linear Diophantine equation has one solution then it has innitely many.
If ax +by = c is a linear Diophantine equation in two unknowns and (x
0
, y
0
) is found to
be one solution, the general solution is given by x = x
0
+
b
d
t, y = y
0

a
d
t , where t Z
If a, b are integers with a > 0, then there are unique integers q, r such that b = aq +r with
0 r < a.
The Euclidean algorithm
In the following, (a, b) denotes the highest common factor of the integers a and b.
If a and b are two integers, a = 0 and b = aq +r where q, r are integers, then
(a, b) = (a, r).
(This result may be used to determine the highest common factor of any two given integers,
and to solve any linear Diophantine equation.)
Multiple-choice questions
1 If m is a positive even integer and n is a positive odd integer the statement which is false is
A m +2n is even B m +n is odd C 3m +2n is even
D m n is even E m
2
+n is even
2 The statement below which is true is
A x 3 > 0 (x 3)(2 x) > 0 B (x 3)(2 x) > 0 x 3 > 0
C (x 3)(2 x) > 0 x 3 > 0 D x 3 > 0 (x 3)(x 2) > 0
E x 3 > 0 (x 3)(x 2) > 0
3 If p and q are positive real numbers, and p > q with p +q = 1, the largest quantity from
the following is
A p B
1
q
C
1
pq
D
1
p
E pq
P1: FXS/ABE P2: FXS
9780521740494c25.xml CUAU033-EVANS October 5, 2008 8:38
R
e
v
i
e
w
618 Essential Advanced General Mathematics
4 If p > q and pq = 0, it is true that
1
p
<
1
q
A always B never C only when p and q are positive
D for all p and q except when both are negative E whenever pq > 0
5 The number of factors that the integer 2
p
3
q
5
r
has is
A
( p +q +r)!
q!q!r!
B pqr C p +q +r
D ( p +1)(q +1)(r +1) E p +q +r +1
6 If an integer of two digits is k times the sum of its digits, the number formed by
interchanging the digits is the sum of the digits multiplied by
A 9 k B 10 k C 11 k D k 5 E k +8
7 The number of pairs of integers (m, n) which satisfy the equation m +n = mn is
A 1 B 2 C 3 D 4 E more than 4
8 If a, b, c are any real numbers, and a > b, the statement which must be true is
A
1
a
>
1
b
B ac > bc C a
2
> b
2
D a +c > b +c E
1
a
<
1
b
9 The number of solutions of the Diophantine equation 3x +5y = 1008, where x and y are
positive integers, is
A 1 B 134 C 68 D 67 E innite
10 If y = (n 1)(n 2)(n 3), where n is a positive integer, then y is not always divisible by
A 6 B 5 C 3 D 2 E 1
Short-answer questions (technology-free)
1 Find the highest common factor of 1885 and 365 using the Euclidean algorithm.
2 Consider 9x +43y = 7. Solve for x and y where
a x Z, y Z b x Z
+
, y Z
+
3 Prove that the product of two consecutive odd integers is odd. (You may assume that the
sum and product of any two integers is an integer.)
4 Using the Euclidean algorithm, nd the highest common factor of 10 659 and 12 121.
5 a Solve the Diophantine equation 5x +7y = 1
b Hence solve the Diophantine equation 5x +7y = 100
c Find {(x, y) : 5x +7y = 1; y x, x, y Z}.
6 The sum of the ages of Tom and Fred is 63. Tom is twice as old as Fred was when Tom was
as old as Fred is now. What are the ages of Tom and Fred?
P1: FXS/ABE P2: FXS
9780521740494c25.xml CUAU033-EVANS October 5, 2008 8:38
R
e
v
i
e
w
Chapter 25 Proof and number 619
Extended-response questions
1 The strips shown below are formed by joining an odd number of triangles together. The
triangles are formed on 1 cm isometric paper as shown.
A strip of length 3
A strip of length 5
A strip of length 7
In the strip of length 3 there are two parallelograms as shown below.
In the strip of length 5 there are six parallelograms.
a Find and draw all the possible parallelograms in a strip of length 7.
b How many parallelograms are there in a strip of length 11?
c Find the number of parallelograms in a strip of length n where n is an odd number.
d Prove your result.
2 A common error for adding fractions is the following:
a
b
+
c
d
=
a +c
b +d
Investigate this error for a, b, c, d N.
3 Twelve cubes are used to build a tower three
cubes high. This is shown with the aid of
isometric dot paper in the diagram.
P1: FXS/ABE P2: FXS
9780521740494c25.xml CUAU033-EVANS October 5, 2008 8:38
R
e
v
i
e
w
620 Essential Advanced General Mathematics
a How many cubes are needed to build a tower of this type which is eighty cubes high?
Work out an expression for the number of cubes needed to build a tower of this type
which is n cubes high.
b The number of cubes needed to build a tower n cubes high, T
n
, is related to the number
of cubes needed to build a tower (n 1) cubes high, T
n1
, by the formula
T
n
= T
n1
+kn, where k is a constant.
What is the value of k?
c How many dots on the isometric dot paper are needed to draw a tower of this type n
cubes high?
d Investigate other types of towers.
4 You have an inexhaustible supply of 5 and 8 stamps.
a List all possible ways of obtaining a total value of 38 with these stamps.
b List all possible ways of obtaining a total of $1.20 with these stamps.
5 The digits of a three-digit number are interchanged so that none of the digits has retained its
original place. Then the new number is subtracted from the original. If the difference is a
two-digit number which is also a perfect square, nd all such two-digit numbers.
6 Find the positive integer n such that n and n +100 have an odd number of divisors.
7 Prove that if a > 0, then a +
1
a
2.
Hint: Use the result that

a
1
a

2
0 for all a R.
8 Show that the sum of three consecutive positive integers divides the sum of the cubes of
these three integers.
Hint: Consider the numbers n 1, n, n +1.
9 Add 1 to the product of four consecutive positive integers. Prove that the result is a perfect
square.
10 Choose any two positive integers which are not divisible by 3. Prove the difference between
their squares is divisible by 3.
11 Prove that for every positive integer n, the expression n
2
(n
4
1) is divisible by 60.
12 Choose any six consecutive positive integers greater than 3. Prove that there are at most two
prime numbers among them.
13 Find all six-digit numbers with the property:
If the rst and last digits are interchanged, then the new six-digit number is six times the
original number.
14 Prove by mathematical induction
a 1
2
+4
2
+7
2
+ +(3n 2)
2
=
1
2
n(6n
2
3n 1)
b 6
n
+4 is divisible by 10 for any natural number n.
P1: FXS/ABE P2: FXS
9780521740494gsy.xml CUAU033-EVANS October 21, 2008 20:47
Glossary
A
absolute value function: [p. 208]
Let f : R R be dened as f (x) =
_
x if x 0
x if x < 0
and is denoted by |x|
Also known as modulus function
acceleration: [p. 466] The acceleration of a particle
is dened as the rate of change of its velocity with
respect to time.
acceleration, average: [p. 466] The average
acceleration of a particle for the time interval [t
1
, t
2
]
is dened by
v
2
v
1
t
2
t
1
where v
2
is the velocity at time
t
2
and v
1
is the velocity at time t
1
.
acceleration, instantaneous: [p. 466] a =
dv
dt
addition formulas: [p. 309]
cos (u v) = cos u cos v +sin u sin v
cos (u +v) = cos u cos v sin u sin v
sin (u +v) = sin u cos v +cos u sin v
sin (u v) = sin u cos v cos u sin v
addition of complex numbers: [p. 418]
If z
1
= a +bi and z
2
= c +di , then
z
1
+ z
2
= (a +c) +(b +d)i .
addition of vectors: [p. 391] Let
a = a
1
i +a
2
j +a
3
k, and b = b
1
i +b
2
j +b
3
k
Then a +b = (a
1
+b
1
)i +(a
2
+b
2
) j +(a
3
+b
3
)k
algebraic numbers: [p. 68] Algebraic numbers are
those which are the solution(s) of an equation of the
form
a
0
x
n
+a
1
x
n1
+ +a
n
= 0, where a
0
, a
1
. . . , a
n
are integers.
amplitude of circular functions: [p. 264] The
distance between the mean position and the
maximum position, e.g., the graph of y = a sin x has
an amplitude of |a|.
angle between a vector and the i direction:
[p. 402] For vector a = a
1
i +a
2
j +a
3
k, where is
the angle between the vector a and the i direction,
cos =
a
1
|a|
.
angle between a vector and the j direction:
[p. 402] For vector a = a
1
i +a
2
j +a
3
k, where is
the angle between the vector a and the j direction,
cos =
a
2
|a|
.
angle between a vector and the k direction:
[p. 402] For vector a = a
1
i +a
2
j +a
3
k, where is
the angle between the vector a and the k direction,
cos =
a
3
|a|
.
angle between planes: [p. 353] If P is any point on
the common line of two planes
1
and
2
and PA and
PB are lines at right angles to the common line in
1
and
2
respectively, then APB is the angle between

1
and
2
.

2
P

B
A
angle of depression: [p. 344] The angle between the
horizontal and a direction below the horizontal.
eye level
angle of depression
cliff
lin
e
o
f s
ig
h
t
angle of elevation: [p. 344] The angle between the
horizontal and a direction above the horizontal.
eye level
angle of elevation
lin
e
o
f sig
h
t
arc: [p. 340] Any two
points on a circle divide the
circle into arcs. The shorter
arc is called the minor arc,
the longer is the major arc,
e.g., arc ACB is a minor arc
and ADB is a major arc in
this diagram.
D
O
A
B
C
621
P1: FXS/ABE P2: FXS
9780521740494gsy.xml CUAU033-EVANS October 21, 2008 20:47
622 Essential Advanced General Mathematics
arc length, l: [p. 341]
The length of arc ACB
is given by: l = r
where
c
= mag AOB
D
O
A
B
C

r
area of a triangle: [p. 337] Area =
1
2
bh
Area of triangle =
1
2
bc sin A, i.e.,
the area is given by half the
product of the length of two
sides and the sine of the
angle included between
them.
A C
h
b
B
Argand diagram: [p. 425] A geometrical
representation of the
set of complex
numbers.
Im(z)
Re(z)
P
0
b
z = a + bi
a

argument of a complex number, arg (z): [p. 430]


arg (z) = , where sin =
Im(z)
|z|
and
cos =
Re(z)
|z|
arg (z) is not dened uniquely.
Argument of a complex number, arg (z): [p. 430]
The single value of arg (z) in the interval (, ]
argument, properties of: [p. 430] The argument of
the product of two complex numbers is the sum of
their arguments, i.e., arg (z
1
z
2
) = arg (z
1
) +arg (z
2
)
Argument, properties of: [p. 431]
Arg (z
1
z
2
) = Arg (z
1
) +Arg (z
2
) +2k
where k = 0, 1 or 1
Arg
_
z
1
z
2
_
= Arg (z
1
) Arg (z
2
) +2k
where k = 0, 1 or 1
Arg
_
1
z
_
= Arg (z)
arithmetic sequence: [p. 122] A sequence in which
each successive term is found by adding a constant
value to the previous term, e.g., 2, 5, 8, 11, . . .
An arithmetic sequence can be dened by a
difference equation of the form:
t
n
= t
n1
+d, where d is the common difference.
The nth term of the sequence can be found using:
t
n
= a +(n 1)d, where a = t
1
arithmetic series: [p. 125] The sum of the terms in
an arithmetic sequence.
The sum of the rst n terms, S
n
, is given by the rule:
S
n
=
n
2
[2a +(n 1)d], where a = t
1
and d = t
n
t
n1
asymptote: [p. 447] A line which a graph
approaches, e.g. y =
1
x
has asymptotes at x = 0 and
y = 0.
asymptotes of hyperbolas: [p. 447] The hyperbola
with equation
x
2
a
2

y
2
b
2
= 1 has asymptotes y =
b
a
x
and y =
b
a
x.
B
bar chart: [p. 502] A visual display of a frequency
distribution when the data are categorical.
Frequencies (vertical axis) are represented by vertical
bars corresponding to each category (horizontal axis).
bearing (or compass bearing): [p. 346] The
direction measured from north clockwise.
bivariate data: [p. 551] Data which arises when two
variables are observed for each subject, such as
height and weight.
box-and-whisker plot: see boxplot
boxplot: [p. 533] A visual display of a numerical
data set, an alternative to a histogram or
stem-and-leaf plot. Once the ve-gure summary has
been determined the boxplot is constructed by
drawing a horizontal box with the ends at the rst and
third quartiles. The minimum value is joined to the
lower end of the box with a horizontal line and the
maximum value joined to the upper end of the box
with a horizontal line. The median is located with a
vertical line.
boxplot with outliers: [p. 536] A more complex
version of the boxplot, with outliers shown. Outliers
are determined according to the formal denition of
an outlier, and illustrated on the plot with an asterisk.
The boxplot is constructed as before but the lower
end of the box is joined by a horizontal line to the
smallest value which is not an outlier, and the upper
end of the box is joined by a horizontal line to the
largest value which is not an outlier.
C
C: [p. 417] The set of complex numbers,
i.e., C = {a +bi : a, b R}.
cartesian equation: [p. 444] An equation
connecting two variables, often called x and y.
cartesian form of a complex number: [p. 417]
A complex number
expressed in the form a + bi,
represented by the ordered
pair (x, y), where x is the
real part of z and y is the
imaginary part of z.
Im(z)
Re(z)
P
0
b
z = a + bi
a

P1: FXS/ABE P2: FXS


9780521740494gsy.xml CUAU033-EVANS October 21, 2008 20:47
Glossary 623
categorical data: [p. 501] Data collected about a
variable which takes values that are categories (not
quantities). For example, data from the variable
gender which can takes the values male and female.
causation: [p. 579] A relationship between two
variables where it is known that a change in the
independent variable causes a change in the
dependent variable. This cannot be determined from
the value of the correlation coefcient.
centre: [p. 521] A measure of the location of the data
values.
chord: [p. 341] An interval with end points on a
circle.
chord length: [p. 341] AB = 2r sin

2
where
c
= mag AOB
O
A
B

r
r
circle, general cartesian equation of: [p. 441]
(x h)
2
+(y k)
2
= r
2
.
The centre of the circle is the point (h, k) and the
radius is r.
circular function equations, solution of: [p. 301]
If cos (x) = a, x = 2n cos
1
(a),
where n Z and a [1, 1]
If tan (x) = a, x = n +tan
1
(a),
where n Z and a R
If sin (x) = a, x = 2n +sin
1
(a)
or x = (2n +1) sin
1
(a),
where n Q and a [1, 1]
circular functions: [pp. 25557] The sine, cosine
and tangent functions.
cis : [p. 430] cos +i sin
class frequency: [p. 506] The number of data values
corresponding to each class interval.
class intervals: [p. 505] A range of values which
forms a sub-group of the values that a variable might
take such as 09.
common difference, d: [p. 122] The difference
between two consecutive terms of an arithmetic
sequence, i.e., d = t
n
t
n1
common ratio, r: [p. 130] The quotient of two
consecutive terms of a geometric sequence,
i.e., r =
t
n
t
n1
compass bearing (or bearing): [p. 346] The
direction measured from north clockwise.
complement of a set: [p. 65] The complement of
A is the set of all members of that are not members
of A.
complementary angles: [p. 298]
sin
_

2

_
= cos cos

2
= sin
sin
_

2
+
_
= cos cos

2
+ = sin
complementary relationships: [p. 298]
sin
_

2

_
= cos cos
_

2

_
= sin
sin
_

2
+
_
= cos cos
_

2
+
_
= sin
complex conjugate, z: [p. 421]
If z = a +bi, then z = Re(z) Im(z)i.
If z = rcis , then z = rcis ().
complex conjugate, properties of: [p. 421]
Let z = a +bi, then z = a bi.
z + z = 2 Re(z) z z = |z|
2
z
1
+ z
2
= z
1
+ z
2
z
1
z
2
= z
1
z
2
complex number: [p. 417] An expression of the
form a +bi , where a and b are real numbers.
composite: [p. 74] A natural number, m, is called a
composite if it can be written as a product m = a b
where a and b are natural numbers greater than 1 and
less than m.
compound angle formulas: [pp. 31011]
cos (x y) = cos x cos y +sin x sin y
cos (x + y) = cos x cos y sin x sin y
sin (x y) = sin x cos y cos x sin y
sin (x + y) = sin x cos y +cos x sin y
tan (x y) =
tan x tan y
1 +tan x tan y
tan (x + y) =
tan x +tan y
1 tan x tan y
conjugate factor theorem: [p. 428] If the
coefcients of
P(z) = a
n
z
n
+a
n1
z
n1
+ +a
1
z +a
0
, a
n
= 0,
where n is a natural number and a
n
, a
n1
, . . . , a
1
, a
0
are real numbers, then the complex roots occur in
conjugate pairs, i.e., if (z
1
) is a factor, so is
(z
1
)
constant acceleration (or kinematics) formulas:
[p. 471] v = u +at
s = ut +
1
2
at
2
v
2
= u
2
+2as
s =
1
2
(u +v)t
continuous data: [p. 501] Data which can take any
value (sometimes within a specied interval), such as
height, often arises from measuring.
convergent series: [p. 138] A geometric series is an
example of a convergent series with a common ratio
1 < r < 1 which will approach a limiting value as
successive terms are added to it, i.e., as n ,
S
n

a
1 r
, where a = t
1
and r =
t
n
t
n1
converse: [p. 602] q p is the converse of p q
P1: FXS/ABE P2: FXS
9780521740494gsy.xml CUAU033-EVANS October 21, 2008 20:47
624 Essential Advanced General Mathematics
conversion of polar to cartesian and vice versa:
[p. 412] For conversion of a complex number from
cartesian to polar and vice versa,
x = r cos , y = r sin and hence x
2
+ y
2
= r
2
.
correlation coefcient: see Pearsons
product-moment correlation coefcient
cosecant function: [p. 304] cosec =
1
sin
,
provided sin = 0
cosine function: [p. 255] Cosine , or cos , is
dened as the x coordinate of
the point P on the unit circle
where OP forms an
angle of radians with
the positive ray of the
x axis.
x
y
P() = (cos , sin )
sin
cos

1
1 0 1
1
cosine function, graph of: [p. 263]

y
1
1
y = cos
3
2

2
2
amplitude = 1
period = 2
cosine rule: [p. 334] For triangle ABC
A
B
C
a
b
c
a
2
= b
2
+c
2
2bc cos A or, equivalently,
cos A =
b
2
+c
2
a
2
2bc
The cosine rule is used to nd unknown quantities in
a triangle when either two sides and an included angle
are given, or three sides are given.
cotangent function: [p. 304] cot =
cos
sin
,
provided sin = 0
cumulative frequency distribution: [p. 510] A table
showing the number of values in the data set less than
or equal to the upper value of each class interval.
cumulative frequency polygon: [p. 510] The graph
of a cumulative frequency distribution, with
horizontal axis showing the values of the variable and
the vertical axis showing cumulative frequency.
cumulative relative frequency distribution:
[p. 509] A table showing the proportion of values in
the data set less than or equal to the upper value of
each class interval.
cumulative relative frequency polygon: [p. 510]
The graph of a cumulative relative frequency
distribution, with horizontal axis showing the values
of the variable and the vertical axis showing
cumulative relative frequency.
Identical in shape to the cumulative frequency
polygon but with a vertical axis scaled from 0 to 1.
D
data: [p. 501] Information collected about group,
such as height, or hair colour or age.
degree of a polynomial: [p. 151] Given by the value
of n, the highest power of x with non-zero coefcient.
dependent variable: [p. 554] The variable in a
bivariate situation which cannot be manipulated by a
researcher, or which could be considered changed as
a result of a change in the independent variable.
determinant of a matrix: [p. 16] For a 2 2 matrix
for A =
_
a b
c d
_
, det (A) = ad bc
diameter: [p. 376] A chord which includes the
centre of a circle.
difference equation (or iterative rule): [p. 116]
A rule which enables each subsequent term of a
sequence to be found using the previous term, e.g.,
t
1
= 1, t
n
= t
n1
+2
dilation from the x axis: [p. 196] In general a
dilation of a units from the x axis is described by the
rule
(x, y) (x, ay)
In general the curve with equation y = f (x) is
mapped to the curve with equation y = af (x) by the
transformation with rule (x, y) (x, ay).
dilation from the y axis: [p. 196] In general a
dilation of a units from the y axis is described by the
rule
(x, y) (ax, y)
In general the curve with equation y = f (x) is
mapped to the curve with equation y = f
_
x
a
_
by the
transformation with rule (x, y) (ax, y).
dimension of a matrix: [p. 1] The size, or
dimension, of the matrix is described by specifying
the number of rows (horizontal lines) and columns
(vertical lines) that occur in the matrix. A matrix with
m rows and n columns is said to be an m n matrix
direct variation: [p. 89] a b
n
, i.e. a varies directly
as b
n
(n R
+
)
This implies a = kb
n
, where k is the constant of
variation (k R
+
).
discrete data: [p. 501] Data that can only take
particular values (often whole numbers), such as
number of children in a family, often arises from
counting.
P1: FXS/ABE P2: FXS
9780521740494gsy.xml CUAU033-EVANS October 21, 2008 20:47
Glossary 625
discriminant, : [p. 154] = b
2
4ac
i If b
2
4ac > 0, the quadratic equation
ax
2
+bx +c = 0 has two real solutions
ii If b
2
4ac = 0, the quadratic equation
ax
2
+bx +c = 0 has one real solution
iii If b
2
4ac < 0, the quadratic equation
ax
2
+bx +c = 0 has no real solutions
displacement: [p. 464] The displacement of a
particle moving in a straight line is dened as the
change in position of the particle.
distribution: [p. 513] A term used to indicate the
pattern of the data values.
division of complex numbers: [p. 421] For
cartesian form:
z
1
z
2
=
z
1
z
2
|z
2
|
2
and z
1
=
z
|z|
2
For polar form:
z
1
z
2
=
r
1
r
2
cis (
1

2
) and z
1
=
1
r
cis ()
double angle formulas: [p. 313]
sin 2x = 2 sin x cos x
cos 2x = cos
2
x sin
2
x
= 1 2 sin
2
x
= 2 cos
2
x 1
tan 2x =
2 tan x
1 tan
2
x
E
ellipse, general cartesian equation of: [p. 446]
(x h)
2
a
2
+
(y k)
2
b
2
= 1
The centre of the ellipse is the point (h, k), the axis of
the ellipse parallel to the x axis is of length 2a units,
and the axis of the ellipse parallel to the y axis is of
length 2b units.
equal complex numbers: [p. 418] If z
1
= a +bi
and z
2
= c +di, then z
1
= z
2
if and only if a = c
and b = d
equilibrium: [p. 485] A particle is said to be in
equilibrium if the resultant force acting on it is zero,
i.e., if F = 0. In this case the particle has zero
acceleration. If the particle is at rest it remains at rest
and if it is moving it will continue to move with
constant velocity.
equivalence of vectors: [p. 398]
Let a = a
1
i +a
2
j +a
3
k, and b = b
1
i +b
2
j +b
3
k
If a = b then a
1
= b
1
, a
2
= b
2
and a
3
= b
3
Euclidean algorithm: [p. 613] A method for nding
the highest common factor of two numbers and for
solving linear Diophantine equations.
even function: [p. 263] A function f for which
f (x) = f (x).
exact values of circular functions: [p. 261]
(

) sin cos tan


0 0 1 0

6
(30

)
1
2

3
2
1

4
(45

)
1

2
1

2
1

3
(60

3
2
1
2

2
(90

) 1 0 undened
F
factor: [p. 74] A natural number, a, is a factor of a
natural number, b, if there exists a natural number, k,
such that b = ak.
factorise: [p. 74] Express as a product of factors
ve-gure summary: [p. 533] The minimum, rst
quartile, median, third quartile, and maximum of a
data set.
xed point iteration: [p. 141] Fixed point iteration
can be used to solve equations of the form f (x) = 0
by nding the sequence of numbers generated by the
equation x
n
= g(x
n1
), as long as this sequence is
convergent.
formula: [p. 42] An equation containing symbols
that states a relationship between two or more
quantities is called a formula.
A = lw (Area = length width) is an example of a
formula. The value of A, called the subject of the
formula, can be found by substituting in given values
of l and w.
frequency distribution: [p. 502] A table showing
each value that a variable can take, and how many
times each of the different values of the variable was
observed in the data set.
frequency histogram: see histogram
frequency polygon: [p. 504] An alternative to a
histogram formed by plotting the values in the
frequency histogram with points, which are then
joined by straight lines.
G
g: [p. 473] The acceleration of a particle owing to
gravity. Close to the Earths surface, the value of g is
approximately 9.8 m/s
2
.
geometric mean: [p. 133] If
a
b
=
c
a
, then a is said
to be the geometric mean of c and b (or sometimes
the mean proportional of b and c).
geometric sequence: [p. 130] A sequence in which
each successive term is found by multiplying the
previous term by a xed value, e.g. 2, 6, 18, 54, . . .
P1: FXS/ABE P2: FXS
9780521740494gsy.xml CUAU033-EVANS October 21, 2008 20:47
626 Essential Advanced General Mathematics
A geometric sequence can be dened by an
iterative equation of the form
t
n
= rt
n1
, where r is the common ratio.
The nth term of the sequence can be found using:
t
n
= ar
n1
, where a = t
1
geometric series: [p. 135] The sum of the terms in a
geometric sequence.
The sum of the rst n terms, S
n
, is given by the rule:
S
n
=
a(r
n
1)
r 1
, where a = t
1
and r =
t
n
t
n1
golden ratio (or golden section), : [p. 244]
=
1 +

5
2
golden rectangle: [p. 245] A rectangle with ratio
of side lengths 1 + :
golden section (or golden ratio), : [p. 245]
=
1 +

5
2
H
highest common factor: [p. 76] The highest
common factor of two natural numbers is the largest
natural number which is a factor of both numbers.
histogram: [p. 503] A visual display of a frequency
distribution when the data are numerical, an
alternative to a stem-and-leaf plot or boxplot.
Frequencies (vertical axis) are represented by vertical
bars corresponding to each number or class interval
(horizontal axis). Sometimes called a frequency
histogram.
hyperbola, general cartesian equation of:
[p. 447]
(x h)
2
a
2

(y k
2
)
b
2
= 1
The centre of the hyperbola is the point (h, k), and
the equations of the asymptotes are:
y k =
b
a
(x h)
I
identities derived from the Pythagorean identity:
[p. 307]
1 +tan
2
x = sec
2
x
1 +cot
2
x = cosec
2
x
cos 2x = 2 cos
2
x 1
= 1 2 sin
2
x
= cos
2
x sin
2
x
imaginary number: [p. 417] The imaginary number
i has the property i
2
= 1.
imaginary part of a complex number: [p. 417]
Im(z) is a function which denes the value of the
imaginary component of z = a +bi, i.e., Im(z) = b
independent variable: [p. 552] The variable in a
bivariate situation which can be manipulated by a
researcher, or which could be considered to be the
cause of a change in the dependent variable.
index laws: [p. 28]
a
m
a
n
= a
m+n
a
m
a
n
= a
mn
a
n
=
1
a
n
n

a = a
1
n
(a
m
)
n
= a
mn
a
0
= 1
(ab)
n
= a
n
b
n
innite geometric series (or sum to innity), S

:
[p. 138] S

=
a
1 r
, where a = t
1
and
r =
t
n
t
n1
, |r| < |
integer value function: [p. 208] The integer value
function I : R Z is dened by l(x) = [x] where
[x] is the greatest integer not exceeding x. For
example, [3, 9] = 3, [4.1] = 5
integers: [p. 63] The elements of {. . . ,2, 1, 0,
1, 2, . . .} are called integers.
integrand: [p. 474] In the expression
_
b
a
f (x) dx, the
function to be integrated, f, is called the integrand.
interquartile range or IQR: [p. 524] A measure of
the spread or variability of the distribution of
numerical data, equal to the difference between the
quartiles. That is, IQR = Q
3
Q
1
.
intersection of sets: [p. 64] The set of all the
elements that are members both of set A and of set B
is called the intersection of A and B. The intersection
of A and B is written A B.
inverse variation: [p. 93] a
1
b
n
, i.e., a varies
inversely as b
n
(n R
+
). This implies a =
k
b
n
where
k is the constant of variation (k R
+
).
irrational numbers: [p. 68] The real numbers which
are not rationals are called irrational (e.g. and

2).
iterative rule (or difference equation): [p. 116]
A rule which enables each subsequent term of a
sequence to be found using the previous term, e.g.,
t
1
= 1, t
n
= t
n1
+2
J
joint variation: [p. 103] One quantity varies with
more than one other variable. This may be a
combination of direct and/or inverse variation.
e.g., V r
2
h implies V = kr
2
h
K
kilogram weight, kg wt: [p. 485] A unit of force.
If a body has mass of one kilogram then the
P1: FXS/ABE P2: FXS
9780521740494gsy.xml CUAU033-EVANS October 21, 2008 20:47
Glossary 627
gravitational force acting on this body is one
kilogram weight.
kinematics (or constant acceleration) formulas:
[p. 471]
v = u +at
s = ut +
1
2
at
2
v
2
= u
2
+2as
s =
1
2
(u +v)t
L
Lamis theorem: [p. 487]
P
q r
p
R Q
Lamis theorem is a
trigonometrically based identity
which simplies problems
involving three forces acting
on a particle in equilibrium
when the angles between the forces are known.
P
sin p

=
Q
sin q

=
R
sinr

least squares regression line: [p. 575] The rule for a


linear relationship determined by minimising the sum
of the vertical deviations of the data points from the
line, that is, such that:
n

i =1
(y
i
a bx
i
)
2
is a minimum,
where x
i
and y
i
are the data values.
like surds: [p. 71] Surds with the same irrational
factor.
linear Diophantine equation: [p. 610] An equation
of the form ax +by = c, where a, b and c are
integers, for which a solution (x
0
, y
0
) may exist for
(x
0
, y
0
) integers. If one solution (x
0
, y
0
) exists then
the general solutions are
x = x
0
+
b
d
t , y = y
0

a
d
t ,
where t Z and d is the highest common factor of
a and b.
linear equation: [p. 33] A linear equation is a
polynomial equation of degree 1, e.g., 2x +1 = 0
linear relationship: [p. 567] A relationship which
can be described by a rule of the form
y = a +bx
where y is the dependent variable, x is the
independent variable, a is the x intercept of the line
and b is the slope of the line.
literal equation: [p. 47] An equation for the variable
x in which the coefcients of x (including the
constants) are pronumerals is known as a literal
equation. e.g., ax +b = c.
locus (plural loci): [p. 440] A set of points which
satises a given condition, e.g., the locus of points P
which satisfy PO = 3, where O is the origin, is the
circle with centre the origin and radius 3.
lower limit of integration: [p. 474] In the
expression
_
b
a
f (x) dx, the number a is called the
lower limit of integration.
M
magnitude of a vector: [p. 383] The length of a
directed line segment corresponding to the vector.
If

AB is represented by the column vector


_
x
y
_
, then
the magnitude, |

AB|, is equal to
_
x
2
+ y
2
.
If

AB is represented by the column vector


_
_
x
y
z
_
_
, then
the magnitude, |

AB|, is equal to
_
x
2
+ y
2
+ z
2
.
mass: [p. 485] The mass of an object is the amount
of matter it contains. Mass is not the same as weight.
matrices addition and subtraction: [p. 6] Addition
will be dened for two matrices only when they have
the same number of rows and the same number of
columns. In this case the sum of two matrices is found
by adding corresponding elements. For example,
_
1 0
0 2
_
+
_
0 3
4 1
_
=
_
1 3
4 3
_
matrices, equal: [p. 3] Two matrices A, B, are
equal, and can be written as A = B when
each has the same number of rows and the same
number of columns
they have the same number or element at
corresponding positions.
matrix multiplication: [p. 11] If A is an m n
matrix and B is an n r matrix, then the product AB
is the m r matrix whose entries are determined as
follows.
To nd the entry in row i and column j of AB single
out row i in matrix A and column j in matrix B.
Multiply the corresponding entries from the row and
column and then add up the resulting products.
Note: The product AB is dened only if the number of
columns of A is the same as the number of rows of B.
matrix multiplication by a scalar: [p. 6] If A is an
m n matrix, and k is a real number, then kA is an
m n matrix whose elements are k times the
corresponding elements of A. For example,
3
_
2 2
0 1
_
=
_
6 6
0 3
_
matrix, inverse of a square matrix: [p. 15] B is
said to be the inverse of A if AB = BA = I. The
inverse of a square matrix A, is denoted by A
1
. The
P1: FXS/ABE P2: FXS
9780521740494gsy.xml CUAU033-EVANS October 21, 2008 20:47
628 Essential Advanced General Mathematics
inverse is unique. It does not exist for every square
matrix.
matrix, multiplicative identity: [p. 14] For square
matrices of a given dimension, e.g. 2 2, a
multiplicative identity I exists.
For example, for 2 2 matrices I =
_
1 0
0 1
_
AI = IA = A, and this result holds for any square
matrix multiplied by the appropriate multiplicative
identity.
matrix, regular: [p. 16] A square matrix is said to
be regular if its inverse exists. Those square matrices
which do not have an inverse are called singular
matrices.
matrix, square: [p. 14] A matrix with the same
number of rows and columns is called a square
matrix.
matrix, zero: [p. 6] The m n matrix with all
elements equal to zero is called the zero matrix.
mean: [p. 521] The most commonly used measure of
centre of numerical data, calculated by summing
all the data values and dividing by the number of
values in the data set. Always denoted x, called
x-bar.
median: [p. 522] A measure of centre of the
distribution of numerical data, found by listing all the
observations in order and locating the middle value,
such that 50% of the data values are less than this
value, and 50% are above. Usually denoted m.
modal class: [p. 506] The interval which has the
highest class frequency.
mode: [p. 523] The value which occurs most
frequently, usually denoted M.
modulus, properties of: [p. 432]
The modulus of the product of two complex
numbers is the product of their moduli, i.e.,
|z
1
z
2
| = |z
1
||z
2
|
The modulus of the quotient of two complex
numbers is the quotient of their moduli, i.e.,

z
1
z
2

=
|z
1
|
|z
2
|
modulus function: [p. 208] see absolute value
function
modulus of a complex number, |z|: [p. 431] The
distance of the complex number from the origin, also
known as the magnitude or absolute value of z.
If z
1
= a +bi , then |z| =

a
2
+b
2
modulus-argument (or polar) form of a complex
number: [p. 431] A complex number expressed in
the form r cis ,
represented by the
ordered pair [r, ],
where r is the
modulus of z and
is an argument of z.
Im(z)
z = a + bi
P
b
a
r
0 Re(z)

multiplication of a complex number by a real


number: [p. 419] If z = a +bi , then
kz = ka +kbi, k R.
If z = r cis , then
kz =
_
_
_
kr cis k > 0
kr cis ( +) k < 0 and < < 0
kr cis ( ) k < 0 and 0 <
multiplication of a complex number by i: [pp. 419,
432] Geometrically, a 90

rotation of the complex


number about the origin in an anticlockwise direction,
i.e., if z
1
= a +bi , then
i z
1
= i (a +bi ) = b +ai
multiplication of a vector by a scalar: [pp. 392,
432] If a = a
1
i +a
2
j +a
3
k, then
ma = ma
1
i +ma
2
j +ma
3
k, m R.
multiplication of complex numbers: [pp. 420, 432]
If z
1
= a +bi and z
2
= c +di , then
z
1
z
2
= (ac bd) +(ad +bc)i
If z
1
= r
1
cis
1
and z
2
= r
2
cis
2
, then
z
1
z
2
= r
1
r
2
cis (
1
+
2
)
Geometrically, the effect of multiplying z
1
by z
2
is to
produce an enlargement of Oz
1
, where O is the
origin, by a factor r
2
and an anticlockwise turn
through an angle
2
about the origin.
N
natural numbers: [p. 63] The elements of
{1, 2, 3, 4, . . .} are called the natural numbers.
negative association: [pp. 561, 565] When smaller
values of y are associated with larger values of x,
identied by a downward trend in a scatterplot.
negatively skewed distribution: [p. 514] A
distribution which has a long tail to the left and a
short tail pointing to the right.
no association: [p. 564] When the values of y are not
related to the values of x, identied by no trend in a
scatterplot.
normal reaction force: [p. 485] If a particle lies on a
smooth surface and exerts a force on the surface, then
the surface exerts a force R N on the particle, which
acts at right angles to the surface and is called the
normal reaction force.
numerical data: [p. 501] Data collected about a
variable which takes values that are quantities (not
categories). For example, data from the variable
height (cm).
O
odd function: [p. 263] A function f for which
f (x) = f (x).
P1: FXS/ABE P2: FXS
9780521740494gsy.xml CUAU033-EVANS October 21, 2008 20:47
Glossary 629
ordered pair: [p. 64] An ordered pair, denoted (x, y),
is a pair of elements x and y in which x is considered
to be the rst element and y the second.
outlier: [p. 536] A value which sits away from the
main body of the data in a plot. Formally dened as
a value more than 1.5 IQR below Q
1
, or more than
1.5 IQR above Q
3
.
P
part variation: [p. 106] The value of one variable is
the sum of two or more quantities each of which is
determined by a variation. In some cases, one of
those quantities may be constant. e.g. A = k
1
r +k
2
r
2
where k
1
, k
2
are constants of variation.
partial fractions: [p. 162] Some rational algebraic
functions may be expressed as a sum of partial
fractions, e.g.
A
ax +b
+
B
(cx +d)
+
C
(cx +d)
2
+
Dx + E
(ex
2
+ f x + g)
particle model: [p. 486] This means that an object is
considered as a point. This can be done when the size
(dimension) of the object can be neglected in
comparison with other lengths in the problem being
considered, or when rotational motion effects can be
ignored.
Pearsons product-moment correlation coefcient:
[p. 564] Also called the correlation coefcient,
denoted r, where 0 r 1 and dened as:
r =
1
n 1
n

i =1
_
x
i
x
s
x
__
y
i
y
s
y
_
where x and s
x
are the mean and standard deviation
of the x scores, y and s
y
are the mean and standard
deviation of the y scores, and n is the number of data
values.
percentage frequency: [p. 509] The frequency
expressed as a percentage of the total number of data
values, obtained by multiplying the relative frequency
by 100.
period of a function: [p. 262] The period of a
function f with domain R is the smallest positive
number a such that f (x +a) = f (x) for a in R.
For example, the period of the sine function is 2 as
sin (x +2) = sin x.
For functions of the form y = a cos(nx +) +c or
y = a sin (nx +) +c the period is given by
2
n
.
For functions of the form y = a tan(nx +) +c the
period is given by

n
.
periodic function: [p. 262] A function which repeats
itself regularly.
polar axis: [p. 411] The polar axis is a ray from a
point O from which the angle of a polar coordinate is
measured.
polar coordinates: [p. 411] The position of a point
in the plane is determined through an ordered pair
[r, ], the polar coordinates. The rst entry is the
distance from the pole and the second is the angle
measured from the polar axis.

pole
O
polar axis
P[r, ]
is measured in an
anticlockwise direction
from the polar axis
polar (or modulus-argument) form of a complex
number: [p. 430]
A complex number
expressed in the form
r cis , represented by
the ordered pair [r, ],
where r is the modulus of z and is an argument of z.
Im(z)
z = a + bi
P
b
a
r
0 Re(z)

pole: [p. 411] A point from which the polar axis


emanates.
polynomial function: [p. 151] A rule of the type
y = a
n
x
n
+a
n1
x
n1
+ a
1
x +a
0
, n N
where a
0
, a
1
, . . . a
n
are numbers called coefcients.
position: [p. 463] The position of a particle moving
in a straight line is determined by its distance from a
xed point O on the line, called the origin, and
whether it is to the right or left of O. Conventionally
the direction to the right of the origin is considered to
be positive.
position vector: [p. 384] A position vector,

OP,
indicates the position in space of the point P relative
to the origin O.
positive association: [pp. 561, 565] When larger
values of y are associated with larger values of x,
identied by an upward trend in a scatterplot.
positively skewed distribution: [p. 514]
A distribution which has a short tail to the left and a
long tail pointing to the right.
prediction: [p. 571] Substituting a value of the
independent variable in the rule for the relationship
between two variables to determine the value of the
dependent variable.
prime: [p. 75] If a natural number greater than 1 has
only factors 1 and itself, it is said to be prime.
prime decomposition: [p. 75] This method of
expressing a composite in terms of a product of
powers of prime numbers is called prime
decomposition.
Pythagoras theorem: [p. 70] For a right-angled
triangle, the square of the hypotenuse is equal to the
sum of the squares of the other two sides, i.e.,
(hyp)
2
= (opp)
2
+(adj)
2
Pythagorean identity: [p. 306] cos
2
+sin
2
= 1
P1: FXS/ABE P2: FXS
9780521740494gsy.xml CUAU033-EVANS October 21, 2008 20:47
630 Essential Advanced General Mathematics
Q
Q
1
or rst quartile: [p. 524] The value such that
25% of the data values are less than this value, and
75% are above, found by locating the median of the
lower half of the data set. Also called the lower
quartile.
Q
2
: [p. 524] The median.
Q
3
or third quartile: [p. 524] The value such that
75% of the data values are less than this value, and
25% are above, found by locating the median of the
upper half of the data set. Also called the upper
quartile.
q-correlation coefcient: [p. 557] A measure of
strength of a relationship between two numerical
variables, with 0 q 1 and dened as
q =
(a +c) (b +d)
a +b +c +d
where a is the number of data points in quadrant A,
b is the number of data points in quadrant B, d is the
number of data points in quadrant C, d is the number
of data points in quadrant D.
quadrant: [p. 558] A section of the scatterplot found
by nding the median of all the x values in the data
set, and drawing a vertical line through this value, and
nding the median of all the y values in the data set,
and drawing a horizontal line through this value. The
resulting four areas are labelled quadrant A, quadrant
B, quadrant C, and quadrant D as shown.
x
y
A B
C
0
D
quadratic formula: [p. 154] An equation of the
form az
2
+bz +c = 0 may be solved quickly by
using the quadratic formula:
z =
b

b
2
4ac
2a
quadratic function: [p. 154] A polynomial function
of degree two with general rule:
y = ax
2
+bx +c, x R
quadratic surd: [p. 69] A number of the form

a, where a is a rational number which is not the


square of another rational number.
R
R
+
: [p. 64] R
+
= {x : x > 0}
R

: [p. 64] R

= {x : x < 0}
R \{0}: [p. 64] R\{0} is the set of real numbers
excluding 0.
R
2
: [p. 64] R
2
= {(x, y) : x, y R}. That is, R
2
is
the set of all ordered pairs of real numbers.
radian: [p. 253] One radian (written 1
c
) is the angle
subtended at the centre of the unit circle by an arc of
length 1 unit.
range of a set of data: [p. 523] The difference
between the smallest and the largest observations.
rate: [p. 158] Describes how a certain quantity
changes with respect to the change in another
quantity (often time).
rational algebraic functions: [p. 162] Functions
which have a rule of the form:
f (x) =
P(x)
Q(x)
, whereP(x) and Q(x)
are polynomials.
rational numbers: [p. 67] The numbers of the form
p
q
with p and q integers, q = 0, are called rational
numbers.
real part of a complex number: [p. 417] Re(z) is a
function which denes the real component of
z = a +bi , i.e., Re(z) = a.
reciprocal circular functions: [p. 304] The
cosecant, secant and cotangent functions.
reciprocal functions: [p. 162] Functions which have
a rule of the form:
f (x) =
1
P(x)
, where P(x) is a polynomial.
rectangular hyperbola: [p. 204] The basic
rectangular hyperbola has equation y =
1
x
.
reection in the x axis: [p. 194] In general a
reection in the x axis is described by the rule
(x, y) (x, y)
In general the curve with equation y = f (x) is
mapped to the curve with equation y = f (x) by
the transformation with rule (x, y) (x, y).
reection in the y axis: [p. 194] In general a
reection in the y axis is described by the rule
(x, y) (x, y)
In general the curve with equation y = f (x) is
mapped to the curve with equation y = f (x) by
the transformation with rule (x, y) (x, y).
reection in y = x: [p. 194] In general a reection
in line y = x is described by the rule
(x, y) (y, x)
In general the curve with equation y = f (x) is
mapped to the curve with equation x = f (y) by
the transformation with rule (x, y) (y, x).
relative frequency: [p. 509] The frequency
expressed as a proportion of the total number of data
values, obtained by dividing the class frequency by
the sample size.
P1: FXS/ABE P2: FXS
9780521740494gsy.xml CUAU033-EVANS October 21, 2008 20:47
Glossary 631
resultant force: [p. 485] The vector sum of the
forces acting at a point.
S
scatterplot: [p. 552] A visual display of bivariate
numerical data, with the independent variable on the
horizontal (x) axis and the dependent variable (y) on
the vertical axis. Individual data values are
represented by a point.
scientic notation: see standard form
secant function: [p. 304] sec =
1
cos
provided
cos = 0
sector: [p. 341] Two radii and
an arc dene a region called
a sector. In this diagram
with circle centre O, the
shaded region is a
minor sector and the
unshaded region is a
major sector.
Area of sector =
1
2
r
2
where
c
= mag AOB
A
C
B
O D
segment: [p. 342] Every
chord divides the interior of
a circle into two regions called
segments. The smaller is called
the minor segment, the larger
is the major segment. In this
diagram the minor segment
has been shaded.
Area of segment =
1
2
r
2
( sin )
where
c
= mag AOB
A
B
O
r

sequence: [p. 115] A set of numbers for which order


is important.
series: [p. 125] The sum of the terms in a sequence.
set notation: [p. 64]
means is an element of
/ means is not an element of
is the empty or null set, containing no elements.
means is a subset of
means the intersection of
means the union of
means the universal set, or the set of all
elements being considered
sets of numbers: [p. 67]
R is the set of real numbers
Q is the set of rational numbers
Z is the set of integers
N is the set of natural numbers
C is the set of complex numbers
sigma: [p. 521] Greek letter, when denoted

used
to mean add up, so that the symbol

x means add
up all the data values.
signs of circular functions: [p. 259] These
symmetry properties can be summarised
for the signs of sin, cos and tan
for the four quadrants as follows:
x
y
A S
T C
1st quadrant all are positive (A)
2nd quadrant sin is positive (S)
3rd quadrant tan is positive (T)
4th quadrant cos is positive (C)
similar gures: [p. 229] Two gures are similar if
one is congruent to an image of the other under an
expansion from the origin of factor k.
similar triangles, conditions of: [p. 230] Two
triangles are similar if one of the following conditions
holds:
triangles have equal angles (AAA)
corresponding sides are in the same ratio (PPP)
A

AB
=
B

BC
=
A

AC
= k, where k is the
expansion (enlargement) factor
two pairs of corresponding sides have the same
ratio and the included angles are equal (PAP)
A

AB
=
A

AC
two pairs of corresponding sides have the same
ratio and two corresponding non-included
angles are equal, provided these angles are right
angles or obtuse.
simplest form: [p. 71] When the number under the
square root has no factors which are squares of a
rational number, then a surd is in its simplest form.
simultaneous equations: [pp. 34, 169] Equations of
two or more lines or curves in a cartesian plane, the
solution of which is the point of intersection of the
pairs of lines or curves.
sine function: [p. 255] Sine ,
or sin , dened as the
y coordinate of the point
P on the unit circle where
OP forms an angle of
radians with the
positive ray of the x axis.
x
y
1
1
1
1 0
cos
sin
P() = (cos , sin )

sine function, graph of: [p. 263]


y
2
1
y = sin
amplitude = 1
period = 2

1
P1: FXS/ABE P2: FXS
9780521740494gsy.xml CUAU033-EVANS October 21, 2008 20:47
632 Essential Advanced General Mathematics
sine rule: [p. 331] For triangle ABC
A
B
C
a c
b
a
sin A
=
b
sin B
=
c
sin C
The sine rule is used to nd unknown quantities in a
triangle when either one side and two angles are
given, or two sides and a non-included angle are
given.
speed: [p. 465] The magnitude of velocity.
speed, average: [p. 465] The average speed of a
particle for a time interval [t
1
, t
2
] is equal to
distance travelled
t
2
t
1
spread: [p. 523] A measure of how spread out the
data values are; a measure of the variability of the
data.
standard deviation: [p. 526] A measure of the
spread or variability of the distribution of numerical
data about the mean, denoted s and dened as
s =
_
1
n 1
n

i =1
(x
i
x)
2
standard form: [p. 30] Standard form involves
expressing the number as a product of a number
between 1 and 10 and a power of ten and is also
called scientic notation.
statistic: [p. 520] Any value computed from a set
of data.
stem-and-leaf plot: [p. 515] A visual display of a
numerical data set, an alternative to a histogram or
boxplot when the data set is small (less than 50).
Leading digits are shown as the stem, and the nal
digit as the leaf in the plot. Also called a
stemplot.
stemplot: see stem-and-leaf plot
subtraction of complex numbers: [p. 418]
If z
1
= a +bi and z
2
= c +di
then z
1
z
2
= (a +bi ) (c +di )
= (a c) +(b d)i
subtraction of vectors: [p. 392] Let
a = a
1
i +a
2
j +a
3
k, and b = b
1
i +b
2
j +b
3
k
Then a b = (a
1
b
1
)i +(a
2
b
2
) j +(a
3
b
3
)k
sum to innity (or innite geometric series), S

:
[p. 138] S

=
a
1 r
, where a = t
1
and r =
t
n
t
n1
summary statistics: [p. 520] Statistics which
numerically summarise special features of a data set,
such as centre and spread.
surd (quadratic surd): [p. 69] A number of the form

a where a is a rational number which is not a square


of another rational number is called a quadratic
surd.
surd (surd of the nth order): [p. 70] If a is a
rational which is not a perfect nth power,
n

a is called
a surd of the nth order.
symmetric distribution: [p. 514] A distribution
which forms a mirror image of itself when folded in
the middle along a vertical axis.
T
tangent function: [p. 257] If a tangent to the unit
circle is drawn at A then the y coordinate of C, the
point of intersection of the
extension of OP and the
tangent is called
tangent , or tan .
x
y
1
1 1
1
0
cos
sin
tan

C(1, y)
B
A
D
P()
transformation: [p. 191] A transformation T is a
mapping from R
2
to R
2
such that if
T(a, b) = T(c, d) then a = c and b = d.
translation: [p. 191] A translation is a
transformation for which each point in the plane is
moved the same distance in the same direction.
trigonometric ratios: [p. 327]
sin =
opposite
hypotenuse
cos =
adjacent
hypotenuse
tan =
opposite
adjacent
opposite side
adjacent side
hypotenuse

U
union of sets: [p. 64] The set of elements that are in
either set A or set B (or both) is the union of sets A
and B. The union of A and B is written A B.
unit vector: [p. 399] A vector of magnitude 1. For a
given vector a the unit vector with the same direction
as a is denoted by a and a =
a
|a|
.
i, j and k are unit vectors in the positive directions of
the x, y and z axes respectively.
P1: FXS/ABE P2: FXS
9780521740494gsy.xml CUAU033-EVANS October 21, 2008 20:47
Glossary 633
V
variance: [p. 526] A measure of the spread or
variability of the distribution of numerical data about
the mean, denoted s
2
and dened as
s
2
=
1
n 1
n

i =1
(x
i
x)
2
vector: [p. 390] A set of equivalent directed line
segments.
vector quantity: [p. 390] A quantity determined by
its magnitude and direction, e.g., displacement,
velocity, acceleration, force.
vectors, properties of: [pp. 39095]
a +b = b +a commutative law
for vector addition
(a +b) +c = a +(b +c) associative law for
vector addition
a +0 = a zero vector
a +a = 0 a is the opposite
or inverse vector
m(a +b) = ma +mb distributive law
where m R
a is parallel to b if there exists k R\{0} such
that a = kb
velocity: [p. 465] The velocity of a particle is dened
as the rate of change of its position with respect to
time.
velocity, average: [p. 465] The average velocity of a
particle is the change in its position over a period of
time, i.e.,
x
2
x
1
t
2
t
1
.
velocity, instantaneous: [p. 465] The instantaneous
velocity of a particle, v =
dx
dt
where x is a function of
time, species the rate of change at a given instant in
time.
velocitytime graphs: [p. 475] These graphs present
information about
acceleration (gradient)
velocity (ordinates)
displacement (signed area or denite integral)
distance travelled (area under curve)
W
weight [p. 485] A mass of m kg, on the Earths
surface, has a force of m kg wt or mg Newtons acting
on it. This force is known as the weight.
Z
zero vector, 0 [p. 392] A line segment of zero length
with no direction.
P1: FXS/ABE P2: FXS
9780521740494ans-1-10.xml CUAU033-EVANS August 22, 2009 11:0
Answers
Chapter 1
Exercise 1A
1 a 2 2 b 2 3 c 1 4 d 4 1
2 a
_
_
_
_
_
_
1 0 0 0 1
0 1 0 1 0
0 0 1 0 0
0 1 0 1 0
1 0 0 0 1
_

_
b
_
_
_
_
_
_
1 1 1 1 1
1 1 1 1 1
1 1 1 1 1
1 1 1 1 1
1 1 1 1 1
_

_
3
_
_
_
_
_
_
1 0 0 0 0
0 1 0 0 0
0 0 1 0 0
0 0 0 1 0
0 0 0 0 1
_

_
Only the seats for top-left
to bottom-right diagonal
are occupied.
4
_
200 180 135 110 56 28
110 117 98 89 53 33
_
5 a [0 x] = [0 4] if x = 4
b
_
4 7
1 2
_
=
_
x 7
1 2
_
if x = 4
c
_
2 x 4
1 10 3
_
=
_
y 0 4
1 10 3
_
=
_
2 0 4
1 10 3
_
if x = 0, y = 2
6 a x = 2, y = 3 b x = 3, y = 2
c x = 4, y = 3 d x = 3, y = 2
7
_
_
_
_
0 3 1 0
3 0 2 1
1 2 0 1
0 1 1 0
_

_
8
_
_
_
_
_
_
21 5 5
8 2 3
4 1 1
14 8 60
0 1 2
_

_
Exercise 1B
1 X +Y =
_
4
2
_
, 2X =
_
2
4
_
,
4Y +X =
_
13
2
_
, X Y =
_
2
2
_
,
3A =
_
3 3
6 9
_
, 3A +B =
_
1 3
7 7
_
2 a
_
6 12 4
8 4 2
_
b
_
5 1 0
18 7 13
_
c
_
_
_
5
3
1
3
0
6
7
3
13
3
_

_
3 2A =
_
2 2
0 4
_
, 3A =
_
3 3
0 6
_
,
6A =
_
6 6
0 12
_
4 a Yes b Yes
5 a
_
6 4
4 4
_
b
_
0 9
12 3
_
c
_
6 5
8 1
_
d
_
6 13
16 7
_
6 a
_
0 1
2 3
_
b
_
2 3
6 3
_
c
_
3 3
1 7
_
7 X =
_
2 4
0 3
_
, Y =
_
_
_
9
2
23
2
1
2
11
_

_
8 X +Y =
_
310 180 220 90
200 0 125 0
_
, representing
the total production at two factories in two
successive weeks.
658
P1: FXS/ABE P2: FXS
9780521740494ans-1-10.xml CUAU033-EVANS August 22, 2009 11:0
A
n
s
w
e
r
s
Answers 659
Exercise 1C
1 AX =
_
4
5
_
, BX =
_
4
1
_
, AY =
_
5
8
_
,
IX =
_
2
1
_
, AC =
_
0 1
1 2
_
,
CA =
_
1 1
0 1
_
, (AC)X =
_
1
0
_
,
C(BX) =
_
9
5
_
, AI =
_
1 2
1 3
_
,
IB =
_
3 2
1 1
_
, AB =
_
1 0
0 1
_
,
BA =
_
1 0
0 1
_
, A
2
=
_
3 8
4 11
_
,
B
2
=
_
11 8
4 3
_
, A(CA) =
_
1 3
1 4
_
,
A
2
C =
_
2 5
3 7
_
2 a AY, CI are dened, YA, XY, X
2
, XI are not
dened.
b AB =
_
0 0
0 0
_
3 No
4 LX = [7], XL =
_
4 2
6 3
_
5 AB and BA are not dened unless m = n.
6
_
1 0
0 1
_
7 One possible answer is
A =
_
1 2
3 4
_
, B =
_
2 1
1.5 0.5
_
8 One possible answer is A =
_
1 2
4 3
_
,
B =
_
0 1
2 3
_
,C =
_
1 2
2 1
_
,
A(B +C) =
_
1 11
4 24
_
,
AB +AC =
_
1 11
4 24
_
,
(B +C)A =
_
11 7
16 12
_
9
_
29
8.50
_
represents John spending 29 minutes
consuming food which cost him $8.50.
_
29 22 12
8.50 8.00 3.00
_
Johns friends spent
$8.00 and $3.00 and took 22 and 12 minutes
respectively to consume their food.
10
_
_
_
_
_
_
6.00
8.00
2.00
11.00
6.50
_

_
represents how much each student
spends in a week on magazines.
11 a SC =
_
s
11
c
1
+s
12
c
2
+s
13
c
3
s
21
c
1
+s
22
c
2
+s
23
c
3
_
b SC represents the income from car sales for
each showroom.
c SC =
_
s
11
c
1
+s
12
c
2
+s
13
c
3
s
11
u
1
+s
12
u
2
+s
13
u
3
s
21
c
1
+s
22
c
2
+s
23
c
3
s
21
u
1
+s
22
u
2
+s
23
u
3
_
represents the income for each showroom for
new car sales and used car sales.
d CV gives the prot on each new car and each
used car for the three models.
Exercise 1D
1 a 1 b
_
2 2
3 2
_
c 2 d
1
2
_
2 2
3 2
_
2 a
_
1 1
4 3
_
b
_
_
_
2
7
1
14
1
7
3
14
_

_
c
_
1 0
0
1
k
_
d
_
cos sin
sin cos
_
3 A
1
=
_
_
1
2
1
2
0 1
_
_
, B
1
=
_
1 0
3 1
_
,
AB =
_
5 1
3 1
_
, (AB)
1
=
_
_
_
1
2
1
2
3
2
5
2
_

_,
A
1
B
1
=
_
1
1
2
3 1
_
,
B
1
A
1
=
_
_
_
_
1
2
1
2
3
2
5
2
_

_
, (AB)
1
= B
1
A
1
4 a
_
_
1
2
3
2
1 2
_
_
b
_
0 7
1 8
_
c
_
_
_
5
2
7
2
11
2
21
2
_

_
5 a
_
_
_
3
8
11
8
1
16
7
16
_

_ b
_
_
_
11
16
17
16
1
4
3
4
_

_
P1: FXS/ABE P2: FXS
9780521740494ans-1-10.xml CUAU033-EVANS August 22, 2009 11:0
A
n
s
w
e
r
s
660 Essential Advanced General Mathematics
6
_
_
_
1
a
11
0
0
1
a
22
_

_
8
_
1 0
0 1
_
,
_
1 0
0 1
_
;
_
1 0
k 1
_
,
_
1 0
k 1
_
;
_
1 k
0 1
_
,
_
1 k
0 1
_
, k R
_
_
a b
1 a
2
b
a
_
_
, b = 0
Exercise 1E
1 a
_
3
10
_
b
_
5
17
_
2 a
_
_
_
1
14
3
14
_

_ b
_
_
_
4
7
2
7
_

_
3 a x =
1
7
, y =
10
7
b x = 4, y = 1.5
c x =
30
7
, y =
2
7
d x = 2.35, y = 0.69
4 (2, 1) 5 books $12, CDs $18
6 a
_
2 3
4 6
_ _
x
y
_
=
_
3
6
_
b
_
2 3
4 6
_
is a singular matrix, not a
regular matrix.
c There is no unique solution for this system, but
a solution can be found.
d The solution set contains an innite number of
pairs.
7 p = 2, q = 4, r = 1, s = 2
Multiple-choice questions
1 B 2 E 3 C 4 E 5 C
6 A 7 E 8 A 9 E 10 D
Short-answer questions
(technology-free)
1 a
_
0 0
12 8
_
b
_
0 0
8 8
_
2
_
_
a
2
3
4
a
_
_
, a R
3 a AB does not exist, AC, CD, BE exist.
b DA =
_
14 0
_
, A
1
=
1
7
_
1 2
3 1
_
4 AB =
_
2 0
2 2
_
, C
1
=
_
_
2 1
3
2

1
2
_
_
5
_
1 2
3 5
_
6 A
2
=
_
_
_
_
4 0 0
0 4 0
0 0 4
_

_
, A
1
=
_
_
_
_
_
_
_
1
2
0 0
0 0
1
2
0
1
2
0
_

_
7 8
8 a i
_
3 5
5 8
_
ii
_
1 18
18 19
_
iii
1
7
_
3 1
1 2
_
b x = 2, y = 1
Extended-response questions
1 a i
_
5 0
6 2
_
ii
_
1 2
4 6
_
iii
_
12 1
17 6
_
iv
_
_
7
2
2
1 7
_
_
b i
_
11 1
18 9
_
ii
1
13
_
4 1
1 3
_
iii
1
13
_
13 2
13 7
_
iv
1
13
_
7 5
22 1
_
2 a
_
_
8 2 11
5 3 1
14 18 7
_
_
b
_
_
2 6 6
3 3 3
15 12 3
_
_
c
_
_
3 3 3
12 6 4
14 9 2
_
_
d
_
_
_
_
_
_
_
50
33
2
11
2
11
7
33
5
11
5
11
1
33
4
11
7
11
_

_
e
1
33
_
_
0 33 0
18 70 10
6 5 29
_
_
f A
1
CBC
1
g C
1
B
3 a i
_
2 3
4 1
_ _
x
y
_
=
_
3
5
_
ii 14,
1
14
_
1 3
4 2
_
iii
1
7
_
9
1
_
iv
_
9
7
,
1
7
_
is the point of intersection
of the two lines
b i
_
2 1
4 2
_ _
x
y
_
=
_
3
8
_
ii 0; A is a singular matrix
c lines represented by the equations are parallel
P1: FXS/ABE P2: FXS
9780521740494ans-1-10.xml CUAU033-EVANS August 22, 2009 11:0
A
n
s
w
e
r
s
Answers 661
4 a
_
79 78 80
80 78 82
_
b
_
_
0.2
0.3
0.5
_
_
c Semester 1: 79.2, Semester 2: 80.4
d Semester 1: 83.8, Semester 2: 75.2
e No aggregate is 318.6
f 3 marks
5 a
_
_
_
_
10 2
8 4
8 8
6 10
_

_
b
_
70
60
_
c Term 1: $820, Term 2: $800,
Term 3: $1040, Term 4: $1020
d
_
_
_
_
2 2 1
2 2 1
3 4 2
3 4 2
_

_
e
_
_
60
55
40
_
_
f Term 1: $270, Term 2: $270,
Term 3: $480, Term 4: $480
g Term 1: $1090, Term 2: $1070,
Term 3: $1520, Term 4: $1500
Chapter 2
Exercise 2A
1 a x
7
b a
2
c x
3
d y
4
e x
12
f p
7
g a
1
6
h a
8
i y
14
j x
15
k a
12
l x
2
m n
2
n 8x
7
2
o a
p x
4
q
1
2n
6
r 8x
2
s a
2
b
5
t 1
2 a 5 b 4 c
4
3
d
1
4
e
6
7
f 3 g 12 h 16 i 27
j
3
2
k 1 l 8
3 a 18.92 b 79.63 c 5.89
d 125 000 e 0.9 f 1.23
g 0.14 h 1.84 i 0.4
4 a a
4
b
7
b 64a
4
b
7
c b
d a
6
b
9
e 2a
4
b
7
f
a
2
b
5
128
5 2
2n4
6 6
3x
7 a
_
1
2
_
1
6
b a
11
20
c 2
5
6
d 2
19
6 e 2
3
5
8 a a
1
3
b b a
5
2
b
1
2
c ab
1
5
d
_
b
a
_
1
2
e a
5
2
b
1
2
c
4
f a
1
5
b
3
5
g a
4
b
7
2
c
5
Exercise 2B
1 a 4.78 10 b 6.728 10
3
c 7.923 10 d 4.358 10
4
e 2.3 10
3
f 5.6 10
7
g 1.200 034 10 h 5.0 10
7
i 2.3 10
10
j 1.3 10
9
k 1.65 10
5
l 1.4567 10
5
2 a 1.0 10
8
b 1.66 10
23
c 5 10
5
d 1.85318 10
3
e 9.463 10
12
f 2.998 10
10
3 a 75 684 000 000 000 b 270 000 000
c 0.000 000 000 000 19
4 a 0.000 0567 b
262
2625
5 a 11.8 b 4.76 10
7
Exercise 2C
1 a x =
8
3
b x = 48 c x =
20
3
d x = 63 e x = 0.7 f x = 2.4
g x = 0.3 h x = 6 i x =
15
92
j x =
21
17
2 a x =
160
9
b x = 19.2 c x =
68
7
d x =
80
51
e x = 6.75 f x =
85
38
g x =
487
13
h x =
191
91
3 a x =
18
13
, y =
14
13
b x =
16
11
, y =
18
11
c x = 12, y = 17 d x = 8, y = 2
e x = 0, y = 2 f x = 1, y = 6
Exercise 2D
1 a 4(x 2) = 60; x = 17
b
_
2x +7
4
_
2
= 49; x = 10.5
c x 5 = 2(12 x) ; x =
29
3
d y = 6x 4 e Q = np
f R = 1.1pS g
60n
5
= 2400
h a =

3
(x +3)
2 $2500 3 Eight dresses and three handbags
4 8.375 m by 25.125 m 5 $56.50
6 Nine 7 20, 34 and 17
8 Annie, Belinda and Cassie scored 165, 150 and
189 respectively
9 15 km/h 10 2.04 10
23
g
P1: FXS/ABE P2: FXS
9780521740494ans-1-10.xml CUAU033-EVANS August 22, 2009 11:0
A
n
s
w
e
r
s
662 Essential Advanced General Mathematics
Exercise 2E
1 140.625 km 2 50
3 10 000 adults and 5000 children
4 Men $220, boys $160
5 127 and 85
6 252 litres of 40% solution and 448 litres of 15%
solution
7 120 and 100, 60
8 370 588 9 500 adults, 1100 students
Exercise 2F
1 a 25 b 330 c 340.47 d 1653.48
e 612.01 f 77.95 g 2.42 h 2.1
i 9.43 j 9.54
2 a a =
v u
t
b l =
2S
n
a
c b =
2A
h
d I =
_
P
R
e a =
2(s ut )
t
2
f v =
_
2E
m
g h =
Q
2
2g
h x =
z
y
i x =
b(c + y)
a c
j x =
b(c +1)
m c
3 a 82.4

F b C =
5(F 32)
9
; 57.22

C
4 a 1080 b n =
S
180
+2 : 9
5 a 115.45 cm
3
b 12.53 cm c 5.00 cm
6 a 66.5 b 4 c 11
Exercise 2G
1 a
13x
6
b
5a
4
c
h
8
d
5x 2y
12
e
3y +2x
xy
f
7x 2
x(x 1)
g
5x 1
(x 2)(x +1)
h
7x
2
36x +27
2(x +3)(x 3)
i
4x +7
(x +1)
2
j
5a
2
+8a 16
8a
k
4(x
2
+1)
5x
l
2x +5
(x +4)
2
m
3x +14
(x 1)(x +4)
n
x +14
(x 2)(x +2)
o
7x
2
+22x +28
(x 2)(x +2)(x +3)
p
(x y)
2
1
x y
q
4x +3
x 1
r
3 2x
x 2
2 a 2xy
2
b
xy
8
c
2
x
d
x
y
2
e
a
3
f
1
2x
g
x 1
x +4
h x +2
i
x 1
x
j
a
4b
k
2x
x +2
l
x 1
4x
m
x +1
2x
n
1
3
x(x +3)
o
x 2
3x(3x 2)(x +5)
3 a
3
x 3
b
4x 14
x
2
7x +12
c
5x 1
x
2
+ x 12
d
2x
2
+10x 6
x
2
+ x 12
e
2x 9
x
2
10x +25
f
5x 8
(x 4)
2
g
1
3 x
h
23 3x
x
2
+ x 12
i
5x
2
3x
x
2
9
j
11 2x
x
2
10x +25
k
12
(x 6)
3
l
9x 25
x
2
7x +12
4 a
3 x

1 x
b
2

x 4 +6
3

x 4
c
5

x +4
d
x +7

x +4
e
12x
2

x +4
f
9x
2
(x +2)
2

x +3
5 a
6x 4
(6x 3)
2
3
b
3
(2x +3)
2
3
c
3 3x
(x 3)
2
3
Exercise 2H
1 a x =
m n
a
b x =
b
b a
c x =
bc
a
d x =
5
p q
e x =
m +n
n m
f x =
ab
1 b
g x = 3a h x = mn
i x =
a
2
b
2
2ab
j x =
p q
p +q
k x =
3ab
b a
l x =
1
3a b
m x =
p
2
+ p
2
t +t
2
q( p +t )
n x =
5a
3
4 a x =
d bc
1 ab
, y =
c ad
1 ab
b x =
a
2
+ab +b
2
a +b
, y =
ab
a +b
c x =
t +s
2a
, y =
t s
2b
d x = a +b, y = a b
P1: FXS/ABE P2: FXS
9780521740494ans-1-10.xml CUAU033-EVANS August 22, 2009 11:0
A
n
s
w
e
r
s
Answers 663
e x = c, y = a
f x = a +1, y = a 1
5 a s = a(2a +1) b s =
2a
2
1 a
c s =
a
2
+a +1
a(a +1)
d s =
a
(a 1)
2
e s = 3a
3
(3a +1) f s =
3a
a +2
g s = 2a
2
1 +
1
a
2
h s =
5a
2
a
2
+6
Exercise 2I
1 a x = a b b x = 7
c x =
a

a
2
+4ab 4b
2
2
d x =
a +c
2
2 a (x 1)(x +1)(y 1)(y +1)
b (x 1)(x +1)(x +2)
c (a
2
12b)(a
2
+4b)
d (a c)(a 2b +c)
3 a x =
a +b +c
a +b
y =
a +b
c
b x =
(a b c)
a +b c
y =
a b +c
a +b c
Multiple-choice questions
1 A 2 A 3 C 4 A 5 B
6 E 7 B 8 B 9 B 10 B
Short-answer questions
(technology-free)
1 a x
12
b y
9
c 15x
11
2
d x
1
2 3.84 10
8
3 a
2x + y
10
b
4y 7x
xy
c
7x 1
(x +2)(x 1)
d
7x +20
(x +2)(x +4)
e
13x
2
+2x +40
2(x +4)(x 2)
f
3(x 4)
(x 2)
2
4 a
2
x
b
x 4
4x
c
x
2
4
3
d 4x
2
5 10
6
seconds or 11
31
54
days
6 50 7 12
8 88 classical, 80 blues and 252 heavy metal
9 a 300 cm
3
b h =
V
r
2
;
117
5
cm
c r =
_
V
h
;
_
128

cm =
8

cm
10 a x =
b
a + y
b x =
a +b
c
c x =
2ab
b a
d x =
ab +b
2
d d
2
d(a +b)
11 a
p
2
+q
2
p
2
q
2
b
x + y
x(y x)
c (x 2)(2x 1) d
2
a
12 A, B and C are 36, 12 and 2 years old
respectively.
13 a a = 8, b = 18 b x = p +q, y = 2q
14 x = 3.5
15 a 4n
2
k
2
b
40cx
2
ab
2
16 x = 1
Extended-response questions
1 a
5x
4
hours b
4x
7
hours c
19x
28
hours
d i x =
14
19
0.737
ii distance from Jack =
140
19
km 7 km
distance from Benny =
560
19
km 29 km
2 a 18 000 cm
3
per minute b V = 18 000t
c h =
45t
4
d after
3 a Thomas, a cards; George,
3a
2
cards; Sally,
a 18 cards; Zeb,
a
3
cards; Henry,
5a
6
cards
b
3a
2
+a 18 +
a
3
= a +
5a
6
+6
c a = 24; Thomas 24 cards, Henry 20 cards,
George 36 cards, Sally 6 cards, Zeb 8 cards
4 a 1.9 10
8
N b m
1
=
Fr
2
10
11
6.67m
2
c 9.8 10
24
kg
5 a V = 1.8 10
7
d b 5.4 10
8
m
3
c k = 9.81 10
3
d 1.325 10
15
J
e 1202 days (to the nearest day)
6
10

3
3
7 40

8
240
11
km/h
9 a 20 r
b i V =
_
20r
2

r
3
3
_

ii r = 5.94 and h = 14.06


10 a
2
3
litre from A,
1
3
litre from B
b 600 mL from A; 400 mL from B
c
( p q)(n +m)
2(np qm)
litres from A and
(n m)( p +q)
2(np qm)
litres from B and
n
m
=
q
p
.
Also
n
m
or
q
p
1 and the other 1.
11 a h = 2(10 r) b V = 2r
2
(10 r)
c r = 3.4985 and h = 13 or r = 9.022
and h = 1.955
P1: FXS/ABE P2: FXS
9780521740494ans-1-10.xml CUAU033-EVANS August 22, 2009 11:0
A
n
s
w
e
r
s
664 Essential Advanced General Mathematics
Chapter 3
Exercise 3A
1
a {4} b {1, 3, 5} c {1, 2, 3, 4, 5} or
d e
2
a {1, 2, 4, 5, 7, 8, 10, 11, 13, 14, 16}
b {1, 3, 5, 7, 9, 11, 13, 15}
c {2, 3, 4, 6, 8, 9, 10, 12, 14, 15, 16}
d {1, 5, 7, 11, 13} e {1, 5, 7, 11, 13}
3
A B
1
3
7
5
9
11
8
12
4
6 2
10

a {1, 2, 3, 5, 6, 7, 9, 10, 11}


b {1, 3, 5, 7, 9, 11} c {2, 4, 6, 8, 10, 12}
d {1, 3, 5, 7, 9, 11} e {1, 3, 5, 7, 9, 11}
4
a {10, 11, 13, 14, 15, 17, 18, 19, 21, 22, 23, 25}
b {11, 12, 13, 14, 16, 17, 18, 19, 21, 22, 23, 24}
c {10, 12, 15, 16, 20, 24, 25}
d {11, 13, 14, 17, 18, 19, 21, 22, 23}
e {11, 13, 14, 17, 18, 19, 21, 22, 23}
5
a {R} b {G, R} c {L, E, A, N}
d {A, N, G, E, L} e {R} f {G, R}
6
a { p, q, u, v} b { p, r, w}
c { p} d { p, q, r, u, v, w}
e {q, r, s, t, u, v, w} f { p}
7
a {5, 7, 8, 9, 10, 11} b {1, 3, 5, 7, 9, 11}
c {1, 3, 5, 7, 8, 9, 10, 11} d {5, 7, 9, 11}
e {1, 2, 3, 4, 6, 8, 10, 12} f {5, 7, 9, 11}
8 a
A B

A'
b
A B

B'
c
A B

A' B'
d
A B

A' B'
e
A B

A B
f
A B

(A B)'
9
a {E, H, M, S} b {C, H, I, M}
c {A, T} d {H, M}
e {C, E, H, I, M, S} f {H, M}
Exercise 3B
1 a Yes b Yes c Yes
2 a No b No c No
3 a
3
11
b
3
25
c
2
7
d
4
11
e
2
9
f
9
20
4 a 0.

28571

4 b 0.

5 c 0.35
d 0.

30769

2 e 0.

058823529411764

7
P1: FXS/ABE P2: FXS
9780521740494ans-1-10.xml CUAU033-EVANS August 22, 2009 11:0
A
n
s
w
e
r
s
Answers 665
Exercise 3C
1 a 2

2 b 2

3 c 3

3 d 5

2
e 3

5 f 11

10 g 7

2 h 6

3
i 5 j 5

3 k 16

2
2 a 3

2 b 6

3 c 4

7
d 5

10 e 28

2 f 0
3 a 11

3 +

14 b 5

7
c 0 d

2 +

3
e 5

2 +15

3 f

2 +

5
4 a

5
5
b

7
7
c

2
2
d
2

3
3
e

6
2
f

2
4
g

2 1 h 2 +

3
i
4 +

10
6
j

6 2 k

5 +

3
2
l 3(

6 +

5)
m 3 +2

2
5 a 6 +4

2 b 9 +4

5 c 1 +

2
d 4 2

3 e
2

3
9
f
8 +5

3
11
g
3 +

5
2
h
6 +5

2
14
6 a 4a 4

a +1
b 3 +2x +2
_
(x +1)(x +2)
7 a 5 3

2 b 7 2

6
Exercise 3D
1 a 2
5
3 5 11 13
b 2
5
3 7 11 13
c 2
5
7 11 13
d 2
5
7 11 13 17
2 a 1 b 27 c 5 d 31 e 6
Exercise 3E
1 a
b i 19 ii 9 iii 23
2 a
14
3
5
9
4
2
6
7
C
A B

b i 23 ii 37 iii 9
3 x = 20 4 x = 7
5 a 5 b 10
6 45
7 a x = 5 b 16 c 0
8 a
X
22
35
26
34
23
29
28
31
32
25
1
49
4 16
9
36
30
33
24
27
21
12
3
6
15
39 18
Y
Z

b i X Y Z = {36}
ii n(X Y) = 5
9 31 students; 15 black, 12 green, 20 red
10 n(M F) = 11 11 1
12 x = 6, 16 procient in Italian 13 102
Multiple-choice questions
1 A 2 D 3 D 4 D 5 C
6 D 7 B 8 B 9 C 10 A
Short-answer questions
(technology-free)
1 a
7
90
b
5
11
c
1
200
d
81
200
e
4
15
f
6
35
2 2
3
3
2
7
3 a
2

2
2
b 4

5 +9 c 2

6 +5
4 23 12

3
5 a 2

6 +6 b
a

a
2
b
2
b
6 a 15 b 15
7 a 1 b 22 c 22
8 5 9 2 cm
2
10 15

7
11 2 12

6 13
51

3
5
14 a 57 b 3 c 32
15 2

2 +3
Extended-response questions
1 c i

11 +

3
ii 2

7 or

7 2

2
iii 3

3 2

6 or 2

6 3

3
2 a a = 6 and b = 5 b p = 26, q = 16
c a = 1, b =
2
3
d i
12

3 19
71
ii 3

3
iii
1

3
2
e Q = {a +0

3; a Q}
P1: FXS/ABE P2: FXS
9780521740494ans-1-10.xml CUAU033-EVANS August 22, 2009 11:0
A
n
s
w
e
r
s
666 Essential Advanced General Mathematics
3 d 2
5 a b = 4, c = 1 b 2 +

3
6 a (20, 21, 29)
7 a i 4 factors ii 8 factors
b n +1 factors
c i 32 factors ii (n +1)(m +1)
d (
1
+1)(
2
+1) . . . (
n
+1) e 24
8 a 1080 = 2
3
3
3
5,
25 200 = 2
4
3
2
5
2
7
b 75 600
d i 3476, 3474, 3472, 3470
ii 1738, 1737, 1736, 1735
9 a i region 8
ii male, red hair, blue eyes
iii male, not red hair, blue eyes
b i 5 ii 182
10 a i students shorter than or equal to 180 cm
ii students who are female or taller than
180 cm
iii students who are male and shorter than or
equal to 180 cm
b
A
(A B)' = A' B' is shaded
B
11 a
A B
C
x
x
65
35
205
35
n(A C) = 0
b 160 c 65 d 0
Chapter 4
Exercise 4A
1 a k = 2
x 2 4 6 8
y 8 32 72 128
b k =
1
3
x
1
2
1
3
2
2
y
1
6
1
3
1
2
2
3
c k = 3
x 4 9 49 900
y 6 9 21 90
d k =
2
5
x
1
32
1 32 1024
y
1
5
2
5
4
5
8
5
2 a V = 262.144 b r 2.924
3 a a 1.058 b b 5.196
4 a 72 cm
2
b 20 cm
5 a
648
113
cm
b 1412.5 g
6 10.125 kg 7 62.035 cm 8 1.898 s
9 a 8.616 km b 14.221 km
10 a i 300% increase ii 41% increase
iii 700% increase
b i 75% decrease ii 29% decrease
iii 87.5% decrease
c i 36% decrease ii 11% decrease
iii 48.8% decrease
d i 96% increase ii 18% increase
iii 174.4% increase
Exercise 4B
1 a k = 2
x 2 4 6 32
y 1
1
2
1
3
1
16
b k =
1
2
x
1
4
1 4 9
y 1
1
2
1
4
1
6
c k = 3
x 1 2 3 6
y 3
3
4
1
3
1
12
d k =
1
3
x
1
8
1 27 125
y
2
3
1
3
1
9
1
15
P1: FXS/ABE P2: FXS
9780521740494ans-1-10.xml CUAU033-EVANS August 22, 2009 11:0
A
n
s
w
e
r
s
Answers 667
2 a a =
1
2
b b 5.657
3 a a = 0.3125 b b =

2
4 2.85 kg/cm
2
5 a 2.4 amperes b 25%
6 64 candela 7 5.15 cm
8 a i 75% decrease ii 29% decrease
iii 87.5% decrease
b i 300% increase ii 41% increase
iii 700% increase
c i 56.25% increase ii 12% increase
iii 95% increase
d i 49% decrease ii 15% decrease
iii 64% decrease
Exercise 4C
1 a direct b direct square c inverse
d direct square root e inverse square
2 b, e 3 a, b, e
4 a y = 3x b y =
3
x
c y =
10
3
x
2
d y = 2

x e y =
1
3

x
f y = 6x
3
5
y
0
4 8 12 16
10
y = 2.4x
2
20
30
40
x
2
6
y
0
1 2 3 4 5
2
4
6
8
x
y = 1.5x
7
y
0
5 10 15 20 25
10
20
30
40
50
1
x
2
2
x
2
y =
8 a y =
1
4

x b y = 2x
5
4
c y = 3.5x
0.4
d y = 10x
2
3
e y = 2x

5
2
f y = 3.2x
0.4
9 a a = 100, b = 0.2 b 158.49
10 a a = 1500, b = 0.5 b 474.34
Exercise 4D
1 k = 5
x 2 4 6 10
z 10 2 60 12.5
y 1 10 0.5 4
2 k =
1
2
x 2 4 1 10
z 10 8 50 3
y 10 16 25 15
3 k = 3
x 2 3 5 10
z 10 4 50
400
3
y
15
2
4
3
6 4
4 a 1.449 5 z 0.397 6 $174
7 360 joules
8 a 500% increase b 78% decrease
9 a 41% increase b 33% increase
10 a 183% increase b 466% increase
11 a The tensions are the same.
b The work done by the second spring is 90%
that of the rst.
Exercise 4E
1 $33.40
2 a overhead charge = $250,
cost per guest = $47.50
b $5000
3 p = 20.5 4 $55.11
5 a 330 m b 67.5 m
6 45 minutes
Multiple-choice questions
1 C 2 A 3 B 4 C 5 B
6 D 7 E 8 D 9 D 10 B
Short-answer questions
(technology-free)
1 a When b = 4, a = 6;
when a = 8, b =
8

3
b When x = 27, y =
30
2
1
3
;
when y =
1
8
, x =
1
256 000
c When x =
1
2
, y =
16
3
; when y =
4
27
,
x = 3
d
1
6
2 a d = 4.91t
2
b 491 m
c 2 s, correct to one decimal place.
3 a 14 m/s b 40 m c v and

s
4 2.4 hours
5 a y is halved b x is halved
c y is doubled d x is doubled
P1: FXS/ABE P2: FXS
9780521740494ans-1-10.xml CUAU033-EVANS August 22, 2009 11:0
A
n
s
w
e
r
s
668 Essential Advanced General Mathematics
6 4.05 cents 7 $35 8 18 amps
9 I
2
=
1
4
I
1
10 34% increase
Extended-response questions
1 a 0.24 kg b 11 cm
2 a h = 0.0003 375 n
2
b 17.1 m
c 218 revs/min
3 13 knots
4
a v =
121.8
P
b 9.6 kg/cm
2
5 a w =
3000
d
b 600 kg c 333 kg
6 a v =
144
p
b i v = 2 ii p = 48
c
y
0
8
9
12
1
18
1
16
1
12
1
p
7 44.8 min 8 $76 9 S
n
=
1
2
n(n +1)
10 a P = 3498.544 N
0.5
b 25 956 c 51 023
11 a t =
3600
d
2
b T = 0.14d
2
c 23.9 mL d 6.3 min e 9 min; 56 min
12 a i T = 0.000 539 R
1.501
ii Mars 1.87; Jupiter 11.86;
Saturn 29.45; Uranus 84.09
Neptune 165.05; Pluto 248.16
b 2.540 10
9
km
13 a a = 11.7, b = 0.41 b 77
c k = 163, p = 1.167 d 7
Chapter 5
Exercise 5A
1 a 3, 7, 11, 15, 19 b 5, 19, 61, 187, 565
c 1, 5, 25, 125, 625 d 1, 1, 3, 5, 7
e 1, 3, 7, 17, 41
2 a t
1
= 1, t
2
=
1
2
, t
3
=
1
3
, t
4
=
1
4
b t
1
= 2, t
2
= 5, t
3
= 10, t
4
= 17
c t
1
= 2, t
2
= 4, t
3
= 6, t
4
= 8
d t
1
= 2, t
2
= 4, t
3
= 8, t
4
= 16
e t
1
= 5, t
2
= 8, t
3
= 11, t
4
= 14
f t
1
= 1, t
2
= 8, t
3
= 27, t
4
= 64
g t
1
= 3, t
2
= 5, t
3
= 7, t
4
= 9
h t
1
= 2, t
2
= 6, t
3
= 18, t
4
= 54
3 a i t
n
= 3n ii t
n
= t
n1
+3, t
1
= 3
b i t
n
= 2
n1
ii t
n
= 2t
n1
, t
1
= 1
c i t
n
=
1
n
2
ii t
n
=
(n 1)
2
n
2
t
n1
, t
1
= 1
d i t
n
= 3(2)
n1
ii t
n
= 2t
n1
, t
1
= 3
e i t
n
= 3n +1
ii t
n
= t
n1
+3, t
1
= 4
f i t
n
= 5n 1
ii t
n
= t
n1
+5, t
1
= 4
4 t
n+1
= 3n +4, t
2n
= 6n +1
5 a t
1
= 15, t
n
= t
n1
+3
b t
n
= 12 +3n c t
13
= 51
6 a t
1
= 94.3, t
n
= 0.96t
n1
b t
n
= 94.3(0.96)
n1
c t
9
= 68.03
7 a t
n
= 1.8t
n1
+20, t
0
= 100
b t
1
= 200, t
2
= 380, t
3
= 704,
t
4
= 1287, t
5
= 2336
8 a $2120 at end of 1st year, $2671.20 at end of
2nd year, $3255.47 at end of 3rd year
b t
n
= 1.06(t
n1
+400), t
1
= 2120
c $8454.02
9 a 1, 4, 7, 10, 13, 16
b 3, 1, 1, 3, 5, 7
c
1
2
, 1, 2, 4, 8, 16
d 32, 16, 8, 4, 2, 1
e 1.1, 1.21, 1.4641, 2.144, 4.595, 21.114
f 27, 18, 12, 8,
16
3
,
32
9
g 1, 3, 11, 27, 59, 123
h 3, 7, 3, 7, 3, 7
10 a t
1
= 1, t
2
= 2, t
3
= 4
b u
1
= 1, u
2
= 2, u
3
= 4
c t
1
= u
1
, t
2
= u
2
, t
3
= u
3
d t
4
= 8, u
4
= 7
11 S
1
= a +b, S
2
= 4a +2b,
S
3
= 9a +3b, S
n+1
S
n
= 2an +a +b
12 t
2
=
3
2
, t
3
=
17
12
, t
4
=
577
408
. The number is

2.
13 t
3
= 2, t
4
= 3, t
5
= 5
Exercise 5B
1 a t
1
= 0, t
2
= 2, t
3
= 4, t
4
= 6
b t
1
= 3, t
2
= 2, t
3
= 7, t
4
= 12
c t
1
=

5, t
2
= 2

5, t
3
= 3

5,
t
4
= 4

5
d t
1
= 11, t
2
= 9, t
3
= 7, t
4
= 5
2 a a = 3, d = 4, t
n
= 4n 1
b a = 3, d = 4, t
n
= 7 4n
c a =
1
2
, d = 2, t
n
= 2n
5
2
d a = 5

5, d =

5,
t
n
=

5n +5 2

5
3 a 31 b 24 c 5 d 6

3
P1: FXS/ABE P2: FXS
9780521740494ans-1-10.xml CUAU033-EVANS August 22, 2009 11:0
A
n
s
w
e
r
s
Answers 669
4 a 14 b 22 c 4th day
5 a 70 b 94 c Row F
6 a 22 b 28 c 20 d 56
7
7
2
8 117 9 t
n
= 156n 450
10 54 11 2
12
62
9
,
88
9
,
38
3
,
140
9
,
166
9
,
64
3
,
218
9
,
244
9
13 7, 9, 11, 13 14 t
n
= a
a(n 1)
m 1
15 27

3 60
16 a 11.5 b
2

2
7
17 16 18 5 20 3
Exercise 5C
1 a 426 b 55 c 60

2 d 108
2 112 3 11 4 680 5 2450
6 a 16.5 km b 45 km c 189 km
7 a 10 days b 25 per day
8 {n : n = 9} 9 20
10 a 86 b 2600 c 224
d 2376 e 5 extra rows
11 $176 400
12 a = 15, d = 3, t
6
= 0, S
6
= 45
13 2160 14 0 15 266
16 a t
n
=
5
4
n +
11
4
b t
n
=
46

5
5
2

5n
17 a b b
n
2
(b +bn)
18 t
5
= 10, S
25
= 1250
19 1575d
20 a S
n1
= 23n 3n
2
20
b t
n
= 20 6n
c a = 14, d = 6
21 7, 12, 17
Exercise 5D
1 a 3, 6, 12, 24 b 3, 6, 12, 24
c 10 000, 1000, 100, 10
d 3, 9, 27, 81
2 a
5
567
b
1
256
c 32 d a
x+5
3 a t
n
= 3
_
2
3
_
n1
b t
n
= 2(2)
n1
c t
n
= 2(

5)
n1
4
2
5
5 t
9
6 a 6 b 9 c 9 d 6 e 8
7 a $5397.31 b 48th year
8 a 21 870 m
2
b 10th day
9 47.46 cm
10 a 57.4 km b 14th day
11 $5 369 000
12 a 24 b 12 288
13
2
3
5
14 16

2
15 t
10
= 2048 16 t
6
= 729
17 a $7092.60 b 12 years
18 $3005.61 19 11.6% p.a. 20 5 weeks
21 a 60 b 2.5 c 1 d x
4
y
7
22 3
Exercise 5E
1 a 5115 b 182 c
57
64
2 a 1094 b 684 c 7812
3 a 1062.9 mL b 5692.27 mL
4 a $18 232.59 b $82 884.47
5 a 49 minutes (to nearest minute)
b 164 minutes c Friday
6
481 835
6561
= 73
2882
6561
7 Biancas is worth $3247.32, Andrews is worth
$3000
8 a 155 b
15

2
2
+15
9 a 8 b {n : n > 19}
10
x
2m+2
+1
x
2
+1
Exercise 5F
1 a
5
4
b
3
5
2
Perimeter of nth triangle =
_
1
2
_
n1
p,
Area of nth triangle =
p
2

3
4
n
,
Sum to innity of perimeters = 2p,
Sum to innity of areas =
p
2

3
3
3 3333
1
3
4 Yes
5 yes, as the number of hours approaches innity,
but problem becomes unrealistic after 4 to 5
hours
6 S

= 8 7
1
2
8 12 m 9 75 m
10 a
4
9
b
1
30
c
31
3
d
7
198
e 1 f
37
9
11 r =
1
2
, t
1
= 16, t
2
= 8 or
r =
1
2
, t
1
= 48, t
2
= 24
12
5
8
13
2
3
P1: FXS/ABE P2: FXS
9780521740494ans-1-10.xml CUAU033-EVANS August 22, 2009 11:0
A
n
s
w
e
r
s
670 Essential Advanced General Mathematics
Exercise 5G
1 a i 3, 2.75, 2.688, 2.672, 2.668, 2.667
ii 1, 2, 1, 2, 1, 2
iii 2, 13, 508, 774193, 1.8 10
12
,
9.7 10
24
iv 3, 3.236, 3.288, 3.300, 3.302, 3.303
b i and iv converge
2 a 0.6736 b 0.6823 c 0.7913
d 1.3532 e 0.3099 f 2.3758
g 0.5222 h 0.3473
Multiple-choice questions
1 D 2 B 3 A 4 A 5 B
6 D 7 E 8 C 9 E 10 D
Short-answer questions
(technology-free)
1 a 3, 1, 5, 9, 13, 17
b 5, 12, 26, 54, 110, 222
2 a 2, 4, 6, 8, 10, 12
b 1, 4, 7, 10, 13, 16
3 a $5250, $6037.50
b t
1
= 5250, t
n
= 1.05(t
n1
+500)
4 147 5 0.1 6 258.75
7 {n : n = 12} 8 1
9 1000 1.035
n
10 t
2
= 6, t
4
=
8
3
or t
2
= 6, t
4
=
8
3
11 96 12 9840 13
3
4
14 x = 8 or x = 2
Extended-response questions
1 a 0.8, 1.5, 2.2, . . . b Yes c 8.5 m
2 a Yes b t
n
= 25n +25 c 650
3 The fth pole is 22
1
7
km from town A and
9
6
7
km from town B.
4 a 20, 36, 52, 68, 84, 100, 116, 132, . . .
b T
n
= 16n +4 c Yes, size 12
5 a D
n
= 7n 5 b 27
6 472 mm 7 520
8 a 99.9999 mg b 100 mg
9 a
1
729
m
b 1.499 m. No, the maximum height the water
will reach is 1.5 m.
10 a 27.49 b 1680.8
11 a 7
1
9
m b 405 m
12 2
64
1 = 1.845 10
19
13 a i t
n
= 3750 +250n
ii S
n
= 3875n +125n
2
iii n = 22
iv m =
T 4000
250
+1
v p = 51
b i S
n
= 37 500(1.08
n
1)
ii Q
B
Q
A
= 37500(1.08
n
1)
3875n 125n
2
; n = 18
14 2.828 43
Chapter 6
Exercise 6A
1 a = 10, b = 0, c = 7
2 a = 1, b = 2
3 a = 2, b = 1, c = 7
4 a = 2, b = 1, c = 3
5 (x +2)
2
4(x +2) +4
6 (x +1)
3
3(x +1)
2
+3(x +1) 1
7 a = 1, b = 2, c = 1
8 a It is impossible to nd a, b and c to satisfy
a = 3, 3ab = 9, 3ab
2
= 8 and
ab
3
+c = 2
b a = 3, b = 1, c = 5
9 a = 1, b = 6, c = 7, d = 1
10 a a =
5
3
b and a = 3b are not possible
unless a and b are both zero, but a +b = 1 so
no constants can be found.
b (n +1)(n +2) 3(n +1) +1
11 a ax
2
+2abx +ab
2
+c
b a
_
x +
b
2a
_
2
+c
b
2
4a
13 a = 3, b =
1
3
, c = 3 or a =
1
3
,
b = 3, c = 3
14 a = 3, b = 3, c = 1
15 If c = 5, a = 1, b = 5; if c = 27,
a = 3, b = 3
Exercise 6B
1 a x = 1 b x = 3
c x = 1

30
5
d x = 1

2
2
e x = 1
3

2
2
f x =
13

145
12
2 a x =
1

32t +1
4
, t
1
32
b x =
1

t +3
2
, t 3
c x =
2

5t 46
5
, t
46
5
d x = 2
_
5t (t 2)
t
, t < 0 or t 2
P1: FXS/ABE P2: FXS
9780521740494ans-1-10.xml CUAU033-EVANS August 22, 2009 11:0
A
n
s
w
e
r
s
Answers 671
3 a x =
3(1

5)
2
b i x =
p
_
p
2
+64
2
ii When p = 0, x = 4 and when p = 6, x = 2
4 x = 2
5 a
18
x(x +3)
b x = 6, 3
6 17 and 19
7 a Average speed of car =
600
x
km/h,
average speed of plane
=
_
600
x
+220
_
km/h
b Average speed of car = 80 km/h,
average speed of plane = 300 km/h
8 x = 20 9 6 km/h
10 a x = 50 b 72 minutes
11 Average speed of slow train = 30 km/h,
average speed of express train = 50 km/h
12 60 km/h
13 Smaller pipe will take 25 minutes and the larger
pipe, 20 minutes
14 Each pipe running alone will ll the tank in 14
minutes
15 Average speed by rail = 43 km/h, average
speed by sea = 18 km/h
16 22 km 17 10 litres
Exercise 6C
1 a
2
x 1
+
3
x +2
b
1
x +1

2
2x +1
c
2
x +2
+
1
x 2
d
1
x +3
+
3
x 2
e
3
5(x 4)

8
5(x +1)
2 a
2
x 3
+
9
(x 3)
2
b
4
1 +2x
+
2
1 x
+
3
(1 x)
2
c
4
9(x +1)
+
4
9(x 2)
+
2
3(x 2)
2
3 a
2
x +1
+
2x +3
x
2
+ x +1
b
x +1
x
2
+2
+
2
x +1
c
x 2
x
2
+1

1
2(x +3)
4 3 +
3
x 1
+
2
x 2
5 It is impossible to nd A and C to satisfy
A = 0, C 2A = 0 and A +C = 10.
6 a
1
2(x 1)

1
2(x +1)
b
2
5(x 2)
+
3
5(x +3)
c
1
x 2
+
2
x +5
d
2
5(2x 1)

1
5(x +2)
e
3
3x 2

1
2x +1
f
2
x 1

2
x
g
1
x
+
3 x
x
2
+1
h
2
x
+
x
x
2
+4
i
1
4(x 4)

1
4x
j
7
4(x 4)

3
4x
k x +
1
x

1
x 1
l x 1
3
x

1
2 x
m
2
3(x +1)
+
x 4
3(x
2
+2)
n
2
3(x 2)
+
1
3(x +1)

1
(x +1)
2
o
2
x
+
1
x
2
+4
p
8
2x +3

5
x +2
q
26
9(x +2)
+
1
9(x 1)

1
3(x 1)
2
r
16
9(2x +1)

8
9(x 1)
+
4
3(x 1)
2
s x 2 +
1
4(x +2)
+
3
4(x 2)
t x
1
x +1
+
2
x 1
u
3
x +1

7
3x +2
Exercise 6D
1 a (1, 1) (0, 0)
b (0, 0)
_
1
2
,
1
2
_
c
_
3 +

13
2
, 4 +

13
_
_
3

13
2
, 4

13
_
2 a (13, 3) (3, 13) b (10, 5) (5, 10)
c (8, 11)(11, 8) d (9, 4) (4, 9)
e (9, 5)(5, 9)
3 a (11, 17) (17, 11) b (37, 14) (14, 37)
c (14, 9)(9, 14)
4 (0, 0) (2, 4)
5
_
5 +

5
2
,
5 +

5
2
__
5

5
2
,
5

5
2
_
6
_
15
2
,
5
2
_
(3, 1)
7
_
130 +80

2
41
,
60 +64

2
41
_
and
_
130 80

2
41
,
60 64

2
41
_
P1: FXS/ABE P2: FXS
9780521740494ans-1-10.xml CUAU033-EVANS August 22, 2009 11:0
A
n
s
w
e
r
s
672 Essential Advanced General Mathematics
8
_
1 +

21
2
,
1

21
2
_
and
_
1

21
2
,
1 +

21
2
_
9
_
4
9
, 2
_
10
_
6

5
5
,
3

5
5
_
11
_
2,
1
2
_
12 (0, 1), (3, 2)
Multiple-choice questions
1 C 2 D 3 D 4 C 5 E
6 E 7 C 8 D 9 B 10 B
Short-answer questions
(technology-free)
1 a = 3, b = 2, c = 1
2 (x 1)
3
+3(x 1)
2
+3(x 1) +1
5 a x = 4 or 3 b x = 1 or 2
c x = 2 or 5 d x =
2

2
2
e x =
1

3t 14
3
f
t

t
2
16t
2t
6 x =
3

73
2
7 a
1
x 3

2
x +2
b
3
x +2
+
4
x 2
c
1
2(x 3)

3
2(x +5)
d
1
x 5
+
2
x +1
e
13
x +2

13
x +3

10
(x +2)
2
f
4
x +4
+
2
x 1

3
(x 1)
2
g
1
x +1

6
x
2
+2
h
1
x 1

x +3
x
2
+ x +1
i
1
3 x

3
x +4
j
2
7(x 3)

16
7(x +4)
8 a
1
x 3

x 10
x
2
+ x +2
b
1
4(x +1)

x 2
4(x
2
x +2)
c 3x +15 +
64
x 4

1
x 1
9 a (0, 0), (1, 1) b (0, 4), (4, 0)
c (1, 4), (4, 1)
10 (4, 1), (2, 1)
Extended-response questions
1 a 24 km/h
b Speed =
a +
_
a(a +480)
2
, a > 0. When
a = 60, speed = 120 km/h, a very fast
constant speed for a train. If we choose this as
an uppermost value for the speed,
0 < a < 60 and 0 < speed < 120
c
a 1 8 14 22 34 43 56 77 118
speed 16 20 24 30 40 48 60 80 120
2 a
b +

b
2
+4a
2
m
b a b x a b x a b x a b x a b x a b x
2 1 2 43 42 43 84 83 84 52 24 26 57 16 19 45 4 9
3 2 3 44 43 44 85 84 85 54 25 27 60 17 20 50 5 10
4 3 4 45 44 45 86 85 86 56 26 28 63 18 21 55 6 11
5 4 5 46 45 46 87 86 87 58 27 29 66 19 22 60 7 12
6 5 6 47 46 47 88 87 88 60 28 30 69 20 23 65 8 13
7 6 7 48 47 48 89 88 89 62 29 31 72 21 24 70 9 14
8 7 8 49 48 49 90 89 90 64 30 32 75 22 25 75 10 15
9 8 9 50 49 50 91 90 91 66 31 33 78 23 26 80 11 16
10 9 10 51 50 51 92 91 92 68 32 34 81 24 27 85 12 17
11 10 11 52 51 52 93 92 93 70 33 35 84 25 28 90 13 18
12 11 12 53 52 53 94 93 94 72 34 36 87 26 29 95 14 19
13 12 13 54 53 54 95 94 95 74 35 37 90 27 30 100 15 20
14 13 14 55 54 55 96 95 96 76 36 38 93 28 31
15 14 15 56 55 56 97 96 97 78 37 39 96 29 32 42 1 7
16 15 16 57 56 57 98 97 98 80 38 40 99 30 33 48 2 8
17 16 17 58 57 58 99 98 99 82 39 41 54 3 9
18 17 18 59 58 59 100 99 100 84 40 42 20 1 5 60 4 10
P1: FXS/ABE P2: FXS
9780521740494ans-1-10.xml CUAU033-EVANS August 22, 2009 11:0
A
n
s
w
e
r
s
Answers 673
a b x a b x a b x a b x a b x a b x
19 18 19 60 59 60 86 41 43 24 2 6 66 5 11
20 19 20 61 60 61 6 1 3 88 42 44 28 3 7 72 6 12
21 20 21 62 61 62 8 2 4 90 43 45 32 4 8 78 7 13
22 21 22 63 62 63 10 3 5 92 44 46 36 5 9 84 8 14
23 22 23 64 63 64 12 4 6 94 45 47 40 6 10 90 9 15
24 23 24 65 64 65 14 5 7 96 46 48 44 7 11 96 10 16
25 24 25 66 65 66 16 6 8 98 47 49 48 8 12
26 25 26 67 66 67 18 7 9 100 48 50 52 9 13 56 1 8
27 26 27 68 67 68 20 8 10 56 10 14 63 2 9
28 27 28 69 68 69 22 9 11 12 1 4 60 11 15 70 3 10
29 28 29 70 69 70 24 10 12 15 2 5 64 12 16 77 4 11
30 29 30 71 70 71 26 11 13 18 3 6 68 13 17 84 5 12
31 30 31 72 71 72 28 12 14 21 4 7 72 14 18 91 6 13
32 31 32 73 72 73 30 13 15 24 5 8 76 15 19 98 7 14
33 32 33 74 73 74 32 14 16 27 6 9 80 16 20
34 33 34 75 74 75 34 15 17 30 7 10 84 17 21 72 1 9
35 34 35 76 75 76 36 16 18 33 8 11 88 18 22 80 2 10
36 35 36 77 76 77 38 17 19 36 9 12 92 19 23 88 3 11
37 36 37 78 77 78 40 18 20 39 10 13 96 20 24 96 4 12
38 37 38 79 78 79 42 19 21 42 11 14 100 21 25
39 38 39 80 79 80 44 20 22 45 12 15 90 1 10
40 39 40 81 80 81 46 21 23 48 13 16 30 1 6 99 2 11
41 40 41 82 81 82 48 22 24 51 14 17 35 2 7
42 41 42 83 82 83 50 23 25 54 15 18 40 3 8
3 a
a +

a
2
+4abc
2ac
b a = 3, b = 1, c =
4
3
etc.
4 a Smaller pipe will take (b +

b
2
ab) minutes
to ll the tank,
larger pipe will take (b a +

b
2
ab)
minutes to ll the tank
b Smaller pipe will take 48 minutes to ll the
tank,
larger pipe will take 24 minutes to ll the
tank
c
a 3 8 15 24 35
b 4 9 16 25 36
Chapter 7
7.1 Multiple-choice questions
1 A 2 B 3 E 4 A 5 B
6 C 7 A 8 B 9 D 10 E
11 B 12 C 13 D 14 B 15 A
16 C 17 B 18 D 19 C 20 D
21 A 22 C 23 C 24 A 25 B
26 D 27 B 28 B 29 B 30 A
31 D 32 A 33 B 34 D 35 A
36 B 37 A 38 A 39 C 40 C
41 A 42 E 43 A 44 C 45 C
46 E 47 D 48 C 49 A
7.2 Extended-response
questions
1 a 8 cm b 7.7 cm c 6 cm d 15 cm
2 a i 178 ii 179
iii 179.5 iv 179.95
b i 180 ii circle
c 20 d n =
360
180 A
e square
3
a Volume of hemisphere =
2
3
t
3
,
Volume of cylinder = t
2
s
Volume of cone =
1
3
t
2
w
b i 6 : 2 : 3 ii 54 cubic units
4 a a = 0.4, b = 148
b C ($)
148
(300, 28)
0 n
c $68 d 248
5 a i OC
1
= R r
1
ii r
1
=
R
3
P1: FXS/ABE P2: FXS
9780521740494ans-1-10.xml CUAU033-EVANS August 22, 2009 11:0
A
n
s
w
e
r
s
674 Essential Advanced General Mathematics
b i OC
2
=
R
3
r
2
ii r
2
=
R
9
c i r =
1
3
ii r
n
=
R
3
n
iii S

=
R
2
iv S

=
R
2
8
6 a a = 6000, b = 15 000
b $57 000 c 2006
7 a i 80n +920
ii A: 2840 tonnes, B: 2465 tonnes
iii 40n(n +24)
iv A: 46 080 tonnes, B: 39 083 tonnes
b April 2006
8 a 4 b 6 c 8 d 2
e i 10 ii P
n
= P
n1
+2
iii P
n
= 2n +2
iv
1
1
1
1
2
1
2
1
2
1
4
1
8
1
8
1
8
1
8
1
8
1
8
1
8
1
8
1
8
1
8
1
8
1
8
1
8
1
8
1
8
1
8
1
8
1
4
1
4
1
4
1
4
1
4
1
4
1
4
1
4
1
2
1
2
9 a 8x cm b 28 8x cm c 7 2x cm
e
x
49
1 2 3 4 5
0
(2, 21)
(5, 84)
A
f A = 21 when x = 2
10 a C = 3500 +0.5x b I = 1.5x
c
x
3500
I
C
0
(3500, 5250)
I/C ($)
d 3500 e 5500
f
x
0
3500
3500
P ($)
P represents prot
11 c 11, 24 and 39
12 b i x =
1
24
ii x =
25
24
13 a r =
x
x +1
; x = 1
b i S

=
4
3
ii S

= 18 iii S

=
9
20
c x >
1
2
and x = 0
14 a i A =
a
3
6
ii 4.5 iii 30
b i A
1
=
a
3
12
ii 486 iii 36
c i A
2
=
1
a

1
b
ii
5
6
iii
11
3
iv 99.99 v 999.999
15 a 14 m b t
n
= 1.5n 1
c 53 d 330 m
16 a i P = 49h
ii
0
h (m)
P (joules)
(1, 49)
iii 1136.8
b i P = 9.8 mh ii 100% increase
iii 50% decrease
c i 14 ii 42
d 4
17 a i a = 50 000, d = 5000
ii The 11th month
iii 4 950 000 litres
b i q
n
= 12 000 (1.1)
n1
ii 256 611 litres
c The 31st month
18 a i 15.4 million tonnes
ii 21.7 million tonnes
b t
n
= 0.9n +9.1 c 371 million tonnes
d 12.1 years e P
n
= 12.5 (1.05)
n1
f 15 years
19 a 1 hr 35 mins b 2.5 km
20 a n(B

T) = n(C T),
n(B C

) = 3n(B

C T

),
n(B C

T) = 4
b
B 21 4 5 8 7 C
T
13
18
n() = 76
c i 5 ii 0
21 a i
_
a
2
+bc ab +bd
ac +cd bc +d
2
_
ii
_
3a 3b
3c 3d
_
Chapter 8
Exercise 8A
1 a (7, 3) b (6, 9) c (2, 7)
d (1, 5) e (1, 7)
2 a
_
4
1
_
b
_
1
1
_
c
_
4
2
_
d
_
7
6
_
e
_
4
3
_
3 a (5, 6) b (2, 2) c (2, 3)
4 a
_
0
5
_
b
_
6
1
_
c
_
5
6
_
d
_
0
0
_
P1: FXS/ABE P2: FXS
9780521740494ans-1-10.xml CUAU033-EVANS August 22, 2009 11:0
A
n
s
w
e
r
s
Answers 675
5
x
y
6
6
5
5
4
4
3
3
2
2
1
1
0
1
1
2
2
3
3
4
4
5
5
6
6
7
Z
C
A B
R
P Q
X
Y
a
_
1
4
_
b
_
1
4
_
c
_
4
3
_
d
_
0
0
_
6 a (4, 12) b (3, 15) c Use
_
2
9
_
7 a (4, 6) b (11, 4)
8 a (1, 0), (2, 1), (3, 4), (4, 9)
b
x
y
(0, 0) (1, 0)
(1, 1)
(2, 1)
(3, 4)
(4, 9) (3, 9)
(2, 4)
c y = (x 1)
2
Exercise 8B
1
x
y
1
0
1
1
2
2
3
3
4
4
5
5
2 3 4 5
C'
B' A' A B
C
2
x
y
1
1
0
1
2
2
2
3 4 5
P'
P Q
Q'
RR'
3
x
y
5
5
4
4
3
3
2
2
1
1
0
Z
Y X
X'
Y' Z'
W'
W
4
x
y
B'
C'
D'
D
A
B
C
A'
1
1
0
1
1
2
2
3
3
4
4
5
5
6
6
7
7
8
8
5 a, b, c
x
y
6
6
5
5
4
4
3
3
2
2
1
0
1
1
2
2
3
3
4
4
5
5
6
6
7
7
8 1
A
2
C
2
C
3
A
3
B
3
C
1
A
1
B
1
B
2
y = x
A
B
C
6 a (2, 6) b (6, 2) c (6, 2)
7 a (1, 0) b (0, 1)
c (0, 1) d (1, 0)
Exercise 8C
1 a (1, 9) b (2, 3) c (4, 3)
2 (x, y) (x, 4y) 3 (x, y) (3x, y)
4 a A(0, 0), B

(0, 3), C

(1, 3), D(1, 0)


b A(0, 0), B(0, 1), C

(3, 1), D

(3, 0)
5 a i A (0, 0) , B (3, 0) , C

(3, 2)
ii A(0, 0), B

_
3
2
, 0
_
, C

_
3
2
, 4
_
b
x
y
B
C
B"
C"
C'
A
0
1
2
3
4
1 2 3
Exercise 8D
1 a i (x, y) (x +5, y +6)
ii no invariant points
b i (x, y) (x, 4y)
ii {(x, 0) : x R}
c i (x, y)
_
1
3
x, y
_
ii {(0, y) : y R}
P1: FXS/ABE P2: FXS
9780521740494ans-1-10.xml CUAU033-EVANS August 22, 2009 11:0
A
n
s
w
e
r
s
676 Essential Advanced General Mathematics
d i (x, y) (x 2, y +3)
ii no invariant points
e i (x, y) (y, x) ii {(x, x) : x R}
f i (x, y) (x, y) ii {(0, y) : y R}
2 a (1, 7) b
_
9,
11
2
_
c
_
3
2
, 1
_
3 An invariant point does not exist.
4 a (5, 3) b
_
2,
11
2
_
c
_
1,
1
3
_
5 a (1, 6) b (0, 0) c (0, 0)
Exercise 8E
1 a i (x, y) (x 1, y +1)
ii (x, y) (x 1, y +1)
b i (x, y) (2x, 2y)
ii (x, y) (2x, 2y)
c i (x, y) (x +4, 3 (y +5))
ii (x, y) (x +4, 3y +5)
d i (x, y) ((x 1), y +2)
ii (x, y) (x 1, y +2)
e i (x, y) (x, y)
ii (x, y) (x, y)
f i (x, y) (4 x, y)
ii (x, y) (4 x, y)
g i (x, y)
_
1
2
x 1, y +2
_
ii (x, y)
_
1
2
(x 1) , y +2
_
h i (x, y) (2x +2, y 3)
ii (x, y) (2(x +2), y 3)
Exercise 8F
1 a y = x +3
x
y = x
y = x + 3
y
3
3
0
b y = (x 1)
2
+4
x
y = (x 1)
2
+ 4
y = x
2
y
4
5
1
c y =
1
x 1
+4
x
y
1
x 1
y = + 4
1
x
y =
4
3
0
1
d y =
_
1
x 1
_
2
+4
x
y
5
1
x
2
y =
y = + 4
1
x 1
2
0 1
4
e (x 1)
2
+(y 4)
2
= 1
y
(x 1)
2
+ ( y 4)
2
= 1
3
4
5
x
x
2
+ y
2
= 1
1
1
0
1
1
2
2 a y = 2x
x
y
y = 2x
y = x
0
b y = 2x
2
x
y
y = 2x
2
y = x
2
0
1 1
c y =
2
x
x
y
0 1
2
1
1
2
x
y =
1
x
y =
2
1
d y =
2
x
2
x
y
1
x
2
y =
2
x
2
y =
1 1
2
1
0
P1: FXS/ABE P2: FXS
9780521740494ans-1-10.xml CUAU033-EVANS August 22, 2009 11:0
A
n
s
w
e
r
s
Answers 677
e x
2
+
y
2
4
= 1
x
y
y
2
4
x
2
+ = 1
x
2
+ y
2
= 1
1 0 1
1
2
2
1
3 a y = 5 x b y = (x 3)
2
+2
c y =
1
3 x
+2 d y =
1
(x 3)
2
+2
e (x 3)
2
+(y 2)
2
= 1
4 a y =
1
2
x +
5
2
b y =
1
4
(x +3)
2
+1
c y =
2
x +3
+1 d y =
4
(x +3)
2
+1
e
1
4
(x +3)
2
+(y 1)
2
= 1
5 y =
1
2
(x +13)
6 a {(x, y) : y = x +4}
x
y
y = x + 4
y = x + 2
0
2
4
2 4
b {(x, y) : y = (x +2)}
x
y
y = x + 2
y = (x + 2)
2
2
0
2
c {(x, y) : y = 4 (x +2)}
x
y
y = 4x + 8
y = x + 2
0
2
8
2
d {(x, y) : y = x 2}
y = x + 2
y = x 2
0
2
2
2
2
x
y
e {(x, y) : y = x +2}
y = x + 2
y = x + 2
0
2
2
2
2
x
y
7 {(x, y) : (x 1)
2
+(y 4)
2
= 4}
1
0
2
4
2
2
2
(x 1)
2
+ ( y 4)
2
= 4
x
2
+ y
2
= 4
x
y
8 a {(x, y) : y = 2x
2
}
b {(x, y) : y = (2x 5)
2
+2}
c
_
(x, y) : y =
1
2
(x 5)
2
+2
_
d {(x, y) : y = (x +2)
2
+1}
e {(x, y) : y = 2

x}
9 a
x
y
0
y = 2x
2
b
x
y
0
2
2.5
y = (2x 5)
2
+ 2
c
x
y
1
2
y = (x 5)
2
+ 2
0
2
5
d
x
y
0
2
1
5
y = (x + 2)
2
+ 1
e
x
y
0
2
4
4
x y = 2 +

P1: FXS/ABE P2: FXS


9780521740494ans-1-10.xml CUAU033-EVANS August 22, 2009 11:0
A
n
s
w
e
r
s
678 Essential Advanced General Mathematics
10 a {(x, y); y = 2
x1
}
b {(x, y) : y = 2
x
+1}
c
_
(x, y) : y =
1
3
_
2
x
2
_
_
d
_
(x, y) : y =
1
3
_
2
x
2
_
_
e
_
(x, y) : y =
1
2
x+2
+4
_
f {(x, y) : y = (2
2x
+2)}
11 a
x
y
0
y = 2
x1
1
2
b
x
y
0
2
2
x
+ 1 y =
1
c
x
y
0
1
3
1
3
y =
x
2
(2 )
d
x
y
0
1
3
1
3
y =
x
2
(2 )
e
x
y
0
1
4
4
4
y =
1
2
x+2

+ 4
f
x
y
0
3
2
y = (2
2x
+ 2)
Exercise 8G
1 a A dilation of factor
1

2
from the y axis or a
dilation of factor 2 from the x axis
b A translation determined by the
vector
_
2
0
_
c Reection in the line y = x
d A dilation of factor
1
2
from the y axis
e A dilation of factor 3 from the x axis
f A translation determined by the
vector
_
3
0
_
g Reection in the y axis
h Reection in the x axis
2 a A dilation of factor 2 from the x axis
followed by a translation determined by the
vector
_
3
0
_
b A dilation of factor 2 from the x axis
followed by a translation determined by the
vector
_
0
3
_
c A dilation of factor 2 from the x axis
followed by a translation determined by the
vector
_
3
1
_
d A dilation of factor 2 from the x axis
followed by a translation determined by the
vector
_
3
0
_
e A dilation of factor 2 from the x axis
followed by a translation determined by the
vector
_
0
3
_
f A reection in the y axis followed by a
translation determined by the vector
_
3
0
_
g A dilation of factor 3 from the y axis
followed by a translation determined by the
vector
_
1
4
_
h A dilation of factor
3
2
from the x axis
followed by a translation determined by the
vector
_
2
0
_
Exercise 8H
1 a {(x, y) : y = |x +1| +3}
x
y
(2, 4)
(1, 3)
4
0
b {(x, y) : x = |y|}
x
y
(5, 5)
(5, 5)
0
c
_
(x, y) : y =

x
4

_
x
y
0
(4, 1) (4, 1)
P1: FXS/ABE P2: FXS
9780521740494ans-1-10.xml CUAU033-EVANS August 22, 2009 11:0
A
n
s
w
e
r
s
Answers 679
d {(x, y) : y = |x|}
x
y
(3, 3) (3, 3)
0
e {(x, y) : x = |y +1| +3}
x
y
x
y
x
y
0
(4, 2)
(3, 1)
4
f {(x, y) : x = |y 3| 1}
x
y
0 2
2
(1, 3)
4
g {(x, y) : y = 2|x|}
x
y
0
(1, 2)
(1, 2)
2 a A translation determined by the
vector
_
0
3
_
b A translation determined by the
vector
_
3
3
_
c A dilation of factor
1
2
from the y axis
d A reection in the x axis followed by a dilation
of factor 2 from the x axis
3 a
_
(x, y) : y =
_
x
2
__
x
y
2 4
2
2
0 2 4
4
6
4
b {(x, y) : y = [x 2]}
x
y
2
4
2
2
0
2 4
4
6
4
c {(x, y) : y = [x] +2}
x
y
2 4
2
2
0
2 4
4
6
4
d {(x, y) : x = [y]}
x
y
2 4
2
2
0
2 4
4
6
4
e {(x, y) : x = [y]}
x
y
2 4
2
2
0 2 4
4
6
4
f {(x, y) : y = [x 4]}
x
y
2
4
2
2
0
2 4
4
6
4
P1: FXS/ABE P2: FXS
9780521740494ans-1-10.xml CUAU033-EVANS August 22, 2009 11:0
A
n
s
w
e
r
s
680 Essential Advanced General Mathematics
g
_
(x, y) : y =
_
x
2
__
x
y
2 4
2
2
0
2 4
4
6
4
h {(x, y) : y = [x] +2}
x
y
2
4
2
2
0
2 4
4
6
4
Exercise 8I
1 a y = f (x 2) +3 b x = f (y)
c y = f (2x) d y = 2 f (x)
2 a 2
x+3
b 4
x
c 2
x
2
d 2
x1
3 4 f
_
x
2
_
= x
2
, a dilation of factor 2 from the y
axis and a dilation of factor 4 from the x axis
take y = f (x) to y = 4 f
_
x
2
_
, a sequence of
transformations that results in the original
function.
4 f (2x 3) +4 =
1
2x 3
+4, a dilation of
factor
1
2
from the y axis followed by a
translation
_
_
3
2
4
_
_
5 3 f (2 x) = 3(x 2)
2
, a reection in the x
axis, a dilation of factor 3 from the x axis and the
translation
_
2
0
_
Exercise 8J
1 a
_
8
3
_
b
_
3a b
a +3b
_
2 (1, 0) (2, 4) , (0, 1) (1, 3),
(3, 2) (4, 6)
3 a (1, 0) (2, 1) , (1, 2) (4, 1)
b (1, 0) (2, 0) , (1, 2) (2, 2)
c (1, 0) (2, 3) , (1, 2) (4, 5)
4 a
_
2 3
3 1
_
b Yes, if the points are not collinear with the
origin.
c
_
1 2
1 2
_
, range is {(x, y) : y = x}
5 a
_
1 0
0 1
_
b
_
0 1
1 0
_
c
_
0 1
1 0
_
d
_
1 0
0 2
_
e
_
3 0
0 3
_
f
_
3 0
0 1
_
Multiple-choice questions
1 C 2 B 3 D 4 C 5 D
6 E 7 A 8 B 9 B 10 B
Short-answer questions
(technology-free)
1 a (9, 1) b (3, 2) c (0, 1)
d (3, 1) e (3, 1) f (1, 3)
2 a y = (x +2)
2
+3
b y =

x c y = x
2
3 a (x, y) (y 2, x +3)
b (x, y) (x, 5y)
c (x, y) (4x 2, y +3)
d (x, y) (x 2, 4 (y +3))
4 a y =
2
3
x 1
b y = 4x 2
c y = 2x +7 d 2x + y +1 = 0
e 2x + y = 1 f y =
1
2
(x +1)
5 a y = 3

x b y = 5(x
2
2)
c y = 5
1
16
(x +2)
2
d y = 20 4(x +2)
2
6 a y = |x|
x
y
0 1 1
1
b y = |2x| +3
x
y
5
3
1 1 0
c y = 4 |2x|
x
y
0 2 2
4
7 a (x, y) (x +3, 2y +4)
b (x, y) (x +4, 2y +3)
c (x, y)
_
x
3
, y 4
_
d (x, y) (x +1, 2y +1)
e (x, y) (x +2, 3 y)
P1: FXS/ABE P2: FXS
9780521740494ans-1-10.xml CUAU033-EVANS August 22, 2009 11:0
A
n
s
w
e
r
s
Answers 681
8 a x
2
= y 1
x
y
1 1
0 1
2
b (x 1)
2
= y +2
x
y
0 1 2
(1, 2)
2
1
1
c (x 2)
2
= 3(y 2)
x
y
(2, 2)
0
1
3
3
d y = [4x]
x
y
1
2
0
1
2
1
2
1
4
3
4
e y = 3|2x 1|
x
y
0
3
1
1
2
f y = 2 3|x 2|
x
y
0
2
2 4
4
Extended-response questions
1 a (4, 6)
b (x, y) (x 3, y)
(3 x, y) (6 x, y)
c (x, y) (x +6, y)
d i translation
_
m
0
_
, reection in the y axis,
translation
_
m
0
_
ii (x, y) (x +2m, y)
e i translation
_
0
n
_
, reection in the x axis,
translation
_
0
n
_
ii (x, y) (x, y +2n)
f i y = x +3 ii y = x +6
iii y = (6 x)
2
iv y = (3 x)
2
2 a A

(1, 3)
b i
1
3
ii 3
c i
q
p
ii A

(q, p)
d (x, y) (y, x)
e i y = x ii x = y
2
iii x
2
+ y
2
= 1 iv y =
1
x
3 a (1, 3) b (a, b)
c y = f (x)
d i (x, y) (6 x, y)
ii y = 3x 19
e (x, y) (2m x, 2n y)
f (x, y) (y n +m, n +m x)
g (x, y) (n +m y, x m +n)
h i x = (1 y)
2
1
ii
x
y
0
(1, 1)
4 a i Dilation from the x axis of factor
3
125
ii (x, y) (x, y)
iii (x, y) (x +25, y +15)
iv (x, y)
_
x +25,
3
125
y +15
_
b i y =
3
125
(x 25)
2
+15
ii (x, y) (x +50, y)
iii y =
3
125
(x 75)
2
+15
c i (x, y)
_
x +
m
2
,
4ny
m
2
+n
_
ii y =
4n
m
2
_
x
m
2
_
2
+n
iii y =
4n
m
2
_
x
3m
2
_
2
+n
5 a i y =
_
x +1
2
_
+3
ii
x
y
4
3
3
2
1
1 1 3 5 7
0
iii reection in the x axis, translation
determined by the vector
_
3
2
_
b i
x
y
1
1 2 3
0
1
2
ii x = 1 or x =
1
3
Chapter 9
Exercise 9A
1 2 parts = 2000, 7 parts = 7000
2 1 part = 3000, 2 parts = 6000
3 3.6 4 264 5 22.5
6 60

, 50

and 70

7 $14
P1: FXS/ABE P2: FXS
9780521740494ans-1-10.xml CUAU033-EVANS August 22, 2009 11:0
A
n
s
w
e
r
s
682 Essential Advanced General Mathematics
8 30 g zinc, 40 g tin
9 16 white and 8 green beads
10 5.625 km 11 $1200 12
3
5
13 : 1 14 1 : 1 15 6 : 7
16 8.75 km
Exercise 9B
1 a AAA, 11.25 cm b AAA, 11
2
3
cm
c AAA, 3 cm d AAA, 7.5 cm
2 a AAA, 6 cm b AAA, 1
1
3
cm
c AAA, 2
2
3
cm d AAA, 7.5 cm
3 AC = 17.5, AE = 16, AB = 20
4 4.42 m 5 7.5 m 6 15 m
7 22.5 m 8 10
10
31
m 9 x = 6
2
3
10 83.6 cm 11 x =
39
46
12 40
1
7
m
13 7.2 m 14 1
14
15
m
15b x = 10 c y = 2

5, z = 5

5
16 a =
36
7
17 7.11 m 18 1.6 m
19 2
1
7
m 20 a = 3

5, x = 5, y = 2

5
Exercise 9C
1 a 1 : 2 : 3 : 4 b 1 : 4 : 9 : 16
c Yes, the second ratio is the square of the rst.
2 a 1 : 2 : 3 : 4 b 1 : 4 : 9 : 16
c Yes, the second ratio is the square of the rst.
3 19
4
9
cm
2
4 4.54 cm
2
5 a

3 cm b
4

3
3
cm c
4
3
6 4 : 5 7 22.5
8 a 1 : 2 : 3 b 1 : 2 : 3 c 1 : 8 : 27
d Yes, the third ratio is the cube of the rst.
9 a i 2 : 3 ii 2 : 3 iii 2 : 3
b 8 : 27
c Yes, the ratios in a are cubed to form the
ratios in b.
10 a 3 : 2 : 5
b Volumes are 36,
32
3
and
500
3
cm
3
.
Ratio of volumes is 27 : 8 : 125
c Yes, the ratios in a are cubed to form the
ratios in b.
11 8 : 1 12 27 : 64 13 2 : 3
14 a 4 : 3 b 4 : 3
15 a 4 : 1 b 8 : 1
16 a 1 : 100 b 1 : 1000
c 1 : 10 d 1 : 1
17
27
16
and 4 litres.
18 125 and 216 mL
19 a 1 : 50 b 1 : 125 000
c 3 cm d 7500 cm
2
20 a 12 : 13 b 1728 : 2197
21 a 4 b 3.75
22 3 : 4 23 4.5 cm
Exercise 9E
2b i 4 ii

10
4 a i 36

ii 72

c 0.62
5
0
= 1,
1
=
1 +

5
2
,
2
=
3 +

5
2
,

3
= 2 +

5,
4
=
7 +3

5
2
,

1 =

5 1
2
,
2
=
3

5
2
,

3
=

5 2,
4
=
7 3

5
2
Multiple-choice questions
1 D 2 B 3 D 4 C 5 B
6 D 7 C 8 E 9 E 10 E
Short-answer questions
(technology-free)
1 b i 20 cm ii 10 cm
c XP : PY = 2 : 1, PQ : YZ = 2 : 3
2 a 3 cm b 5 : 3 c 3 : 5
3
210
23
m 4
15
8
5 12.25 6 12
7 a 96 g b 2 : 1
c 1000 cm
3
d 100 mm
8 b 25 : 36 c 48 cm
9 a 20 : 3 b 1.6 m
2
c
8
27
m
3
10 a 2% b 3%
11 a
1
3
b
1
3
c
2
3
d
2
3
e
1
9
f
4
9
Extended-response questions
1 a EBC c
h
q
=
x
x + y
e
20
9
2 a Rhombus, CF = 1 c ACF
e
1 +

5
2
3 x = 8 or x = 11
4 a BDR and CDS, BDT and
BCS, RSB and DST
P1: FXS/ABE P2: FXS
9780521740494ans-1-10.xml CUAU033-EVANS August 22, 2009 11:0
A
n
s
w
e
r
s
Answers 683
b
z
y
=
p
p +q
c
z
x
=
q
p +q
5 a i 9 cm ii 12 cm
iii
1
16
iv
9
16
b i 16a cm
2
ii 3a cm
2
7 15

26 m
Chapter 10
Exercise 10A
1 a

3
b
4
5
c
4
3
d
11
6
e
7
3
f
8
3
2 a 120

b 150

c 210

d 162

e 100

f 324

g 220

h 324

3 a 34.38

b 108.29

c 166.16

d 246.94

e 213.14

f 296.79

g 271.01

h 343.77

4 a 0.66 b 1.27 c 1.87 d 2.81


e 1.47 f 3.98 g 2.39 h 5.74
5 a 60

b 720

c 540

d 180

e 300

f 330

g 690

h 690

6 a 2 b 3 c
4
3
d 4 e
11
6
f
7
6
Exercise 10B
1 a 0, 1 b 1, 0 c 1, 0
d 1, 0 e 0, 1 f 1, 0
g 1, 0 h 0, 1
2 a 0.95 b 0.75 c 0.82
d 0.96 e 0.50 f 0.03
g 0.86 h 0.61
3 a 0, 1 b 1, 0 c 1, 0
d 1, 0 e 1, 0 f 0, 1
g 0, 1 h 0, 1
Exercise 10C
1 a 34.23 b 2.57 c 0.97 d 1.38
e 0.95 f 0.75 g 1.66
2 a 0 b 0 c undened d 0
e undened f undened
3 a 0 b 0 c 0 d 0 e 0 f 0
Exercise 10D
1 a 67

59

b 4.5315 c 2.5357
d 6.4279 e 50

12

f 3.4202
g 2.3315 h 6.5778 i 6.5270
2 a a = 0.7660, b = 0.6428
b c = 0.7660, d = 0.6428
c i cos 140

= 0.7660,
sin 140

= 0.6428
ii cos 140

= cos 40

Exercise 10E
1 a 0.42 b 0.7 c 0.42 d 0.38
e 0.42 f 0.38 g 0.7 h 0.7
2 a 0.7 b 0.6 c 0.4 d 0.6
e 0.7 f 0.7 g 0.4 h 0.6
3 a a =
1
2
b b =

3
2
c c =
1
2
d d =

3
2
e tan( ) =

3
f tan() =

3
4 a

3
2
b
1
2
c

3
d

3
2
e
1
2
Exercise 10F
1 a

3
2
b
1

2
c
1

3
d
1
2
e
1

2
f

3
g

3
2
h
1

2
i
1

3
2 a sin =

3
2
, cos =
1
2
, tan =

3
b sin =
1

2
, cos =
1

2
, tan = 1
c sin =
1
2
, cos =

3
2
, tan =
1

3
d sin =

3
2
, cos =
1
2
, tan =

3
e sin =
1

2
, cos =
1

2
, tan = 1
f sin =
1
2
, cos =

3
2
, tan =
1

3
g sin =

3
2
, cos =
1
2
, tan =

3
h sin =
1

2
, cos =
1

2
, tan = 1
i sin =

3
2
, cos =
1
2
, tan =

3
j sin =

3
2
, cos =
1
2
, tan =

3
P1: FXS/ABE P2: FXS
9780521740494ans-1-10.xml CUAU033-EVANS August 22, 2009 11:0
A
n
s
w
e
r
s
684 Essential Advanced General Mathematics
3 a

3
2
b
1

2
c
1

3
d not dened e 0 f
1

2
g
1

2
h 1
Exercise 10G
1 a 2 and 2 b and 3
c
2
3
and
1
2
d 4 and 3
e
2
3
and 4 f

2
and
1
2
g 4 and 2
2 a
x
y
0
3

2
3
Amplitude = 3, Period =
b
y
2
3
4
3
2
2
2
0
Amplitude = 2, Period =
2
3
c
y
4
4
0
2 3 4

Amplitude = 4, Period = 4
d
y
1
2
1
2
0
2
3
4
3
2

Amplitude =
1
2
, Period =
2
3
e
y
4
0
4
4
3
2
3
2
x
Amplitude = 4, Period =
2
3
f
x
y
5
5
0

2
4
3
4
5
2
3
4
7
2
Amplitude = 5, Period =
g
x
y
3
0
3
3 4 2
Amplitude = 3, Period = 4
h
x
y

2
8
3
8

8
5
8
7
4
3
2
0
2
Amplitude = 2, Period =

2
i
x
y
2
2
0
2
3
2
9 6 3
Amplitude = 2, Period = 6
P1: FXS/ABE P2: FXS
9780521740494ans-1-10.xml CUAU033-EVANS August 22, 2009 11:0
A
n
s
w
e
r
s
Answers 685
3 a
x
y
2
3
2
3
2

2
0 2
1
1
b
6 3 3 6
2
0
2
x
y
c
x
y
2
3
2
3
4
3
5
3
2
6
5
6
7
2

2
0
2
6
11
d
3
4
3
2
3

3
5 2
x
y

2
0
2
4
2
5
x
y
2
3
2
5
2,
5
4
0
3
5 a
x
y
2
0
y = cos x
y = sin x
b

4
,
5
4
Exercise 10H
1 a
x
y

2 4
3
4
4
0
y = 4cos(2x)

Period = , Amplitude = 4, y = 4
b
4
3
y

2
2
f() = sin(2) 2
0
Period = , Amplitude =

2, y =

2
c
3
2
6

2
x
y
2
0
2
f (x) = 2sin(3x)
Period =
2
3
, Amplitude = 2, y = 2
2 a
y

2 2
3
2
5 2
3
0
3

Period = 2, Amplitude = 3, y = 3
b
2
1
0
1

y
Period = , Amplitude = 1, y = 1
P1: FXS/ABE P2: FXS
9780521740494ans-1-10.xml CUAU033-EVANS August 22, 2009 11:0
A
n
s
w
e
r
s
686 Essential Advanced General Mathematics
c
12
5
12
13
2
0
2

y
Period =
2
3
, Amplitude = 2, y = 2
d
y
3
3

2
3
0

Period = , Amplitude =

3, y =

3
e

2
y

3
0
3

Period = , Amplitude = 3, y = 3
f
2
2
0
y
12
5
4

12


Period =
2
3
, Amplitude = 2, y = 2
g
y
3
4
6
5
3
2
2
0
Period = , Amplitude =

2, y =

2
h

2
y
3
0
3


Period = , Amplitude = 3, y = 3
i

2
3
0
3
y

2
Period = , Amplitude = 3, y = 3
3 a f (0) =
1
2
, f (2) =
1
2
b
2,
1
2
0,
1
2
5
6 6
11
4
3
, 1

3
, 1
1
0
1
x
4 a f (0) =

3
2
, f (2) =

3
2
b
5
6
4
3 6
11
x
y
3
2
2,

3
1
0
1
3
2

5 a f () =
1

2
, f () =
1

2
b
x
y
1
0
1

2
2

,
1
,
1
_
2
0,
1

P1: FXS/ABE P2: FXS


9780521740494ans-1-10.xml CUAU033-EVANS August 22, 2009 11:0
A
n
s
w
e
r
s
Answers 687
Exercise 10I
1 a
5
4
and
7
4
b

4
and
7
4
2 a
3
3
,
3
4
b

3
,
2
3
c
2
3
,
2
3
3 a 150 and 210 b 30 and 150
c 120 and 240 d 120 and 240
e 60 and 120 f 45 and 135
4 a 0.93 and 2.21 b 4.30 and 1.98
c 3.50 and 5.93 d 0.41 and 2.73
e 2.35 and 3.94 f 1.77 and 4.51
5 a 0.64, 2.498, 6.93, 8.781
b
5
4
,
7
4
,
13
4
,
15
4
c

3
,
2
3
,
7
3
,
8
3
6 a
7
12
,
11
12
,
19
12
,
23
12
b

12
,
11
12
,
13
12
,
23
12
c

12
,
5
12
,
13
12
,
17
12
d
5
12
,
7
12
,
13
12
,
15
12
,
21
12
,
23
12
e
5
12
,
7
12
,
17
12
,
19
12
f
5
8
,
7
8
,
13
8
,
15
8
7 a 2.034, 2.678, 5.176, 5.820
b 1.892, 2.820, 5.034, 5.961
c 0.580, 2.562, 3.721, 5.704
d 0.309, 1.785, 2.403, 3.880, 4.498, 5.974
8
x
y
,
5
3
1
2
,
4
3
1
2

,
2
3
1
2
,
2
3
1
2

,
4
3
1
2

,
5
3
1
2

,
1
2

3
,
1
2

1
0
1
Exercise 10J
1 a
x
y
1
3
0
1
6
11 7
6
b
x
y
2 3
2 3
6

6
7
3
4
0
3

c
1 + 2
1 2
4
5
4
3
0 x
y
2
d
4

4
5
x
y
2
4
0
e
2
3
0 x
y
1 + 2
1 2
(2, 0)
2 a
x
y
6
11
6
7
2

6
5
2
3
2 2
(2, 2)
(2, 2)
2
0
4
P1: FXS/ABE P2: FXS
9780521740494ans-1-10.xml CUAU033-EVANS August 22, 2009 11:0
A
n
s
w
e
r
s
688 Essential Advanced General Mathematics
b
12
23
12
19
12
15
12
11
12
7
12
21
12
17
12
13
12
3
12

12
9
12
5
(2, 1.414)
(2, 1.414)
x
y
2 2

2
0
2
c
(2, 3)
(2, 3)
x
y
2 2 0
3
1
5
d
3
5
3
4
3
2
(2, 3) (2, 3)
x
y
2
1
1
0
3

3
2
3
4
3
5
2
3
3

e
x
y
6
5
2
3
2
3
6
7
6
11
6

2

2

2
(2, 2)
(2, 2)
2
2
3
1
0
f
12
19
4
7
12
17
4
5
12
7
4
3
4

x
y
12
5
2 2
1
3
0
(2, 1+ 3)
(2, 1 + 3) 1 + 3
P1: FXS/ABE P2: FXS
9780521740494ans-1-10.xml CUAU033-EVANS August 22, 2009 11:0
A
n
s
w
e
r
s
Answers 689
3 a
4
3
12
7
4

12
5
(, 1 + 3) (, 1 + 3)
1 + 3
x
y

3
0
1
b
4
3
4

12
11
12

(, 3 + 1)
(, 3 + 1)

3
0
1
x
y
3 + 1
c
y
x
3
3 2
0
2 + 3
3 (, )
3 (, )
6
5
3
2
6

Exercise 10K
1
a
_
_
_
_
_

3
2

1
2
1
2

3
2
_

_
,
_

3
2
,
1
2
_
b
_
_
_
_
1

2
1

2
1

2
_

_
,
_
1

2
,
1

2
_
c
_
0 1
1 0
_
, (0, 1)
d
_
_
_
_
_

1
2

3
2

3
2

1
2
_

_
,
_

3
2
,
1
2
_
e
_
_
_
_
_

2
1

2

1

2
_

_
, (0, 1)
f
_
_
_
_
_

3
2
1
2

1
2

3
2
_

_
,
_

1
2
,

3
2
_
2 a rotation of

6
about O in an anticlockwise
direction
b rotation of
4
3
about O in an anticlockwise
direction
c rotation of = cos
1
_
4
5
_
, where

_
0,

2
_
about O in an anticlockwise
direction
3 a
_
_
_
_
1

2
1

2
1

2
1

2
_

_
b
_
1

2
,
3

2
_
c
i
_
_
_
_
1

2
1

2
1

2
1

2
_

_
ii
_
0 1
1 0
_
iii
_
_
_
_

3
2
1
2
1
2

3
2
_

_
iv
_
_
_
_
1

2
1

2
1

2
1

2
_

_
d sin =
3
5
,
_
_
_
4
5
3
5
3
5
4
5
_

_
Exercise 10L
1 a i 1.83 10
3
hours
ii 11.79 hours
b April 26 (t = 3.856), August 14 (t = 7.477)
2 a
13
D
10
7
0
3 6 12 18 24 t
b {t : D (t ) 8.5} = {t : 0 t 7}
{t : 11 t 19} {t : 23 t 24}
c 12.9 m
3 a p = 5, q = 2
b D
7
5
3
0
6 12 t
c A ship can enter 2 hours after low tide.
P1: FXS/ABE P2: FXS
9780521740494ans-1-10.xml CUAU033-EVANS August 22, 2009 11:0
A
n
s
w
e
r
s
690 Essential Advanced General Mathematics
4 a 5 m b 1 m
c t = 0.524 s, 2.618 s, 4.712 s
d t = 0 s, 1.047 s, 2.094 s
e Particle oscillates about the point x = 3 from
x = 1 to x = 5
5 a 19.5

C b D = 1 +2 cos
_
t
12
_
c
3
2
1
0
1
6 12 18 24 t
D
d {t : 4 < t < 20}
6 a 2 am b 8 am and 8 pm
7 a i
3
2
ii 12
iii d (t ) =
7
2

3
2
cos

6
t iv 1.5 m
b Between 9 pm and 3 am, and 9 am and
3 pm, each day.
8 a
5
4
3
2
1
4 8 12 16 20 24
0
D
t
b The boat can enter 4 hours before noon and
must leave by 4 pm.
c The boat can enter at 6.40 am and must leave
by 5.20 pm.
9 a i 52 weeks ii 3000
iii [1000, 7000]
b i N(0) = 1194.95; N(100) = 1021.87
ii
1000
23 52 75 100
7000
N
(100, 1021.87)
(0, 1194.95)
t (weeks)
c i t = 23, 75
ii 49
d
_
14
1
3
, 31
2
3
_

_
66
1
3
, 83
2
3
_
e d = 25 000, a = 15 000, b = 10, c = 5
Multiple-choice questions
1 B 2 A 3 D 4 D 5 C
6 D 7 E 8 E 9 B 10 B
Short-answer questions
(technology-free)
1 a
11
6
b
9
2
c 6 d
23
4
e
3
4
f
9
4
g
13
6
h
7
3
i
4
9
2 a 150

b 315

c 495

d 45

e 1350

f 135

g 45

h 495

i 1035

3 a
1

2
b
1

2
c
1
2
d

3
2
e

3
2
f
1
2
g
1
2
h
1

2
4 a 2, 4 b 3,

2
c
1
2
,
2
3
d 3, e 4, 6 f
2
3
, 3
5 a y = 2 sin 2(2x)
x
y
2
0
2
4

8
3
b y = 3 cos
x
3
3
0
3
2
3
2
9
x
y
6 3
c y = 2 sin 3x
x

2
0
2
6

3
2
y
d y = 2 sin
x
3
2
0
2
2
9
2
3
x
y
3 6
P1: FXS/ABE P2: FXS
9780521740494ans-1-10.xml CUAU033-EVANS August 22, 2009 11:0
A
n
s
w
e
r
s
Answers 691
e y = sin
_
x

4
_
x
y
4
5
4
3
4
7
2
3
1
0
1
4

2
f y = sin
_
x +
2
3
_
3
4
1
0
1
3
2
x
y
g y = 2 cos
_
x
5
6
_
3
4
6
5
2
0
2
3

2
x
y
6
11
h y = 3 cos
_
x +

6
_
3
4
2
33
3
0
3
3
2
x
y
6 a
2
3
,

3
b

3
,

6
,
2
3
,
5
6
c

6
,
3
2
d
7
6
e

2
,
7
6
7 a f (x) = 2 sin 2x +1
x
y
3
1
0
1

b f (x) = 1 2 cos x
2
x
y
3

3

3
1
0
1
c f (x) = 3 cos
_
x +

3
_
2
6

3

x
y
3
0
3
6
5
6
7
6
10
d f (x) = 2 cos
_
x +

3
_
3
1
0
x
y
2
3

3
2
3
5
1
1.5
e f (x) = 1 2 sin 3x
x
y
3
1
1
2
0
18
17
18
5
18

Extended-response questions
1 a i 13.4 ii 2 iii 12
b 3 am, 9 am, 3 pm, 9 pm
c 2 < t < 10, 14 < t < 22
2 a 7.3

b min = 7

, max = 23

c Between 9.40 am and 4.30 pm


d
(0, 7.32)
0
(1.08, 7)
(13.08, 23)
(24, 7.32)
x
y
3 a

6
P1: FXS/ABE P2: FXS
9780521740494ans-1-10.xml CUAU033-EVANS August 22, 2009 11:0
A
n
s
w
e
r
s
692 Essential Advanced General Mathematics
b
t
x
, 3
1
3
, 3
4
3
, 3
0
5
6
, 3
11
6
2,
3
2

3
2

1
12
7
12
13
12
19
12
c 3 m d
5
6
s
e 1 s f
1
4
s
g i 24 m ii 30 m
4 a p = 6, q = 4.2 b 3 am, 3 pm c 6 m
d 7 am, 11 am, 7 pm, 11 pm e 8 hours
5 a i 1 < k < 1, 1 < k < 3
ii k = 1 or k = 3
iii k < 1 or k > 3
b A translation of 1 unit in the negative
direction of the y axis, followed by a dilation
of factor
1
2
from the x axis, and a
dilation of factor 3 from the y axis.
c i h =

2
ii h =

6
6 a A translation of

2
units in the positive
direction of the x axis
b A dilation of factor
1
2
from the y axis,
followed by a translation of

4
units in the
negative direction of the x axis, and a dilation
of factor
1
4
from the x axis.
c i y = sin
_
x
2
_
+4
ii range = [3, 5], period = 4
7 a x = 4, y = 9
b i 4 ii 2
c i 8 and 0 ii 12 and 8
d i
2
3
ii
e

6
s f
5
6
,
3
2
,
13
6
,
17
6
g
5
6
,
11
6
,
17
6
,
23
6
h
t
9
4
0
2

x = 4sin(3t) + 4
, 12

3
,
8

6
,
8
5
6
(, 9)
(, 4)
6

y = 2sin 2t + 10
i
5
6
s j 2 s
8 a One possible set of values is
a = 4, b = 4, n = 2 and
c = 3, d = 11, m = 2
b
11
4
0
(1, 11)
(1, 4)
, 14
3
4
, 8
1
4
3
4
x = 4 sin(2t) + 4
y = 3 sin(2t) + 11
t
9 a For N: max = 7000, min = 1000 minimum
occurs in Ocober (when t = 10) maximum
occurs in April (when t = 4) For
M: max = 8500, min = 2500 minimum
occurs at end of January and November
(t = 1 and t = 11) maximum occurs in June
(when t = 6)
b t = 4.31 (April) population is 6961 t = 0.24
(Jan) population is 2836
c 145 556 in May (t = 5.19)
d t = 7.49 (July)
10 a
31.5
16.5
1.5
1
3
.
5
3
1
.
5
4
9
.
5
6
7
.
5
8
5
.
5
1
0
3
.
5
1
2
0
t (minutes)
h (m)
(0, 5.89)
(120, 30.99)
0
b 5.89 m c 27.51 s d 6 times
e 20 times f 4.21 m g 13.9 m
11 a i R

=
_
cos cos sin sin
sin cos +cos sin
cos sin sin cos
sin sin +cos cos
_
cos ( +) = cos cos sin sin ,
sin ( +) = sin cos +cos sin
ii R
2

=
_
cos
2
sin
2
2 sin cos
2 sin cos cos
2
sin
2

_
iii R
3

=
_
4 cos
3
3 cos (3 sin 4 sin
3
)
3 sin 4 sin
3
4 cos
3
3 cos
_
b R
1

=
_
cos sin
sin cos
_
=
_
cos () sin ()
sin () cos ()
_
, which
represents a rotation of

about O in a
clockwise direction
P1: FXS/ABE P2: FXS
9780521740494ans-11-15.xml CUAU033-EVANS August 22, 2009 11:5
A
n
s
w
e
r
s
Answers 693
c i x

= (x a) cos
(y b) sin +a,
y

= (x a) sin +(y b) cos +b


ii x

= (x 1) cos
(y 1) sin +1,
y

= (x 1) sin +(y 1) cos +1


d i x

= x cos y sin ,
y

= x sin + y cos
ii y =

2 2x

2 8

2x
2
iii y =
cos +sin
cos sin
x
e i (cos 2, sin 2) and (sin 2, cos 2)
ii a = cos 2, b = sin 2,
c = sin 2, d = cos 2
f y =

x
Chapter 11
Exercise 11A
1 a 0.6 b 0.6 c 0.7 d 0.3
e 0.3 f
10
7
g 0.3
h 0.6 i 0.6 j 0.3
Exercise 11B
1 a y
y = 2sin + cos

2
3
2
2
2
1
0
1
2

b
y
y = 3cos2 + 2sin2

2
3
2
1
0
1
2
3


c
y
1
0
1

y = sin2 cos
1
2
1
2
1
2
d
y
3
2
1
0
1
2
3

2
y = 3sin + cos2

e y
4
2
0
2
4
2
y = 4sin 2cos

Exercise 11C
1 a y
4
2
0
2
4

4
3
4
x
b
y
4
2
0
2
4
x

6
2
3
c
y
2
0
2
4
x
2
4
7
4
5
4
3
4
P1: FXS/ABE P2: FXS
9780521740494ans-11-15.xml CUAU033-EVANS August 22, 2009 11:5
A
n
s
w
e
r
s
694 Essential Advanced General Mathematics
d
y
2
1
0
2
1
3
4
x
2
2
3
2
e
y
2
0
2
4
x
2
2
3
2
f
y
4
2
0
2
x

2
3
4 4
Exercise 11D
1 a x =
(12n +1)
6
or x =
(12n +5)
6
b x =
(12n 1)
18
c x =
(3n +2)
3
2 a x =

6
or x =
5
6
b x =

18
or x =
11
18
c x =
2
3
or x =
5
3
3 x = n or x =
(4n 1)
4
;
x =
5
4
, ,

4
, 0,
3
4
, or
7
4
4 x =
n
3
; x = ,
2
3
,

3
, 0
5 x =
6n 1
12
or x =
3n +2
6
;
x =
2
3
,
7
12
,
1
6
,
1
12
,
1
3
,
5
12
,
5
6
,
11
12
Exercise 11E
1 a 1 b 1 c
2

3
=
2

3
3
d 1
e 2 f 2
g
1

3
=

3
3
h 2
2 a 1 b
2

3
=
2

3
3
c 1 d
1

3
=

3
3
e

2 f
2

3
=
2

3
3
g 1
h
2

3
=
2

3
3
i
1

3
=

3
3
3 a

6
,
5
6
b

6
,
7
6
c
3
4
,
5
4
d

4
,
5
4
4 a cos =
8
17
b sin =
15
17
c tan =
15
8
5 cos =
24
25
, sin =
7
25
6

29
5
7
8
31
8
15
4(6 +

5)
=
15(6

5)
124
Exercise 11F
1 a

2 +

6
4
b
1

3
2

2
=

6
4
2 a

2
4
b

3 +1

3 1
= 2 +

3
3 a

3 1
2

2
=

2
4
b

3 1
2

2
=

2
4
c
1

3
1 +

3
= 2 +

3
4 0 < u, v <

2
, sin (u +v) =
63
65
,

2
< u, v < , sin (u +v) =
63
65
,
0 < u <

2
,

2
< v < ,
sin (u +v) =
33
65
,

2
< u < ,
0 < v <

2
, sin (u +v) =
33
65
5 a

3
2
sin +
1
2
cos
b
1

2
(cos +sin )
c
tan +

3
1

3 tan
d
1

2
(sin cos )
6 a sin u b cos u
P1: FXS/ABE P2: FXS
9780521740494ans-11-15.xml CUAU033-EVANS August 22, 2009 11:5
A
n
s
w
e
r
s
Answers 695
7 a
119
169
b
24
25
c
24
7
d
169
119
e
33
65
f
16
65
g
65
33
h
7
24
8 a
63
16
b
24
7
c
56
65
d
24
25
9 a
7
25
b
3
5
c
117
44
d
336
625
10 a

3
2

for =
5
3

b
1
2
11 a 1 sin 2 b cos 2
Exercise 11G
1 a max = 5, min = 5
b max = 2, min = 2
c max =

2, min =

2
d max =

2, min =

2
e max = 2

3, min = 2

3
f max = 2, min = 2
g max = 4, min = 0
h max = 5 +

13, min = 5

13
2 a

2
, b 0,
2
3
, 2 c

6
,
3
2
d 0,
5
3
, 2 e 53.13

f 95.26

, 155.26

3 2 cos

2x +

6

2 sin

3x
5
4

5 a f (x) = sin x cos x =

2 cos

x
3
4

2 sin

x +
7
4

2 sin

x

4

x
y
2
2
0
1
2

4
3
4
5
4
7
4
b
f (x) =

3 sin x +cos x = 2 cos

x

3

= 2 sin

x +

6

x
y
2
0
1
2

3
4
3
5
6
11
6
2
c
f (x) = sin x +cos x =

2 cos

x

4

2 sin

x +

4

x
y
2
2
0
1
2
4
7
4
3
4
5
4
d f (x) = sin x

3 cos x
= 2 cos

x
5
6

= 2 sin

x +
5
3

= 2 sin

x

3

y
2
0
2
1
x

3
5
6
4
3
11
6
2
3
Multiple-choice questions
1 A 2 A 3 B 4 A 5 C
6 E 7 C 8 E 9 A 10 D
Short-answer questions
(technology-free)
2 a 5, 1 b 4, 2 c 4, 4
d 2, 0 e 1,
1
3
3 a

6
,
5
6
,
7
6
,
11
6
b

12
,
5
12
,
13
12
,
17
12
c

18
,
11
18
,
13
18
,
23
18
,
25
18
,
35
18
d

4
,
3
4
,
5
4
,
7
4
e

6
,
5
6
,
7
6
,
11
6
f
3
8
,
7
8
,
11
8
,
15
8
g

2
,
7
6
,
11
6
h

8
,
7
8
,
9
8
,
15
8
4 60

, 300

, 0

, 180

, 360

5 a
140
221
b
21
221
c
171
140
6 a
1
2
b 1
P1: FXS/ABE P2: FXS
9780521740494ans-11-15.xml CUAU033-EVANS August 22, 2009 11:5
A
n
s
w
e
r
s
696 Essential Advanced General Mathematics
7 a 1 b 0
8 a 5, 1 b 9, 1
10 a
1
9
b
4

5
9
c
8

5
81
12 a 2

3 b sin (x + y) +sin (x y)
13 a (0, 2

3) b

3
, 0

4
3
, 0

11
6
, 4

5
6
, 4

x
y
0
4
23
4
2
3
4
3
11
6
5
6
f (x) = 2

3 cos x 2 sin x
14
a x = 0,

2
, 2
b x =
7
6
,
11
6
c x = 0, , 2
d x =

2
,
3
2
e x =

6
,

3
,
7
6
,
4
3
f x =
7
12
,
3
4
,
19
12
,
7
4
15 a y = 2 cos
2
x
x
y
0
2
2

2
3
2

b y = 1 2 sin

2

x
2

x
y
0
1
3
4 2 10
3
2
3
c f (x) = tan 2x
x
y
2
2
3
2

2
1
0
1

16
2
9
17 a

85 cos ( ) where
= cos
1

85

b i

85 ii
2

85
iii = cos
1

85

+cos
1

85

Extended-response questions
1 b P = 10

5 cos ( ) where
= cos
1

; = 70.88

c k = 25 d = 45

2 a AD = cos +2 sin
b AD =

5 cos ( 63)

5 cos ( ) where = cos


1

c maximum length of AD is

5 m when
is 63

d = 79.38

3 b ii a = 1, b = 1
c
1 +

3
1 +

3 +

6
=
2

3 1

3 1
=

6 +

3 2
4 a i h
1
= cos ii h
2
= cos sin
iii h
3
= sin
2
cos
iv h
n
= sin
n1
cos , n N
c 19.47

5 a ii 2 cos

5
b iii 4 cos
2

2 cos

5
1 = 0
iv
1 +

5
4
6 b
2
3
or
1
2
Chapter 12
Exercise 12A
1 a 4.10 b 0.87 c 2.94
d 4.08 e 33.69

f 11.92
2
40

3
3
cm 3 66.42

, 66.42

and 47.16

4 23 m
5 a 9.59

35 m
6 a 60

b 17.32 m
7 a 6.84 m b 6.15 m
8 12.51

9 182.7 m 10 1451 m
11 a 5

2 cm b 90

12 3.07 13 37.8 cm 14 31.24 m


15 4.38 m 16 57.74 m
P1: FXS/ABE P2: FXS
9780521740494ans-11-15.xml CUAU033-EVANS August 22, 2009 11:5
A
n
s
w
e
r
s
Answers 697
Exercise 12B
1 a 8.15 b 3.98 c 11.75 d 9.46
2 a 56.32

b 36.22

c 49.54

d 98.16

or 5.84

3 a A = 48

, b = 13.84 cm, c = 15.44 cm


b a = 7.26, C = 56.45

, c = 6.26
c B = 19.8

, b = 4.66, c = 8.27
d C = 117

, b = 24.68, c = 34.21
e C = 30

, a = 5.40, c = 15.56
4 a B = 59.12

, A = 72.63

, a = 19.57 or
B = 120.88

, A = 10.87

, a = 3.87
b C = 26.69

, A = 24.31

, a = 4.18
c B = 55.77

, C = 95.88

, c = 17.81 or
B = 124.23

, C = 27.42

, c = 8.24
5 554.26 m 6 35.64 m 7 1659.86 m
8 a 26.60 m b 75.12 m
Exercise 12C
1 5.93 cm
2 ABC = 97.90

, ACB = 52.41

3 a 26 b 11.74 c 49.29

d 73
e 68.70 f 47.22

g 7.59 h 38.05

4 2.626 km 5 3.23 km
6 a 8.23 cm b 3.77 cm
7 55.93 cm
8 a 7.326 cm b 5.53 cm
9 a 83.62

b 64.46

10 a 87.61 m b 67.7 m
Exercise 12D
1 a 11.28 cm
2
b 15.10 cm
2
c 10.99 cm
2
d 9.58 cm
2
2 a 6.267 cm
2
b 15.754 cm
2
c 19.015 cm
2
d 13.274 cm
2
e 24.105 cm
2
or 29.401 cm
2
f 2.069 cm
2
Exercise 12E
1 45.81 cm
2 a 95

30

b 112

53

3 a 6.20 cm b 2.73 cm
2
4
x
y
4
2
0 4
4
4
A
y = 2
B
Area of A B = 9.83 square units
6 61.42 cm
2
7 a 125.66 m b 41.96%
8 a 10.47 m b 20.94 m
2
9 6.64 cm
2
10 r = 7 cm, =

18
7

c
or r = 9 cm,
=

14
9

c
11 247.33 cm
12 a 81.96 cm b 4.03 cm
2
Exercise 12F
1 400.10 m 2 34.77 m 3 575.18 m
4 109.90 m 5 16.51 m 6 056

7 a 034

b 214

8 a 3583.04 m b 353

9 027

10 ASB = 113

11 22.01

12 a BAC = 49

b 264.24 km
13 10.63 km
Exercise 12G
1 a 13 cm b 15.26 cm
c 31.61

d 38.17

2 a 4 cm b 71.57

c 12.65 cm
d 13.27 cm e 72.45

f 266.39 cm
2
3 10.31

at B, 14.43

at A and C
4 a 85 m b 45.04 m
5 17.58

6 1702.55 m
7 a 24.78

b 65.22

c 20.44

8 42.40 m 9 1945.54 m
10 a 6.96 cm b 16.25 cm
2
11 a 5 km b 215.65

c 6

33

Exercise 12H
1 a 4a
2
, 3a
2
and 12a
2
square units
respectively
b 14.04

c 18.43

d 11.31

2 a 35.26

b 45

3 a 0.28 b 15.78

4 a 15.51 cm b 20 cm c 45.64

5 a i 107 m ii 87 m iii 138 m


b 43.00

6 a 5

11 cm b 64.76

c 71.57

d 95.74

7 26.57

8 a 54.74

b 70.53

9 1.67 km 10 34.14 cm
11 a 141.42 m b 20.70

12 16 cm
13 a
a

3
2
cm b
a
2
14 a 26.57

b 39.81

c 38.66

P1: FXS/ABE P2: FXS


9780521740494ans-11-15.xml CUAU033-EVANS August 22, 2009 11:5
A
n
s
w
e
r
s
698 Essential Advanced General Mathematics
Multiple-choice questions
1 D 2 C 3 C 4 B 5 A
6 A 7 D 8 B 9 C 10 A
Short-answer questions
(technology-free)
1 a 5

11 b sin
1

5
6

or sin
1

5
6

2 a 20

3 cm b 20 cm
3 4

19 km
4 a 5

3 cm b
25

3
4
cm
2
c
105
4
cm
2
d
5(21 +5

3)
4
cm
2
5 143

6
17
28
7
3

93
31
8
11
c
6
9 a i 30

ii 15

b AT = 300(1 +

3) m,
BT = 150(

6 +

2) m
10

181 km
11 a AC =
12

3
5
km, BC = 2.4 km
b 57.6 km/h
12 a 26 tan
1

12
5

cm
b 169

tan
1

12
5

cm
2
13 180 cm
2
14 21.4 cm 15 11 m
Extended-response questions
1 a ACB = 12

, CBO = 53

,
CBA = 127

b 189.33 m c 113.94 m
2 a 4.77 cm b 180 cm
2
c 9.55 cm
3 a TAB = 3

, ABT = 97

, ATB = 80

b 2069.87 m c 252.25 m
4 a 184.78 m b 199.71 m c 14.93 m
5 a 370.17 m b 287.94 m c 185.08 m
6 a 8

2 cm b 10 cm c 10 cm
d 68.90

Chapter 13
13.1 Multiple-choice
questions
1 A 2 B 3 B 4 C 5 D 6 B
7 D 8 B 9 C 10 E 11 E 12 B
13 E 14 D 15 A 16 D 17 C 18 A
19 E 20 A 21 E 22 D 23 E 24 A
25 A 26 C 27 A 28 B 29 C 30 D
31 E 32 E 33 C 34 C 35 C 36 B
37 C 38 C 39 E 40 D 41 E
13.2 Extended-response
questions
1 a (x, y) (x +6, y +3)
b, c
x
y
8 4 6
2
1
5
1
5
A'
A''
B'
B''
C'
C''
A
B
C
0
d y = 2(x +3)
2
+2
e (x, y) (x +3, 2y +4)
f
x
y
1
0
2 3 4
16
(1.18, 7.27)
0.33
(2.82, 0.73)
2 a i ABC = 11.81

, BCA = 138.19

ii ABC

=108.19

, BC

A = 41.81

b i 24.56 units ii 114.00 units


iii 89.44 units
c ii 1788.85 sq units iii 3027.9 sq units
iv 1239.0 sq units
3 a (4, 1)
b i The image is a 4 1 rectangle
ii 1 sq unit iii 4 sq units
iv k sq units
c (x, y) (4x, y)
d i y =
1
16
x
2
ii y =
1
16
(x 2)
2
1
iii
x
y
6 2
1
0.75
y = x
2
y =
1
16
(x 2)
2
1
0
(2, 1)
e (x, y)

x +2,
y +3
5

4 a Rotation by cos
1
3
5
clockwise
about the origin.
b i x
2
+(y 1)
2
= 1
ii

x
4
5

2
+

y
3
5

2
= 1
c (0, 0) and

4
5
,
8
5

P1: FXS/ABE P2: FXS


9780521740494ans-11-15.xml CUAU033-EVANS August 22, 2009 11:5
A
n
s
w
e
r
s
Answers 699
5 a (3, 11) b
1
10

3 1
2 4

c a = 2, b = 3 d (5a, 5a)
e = 2, b = 2a or = 5, a = b
6
a

2
1

2
1

2
1

2
1

2
1

2
1

c a =

2, b = 0
d c =
3

2
, d =
1

2
e i x =
1

2
(x

+ y

), y =
1

2
(y

)
ii y x =
1

2
(x + y)
2
7 a i 5 m ii 8 m
b
0
3 6 9 12 15 18 21 24
2
5
8
(3, 8) (15, 8)
x (m)
t (s)
(9, 2) (21, 2)
c i 8 m ii 2 m
d i t = 0, 6, 12, 18, 24
ii t = 0.65, 5.35, 12.65, 17.35
8 a i

c
6
ii
5
3
cm
b 19.78 cm
c i 14.62 cm
2
ii 11.25 cm
2
iii 288.29 cm
2
9 b
1
2
c
a
2
8
d
1
4
e
a
2
32
f i A
n
=
1
2
a
2

1
4

n1
= a
2
(2)
12n
ii
2
3
a
2
10 a k
1
= 2, k
2
= 3
b i
x
y
3k
1
x = 3
3
y = k
1
(x 3)
0
ii c =
3

k
2
k
1
, the x axis intercept
of the image is
3

k
2
k
1
+3
iii e

= e +3, f

= f
iv y = k
1
(x 3)
k
2
(x 3)
2
11 a i (3, 1)
ii A

(3, 1), B

(5, 1), C

(3, 3)
b ii (1, 1), (2, 2)
iii
x
y
5
5
3
3
1
1
0
5
5
A' B'
C' A
B
C
y = x
iv (1, 1), (2, 2),

1
2

,
1
2

1 +

and

1
2

1 +

,
1
2

13 a 155 m
b i 16.00 m ii 29.04 m iii 17

c 32.7 cm
2
14 a 12.05 pm b 2752 km c 26.05

15 a a = 35, b = 25, c =
1
3
b i
5
3
,
11
3
s
ii 0.13, 1.20, 2.13, 3.20 s
c
1 2 3 4
10
20
30
40
50
60
47.5
h (cm)
t (s)
0
(4, 47.5)
h(t) = 35 + 25cos t +
1
3
3
, 60
5
, 10
8
3
, 10
2
3
, 60
11
3
16 a x =
1
2

p
2
+q
2
2pq cos
b y =
1
2

p
2
+q
2
+2pq cos
d

31 cm
17 b i 51.48 cm ii 4764.95 cm
2
iii 94.8%
18 b AC
2
= 61 +60 cos
c i 9.12 cm ii 43.18 cm
2
P1: FXS/ABE P2: FXS
9780521740494ans-11-15.xml CUAU033-EVANS August 22, 2009 11:5
A
n
s
w
e
r
s
700 Essential Advanced General Mathematics
Chapter 14
Exercise 14A
1 a x = 100, y = 50
b x = 126, y = 252, z = 54
c y = 145, z = 290
d x = 180, y = 90
e x = 45, y = 90, z = 270
f x = 110, y = 100
2 a x = 68, y = 121 b x = 112, y = 87
c x = 50, y = 110
3 110

, 110

and 140

4 ABC = 98

, BCD = 132

,
CDE = 117

, DEA = 110

,
EAB = 83

7 60

or 120

Exercise 14B
1 a x = 73, y = 81 b x = 57, q = 57
c x = 53, y = 74, z = 53
d x = 60, y = 60, z = 20, w = 100
e w = 54, x = 54, y = 72, z = 54
2 a 40

b 40

c 80

3 ACB = 40

, ABC = 70

, BAT = 40

4 32

and 148

Exercise 14C
1 a 10 cm
b 6 cm
2 7 cm 3 5

6 cm
Multiple-choice questions
1 B 2 A 3 E 4 A 5 C
6 A 7 C 8 B 9 A 10 A
Short-answer questions
(technology-free)
1 a x = 110, y = 70
b x = 35, y = 35
c x = 47, y = 53, z = 100
d x = 40, y = 40, z = 70
5 a x = 66 b x = 116
c x = 66, y = 114
7 3 cm
Extended-response questions
4 b 24 cm
2
Chapter 15
Exercise 15A
1 a
5
1
b
2
}
c
1
2
d
3
4
2 a = 5, b = 1 3 a = 3, b = 15
4 a

1
2

2
2

1
3

2
3

1
3

5 a
x
y
0
2
3
b
x
y
0
2 4
3
c
x
y
0
1 4
4
d
x
y
0
1
4
e
x
y
0
1 4
4
6 a
x
y
0
1
2
b
x
y
0
4
3
P1: FXS/ABE P2: FXS
9780521740494ans-11-15.xml CUAU033-EVANS August 22, 2009 11:5
A
n
s
w
e
r
s
Answers 701
c
x
y
0
4
3
3 1
d
x
y
0
4
3
2 4
e
x
0
2
2 1
5
y
f
x
0
1
3
3
y
7 a and c
8 a, b
x
y
0
1
1
3
3
2
2
1
1
4
4
C
D
B
A
d parallelogram
9 a i

2
1

ii

5
0

iii

4
2

b a +b = c
10 m = 11, n = 7
11 a i b
1
2
a ii b
b

MN =

AD
12 a

CB = a b,

MN =
1
2
(b a)
b

CB = 2

MN
13 a a b b c 2a d 2b
e a f b a g a +b
14 a a b b c a +b
d a b e b a
15 a a b b
1
3
(b a) c
1
3
(a +2b)
d
1
9
(a +2b) e
1
9
(4a b)
16 a u +v b v +w c u +v +w
17 a

OB = u +v,

OM = u +
1
2
v
b u
1
2
v c
2
3

u
1
2
v

d

OP =
2
3
(u +v) =
2
3

OB
e 2 : 1
Exercise 15B
1 2i 7 j
2 a 5i +6 j b 5i +6 j c 5i 6 j
3 a 5 b 2 c 5 d 13
4 a 13 b x = 2, y = 7
5 7i +
5
2
j
6 a i
2
5
i ii
2
5
i + j
iii
1
6

2
5
i + j

iv
1
3
i +
1
6
j
v 2i + j
b i

ON =
1
6

OA ii 1 : 5
7 4

2 units
8 a k =
3
2
, l = 1 b x = 6, y = 2
c x = 3, y = 3 d k =
1
3
, l =
5
3
9 a 3i 2 j b

13
10 a 2i +4 j b 6i + j c 5
11 a D = (6, 3) b F = (4, 3)
c G =

3
2
,
3
2

12 A = (1, 4), B = (2, 2), C = (0, 10)


13 a i 2i j ii 5i +4 j
iii i +7 j iv 6i +3 j
v 6i +3 j
b D = (8, 2)
14 a

OP = 12i +5 j ,

PQ = 6i +8 j
b 13, 10
15 a i

29 ii

116 iii

145
b (

29)
2
+(

116)
2
= (

145)
2
16 a i i 3 j ii 4i +2 j
iii 3i + j
b i

10 ii 2

5 iii

10
17 a i 3i +2 j ii 7 j
iii 3i 5 j iv
1
2
(3i 5 j )
b M =

3
2
,
9
2

18 a
1
5
(3i +4 j ) b
1

10
(3i j )
c
1

2
(i + j ) d
1

2
(i j )
e
6

13

1
2
i +
1
3
j

f
1

13
(3i 2 j )
Exercise 15C
1 a i +2 j k b 3i 5 j +6k
c

14 d 3

2 e 5i +6 j k
2 a i
3

11
i +
1

11
j
1

11
k
ii
6

11
i
2

11
j +
2

11
k
P1: FXS/ABE P2: FXS
9780521740494ans-11-15.xml CUAU033-EVANS August 22, 2009 11:5
A
n
s
w
e
r
s
702 Essential Advanced General Mathematics
b
15

11
i +
5

11
j
5

11
k
3

14
3

3
(i j +5k)
4 a i 3 j b

10 c
3
2
i +
1
2
j k
5 a 2 j +2k b i +2 j c i +2k
d i +2 j +2k e 2 j f 2 j +2k
g i +2 j 2k h i 2 j 2k
6 a
1
6
i +2 j +2k b
17
6
Exercise 15D
1 a i
4
5
p ii
1
5
p iii q p
iv
1
5
(q p) v
1
5
q
b RS and OQ are parallel
c ORSQ is a trapezium
d 120 cm
2
2 a i
1
3
a +
2
3
b ii
k
7
a +
6
7
b
b i 3 ii
7
2
3 a i

OD = 2i 0.5 j ,

OE =
15
4
i +
9
4
j
ii

170
4
b i p

15
4
i +
9
4
j

ii (q +2)i +(4q 0.5) j


c p =
2
3
, q =
1
2
5 a r +t b
1
2
(s +t )
Multiple-choice questions
1 C 2 C 3 E 4 A 5 B
6 B 7 A 8 C 9 D 10 C
Short-answer questions
(technology-free)
1 a
12
7
b 9
2 A(2, 1), B(5, 3), C(3, 8), D(0, 4)
3 p =
1
6
, q =
11
12
4 a 3

10 b
1
3

10
(i 5 j +8k)
5 6
6 a
1
5
(4i +3 j ) b
16
25
(4i +3 j )
7 a i a +b ii
1
3
(a +b)
iii b a iv
1
3
(2a b)
v
2
3
(2a b)
b

TR = 2

PT P, T and R are collinear.


8 a i 2 ii 5 iii 2
b

33
9

109 units
10 a 11i 2 j +3k b

30
c
1

30
(5i +2 j +k) d 2i +4 j
11 a (1, 10) b h = 3, k = 2
12 m = 2, n = 1
13 a b = a +c b b =
2
5
a +
3
5
c
Extended-response questions
1 a

31
32

15
20

c |OR| = 25
2 a

34 b

10

20 c r = i 9 j
3 a
1
2
b x = 2, y = 2
c p = 4, q = 2, r = 2
4 a (25, 7)

7
24

20
15

5 a (12, 4) b

k 12
4

160, k,

(k 12)
2
+16, k =
40
3
d 34.7

Chapter 16
Exercise 16A
1 a
Z O
A
2
30
b
Z O
B
3
45
c
Z O
C
2
2
60
120
d
Z O
D
4
30
P1: FXS/ABE P2: FXS
9780521740494ans-16-19.xml CUAU033-EVANS August 24, 2009 8:39
A
n
s
w
e
r
s
Answers 703
e
Z O
E
5
50
f
Z O
F
5
5
50
130
g
Z
O
G
130
5 h
Z
O
H
130
5
2 a
Z
O
A
1

b
Z
B
O
1
c
Z
C
O
2

2
d
Z
D
O
3
4
3
3 a
_
4

2,

4
_
= [4

2, 45

]
b
_
2,

3
_
= [2, 60

]
c
_
4,

6
_
= [4, 30

]
d
_
13, cos
1
_
5
13
__
[13, 113

]
e
_

61, sin
1
_
5

61
__
[

16, 40

]
f
_
2,

6
_
= [2, 30

]
g
_
13, +cos
1
_
5
13
__
[13, 113

]
h
_
5, cos
1
_
4
5
__
[5, 37

]
4 a (

3, 1) b (0, 4)
c
_
1

2
,
1

2
_
d (4, 0)
e (

3, 1) f
_
5
2
,
5

3
2
_
g (2, 0) h
_

1
2
,

3
2
_
5 a r =
3
cos
Z O
3
b r =
4
sin
Z O
4,

2
c r = 2 cos
Z O
1
[2, 0]
d r = 2, 0 6
Z O
[12, 6]
[10, 5]
[8, 4]
[4, 2]
2
11
11,
2
9
9,
e r =

6
4
Z O [1, ]
2,

2
6,

6
2
3 2
3
,
1
2
,
2
1
4
, 4
f r = cos 2
Z O
1,

2
1,
2
3
[1, ]
[1, 0]
g r = 5(1 +cos )
Z O
5,

2
5,
2
3
[10, 0]
h r = 2(1 sin )
Z
O
4,
2
3
[2, 0]
[2, ]
P1: FXS/ABE P2: FXS
9780521740494ans-16-19.xml CUAU033-EVANS August 24, 2009 8:39
A
n
s
w
e
r
s
704 Essential Advanced General Mathematics
i r = 3 cos +2
Z O
2,
2
3
[1, ]
[5, 2]
2,

2
j r =

cos 2
Z
O
[1, ]
[1, 0]
k r =
_

, 0 6
Z
2,

2
2,
[1, ]
O
6
3
,
2
3
2
2
, 2
6
6
, 6
l r = 2 sin 2
Z
O
2,
4
5
2,
4
7
2,
4
3
2,

4
6 a r = 4 b r =
6
cos +sin
c r =
sin
cos
2

d r
2
=
4
1 +3 sin
2

7 a x
2
+ y
2
= 4
b x
2
+ y
2
ax = a
_
x
2
+ y
2
c x
2
+ y
2
= ax
d (x
2
+ y
2
)
3
2
= 2a(x
2
+ y
2
+2xy)
e y
2
= a(a 2x) f y =
1
2a
(a
2
x
2
)
Exercise 16B
1
Re(z) Im(z)
a 2 3
b 4 5
c
1
2

3
2
d 4 0
e 0 3
f

2 2

2
2 a a = 2, b = 2
b a = 3, b = 2 or a = 2, b = 3
c a = 5, b = 0 d a =
2
3
, b =
1
3
3 a 6 8i b 6 i
c 6 2i d 7 3

2i
e 2 3i f 4 +2i
g 6 4i h 4 +6i
i 1 +11i j 1
4 a 4i b 6i c

2i
d i e 1 f 1
g 2 h 12 i 4
5 a 1 +2i b 3 +4i
c

2 2i d

6 3i
Exercise 16C
1 a 15 +8i b 8i c 2 +16i
d 2i e 5 f 4 +19i
2 a 2 +5i b 1 3i
c

5 +2i d 5i
3 a 2 +i b 3 2i c 4 +7i
d 4 7i e 4 7i f 1 +i
g 1 i h 1 i
4 a 2 +4i b 20 c 4
d 8 16i e 8i f 8
g
1
10
(1 +2i ) h 4 2i
5 a =
1
29
, b =
17
29
6 a
7
17

6
17
i
b i
c
7
2

1
2
i d
1
2

1
2
i
e
2
13
+
3
13
i f
3
20
+
1
20
i
7 a =
5
2
, b =
3
2
8 a
2
5
(3 +4i ) b
1
2
(1 i )
c
1
17
(4 +i ) d
1
130
(6 +43i ) e 2 2i
Exercise 16D
1 A 3 +i B 2i C 3 4i
D 2 2i E 3 F 1 i
2
x
y
1
0
1
2 3 4 1
1
2
3
4
2 3 4 5
Im(z)
Re(z)
a
e
c
f
d
b
P1: FXS/ABE P2: FXS
9780521740494ans-16-19.xml CUAU033-EVANS August 24, 2009 8:39
A
n
s
w
e
r
s
Answers 705
3 a z
1
+ z
2
= 3 i
x
0
4
6 3
1
1 2 3
5
Im(z)
Re(z)
z
1
+ z
2
z
1
z
2
b z
1
z
2
= 9 9i
x
Im(z)
Re(z) 9 6
0
4
3
5
9
z
1
z
2
z
1
z
2
4
x
Im(z)
Re(z)
3
8
6
3
1
0
c (8 + 6i)
d (1 3i)
b (1 3i)
a (1 + 3i)
1
10
3
10
i e
5
x
Im(z)
Re(z) 0 2
2
2
5
5
2
5
5
b (5 + 2i)
c (2 + 5i)
d (5 2i)
a, e (2 5i)
Exercise 16E
1 a 2i b 3i c

5i
d 2 4i e 1 7i f 1

2i
g
1
2
(3

3i ) h
1
4
(5

7i )
i
1
6
(1

23i ) j 1 2i
k
1
2
(3

11i ) l 3

5i
Exercise 16F
1 a 2 cis

3
b

2 cis
_

4
_
c 4 cis
5
6
d 4

2 cis
3
4
e 24 cis

3
f
1

2
cis
3
4
2 a 3i b
1

2
(1 +

3i ) or

2
2
(1 +

3i )
c

3 +i d
5

2
2
(1 i ) or
5

2
(1 i )
e 6(

3 i ) f 3(1 i )
g
5
2
(1 +

3i ) h
5
2
(1 +

3i )
3 a 3

2(1 +i ) b 6(1 +

3i )
c
5
2
(1

3i ) d 18(1 +

3i )
e 18(1 +

3i ) f

3(1 +i )
g

3 +i h 4
i 4(1

3i ) j
5
2
Multiple-choice questions
1 D 2 C 3 B 4 D 5 E
6 D 7 E 8 C 9 D 10 C
Short-answer questions
(technology free)
1 a
Z
O
[3, ]
b
Z
O

3
2,
c
Z
O
210
[2, 210]
d
_
3,
11
6
_
Z
O
6
11
2 a (3, 0) b (1,

3)
c (

3, 1) d
_
3

3
2
,
3
2
_
P1: FXS/ABE P2: FXS
9780521740494ans-16-19.xml CUAU033-EVANS August 24, 2009 8:39
A
n
s
w
e
r
s
706 Essential Advanced General Mathematics
3 a {[r, ] : r = 3}
Z O
3
b
_
[r, ] : =

3
_
Z
O

3
c {[r, ] : r = 4}
Z O
4
d
_
[r, ] : =
5
4
_
Z
O
4
5
4 a
_
3

2,

4
_
b
_
1,

6
_
c
_
5,
4
3
_
d
_
8,

4
_
5 a r = 4 b r = 3
c r = 0 or r =
8 cos
sin
2

= 8 cot cosec
d r = 0 or r =
4 sin
cos
2

= 4 tan sec
e r =
8
_
1 +3 sin
2

f r =
2
2 cos sin
6 a x
2
+ y
2
= 25 b x
2
+
_
y
3
2
_
2
=
9
4
c x
2
y
2
= 9
d 3
_
x +
16
3
_
2
y
2
=
64
3
e 3
_
x
7
3
_
2
+4y
2
=
196
3
f y =
1
2
(x
2
1)
7 a (2m +3p) +i (2n +3q) b p i q
c (mp +nq) +i(np mq)
d
(mp +nq) +i(np mq)
p
2
+q
2
e 2m
f (m
2
n
2
p
2
+q
2
) +i (2mn 2pq)
g
m i n
m
2
+n
2
h
(mp +nq) +i(mq np)
m
2
+n
2
i
3[(mp +nq) +i(np mq)]
p
2
+q
2
8
z
0
2
2
4
2
2
4
4 6 8
Im(z)
Re(z)
a
e
c
f
d
b
a 1

3i b 2 2

3i
c 8 d
1
4
(1 +

3i )
e 1 +

3i
f
1
4
(1

3i )
9 a

2 cis

4
b 2 cis
_

3
_
c

13 cis
_
tan
1

3
6
_
d 6 cis

4
e 6 cis
_
3
4
_
f 2 cis
_

6
_
10 a 1

3i b
3

2
2
+
3

2
2
i
c
3

2
2
+
3

2
2
i d
3

2
2
+
3

2
2
i
e
3

3
2

3
2
i
f 1 i
Extended-response questions
1 a
Z
O
Z
O
b x
2
+(y 1)
2
= 1 and (x 1)
2
+ y
2
= 1
c x
2
+(y a)
2
= a
2
, a circle with centre (0, a)
and radius a and (x a)
2
+ y
2
= a
2
, a circle
with centre (a, 0) and radius a
3 a z =

3 +i or z =

3 i
b i
Im(z)
Re(z)
0
1
1
3
3 z = + i
3 z = i
ii x
2
+ y
2
= 4 iii a = 2
4 a 6

2 b 6
5 a
Im(z)
Re(z) O A
Z P
Q
b

2 +1
P1: FXS/ABE P2: FXS
9780521740494ans-16-19.xml CUAU033-EVANS August 24, 2009 8:39
A
n
s
w
e
r
s
Answers 707
Chapter 17
Exercise 17A
1 a x =
9
2
x
y
O 9
2
b y = 10
x
y
O
10
c x
2
+ y
2
= 9
x
y
O
3
3
3
3
d y = 6 or y = 6
x
y
O
6
6
2 10y = 4x +29 3 x + y = 4
4 x
2
+ y
2
= 9
5
(3x 2)
2
4

3 (y 5)
2
4
= 1
6
_
x +
16
3
_
2
+ y
2
=
964
9
7 y = x
8
_
x +
4
3
_
2
+
_
y
2
3
_
2
=
80
9
9 x = 1 or x = 1
10
_
x
1
2
_
2
+
_
y
1
2
_
2
=
1
2
11
_
x
16
3
_
2
+ y
2
=
64
9
12 x + y = 1
13 y = 2x +1 14 y = 1 or y = 5
15 y
2
= 12 (x +1)
16 y =
1
12
x
2
1
17 The parabola with equation y
2
= 3x can be
described as the locus of points P(x, y) which
satisfy the property that the distance of P to the
point F
_
3
4
, 0
_
is equal to the distance PM, the
perpendicular distance to the line with equation
x =
3
4
Exercise 17B
1 a
x
y
O
8
8
3 3
b
x
y
O
10
10
5 5
c
x
y
O
3
3
8 8
d
x
y
O 3 3
5
5
2 a Centre = (3, 4)
O
4
12
4
3
33
2
3 +
33
2
x
y
b Centre = (3, 4)
1
O
4
9
6
5
24
5
x
y
c Centre = (2, 3)
O
1
7
3
2 +
7
2
2
7
2
x
y
P1: FXS/ABE P2: FXS
9780521740494ans-16-19.xml CUAU033-EVANS August 24, 2009 8:39
A
n
s
w
e
r
s
708 Essential Advanced General Mathematics
d Centre = (5, 0)
O
5
5
2 8
x
y
3
x
2
25
+
y
2
9
= 1 4
x
2
25
+
y
2
9
= 1
5
O
4
8
4
8
x
y
x
2
64
+
y
2
16
= 1
6
(x 4)
2
16
+
y
2
12
= 1
7
3x
2
16
+
(3y 28)
2
64
= 1
Exercise 17C
1 a
x
y
3
O
3
y =
8
3
x y =
8
3
x
b
x
y
y = 2x
y = 2x
5 5
O
c
x
y
3
O
3
y =
3
8
x
y =
3
8
x
d
x
y
3
O
3
y =
5
3
x y =
5
3
x
2 a Centre = (3, 4)
x
y
O
y = x 4
8
3
y = x + 12
8
3
3
35
2
3 +
35
2
4
4
b Centre = (3, 4)
x
y
y = x + 1
5
3
3
341
5
3
9
341
5
5
3
y =
x 9
4
O
c Centre = (2, 3)
x
y
2 4
y = 2x 1
y = 2x + 7
3 + 42
3 42
O
d Centre = (5, 0)
x
y y = x
5
3
25
3
25
3
20
3
y =
5
3
25
3
x +
20
3
25
3


2
5
8
O
e Centre = (0, 0)
y = x y = x
x
y
2 2
O
P1: FXS/ABE P2: FXS
9780521740494ans-16-19.xml CUAU033-EVANS August 24, 2009 8:39
A
n
s
w
e
r
s
Answers 709
f Centre = (0, 0)
y = 2x y = 2x
x
y
2 2
O
g Centre = (2, 1)
x
y
2 + 25
2 25
1
2
y = x
1
2
y = x 2
1
2
2 4 O
h Centre = (5, 3)
x
y
3
5
y =
x
5 + 52
5 52
3
5
y = x + 6
5 10
3
6
O
3
x
2
9

y
2
7
= 1 4
(x +6)
2
16

y
2
48
= 1
5
(15y 56)
2
256

15x
2
256
= 1 6
x
2
4

y
2
5
= 1
Multiple-choice questions
1 C 2 D 3 B 4 D 5 D
6 A 7 B 8 D 9 A 10 C
Short-answer questions
(technology free)
1 Centre = (2, 4) , radius = 2

5
2 Centre = (2, 0), axes intercept at (4, 0) and
(0, 0)
3 y = 3x 8 4 (x 3)
2
+(y 2)
2
= 36
5 y =
2x
3
, y =
2x
3
6 (x 8)
2
+
_
y +
2
3
_
2
=
160
9
7 Centre (2, 0)
x
y
5 1
25
3

25
3
O
8 x =
k
16
, where k is the given constant.
Extended-response questions
1 a y = x b (x 6)
2
+ y
2
= 36
c (x +2)
2
+(y 8)
2
= 32
d (x +10)
2
+ y
2
= 64
e (x 10)
2
+ y
2
= 64
f
x
2
100
+
y
2
64
= 1 g y = 0, 6 x 6
h
4x
2
25

4y
2
119
= 1, x
5
2
i
4x
2
25

4y
2
119
= 1, x
5
2
2 a y =
1
4
x
2
+3 b
3x
2
4
+
(3y 14)
2
16
= 1
c x
2
3y
2
+8y = 0
3 a Let triangle OAB have vertices with
coordinates O (0, 0), A (a, 0) and B (x, y).
The locus of the point B is a circle with
equation (x +a)
2
+ y
2
= 4k
2
where k is a
constant.
b Let triangle OAB have vertices with
coordinates O (0, 0), A (a, 0) and B (x, y).
The locus of the point B is a circle with
equation
_
x
ak
2
k
2
1
_
2
+ y
2
=
a
2
k
2
(k
2
1)
2
where k is a constant. (k = 1, a straight line)
c There are three cases to consider:
Case 1: Let quadrilateral OBAC have vertices
with coordinates O (0, 0), A(a, 0),
B(b, c) and C(x, y). The locus of the
point C is y =
2k
a
where k is a
constant.
Case 2: Let quadrilateral OABC have vertices
with coordinates O (0, 0), A(a, 0),
B(b, c) and C(x, y). The locus of the
point C is y =
c
b
x
2k
b
where k is a
constant.
Case 3: Let quadrilateral OACB have
vertices with coordinates O (0, 0),
A(a, 0), B(b, c) and C(x, y).
The locus of the point C is
y =
c
b a
x +
bc 2ac 2k
b a
where k is a constant.
P1: FXS/ABE P2: FXS
9780521740494ans-16-19.xml CUAU033-EVANS August 24, 2009 8:39
A
n
s
w
e
r
s
710 Essential Advanced General Mathematics
Chapter 18
18.1 Multiple-choice questions
1 B 2 C 3 B 4 D 5 C
6 C 7 B 8 D 9 B 10 D
11 B 12 A 13 B 14 D 15 A
16 B 17 C 18 B 19 B 20 B
21 D 22 E 23 A 24 C 25 C
26 A 27 C 28 E 29 C 30 D
31 E 32 C 33 C 34 B 35 D
36 E
18.2 Extended-response
questions
1 a

AE =
1
t +1
(2a +t b)
b

AE =
1
8
(7a +

AF) d t =
9
7
2 b (n 1) a nb +c
3 a

AB = b a,

PQ =
3
10
a +
1
2
b
b i n
_
3
10
a +
1
2
b
_
ii
_
k +
1
2
_
b
1
2
a
c n =
5
3
, k =
1
3
4 b BCA = x

, BOA = 2x

TAB = x

, TBA = x

5 a 4

2 blowing from SW b 200 m downstream


c true velocity = 43.1 km/h at bearing 80

d 222

6 b ii

ZG =
1
3
h

ZH +
1
3
k

ZK
iii
1
h
+
1
k
= 3 iv h =
2
3
(similarity)
v
4
9
cm
2
vi h =
1
2
; H is the midpoint of

ZX, K = Y
vii
1
1
1
2
1
2
1
2
k 1;
1
2
h 1
k
h
viii
1
1
1
2
1
2
2
3
,
4
9
k
A
Chapter 19
Exercise 19A
1 a 12 cm to the right of O
b 2 cm to the right of O
c moving to the left at 7 cm/s
d when t = 3.5 s and the particle is 0.25 cm to
the left of O
e 2 cm/s f 2.9 cm/s
2 a after 3.5 s b 2 m/s
2
c 14.5 m
d when t = 2.5 s and the particle is 1.25 m to the
left of O
3 a 3 cm to the left of O moving to the right at 24
cm/s
b v = 3t
2
22t +24 c after
4
3
s and 6 s
d 11
22
27
cm to the right of O and 39 cm to the left
of O
e 4
2
3
s f a = 6t 22
g when t =
11
3
s and the particle is
13
16
27
cm left of O moving to the left at
16
1
3
cm/s
4 a when t =
2
3
s and a = 2 cm/s
2
, and when
t = 1 and a = 2 cm/s
2
b when t =
5
6
s and the particle is moving to the
left at
1
6
cm/s
5 when t = 2 s, v = 6 cm/s, a = 14 cm/s
2
, when
t = 3, v = 5 cm/s, a = 8 cm/s
2
, when
t = 8 s, v = 30 cm/s, a = 22 cm/s
2
6 a t = 4 s and t = 1 b t =
3
2
s
Exercise 19B
1 a x = 2t
2
6t b at the origin O
c 9 cm d 0 cm/s e 3 cm/s
2 a x = t
3
4t
2
+5t +4, a = 6t 8
b when t = 1, x = 6, when t =
5
3
, x = 5
23
27
c when t = 1, a = 2 cm/s
2
, when
t =
5
3
, a = 2 cm/s
2
3 20 m to the left of O 4 x = 215
1
3
, v = 73
5 a v = 10t +25 b x = 5t
2
+25t
c 2.5 s d 31
1
4
m e 5 s
6 the 29th oor
Exercise 19C
1 2

10 s 2 37.5 m
3 a 3 m/s
2
b 6
2
3
s
c 337.5 m d
500
27
s
P1: FXS/ABE P2: FXS
9780521740494ans-16-19.xml CUAU033-EVANS September 4, 2009 1:5
A
n
s
w
e
r
s
Answers 711
4 a 2.5 m/s
2
b 31.25 m
5 a 50 s b 625 m
6 a 20 s b 10 m/s
7 a 19.2 m/s b 1.6 m
8 a 59.2 m/s b 158.4 m
9 a 10 s b after 3 s and 7 s
10 a 4.9(1 2t ) m/s b 4.9t (1 t ) +3 m
c 4.225 m d
10
7
s
11 a 2 s b 44.1 m c 4 s d 5 s
12 10

10 m/s
Exercise 19D
1 65 m
2 a 562.5 m b 450 m c 23.75 s
3
200
3
m/s 4 210 m
5 a 500 m b 375 m c 17.57 s
6 a 12.5 s b 187.5 m
7 No, the rst train will stop after 6.25 km and the
second train will stop after 6 km.
8 a 57.6 km/h b 1 min 6
2
3
s c 0.24
Multiple-choice questions
1 A 2 E 3 C 4 C 5 E
6 C 7 D 8 E 9 A 10 D
Short-answer questions
(technology-free)
1 a 5 cm to the left of O
b 8 cm to the left of O c 4 cm/s
d t = 2 s, 9 cm to the left e 1 cm/s
f 1
2
3
cm/s
2 a 8 cm to the right, 0 cm/s, 4 cm/s
2
b at 0 s, 8 cm to the right,
4 cm/s
2
and at
4
3
s, 6
22
27
to the right,
4 cm/s
2
3 a 3.5 s, 40.5 cm/s, 36 cm/s
2
b 2 s c 31 cm
4 a i
1
8
cm to the left
ii 1 cm/s
2
iii 1 cm/s
b i 0 s, 2 s ii
32
27
cm
5 a 12 m/s b s = t
3
c 17.57 s
6 a 4 s b 18
2
3
m to the right
c 5m/s
2
d 1.5 s e 6
1
4
m/s
7 a
1
12
m to the left b 1 m/s c 5 m/s
2
8 a a =
1
t
3
b s =
1
2

1
2t
9 a a = 3t
2
22t +24 b 15 m/s
2
c 2
1
12
m to the left, 60
7
12
m
10 40 m
11 a 2.5 m/s
2
b 8 s c 500 m
d
100
9
s
12 a 41
2
3
s b 347
2
9
m
13 a 7.143 s b 2
6
7
s, 4
2
7
s
14 a 2 s b 39.6 m c 4 s d 4.84 s
15 437.5 m
16 a 288 m b 16 s
17 16 m/s
18
80
81
m/s
19 a 0 m/s b 3 m/s
2
c 4 m/s
d 4
2
3
m e
11
12
m to the left
20 a 2t t
2
+8 b t
2

t
3
3
+8t
21 a 4t 2t
2
+6
b i 8 m/s ii 2 s iii 18 m
22 a 27 m/s
2
b 50 m/s c 4.5 s
23 a 10 m/s b 0 m
24 a 4 s, 6 s b 36 m c 0 t < 5
Extended-response questions
1 a 2
1
3
cm to the left of O
b 4 cm/s c 2 cm/s
2
d at 2 s
e
1
3
cm to the right of O
f at 1 s
3 a after 6 s at 36 m/s
b when t = 0 or 4, when t = 4 the maximum
height is 32 m
c after 2 s
4
x(1) x(0) = 15.1, x(2) x(1) = 5.3,
x(3) x(2) = 4.5, x(4) x(3) = 14.3,
x(5) x(4) = 24.1, x(6) x(5) = 33.9,
x(7) x(6) = 43.7, x(8) x(7) = 53.5,
x(9) x(8) = 63.3, x(10) x(9) = 73.1
The constant difference between successive
numbers is 9.8, the acceleration due to gravity.
6 33 m
7 a v = 5t +25, 0 t 5 b 62.5 m
8 25 m to the left of O
9 When T =
2u
g
, the second particle is projected
upwards at the instant the rst particle lands.
When T >
2u
g
, the second particle is projected
upwards after the rst particle has landed. Hence
for T >
2u
g
there is no collision.
P1: FXS/ABE P2: FXS
9780521740494ans-20-25.xml CUAU033-EVANS August 24, 2009 8:40
A
n
s
w
e
r
s
712 Essential Advanced General Mathematics
Chapter 20
Exercise 20A
1 T
1
= T
2
=
5

2
2
kg wt 2 90

3 T
1
= 14.99 kg wt, T
2
= 12.10 kg wt
4 28.34 kg wt, W48.5

S
5 T = 40 kg wt, N = 96 kg wt
6 F = 6.39 kg wt
7 a No b Yes
8 146

53

, 51

19

, 161

48

9 a 7.5 kg wt b 9.64 kg wt c 7.62 kg wt


10 32.97 kg wt, 26.88 kg wt, 39.29 kg wt,
W = 39.29 kg
Exercise 20B
1 13.05 kg wt 2 5.74 kg wt
3 3.73 kg wt, 8.83 kg wt 4 4.13 kg wt
5 6.93 kg wt 6 31.11 kg, 23.84 kg wt
7 44.10 kg, 22

29

to the vertical
8 6.43 kg wt, 7.66 kg wt, 11.92 kg
9 3.24 kg wt
Multiple-choice questions
1 E 2 E 3 A 4 C 5 C
6 B 7 B 8 A 9 C 10 B
Short-answer questions
(technology-free)
1 T
1
= 9 kg wt, T
2
= 12 kg wt
2 10

3 kg wt, 150

to the 10 kg wt
3 14

5 kg wt, 28

5 kg wt
4
15

3
3
kg wt 5
7
8
6
40

3
3
kg wt
7
15

2
2
kg wt
8 28 kg, 14

3 kg wt
9 4

3 kg wt
Chapter 21
21.1 Multiple-choice questions
1 D 2 E 3 B 4 B 5 C
6 C 7 A 8 D 9 C 10 B
11 D 12 D 13 C 14 E 15 D
16 A 17 D 18 B 19 A 20 C
21 C 22 E 23 B
21.2 Extended-response
questions
1 a p = 4, q = 3 b 4
2
9
m/s
2 a 14.7 m/s b 24.66 m/s
c 1.043 s, 1.957 s d 3.81 s
3 b i
2V(a +r)
3ar
ii
7V(a +r)
6ar
c
4
7
V
4
16 000
49
m; 52.567 m/s
5 a i 4 10t 3t
2
ii 10 6t
b i 4 m/s ii 10 m/s
2
c 2
1
3
s
d Y : 9
2
3
m/s, X : 35
2
3
m/s, X and Y
travelling in the same direction, X moving
faster catches up to Y.
Chapter 22
Exercise 22A
1 a categorical b numerical
c categorical d numerical
e categorical f categorical
2 a numerical b categorical
3 a discrete b discrete
c continuous d continuous
e discrete
Exercise 22B
1 a
H
a
m
b
u
r
g
e
r
s
C
h
i
c
k
e
n
F
i
s
h

&

c
h
i
p
s
C
h
i
n
e
s
e
P
i
z
z
a
O
t
h
e
r
Food type
N
o
.

o
f

s
t
u
d
e
n
t
s
5
10
15
20
25
b Hamburgers
2 a
N
o
.

o
f

r
e
s
p
o
n
s
e
s
50
100
150
A
g
r
e
e
D
o
n

t

k
n
o
w
D
i
s
a
g
r
e
e
S
t
r
o
n
g
l
y

d
i
s
a
g
r
e
e
Attitude to
capital punishment
S
t
r
o
n
g
l
y

a
g
r
e
e
b 32
P1: FXS/ABE P2: FXS
9780521740494ans-20-25.xml CUAU033-EVANS August 24, 2009 8:40
A
n
s
w
e
r
s
Answers 713
3 a
50
100
N
o
.

o
f

b
o
r
r
o
w
e
r
s
C
o
m
e
d
y
D
r
a
m
a
H
o
r
r
o
r
M
u
s
i
c
O
t
h
e
r
Type of film
b Music
4 a
P
e
r
c
e
n
t
a
g
e

o
f

s
t
u
d
e
n
t
s
25
50
W
a
t
c
h
T
V
R
e
a
d
L
i
s
t
e
n

t
o

m
u
s
i
c
W
a
t
c
h

a

v
i
d
e
o
P
h
o
n
e

f
r
i
e
n
d
s
O
t
h
e
r
Leisure activity
b Watching TV
Exercise 22C
1
Number 0 1 2 3 4 5 6
Frequency 4 4 4 4 3 2 1
2 a 4 b 2 c 5 d 28
3 a 0 b 48 c 6069 d 33
4 a, b
Temperatures Relative
(

C) Frequency frequency
0 1 0.03
5 0 0
10 1 0.03
15 9 0.28
20 4 0.13
25 5 0.16
30 7 0.22
35 4 0.13
40 0 0
45 1 0.03
c
0
2
4
6
8
10
5 10 15 20 25 30 35 40 45
Temperature
N
o
.

o
f

c
i
t
i
e
s
d 47%
5 a
0
2
4
6
N
o
.

o
f

b
o
o
k
s
Price
5 10 15 20 25 30 35 40 45
b $5.00$5.99
c
Prices ($) Cumulative frequency
less than 5 3
less than 10 9
less than 15 12
less than 20 15
less than 25 19
less than 30 19
less than 35 20
less than 40 20
less than 45 21
Price
C
u
m
u
l
a
t
i
v
e

f
r
e
q
u
e
n
c
y
5
0
5
10
15
20
10 15 20 25 30 35 40 45
6 a
0
28 29 30 31 32 33 34
2
4
6
8
10
Measurement
F
r
e
q
u
e
n
c
y
b
0
28
28
29 30 31 32 33 34
Measurement
C
u
m
u
l
a
t
i
v
e

f
r
e
q
u
e
n
c
y
c The students estimates ranged from
28.9 cm to 33.3 cm, with most students (86%)
over-estimating the 30 cm measure.
7 a
0
10 20 30 40 50 60 70 80 90 100
2
4
6
8
Marks
N
o
.

o
f

s
t
u
d
e
n
t
s
b
0
10 20 30 40 50 60 70 80 90 100
5
10
15
20
25
30
Marks
C
u
m
u
l
a
t
i
v
e

f
r
e
q
u
e
n
c
y
c The students marks ranged from 21 to 99,
with most students (over 70%) scoring more
than 50% on the test.
P1: FXS/ABE P2: FXS
9780521740494ans-20-25.xml CUAU033-EVANS August 24, 2009 8:40
A
n
s
w
e
r
s
714 Essential Advanced General Mathematics
8 a
Length of hole
N
o
.

o
f

h
o
l
e
s
0
240 260 280 300 320 340 360 380 400 420
2
4
6
8
10
12
14
b
Length of hole
0
240 260 280 300 320 340 360 380 400 420
10
20
30
40
50
C
u
m
u
l
a
t
i
v
e

f
r
e
q
u
e
n
c
y
c i below 300 m =
4
25
ii no. of holes 360 m = 17,
proportion =
17
50
iii approximately 305 m
Exercise 22D
1 a centre b neither c both
2 a positively skewed
b negatively skewed c symmetric
3 symmetric 4 symmetric 5 symmetric
Exercise 22E
1 a
4 8 9 9 5 | 0 represents 50 mm
5 0 2 7 7 8 9 9
6 0 7
b two months
2 a
0 4
1
1 6 8 9 2 | 5 represents 25 hours
2 1 1 3 (truncated)
2 5 5 5 6 7 7 9 9
3 1 1 2 3 3
3 6 9
4 1
4 6
b nine batteries
3 a
0 0
1 0 0 4 5 5 6 9
2 0 0 1 3 7 8 9
3 3 7 9
4 6 4 | 6 represents 46 minutes
5
6 3 7
7 0
b three students
c positively skewed
4 a 2 5 8
3 5 6 9
4 5 6 9
5 2
6 8
7 5 5 6 8 9
8 2 4
9 5
10 16 | 4 represents $164
11 (truncated)
12
13
14 9
15
16 4
17
18
19
20
21 0
b approximately symmetric
5 a Father's age Mother's age
3 7 8 8 9
4 4 4 3 3 3 1 1 0 4 0 0 0 1 2 3 3 3 3 3 4 4
9 8 8 8 8 7 7 6 6 6 5 4 5 6 7 8 9 9 9
4 2 1 1 0 5 0 0 0
5 5
0 | 4 represents 40 years 4 | 0 represents 40 years
b Both distributions are approximately
symmetric. Fathers, with ages centred in the
late forties, tend to be older than mothers, with
ages centred in the early forties. The spread is
similar for both distributions.
6 a
Class B Class A
3 2 1 9
2 2
3 9
4 5 7 8
5 5 8
9 6 5 8
6 4 3 3 2 2 1 0 0 7 1 6 7 9 9
8 8 4 4 3 2 1 1 0 0 8 0 1 2 2 5 5 9
8 1 9 1 9
9 | 6 represents 69 marks 7 | 1 represents 71 marks
b Six students in class A and two in class B
c Class B performed better as more students
scored in the higher values of 70s to 90s.
Exercise 22F
1 a mean = 18.36, median = 14
b mean = 9.19, median = 10
c mean = 7.41, median = 7.65
d mean = 1.62, median = 1.15
P1: FXS/ABE P2: FXS
9780521740494ans-20-25.xml CUAU033-EVANS August 24, 2009 8:40
A
n
s
w
e
r
s
Answers 715
2 a mean = 3.24, median = 3
b mean = 0.38, median = 0
3 mean = $193 386, median = $140 000; the
median is a better measure of centre as it is
typical of more house prices.
4 mean = 4.06, median = 4; both are reasonable
measures of centre in this example.
5 a range = 602, IQR = 455
b range = 5.3, IQR = 3.2
c range = 0.57, IQR = 0.21
d range = 7, IQR = 3.5
6 a 145 b 42
7 a 2.4 kg b 1.0 kg
8 a 12.39 b 1.33 c 281.24 d 3.04
9 a i mean = 17.61, s = 15.96
ii mean = 195.3, s = 52.9
b i 94% ii 100%
10 a
0
C
u
m
u
l
a
t
i
v
e

r
e
l
a
t
i
v
e





f
r
e
q
u
e
n
c
y
Age
17 22 27 32 37 42 47
0.2
0.4
0.6
0.8
1.0
median 18, IQR 2
b mean = 20.97, s = 7.37 c 92%
11 a i mean = 6.79, median = 6.75
ii IQR = 1.45, s = 0.93
b i mean = 13.54, median = 7.35
ii IQR = 1.7, s = 18.79
c The error does not affect the median or
interquartile range very much. It doubles the
mean and increases the standard deviation by
a factor of 20.
12 Approximately 95% of share prices lie between
$44 and $56.
13 About 95% of days lie in this interval.
Exercise 22G
1 a m = 154, Q
1
= 141.5, Q
3
= 161.5,
min = 123, max = 180
b
120 130 140 150 160 170 180
c The distribution of heights is slightly
negatively skewed, centred at 154 cm, with the
middle 50% of heights ranging from 141.5 cm
to 161.5 cm.
2 a m = 3, Q
1
= 0, Q
3
= 13, min = 0,
max = 52
b 38, 52
c
* *
0 10 20 30 40 50
d The distribution of number of books borrowed
is positively skewed, centred
at 3. While 75% of people borrowed
13 books or less, one student borrowed
38 books and another borrowed 52.
3 a
0 1000000 3000000
2 000000 4 000000
5000000
6 000000
7 000000
winnings
* *
b The distribution of winnings is extremely
positively skewed with a median value of
$854 533. The middle 50% of players won
from $752 228.50 to $1 573 674. There were
two outliers: Lleyton Hewitt, who is only
just an outlier, winning $2 766 051, and
Roger Federer, clearly an outlier, winning
$6 357 547.
4 a
0 5 10 15 20
b The distribution is symmetric, centred at
$10.00. The middle fty percent of students
earn between $8.15 and $11.85 per hour.
5 a
0 100 200 300 400 500 600
,
000
*
d The distribution is approximately symmetric,
centred at about 210 000, with an outlier at
570 000. The middle 50% of papers have
circulations from about 88 000 to 270 000.
Exercise 22H
1 a
0 10 20 30 40 50
*
*
*
After
Before
b The distribution for the number of sit-ups is
negatively skewed before the course, centred
at 26. After the course, the distribution is more
symmetric, centred at 30, indicating that the
course has been effective. The distribution
after the course is more variable than before
the course showing the course has not had the
same effect on all participants. There is one
outlier in the before group, who can achieve
46 sit-ups, and two in the after group,
recording 50 and
54 sit-ups respectively.
P1: FXS/ABE P2: FXS
9780521740494ans-20-25.xml CUAU033-EVANS August 24, 2009 8:40
A
n
s
w
e
r
s
716 Essential Advanced General Mathematics
2
0 1 3 5 7 2 4 6 8 9
Year 12
Year 8
a Year 12 b Year 12
3 a
15 20 30 40 50 25 35 45
1990
1970
b The distributions of ages in both groups are
slightly positively skewed, with the mothers in
1970 (median = 24.5) generally younger than
the mothers in 1990 (median = 28). The
variability in both groups is the same
(IQR = 10 for both groups).
Multiple-choice questions
1 D 2 B 3 D 4 C 5 D
6 C 7 A 8 D 9 C 10 A
Short-answer questions
(technology-free)
1 a numerical b categorical
2 a categorical b 7.5%
3
0
F
r
e
q
u
e
n
c
y
0
2
4
6
8
10
12
10 20
No. of cigarettes smoked
30 40
4 a 2 2
3 9 4 | 7 represents 47 minutes
4 3 4 5 7 9
5 0 1 1 2 2 4 5 6 6 7 9
6 5 8 9
7 2
b m = 52, Q
1
= 47, Q
3
= 57
5 x = $283.57, m = $267.50
6 a x = 178.89, s = 13.990 b 92.9%
7
0 5 10 15 20 25
8
0 10 20 30 40 50
*
Extended-response questions
1 a numerical
b 0 0 5 6 6 8 9
1 4 4 5 8 9 3 | 2 represents 32%
2 5 6 7 8
3 2 2
4 4
5 3
c positively skewed d 21.0%
e x = 20.05, m = 18
f
0 10 20 30 40 50 60
0
1
2
3
4
5
6
Divorce rate
F
r
e
q
u
e
n
c
y
i positively skewed ii 5
g
10 20 30 40 50 60 0
0.0
20.0
40.0
60.0
80.0
100.0
Divorce rate
C
u
m
u
l
a
t
i
v
e

f
r
e
q
u
e
n
c
y

%
i 58% ii 17%
2 a
60
0
2
4
6
8
10
12
70 80 90
Travel time
F
r
e
q
u
e
n
c
y
i 21.4% ii positively skewed
iii 38.1%
P1: FXS/ABE P2: FXS
9780521740494ans-20-25.xml CUAU033-EVANS August 24, 2009 8:40
A
n
s
w
e
r
s
Answers 717
b x = 69.60, s = 9.26, min = 57,
Q
1
= 62, m = 68, Q
3
= 76, max = 90
c i 69.60 ii 68
iii 33, 14 iv 76
v 9.26 vi 51.08, 88.12
d
50
Hillside
Met
60 70 80 90 100
e The distributions of travel times are both
positively skewed. The travel times for the
Met (median = 70) tend to be longer than the
travel times for Hillside trains (median = 68).
The spread of times is also longer for the Met
(IQR = 24) than the travel times for Hillside
trains (IQR = 14).
3 a
Method 1
Method 2
Method 3
50 60 70 80 90 100
b The distributions of scores are negatively
skewed for methods 1 and 3 and symmetric
for method 2. The scores for method 1 are
higher than for methods 2 and 3 (90, 79 & 70
respectively), and are also less variable than
method 2. They show similar variation to the
scores for method 3.
c Thus training method 1 would be
recommended, as it consistently produces
higher scores.
4 a
First born
Second born
Third born
10 20 30 40 50
b The distribution for the rst-born is
symmetrical, while for the second and
third-born the distributions are positively
skewed. The centre for the rst born is higher
than for the second which is higher than the
third (35, 21 & 12 respectively), whilst the
variability is most for the rst-born, followed
by the second-born and then the third-born.
Chapter 23
Exercise 23A
1 a
1.0 2.0 3.0 4.0 5.0 6.0
0
10
20
30
40
50
60
70
Drug dose (mg)
R
e
s
p
o
n
s
e

t
i
m
e

(
m
i
n
)
b negative association c no outliers
2 a
100 200 300 400 500 600
5
10
15
0
B
u
s
i
n
e
s
s

(
$

0
0
0
)
Advertising ($)
b positive association c no outliers
3 a
100 200 300 400
400
500
600
700
800
No. of seats
A
i
r
s
p
e
e
d

(
k
m
/
h
)
b positive association
c (122, 378) is an outlier
4 a
2 4 6
5
10
15
0
8 10 12
P
r
i
c
e

(
$

0
0
0
)
Age (years)
b negative association
c (10, 8700) is an outlier
Exercise 23B
1 a no correlation
b weak negative correlation
c strong negative correlation
d weak positive correlation
e strong positive correlation
f strong negative correlation
g strong positive correlation
P1: FXS/ABE P2: FXS
9780521740494ans-20-25.xml CUAU033-EVANS August 24, 2009 8:40
A
n
s
w
e
r
s
718 Essential Advanced General Mathematics
h no correlation
i strong negative correlation
j weak positive correlation
k strong positive correlation
l moderate negative correlation
2 a 0.71 b 0.78 c 0.82 d 0.92
3 a 0.6 b moderate negative correlation
4 a 0.67 b moderate positive correlation
5 a 1 b perfect positive correlation
6 a 0.43 b weak negative correlation
Exercise 23C
1 a no linear relationship
b weak negative linear relationship
c strong negative linear relationship
d weak positive linear relationship
e strong positive linear relationship
f strong negative linear relationship
g strong positive linear relationship
h no linear relationship
i moderate negative linear relationship
j weak positive linear relationship
k perfect positive linear relationship
l perfect negative linear relationship
2 a 0.8 b 0.8 c 0.7
d 0.8 e 0.7 f 0.2
3 a 0.86
b strong negative linear relationship
4 a 0.95
b strong positive linear relationship
5 a 0.77
b strong positive linear relationship
6 a 0.77
b strong negative linear relationship
7
50
50
55
55
60
60
65
65
70
70
40 45
Attempt 1
A
t
t
e
m
p
t

2
a a strong positive relationship
b Yes, the data is numeric and the relationship is
linear. There are no outliers.
8
210
230
250
270
290
310
330
200 240 220 260 280 300 320 340
0
T
e
s
t

2
Test 1
a There is a strong positive linear relationship
between the scores on Test 1 and Test 2.
b Yes, the data is numeric and the relationship is
linear.
c q = 0.71, r = 0.87
d q: moderate positive linear relationship
r: strong positive linear relationship
e i q = 0.43, r = 0.004
ii The error in the data has a much greater
effect on Pearsons correlation coefcient.
Exercise 23D
Note: Answers will vary for lines drawn by eye.
1
x
y
1 2 3 4 5 6 7 8
5
10
15
0
y = 1 + 2x
2
x
y
0
1 2 3 4
5
5
1 2 3
20
15
10
y =

4.5

3.75x
3 a
2000
2000
4000
4000
6000
6000
K
B
A
J
D
E C
G
H
I
F
Year 1
Y
e
a
r

2
0
b y = 424 +0.794x
c The positive slope indicates that districts with
high rates in Year 1 also had high rates in
Year 2.
4 a
85
90
95
36 40 44 48 52 56 60
H
e
i
g
h
t

(
c
m
)
Age (months)
b y = 72 +0.4x
c The intercept (72 cm) is the predicted height
at age 0. The slope predicts an increase of
0.4 cm in height each month.
d i 89 cm ii 158 cm
P1: FXS/ABE P2: FXS
9780521740494ans-20-25.xml CUAU033-EVANS August 24, 2009 8:40
A
n
s
w
e
r
s
Answers 719
e Part i is reasonable as it is a value close to the
data. Part ii is not reliable as the relationship
may no longer be linear here.
5 a
150
150
160
160
170
170
180
180
Mother
D
a
u
g
h
t
e
r
b y = 18.3 +0.91x c 173 cm
6 a
160 180 100
2.5
3.0
3.5
4.0
200 140 120
C
o
s
t

(
$

0
0
0
)
Number of MP3 players
b y = 1300 +13x c $1300
d $13
7 a y = 70 14x
b The intercept is the predicted time taken to
experience pain relief if no drug is given.
From the slope we predict a reduction of
14 minutes in time taken to experience pain
relief for each mg of drug administered.
c 14 mins, which is not a realistic answer
8 a y = 18.2x
b Intercept predicts zero sales if nothing is spent
on advertising. The slope means that on
average, each $1 spent on advertising is
associated with an increase of $18.20 in sales.
c i $18 200 ii $0
Exercise 23E
1 a y = 68.2 +0.46x
b The y intercept is the predicted height at birth.
From the slope, we predict an increase in
height of 0.46 cm each month.
c i 88 cm ii 168 cm
d The height at 42 months is reliable since this
in within the range of data given
(interpolation). The height at 18 years is less
reliable since this is outside the range of data
given (extrapolation).
2 y = 487.6 +0.77x
3 a y = 50.2 +0.72x
b An increase of 1 cm in the mothers height is
associated with an increase of 0.72 cm in the
daughters height, on average.
c 172 cm (to the nearest cm)
4 a y = 1330 +12x
b $1330
c $12
5 a response time = 57.0 10.2 drug dose
b The intercept of 57.0 minutes is the predicted
time for pain relief when no drug is given.
From the slope, we predict a 10.2 minute
decrease in response time for each 1 mg of
drug given.
c 4.2 minutes, which is not a realistic answer.
6 a business = 1123.8 +18.9 advertising
b Intercept is the volume of business with no
advertising. From the slope we predict an
increase in business of $18.90 for every dollar
spent on advertising.
c i $20 044 (to the nearest dollar)
ii $1124 (to the nearest dollar)
Multiple-choice questions
1 D 2 E 3 A 4 B 5 B
6 E 7 C 8 C 9 A 10 D
Short-answer questions
1 a
30 40
40
50 60
60
70
80
100
120
140
Inside 50
S
c
o
r
e

(
p
o
i
n
t
s
)
b Positive
2 0 3 0.927
4 weight = 200 +2 height, approximately
5 errors = 14.9 0.533 time
6 a Intercept: no sensible interpretation. Slope:
For each additional second taken to complete
the task on average the number of errors is
reduced by about
1
2
an error.
b 9.6
Extended-response questions
1 a IV = Exam score,
DV = Number of newclients
P1: FXS/ABE P2: FXS
9780521740494ans-20-25.xml CUAU033-EVANS August 24, 2009 8:40
A
n
s
w
e
r
s
720 Essential Advanced General Mathematics
b
7.00
70.00
5.00
8.00
80.00
6.00
60.00
9.00
10.00
11.00
65.00 75.00 85.00
Exam score
N
u
m
b
e
r

o
f

n
e
w

c
l
i
e
n
t
s
c positive d 1, strong positive
e 0.748, moderate positive
f Number of newclients =
4.00 +0.173 Examscore
g Intercept: no sensible interpretation. Slope: On
average each extra 1 mark in the nal exam is
associated with an increase of 0.173 clients.
h 13
i Not very reliable as it is outside the range of
the data.
2 a IV = Assignment mark,
DV = Final exammark
b
55.00
55.00
60.00
60.00
65.00
65.00
70.00
70.00
80.00
80.00
85.00
75.00
75.00
50.00
Assignment mark
F
i
n
a
l

e
x
a
m

m
a
r
k
c positive d 1, strong positive
e 0.758, strong positive
f The statement is incorrect as it is a causal
statement. It can only be said that there is an
association between the Assignment mark and
the Final exam mark.
g Final exam mark =
24.74 +0.699 Assignment mark
h Intercept: On average those who score 0 on
the assignment will score about 25 on the nal
exam. Slope: On average each extra 1 mark in
the assignment is associated with an increase
of 0.7 marks on the nal exam.
i 60
j Reliable, as it is within the range of the data.
3 a IV = Number of times the song was played,
DV = Weekly sales
b
25 30 35 40 45 50 55
2000
2500
3000
3500
4000
4500
Number of times played
W
e
e
k
l
y

s
a
l
e
s
c positive d 1, strong positive
e 0.946, strong positive
f Weekly sales = 293.06 +74.281
Number of times song played
g Intercept: On average songs which get no
airplay will have sales of around 293. Slope:
On average each extra 1 play of the CD is
associated with an increase of about 73 sales
per week.
h 4750
i Not reliable as it is outside the range of the
data.
Chapter 24
Multiple-choice questions
1 C 2 A 3 E 4 D 5 C
6 B 7 D 8 C 9 B 10 D
11 B 12 B 13 B 14 B 15 D
16 C 17 B 18 E 19 B 20 B
21 B 22 A 23 A 24 C 25 C
26 C 27 C 28 A 29 C 30 D
Extended-response questions
1 a
Aggressive Passive
1 3 3 5 5 7 8 8
5 2 2 2 5 5 7
9 7 3 0 0 3 0 0 3
5 5 5 3 3 3 4
8 5 5
0 6
5 | 2 represents 25 1 | 5 represents 15
b Aggressive Passive
min = 25 min = 13
Q
1
= 33 Q
1
= 15
median = 43 median = 22
Q
3
= 45 Q
3
= 27
max = 60 max = 33
c
10 20 30 40 50 60
Passive
Aggressive
P1: FXS/ABE P2: FXS
9780521740494ans-20-25.xml CUAU033-EVANS August 24, 2009 8:40
A
n
s
w
e
r
s
Answers 721
d Aggressive: mean = 42.07 cm,
s = 10.25 cm; Passive:
mean = 21.53 cm, s = 6.58 cm
e Both distributions are approximately
symmetrical. The centre for passive is much
lower than the centre for aggressive which
indicates that individuals allow a passive
person to approach closer than an aggressive
person. The variability for both groups is
similar.
2 a
0.05% blood alcohol 0% blood alcohol
2 5 9
4 2 2 2 3 0 1 4
9 9 8 6 5 3 5 5 5 6 8 9
4 3 2 1 1 0 4 0 0 2 3 4 4
6 6 4 6 8
0 0 5
6 5
6 4
0 | 4 represents 40 4 | 0 represents 40
b
0.05% 0%
min = 32 min = 25
Q
1
= 35.5 Q
1
= 34.5
median = 40.5 median = 38.5
Q
3
= 45 Q
3
= 43.5
max = 56 max = 64
c
20 30 40 50 60 70
0%
0.05%
*
d 0%: mean = 38.9, s = 8.4 cm;
0.05%: mean = 40.8, s = 6.6
e The distributions are both approximately
symmetrical. The 0% reading has one outlier.
The centre for 0% is slightly lower than the
centre for 0.05% and the spread of each is
similar. The effect of 0.05% alcohol on
performance has been minimal.
3 a
Secondhand New
9 7 5 0 10 7 8
9 9 8 6 11 5 6 6
6 12 0 5
7 13 2 4
7 6 14
9 9 8 7 0 15 0 1 4 5
6 16 4 7 7
3 17 2
18 0 1
0 | 15 represents 150 15 | 0 represents 150
b New Secondhand
min = 10 700 min = 9950
Q
1
= 11 595 Q
1
= 11 550
median = 14 995 median = 13 650
Q
3
= 16 660 Q
3
= 15 750
max = 18 050 max = 17 250
Prices rounded to nearest hundred
c
9 10 11 12 13 14 15 16 17 18 19
New
Secondhand
d New: mean = $14 275, s = $2508;
Secondhand: mean = $13 497, s = $2349
e The distribution of the secondhand price is
symmetrical while that of the new price is
negatively skewed. The centre for new cars is
higher than for secondhand, indicating they
are more expensive and new prices are more
variable than secondhand prices.
4 a Choice No choice
1 7
6 5 5 2 1 4 6 8
6 6 6 5 5 0 0 3 5 5 6 8
8 4 0 4 2 2 4 4 89
0 0 5
5 | 2 represents 25 3 | 5 represents 35
b No choice Choice
min = 17 min = 25
Q
1
= 26 Q
1
= 30
median = 36 median = 36
Q
3
= 44 Q
3
= 44
max = 49 max = 50
c
0 10 20 30 40 50
Choice
No choice
d No choice: mean = 35.3, s = 10.0;
Choice: mean= 36.4, s = 8.6
e Giving students a choice of assignment has
not affected the centre of the distribution.
The typical mark for both groups is around
36. However, marks are less variable when
students are allowed to choose.
5 a
0 10 20 30 40
Normal
Disabled
b The distribution of scores for normal
children is symmetric and tightly clustered
around the centre at 28. The distribution for
learning disabled children is negatively
skewed, centred lower at 25 and shows much
more variability in scores.
6 a
40
30
20
10
0
1 2 3 4 5
Sleep deprivation (hrs)
N
u
m
b
e
r

R
E
M
The scatterplot shows a moderate positive
linear relationship.
P1: FXS/ABE P2: FXS
9780521740494ans-20-25.xml CUAU033-EVANS August 24, 2009 8:40
A
n
s
w
e
r
s
722 Essential Advanced General Mathematics
b q = 0.6 c r = 0.748
d REM = 8.45 +4.88 sleep
e From the slope, we predict an increase of
4.88 in REM for each additional hour of
sleep deprivation.
f REM = 33
7 a
10
4
0
8 12 16 20 24 28
15
20
25
30
35
T
e
s
t

s
c
o
r
e
Hours
The scatterplot has a strong positive linear
relationship.
b q = 1 c r = 0.94
d test score = 12.3 +0.93 hours
e From the slope, we predict an increase of
about 1 mark in test score for each additional
hour of study.
f Test score = 22
8 a
Year
0
2000
2050
2100
2150
81 86 91
N
o
.

o
f

s
c
h
o
o
l
s
The scatterplot has a strong negative linear
relationship.
b q = 1 c r = 0.98
d schools = 2169.4 12.5 year where year
is the no. of years since 1980, i.e. 1981 = 1
e The slope tells us that over the time period in
question, an average of 12.5 schools closed
each year.
f schools = 1919
g This is not a reliable estimate because it is a
long way outside the range of data we have.
The government may decide to change policy
with regard to closing schools.
9 a
Weight
85
90
100
105
110
115
75
80
95
120
60 70 80 90
G
l
u
c
o
s
e
The scatterplot has a weak positive linear
relationship.
b r = 0.60
c glucose = 59.2 +0.55 weight
d From the slope, we predict an increase in
glucose of 0.55 mg/100 mL for each kg
increase in weight.
e Glucose = 97.7 mg/100 mL
10 a
12 13 14 15 16
15
20
25
E
x
p
e
n
d
i
t
u
r
e

(
$

0
0
)
Income ($000)
The graph shows a strong positive linear
relationship.
b q = 1 c r = 0.94
d expenditure = 1547 +0.24 income
e From the slope, we predict that for every
extra $1 in income expenditure increases by
24c.
f Expenditure = $2373
g Since $30 000 is well outside our range of
data the prediction would not be reliable.
11 a
2
0
30
20
10
1 3
M
a
n
g
o
e
s
Price
The graph shows a strong negative linear
relationship.
b q = 1 c r = 0.95
d boxes = 34 10.3 price
e Slope tells us that for every dollar increase in
price, 10 fewer boxes are sold on average.
f 20 boxes
g 5 cents each is outside the range of data that
we have and any prediction would be
unreliable.
12 a
Magnesium (mg)
T
a
s
t
e
0
8 10 12 14 16 18
20
40
60
80
100
The graph shows a moderate positive linear
relationship.
P1: FXS/ABE P2: FXS
9780521740494ans-20-25.xml CUAU033-EVANS August 24, 2009 8:40
A
n
s
w
e
r
s
Answers 723
b q = 0.5 c r = 0.73
d taste = 22.4 +7.3 magnesium
e Slope tells us that each 1 mg of magnesium
in the water predicts an increase in the taste
rating of 7.3
f taste rating = 562
13 a i
0 20 40 60 80 100
* *
ii The boxplot shows that the distribution of
matches won is positively skewed, centred
at 8 with an outlier at 93. The middle 50%
of captains led their teams about 224
times.
iii Allan Border
b i
0 10 20 30 40 50 60 70 80 90 100
ii The distribution is approximately
symmetric, centred at about 44%. The
middle 50% of captains have won from
about 18% to 63% of their matches.
iii Four captains, H H Massie, H Trumble,
W A Brown and R N Harvey, have a
success rate of 100%.
c i
0
0
10
20
30
40
50
20 40 60 80 100
Tests played
T
e
s
t
s

w
o
n
ii There is a strong positive linear
association between the number of
matches played and the number of
matches won.
iii r = 0.915
iv Matches won =
0.220 +0.437 matches played
v Approximately, for each extra ve test
matches played, we predict that two tests
are won.
vi
0 20 40 60 80 100
Tests played
T
e
s
t
s

w
o
n
50
40
30
20
10
0
Steve Waugh
Steve Waugh has the largest positive
residual, so if this is used as a measure of
success, he is the most successful captain.
Chapter 25
Exercise 25A
1 a b c d
2 a False: Consider A = {2, 4, 6, 8, 10} and
B = {6, 8, 10} , n (A B) = 5
b True: x A x B
x A B x A
and x A x A B
c False: A = {2, 4, 6} and
B = {8, 10} , A B =
d True
3 a n (A B) = 8 n (A) = 5 and n (B) = 3:
False
b A B = A A B: True
c A = or B = A B = : True
d A = A

= : True
4 a n
2
is odd implies n is odd: True if n is an
integer
b N
2
is divisible by 9 N is divisible by 3:
True if n is an integer
c x
2
> 4 x < 2: False
5 a The sum of three consecutive odd numbers is
divisible by 3.
(Hint: consider 2n 1, 2n +1, 2n +3)
b The sum of four consecutive odd numbers is
divisible by 8.
7 a 3
2
+4
2
= 5
2
, (3 +4)
2
= 7
2
b 14 c

1 +4 =

1 +

4 = 3
d
1
2

1 +
2 +4
2

= 2 and
1
2

1 +2
2
+4

=
11
4
e 7 +3 = 10
f
1
100 +10
=
1
110
;
1
100
+
1
10
=
11
100
P1: FXS/ABE P2: FXS
9780521740494ans-20-25.xml CUAU033-EVANS August 24, 2009 8:40
A
n
s
w
e
r
s
724 Essential Advanced General Mathematics
8 a a > b implies a b is positive: True
b x = y implies x = 0 and y = 0: False
c x = y implies x + y = 0: True
d xy is even implies x is even and y is odd:
False
e A perfect square being even implies its
square root is even: True
13 m = 5 and n = 12 or m = 6 and n = 8
15 6, 8, 10 and 5, 12, 13
Exercise 25C
1 a x = 2 +3t, y = 7 11t, t Z
b x = 8 +7t, y = 2 2t, t Z
c x = 264 +21t, y = 99 8t, t Z
(Hint: Use your graphics calculator with
Y
1
=
8
21
x +
33
21
and nd the rst solution
through Table.)
x = 9 +21t, y = 5 8t, t Z is also a
solution.
d x = 2 +3t, y = 7 11t, t Z
e x = 4 +7t, y = 2 2t, t Z
f x = 4 +7t, y = 2 2t, t Z
4 a 8s +6b = 54
b s = 6, b = 1 or s = 3, b = 5
5
50c coins 0 2 4 6 8 10
20c coins 25 20 15 10 5 0
6 x = 17, y = 20. The only solution to
19x +98y = 1998 is x = 2, y = 20 for
x, y Z
+
besides x = 100, y = 1
7 (10, 0), (9, 5), (8, 10), (7, 15), (6, 20), (5, 25),
(4, 30), (3, 35), (2, 40), (1, 45), (0, 50)
8 63x 23y = 7; x = 5 +23t,
y = 14 +63t ; t 0 and t Z
9 5 and 15
10 20; 20 +77t for t N {0}
11 Pour two full 5 litre jugs into a container and
remove one 3 litre jugful.
12 All amounts in excess of 3c except
4c and 7c.
13 8 of type A and 16 of type B
Exercise 25D
1 a 43 = 8 5 +3
(43, 5) = (5, 3) = 1
b 39 = 3 13 +0
(39, 13) = (13, 0) = 13
c 37 = 2 17 +3
(37, 17) = (17, 3) = 1
d 128 = 16 8 +0
(128, 16) = (16, 0) = 16
3 a l b 27 c 6 d 5
4 a x = 44 +393t, y = 15 134t ; t Z
b x = 1 +4t, y = 1 3t ; t Z
c x = 1434 +4t, y = 956 3t ; t Z
d x = 1 +5t, y = 7 +3t ; t Z
e x = 107 +224t, y = 32 67t ; t Z
f x = 37 +336t, y = 25 227t ; t Z
Multiple-choice questions
1 E 2 D 3 C 4 E 5 D
6 C 7 B 8 D 9 D 10 B
Short-answer questions
(technology-free)
1 5
2 a x = 39 +43t, y = 8 9t ; t Z
b no solutions
4 17
5 a x = 3 +7t, y = 2 5t ; t Z
b x = 300 +7t, y = 200 5t ; t Z
c x = 3 +7t, y = 2 5t ; t Z and t 0
6 Tom is 36 and Fred is 27
Extended-response questions
1 a
b 30 c
n
2
1
4
d Proof is by induction or by observing an
arithmetic sequence is formed.
2 It is constructive to show if
a
b
<
c
d
where
a
b
,
c
d
Q
+
;
a
b
<
a +c
b +d
<
c
d
3 a 6480 cubes b k = 2 c n
2
+6n +3
4 a six 5c stamps and one 8c stamp
b 5c 8c
8 10
16 5
0 15
24 0
5 36, 81 6 576
13 No such numbers exist.

You might also like